Вы находитесь на странице: 1из 597

State Educational Institution

of the Higher Professional Education


VOLGOGRAD STATE MEDICAL UNIVERSITY
of Federal Agency for Public Health and Social Development
Pharmacology Department

MULTIPLE-CHOICE TESTS
IN
PHARMACOLOGY

VOLOGOGRAD
2006
2

ГОСУДАРСТВЕННОЕ ОБРАЗОВАТЕЛЬНОЕ УЧРЕЖДЕНИЕ


ВЫСШЕГО ПРОФЕССИОНАЛЬНОГО ОБРАЗОВАНИЯ
ВОЛГОГРАДСКИЙ ГОСУДАРСТВЕННЫЙ
МЕДИЦИНСКИЙ УНИВЕРСИТЕТ
ФЕДЕРАЛЬНОГО АГЕНСТВА ПО ЗДРАВООХРАНЕНИЮ И
СОЦИАЛЬНОМУ РАЗВИТИЮ
КАФЕДРА ФАРМАКОЛОГИИ

В.А. КОСОЛАПОВ, М.В. ЧЕРНИКОВ, О.Ю. ГРЕЧКО,


И.Н. ИЕЖИЦА, А.Ф. КУЧЕРЯВЕНКО

ТЕСТОВЫЕ ВОПРОСЫ ПО ФАРМАКОЛОГИИ

УЧЕБНОЕ ПОСОБИЕ

Под редакцией Член-корреспондента РАМН, заслуженного деятеля науки


РФ, доктора медицинских наук, профессора А.А. Спасова и кандидата
филологических наук В.В. Жура

Рекомендовано к печати

УЧЕБНО-МЕТОДИЧЕСКИМ ОБЪЕДИНЕНИЕМ ПО МЕДИЦИНСКОМУ И


ФАРМАЦЕВТИЧЕСКОМУ ОБРАЗОВАНИЮ ВУЗОВ РОССИИ
В КАЧЕСТВЕ УЧЕБНОГО ПОСОБИЯ ДЛЯ ИНОСТРАННЫХ СТУДЕНТОВ
МЕДИЦИНСКИХ ВУЗОВ, ОБУЧАЮЩИХСЯ НА АНГЛИЙСКОМ ЯЗЫКЕ
ОТ ________ УМО-_____

Волгоград 2006
3

УДК 615(07)

ТЕСТОВЫЕ ВОПРОСЫ ПО ФАРМАКОЛОГИИ:


Учебное пособие / В.А. КОСОЛАПОВ, М.В. ЧЕРНИКОВ, О.Ю. ГРЕЧКО, И.Н.
ИЕЖИЦА, А.Ф. КУЧЕРЯВЕНКО / Под ред. АА. Спасова, В.В Жура. –
Волгоград, 2005. –___с.

РЕЦЕНЗЕНТЫ:
Зав. кафедрой фармакологии и биофармации ФУВ ВолгМУ,
д.м.н., профессор И.Н. Тюренков
Зав. кафедрой фармакологии фармацевтического факультета Московской
медицинской академии им. Сеченова, профессор, д.м.н. Р.Н. Аляутдин
Зав. кафедрой фармакологии Саратовского медицинского университета,
д.м.н., профессор А.А. Свистунов

© Волгоградский государственный медицинский университет 2006


4
Contents

Chapter Pages
(1) GENERAL PRINCIPLES OF PHARMACOLOGY............................................................................................... 5
PART I PHARMACOKINETICS ........................................................................................................................................ 5
PART II PHARMACODYNAMICS.................................................................................................................................... 8
(2) AGENTS, CONTROLLING THE FUNCTIONS OF THE PERIPHERAL NERVOUS SYSTEM ................. 12
PART I LOCAL ANESTHETICS .............................................................................................................................................. 12
PART II CHOLINOMIMETIC DRUGS ...................................................................................................................................... 16
PART III CHOLINORECEPTOR BLOCKING DRUGS ................................................................................................................. 19
PART IV ADRENORECEPTOR ACTIVATING DRUGS ............................................................................................................... 24
PART V ADRENORECEPTOR ANTAGONIST DRUGS ............................................................................................................... 29
(3) AGENTS, CONTROLLING THE FUNCTIONS OF THE CENTRAL NERVOUS SYSTEM........................ 34
PART I HYPNOTIC DRUGS ................................................................................................................................................... 34
PART II ANTISEIZURE DRUGS ............................................................................................................................................. 38
PART III ANTIPARKINSONIAN AGENTS................................................................................................................................ 41
PART IV ETHYL ALCOHOL .................................................................................................................................................. 44
PART V NARCOTIC ANALGESICS......................................................................................................................................... 46
PART VI NON-NARCOTIC ANALGESICS ............................................................................................................................... 49
PART VII ANTIPSYCHOTIC AGENTS .................................................................................................................................... 51
PART VIII ANTIDEPRESSANT AGENTS ................................................................................................................................ 54
PART IX ANXIOLYTIC AGENTS ........................................................................................................................................... 57
PART X CNS STIMULANTS ................................................................................................................................................. 60
PART XI DRUGS OF ABUSE ................................................................................................................................................. 63
PART XII GENERAL ANESTHETICS ...................................................................................................................................... 65
(4) ORGANOTROPIC AGENTS.................................................................................................................................. 68
PART I DRUGS ACTING ON RESPIRATORY SYSTEM .............................................................................................................. 68
PART II DRUGS USED IN GASTROINTESTINAL DISEASES ...................................................................................................... 70
PART III DRUGS ACTING ON HEMATOPOIETIC SYSTEM........................................................................................................ 73
PART IV DRUGS USED IN DISORDERS OF COAGULATION ..................................................................................................... 74
PART V DRUGS USED FOR TREATMENT OF HEART FAILURE ................................................................................................ 76
PART VI ANTIARRHYTHMIC AGENTS .................................................................................................................................. 78
PART VII DRUGS FOR ANGINA PECTORIS TREATMENT ....................................................................................................... 80
PART VIII ANTIHYPERTENSIVE DRUGS............................................................................................................................... 83
PART IX HYPERTENSIVE (ANTI-HYPOTENSIVE) DRUGS. DRUGS INFLUENCING CEREBRAL BLOOD FLOW. ANTI-MIGRAINE
AGENTS ................................................................................................................................................................................ 85

(5) METABOLIC PROFILE DRUGS .......................................................................................................................... 88


PART I HYPOTHALAMIC & PITUITARY HORMONES, THYROID & ANTITHYROID DRUGS ..................................................... 88
PART II PANCREATIC HORMONES & ANTIDIABETIC DRUGS ............................................................................................... 91
PART III THE GONADAL HORMONES & INHIBITORS ........................................................................................................... 93
PART IV GLUCOCORTICOID, STEROIDAL & NONSTEROIDAL ANTI-INFLAMMATORY DRUGS .............................................. 95
PART V IMMUNOTROPIC & ANTIALLERGIC AGENTS .......................................................................................................... 98
PART VI VITAMINS, VITAMIN-LIKE COMPOUNDS, ANTIVITAMINS, ENZYMES & ANTIENZYMES ...................................... 104
PART VII ANTIHYPERLIPIDEMIC DRUGS & DRUGS USED IN THE TREATMENT OF GOUT .................................................. 110
PART VIII AGENTS THAT AFFECT BONE MINERAL HOMEOSTASIS .................................................................................. 114
PART IX MINERALOCORTICOID, MINERALOCORTICOID ANTAGONISTS, DIURETICS, PLASMA EXPANDERS ...................... 120
(6) CHEMOTHERAPEUTIC DRUGS....................................................................................................................... 124
PART I ANTIBIOTICS .................................................................................................................................................... 124
PART II SYNTHETIC ANTIBACTERIAL DRUGS ....................................................................................................... 129
PART III ANTIPROTOZOAL AND ANTHELMINTIC DRUGS................................................................................... 132
PART IV ANTIVIRAL AGENTS. AGENTS FOR CHEMOTHERAPY OF CANCER .................................................. 135
5
(1) GENERAL PRINCIPLES OF PHARMACOLOGY

PART I PHARMACOKINETICS
001. Pharmacokinetics is:
a) The study of biological and therapeutic effects of drugs
b) The study of absorption, distribution, metabolism and excretion of drugs
c) The study of mechanisms of drug action
d) The study of methods of new drug development
002. What does “pharmacokinetics” include?
a) Complications of drug therapy
b) Drug biotransformation in the organism
c) Influence of drugs on metabolism processes
d) Influence of drugs on genes
002. What does “pharmacokinetics” include?
a) Pharmacological effects of drugs
b) Unwanted effects of drugs
c) Chemical structure of a medicinal agent
d) Distribution of drugs in the organism
003. What does “pharmacokinetics” include?
a) Localization of drug action
b) Mechanisms of drug action
c) Excretion of substances
d) Interaction of substances
004. The main mechanism of most drugs absorption in GI tract is:
a) Active transport (carrier-mediated diffusion)
b) Filtration (aqueous diffusion)
c) Endocytosis and exocytosis
d) Passive diffusion (lipid diffusion)
005. What kind of substances can’t permeate membranes by passive diffusion?
a) Lipid-soluble
b) Non-ionized substances
c) Hydrophobic substances
d) Hydrophilic substances
006. A hydrophilic medicinal agent has the following property:
a) Low ability to penetrate through the cell membrane lipids
b) Penetrate through membranes by means of endocytosis
c) Easy permeation through the blood-brain barrier
d) High reabsorption in renal tubules
007. What is implied by «active transport»?
a) Transport of drugs trough a membrane by means of diffusion
b) Transport without energy consumption
c) Engulf of drug by a cell membrane with a new vesicle formation
d) Transport against concentration gradient
008. What does the term “bioavailability” mean?
a) Plasma protein binding degree of substance
b) Permeability through the brain-blood barrier
c) Fraction of an uncharged drug reaching the systemic circulation following any route administration
d) Amount of a substance in urine relative to the initial doze
009. The reasons determing bioavailability are:
a) Rheological parameters of blood
b) Amount of a substance obtained orally and quantity of intakes
c) Extent of absorption and hepatic first-pass effect
d) Glomerular filtration rate
010. Pick out the appropriate alimentary route of administration when passage of drugs through liver is minimized:
a) Oral
b) Transdermal
c) Rectal
d) Intraduodenal
011. Which route of drug administration is most likely to lead to the first-pass effect?
a) Sublingual
6
b) Oral
c) Intravenous
d) Intramuscular
012. What is characteristic of the oral route?
a) Fast onset of effect
b) Absorption depends on GI tract secretion and motor function
c) A drug reaches the blood passing the liver
d) The sterilization of medicinal forms is obligatory
013. Tick the feature of the sublingual route:
a) Pretty fast absorption
b) A drug is exposed to gastric secretion
c) A drug is exposed more prominent liver metabolism
d) A drug can be administrated in a variety of doses
014. Pick out the parenteral route of medicinal agent administration:
a) Rectal
b) Oral
c) Sublingual
d) Inhalation
015. Parenteral administration:
a) Cannot be used with unconsciousness patients
b) Generally results in a less accurate dosage than oral administration
c) Usually produces a more rapid response than oral administration
d) Is too slow for emergency use
016. What is characteristic of the intramuscular route of drug administration?
a) Only water solutions can be injected
b) Oily solutions can be injected
c) Opportunity of hypertonic solution injections
d) The action develops slower, than at oral administration
017. Intravenous injections are more suitable for oily solutions:
a) True
b) False
018. Correct statements listing characteristics of a particular route of drug administration include all of the following EXCEPT:
a) Intravenous administration provides a rapid response
b) Intramuscular administration requires a sterile technique
c) Inhalation provides slow access to the general circulation
d) Subcutaneous administration may cause local irritation
019. Most of drugs are distributed homogeneously.
a) True
b) False
020. Biological barriers include all except:
a) Renal tubules
b) Cell membranes
c) Capillary walls
d) Placenta
021. What is the reason of complicated penetration of some drugs through brain-blood barrier?
a) High lipid solubility of a drug
b) Meningitis
c) Absence of pores in the brain capillary endothelium
d) High endocytosis degree in a brain capillary
022. The volume of distribution (Vd) relates:
a) Single to a daily dose of an administrated drug
b) An administrated dose to a body weight
c) An uncharged drug reaching the systemic circulation
d) The amount of a drug in the body to the concentration of a drug in plasma
023. For the calculation of the volume of distribution (Vd) one must take into account:
a) Concentration of a substance in plasma
b) Concentration of substance in urine
c) Therapeutical width of drug action
d) A daily dose of drug
024. A small amount of the volume of distribution is common for lipophylic substances easy penetrating through barriers and
widely distributing in plasma, interstitial and cell fluids:
7
a) True
b) False
025. The term “biotransformation” includes the following:
a) Accumulation of substances in a fat tissue
b) Binding of substances with plasma proteins
c) Accumulation of substances in a tissue
d) Process of physicochemical and biochemical alteration of a drug in the body
026. Biotransformation of the drugs is to render them:
a) Less ionized
b) More pharmacologically active
c) More lipid soluble
d) Less lipid soluble
027. Tick the drug type for which microsomal oxidation is the most prominent:
a) Lipid soluble
b) Water soluble
c) Low molecular weight
d) High molecular weight
028. Pick out the right statement:
a) Microsomal oxidation always results in inactivation of a compound
b) Microsomal oxidation results in a decrease of compound toxicity
c) Microsomal oxidation results in an increase of ionization and water solubility of a drug
d) Microsomal oxidation results in an increase of lipid solubility of a drug thus its excretion from the organism is facilitated
029. Stimulation of liver microsomal enzymes can:
a) Require the dose increase of some drugs
b) Require the dose decrease of some drugs
c) Prolong the duration of the action of a drug
d) Intensify the unwanted reaction of a drug
030. Metabolic transformation (phase 1) is:
a) Acetylation and methylation of substances
b) Transformation of substances due to oxidation, reduction or hydrolysis
c) Glucuronide formation
d) Binding to plasma proteins
031. Biotransformation of a medicinal substance results in:
a) Faster urinary excretion
b) Slower urinary excretion
c) Easier distribution in organism
d) Higher binding to membranes
032. Conjugation is:
a) Process of drug reduction by special enzymes
b) Process of drug oxidation by special oxidases
c) Coupling of a drug with an endogenous substrate
d) Solubilization in lipids
033. Which of the following processes proceeds in the second phase of biotransformation?
a) Acetylation
b) Reduction
c) Oxidation
d) Hydrolysis
034. Conjugation of a drug includes the following EXCEPT:
a) Glucoronidation
b) Sulfate formation
c) Hydrolysis
d) Methylation
035. Metabolic transformation and conjugation usually results in an increase of a substance biological activity:
a) True
b) False
036. In case of liver disorders accompanied by a decline in microsomal enzyme activity the duration of action of some drugs
is:
a) Decreased
b) Enlarged
c) Remained unchanged
d) Changed insignificantly
8
037. Half life (t ½) is the time required to:
a) Change the amount of a drug in plasma by half during elimination
b) Metabolize a half of an introduced drug into the active metabolite
c) Absorb a half of an introduced drug
d) Bind a half of an introduced drug to plasma proteins
038. Half life (t ½) doesn’t depend on:
a) Biotransformation
b) Time of drug absorption
c) Concentration of a drug in plasma
d) Rate of drug elimination
039. Elimination is expressed as follows:
a) Rate of renal tubular reabsorption
b) Clearance speed of some volume of blood from substance
c) Time required to decrease the amount of drug in plasma by one-half
d) Clearance of an organism from a xenobiotic
040. Elimination rate constant (Kelim) is defined by the following parameter:
a) Rate of absorption
b) Maximal concentration of a substance in plasma
c) Highest single dose
d) Half life (t ½)
041. The most rapid eliminated drugs are those with high glomerular filtration rate and actively secreted but aren’t passively
reabsorbed:
a) True
b) False
042. Systemic clearance (CLs) is related with:
a) Only the concentration of substances in plasma
b) Only the elimination rate constant
c) Volume of distribution, half life and elimination rate constant
d) Bioavailability and half life

PART II PHARMACODYNAMICS
001. Pharmacodynamics involves the study of following EXCEPT:
a) Biological and therapeutic effects of drugs
b) Absorption and distribution of drugs
c) Mechanisms of drug action
d) Drug interactions
002. Pharmacodynamics involves the study of following?
a) Mechanisms of drug action
b) Biotransformation of drugs in the organism
c) Distribution of drugs in the organism
d) Excretion of drug from the organism
003. Pharmacodynamics involves the following?
a) Information about main mechanisms of drug absorption
b) Information about unwanted effects
c) Information about biological barriers
d) Information about excretion of a drug from the organism
004. Pick out the answer which is the most appropriate to the term “receptor”
a) All types of ion channels modulated by a drug
b) Enzymes of oxidizing-reducing reactions activated by a drug
c) Active macromolecular components of a cell or an organism which a drug molecule has to combine with in
order to elicit its specific effect
d) Carriers activated by a drug
005. What does “affinity” mean?
a) A measure of how tightly a drug binds to plasma proteins
b) A measure of how tightly a drug binds to a receptor
c) A measure of inhibiting potency of a drug
d) A measure of bioavailability of a drug
006. Target proteins which a drug molecule binds are:
a) Only receptors
b) Only ion channels
c) Only carriers
9
d) All of the above
007. An agonist is a substance that:
a) Interacts with the receptor without producing any effect
b) Interacts with the receptor and initiates changes in cell function, producing various effects
c) Increases concentration of another substance to produce effect
d) Interacts with plasma proteins and doesn’t produce any effect
008. If an agonist can produce maximal effects and has high efficacy it’s called:
a) Partial agonist
b) Antagonist
c) Agonist-antagonist
d) Full agonist
009. If an agonist can produce submaximal effects and has moderate efficacy it’s called:
a) Partial agonist
b) Antagonist
c) Agonist-antagonist
d) Full agonist
010. An antagonist is a substance that:
a) Binds to the receptors and initiates changes in cell function, producing maximal effect
b) Binds to the receptors and initiates changes in cell function, producing submaximal effect
c) Interacts with plasma proteins and doesn’t produce any effect
d) Binds to the receptors without directly altering their functions
011. A competitive antagonist is a substance that:
a) Interacts with receptors and produces submaximal effect
b) Binds to the same receptor site and progressively inhibits the agonist response
c) Binds to the nonspecific sites of tissue
d) Binds to one receptor subtype as an agonist and to another as an antagonist
012. The substance binding to one receptor subtype as an agonist and to another as an antagonist is called:
a) Competitive antagonist
b) Irreversible antagonist
c) Agonist-antagonist
d) Partial agonist
013. Irreversible interaction of an antagonist with a receptor is due to:
a) Ionic bonds
b) Hydrogen bonds
c) Covalent bonds
d) All of the above
014. Mechanisms of transmembrane signaling are the following EXCEPT:
a) Transmembrane receptors that bind and stimulate a protein tyrosine kinase
b) Gene replacement by the introduction of a therapeutic gene to correct a genetic effect
c) Ligand-gated ion channels that can be induced to open or close by binding a ligand
d) Transmembrane receptor protein that stimulates a GTP-binding signal transducer protein (G-protein) which in turn
generates an intracellular second messenger
015. Tick the second messenger of G-protein-coupled (metabotropic) receptor:
a) Adenylyl cyclase
b) Sodium ions
c) Phospholipase C
d) cAMP
016. Tick the substance which changes the activity of an effector element but doesn’t belong to second messengers:
a) cAMP
b) cGMP
c) G–protein
d) Calcium ions
017. The increase of second messengers’ (cAMP, cGMP, Ca2+ etc.) concentration leads to:
a) Inhibition of intracellular protein kinases and protein phosphorylation
b) Proteinkinases activation and protein phosphorylation
c) Blocking of interaction between a receptor and an effector
d) Antagonism with endogenous ligands
018. Tick the substances whose mechanisms are based on interaction with ion channels
a) Sodium channel blockers
b) Calcium channel blockers
c) Potassium channels activators
10
d) All of the above
019. All of the following statements about efficacy and potency are true EXCEPT:
a) Efficacy is usually a more important clinical consideration than potency
b) Efficacy is the maximum effect of a drug
c) Potency is a comparative measure, refers to the different doses of two drugs that are needed to produce the same
effect
d) The ED50 is a measure of drug’s efficacy
020. Give the definition for a therapeutical dose:
a) The amount of a substance to produce the minimal biological effect
b) The amount of a substance to produce effects hazardous for an organism
c) The amount of a substance to produce the required effect in most patients
d) The amount of a substance to accelerate an increase of concentration of medicine in an organism
021. Pick out the correct definition of a toxic dose:
a) The amount of substance to produce the minimal biological effect
b) The amount of substance to produce effects hazardous for an organism
c) The amount of substance to produce the necessary effect in most of patients
d) The amount of substance to fast creation of high concentration of medicine in an organism
022. Which effect may lead to toxic reactions when a drug is taken continuously or repeatedly?
a) Refractoriness
b) Cumulative effect
c) Tolerance
d) Tachyphylaxis
023. What term is used to describe a more gradual decrease in responsiveness to a drug, taking days or weeks to develop?
a) Refractoriness
b) Cumulative effect
c) Tolerance
d) Tachyphylaxis
024. What term is used to describe a decrease in responsiveness to a drug which develops in a few minutes?
a) Refractoriness
b) Cumulative effect
c) Tolerance
d) Tachyphylaxis
025. Tachyphylaxis is:
a) A drug interaction between two similar types of drugs
b) Very rapidly developing tolerance
c) A decrease in responsiveness to a drug, taking days or weeks to develop
d) None of the above
026. Drug resistance is a term used to describe the loss of effectiveness of antimicrobial or antitumour drugs. This
consideration is:
a) True
b) False
027. Tolerance and drug resistance can be a consequence of:
a) Drug dependence
b) Increased metabolic degradation
c) Depressed renal drug excretion
d) Activation of a drug after hepatic first-pass
028. Tolerance and drug resistance can be a consequence of:
a) Change in receptors, loss of them or exhaustion of mediators
b) Increased receptor sensitivity
c) Decreased metabolic degradation
d) Decreased renal tubular secretion
029. Tolerance develops because of:
a) Diminished absorption
b) Rapid excretion of a drug
c) Both of the above
d) None of the above
030. Dependence is often associated with tolerance to a drug, a physical abstinence syndrome, and psychological
dependence (craving). This consideration is:
a) True
b) False
11
031. The situation when failure to continue administering the drug results in serious psychological and somatic disturbances is
called?
a) Tachyphylaxis
b) Sensibilization
c) Abstinence syndrome
d) Idiosyncrasy
032. What is the type of drug-to-drug interaction which is connected with processes of absorption, biotransformation,
distribution and excretion?
a) Pharmacodynamic interaction
b) Physical and chemical interaction
c) Pharmaceutical interaction
d) Pharmacokinetic interaction
033. What is the type of drug-to-drug interaction which is the result of interaction at receptor, cell, enzyme or organ level?
a) Pharmacodynamic interaction
b) Physical and chemical interaction
c) Pharmaceutical interaction
d) Pharmacokinetic interaction
034. What phenomenon can occur in case of using a combination of drugs?
a) Tolerance
b) Tachyphylaxis
c) Accumulation
d) Synergism
035. If two drugs with the same effect, taken together, produce an effect that is equal in magnitude to the sum of the effects of
the drugs given individually, it is called as:
a) Antagonism
b) Potentiation
c) Additive effect
d) None of the above
036. What does the term “potentiation” mean?
a) Cumulative ability of a drug
b) Hypersensitivity to a drug
c) Fast tolerance developing
d) Intensive increase of drug effects due to their combination
037. The types of antagonism are:
a) Summarized
b) Potentiated
c) Additive
d) Competitive
038. The term “chemical antagonism” means that:
a) two drugs combine with one another to form an inactive compound
b) two drugs combine with one another to form a more active compound
c) two drugs combine with one another to form a more water soluble compound
d) two drugs combine with one another to form a more fat soluble compound
039. A teratogenic action is:
a) Toxic action on the liver
b) Negative action on the fetus causing fetal malformation
c) Toxic action on blood system
d) Toxic action on kidneys
040. Characteristic unwanted reaction which isn’t related to a dose or to a pharmacodynamic property of a drug is called:
a) Idiosyncrasy
b) Hypersensitivity
c) Tolerance
d) Teratogenic action
041. Idiosyncratic reaction of a drug is:
a) A type of hypersensitivity reaction
b) A type of drug antagonism
c) Unpredictable, inherent, qualitatively abnormal reaction to a drug
d) Quantitatively exaggerated response
042. Therapeutic index (TI) is:
a) A ratio used to evaluate the safety and usefulness of a drug for indication
b) A ratio used to evaluate the effectiveness of a drug
12
c) A ratio used to evaluate the bioavailability of a drug
d) A ratio used to evaluate the elimination of a drug

(2) AGENTS, CONTROLLING THE FUNCTIONS OF THE PERIPHERAL NERVOUS SYSTEM

PART I Local anesthetics


001. Local anesthetics produce:
a) Analgesia, amnesia, loss of consciousness
b) Blocking pain sensation without loss of consciousness
c) Alleviation of anxiety and pain with an altered level of consciousness
d) A stupor or somnolent state
002. A good local anesthetic agent shouldn’t cause:
a) Local irritation and tissue damage
b) Systemic toxicity
c) Fast onset and long duration of action
d) Vasodilatation
003. Most local anesthetic agents consist of:
a) Lipophylic group (frequently an aromatic ring)
b) Intermediate chain (commonly including an ester or amide)
c) Amino group
d) All of the above
004. Which one of the following groups is responsible for the duration of the local anesthetic action?
a) Intermediate chain
b) Lipophylic group
c) Ionizable group
d) All of the above
005. Indicate the local anesthetic agent, which has a shorter duration of action:
a) Lidocaine
b) Procaine
c) Bupivacaine
d) Ropivacaine
006. Which one of the following groups is responsible for the potency and the toxicity of local anesthetics?
a) Ionizable group
b) Intermediate chain
c) Lipophylic group
d) All of the above
007. Indicate the drug, which has greater potency of the local anesthetic action:
a) Lidocaine
b) Bupivacaine
c) Procaine
d) Mepivacaine
008. Ionizable group is responsible for:
a) The potency and the toxicity
b) The duration of action
c) The ability to diffuse to the site of action
d) All of the above
009. Which one of the following local anesthetics is an ester of benzoic acid?
a) Lidocaine
b) Procaine
c) Ropivacaine
d) Cocaine
010. Indicate the local anesthetic, which is an ester of paraaminobenzoic acid:
a) Mepivacaine
b) Cocaine
c) Procaine
d) Lidocaine
011. Which of the following local anesthetics is an acetanilide derivative?
a) Tetracaine
b) Lidocaine
c) Cocaine
d) Procaine
13
012. Indicate the local anesthetic, which is a toluidine derivative:
a) Lidocaine
b) Bupivacaine
c) Prilocaine
d) Procaine
013. Which of the following local anesthetics is a thiophene derivative?
a) Procaine
b) Ultracaine
c) Lidocaine
d) Mepivacaine
014. Local anesthetics are:
a) Weak bases
b) Weak acids
c) Salts
d) None of the above
015. For therapeutic application local anesthetics are usually made available as salts for the reasons of:
a) Less toxicity and higher potency
b) Higher stability and greater lipid solubility
c) Less local tissue damage and more potency
d) More stability and greater water solubility
016. Which of the following statements is not correct for local anesthetics?
a) In a tissue they exist either as an uncharged base or as a cation
b) A charged cationic form penetrates biologic membranes more readily than an uncharged form
c) Local anesthetics are much less effective in inflamed tissues
d) Low ph in inflamed tissues decreases the dissociation of nonionized molecules
017. Which one of the following statements about the metabolism of local anesthetics is incorrect?
a) Metabolism of local anesthetics occurs at the site of administration
b) Metabolism occurs in the plasma or liver but not at the site of administration
c) Ester group of anesthetics like procaine, are metabolized systemically by pseudocholinesterase
d) Amides such as lidocaine, are metabolized in the liver by microsomal mixed function oxidases
018. Indicate the anesthetic agent of choice in patient with a liver disease:
a) Lidocaine
b) Bupivacaine
c) Procaine
d) Etidocaine
019. Which of the following local anesthetics is preferable in patient with pseudocholinesterase deficiency?
a) Procaine
b) Ropivacaine
c) Tetracaine
d) Benzocaine
020. The primary mechanism of action of local anesthetics is:
a) Activation of ligand-gated potassium channels
b) Blockade of voltage-gated sodium channels
c) Stimulation of voltage-gated N-type calcium channels
d) Blockade the GABA-gated chloride channels
021. Which of the following local anesthetics is more water-soluble?
a) Tetracaine
b) Etidocaine
c) Procaine
d) Bupivacaine
022. Indicate the local anesthetic, which is more lipid-soluble:
a) Bupivacaine
b) Lidocaine
c) Mepivacaine
d) Procaine
023. The more lipophylic drugs:
a) Are more potent
b) Have longer duration of action
c) Bind more extensively to proteins
d) All of the above
024. Which of the following fibers is the first to be blocked?
14
a) Type A alpha fibers
b) B and C fibers
c) Type A beta fibers
d) Type A gamma fibers
025. Indicate the function, which the last to be blocked:
a) Pain, temperature
b) Muscle spindles
c) Motor function
d) Touch, pressure
026. Which of the following fibers participates in high-frequency pain transmission?
a) Type A delta and C fibers
b) Type A alpha fibers
c) Type B fibers
d) Type A beta fibers
027. Which of the following local anesthetics is an useful antiarrhythmic agent?
a) Cocaine
b) Lidocaine
c) Bupivacaine
d) Ropivacaine
028. Indicate the route of local anesthetic administration, which is associated with instillation within epidural or subarachnoid
spaces:
a) Topical anesthesia
b) Infiltrative anesthesia
c) Regional anesthesia
d) Spinal anesthesia
029. The choice of a local anesthetic for specific procedures is usually based on:
a) The duration of action
b) Water solubility
c) Capability of rapid penetration through the skin or mucosa with limited tendency to diffuse away from the site of
application
d) All of the above
030. Which of the following local anesthetics is a short-acting drug?
a) Procaine
b) Tetracaine
c) Bupivacaine
d) Ropivacaine
031. Indicate the local anesthetic, which is a long-acting agent:
a) Lidocaine
b) Bupivacaine
c) Procaine
d) Mepivacaine
032. The anesthetic effect of the agents of short and intermediate duration of action can not be prolonged by adding:
a) Epinephrine
b) Norepinephrine
c) Dopamine
d) Phenylephrine
033. A vasoconstrictor does not:
a) Retard the removal of drug from the injection site
b) Hence the chance of toxicity
c) Decrease the blood level
d) Reduce a local anesthetic uptake by the nerve
034. Vasoconstrictors are less effective in prolonging anesthetic properties of:
a) Procaine
b) Bupivacaine
c) Lidocaine
d) Mepivacaine
035. Which of the following local anesthetics is only used for surface or topical anesthesia?
a) Cocaine
b) Tetracaine
c) Procaine
d) Bupivacaine
15
036. Indicate the local anesthetic, which is mainly used for regional nerve block anesthesia:
a) Dibucaine
b) Bupivacaine
c) Tetracaine
d) Cocaine
037. Which of the following local anesthetics is used for infiltrative and regional anesthesia?
a) Procaine
b) Lidocaine
c) Mepivacaine
d) All of the above
038. Indicate the local anesthetic, which is used for spinal anesthesia:
a) Tetracaine
b) Cocaine
c) Dibucaine
d) Bupivacaine
039. Which of the following local anesthetics is called a universal anesthetic?
a) Procaine
b) Ropivacaine
c) Lidocaine
d) Bupivacaine
040. Most serious toxic reaction to local anesthetics is:
a) Seizures
b) Cardiovascular collapse
c) Respiratory failure
d) All of the above
041. Correct statements concerning cocaine include all of the following EXCEPT:
a) Cocaine is often used for nose and throat procedures
b) Limited use because of abuse potential
c) Myocardial depression and peripheral vasodilatation
d) Causes sympathetically mediated tachycardia and vasoconstriction
042. Which of the following local anesthetics is more cardiotoxic?
a) Procaine
b) Bupivacaine
c) Lidocaine
d) Mepivacaine
043. Most local anesthetics can cause:
a) Depression of abnormal cardiac pacemaker activity, excitability, conduction
b) Depression of the strength of cardiac contraction
c) Cardiovascular collapse
d) All of the above
044. Which one of the following local anesthetics causes methemoglobinemia?
a) Prilocaine
b) Procaine
c) Lidocaine
d) Ropivacaine
045. Procaine has all of the following properties EXCEPT:
a) It has ester linkage
b) Its metabolic product can inhibit the action of sulfonamides
c) It readily penetrates the skin and mucosa
d) It is relatively short-acting
046. Correct statements concerning lidocaine include all of the following EXCEPT:
a) It is an universal anesthetic
b) It has esteratic linkage
c) It widely used as an antiarrhythmic agent
d) It is metabolized in liver
047. Which of the following local anesthetics is more likely to cause allergic reactions?
a) Lidocaine
b) Bupivacaine
c) Procaine
d) Ropivacaine
048. Tetracaine has all of the following properties EXCEPT:
16
a) Slow onset
b) Low potency
c) Long duration
d) High toxicity
049. Correct statements concerning bupivacaine include all of the following EXCEPT:
a) It has low cardiotoxicity
b) It has amide linkage
c) It is a long-acting drug
d) An intravenous injection can lead to seizures

PART II Cholinomimetic drugs


001. Acetylcholine is not a specific neurotransmitter at:
a) Sympathetic ganglia
b) Sympathetic postganglionic nerve endings
c) Parasympathetic ganglia
d) Parasympathetic postganglionic nerve endings
002. Muscarinic receptors are located in:
a) Autonomic ganglia
b) Skeletal muscle neuromuscular junctions
c) Autonomic effector cells
d) Sensory carotid sinus baroreceptor zone
003. Indicate the location of M2 cholinoreceptor type:
a) Heart
b) Glands
c) Smooth muscle
d) Endothelium
004. The symptoms of mushroom poisoning include all of the following EXCEPT:
a) Salivation, lacrimation, nausea, vomiting
b) Dryness of mouth, hyperpyrexia, hallucination
c) Headache, abdominal colic
d) Bradycardia, hypotension and shock
005. Which of the following cholinomimetics activates both muscarinic and nicotinic receptors?
a) Lobeline
b) Pilocarpine
c) Nicotine
d) Bethanechol
006. Indicate a cholinomimetic agent, which is related to direct-acting drugs:
a) Edrophonium
b) Physostigmine
c) Carbachol
d) Isoflurophate
007. Characteristics of carbachol include all of the following EXCEPT:
a) It decreases intraocular pressure
b) It causes mydriasis
c) It exerts both nicotinic and muscarinic effects
d) It is resistant to acethylcholiesterase
008. Acetylcholine is not used in clinical practice because:
a) It is very toxic
b) The doses required are very high
c) It is very rapidly hydrolyzed
d) It is very costly
009. Parasympathomimetic drugs cause:
a) Bronchodilation
b) Mydriasis
c) Bradycardia
d) Constipation
010. Which of the following direct-acting cholinomimetics is mainly muscarinic in action?
a) Bethanechol
b) Carbachol
c) Acetylcholine
d) None of the above
17
011. Which of the following direct-acting cholinomimetics has the shortest duration of action?
a) Acetylcholine
b) Methacholine
c) Carbachol
d) Bethanechol
012. Bethanechol has all of the following properties EXCEPT:
a) It is extremely resistant to hydrolysis
b) Purely muscarinic in its action
c) It is used for abdominal urinary bladder distention
d) It exerts both nicotinic and muscarinic effects
013. A M-cholinimimetic agent is:
a) Carbachol
b) Pilocarpine
c) Acetylcholine
d) Bethanechol
014. Characteristics of pilocarpine include all of the following EXCEPT:
a) It is a tertiary amine alkaloid
b) It causes miosis and a decrease in intraocular pressure
c) Causes a decrease in secretory and motor activity of gut
d) It is useful in the treatment of glaucoma
015. Which of the following cholinomimetics is a plant derivative with lower potency than nicotine but with a similar spectrum
of action?
a) Lobeline
b) Pilocarpine
c) Carbochol
d) Acetylcholine
016. Which of the following cholinomimetics is indirect-acting?
a) Lobeline
b) Edrophonium
c) Pilocarpine
d) Carbachol
017. The mechanism of action of indirect-acting cholinomimetic agents is:
a) Binding to and activation of muscarinic or nicotinic receptors
b) Inhibition of the hydrolysis of endogenous acetylcholine
c) Stimulation of the action of acetylcholinesterase
d) Releasing acetylcholine from storage sites
018. Indicate a reversible cholinesterase inhibitor:
a) Isoflurophate
b) Carbochol
c) Physostigmine
d) Parathion
019. Which of the following cholinesterase inhibitors is irreversible?
a) Physostigmine
b) Edrophonium
c) Neostigmine
d) Isoflurophate
020. Indicate cholinesterase activator:
a) Pralidoxime
b) Edrophonium
c) Pilocarpine
d) Isoflurophate
021. Isofluorophate increases all of the following effects except:
a) Lacrimation
b) Bronchodilation
c) Muscle twitching
d) Salivation
022. Indicate a cholinesterase inhibitor, which has an additional direct nicotinic agonist effect:
a) Edrophonium
b) Carbochol
c) Neostigmine
d) Lobeline
18
023. Сholinesterase inhibitors do not produce:
a) Bradycardia, no change or modest fall in blood pressure
b) Increased strength of muscle contraction, especially in muscles weakened by myasthenia gravis
c) Miosis and reduction of intraocular pressure
d) Dramatic hypertension and tachycardia
024. Which of the following cholinomimetics is commonly used in the treatment of glaucoma?
a) Pilocarpine
b) Lobeline
c) Acethylcholine
d) Neostigmine
025. Indicate the organophosphate cholinesterase inhibitor, which can be made up in an aqueous solution for ophthalmic use
and retains its activity within a week:
a) Physoctigmine
b) Edrophonium
c) Echothiophate
d) Neostigmine
026. Which of the following cholinomimetics is most widely used for paralytic ileus and atony of the urinary bladder?
a) Lobeline
b) Neostigmine
c) Pilocarpine
d) Echothiophate
027. Chronic long-term therapy of myasthenia is usually accomplished with:
a) Edrophonium
b) Neostigmine
c) Echothiophate
d) Carbachol
028. Which of the following cholinomimetics is a drug of choice for reversing the effects of nondepolarizing neuromuscular
relaxants?
a) Echothiophate
b) Physostigmine
c) Edrophonium
d) Pilocarpine
029. Indicate the reversible cholinesterase inhibitor, which penetrates the blood-brain barrier:
a) Physostigmine
b) Edrophonium
c) Neostigmine
d) Piridostigmine
030. Which of the following cholinomimetics is used in the treatment of atropine intoxication?
a) Neostigmine
b) Carbochol
c) Physostigmine
d) Lobeline
031. The symptoms of excessive stimulation of muscarinic receptors include all of the following EXCEPT:
a) Abdominal cramps, diarrhea
b) Increased salivation, excessive bronchial secretion
c) Miosis, bradycardia
d) Weakness of all skeletal muscles
032. The excessive stimulation of muscarinic receptors by pilocarpine and choline esters is blocked competitively by:
a) Edrophonium
b) Atropine
c) Pralidoxime
d) Echothiophate
033. The toxic effects of a large dose of nicotine include all of the following EXCEPT:
a) Hypotension and bradycardia
b) Convulsions, coma and respiratory arrest
c) Skeletal muscle depolarization blockade and respiratory paralysis
d) Hypertension and cardiac arrhythmias
034. The dominant initial sights of acute cholinesterase inhibitors intoxication include all of the following except:
a) Salivation, sweating
b) Mydriasis
c) Bronchial constriction
19
d) Vomiting and diarrhea
035. Which of the following drugs is used for acute toxic effects of organophosphate cholinesterase inhibitors?
a) Atropine
b) Pilocarpine
c) Pralidoxime
d) Edrophonium

PART III Cholinoreceptor blocking drugs


001. The group of nicotinic receptor-blocking drugs consists of:
a) Ganglion-blockers
b) Atropine-similar drugs
c) Neuromuscular junction blockers
d) Both a and c
002. M3 receptor subtype is located:
a) In the myocardium
b) In sympathetic postganglionic neurons
c) On effector cell membranes of glandular and smooth muscle cells
d) On the motor end plates
003. Which of the following drugs is both a muscarinic and nicotinic blocker?
a) Atropine
b) Benztropine
c) Hexamethonium
d) Succinylcholine
004. Indicate a muscarinic receptor-blocking drug:
a) Scopolamine
b) Pipecuronium
c) Trimethaphan
d) Pilocarpine
005. Which of the following agents is a ganglion-blocking drug?
a) Homatropine
b) Hexamethonium
c) Rapacuronium
d) Edrophonium
006. Indicate the skeletal muscle relaxant, which is a depolarizing agent:
a) Vencuronium
b) Scopolamine
c) Succinylcholine
d) Hexamethonium
007. Which of the following drugs is a nondepolarizing muscle relaxant?
a) Pancuronium
b) Succinylcholine
c) Hexamethonium
d) Scopolamine
008. Indicate the drug, which is rapidly and fully distributed into CNS and has a greater effect than most other antimuscarinic
agents?
a) Atropine
b) Scopolamine
c) Homatropine
d) Ipratropium
009. The effect of the drug on parasympathetic function declines rapidly in all organs EXCEPT:
a) Eye
b) Heart
c) Smooth muscle organs
d) Glands
010. The mechanism of atropine action is:
a) Competitive ganglion blockade
b) Competitive muscarinic blockade
c) Competitive neuromuscular blockade
d) Noncompetitive neuromuscular blockade
011. The tissues most sensitive to atropine are:
20
a) The salivary, bronchial and sweat glands
b) The gastric parietal cells
c) Smooth muscle and autonomic effectors
d) The heart
012. Atropine is highly selective for:
a) M1 receptor subtype
b) M2 receptor subtype
c) M3 receptor subtype
d) All of the above
013. Which of the following antimuscarinic drugs is often effective in preventing or reversing vestibular disturbances,
especially motion sickness?
a) Atropine
b) Ipratropium
c) Scopolamine
d) Homatropine
014. Atropine causes:
a) Miosis, a reduction in intraocular pressure and cyclospasm
b) Mydriasis, a rise in intraocular pressure and cycloplegia
c) Miosis, a rise in intraocular pressure and cycloplegia
d) Mydriasis, a rise in intraocular pressure and cyclospasm
015. Patients complain of dry or “sandy” eyes when receiving large doses of:
a) Atropine
b) Hexamethonium
c) Pilocarpine
d) Carbachol
016. All of the following parts of the heart are very sensitive to muscarinic receptor blockade except:
a) Atria
b) Sinoatrial node
c) Atrioventricular node
d) Ventricle
017. Atropine causes:
a) Bradycardia, hypotension and bronchoconstriction
b) Tachycardia, little effect on blood pressure and bronchodilation
c) Decrease in contractile strength, conduction velocity through the AV node
d) Tachycardia, hypertensive crisis and bronchodilation
018. Atropine is frequently used prior to administration of inhalant anesthetics to reduce:
a) Muscle tone
b) Secretions
c) Nausea and vomiting
d) All of the above
019. Atropine is now rarely used for the treatment of peptic ulcer because of:
a) Slow gastric empting and prolongation of the exposure of the ulcer bed to acid
b) Low efficiency and necessity of large doses
c) Adverse effects
d) All of the above
020. Which of the following antimuscarinic drugs is a selective M1 blocker?
a) Atropine
b) Scopolamine
c) Pirenzepine
d) Homatropine
021. Atropine causes:
a) Spasmolitic activity
b) Intestinal hypermotility
c) Stimulation of contraction in the gut
d) Stimulation of secretory activity
022. Which of the following drugs is useful in the treatment of uterine spasms?
a) Carbachol
b) Vecuronium
c) Atropine
d) Edrophonium
023. Atropine may cause a rise in body temperature (atropine fever):
21
a) In adults
b) In pregnant women
c) In infants and children
d) All of the above
024. The pharmacologic actions of scopolamine most closely resemble those of:
a) Hexamethonium
b) Atropine
c) Succinylcholine
d) Pilocarpine
025. Compared with atropine, scopolamine has all of the following properties EXCEPT:
a) More marked central effect
b) Less potent in decreasing bronchial, salivary and sweat gland secretion
c) More potent in producing mydriasis and cycloplegia
d) Lower effects on the heart, bronchial muscle and intestines
026. Which of the following drugs is useful in the treatment of Parkinson′s disease?
a) Benztropine
b) Edrophonium
c) Succinylcholine
d) Hexamethonium
027. Indicate the antimuscarinic drug, which is used as a mydriatic:
a) Pilocarpine
b) Neostigmine
c) Homatropine
d) Ipratropium
028. Which of the following agents is used as an inhalation drug in asthma?
a) Atropine
b) Ipratropium
c) Lobeline
d) Homatropine
029. Which of the following agents is most effective in regenerating cholinesterase associated with skeletal muscle
neuromuscular junctions?
a) Suscinilcholine
b) Pralidoxime
c) Pirenzepine
d) Propiverine
030. Indicate an antimuscarinic drug, which is effective in the treatment of mushroom poising:
a) Pralidoxime
b) Pilocarpine
c) Homatropine
d) Atropine
031. Antimuscarinics are used in the treatment of the following disorders EXCEPT:
a) Motion sickness
b) Glaucoma
c) Hyperhidrosis
d) Asthma
032. The atropine poisoning includes all of the following symptoms EXCEPT:
a) Mydriasis, cycloplegia
b) Hyperthermia, dry mouth, hot and flushed skin
c) Agitation and delirium
d) Bradicardia, orthostatic hypotension
033. The treatment of the antimuscarinic effects can be carried out with:
a) Neostigmine
b) Hexametonium
c) Homatropine
d) Acetylcholine
034. Contraindications to the use of antimuscarinic drugs are all of the following except:
a) Glaucoma
b) Myasthenia
c) Bronchial asthma
d) Paralytic ileus and atony of the urinary bladder
035. Hexamethonium blocks the action of acethylcholine and similar agonists at:
22
a) Muscarinic receptor site
b) Neuromuscular junction
c) Autonomic ganglia
d) Axonal transmission
036. The applications of the ganglion blockers have disappeared because of all of the following reasons EXCEPT:
a) Orthostatic hypotension
b) Lack of selectivity
c) Homeostatic reflexes block
d) Respiratory depression
037. Which of the following agents is a short-acting ganglion blocker?
a) Homatropine
b) Trimethaphane
c) Hexamethonium
d) Pancuronium
038. Indicate the ganglion-blocking drug, which can be taken orally for the treatment of hypertension?
a) Mecamylamine
b) Scopolamine
c) Trimethaphane
d) Vecocuronium
039. The systemic effects of hexamethonium include all of the following EXCEPT:
a) Reduction of both peripheral vascular resistance and venous return
b) Partial mydriasis and loss of accommodation
c) Constipation and urinary retention
d) Stimulation of thermoregulatory sweating
040. Ganglion blocking drugs are used for the following emergencies EXCEPT:
a) Hypertensive crises
b) Controlled hypotension
c) Cardiovascular collapse
d) Pulmonary edema
041. Agents that produce neuromuscular blockade act by inhibiting:
a) Interaction of acetylcholine with cholinergic receptors
b) Release of acetylcholine from prejunctional membrane
c) Packaging of acetylcholine into synaptic vesicles
d) Reuptake of acetylcholine into the nerve ending
042. Skeletal muscle relaxation and paralysis can occur from interruption of functions at several sites, including all of the
following EXCEPT:
a) Nicotinic acethylcholine receptors
b) Muscarinic acethylcholine receptors
c) The motor end plate
d) Contractile apparatus
043. Nondepolarisation neuromuscular blocking agents:
a) Block acetylcholine reuptake
b) Prevent access of the transmitter to its receptor and depolarization
c) Block transmission by an excess of a depolarizing agonist
d) All of the above
044. Which of the following drugs has “double-acetylcholine” structure?
a) Rocuronium
b) Carbachol
c) Atracurium
d) Succylcholine
045. Indicate the long-acting neuromuscular blocking agent:
a) Rapacuronium
b) Mivacurium
c) Tubocurarine
d) Rocuronium
046. Which of the following neuromuscular blocking drugs is an intermediate-duration muscle relaxant?
a) Vecuronium
b) Tubocurarine
c) Pancuronium
d) Rapacuronium
047. Indicate the nondepolarizing agent, which has the fastest onset of effect?
23
a) Succinylcholine
b) Rapacuronium
c) Pancuronium
d) Tubocurarine
048. Indicate the neuromuscular blocker, whose breakdown product readily crosses the blood-brain barrier and may cause
seizures:
a) Pancuronium
b) Succinylcholine
c) Tubocurarine
d) Atracurium
049. Which competitive neuromuscular blocking agent could be used in patients with renal failure?
a) Atracurium
b) Succinylcholine
c) Pipecuronium
d) Doxacurium
050. Indicate the nondepolarizing agent, which has short duration of action:
a) Succinylcholine
b) Tubocurarine
c) Mivacurium
d) Pancuronium
051. Which depolarizing agent has the extremely brief duration of action?
a) Mivacurium
b) Rapacuronium
c) Rocuronium
d) Succinylcholine
052. Neuromuscular blockade by both succinylcholine and mivacurium may be prolonged in patients with:
a) Renal failure
b) An abnormal variant of plasma cholinesterase
c) Hepatic disease
d) Both b and c
053. Depolarizing agents include all of the following properties EXCEPT:
a) Interact with nicotinic receptor to compete with acetylcholine without receptor activation
b) React with the nicotinic receptor to open the channel and cause depolarisation of the end plate
c) Cause desensitization, noncompetive block manifested by flaccid paralysis
d) Cholinesterase inhibitors do not have the ability to reverse the blockade
054. Which of the following neuromuscular blockers causes transient muscle fasciculations?
a) Mivacurium
b) Pancuronium
c) Succinylcholine
d) Tubocurarine
055. Indicate muscles, which are more resistant to block and recover more rapidly:
a) Hand
b) Leg
c) Neck
d) Diaphragm
056. Which neuromuscular blocking agent has the potential to cause the greatest release of histamine?
a) Succylcholine
b) Tubocurarine
c) Pancuronium
d) Rocuronium
057. Which of the following muscular relaxants causes hypotension and bronchospasm?
a) Vecuronium
b) Succinylcholine
c) Tubocurarine
d) Rapacuronium
058. Indicate the neuromuscular blocker, which causes tachycardia:
a) Tubocurarine
b) Atracurium
c) Pancuronium
d) Succinylcholine
059. Which of the following neuromuscular blocking agents cause cardiac arrhythmias?
24
a) Vecuronium
b) Tubocurarine
c) Rapacuronium
d) Succinylcholine
060. Effects seen only with depolarizing blockade include all of the following EXCEPT:
a) Hypercaliemia
b) A decrease in intraocular pressure
c) Emesis
d) Muscle pain
061. Which neuromuscular blocking agent is contraindicated in patients with glaucoma?
a) Tubocurarine
b) Succinylcholine
c) Pancuronium
d) Gallamine
062. Indicate the following neuromuscular blocker, which would be contraindicated in patients with renal failure:
a) Pipecuronium
b) Succinylcholine
c) Atracurium
d) Rapacuronium
063. All of the following drugs increase the effects of depolarizing neuromuscular blocking agents EXCEPT:
a) Aminoglycosides
b) Antiarrhythmic drugs
c) Nondepolarizing blockers
d) Local anesthetics
064. Which of the following diseases can augment the neuromuscular blockade produced by nondepolarizing muscle
relaxants?
a) Myasthenia gravis
b) Burns
c) Asthma
d) Parkinsonism
065. Indicate the agent, which effectively antagonizes the neuromuscular blockade caused by nondepolarizing drugs:
a) Atropine
b) Neostigmine
c) Acetylcholine
d) Pralidoxime

PART IV Adrenoreceptor activating drugs


001. Sympathetic stimulation is mediated by:
a) Release of norepinephrine from nerve terminals
b) Activation of adrenoreceptors on postsynaptic sites
c) Release of epinephrine from the adrenal medulla
d) All of the above
002. Characteristics of epinephrine include all of the following EXCEPT:
a) It is synthesized into the adrenal medulla
b) It is synthesized into the nerve ending
c) It is transported in the blood to target tissues
d) It directly interacts with and activates adrenoreceptors
003. Which of the following sympathomimetics acts indirectly?
a) Epinephrine
b) Norepinephrine
c) Ephedrine
d) Methoxamine
004. Indirect action includes all of the following properties EXCEPT:
a) Displacement of stored catecholamines from the adrenergic nerve ending
b) Inhibition of reuptake of catecholamines already released
c) Interaction with adrenoreceptors
d) Inhibition of the release of endogenous catecholamines from peripheral adrenergic neurons
005. Catecholamine includes following EXCEPT:
a) Ephedrine
b) Epinephrine
c) Isoprenaline
25
d) Norepinephrine
006. Epinephrine decreases intracellular camp levels by acting on:
a) α1 receptor
b) α2 receptor
c) beta1 receptor
d) beta2 receptor
007. Which of the following statements is not correct?
a) ALFA receptors increase arterial resistence, whereas beta2 receptor promote smooth muscle relaxation
b) The skin and splanchic vessels have predominantly alfa receptors
c) Vessels in a skeletal muscle may constrict or dilate depending on whether alfa or beta2 receptors are activated
d) Skeletal muscle vessels have predominantly alfa receptors and constrict in the presence of epinephrine and
norepinephrine
008. Direct effects on the heart are determined largely by:
a) Alfa1 receptor
b) Alfa2 receptor
c) Beta1 receptor
d) Beta2 receptor
009. Which of the following effects is related to direct beta1-adrenoreceptor stimulation?
a) Bronchodilation
b) Vasodilatation
c) Tachycardia
d) Bradycardia
010. Distribution of alfa adrenoreceptor subtypes is associated with all of the following tissues except those of:
a) Heart
b) Blood vessels
c) Prostate
d) Pupillary dilator muscle
011. Beta adrenoreceptor subtypes is contained in all of the following tissues EXCEPT:
a) Bronchial muscles
b) Heart
c) Pupillary dilator muscle
d) Fat cells
012. In which of the following tissues both alfa and beta1 adrenergic stimulation produces the same effect?
a) Blood vessels
b) Intestine
c) Uterus
d) Bronchial muscles
013. The effects of sympathomimetics on blood pressure are associated with their effects on:
a) The heart
b) The peripheral resistance
c) The venous return
d) All of the above
014. A relatively pure alfa agonist causes all of the following effects EXCEPT:
a) Increase peripheral arterial resistance
b) Increase venous return
c) Has no effect on blood vessels
d) Reflex bradycardia
015. A nonselective beta receptor agonist causes all of the following effects EXCEPT:
a) Increase cardiac output
b) Increase peripheral arterial resistance
c) Decrease peripheral arterial resistance
d) Decrease the mean pressure
016. Which of the following statement is not correct?
a) Αlfa agonists cause miosis
b) Αlfa agonists cause mydriasis
c) Beta antagonists decrease the production of aqueous humor
d) Αlfa agonists increase the outflow of aqueous humor from the eye
017. A bronchial smooth muscle contains:
a) Αlfa1 receptor
b) Αlfa2 receptor
c) Beta 1 receptor
26
d) Beta 2 receptor
018. All of the following agents are beta receptor agonists EXCEPT:
a) Epinephrine
b) Isoproterenol
c) Methoxamine
d) Dobutamine
019. Which of the following drugs causes bronchodilation without significant cardiac stimulation?
a) Isoprenaline
b) Terbutaline
c) Xylometazoline
d) Methoxamine
020. Αlfa-receptor stimulation includes all of the following effects EXCEPT:
a) Relaxation of gastrointestinal smooth muscle
b) Contraction of bladder base, uterus and prostate
c) Stimulation of insulin secretion
d) Stimulation of platelet aggregation
021. Beta1 receptor stimulation includes all of the following effects EXCEPT:
a) Increase in contractility
b) Bronchodilation
c) Tachycardia
d) Increase in conduction velocity in the atrioventricular node
022. Beta2 receptor stimulation includes all of the following effects EXCEPT:
a) Stimulation of renin secretion
b) Fall of potassium concentration in plasma
c) Relaxation of bladder, uterus
d) Tachycardia
023. Hyperglycemia induced by epinephrine is due to:
a) Gluconeogenesis (beta2)
b) Inhibition of insulin secretion (alfa)
c) Stimulation of glycogenolysis (beta2)
d) All of the above
024. Which of the following effects is associated with beta3-receptor stimulation?
a) Lipolysis
b) Decrease in platelet aggregation
c) Bronchodilation
d) Tachycardia
025. Which of the following statements is not correct?
a) Epinephrine acts on both alfa- and beta-receptors
b) Norepinephrine has a predominantly beta action
c) Methoxamine has a predominantly alfa action
d) Isoprenaline has a predominantly beta action
026. Indicate the drug, which is a direct-acting both alfa- and beta-receptor agonist:
a) Norepinephrine
b) Methoxamine
c) Isoproterenol
d) Ephedrine
027. Which of the following agents is an alfa1 alfa2 beta1 beta2 receptor agonist?
a) Methoxamine
b) Albuterol
c) Epinephrine
d) Norepinephrine
028. Indicate the direct-acting sympathomimetic, which is an alfa1 alfa2 beta1 receptor agonist:
a) Isoproterenol
b) Ephedrine
c) Dobutamine
d) Norepinephrine
029. Which of the following agents is an alfa1-selective agonist?
a) Norepinephrine
b) Methoxamine
c) Ritodrine
d) Ephedrine
27
030. Indicate the alfa2-selective agonist:
a) Xylometazoline
b) Epinephrine
c) Dobutamine
d) Methoxamine
031. Which of the following agents is a nonselective beta receptor agonist?
a) Norepinephrine
b) Terbutaline
c) Isoproterenol
d) Dobutamine
032. Indicate the beta1-selective agonist:
a) Isoproterenol
b) Dobutamine
c) Metaproterenol
d) Epinephrine
033. Which of the following sympathomimetics is a beta2-selective agonist?
a) Terbutaline
b) Xylometazoline
c) Isoproterenol
d) Dobutamine
034. Indicate the indirect-acting sympathomimetic agent:
a) Epinephrine
b) Phenylephrine
c) Ephedrine
d) Isoproterenol
035. Epinephrine produces all of the following effects EXCEPT:
a) Positive inotropic and chronotropic actions on the heart (beta1 receptor)
b) Increase peripheral resistance (alfa receptor)
c) Predominance of alfa effects at low concentration
d) Skeletal muscle blood vessel dilatation (beta2 receptor)
036. Epinephrine produces all of the following effects EXCEPT:
a) Decrease in oxygen consumption
b) Bronchodilation
c) Hyperglycemia
d) Mydriasis
037. Epinephrine is used in the treatment of all of the following disorders EXCEPT:
a) Bronchospasm
b) Anaphylactic shock
c) Cardiac arrhythmias
d) Open-angle glaucoma
038. Compared with epinephrine, norepinephrine produces all of the following effects EXCEPT:
a) Similar effects on beta1 receptors in the heart and similar potency at an alfa receptor
b) Decrease the mean pressure below normal before returning to the control value
c) Significant tissue necrosis if injected subcutaneously
d) Increase both diastolic and systolic blood pressure
039. Norepinephrine produces:
a) Vasoconstriction
b) Vasodilatation
c) Bronchodilation
d) Decresed potassium concentration in the plasma
040. Which of the following direct-acting drugs is a relatively pure alfa agonist, an effective mydriatic and decongestant and
can be used to raise blood pressure?
a) Epinephrine
b) Norepinephrine
c) Phenylephrine
d) Ephedrine
041. Characteristics of methoxamine include all of the following EXCEPT:
a) It is a direct-acting alfa1-receptor agonist
b) It increases heart rate, contractility and cardiac output
c) It causes reflex bradycardia
d) It increases total peripheral resistance
28
042. Which of the following agents is an alfa2-selective agonist with ability to promote constriction of the nasal mucosa?
a) Xylometazoline
b) Phenylephrine
c) Methoxamine
d) Epinephrine
043. Indicate the sympathomimetic, which may cause hypotension, presumably because of a clonidine-like effect:
a) Methoxamine
b) Phenylephrine
c) Xylometazoline
d) Isoproterenol
044. Isoproterenol is:
a) Both an alfa- and beta-receptor agonist
b) beta1-selective agonist
c) beta2-selective agonist
d) Nonselective beta receptor agonist
045. Isoproterenol produces all of the following effects EXCEPT:
a) Increase in cardiac output
b) Fall in diastolic and mean arterial pressure
c) Bronchoconstriction
d) Tachycardia
046. Characteristics of dobutamine include all of the following EXCEPT:
a) It is a relatively beta1-selective synthetic catecholamine
b) It is used to treat bronchospasm
c) It increases atrioventricular conduction
d) It causes minimal changes in heart rate and systolic pressure
047. Characteristics of salmeterol include all of the following EXCEPT:
a) It is a potent selective beta2 agonist
b) It causes uterine relaxation
c) It stimulates heart rate, contractility and cardiac output
d) It is used in the therapy of asthma
048. Characteristics of ephedrine include all of the following EXCEPT:
a) It acts primarily through the release of stored cathecholamines
b) It is a mild CNS stimulant
c) It causes tachyphylaxis with repeated administration
d) It decreases arterial pressure
049. Ephedrine causes:
a) Miosis
b) Bronchodilation
c) Hypotension
d) Bradycardia
050. Compared with epinephrine, ephedrine produces all of the following features EXCEPT:
a) It is a direct-acting sympathomimetic
b) It has oral activity
c) It is resistant to MAO and has much longer duration of action
d) Its effects are similar, but it is less potent
051. Which of the following sympathomimetics is preferable for the treatment of chronic orthostatic hypotension?
a) Epinephrine
b) Norepinephrine
c) Ephedrine
d) Salmeterol
052. Indicate the sympathomimetic drug, which is used in a hypotensive emergency:
a) Xylometazoline
b) Ephedrine
c) Terbutaline
d) Phenylephrine
053. Which of the following sympathomimetics is preferable for the emergency therapy of cardiogenic shock?
a) Epinephrine
b) Dobutamine
c) Isoproterenol
d) Methoxamine
29
054. Indicate the sympathomimetic agent, which is combined with a local anesthetic to prolong the duration of infiltration
nerve block:
a) Epinephrine
b) Xylometazoline
c) Isoproterenol
d) Dobutamine
055. Which of the following sympathomimetics is related to short-acting topical decongestant agents?
a) Xylometazoline
b) Terbutaline
c) Phenylephrine
d) Norepinephrine
056. Indicate the long-acting topical decongestant agents:
a) Epinephrine
b) Norepinephrine
c) Phenylephrine
d) Xylometazoline
057. Which of the following topical decongestant agents is an alfa2-selective agonist?
a) Phenylephrine
b) Xylometazoline
c) Ephedrine
d) Epinephrine
058. Indicate the sympathomimetic, which may be useful in the emergency management of cardiac arrest:
a) Methoxamine
b) Phenylephrine
c) Epinephrine
d) Xylometazoline
059. Which of the following sympathomimetics is used in the therapy of bronchial asthma?
a) Formoterol
b) Norepinephrine
c) Methoxamine
d) Dobutamine
060. Indicate the agent of choice in the emergency therapy of anaphylactic shock:
a) Methoxamine
b) Terbutaline
c) Norepinephrine
d) Epinephrine
061. Which of the following sympathomimetics is an effective mydriatic?
a) Salmeterol
b) Phenylephrine
c) Dobutamine
d) Norepinephrine
062. The adverse effects of sympathomimetics include all of the following EXCEPT:
a) Drug-induced parkinsonism
b) Cerebral hemorrhage or pulmonary edema
c) Myocardial infarction
d) Ventricular arrhythmias

PART V Adrenoreceptor antagonist drugs


001. Which of the following drugs is a nonselective alfa receptor antagonist?
a) Prazosin
b) Phentolamine
c) Metoprolol
d) Reserpine
002. Indicate the alfa1-selective antagonist:
a) Phentolamine
b) Dihydroergotamine
c) Prazosin
d) Labetalol
003. Which of the following agents is an alfa2–selective antagonist?
a) Yohimbine
b) Tamsulosin
30
c) Tolazoline
d) Prazosin
004. Indicate the irreversible alfa receptor antagonist:
a) Tolazoline
b) Labetalol
c) Prazosin
d) Phenoxybenzamine
005. Which of the following drugs is an nonselective beta receptor antagonist?
a) Metoprolol
b) Atenolol
c) Propranolol
d) Acebutolol
006. Indicate the beta1-selective antagonist:
a) Propranolol
b) Metoprolol
c) Carvedilol
d) Sotalol
007. Which of the following agents is a beta2–selective antagonist?
a) Tolazolin
b) Pindolol
c) Ergotamin
d) Butoxamine
008. Indicate the beta adrenoreceptor antagonist, which has partial beta–agonist activity:
a) Propranolol
b) Metoprolol
c) Pindolol
d) Betaxolol
009. Which of the following drugs is a reversible nonselective alfa, beta antagonist?
a) Labetalol
b) Phentolamine
c) Metoprolol
d) Propranolol
010. Indicate the indirect-acting adrenoreceptor blocking drug:
a) Tolazoline
b) Reserpine
c) Carvedilol
d) Prazosin
011. The principal mechanism of action of adrenoreceptor antagonists is:
a) Reversible or irreversible interaction with adrenoreceptors
b) Depletion of the storage of catecholamines
c) Blockade of the amine reuptake pumps
d) Nonselective MAO inhibition
012. Characteristics of alfa-receptor antagonists include all of the following EXCEPT:
a) They cause a fall in peripheral resistance and blood pressure
b) They cause epinephrine reversal (convert a pressor response to a depressor response)
c) Bronchospasm
d) They may cause postural hypotension and reflex tachycardia
013. Which of the following drugs is an imidazoline derivative and a potent competitive antagonist at both alfa1 and alfa2
receptors?
a) Prazosin
b) Labetalol
c) Phenoxybenzamine
d) Phentolamine
014. Characteristics of phentolamine include all of the following EXCEPT:
a) Reduction in peripheral resistance
b) Stimulation of responses to serotonin
c) Tachycardia
d) Stimulation of muscarinic, H1 and H2 histamine receptors
015. The principal mechanism of phentolamine-induced tachycardia is:
a) Antagonism of presynaptic alfa2 receptors enhances norepinephrine release, which causes cardiac
stimulation via unblocked beta receptors
31
b) Baroreflex mechanism
c) Direct effect on the heart by stimulation of beta1 receptors
d) Inhibition of transmitter reuptake at noradrenergic synapses
016. Nonselective alfa-receptor antagonists are most useful in the treatment of:
a) Asthma
b) Cardiac arrhythmias
c) Pheochromocytoma
d) Chronic hypertension
017. The main reason for using alfa-receptor antagonists in the management of pheochromocytoma is:
a) Inhibition of the release of epinephrine from the adrenal medulla
b) Blockade of alfa2 receptors on vascular smooth muscle results in epinephrine stimulation of unblocked alfa2
receptors
c) Direct interaction with and inhibition of beta2 adrenoreceptors
d) Antagonism to the release of renin
018. Which of the following drugs is useful in the treatment of pheochromocytoma?
a) Phenylephrine
b) Propranolol
c) Phentolamine
d) Epinephrine
019. Indicate adrenoreceptor antagonist agents, which are used for the management of pheochromocytoma:
a) Selective beta2-receptor antagonists
b) Nonselective beta-receptor antagonists
c) Indirect-acting adrenoreceptor antagonist drugs
d) Αlfa-receptor antagonists
020. The principal adverse effects of phentolamine include all of the following EXCEPT:
a) Diarrhea
b) Bradycardia
c) Arrhythmias
d) Myocardial ischemia
021. Indicate the reversible nonselective alfa-receptor antagonist, which is an ergot derivative:
a) Ergotamine
b) Prazosin
c) Phenoxybenzamine
d) Carvedilol
022. Indicate an alfa-receptor antagonist, which binds covalently to alfa receptors, causing irreversible blockade of long
duration (14-48 hours or longer):
a) Phentolamine
b) Phenoxybenzamine
c) Ergotamine
d) Prazosin
023. Compared with phentolamine, prazosin has all of the following features EXCEPT:
a) Irreversible blockade of alfa receptors
b) Highly selective for alfa1 receptors
c) The relative absence of tachycardia
d) Persistent block of alfa1 receptors
024. Which of the following statements is not correct?
a) There are at least three subtypes of alfa1 receptors, designated alfa1a, alfa1b and alfa1d
b) ALFA1a subtype mediates prostate smooth muscle contraction
c) ALFA1b subtype mediates vascular smooth muscle contraction
d) ALFA1a subtype mediates both vascular and prostate smooth muscle contraction
025. Indicate an alfa1 adrenoreceptor antagonist, which has great selectivity for alfa1a subtype:
a) Prazosin
b) Tamsulosin
c) Phenoxybenzamine
d) Phentolamine
026. Subtype-selective alfa1 receptor antagonists such as tamsulosin, terazosin, alfusosin are efficacious in:
a) Hyperthyroidism
b) Cardiac arrhythmias
c) Benign prostatic hyperplasia (BPH)
d) Asthma
32
027. Indicate an alfa receptor antagonist, which is an efficacious drug in the treatment of mild to moderate systemic
hypertension:
a) Phentolamine
b) Tolazoline
c) Ergotamine
d) Prazosin
028. Which of the following alfa receptor antagonists is useful in reversing the intense local vasoconstriction caused by
inadvertent infiltration of norepinephrine into subcutaneous tissue during intravenous administration?
a) Propranolol
b) Phentolamine
c) Tamsulosin
d) Ergotamine
029. Beta-blocking drugs-induced chronically lower blood pressure may be associated with theirs effects on:
a) The heart
b) The blood vessels
c) The renin-angiotensin system
d) All of the above
030. Characteristics of beta-blocking agents include all of the following EXCEPT:
a) They occupy beta receptors and competitively reduce receptor occupancy by catecholamines or other beta agonists
b) They do not cause hypotension in individuals with normal blood pressure
c) They induce depression and depleted stores of catecholamines
d) They can cause blockade in the atrioventricular node
031. Beta-receptor antagonists have all of the following cardiovascular effects EXCEPT:
a) The negative inotropic and chronotropic effects
b) Acute effects of these drugs include a fall in peripheral resistance
c) Vasoconstriction
d) Reduction of the release of renin
032. Beta-blocking agents have all of the following effects except:
a) Increase plasma concentrations of HDL and decrease of VLDL
b) Bronchoconstriction
c) Decrease of aqueous humor prodaction
d) “membrane-stabilizing” action
033. Beta-receptor antagonists cause:
a) Stimulation of lipolysis
b) Stimulation of gluconeogenesis
c) Inhibition of glycogenolysis
d) Stimulation of insulin secretion
034. Propranolol has all of the following cardiovascular effects EXCEPT:
a) It decreases cardiac work and oxygen demand
b) It reduces blood flow to the brain
c) It inhibits the renin secretion
d) It increases the atrioventricular nodal refractory period
035. Propranolol-induced adverse effects include all of the following EXCEPT:
a) Bronchoconstriction
b) “supersensitivity” of beta-adrenergic receptors (rapid withdrawal)
c) Hyperglycemia
d) Sedation, sleep disturbances, depression and sexual dysfunction
036. Propranolol is used in the treatment all of the following diseases EXCEPT:
a) Cardiovascular diseases
b) Hyperthyroidism
c) Migraine headache
d) Bronchial asthma
037. Metoprolol and atenolol:
a) Are members of the beta1-selective group
b) Are nonselective beta antagonists
c) Have intrinsic sympathomimetic activity
d) Have an anesthetic action
038. Which of the following beta receptor antagonists is preferable in patients with asthma, diabetes or peripheral vascular
diseases?
a) Propranolol
b) Metoprolol
33
c) Nadolol
d) Timolol
039. Indicate a beta receptor antagonist, which has very long duration of action:
a) Metoprolol
b) Propranolol
c) Nadolol
d) Pindolol
040. Indicate a beta1-selective receptor antagonist, which has very long duration of action:
a) Betaxolol
b) Sotalol
c) Nadolol
d) Metoprolol
041. Which of the following drugs is a nonselective beta-blocker without intrinsic sympathomimetic or local anesthetic activity
and used for the treatment of life-threatening ventricular arrhythmias?
a) Propranolol
b) Oxprenolol
c) Sotalol
d) Atenolol
042. Indicate a beta receptor antagonist with intrinsic sympathomimetic activity:
a) Propranolol
b) Oxprenolol
c) Metoprolol
d) Carvedilol
043. Pindolol, oxprenolol have all of the following properties EXCEPT:
a) They are nonselective beta antagonists
b) They have no partial agonist activity
c) They are less likely to cause bradycardia and abnormalities in plasma lipids
d) They are effective in hypertension and angina
044. Which of the following drugs has both alfa1-selective and beta-blocking effects?
a) Labetalol
b) Betaxolol
c) Propranolol
d) Timolol
045. Characteristics of carvedilol include all of the following EXCEPT:
a) It is a beta1-selective antagonist
b) It has both alfa1-selective and beta-blocking effects
c) It attenuates oxygen free radical-initiated lipid peroxidation
d) It inhibits vascular smooth muscle mitogenesis
046. Indicate the adrenoreceptor antagonist drug, which is a rauwolfia alkaloid:
a) Prazosin
b) Propranolol
c) Reserpine
d) Phentolamine
047. Characteristics of reserpine include all of the following EXCEPT:
a) It inhibits the uptake of norepinephrine into vesicles and MAO
b) It decreases cardiac output, peripheral resistance and inhibits pressor reflexes
c) It may cause a transient sympathomimetic effect
d) It depletes stores of catecholamines and serotonin in the brain
048. Indicate a beta-blocker, which is particularly efficacious in thyroid storm:
a) Pindolol
b) Sotalol
c) Phentolamine
d) Propranolol
049. Beta-receptor blocking drugs are used in the treatment all of the following diseases EXCEPT:
a) Hypertension, ischemic heart disease, cardiac arrhythmias
b) Glaucoma
c) Pheochromocytoma
d) Hyperthyroidism
050. Beta-blocker-induced adverse effects include all of the following EXCEPT:
a) Bronchoconstriction
b) Depression of myocardial contractility and excitability
34
c) “supersensitivity” of beta-receptors associated with rapid withdrawal of drugs
d) Hyperglycemia

(3) AGENTS, CONTROLLING THE FUNCTIONS OF THE CENTRAL NERVOUS SYSTEM

PART I Hypnotic drugs


001. Hypnotic drugs are used to treat:
a) Psychosis
b) Sleep disorders
c) Narcolepsy
d) Parkinsonian disorders
002. Hypnotic drugs should:
a) Reduce anxiety and exert a calming effect
b) Induce absence of sensation
c) Produce drowsiness, encourage the onset and maintenance of sleep
d) Prevent mood swings in patients with bipolar affective disorders
003. Which of the following chemical agents are used in the treatment of insomnia?
a) Benzodiazepines
b) Imidazopyridines
c) Barbiturates
d) All of the above
004. Select a hypnotic drug, which is a benzodiazepine derivative:
a) Zolpidem
b) Flurazepam
c) Secobarbital
d) Phenobarbitone
005. Tick a hypnotic agent – a barbituric acid derivative:
a) Flurazepam
b) Zaleplon
c) Thyopental
d) Triazolam
006. Select a hypnotic drug, which is an imidazopyridine derivative:
a) Pentobarbital
b) Temazepam
c) Zolpidem
d) Chloral hydrate
007. Which of the following hypnotic agents is absorbed slowly?
a) Phenobarbital
b) Flurazepam
c) Triazolam
d) Temazepam
008. Which of the following barbiturates is an ultra-short-acting drug?
a) Secobarbital
b) Amobarbital
c) Thiopental
d) Phenobarbital
009. Indicate the barbituric acid derivative, which has 4-5 days elimination half-life:
a) Secobarbital
b) Thiopental
c) Phenobarbital
d) Amobarbital
010. Indicate the hypnotic benzodiazepine, which has the shortest elimination half-life:
a) Temazepam
b) Triazolam
c) Flurazepam
d) Diazepam
011. Which of the following hypnotic drugs is more likely to cause cumulative and residual effects?
a) Zolpidem
b) Temazepam
c) Phenobarbital
d) Triazolam
35
012. Which of the following hypnotic drugs increases the activity of hepatic drug-metabolizing enzyme systems?
a) Phenobarbital
b) Zolpidem
c) Flurazepam
d) Zaleplon
013. Hepatic microsomal drug-metabolizing enzyme induction leads to:
a) Barbiturate tolerance
b) Cumulative effects
c) Development of physical dependence
d) “hangover” effects
014. Hypnotic benzodiazepines are more powerful enzyme inducers than barbiturates.
a) True
b) False
015. Indicate the hypnotic drug, which does not change hepatic drug-metabolizing enzyme activity?
a) Flurazepam
b) Zaleplon
c) Triazolam
d) All of the above
016. Barbiturates increase the rate of metabolism of:
a) Anticoagulants
b) Digitalis compounds
c) Glucocorticoids
d) All of the above
017. Which of the following agents inhibits hepatic metabolism of hypnotics?
a) Flumasenil
b) Cimetidin
c) Phenytoin
d) Theophylline
018. Which of the following factors can influence the biodisposition of hypnotic agents?
a) Alterations in the hepatic function resulting from a disease
b) Old age
c) Drug-induced increases or decreases in microsomal enzyme activities
d) All of the above
019. Which of the following hypnotics is preferred for elderly patients?
a) Phenobarbital
b) Flurozepam
c) Temazepam
d) Secobarbital
020. Which of the following hypnotics is preferred in patients with limited hepatic function?
a) Zolpidem
b) Amobarbital
c) Flurozepam
d) Pentobarbital
021. Indicate the mechanism of barbiturate action (at hypnotic doses):
a) Increasing the duration of the GABA-gated Cl- channel openings
b) Directly activating the chloride channels
c) Increasing the frequency of Cl- channel opening events
d) All of the above
022. Imidazopyridines are:
a) Partial agonists at brain 5-TH1A receptors
b) Selective agonists of the BZ1 (omega1) subtype of BZ receptors
c) Competitive antagonists of BZ receptors
d) Nonselective agonists of both BZ1 and BZ2 receptor subtypes
023. Which of the following hypnotic agents is a positive allosteric modulator of GABAA receptor function?
a) Zaleplon
b) Flurazepam
c) Zolpidem
d) All of the above
024. Indicate a hypnotic drug - a selective agonist at the BZ1 receptor subtype:
a) Flurazepam
b) Zolpidem
36
c) Triazolam
d) Flumazenil
025. Which of the following hypnotic agents is able to interact with both BZ1 and BZ2 receptor subtypes?
a) Zaleplon
b) Phenobarbital
c) Flurazepam
d) Zolpidem
026. Indicate the competitive antagonist of BZ receptors:
a) Flumazenil
b) Picrotoxin
c) Zolpidem
d) Temazepam
027. Flumazenil blocks the actions of:
a) Phenobarbital
b) Morphine
c) Zolpidem
d) Ethanol
028. Indicate the agent, which interferes with GABA binding:
a) Flurazepam
b) Bicuculline
c) Thiopental
d) Zolpidem
029. Which of the following agents blocks the chloride channel directly?
a) Secobarbital
b) Flumazenil
c) Zaleplon
d) Picrotoxin
030. Which of the following agents is preferred in the treatment of insomnia?
a) Barbiturates
b) Hypnotic benzodiazepines
c) Ethanol
d) Phenothiazide
031. Barbiturates are being replaced by hypnotic benzodiazepines because of:
a) Low therapeutic index
b) Suppression in REM sleep
c) High potential of physical dependence and abuse
d) All of the above
032. Which of the following benzodiazepines is used mainly for hypnosis?
a) Clonozepam
b) Lorazepam
c) Flurazepam
d) Midazolam
033. Indicate the main claim for an ideal hypnotic agent:
a) Rapid onset and sufficient duration of action
b) Minor effects on sleep patterns
c) Minimal “hangover” effects
d) All of the above
034. Which stage of sleep is responsible for the incidence of dreams?
a) REM sleep
b) Slow wave sleep
c) Stage 2NREM sleep
d) All of the above
035. During slow wave sleep (stage 3 and 4 NREM sleep):
a) Dreams occur
b) The secretion of adrenal steroids is at its highest
c) Somnambulism and nightmares occur
d) The secretion of somatotropin is at its lowest
036. All of the hypnotic drugs induce:
a) Increase the duration of REM sleep
b) Decrease the duration of REM sleep
c) Do not alter the duration of REM sleep
37
d) Increase the duration of slow wave sleep
037. Which of the following hypnotic drugs causes least suppression of REM sleep?
a) Flumazenil
b) Phenobarbital
c) Flurazepam
d) Secobarbital
038. Although the benzodiazepines continue to be the agents of choice for insomnia, they have:
a) The possibility of psychological and physiological dependence
b) Synergistic depression of CNS with other drugs (especially alcohol)
c) Residual drowsiness and daytime sedation
d) All of the above
039. Hypnotic benzodiazepines can cause:
a) A dose-dependent increase in both REM and slow wave sleep
b) Do not change sleep patterns
c) A dose-dependent decrease in both REM and slow wave sleep
d) A dose-dependent increase in REM sleep and decrease in slow wave sleep
040. Which one of the following hypnotic benzodiazepines is more likely to cause rebound insomnia?
a) Triazolam
b) Flurazepam
c) Temazepam
d) All of the above
041. Which of the following hypnotic benzodiazepines is more likely to cause “hangover” effects such as drowsiness,
dysphoria, and mental or motor depression the following day?
a) Temazepam
b) Triazolam
c) Flurazepam
d) None of the above
042. Indicate the hypnotic drug, which binds selectively to the BZ1 receptor subtype, facilitating GABAergic inhibition:
a) Thiopental
b) Zolpidem
c) Flurazepam
d) Phenobarbital
043. Which of the following statements is correct for zolpidem?
a) Causes minor effects on sleep patterns
b) The risk of development of tolerance and dependence is less than with the use of hypnotic benzodiazepines
c) Has minimal muscle relaxing and anticonvulsant effects
d) All of the above
044. Which agent exerts hypnotic activity with minimal muscle relaxing and anticonvulsant effects?
a) Flurazepam
b) Triazolam
c) Zaleplon
d) None of the above
045. Zolpidem and zaleplon have effectiveness similar to that of hypnotic benzodiazepines in the management of sleep
disorders.
a) True
b) False
046. Which of the following hypnotic drugs is used intravenously as anesthesia?
a) Thiopental
b) Phenobarbital
c) Flurazepam
d) Zolpidem
047. Indicate the usual cause of death due to overdose of hypnotics:
a) Depression of the medullar respiratory center
b) Hypothermia
c) Cerebral edema
d) Status epilepticus
048. Toxic doses of hypnotics may cause a circulatory collapse as a result of:
a) Blocking alfa adrenergic receptors
b) Increasing vagal tone
c) Action on the medullar vasomotor center
d) All of the above
38
PART II Antiseizure drugs
001. The mechanism of action of antiseizure drugs is:
a) Enhancement of GABAergic (inhibitory) transmission
b) Diminution of excitatory (usually glutamatergic) transmission
c) Modification of ionic conductance
d) All of the above mechanisms
002. Which of the following antiseizure drugs produces enhancement of GABA-mediated inhibition?
a) Ethosuximide
b) Carbamazepine
c) Phenobarbital
d) Lamotrigine
003. Indicate an antiseizure drug, which has an impotent effect on the T-type calcium channels in thalamic neurons?
a) Carbamazepin
b) Lamotrigine
c) Ethosuximide
d) Phenytoin
004. Which of the following antiseizure drugs produces a voltage-dependent inactivation of sodium channels?
a) Lamotrigine
b) Carbamazepin
c) Phenytoin
d) All of the above
005. Indicate an antiseizure drug, inhibiting central effects of excitatory amino acids:
a) Ethosuximide
b) Lamotrigine
c) Diazepam
d) Tiagabine
006. The drug for partial and generalized tonic-clonic seizures is:
a) Carbamazepine
b) Valproate
c) Phenytoin
d) All of the above
007. Indicate an anti-absence drug:
a) Valproate
b) Phenobarbital
c) Carbamazepin
d) Phenytoin
008. The drug against myoclonic seizures is:
a) Primidone
b) Carbamazepine
c) Clonazepam
d) Phenytoin
009. The most effective drug for stopping generalized tonic-clonic status epilepticus in adults is:
a) Lamotrigine
b) Ethosuximide
c) Diazepam
d) Zonisamide
010. Select the appropriate consideration for phenytoin:
a) It blocks sodium channels
b) It binds to an allosteric regulatory site on the GABA-BZ receptor and prolongs the openings of the Cl-channels
c) It effects on Ca2+ currents, reducing the low-threshold (T-type) current
d) It inhibits GABA-transaminase, which catalyzes the breakdown of GABA
011. Phenytoin is used in the treatment of:
a) Petit mal epilepsy
b) Grand mal epilepsy
c) Myoclonic seizures
d) All of the above
012. Dose-related adverse effect caused by phenytoin is:
a) Physical and psychological dependence
b) Exacerbated grand mal epilepsy
c) Gingival hyperplasia
d) Extrapyramidal symptoms
39
013. Granulocytopenia, gastrointestinal irritation, gingival hyperplasia, and facial hirsutism are possible adverse effects of:
a) Phenobarbital
b) Carbamazepin
c) Valproate
d) Phenytoin
014. The antiseizure drug, which induces hepatic microsomal enzymes, is:
a) Lamotrigine
b) Phenytoin
c) Valproate
d) None of the above
015. The drug of choice for partial seizures is:
a) Carbamazepin
b) Ethosuximide
c) Diazepam
d) Lamotrigine
016. The mechanism of action of carbamazepine appears to be similar to that of:
a) Benzodiazepines
b) Valproate
c) Phenytoin
d) Ethosuximide
017. Which of the following antiseizure drugs is also effective in treating trigeminal neuralgia?
a) Primidone
b) Topiramat
c) Carbamazepine
d) Lamotrigine
018. The most common dose-related adverse effects of carbamazepine are:
a) Diplopia, ataxia, and nausea
b) Gingival hyperplasia, hirsutism
c) Sedation, physical and psychological dependence
d) Hemeralopia, myasthenic syndrome
019. Indicate the drug of choice for status epilepticus in infants and children:
a) Phenobarbital sodium
b) Clonazepam
c) Ethosuximide
d) Phenytoin
020. Barbiturates are used in the emergency treatment of status epilepticus in infants and children because of:
a) They significantly decrease of oxygen utilization by the brain, protecting cerebral edema and ischemia
b) Short onset and duration of action
c) They do not have effect on sleep architecture
d) All of the above
021. Which of the following antiseizure drugs binds to an allosteric regulatory site on the GABA-BZ receptor, increases the
duration of the Cl-channels openings:
a) Diazepam
b) Valproate
c) Phenobarbital
d) Topiramate
022. Adverse effect caused by phenobarbital is:
a) Physical and phychological dependence
b) Exacerbated petit mal epilepsy
c) Sedation
d) All of the above
023. Which of the following antiseizure drugs is a prodrug, metabolized to phenobarbital?
a) Phenytoin
b) Primidone
c) Felbamate
d) Vigabatrin
024. Indicate the antiseizure drug, which is a phenyltriazine derivative:
a) Phenobarbital
b) Clonazepam
c) Lamotrigine
d) Carbamazepin
40
025. Lamotrigine can be used in the treatment of:
a) Partial seizures
b) Absence
c) Myoclonic seizures
d) All of the above
026. The mechanism of vigabatrin′s action is:
a) Direct action on the GABA receptor-chloride channel complex
b) Inhibition of GABA aminotransferase
c) NMDA receptor blockade via the glycine binding site
d) Inhibition of GABA neuronal reuptake from synapses
027. Indicate an irreversible inhibitor of GABA aminotransferase (GABA-T):
a) Diazepam
b) Phenobarbital
c) Vigabatrin
d) Felbamate
028. Tiagabine:
a) Blocks neuronal and glial reuptake of GABA from synapses
b) Inhibits GABA-T, which catalyzed the breakdown of GABA
c) Blocks the T-type Ca2+ channels
d) Inhibits glutamate transmission at AMPA/kainate receptors
029. The mechanism of both topiramate and felbamate action is:
a) Reduction of excitatory glutamatergic neurotransmission
b) Inhibition of voltage sensitive Na+ channels
c) Potentiation of GABAergic neuronal transmission
d) All of the above
030. The drug of choice in the treatment of petit mal (absence seizures) is:
a) Phenytoin
b) Ethosuximide
c) Phenobarbital
d) Carbamazepin
031. The dose-related adverse effect of ethosuximide is:
a) Gastrointestinal reactions, such as anorexia, pain, nausea and vomiting
b) Exacerbated grand mal epilepsy
c) Transient lethargy or fatigue
d) All of the above
032. Valproate is very effective against:
a) Absence seizures
b) Myoclonic seizures
c) Generalized tonic-clonic seizures
d) All of the above
033. The drug of choice in the treatment of myoclonic seizures is:
a) Valproate
b) Phenobarbital
c) Phenytoin
d) Felbamate
034. The reason for preferring ethosuximide to valproate for uncomplicated absence seizures is:
a) More effective
b) Valproate′s idiosyncratic hepatotoxicity
c) Greater CNS depressant activity
d) All of the above
035. The mechanism of valproate action is:
a) Facilitation glutamic acid decarboxylase, the enzyme responsible for GABA synthesis and inhibition of GABA-
aminotransferase, the enzyme responsible for the breakdown of GABA (enhance GABA accumulation)
b) Inhibition of voltage sensitive Na+ channels
c) Inhibition of low threshold (T-type) Ca2+ channels
d) All of the above
036. Indicate the antiseizure drug, which is a sulfonamide derivative, blocking Na+ channels and having additional ability to
inhibit T-type Ca2+ channels:
a) Tiagabine
b) Zonisamide
c) Ethosuximide
41
d) Primidone
037. Indicate the antiseizure drug – a benzodiazepine receptor agonist:
a) Phenobarbital
b) Phenytoin
c) Carbamazepine
d) Lorazepam
038. Which of the following antiseizure drugs acts directly on the GABA receptor-chloride channel complex?
a) Vigabatrin
b) Diazepam
c) Gabapentin
d) Valproate
039. Benzodiazepine΄s uselfulness is limited by:
a) Tolerance
b) Atropine-like symptoms
c) Psychotic episodes
d) Myasthenic syndrome
040. A long-acting drug against both absence and myoclonic seizures is:
a) Primidone
b) Carbamazepine
c) Clonazepam
d) Phenytoin
041. Which of the following antiseizure drugs may produce teratogenicity?
a) Phenytoin
b) Valproate
c) Topiramate
d) All of the above
042. The most dangerous effect of antiseizure drugs after large overdoses is:
a) Respiratory depression
b) Gastrointestinal irritation
c) Alopecia
d) Sedation

PART III Antiparkinsonian agents


001. Which neurons are involved in parkinsonism?
a) Cholinergic neurons
b) GABAergic neurons
c) Dopaminergic neurons
d) All of the above
002. The pathophysiologic basis for antiparkinsonism therapy is:
a) A selective loss of dopaminergic neurons
b) The loss of some cholinergic neurons
c) The loss of the GABAergic cells
d) The loss of glutamatergic neurons
003. Which of the following neurotransmitters is involved in Parkinson′s disease?
a) Acetylcholine
b) Glutamate
c) Dopamine
d) All of the above
004. The concentration of dopamine in the basal ganglia of the brain is reduced in parkinsonism.
a) True
b) False
005. Principal aim for treatment of Parkinsonian disorders is:
a) To restore the normal balance of cholinergic and dopaminergic influences on the basal ganglia with antimuscarinic
drugs
b) To restore dopaminergic activity with levodopa and dopamine agonists
c) To decrease glutamatergic activity with glutamate antagonists
d) All of the above
006. Indicate the drug that induces parkinsonian syndromes:
a) Chlorpromazine
b) Diazepam
42
c) Triazolam
d) Carbamazepine
007. Which of the following drugs is used in the treatment of Parkinsonian disorders?
a) Phenytoin
b) Selegiline
c) Haloperidol
d) Fluoxetine
008. Select the agent, which is preferred in the treatment of the drug-induced form of parkinsonism:
a) Levodopa
b) Bromocriptine
c) Benztropine
d) Dopamine
009. Which of the following agents is the precursor of dopamine?
a) Bromocriptine
b) Levodopa
c) Selegiline
d) Amantadine
010. The main reason for giving levodopa, the precursor of dopamine, instead of dopamine is:
a) Dopamine does not cross the blood-brain barrier
b) Dopamine may induce acute psychotic reactions
c) Dopamine is intensively metabolized in humans
d) All of the above
011. Indicate a peripheral dopa decarboxylase inhibitor:
a) Tolcapone
b) Clozapine
c) Carbidopa
d) Selegiline
012. The mechanism of carbidopa′s action is:
a) Stimulating the synthesis, release, or reuptake of dopamine
b) Inhibition of dopa decarboxilase
c) Stimulating dopamine receptors
d) Selective inhibition of catecol-O-methyltransferase
013. Carbidopa is unable to penetrate the blood-brain barrier, it acts to reduce the peripheral conversion of levodopa to
dopamine.
a) True
b) False
014. When carbidopa and levodopa are given concomitantly:
a) Levodopa blood levels are increased, and drug half-life is lengthened
b) The dose of levodopa can be significantly reduced (by 75%), also reducing toxic side effects
c) A shorter latency period precedes the occurrence of beneficial effects
d) All of the above
015. Which of the following preparations combines carbidopa and levodopa in a fixed proportion?
a) Selegiline
b) Sinemet
c) Tolkapone
d) Biperiden
016. Which of the following statements is correct for levodopa?
a) Tolerance to both beneficial and adverse effects develops gradually
b) Levodopa is most effective in the first 2-5 years of treatment
c) After 5 years of therapy, patients have dose-related dyskinesias, inadequate response or toxicity
d) All of the above
017. Gastrointestinal irritation, cardiovascular effects, including tachycardia, arrhythmias, and orthostatic hypotension, mental
disturbances, and withdrawal are possible adverse effects of:
a) Amantadine
b) Benztropine
c) Levodopa
d) Selegiline
018. Which of the following agents is the most helpful in counteracting the behavioral complications of levodopa?
a) Tolkapone
b) Clozapine
c) Carbidopa
43
d) Pergolide
019. Which of the following vitamins reduces the beneficial effects of levodopa by enhancing its extracerebral metabolism?
a) Pyridoxine
b) Thiamine
c) Tocopherol
d) Riboflavin
020. Which of the following drugs antagonizes the effects of levodopa because it leads to a junctional blockade of dopamine
action?
a) Reserpine
b) Haloperidol
c) Chlorpromazine
d) All of the above
021. Levodopa should not be given to patients taking:
a) Bromocriptine
b) Monoamine oxydase A inhibitors
c) Carbidopa
d) Nonselective beta-adrenergic antagonists
022. Indicate D2 receptor agonist with antiparkinsonian activity:
a) Sinemet
b) Levodopa
c) Bromocriptine
d) Selegiline
023. Which of the following antiparkinsonian drugs has also been used to treat hyperprolactinemia?
a) Benztropine
b) Bromocriptine
c) Amantadine
d) Levodopa
024. Indicate a selective inhibitor of monoamine oxidase B:
a) Levodopa
b) Amantadine
c) Tolcapone
d) Selegiline
025. Which of the following statements is correct?
a) MAO-A metabolizes dopamine; MAO-B metabolizes serotonin
b) MAO-A metabolizes norepinephrine and dopamine; MAO-B metabolizes serotonin
c) MAO-A metabolizes norepinephrine and serotonin; MAO-B metabolizes dopamine
d) MAO-A metabolizes dopamine; MAO-B metabolizes norepinephrine and serotonin
026. Treatment with selegilin postpones the need for levodopa for 3-9 months and may retard the progression of Parkinson′s
disease.
a) True
b) False
027. The main reason for avoiding the combined administration of levodopa and an inhibitor of both forms of monoamine
oxidase is:
a) Respiratory depression
b) Hypertensive emergency
c) Acute psychotic reactions
d) Cardiovascular collapse and CNS depression
028. Indicate selective catechol-O-methyltransferase inhibitor, which prolongs the action of levodopa by diminishing its
peripheral metabolism:
a) Carbidopa
b) Clozapine
c) Tolcapone
d) Rasagiline
029. Which of the following antiparkinsonian drugs is an antiviral agent used in the prophylaxis of influenza A2?
a) Selegiline
b) Sinemet
c) Pergolide
d) Amantadine
030. The mechanism of amantadine action is:
a) Stimulating the glutamatergic neurotransmission
b) Blocking the excitatory cholinergic system
44
c) Inhibition of dopa decarboxilase
d) Selective inhibition of catechol-O-methyltransferase
031. Which of the following antiparkinsonism drugs is an anticholinergic agent?
a) Amantadine
b) Selegilin
c) Trihexyphenidyl
d) Bromocriptine
032. Mental confusion and hallucinations, peripheral atropine-like toxicity (e.g. Cycloplegia, tachycardia, urinary retention, and
constipation) are possible adverse effects of:
a) Sinemet
b) Benztropine
c) Tolkapone
d) Bromocriptine
033. Indicate the antiparkinsonism drug which should be avoided in patients with glaucoma:
a) Selegilin
b) Levodopa
c) Bromocriptine
d) Trihexyphenidyl

PART IV Ethyl alcohol


001. Alcohol may cause:
a) CNS depression
b) Vasodilatation
c) Hypoglycemia
d) All of the above
002. Alcohol:
a) Increases body temperature
b) Decreases body heat loss
c) Increases body heat loss
d) Does not affect body temperature
003. It is undesirable to take alcohol before going outdoors when it extremely cold, but it may be harmless to take some after
coming into a warm place from the cold.
a) True
b) False
004. The most common medical complication of alcohol abuse is:
a) Liver failure including liver cirrhosis
b) Tolerance and physical dependence
c) Generalized symmetric peripheral nerve injury, ataxia and dementia
d) All of the above
005. Effect of moderate consumption of alcohol on plasma lipoproteins is:
a) Raising serum levels of high-density lipoproteins
b) Increasing serum concentration of low-density lipoproteins
c) Decreasing the concentration of high-density lipoproteins
d) Raising serum levels of very low-density lipoproteins
006. Which of the following metabolic alterations may be associated with chronic alcohol abuse?
a) Hyperglycemia
b) Increased serum concentration of phosphate
c) Severe loss of potassium and magnesium
d) Decreased serum concentration of sodium
007. Alcohol potentiates:
a) SNS depressants
b) Vasodilatators
c) Hypoglycemic agents
d) All of the above
008. Which of the following drugs is most commonly used for causing a noxious reaction to alcohol by blocking its
metabolism?
a) Naltrexone
b) Disulfiram
c) Diazepam
d) Morphine
45
009. Which of the following agents is an inhibitor of aldehyde dehydrogenase?
a) Fomepizole
b) Ethanol
c) Disulfiram
d) Naltrexone
010. Indicate the drug, which alters brain responses to alcohol:
a) Naltrexone
b) Disulfiram
c) Amphetamine
d) Chlorpromazine
011. Which of the following agents is an opioid antagonist?
a) Amphetamine
b) Naltrexone
c) Morphine
d) Disulfiram
012. Alcohol causes an acute increase in the local concentrations of:
a) Dopamine
b) Opioid
c) Serotonine
d) All of the above
013. Management of alcohol withdrawal syndrome contains:
a) Restoration of potassium, magnesium and phosphate balance
b) Thiamine therapy
c) Substituting a long-acting sedative-hypnotic drug for alcohol
d) All of the above
014. Indicate the drug, which decreases the craving for alcohol or blunts pleasurable “high” that comes with renewed drinking:
a) Disulfiram
b) Amphetamine
c) Naltrexone
d) Diazepam
015. The symptoms resulting from the combination of disulfiram and alcohol are:
a) Hypertensive crisis leading to cerebral ischemia and edema
b) Nausea, vomiting
c) Respiratory depression and seizures
d) Acute psychotic reactions
016. The combination of disulfiram and ethanol leads to accumulation of:
a) Formaldehyde
b) Acetate
c) Formic acid
d) Acetaldehyde
017. The combination of naltrexone and disulfiram should be avoided since both drugs are potential hepatotoxins.
a) True
b) False
018. Indicate the “specific” modality of treatment for severe methanol poisoning:
a) Dialysis to enhance removal of methanol
b) Alkalinization to counteract metabolic acidosis
c) Suppression of metabolism by alcohol dehydrogenase to toxic products
d) All of the above
019. Which of the following agents may be used as an antidote for ethylene glycol and methanol poisoning?
a) Disulfiram
b) Fomepizol
c) Naltrexone
d) Amphetamine
020. The principal mechanism of fomepizol action is associated with inhibition of:
a) Aldehyde dehydrogenase
b) Acethylholinesterase
c) Alcohol dehydrogenase
d) Monoamine oxidase
46
PART V Narcotic analgesics
001. Narcotics analgesics should:
a) Relieve severe pain
b) Induce loss of sensation
c) Reduce anxiety and exert a calming effect
d) Induce a stupor or somnolent state
002. Second-order pain is:
a) Sharp, well-localized pain
b) Dull, burning pain
c) Associated with fine myelinated A-delta fibers
d) Effectively reduced by non-narcotic analgesics
003. Chemical mediators in the nociceptive pathway are all of the following EXCEPT:
a) Enkephalins
b) Kinins
c) Prostaglandins
d) Substance P
004. Indicate the chemical mediator in the antinociceptive descending pathways:
a) BETA-endorphin
b) Met- and leu-enkephalin
c) Dynorphin
d) All of the above
005. 5. Which of the following mediators is found mainly in long descending pathways from the midbrain to the dorsal horn?
a) Prostaglandin E
b) Dynorphin
c) Enkephalin
d) Glutamate
006. Select the brain and spinal cord regions, which are involved in the transmission of pain?
a) The limbic system, including the amygdaloidal nucleus and the hypothalamus
b) The ventral and medial parts of the thalamus
c) The substantia gelatinosa
d) All of the above
007. Mu (μ) receptors are associated with:
a) Analgesia, euphoria, respiratory depression, physical dependence
b) Spinal analgesia, mydriasis, sedation, physical dependence
c) Dysphoria, hallucinations, respiratory and vasomotor stimulation
d) Analgesia, euphoria, respiratory stimulation, physical dependence
008. Which of the following opioid receptor types is responsible for euphoria and respiratory depression?
a) Kappa-receptors
b) Delta-receptors
c) Mu-receptors
d) All of the above
009. Indicate the opioid receptor type, which is responsible for dysphoria and vasomotor stimulation:
a) Kappa-receptors
b) Delta-receptors
c) Mu-receptors
d) All of the above
010. Kappa and delta agonists:
a) Inhibit postsynaptic neurons by opening K+ channels
b) Close a voltage-gated Ca2+ channels on presynaptic nerve terminals
c) Both a and b
d) Inhibit of arachidonate cyclooxygenase in CNS
011. Which of the following supraspinal structures is implicated in pain-modulating descending pathways?
a) The midbrain periaqueductal gray
b) The hypothalamus
c) The aria postrema
d) The limbic cortex
012. Indicate the neurons, which are located in the locus ceruleus or the lateral tegmental area of the reticular formation:
a) Dopaminergic
b) Serotoninergic
c) Nonadrenergic
d) Gabaergic
47
013. Which of the following analgesics is a phenanthrene derivative?
a) Fentanyl
b) Morphine
c) Methadone
d) Pentazocine
014. Tick narcotic analgesic, which is a phenylpiperidine derivative:
a) Codeine
b) Dezocine
c) Fentanyl
d) Buprenorphine
015. Which of the following opioid analgesics is a strong mu receptor agonist?
a) Naloxone
b) Morphine
c) Pentazocine
d) Buprenorphine
016. Indicate the narcotic analgesic, which is a natural agonist:
a) Meperidine
b) Fentanyl
c) Morphine
d) Naloxone
017. Select the narcotic analgesic, which is an antagonist or partial mu receptor agonist:
a) Fentanyl
b) Pentazocine
c) Codeine
d) Methadone
018. Which of the following agents is a full antagonist of opioid receptors?
a) Meperidine
b) Buprenorphine
c) Naloxone
d) Butorphanol
019. The principal central nervous system effect of the opioid analgesics with affinity for a mu receptor is:
a) Analgesia
b) Respiratory depression
c) Euphoria
d) All of the above
020. Which of the following opioid analgesics can produce dysphoria, anxiety and hallucinations?
a) Morphine
b) Fentanyl
c) Pentazocine
d) Methadone
021. Indicate the opioid analgesic, which has 80 times analgesic potency and respiratory depressant properties of morphine,
and is more effective than morphine in maintaining hemodynamic stability?
a) Fentanyl
b) Pentazocine
c) Meperidine
d) Nalmefene
022. Which of the following opioid analgesics is used in combination with droperidol in neuroleptanalgesia?
a) Morphine
b) Buprenorphine
c) Fentanyl
d) Morphine
023. Fentanyl can produce significant respiratory depression by:
a) Inhibiting brain stem respiratory mechanisms
b) Suppression of the cough reflex leading to airway obstruction
c) Development of truncal rigidity
d) Both a and c
024. Most strong mu receptor agonists cause:
a) Hypertension
b) Increasing the pulmonary arterial pressure and myocardial work
c) Cerebral vasodilatation, causing an increase in intracranial pressure
d) All of the above
48
025. Which of the following opioid analgesics can produce an increase in the pulmonary arterial pressure and myocardial
work?
a) Morphine
b) Pentazocine
c) Meperidine
d) Methadone
026. Morphine causes the following effects EXCEPT:
a) Constipation
b) Dilatation of the biliary duct
c) Urinary retention
d) Bronchiolar constriction
027. Therapeutic doses of the opioid analgesics:
a) Decrease body temperature
b) Increase body temperature
c) Decrease body heat loss
d) Do not affect body temperature
028. Which of the following opioid analgesics is used in obstetric labor?
a) Fentanyl
b) Pentazocine
c) Meperidine
d) Buprenorphine
029. Indicate the opioid analgesic, which is used for relieving the acute, severe pain of renal colic:
a) Morphine
b) Naloxone
c) Methadone
d) Meperidine
030. Which of the following opioid analgesics is used in the treatment of acute pulmonary edema?
a) Morphine
b) Codeine
c) Fentanyl
d) Loperamide
031. The relief produced by intravenous morphine in dyspnea from pulmonary edema is associated with reduced:
a) Perception of shortness of breath
b) Patient anxiety
c) Cardiac preload (reduced venous tone) and afterload (decreased peripheral resistance)
d) All of the above
032. Rhinorrhea, lacrimation, chills, gooseflesh, hyperventilation, hyperthermia, mydriasis, muscular aches, vomiting,
diarrhea, anxiety, and hostility are effects of:
a) Tolerance
b) Opioid overdosage
c) Drug interactions between opioid analgesics and sedative-hypnotics
d) Abstinence syndrome
033. The diagnostic triad of opioid overdosage is:
a) Mydriasis, coma and hyperventilation
b) Coma, depressed respiration and miosis
c) Mydriasis, chills and abdominal cramps
d) Miosis, tremor and vomiting
034. Which of the following opioid agents is used in the treatment of acute opioid overdose?
a) Pentazocine
b) Methadone
c) Naloxone
d) Remifentanyl
035. Indicate the pure opioid antagonist, which has a half-life of 10 hours:
a) Naloxone
b) Naltrexone
c) Tramadol
d) Pentazocine
036. In contrast to morphine, methadone:
a) Causes tolerance and physical dependence more slowly
b) Is more effective orally
c) Withdrawal is less severe, although more prolonged
49
d) All of the above
037. Which of the following opioid analgesics is a partial mu receptor agonist?
a) Morphine
b) Methadone
c) Buprenorphine
d) Sufentanyl
038. Indicate a partial mu receptor agonist, which has 20-60 times analgesic potency of morphine, and a longer duration of
action:
a) Pentazocine
b) Buprenorphine
c) Nalbuphine
d) Naltrexone
039. Which of the following opioid analgesics is a strong kappa receptor agonist and a mu receptor antagonist?
a) Naltrexone
b) Methadone
c) Nalbuphine
d) Buprenorphine
040. Which of the following drugs has weak mu agonist effects and inhibitory action on norepinephrine and serotonin reuptake
in the CNS?
a) Loperamide
b) Tramadol
c) Fluoxetine
d) Butorphanol

PART VI Non-narcotic analgesics


001. Non-narcotic analgesics are mainly effective against pain associated with:
a) Inflammation or tissue damage
b) Trauma
c) Myocardial infarction
d) Surgery
002. Non-narcotic agents cause:
a) Respiratory depression
b) Antipyretic effect
c) Euphoria
d) Physical dependence
003. Non-narcotic analgesics are all of the following drugs EXCEPT:
a) Paracetamol
b) Acetylsalicylic acid
c) Butorphanol
d) Ketorolac
004. Select the non-narcotic drug, which is a paraaminophenol derivative:
a) Analgin
b) Aspirin
c) Baclophen
d) Paracetamol
005. Which of the following non-narcotic agents is salicylic acid derivative?
a) Phenylbutazone
b) Ketamine
c) Aspirin
d) Tramadol
006. Tick pirazolone derivative:
a) Methylsalicylate
b) Analgin
c) Paracetamol
d) Ketoralac
007. Which one of the following non-narcotic agents inhibits mainly cyclooxygenase (COX) in CNS?
a) Paracetamol
b) Ketorolac
c) Acetylsalicylic acid
d) Ibuprofen
50
008. Most of non-narcotic analgetics have:
a) Anti-inflammatory effect
b) Analgesic effect
c) Antipyretic effect
d) All of the above
009. Indicate the non-narcotic analgesic, which lacks an anti-inflammatory effect:
a) Naloxone
b) Paracetamol
c) Metamizole
d) Aspirin
010. Correct statements concerning aspirin include all of the following EXCEPT:
a) It inhibits mainly peripheral COX
b) It does not have an anti-inflammatory effect
c) It inhibits platelet aggregation
d) It stimulates respiration by a direct action on the respiratory center
011. For which of the following conditions could aspirin be used prophylactically?
a) Noncardiogenic pulmonary edema
b) Peptic ulcers
c) Thromboembolism
d) Metabolic acidosis
012. All of the following are undesirable effects of aspirin EXCEPT:
a) Gastritis with focal erosions
b) Tolerance and physical addiction
c) Bleeding due to a decrease of platelet aggregation
d) Reversible renal insufficiency
013. Characteristic findinds of salicylism include:
a) Headache, mental confusion and drowsiness
b) Tinnitus and difficulty in hearing
c) Hyperthermia, sweating, thirst, hyperventilation, vomiting and diarrhea
d) All of the above
014. Analgin usefulness is limited by:
a) Agranulocytosis
b) Erosions and gastric bleeding
c) Methemoglobinemia
d) Hearing impairment
015. Methemoglobinemia is possible adverse effect of:
a) Aspirin
b) Paracetamol
c) Analgin
d) Ketorolac
016. Correct the statements concerning ketorolac include all of the following EXCEPT:
a) It inhibits COX
b) It is as effective as morphine for a short-term relief from moderate to severe pain
c) It has a high potential for physical dependence and abuse
d) It does not produce respiratory depression
017. Indicate the nonopioid agent of central effect with analgesic activity:
a) Reserpine
b) Propranolol
c) Clopheline
d) Prazosin
018. Select the antiseizure drug with an analgesic component of effect:
a) Carbamazepine
b) Ethosuximide
c) Phenytoin
d) Clonazepam
019. Which of the following nonopioid agents is an antidepressant with analgesic activity?
a) Fluoxetine
b) Moclobemide
c) Tranylcypramine
d) Amitriptyline
020. Tick mixed (opioid/non-opioid) agent:
51
a) Paracetamol
b) Tramadol
c) Sodium valproate
d) Butorphanol

PART VII Antipsychotic agents


001. Neuroleptics are used to treat:
a) Neurosis
b) Psychosis
c) Narcolepsy
d) Parkinsonian disorders
002. Most antipsychotic drugs:
a) Strongly block postsynaptic d2receptor
b) Stimulate postsynaptic D2 receptor
c) Block NMDA receptor
d) Stimulate 5-HT2 receptor
003. Which of the following dopaminergic systems is most closely related to behavior?
a) The hypothalamic-pituitary system
b) The extrapyramidal system
c) The mesolimbic and mesofrontal systems
d) The chemoreceptor trigger zone of the medulla
004. Hyperprolactinemia is caused by blockade of dopamine in:
a) The chemoreceptor trigger zone of the medulla
b) The pituitary
c) The extrapiramidal system
d) The mesolimbic and mesofrontal systems
005. Parkinsonian symptoms and tarditive dyskinesia are caused by blockade dopamine in:
a) The nigrostriatal system
b) The mesolimbic and mesofrontal systems
c) The chemoreceptor trigger zone of the medulla
d) The tuberoinfundibular system
006. Extrapyramidal reactions can be treated by:
a) Levodopa
b) Benztropine mesylate
c) Bromocriptine
d) Dopamine
007. Which of the following statements is true?
a) D1 postsynaptic receptors are located in striatum
b) D2 pre- and postsynaptic receptors are located in striatum and limbic areas
c) D4 postsynaptic receptors are located in frontal cortex, mesolimbic system
d) All of the above
008. Which of the following antipsychotic drugs is typical?
a) Clozapine
b) Quetiapine
c) Haloperidol
d) Olanzapine
009. Indicate the atypical antipsychotic drug:
a) Haloperidol
b) Clozapine
c) Thioridazine
d) Thiothixene
010. Atypical antipsychotic agents (such as clozapine) differ from typical ones:
a) In reduced risks of extrapyramidal system dysfunction and tardive dyscinesia
b) In having low affinity for D1 and D2 dopamine receptors
c) In having high affinity for D4 dopamine receptors
d) All of the above
011. Tardive dyskinesia is the result of:
a) Degeneration of dopaminergic and cholinergic fibers
b) Hyperactive dopaminergic state in the presence of dopamine blockers
c) Degeneration of histaminergic fibers
d) Supersensitivity of cholinergic receptors in the caudate-putamen
52
012. Which of the following antipsychotic drugs has high affinity for D4 and 5-HT2 receptors?
a) Clozapine
b) Fluphenazine
c) Thioridazine
d) Haloperidole
013. Indicate the antipsychotic drug, which is a phenothiazine aliphatic derivative:
a) Thiothixene
b) Risperidone
c) Chlorpromazine
d) Clozapine
014. Indicate the antipsychotic drug, which is a butyrophenone derivative:
a) Droperidol
b) Thioridazine
c) Sertindole
d) Fluphenazine
015. Indicate the antipsychotic drug, which is a thioxanthene derivative:
a) Haloperidol
b) Clozapine
c) Chlorpromazine
d) Thiothixene
016. Indicate the antipsychotic agent – a dibenzodiazepine derivative:
a) Fluphenazine
b) Clozapine
c) Risperidone
d) Droperidol
017. The strong antiemetic effect of the phenothiazine derivatives is due to dopamine receptor blockade:
a) In the chemoreceptor trigger zone of the medulla
b) Of the receptors in the stomach
c) The medullar vomiting centre
d) All of the above
018. Phenothiazine derivatives are able to:
a) Alter temperature-regulating mechanisms producing hypothermia
b) Decrease levels of prolactin
c) Increase corticotrophin release and secretion of pituitary growth hormone
d) Decrease appetite and weight
019. Most phenothiazine derivatives have:
a) Antihistaminic activity
b) Anticholinergic activity
c) Antidopaminergic activity
d) All of the above
020. Indicate the antipsychotic drug having significant peripheral alpha-adrenergic blocking activity:
a) Haloperidol
b) Chlorpromazine
c) Clozapine
d) Risperidone
021. Indicate the antipsychotic drug having a muscarinic-cholinergic blocking activity:
a) Chlorpromazine
b) Clorprothixene
c) Risperidone
d) Haloperidol
022. Indicate the antipsychotic drug having H1-antihistaminic activity:
a) Clozapine
b) Chlorpromazine
c) Olanzapine
d) All of the above
023. Parkinson′s syndrome, acute dystonic reactions, tardive dyskinesia, antimuscarinic actions, orthostatic hypotension,
galactorrhea are possible adverse effects of:
a) Haloperidol
b) Clozapine
c) Chlorpromazine
d) Risperidone
53
024. Orthostatic hypotension can occur as a result of:
a) The central action of phenothiazines
b) Inhibition of norepinephrine uptake mechanisms
c) Alpha adrenoreceptor blockade
d) All of the above
025. Adverse peripheral effects, such as loss of accommodation, dry mouth, tachycardia, urinary retention, constipation are
related to:
a) Alpha adrenoreceptor blockade
b) Muscarinic cholinoreceptor blockade
c) Supersesitivity of the dopamine receptor
d) Dopamine receptor blockade
026. Which of the following phenothiazine derivatives is a potent local anesthetic?
a) Fluphenazine
b) Thioridazine
c) Chlorpromazine
d) None of the above
027. Which of the following phenothiazine derivatives may produce cardiac toxicity, including ventricular arrhythmias, cardiac
conduction block, and sudden death?
a) Thioridazine
b) Chlorpromazine
c) Perphenazine
d) Fluphenazine
028. Which of the following antipsychotic agents is preferable in patients with coronary and cerebrovascular disease?
a) Chlorpromazine
b) Fluphenazine
c) Haloperidol
d) Perphenazine
029. Which of the following antipsychotic agents is used in combination with an opioid drug fentanyl in neuroleptanalgesia?
a) Haloperidol
b) Droperidol
c) Chlorpromazine
d) Clozapine
030. The mechanism of haloperidol antipsychotic action is:
a) Blocking D2 receptors
b) Central alpha-adrenergic blocking
c) Inhibition of norepinephrine uptake mechanisms
d) All of the above
031. Which of the following statements is correct for clozapine?
a) Has potent anticholinergic activity
b) Has high affinity for D1 and D2 dopamine receptors
c) Produces significant extrapyramidal toxicity
d) Is related to typical antipsychotic agents
032. Which of the following antipsychotic drugs has the high risk of potentially fatal agranulocytosis and risk of seizures at
high doses?
a) Haloperidol
b) Risperidone
c) Clozapine
d) Chlorpromazine
033. Which of the following antipsychotic drugs has high affinity for D2 and 5-HT2 receptors?
a) Droperidol
b) Clozapine
c) Thiothixene
d) Risperidone
034. Lithium carbonate is useful in the treatment of:
a) Petit mal seizures
b) Bipolar disorder
c) Neurosis
d) Trigeminal neuralgia
035. The drug of choice for manic-depressive psychosis is:
a) Imipramine
b) Chlordiazepoxide
54
c) Isocarboxazid
d) Lithium carbonate
036. The lithium mode of action is:
a) Effect on electrolytes and ion transport
b) Effect on neurotransmitters
c) Effect on second messengers
d) All of the above
037. Which of the following statements is correct for lithium?
a) Stimulate dopamine and beta-adrenergic receptors
b) Decrease catecholamine-related activity
c) Stimulate the development of dopamine receptor supersensitivity
d) Decrease cholinergic activity
038. Which of the following adverse effects is associated with lithium treatment?
a) Cardiovascular anomalies in the newborn
b) Thyroid enlargement
c) Nephrogenic diabetes insipidus
d) All of the above

PART VIII Antidepressant agents


001. The principal mechanism of action of antidepressant agents is:
a) Stabilization of dopamine and beta-adrenergic receptors
b) Inhibition of the storage of serotonin and epinephrine in the vesicles of presynaptic nerve endings
c) Blocking epinephrine or serotonin reuptake pumps
d) Stimulation of alfa2-norepinephrine receptors
002. Which of the following agents is related to tricyclic antidepressants?
a) Nefazodon
b) Amitriptyline
c) Fluoxetine
d) Isocarboxazid
003. Indicate the second-generation heterocyclic drug:
a) Maprotiline
b) Imipramine
c) Phenelzine
d) Fluoxetine
004. Which of the following agents is related to the third-generation heterocyclic antidepressants?
a) Amitriptyline
b) Maprotiline
c) Nefazodone
d) Tranylcypromine
005. Which of the following antidepressants is a selective serotonin reuptake inhibitor?
a) Phenelzine
b) Desipramine
c) Maprotiline
d) Fluoxetine
006. Which of the following antidepressant agents is a selective inhibitor of norepinephrine reuptake?
a) Fluvoxamine
b) Maprotiline
c) Amitriptyline
d) Tranylcypromine
007. Indicate the antidepressant, which blocks the reuptake pumps for serotonin and norepinephrine:
a) Amitriptyline
b) Fluoxetine
c) Maprotiline
d) Phenelzine
008. Which of the following antidepressants is an unselective MAO blocker and produces extremely long-lasting inhibition of
the enzyme?
a) Moclobemide
b) Tranylcypramine
c) Selegiline
d) Fluoxetine
55
009. Indicate the irreversible MAO inhibitor, which is a hydrazide derivative:
a) Moclobemide
b) Selegiline
c) Tranylcypramine
d) Phenelzine
010. Which of the following MAO inhibitors has amphetamine-like activity and is related to nonhydrazide derivatives:
a) Phenelzine
b) Moclobemide
c) Tranylcypramine
d) All of the above
011. Which of the following antidepressants is a selective short-acting MAO-A inhibitor?
a) Maprotiline
b) Amitriptyline
c) Moclobemide
d) Selegiline
012. Monoamine Oxydase A:
a) Is responsible for norepinephrine, serotonin, and tyramine metabolism
b) Is more selective for dopamine
c) Metabolizes norepinephrine and dopamine
d) Deaminates dopamine and serotonin
013. Which synapses are involved in depression?
a) Dopaminergic synapses
b) Serotoninergic synapses
c) Cholinergic synapses
d) All of the above
014. Block of which type of Monoamine Oxydase might be more selective for depression?
a) MAO-A
b) MAO-B
c) Both MAO-A and MAO-B
d) MAO-C
015. The principal mechanism of MAO inhibitor action is:
a) Blocking the amine reuptake pumps, which permits to increase the concentration of the neurotransmitter at the
receptor site
b) Blocking a major degradative pathway for the amine neurotransmitters, which permits more amines to
accumulate in presynaptic stores
c) Inhibition the storage of amine neurotransmitters in the vesicles of presynaptic nerve endings
d) Antagonism of alfa2-norepinephrine receptors
016. The irreversible MAO inhibitors have a very high risk of developing:
a) Respiratory depression
b) Cardiovascular collapse and CNS depression
c) Hypertensive reactions to tyramine ingested in food
d) Potentially fatal agranulocytosis
017. The most dangerous pharmacodynamic interaction is between MAO inhibitors and:
a) Selective serotonin reuptake inhibitors
b) Tricyclics
c) Sympathomimetics
d) All of the above
018. Serotonin syndrome is a result of:
a) Increased stores of monoamine
b) Significant accumulation of amine neurotransmitters in the synapses
c) Both a and b
d) Depleted stores of biogenic amines
019. The therapeutic response to antidepressant drugs is usually over a period of:
a) 2-3 days
b) 2-3 weeks
c) 24 hours
d) 2-3 month
020. Which of the following antidepressants may have latency period as short as 48 hours?
a) Tranylcypromine
b) Imipramine
c) Fluoxetine
56
d) Amitrityline
021. Which of the following features do MAO inhibitors and tricyclic antidepressants have in common?
a) Act postsynaptically to produce their effect
b) Can precipitate hypotensive crises if certain foods are ingested
c) Increase levels of biogenic amines
d) Are useful for the manic phase of bipolar disorder
022. Tricyclic antidepressants are:
a) Highly selective serotonin reuptake inhibitors
b) Monoamine oxidase inhibitors
c) Selective norepinephrine reuptake inhibitors
d) Mixed norepinephrine and serotonin reuptake inhibitors
023. Which of the following autonomic nervous system effects is common for tricyclic antidepressants?
a) Antimuscarinic action
b) Antihistaminic action
c) Alfa adrenoreceptor-blocking action
d) All of the above
024. Indicate an effective antidepressant with minimal autonomic toxicity:
a) Amitrityline
b) Fluoxetine
c) Imipramine
d) Doxepin
025. Fluoxetine has fewer adverse effects because of:
a) Mixed norepinephrine and serotonin reuptake inhibition
b) Depleted stores of amine neurotransmitters
c) Minimal binding to cholinergic, histaminic, and alfa-adrenergic receptors
d) All of the above
026. Which of the following tricyclic and heterocyclic antidepressants has the greatest sedation?
a) Doxepin
b) Amitriptyline
c) Trazodone
d) All of the above
027. Which of the following tricyclic and heterocyclic agents has the least sedation?
a) Protriptyline
b) Trazodone
c) Amitriptyline
d) Mitrazapine
028. Indicate a tricyclic or a heterocyclic antidepressant having greatest antimuscarinic effects:
a) Desipramine
b) Amitriptyline
c) Trazodone
d) Mirtazapine
029. Indicate a tricyclic or a heterocyclic antidepressant having least antimuscarinic effects:
a) Trazodone
b) Buprorion
c) Mirtazapine
d) All of the above
030. Which of the following antidepressants has significant alfa2-adrenoreceptor antagonism?
a) Amitriptyline
b) Nefazodone
c) Mirtazapine
d) Doxepin
031. Indicate the main claim for an ideal antidepressant agent:
a) A faster onset of action
b) Fewer adverse sedative and autonomic effects
c) Fewer toxicity when overdoses are taken
d) All of the above
032. Sedation, peripheral atropine-like toxicity (e.g. Cycloplegia, tachycardia, urinary retention, and constipation), orthostatic
hypotension, arrhythmias, weight gain and sexual disturbances are possible adverse effects of:
a) Sertaline
b) Amitriptyline
c) Phenelsine
57
d) Bupropion
033. Which of the following drugs is least likely to be prescribed to patients with prostatic hypertrophy, glaucoma, coronary
and cerebrovascular disease?
a) Amitriptyline
b) Paroxetine
c) Bupropion
d) Fluoxetine
034. Indicate the antidepressant agent, which is a phenyltolylpropylamine derivative:
a) Paroxetine
b) Maprotiline
c) Fluoxetine
d) Amitriptyline
035. The mechanism of fluoxetine action includes:
a) Selective inhibition of serotonine uptake in the CNS
b) Little effect on central norepinephrine or dopamine function
c) Minimal binding to cholinergic, histaminic, and alfa-adrenergic receptors
d) All of the above
036. Which of the following antidepressants is used for treatment of eating disorders, especially buliemia?
a) Amitriptyline
b) Fluoxetine
c) Imipramine
d) Tranylcypromine
037. Sertaline and paroxetine are similar to fluoxetine in the mechanism of action and therapeutic use, sertaline is less likely
to interact adversely with other drugs.
a) True
b) False
038. A highly selective serotonine reuptake inhibitor is:
a) Sertaline
b) Paroxetine
c) Fluoxetine
d) All of the above

PART IX Anxiolytic agents


001. Anxiolytics are used to treat:
a) Neurosis
b) Psychosis
c) Narcolepsy
d) Bipolar disorders
002. Anxiolytic agents should:
a) Relieve pain
b) Reduce anxiety and exert a calming effect
c) Improve mood and behavior in patient with psychotic symptoms
d) Produce drowsiness, encourage the onset and maintenance of a state of sleep
003. Anxiolytics are also useful for:
a) Treatment of epilepsy and seizures
b) Insomnia
c) Muscle relaxation in specific neuromuscular disorders
d) All of the above
004. Indicate the agents of choice in the treatment of most anxiety states:
a) Barbiturates
b) Benzodiazepines
c) Lithium salts
d) Phenothiazines
005. The choice of benzodiazepines for anxiety is based on:
a) A relatively high therapeutic index
b) Availability of flumazenil for treatment of overdose
c) A low risk of physiologic dependence
d) All of the above
006. Which of the following anxiolitics is a benzodiazepine derivative:
a) Buspirone
58
b) Clordiazepoxide
c) Meprobamate
d) Chloral hydrate
007. Indicate the benzodiazepine, which has the shortest elimination half-life:
a) Quazepam
b) Triazolam
c) Diazepam
d) Clorazepate
008. Which of the following benzodiazepines has the shortest duration of action?
a) Triazolam
b) Clorazepate
c) Prazepam
d) Clordiazepoxide
009. Which of the following benzodiazepines is less likely to cause cumulative and residual effects with multiple doses?
a) Clorazepate
b) Quazepam
c) Lorazepam
d) Prazepam
010. Anxiolytic dosage reduction is recommended:
a) In patients taking cimetidine
b) In patients with hepatic dysfunction
c) In elderly patients
d) All of the above
011. Which of the following benzodiazepines is preferred for elderly patients?
a) Clorazepate
b) Clordiazepoxide
c) Triazolam
d) Prazepam
012. Which of the following anxiolytics is preferred in patient with limited hepatic function?
a) Buspirone
b) Quazepam
c) Diazepam
d) Chlordiazepoxide
013. Indicate the mechanism of hypnotic benzodiazepine action:
a) Increasing the duration of the GABA-gated Cl- channel openings
b) Directly activating the chloride channels
c) Increasing the frequency of Cl- channel opening events
d) All of the above
014. Which of the following anxiolytics is a partial agonist of brain 5-HT1A receptors?
a) Buspirone
b) Alprozolam
c) Chlorazepat
d) Lorazepam
015. Indicate the competitive antagonist of BZ receptors:
a) Flumazenil
b) Buspirone
c) Picrotoxin
d) Diazepam
016. Indicate the agent, which interferes with GABA binding:
a) Chlordiazepoxide
b) Bicuculline
c) Thiopental
d) Picrotoxin
017. Antianxiety agents have:
a) Sedative and hypnotic activity
b) Muscle relaxing and anticonvulsant effects
c) Amnesic properties
d) All of the above
018. Which of the following disadvantages does not limit using benzodiazepines as antianxiety agents?
a) Tendency to develop psychologic dependence
b) A high risk of drug interactions based on liver enzyme induction
59
c) Synergic CNS depression with concomitant use of other drugs
d) The formation of active metabolites
019. Indicate the anxiolitic agent, which relieves anxiety without causing marked sedative effects:
a) Diazepam
b) Chlordiazepoxid
c) Buspirone
d) Clorazepate
020. Which of the following anxiolytics has minimal abuse liability?
a) Oxazepam
b) Buspirone
c) Flumazenil
d) Alprazolam
021. In contrast to benzodiazepines, buspirone:
a) Interact directly with gabaergic system
b) Has more marked hypnotic, anticonvulsant, or muscle relaxant properties
c) Causes less psychomotor impairment and does not affect driving skills
d) Has maximal abuse liability
022. Which of the following sedative-hypnotic drugs does not potentiate the CNS depressant effects of ethanol,
phenothiazines, or tricyclic antidepressants?
a) Buspirone
b) Phenobarbital
c) Diazepam
d) Chloralhydrate
023. Limitation of buspirone is:
a) A low therapeutic index
b) An extremely slow onset of action
c) A high potential of development of physical dependence
d) Impairment of mentation or motor functions during working hours
024. Which drugs may be used as antianxiety agents?
a) BETA-blocking drugs
b) Clonidine - a partial agonist of alfa2 receptors
c) Tricyclic antidepressants
d) All of the above
025. Which of the following benzodiazepines is more likely to cause “hangover” effects such as drowsiness, dysphoria, and
mental or motor depression the following day?
a) Oxazepam
b) Triazolam
c) Clorazepat
d) Lorazepam
026. Additive CNS depression can be predicted if benzodiazepines are used with:
a) Ethanol
b) Morphine
c) Clorpromazine
d) All of the above
027. Which dosage of benzodiazepines for 60-90 days may produce severe withdrawal symptoms?
a) 50-60 mg/d
b) Less than 400 mg/d
c) More than 800 mg/d
d) Less than 40 mg/d
028. Restlessness, anxiety, orthostatic hypotension, generalized seizures, severe tremor, vivid hallucination, and psychosis
are possible symptoms of:
a) Tolerance
b) Withdrawal
c) Drug interactions between barbiturate and diazepam
d) None of the above
029. Flumazenil is used to:
a) Reverse the CNS depressant effects of hypnotic benzodiazepines overdose
b) Hasten recovery following use of hypnotic benzodiazepines in anesthetic and diagnostic procedure
c) Reverse benzodiazepine-induced respiratory depression
d) All of the above
030. Flumazenil given intravenously:
60
a) Has intermediate onset and duration of action about 2 hours
b) Acts rapidly but has a short half-life
c) Has an effect lasting 3-5 hours
d) Has duration of action longer than 6 hours

PART X CNS stimulants


001. Agents, stimulating CNS are all of the following except:
a) Fluoxetine
b) Clozapine
c) Nootropil
d) Sydnocarb
002. Which of the following CNS stimulants are the agents of selective effect?
a) Analeptics
b) General tonics
c) Psychostimulants
d) Actoprotectors
003. Indicate CNC stimulating drugs, which are the agents of general action:
a) Nootropic agents
b) Analeptics
c) Psychostimulants
d) Antidepressants
004. Which of the following agents belongs to psychostimulants?
a) Meridil
b) Camphor
c) Piracetam
d) Pantocrin
005. Indicate the nootropic agent:
a) Sydnocarb
b) Eleuterococci extract
c) Fluoxetine
d) Piracetam
006. Which of the following agents is a respiratory analeptic?
a) Piracetam
b) Sydnocarb
c) Bemegride
d) Pantocrin
007. Indicate the CNC stimulating drug, which belongs to adaptogens:
a) Amphetamine
b) Eleuterococci extract
c) Caffeine
d) Sydnocarb
008. Actoprotectors are:
a) Stimulators, improving physical efficiency
b) Cognition enhancers, improving the highest integrative brain function
c) Stimulants, raising non-specific resistance towards stresses
d) Agents, stimulating the bulbar respiratory and vasomotor centers
009. Adaptogens cause:
a) Improvment of efficiency using physical loads and acceleration of recovery after the load
b) Stimulation of respiratory and vasomotor centers
c) Temporary relief of the feeling of tiredness, facilitating the professional work and fighting somnolence
d) Increased resistance towards stress situations and adaptation to extreme conditions
010. Indicate the CNS stimulants, which mitigate conditions of weakness or lack of tone within the entire organism or in
particular organs?
a) Psychostimulants
b) Analeptics
c) General tonics
d) Antidepressants
011. Which of the following agents is a general tone-increasing drug of plant origin?
a) Meridil
b) Eleuterococci′s extract
c) Pantocrin
61
d) Caffeine
012. Indicate a general tone-increasing drug, which is an agent of animal origin?
a) Pantocrin
b) Amphetamine
c) Sydnocarb
d) Camphor
013. Amphetamine:
a) Is a powerful stimulant of the CNS
b) Stimulates the medullar respiratory center and has an analeptic action
c) Increases motor and speech activity, mood, decreases a sense of fatigue
d) All of the above
014. The mechanism of amphetamine action is related to:
a) Direct catecholamiergic agonist action
b) Inhibition of monoamine oxydase
c) Increasing a release of catecholaminergic neurotransmitters
d) All of the above
015. Indicate the CNS stimulant, which is a piperidine derivative:
a) Meridil
b) Amphetamine
c) Caffeine
d) Sydnophen
016. Which of the following CNS psychostimulants is a sydnonymine derivative?
a) Caffeine
b) Sydnocarb
c) Meridil (methylphenidate hydrochloride)
d) Amphetamine
017. Sydnocarb causes:
a) Decreased sense of fatigue, it facilitates the professional work and fights somnolence
b) The feeling of prosperity, relaxation and euphoria
c) Influx of physical and mental forces, locomotive and speech excitation
d) Peripheral sympathomimetic action
018. Indicate the psychostimulant, which is a methylxantine derivative:
a) Caffeine
b) Sydnocarb
c) Amphetamine
d) Meridil
019. Which of the following psychostimulants acts centrally mainly by blocking adenosine receptors?
a) Meridil
b) Caffeine
c) Amphetamine
d) Sydnophen
020. Principal properties of caffeine include all of the following EXEPT:
a) Cardiac analeptic (increase the rate and the force of the cardiac contraction)
b) Adaptogenic (rise non-specific resistance towards stresses and adapt to extraordinary challenges)
c) Psychoanaleptic (decrease the feeling of tiredness, facilitates the professional work and fights somnolence)
d) Respiratory analeptic (stimulate the bulbar respiratory center)
021. Caffeine can produce all of the following effects except:
a) Coronary vasodialation
b) Relaxation of bronchial and biliary tract smooth muscles
c) Vasodialation of cerebral vessels
d) Reinforcement of the contractions and increase of the striaated muscle work
022. Caffeine does not cause:
a) Inhibition of gastric secretion
b) Hyperglycemia
c) Moderate diuretic action
d) Increase in free fatty acids
023. Therapeutic uses of caffeine include all of the following EXCEPT:
a) Cardiovascular collapse and respiratory insufficiency
b) Migraine
c) Somnolence
d) Gastric ulceration
62
024. Adverse effects of caffeine include all of the following EXCEPT:
a) Arrhythmias
b) Insomnia
c) Hypotension
d) Psychomotor excitation
025. Principal properties of cordiamine include all of the following EXCEPT:
a) Cardiac analeptic
b) Respiratory analeptic
c) Coronarodilatator
d) Significant abuse potential
026. Сharacteristics of cordiamine include all of the following EXCEPT:
a) It stimulates the CNS and facilitates the movement coordination
b) It is a respiratory analeptic of mixed action (stimulates both the medullar respiratory center and chemoreceptor of
carotid sinus zone)
c) It decreases the aortic and coronary flow
d) It counteracts the central depression produced by other drugs (barbiturates)
027. Cordiamine is useful in the treatment of:
a) Hypotension
b) Coronary insufficiency
c) Respiratory insufficiency
d) All of the above
028. Respiratory and cardiac analeptics are all of the following agents EXCEPT:
a) Cordiamine
b) Bemegride
c) Caffeine
d) Camphor
029. Bemegride:
a) Stimulates the medullar respiratory center (central effect)
b) Stimulates hemoreceptors of carotid sinus zone (reflector action)
c) Is a mixed agent (both central and reflector effects)
d) Is a spinal analeptic
030. Which of the following CNS stimulants belongs to nootropics?
a) Camphor
b) Pantocrin
c) Sydnocarb
d) Piracetam
031. Characteristics of nootropics include all of the following EXCEPT:
a) Selective influence on the brain
b) Improvement the ability to communicate with peers
c) Decline in the highest integrative brain functions
d) Increase in energetic exchange of the brain cells
032. Which of the following statements concerning nootropics is not correct?
a) They improve the highest integrative brain functions (memory, learning, understanding, thinking and the capacity for
concentration)
b) They stimulate the bulbar respiratory center
c) They stimulate existing neuronal synapses to optimum performance (adaptive capacity)
d) They stimulate existing neuronal synapses to damaging influences, such as disturbances of the energy and
neurotransmitter metabolism or ischemia (protective capacity)
033. Features of piracetam include all of the following EXCEPT:
a) It is a GABA derivative
b) It does not influence the neuro-vegetative function
c) Improvement begins in the 3′rd week
d) It has a high potential of toxicity
034. Piracetam can produce all of the following effects EXCEPT:
a) Antipsychotic
b) Anticonvulsant
c) Psychometabolic
d) Antihypoxic
035. Piracetam is widely used for the treatment of:
a) Senile dementia
b) Asthenia
63
c) Chronic alcoholism
d) All of the above
036. Indicate the CNS stimulant, which is used in pediatric medicine, as it improves the communication with the child,
increases the ability to study and communication with peers, improves school-performance?
a) Meridil
b) Piracetam
c) Bemegride
d) Amphetamine
037. Which of the following CNS stimulants is used for the cerebral stroke treatment?
a) Pantocrin
b) Sydnocarb
c) Piracetam
d) Caffeine

PART XI Drugs of abuse


001. Psychologic dependence is:
a) Decreased responsiveness to a drug following repeated exposure
b) A combination of certain drug-specific symptoms that occur on sudden discontinuation of a drug
c) Compulsive drug-seeking behavior
d) All of the above
002. Tolerance is associated with:
a) An ability to compensate for the drug effect
b) Increased disposition of the drug after chronic use
c) Compensatory changes in receptors, effector enzymes, or membrane actions of the drug
d) All of the above
003. Addiction is associated with the existence of:
a) Psychological dependence
b) Physiological dependence
c) Tolerance
d) All of the above
004. Substances causing narco- and glue sniffings are all of the following EXCEPT :
a) Stimulants
b) Antipsychotic drugs
c) Psychedelics
d) Sedative drugs
005. Which of the following abused drugs do not belong to sedative agents?
a) Barbiturates
b) Tranquilizers
c) Cannabinoids
d) Opioids
006. Psychedelics are all of following agents EXCEPT:
a) Cocaine
b) LSD
c) Marijuana
d) Volatile substances (glues, solvents, volatile nitrites and nitrous oxide)
007. In contrast to morphine, heroin is:
a) Used clinically
b) More addictive and fast-acting
c) More effective orally
d) Less potent and long-acting
008. Symptoms of opioid withdrawal begin 8-10 hours after the last dose.
a) True
b) False
009. The acute course of opioid withdrawal may last:
a) 3-4 days
b) 7-10 days
c) 3-4 weeks
d) 26-30 weeks
010. Indicate the sedative-hypnotic agent, which has the highest abuse potential:
a) Buspirone
64
b) Diazepam
c) Phenobarbital
d) Zolpidem
011. Characteristics of barbiturate intoxication (2-3 dose) include all of the following EXCEPT:
a) Pleasant feelings of the “blow” in the head, vertigo, myasthenia, stupor
b) Perceptual distortion of surroundings, disorders of thinking, behavior
c) Locomotive, speech excitation, sharp swings from a cheerful mood to an aggressive state
d) Sleep with the subsequent weakness and headaches
012. Barbiturate abstinent syndrome is shown by:
a) Crisis by 3 day of abstention
b) Anxiety, mydriasis, myasthenia, muscular convulsions, vomiting, diarrhea
c) Psychosis as delirium (color visual and auditory hallucinations)
d) All of the above
013. Which one of the following tranquilizers belongs to strong euphorizing agents?
a) Mebicarum
b) Buspirone
c) Diazepam
d) Chlordiazepoxide
014. Tranquilizers intoxication (5-10 tablets) features include:
a) Euphoria, burst of energy, increase in motor activity, wave warmth all over the body
b) Visual hallucinations, a distorted feelling of time and space
c) Physical bliss, body lightness, a wish to fly, motionlessness
d) Synaesthesia (the sounds can be tensed, the colors can be heard)
015. Which of the following abused drugs is related to stimulants?
a) Cocaine
b) Amphetamine
c) Caffeine
d) All of the above
016. Cocaine exerts its central action by:
a) Inhibiting phosphodiesterase
b) Increasing a release of catecholaminergic neurotransmitters, including dopamine
c) Inhibiting dopamine and norepinephrine reuptake
d) Altering serotonin turnover
017. “Crack” is a derivative of:
a) Opium
b) LSD
c) Cocaine
d) Cannabis
018. Cocaine intoxication appears by:
a) Short clouding of consciousness, lightness of body and a feeling of flight
b) Wave warmth all over the body, physical bliss, motionlessness
c) Clear consciousness, improved mood, influx of physical and spiritual forces, locomotive and speech excitation,
reappraisal of personality
d) All of the above
019. Which of the following stimulants is related to psychedelics?
a) “ecstasy” (methylenedioxymethamphetamine)
b) Cocaine
c) “crack” (cocaine free base)
d) Caffeine
020. Cocaine may cause:
a) Powerful vasoconstrictive reactions resulting in myocardial infarctions
b) The multiple brain perfusion defects
c) Spontaneous abortion during pregnancy
d) All of the above
021. Characteristics of cocaine abstinent syndrome include all of the following phases EXCEPT:
a) Feeling of depression, irritability, confusion, insomnia (the first 3 days)
b) Depression, apathy, excessive appetite, a wish to sleep (the subsequent 1-2 days)
c) Psychosis as color visual and auditory hallucinations (for 3 day)
d) New attack of depression, anxiety, irritability, dullness, intense thirst for cocaine (after 1-5 days improvement)
022. Overdoses of cocaine are usually rapidly fatal from:
a) Respiratory depression
65
b) Arrhythmias
c) Seizures
d) All of the above
023. Which of the following agents is related to hallucinogens?
a) Heroin
b) LSD
c) Cocaine
d) Opium
024. LSD produces:
a) Mood swings
b) Impaired memory, difficulty in thinking, poor judgment
c) Perceptual distortion
d) All of the above
025. LSD decreases in brain:
a) 5-HT2 receptor densities
b) GABAA-benzodiazepine receptor densities
c) Adrenergic receptor densities
d) D2 receptor densities
026. Which of the following agents is related to cannabis?
a) Heroin
b) Ecstasy
c) Hashish
d) Crack
027. The early stage of cannabis intoxication is characterized by:
a) Euphoria, uncontrolled laugher
b) Alteration of time sense, depersonalization
c) Sharpened vision
d) All of the above
028. Which of the following physiologic signs is a characteristic of cannabis intoxication?
a) Bradycardia
b) Reddening of the conjunctiva
c) Miosis
d) Nausea and vomiting
029. Industrial solvent inhalation causes:
a) Quick intoxication, lasting only 5-15 minutes
b) Euphoria, relaxed “drunk” feeling
c) Disorientation, slow passage of time and possible hallucinations
d) All of the above
030. Indicate the drugs of choice for reversing the withdrawal syndrome:
a) Benzodiazepines
b) Neuroleptics
c) Antidepressants
d) All of the above

PART XII General anesthetics


001. The state of “general anesthesia” usually includes:
a) Analgesia
b) Loss of consciousness, inhibition of sensory and autonomic reflexes
c) Amnesia
d) All of the above
002. Inhaled anesthetics and intravenous agents having general anesthetic properties:
a) Directly activate GABAA receptors
b) Facilitate GABA action but have no direct action on GABAA receptors
c) Reduce the excitatory glutamatergic neurotransmission
d) Increase the duration of opening of nicotine-activated potassium channels
003. Indicate the anesthetic, which is an inhibitor of NMDA glutamate receptors:
a) Thiopental
b) Halothane
c) Ketamine
d) Sevoflurane
66
004. An ideal anesthetic drug would:
a) Induces anesthesia smoothly and rapidly and secure rapid recovery
b) Posses a wide margin of safety
c) Be devoid of adverse effects
d) All of the above
005. Which of the following general anesthetics belongs to inhalants?
a) Thiopental
b) Desfluran
c) Ketamine
d) Propofol
006. Indicate the anesthetic, which is used intravenously:
a) Propofol
b) Halothane
c) Desflurane
d) Nitrous oxide
007. Which of the following inhalants is a gas anesthetic?
a) Halothane
b) Isoflurane
c) Nitrous oxide
d) Desflurane
008. Sevoflurane has largely replaced halothane and isoflurane as an inhalation anesthetic of choice because:
a) Induction of anesthesia is achieved more rapidly and smoothly
b) Recovery is more rapid
c) It has low post- anesthetic organ toxicity
d) All of the above
009. The limitation of sevoflurane is:
a) High incidence of coughing and laryngospasm
b) Chemically unstable
c) Centrally mediated sympathetic activation leading to a rise of BP and HR
d) Hepatotoxicity
010. Which of the following inhalants lacks sufficient potency to produce surgical anesthesia by itself and therefore is
commonly used with another inhaled or intravenous anesthetic?
a) Halothane
b) Sevoflurane
c) Nitrous oxide
d) Desflurane
011. Which of the following inhaled anesthetics has rapid onset and recovery?
a) Nitrous oxide
b) Desflurane
c) Sevoflurane
d) All of the above
012. Indicate the inhaled anesthetic, which reduces arterial pressure and heart rate:
a) Isoflurane
b) Halothane
c) Desflurane
d) Nitrous oxide
013. Which of the following inhaled anesthetics causes centrally mediated sympathetic activation leading to a rise in blood
pressure and heart rate?
a) Desflurane
b) Sevoflurane
c) Nitrous oxide
d) Isofurane
014. Indicated the inhaled anesthetic, which decreases the ventilatory response to hypoxia:
a) Sevoflurane
b) Nitrous oxide
c) Desflurane
d) Halothane
015. Which of the following inhaled anesthetics is an induction agent of choice in patient with airway problems?
a) Desfurane
b) Nitrous oxide
c) Halothane
67
d) None of the above
016. Indicate the inhaled anesthetic, which causes the airway irritation:
a) Nitrous oxide
b) Sevoflurane
c) Halothane
d) Desflurane
017. Which of the following inhaled anesthetics increases cerebral blood flow least of all?
a) Sevoflurane
b) Nitrous oxide
c) Isoflurane
d) Desflurane
018. Indicate the inhaled anesthetic, which should be avoided in patients with a history of seizure disorders:
a) Enflurane
b) Nitrous oxide
c) Sevoflurane
d) Desflurane
019. Which of the following inhaled anesthetics can produce hepatic necrosis?
a) Soveflurane
b) Desflurane
c) Halothane
d) Nitrous oxide
020. Indicated the inhaled anesthetic, which may cause nephrotoxicity:
a) Halothane
b) Soveflurane
c) Nitrous oxide
d) Diethyl ether
021. Which of the following inhaled anesthetics decreases metheonine synthase activity and causes megaloblastic anemia?
a) Desflurane
b) Halothane
c) Nitrous oxide
d) Soveflurane
022. Unlike inhaled anesthetics, intravenous agents such as thiopental, etomidate, and propofol:
a) Have a faster onset and rate of recovery
b) Provide a state of conscious sedation
c) Are commonly used for induction of anesthesia
d) All of the above
023. Indicate the intravenous anesthetic, which is an ultra-short-acting barbiturate:
a) Fentanyl
b) Thiopental
c) Midazolam
d) Ketamine
024. Indicate the intravenous anesthetic, which is a benzodiazepine derivative:
a) Midazolam
b) Thiopental
c) Ketamin
d) Propofol
025. Which of the following agents is used to accelerate recovery from the sedative actions of intravenous benzodiazepines?
a) Naloxone
b) Flumazenil
c) Ketamine
d) Fomepizole
026. Neuroleptanalgesia has all of the following properties EXCEPT:
a) Droperidol and fentanyl are commonly used
b) It can be used with nitrous oxide to provide neuroleptanesthesia
c) Hypertension is a common consequence
d) Confusion and mental depression can occur as adverse effects
027. Which of the following intravenous anesthetics has antiemetic actions?
a) Thiopental
b) Propofol
c) Ketamine
d) Fentanyl
68
028. Indicate the intravenous anesthetic, which causes minimal cardiovascular and respiratory depressant effects:
a) Propofol
b) Thiopental
c) Etomidate
d) Midazolam
029. Indicate the intravenous anesthetic, which produces dissociative anesthesia:
a) Midazolam
b) Ketamine
c) Fentanyl
d) Thiopental
030. Ketamine anesthesia is associated with:
a) Cardiovascular stimulation
b) Increased cerebral blood flow, oxygen consumption and intracranial pressure
c) Disorientation, sensory and perceptual illusions, and vivid dreams following anesthesia
d) All of the above

(4) ORGANOTROPIC AGENTS

PART I Drugs acting on respiratory system


001. Following drugs directly activate the respiratory center EXCEPT:
a) Bemegride
b) Caffeine
c) Aethymizole
d) Cytiton
002. The mechanism of Cytiton action is:
a) Direct activation of the respiratory center
b) The reflex mechanism
c) The mixed mechanism
d) None of the above
003. Indicate the drug belonging to antitussives of narcotic type of action:
a) Glaucine hydrochloride
b) Aethylmorphine hydrochloride
c) Tusuprex
d) Libexine
004. Tick out the drug belonging to non-narcotic antitussives:
a) Libexine
b) Tusuprex
c) Codeine
d) Aethylmorphine hydrochloride
005. Indicate the expectorant with the reflex mechanism:
a) Sodium benzoate
b) Derivatives of Ipecacucnha and Thermopsis
c) Trypsin
d) Ambroxol
006. Tick the antitussive agent with a peripheral effect:
a) Codeine
b) Tusuprex
c) Libexine
d) Glaucine hydrochloride
007. Chymotrypsin is an agent containing free sulfhydryl groups. It’s:
a) True
b) False
008. All of these drugs contain free sulfhydryl groups EXCEPT:
a) Acetylcysteine
b) Ambroxol
c) Bromhexin
d) Trypsin
009. Which of the following drugs is proteolytic enzyme?
a) Potassium iodide
b) Desoxiribonuclease
c) Carbocysteine
69
d) Acetylcysteine
010. All of the following drugs destroy disulfide bonds of proteoglycans, which causes depolymerization and reduction of
viscosity of sputum, EXCEPT:
a) Acetylcysteine
b) Ambroxol
c) Desoxiribonuclease
d) Bromhexin
011. Which of these groups of drugs is used for asthma treatment?
a) Methylxanthines
b) M-cholinoblocking agents
c) Beta2 - stimulants
d) All of above
012. Tick the drug belonging to non-selective beta2-adrenomimics:
a) Salbutamol
b) Isoprenaline
c) Salmeterol
d) Terbutaline
013. Select the side-effect characteristic for non-selective beta2-adrenomimics:
a) Depression of the breathing centre
b) Tachycardia
c) Peripheral vasoconstriction
d) Dry mouth
014. Pick out the bronchodilator drug related to xanthine:
a) Atropine
b) Orciprenaline
c) Adrenaline
d) Theophylline
015. Pick out the bronchodilator drug belonging to sympathomimics:
a) Isoprenaline
b) Ephedrine
c) Atropine
d) Salbutamol
016. The property of prolonged theophyllines is the prevention of night asthmatic attacks. It’s:
a) True
b) False
017. The mechanism of methylxanthines action is:
a) Inhibition of the enzyme phosphodiesterase
b) Beta2 -adrenoreceptor stimulation
c) Inhibition of the production of inflammatory cytokines
d) Inhibition of M-cholinoreceptors
018. Which of the following M-cholinoblocking agents is used especially as an anti-asthmatic?
a) Atropine
b) Ipratropium
c) Platiphylline
d) Metacin
019. Indicate the side effect of Theophylline:
a) Bradycardia
b) Increased myocardial demands for oxygen
c) Depression of respiratory centre
d) Elevation of the arterial blood pressure
020. All of the following drugs are inhaled glucocorticoids EXCEPT:
a) Triamcinolone
b) Beclometazone
c) Sodium cromoglycate
d) Budesonide
021. Choose the drug belonging to membranestabilizing agents:
a) Zileutin
b) Sodium cromoglycate
c) Zafirlucast
d) Montelucast
022. Tick the drug which is a 5-lipoxygenase inhibitor:
70
a) Budesonide
b) Sodium cromoglycate
c) Zileutin
d) Beclometazone
023. Zileutin prevents the production of leukotrienes. This statement is:
a) True
b) False
024. Indicate the drug which is a leucotriene receptor antagonist:
a) Sodium cromoglycate
b) Zafirlucast
c) Zileutin
d) Triamcinolone
025. Zafirlucast prevents aspirin-sensitive asthma. This consideration is:
a) True
b) False

PART II Drugs used in gastrointestinal diseases


001. Tick the main approach of peptic ulcer treatment:
a) Neutralization of gastric acid
b) Eradication of Helicobacter pylori
c) Inhibition of gastric acid secretion
d) All the above
002. Gastric acid secretion is under the control of the following agents EXCEPT:
a) Histamine
b) Acetylcholine
c) Serotonin
d) Gastrin
003. Indicate the drug belonging to proton pump inhibitors:
a) Pirenzepine
b) Ranitidine
c) Omeprazole
d) Trimethaphan
004. All of the following agents intensify the secretion of gastric glands EXCEPT:
a) Pepsin
b) Gastrin
c) Histamine
d) Carbonate mineral waters
005. Which of the following drugs is an agent of substitution therapy?
a) Gastrin
b) Hydrochloric acid
c) Hystamine
d) Carbonate mineral waters
006. Choose the drug which is a H2-receptor antagonist:
a) Omeprazole
b) Pirenzepine
c) Carbenoxolone
d) Ranitidine
007. All of the following drugs are proton pump inhibitors EXCEPT:
a) Pantoprozole
b) Omeprazole
c) Famotidine
d) Rabeprazole
008. Indicate the drug belonging to M1-cholinoblockers:
a) Cimetidine
b) Ranitidine
c) Pirenzepin
d) Omeprazole
009. Which of the following drugs may cause reversible gynecomastia?
a) Omeprazole
b) Pirenzepine
71
c) Cimetidine
d) Sucralfate
010. Cimetidine has no effect on hepatic drug metabolism. It’s
a) True
b) False
011. Tick the drug forming a physical barrier to HCL and Pepsin:
a) Ranitidine
b) Sucralfate
c) Omeprazole
d) Pirenzepine
012. Which drug is an analog of prostaglandin E1?
a) Misoprostole
b) De-nol
c) Sucralfate
d) Omeprazole
013. Select the drug stimulating the protective function of the mucous barrier and the stability of the mucous membrane
against damaging factors:
a) De-nol
b) Sucralfate
c) Misoprostol
d) Omeprazole
014. Antacids are weak bases that react with gastric hydrochloric acid to form salt and water. It’s
a) True
b) False
015. Most of drugs are antacids EXCEPT:
a) Misoprostol
b) Maalox
c) Mylanta
d) Almagel
016. Indicate the drug that cause metabolic alkalosis:
a) Sodium bicarbonate
b) Cimetidine
c) Pepto-Bismol
d) Carbenoxolone
017. Choose the drug that causes constipation:
a) Sodium bicarbonate
b) Aluminium hydroxide
c) Calcium carbonate
d) Magnesium oxide
018. All of the following drugs stimulate appetite EXCEPT:
a) Vitamins
b) Bitters
c) Fepranone
d) Insulin
019. Ethyl alcohol is an agent decreasing appetite. It’s:
a) True
b) False
020. Select an anorexigenic agent affecting serotoninergic system:
a) Fenfluramine
b) Fepranone
c) Desopimone
d) Masindole
021. All of the following drugs intensify gastrointestinal motility EXCEPT:
a) Papaverine
b) Metoclopramide
c) Domperidone
d) Cisapride
022. Metoclopramide is a potent dopamine antagonist.It’s
a) True
b) False
72
023. Choose an emetic drug of central action:
a) Ipecacuanha derivatives
b) Promethazine
c) Tropisetron
d) Apomorphine hydrochloride
024. Tick the mechanism of Metoclopramide antiemetic action:
a) H1 and H2-receptor blocking effect
b) M-cholinoreceptor stimulating effect
c) D2-dopamine and 5-HT3-serotonin receptor blocking effect
d) M-cholinoblocking effect
025. Select the emetic agent having a reflex action:
a) Ipecacuanha derivatives
b) Apomorphine hydroclorid
c) Chlorpromazine
d) Metoclopramide
026. All of the following drugs are antiemetics EXCEPT:
a) Metoclopramide
b) Ondansetron
c) Chlorpromazine
d) Apomorphine hydrochloride
027. Indicate an antiemetic agent which is related to neuroleptics:
a) Metoclopramide
b) Nabilone
c) Tropisetron
d) Prochlorperazine
028. All of these drugs reduce intestinal peristalsis EXCEPT:
a) Loperamide
b) Cisapride
c) Methyl cellulose
d) Magnesium aluminium silicate
029. Indicate the laxative drug belonging to osmotic laxatives:
a) Docusate sodium
b) Bisacodyl
c) Phenolphthalein
d) Sodium phosphate
030. The mechanism of stimulant purgatives is:
a) Increasing the volume of non-absorbable solid residue
b) Increasing motility and secretion
c) Altering the consistency of the feces
d) Increasing the water content
031. Choose the drug irritating the gut and causing increased peristalsis:
a) Phenolphthalein
b) Methyl cellulose
c) Proserine
d) Mineral oil
032. All of the following drugs stimulate bile production and bile secretion EXCEPT:
a) Chenodiol
b) Cholenszyme
c) Oxaphenamide
d) Cholosas
033. Tick the stimulant of bile production of vegetable origin:
a) Oxaphenamide
b) Papaverine
c) Cholenzyme
d) Cholosas
034. Select the drug which inhibits peristalsis:
a) Castor oil
b) Bisacodyl
c) Loperamide
d) Sorbitol
035. Choose the drug affecting the biliary system and relaxing Oddy sphincter:
73
a) Cholosas
b) Oxaphenamide
c) No-spa
d) Cholenzyme

PART III Drugs acting on hematopoietic system


001. Following drugs stimulate erythrogenesis EXCEPT:
a) Iron dextran
b) Vitamine B12
c) Methotrexate
d) Folic acid
002. Choose the drug depressing erythrogenesis:
a) Radioactive phosphorus 32
b) Ferrous sulfate
c) Molgramostim
d) Folic acid
003. Which drug does not influence leucopoiesis?
a) Filgrastim
b) Erythropoetin
c) Doxorubicin
d) Methotrexate
004. Iron deficiency anemia leads to pallor, fatigue, dizziness, exertional dyspnea and other symptoms of tissue ischemia. It’s:
a) True
b) False
005. All of the following drugs used for iron deficiency anemia EXCEPT:
a) Ferrous sulfate
b) Folic acid
c) Ferrous gluconate
d) Ferrous fumarate
006. Tick the drug for parenteral iron therapy:
a) Ferrous sulfate
b) Fercoven
c) Ferrous lactate
d) Ferrous fumarate
007. Indicate the drug which increases absorption of iron from intestine:
a) Cyanocobalamin
b) Folic acid
c) Ascorbic acid
d) Erythropoetin
008. The drugs used for oral administration EXCLUDE:
a) Ferrous sulfate
b) Fercoven
c) Ferrous lactate
d) Ferrous fumarate
009. Pernicious anemia is developed due to deficiency of:
a) Erythropoetin
b) Vitamin B12
c) Iron
d) Vitamin B6
010. Select the drug used for pernicious anemia:
a) Ferrous lactate
b) Cyanocobalamin
c) Iron dextran
d) Ferrous gluconate
011. An adverse effect of oral iron therapy is:
a) Anemia
b) Thrombocytopenia
c) Headache
d) Constipation
012. Choose the drug which contains cobalt atom:
74
a) Folic acid
b) Iron dextran
c) Cyanocobalamine
d) Ferrous gluconate
013. Tick the drug used in aplastic anemia:
a) Fercoven
b) Cyanocobalamine
c) Epoetin alpha
d) Folic acid
014. Folic acid is recommended for treatment of megaloblastic anemia. This statement is:
a) True
b) False
015. Select the drug of granulocyte colony-stimulating factor:
a) Filgrastim
b) Methotrexate
c) Erythropoetin
d) Doxorubicin

PART IV Drugs used in disorders of coagulation


001. All of the following physiologic reactions are involved in the control of bleeding EXCEPT:
a) Platelet adhesion reaction
b) Platelet release reaction
c) Activation of the antifibrinolytic system
d) Triggering of the coagulation process
002. Which of the following substances is synthesized within vessel walls and inhibits thrombogenesis?
a) Thromboxane A2 (TXA2)
b) Prostacyclin (PGI2)
c) Prostaglandin ((PGE)
d) None of the above
003. All of the following groups of drugs are for thrombosis treatment EXCEPT:
a) Anticoagulant drugs
b) Antifibrinolitic drugs
c) Fibrinolitic drugs
d) Antiplatelet drugs
004. Pick out the drug belonging to anticoagulants of direct action:
a) Aspirin
b) Heparin
c) Dicumarol
d) Phenprocoumon
005. Which of the following drugs has low-molecular weight?
a) Dicumarol
b) Enoxaparin
c) Phenprocoumon
d) Heparin
006. Indicate the drug belonging to antagonists of heparin:
a) Aspirin
b) Dicumarol
c) Dalteparin
d) Protamine sulfate
007. Tick the drug used as an oral anticoagulant:
a) Heparin
b) Daltreparin
c) Dicumarol
d) Enoxaparin
008. All of the following drugs are indirect acting anticoagulants EXCEPT:
a) Dicumarol
b) Warfarin
c) Dalteparin
d) Phenindione
009. Which of the following drugs belongs to coumarin derivatives?
75
a) Heparin
b) Enoxaparin
c) Dalteparin
d) Warfarin
010. Heparin is effective when administred orally. This consideration is:
a) True
b) False
011. All of these drugs are antiplatelet agents EXCEPT:
a) Aspirin
b) Urokinase
c) Ticlopidine
d) Clopidogrel
012. The use of heparin is recommended for treatment of deep venous thrombosis. This statement is:
a) True
b) False
013. Mechanism of aspirin action is:
a) Converts inactive plasminogen into active plasmin
b) Inhibits COX and thus thromboxane synthesis
c) Enhances the interaction between antitrombin III and both thrombin and the factors involved in the intrinsic clotting
cascade
d) Inhibits the glycoprotein IIb/IIIa complex
014. Which doses of Aspirin may be more effective in inhibiting Tromboxane A2?
a) Low
b) High
015. Which of the following drugs is an inhibitor of platelet glycoprotein IIb/IIIa receptors?
a) Aspirin
b) Clopidogrel
c) Ticlopidine
d) Abciximab
016. Ticlopidine is an inhibitor of ADP-induced platelet aggregation. It’s:
a) True
b) False
017. Which of the following drugs is fiibrinolytic?
a) Ticlopidine
b) Streptokinase
c) Aspirin
d) Warfarin
018. Mechanism of urokinase action is an inhibition of Thromboxane A2. This statement is:
a) True
b) False
019. Fibrinolytic drugs are used for following EXCEPT:
a) Central deep venous thrombosis
b) Multiple pulmonary emboli
c) Heart failure
d) Acute myocardial infarction
020. Indicate the drug belonging to fibrinoliytic inhibitors:
a) Aminocapronic acid
b) Ticlopidine
c) Streptokinase
d) Vitamin K
021. Aminocapronic acid is a drug of choice for treatment of:
a) Acute myocardial infarction
b) Bleeding from fibrinolytic therapy
c) Heart failure
d) Multiple pulmonary emboli
022. Tranexamic acid is an analog of aminocapronic acid. It’s:
a) True
b) False
76
PART V Drugs used for treatment of heart failure
001. All of the following are normally involved in the pathogenesis of heart failure EXCEPT:
a) A cardiac lesion that impairs cardiac output
b) An increase in peripheral vascular resistance
c) A decrease in preload
d) An increase in sodium and water retention
002. All of the following are compensatory mechanisms that occur during the pathogenesis of congestive heart failure
EXCEPT:
a) An increase in ventricular end-diastolic volume
b) An increase in the concentration of plasma catecholamines
c) An increase in vagal tone
d) Increased activity of the renin-angiotensin-aldosterone system
003. All of the following are recommended at the initial stages of treating patients with heart failure EXCEPT:
a) Reduced salt intake
b) Verapamil
c) ACE inhibitors
d) Diuretics
004. All of the following agents belong to cardiac glycosides EXCEPT:
a) Digoxin
b) Strophantin K
c) Amrinone
d) Digitoxin
005. The non-glycoside positive inotropic drug is:
a) Digoxin
b) Strophantin K
c) Dobutamine
d) Digitoxin
006. Sugar molecules in the structure of glycosides influence:
a) Cardiotonic action
b) Pharmacokinetic properties
c) Toxic properties
d) All of the above
007. Aglycone is essential for:
a) Plasma protein binding
b) Half-life
c) Cardiotonic action
d) Metabolism
008. Choose the derivative of the plant Foxglove (Digitalis):
a) Digoxin
b) Strophantin K
c) Dobutamine
d) Amrinone
009. All of the following statements regarding cardiac glycosides are true EXCEPT:
a) They inhibit the Na+/K+-ATPase and thereby increase intracellular Ca++ in myocardial cells
b) They cause a decrease in vagal tone
c) Children tolerate higher doses of digitalis than do adults
d) The most frequent cause of digitalis intoxication is concurrent administration of diuretics that deplete K+
010. An important action of digitalis is to increase vagal tone. It’s:
a) True
b) False
011. Digoxin is thought to increase intracellular concentrations of calcium in myocardial cells by indirectly slowing the action of
the sodium-calcium exchanger. This consideration is:
a) True
b) False
012. Compare the half-life of digoxin and the half-life of digitoxin:
a) Digoxin is greater than digitoxin
b) Digitoxin is greater than digoxin
013. All of the following statements regarding cardiac glycosides are true EXCEPT:
a) They inhibit the activity of the Na+/K+-ATPase
b) They decrease intracellular concentrations of calcium in myocytes
77
c) They increase vagal tone
d) They have a very low therapeutic index
014. All of the following statements regarding cardiac glycosides are true EXCEPT:
a) Digoxin is a mild inotrope
b) Digoxin increases vagal tone
c) Digoxin has a longer half-life than digitoxin
d) Digoxin acts by inhibiting the Na+/K+ ATPase
015. The most cardiac manifestation of glycosides intoxication is:
a) Atrioventricular junctional rhythm
b) Second-degree atrioventricular blockade
c) Ventricular tachycardia
d) All the above
016. The manifestations of glycosides intoxication are:
a) Visual changes
b) Ventricular tachyarrhythmias
c) Gastrointestinal disturbances
d) All the above
017. For digitalis-induced arrhythmias the following drug is favored:
a) Verapamil
b) Amiodarone
c) Lidocaine
d) Propanolol
018. In very severe digitalis intoxication the best choice is to use:
a) Lidocaine
b) Digibind (Digoxin immune fab)
c) Oral potassium supplementation
d) Reducing the dose of the drug
019. All of the following statements regarding cardiac glycoside-induced ventricular tachyarrhythmias are true EXCEPT:
a) Lidocaine is a drug of choice in treatment
b) Digibind should be used in life-threatening cases
c) They occur more frequently in patients with hyperkalemia than in those with hypokalemia
d) They are more likely to occur in patients with a severely damaged heart
020. This drug is a selective beta-1 agonist:
a) Digoxin
b) Dobutamine
c) Amrinone
d) Dopamine
021. Tolerance to this inotropic drug develops after a few days:
a) Amrinone
b) Amiodarone
c) Dobutamine
d) Adenosine
022. This drug inhibits breakdown of cAMP in vascular smooth muscle:
a) Digoxin
b) Dobutamine
c) Amrinone
d) Dopamine
023. This drug is useful for treating heart failure because it increases the inotropic state and reduces afterload:
a) Amiodarone
b) Amrinone
c) Propanolol
d) Enalapril
024. This drug acts by inhibiting type III cyclic nucleotide phosphodiesterase:
a) Amiodarone
b) Milrinone
c) Propanolol
d) Enalapril
025. All of the following statements regarding inhibitors of type III phosphodiesterase are true EXCEPT:
a) They raise cAMP concentrations in cardiac myocytes
b) They reduce afterload
c) They show significant cross-tolerance with beta-receptor agonists
78
d) They are associated with a significant risk for cardiac arrhythmias
026. All of the following drugs are used in the treatment of severe congestive heart failure EXCEPT:
a) Verapamil
b) Digoxin
c) Dobutamine
d) Dopamine
027. Drugs most commonly used in chronic heart failure are:
a) Cardiac glycosides
b) Diuretics
c) Angiotensin-converting enzyme inhibitors
d) All the above
028. All of the following statements concerning angiotensin converting enzyme (ACE) inhibitors are true EXCEPT:
a) They act by inhibiting the ability of renin to convert angiotensinogen to angiotensin I.
b) Enalapril is a prodrug that is converted to an active metabolite
c) They reduce secretion of aldosterone
d) They can produce hyperkalemia in combination with a potassium-sparing diuretic
029. All of the following effects of ACE inhibitors may be useful in treating heart failure EXCEPT:
a) They decrease afterload
b) They increase circulating catecholamine levels
c) They reduce reactive myocardial hypertrophy
d) They increase myocardial beta-1 adrenergic receptor density
030. All of the following statements concerning the use of angiotensin-converting enzyme (ACE) inhibitors in the treatment of
heart failure are true EXCEPT:
a) They improve hemodynamics by decreasing afterload
b) They can increase plasma cholesterol levels
c) They may slow the progression of heart failure by preventing myocardial and vascular remodeling
d) They are effective first-line agents in the treatment of chronic heart failure

PART VI Antiarrhythmic agents


001. This drug is a Class IA antiarrhythmic drug:
a) Sotalol
b) Propranolol
c) Verapamil
d) Quinidine
002. This drug is a Class IC antiarrhythmic drug:
a) Flecainide
b) Sotalol
c) Lidocaine
d) Verapamil
003. This drug is a Class IC antiarrhythmic drug:
a) Flecainide
b) Sotalol
c) Lidocaine
d) Verapamil
004. This drug is a Class II antiarrhythmic drug:
a) Flecainide
b) Propranolol
c) Lidocaine
d) Verapamil
005. This drug is a Class III antiarrhythmic drug:
a) Flecainide
b) Sotalol
c) Lidocaine
d) Verapamil
006. This drug prolongs repolarization:
a) Flecainide
b) Sotalol
c) Lidocaine
d) Verapamil
007. This drug is a Class IV antiarrhythmic drug:
79
a) Flecainide
b) Sotalol
c) Lidocaine
d) Verapamil
008. This drug is used in treating supraventricular tachycardias:
a) Digoxin
b) Dobutamine
c) Amrinone
d) Dopamine
009. This drug is associated with Torsades de pointes.
a) Flecainide
b) Sotalol
c) Lidocaine
d) Verapamil
010. This drug has beta-adrenergic blocking activity:
a) Flecainide
b) Sotalol
c) Lidocaine
d) Verapamil
011. This drug is useful in terminating atrial but not ventricular tachycardias:
a) Flecainide
b) Sotalol
c) Lidocaine
d) Verapamil
012. This is a drug of choice for acute treatment of ventricular tachycardias:
a) Flecainide
b) Sotalol
c) Lidocaine
d) Verapamil
013. The calcium channel blockers have direct negative inotropic effects because they reduce the inward movement of
calcium during the action potential. Thic consideration is:
a) True
b) False
014. Common unwanted effects of the dihydropyridines are due to vasodilation. It’s:
a) True
b) False
015. Verapamil is a more potent vasodilator than nifedipine. This statement is:
a) True
b) False
016. This drug is contraindicated in patients with moderate to severe heart failure:
a) Nifedipine
b) Verapamil
c) Both of the above
d) None of the above
017. This drug is an effective bronchodilator:
a) Nifedipine
b) Verapamil
c) Both of the above.
d) None of the above
018. This drug is used intravenously to terminate supraventricular tachycardias:
a) Nifedipine
b) Verapamil
c) Both of the above
d) None of the above
019. This drug has a little or no direct effect on chronotropy and dromotropy at normal doses
a) Nifedipine
b) Diltiazem
c) Verapamil
d) All of the above
020. Verapamil has a significant effect on automaticity in the SA node. It’s:
80
a) True
b) False
021. This drug acts by inhibiting slow calcium channels in the SA and AV nodes:
a) Quinidine
b) Adenosine
c) Flecainide
d) Diltiazem
022. All of the following statements regarding verapamil are true EXCEPT:
a) It blocks L-type calcium channels
b) It increases heart rate
c) It relaxes coronary artery smooth muscle
d) It depresses cardiac contractility
023. All of the following calcium channel blockers are useful in the treatment of cardiac arrhythmias EXCEPT:
a) Bepridil
b) Diltiazem
c) Verapamil
d) Nifedipine
024. All of the following are common adverse effects of calcium channel blockers EXCEPT:
a) Skeletal muscle weakness
b) Dizziness
c) Headache
d) Flushing
025. Tick the adverse reactions characteristic for lidocaine:
a) Agranulocytosis, leucopenia
b) Extrapyramidal disorders
c) Hypotension, paresthesias, convulsions
d) Bronchospasm, dyspepsia

PART VII Drugs for Angina Pectoris treatment


001. Angina pectoris is:
a) Severe constricting chest pain, often radiating from the precordium to the left shoulder and down the arm,
due to insufficient blood supply to the heart that is usually caused by coronary disease
b) An often fatal form of arrhythmia characterized by rapid, irregular fibrillar twitching of the ventricles of the heart instead
of normal contractions, resulting in a loss of pulse
c) The cardiovascular condition in which the heart ability to pump blood weakens
d) All of the above
002. All these drug groups useful in angina both decrease myocardial oxygen requirement (by decreasing the determinations
of oxygen demand) and increase myocardial oxygen delivery (by reversing coronary arterial spasm), EXCEPT:
a) Nitrates and nitrite drugs (Nitroglycerin, Isosorbide dinitrate)
b) Calcium channel blockers (Nifedipine, Nimodipine)
c) Beta-adrenoceptor-blocking drugs (Atenolol, Metoprolol)
d) Potassium channel openers (Minoxidil)
003. This drug group useful in angina decreases myocardial oxygen requirement (by decreasing the determinations of oxygen
demand) and does not increase myocardial oxygen delivery (by reversing coronary arterial spasm):
a) Nitrates and nitrite drugs (Nitroglycerin, Isosorbide dinitrate)
b) Myotropic coronary dilators (Dipyridamole)
c) Potassium channel openers (Minoxidil)
d) Beta-adrenoceptor-blocking drugs (Atenolol, Mtoprolol)
004. This drug group useful in angina increase myocardial oxygen delivery (by reversing coronary arterial spasm) and does
not decrease myocardial oxygen requirement (by decreasing the determinations of oxygen demand):
a) Beta-adrenoceptor-blocking drugs (Atenolol, Metoprolol):
b) Myotropic coronary dilators (Dipyridamole)
c) Calcium channel blockers (Nifedipine, Nimodipine)
d) Potassium channel openers (Minoxidil)
005. Which of the following statements concerning nitrate mechanism of action is True?
a) Therapeutically active agents in this group are capable of releasing nitric oxide (NO) in to vascular smooth muscle
target tissues
b) Nitric oxide (NO) is an effective activator of soluble guanylyl cyclase and probably acts mainly through this mechanism
c) Nitrates useful in angina decrease myocardial oxygen requirement (by decreasing the determinations of oxygen
demand) and increase myocardial oxygen delivery (by reversing coronary arterial spasm)
d) All of the above
81
006. Which of the following nitrates and nitrite drugs are long-acting?
a) Nitroglycerin, sublingual
b) Isosorbide dinitrate, sublingual (Isordil, Sorbitrate)
c) Amyl nitrite, inhalant (Aspirols, Vaporole)
d) Sustac
007. Which of the following nitrates and nitrite drugs is a short-acting drug?
a) Nitroglycerin, 2% ointment (Nitrol)
b) Nitroglycerin, oral sustained-release (Nitrong)
c) Amyl nitrite, inhalant (Aspirols, Vaporole)
d) Sustac
008. Which of the following nitrates and nitrite drugs is used for prevention of angina attack?
a) Nitroglycerin, 2% ointment (Nitrol)
b) Nitroglycerin, oral sustained-release (Nitrong)
c) Isosorbide mononitrate (Ismo)
d) All of the above
009. Duration of nitroglycerin action (sublingual) is:
a) 10-30 minutes
b) 6-8 hours
c) 3-5 minutes
d) 1.5-2 hours
010. The following statements concerning mechanism of nitrate beneficial clinical effect are true, EXCEPT?
a) Decreased myocardial oxygen requirement
b) Relief of coronary artery spasm
c) Improved perfusion to ischemic myocardium
d) Increased myocardial oxygen consumption
011. Side effect of nitrates and nitrite drugs are, EXCEPT:
a) Orthostatic hypotension, tachycardia
b) GI disturbance
c) Throbbing headache
d) Tolerance
012. The following statements concerning mechanism of calcium channel blockers’ action are true, EXCEPT:
a) Therapeutically active agents in this group are capable of releasing nitric oxide (NO) in vascular smooth
muscle target tissues
b) Calcium channel blockers bind to L-type calcium channel sites
c) Calcium channel blockers useful in angina decrease myocardial oxygen requirement (by decreasing the
determinations of oxygen demand) and increase myocardial oxygen delivery (by reversing coronary arterial spasm)
d) Calcium channel blockers decrease transmembrane calcium current associated in smooth muscle with long-lasting
relaxation and in a cardiac muscle with a reduction in contractility
013. Which of the following antianginal agents is a calcium channel blocker?
a) Nitroglycerin
b) Dipyridamole
c) Minoxidil
d) Nifedipine
014. Which of the following cardiovascular system effects refers to a calcium channel blocker?
a) The reduction of peripheral vascular resistance
b) The reduction of cardiac contractility and, in some cases, cardiac output
c) Relief of coronary artery spasm
d) All of the above
015. Main clinical use of calcium channel blockers is:
a) Angina pectoris
b) Hypertension
c) Supraventricular tachyarrhythmias
d) All of the above
016. Which of the following antianginal agents is a myotropic coronary dilator:
a) Dipyridamole
b) Validol
c) Atenolol
d) Alinidine
017. Which of the following antianginal agents is a beta-adrenoceptor-blocking drug:
a) Dipyridamole
b) Validol
82
c) Atenolol
d) Alinidine
018. The following agents are cardioselective beta1-adrenoceptor-blocking drugs labeled for use in angina, EXCEPT:
a) Metoprolol
b) Talinolol
c) Atenolol
d) Propranolol
019. Which of the following statements concerning beta-adrenoceptor-blocking drugs are true:
a) These agents decrease transmembrane calcium current associated in a smooth muscle with long-lasting relaxation
and in a cardiac muscle with a reduction in contractility
b) These agents has a moderate reflex and vascular dilative action caused by the stimulation of sensitive nerve endings
c) Beneficial effects of these agents are related primarily to their hemodynamic effects – decreased heart rate,
blood pressure, and contractility – which decrease myocardial oxygen requirements at rest and during
exercise
d) These agents increase the permeability of K channels, probably ATP-dependent K channels, that results in stabilizing
the membrane potential of excitable cells near the resting potential
020. Which of the following antianginal agents refers to reflex coronary dilators:
a) Dipyridamole
b) Validol
c) Atenolol
d) Alinidine
021. Which of the following statements concerning Validol is true:
a) Validol has a moderate reflex and vascular dilative action caused by the stimulation of sensitive nerve endings
b) At sublingual administration the effect is produced in five minutes and 70 % of the preparation is released in 3 minutes
c) It is used in cases of angina pectoris, motion sickness, nausea, vomiting when seasick or airsick and headaches due
to taking nitrates
d) All of the above
022. Which of the following antianginal agents is the specific bradycardic drug:
a) Dipyridamole
b) Validol
c) Atenolol
d) Alinidine
023. Following statements concerning specific bradycardic agents (Falipamil, Alinidine) are true, EXCEPT:
a) Bradycardic drugs have a moderate reflex and vascular dilative action caused by the stimulation of sensitive
nerve endings
b) The predominant effect of bradycardic drugs is a decrease in heart rate without significant changes in arterial pressure
c) The protective effect of bradycardic drugs is likely due to a reduced O2 demand
d) Specific bradycardic agents are used in the management of a wide range of cardiovascular disorders, including sinus
tachyarrhythmias and angina pectoris
024. Which of the following statements concerning Dipyridamole is true?
a) Dipyridamole is an agent that blocks the reabsorption and breakdown of adenosine that results in an increase of
endogenous adenosine and vasodilatation
b) The drug causes relative hypoperfusion of myocardial regions served by coronary arteries with haemodynamically
significant stenoses
c) Dipyridamole is a platelet aggregation inhibitor
d) All of the above
025. Which of the following antianginal agents is a potassium channel opener:
a) Dipyridamole
b) Validol
c) Atenolol
d) Minoxidil
026. Which of the following statements concerning potassium channel openers is true?
a) These agents decrease transmembrane calcium current associated in a smooth muscle with long-lasting relaxation
and in a cardiac muscle with a reduction in contractility
b) These agents has a moderate reflex and vascular dilative action caused by the stimulation of sensitive nerve endings
c) Beneficial effects of these agents are related primarily to their hemodynamic effects – decreased heart rate, blood
pressure, and contractility – which decrease myocardial oxygen requirements at rest and during exercise
d) These agents increase the permeability of K channels, probably ATP-dependent K channels, that results in
stabilizing the membrane potential of excitable cells near the resting potential
83
PART VIII Antihypertensive drugs
001. This drug reduces blood pressure by acting on vasomotor centers in the CNS:
a) Labetalol
b) Clonidine
c) Enalapril
d) Nifedipine
002. All of the following are central acting antihypertensive drugs EXCEPT:
a) Methyldopa
b) Clonidine
c) Moxonidine
d) Minoxidil
003. A ganglioblocking drug for hypertension treatment is:
a) Hydralazine
b) Tubocurarine
c) Trimethaphan
d) Metoprolol
004. Pick out the sympatholythic drug:
a) Labetalol
b) Prazosin
c) Guanethidine
d) Clonidine
005. Tick the drug with nonselective beta-adrenoblocking activity:
a) Atenolol
b) Propranolol
c) Metoprolol
d) Nebivolol
006. Choose the selective blocker of beta-1 adrenoreceptors:
a) Labetalol
b) Prazosin
c) Atenolol
d) Propranolol
007. Pick out the drug – an alpha and beta adrenoreceptors blocker:
a) Labetalol
b) Verapamil
c) Nifedipine
d) Metoprolol
008. This drug inhibits the angiotensin-converting enzyme:
a) Captopril
b) Enalapril
c) Ramipril
d) All of the above
009. This drug is a directly acting vasodilator:
a) Labetalol
b) Clonidine
c) Enalapril
d) Nifedipine
010. Pick out the diuretic agent for hypertension treatment:
a) Losartan
b) Dichlothiazide
c) Captopril
d) Prazosin
011. This drug blocks alpha-1 adrenergic receptors:
a) Prazosin
b) Clonidine
c) Enalapril
d) Nifedipine
012. This drug activates alpha-2 adrenergic receptors:
a) Labetalol
b) Phentolamine
c) Clonidine
d) Enalapril
84
013. This drug is an inhibitor of renin synthesis:
a) Propranolol
b) Enalapril
c) Diazoxide
d) Losartan
014. This drug is a non-peptide angiotensin II receptor antagonist:
a) Clonidine
b) Captopril
c) Losartan
d) Diazoxide
015. This drug is a potassium channel activator:
a) Nifedipine
b) Saralasin
c) Diazoxide
d) Losartan
016. All of the following statements regarding angiotensin II are true EXCEPT:
a) It is a peptide hormone
b) It stimulates the secretion of aldosterone
c) Angiotensin I is almost as potent as angiotensin II
d) It is a potent vasoconstrictor
017. This drug is contraindicated in patients with bronchial asthma:
a) Propranolol
b) Clonidine
c) Enalapril
d) Nifedipine
018. This drug is converted to an active metabolite after absorption:
a) Labetalol
b) Clonidine
c) Enalapril
d) Nifedipine
019. This drug routinely produces some tachycardia:
a) Propranolol
b) Clonidine
c) Enalapril
d) Nifedipine
020. All of the following statements regarding vasodilators are true EXCEPT:
a) Hydralazine causes tachycardia
b) Nifedipine is a dopamine receptor antagonist
c) Nitroprusside dilates both arterioles and veins
d) Minoxidil can cause hypertrichosis
021. All of the following statements regarding verapamil are true EXCEPT:
a) It blocks L-type calcium channels
b) It increases heart rate
c) It relaxes coronary artery smooth muscle
d) It depresses cardiac contractility
022. Choose the unwanted effects of clonidine:
a) Parkinson’s syndrome
b) Sedative and hypnotic effects
c) Agranulocytosis and aplastic anemia
d) Dry cough and respiratory depression
023. The reason of beta-blockers administration for hypertension treatment is:
a) Peripheral vasodilatation
b) Diminishing of blood volume
c) Decreasing of heart work
d) Depression of vasomotor center
024. An endogenous vasoconstrictor that can stimulate aldosterone release from suprarenal glands:
a) Angiotensinogen
b) Angiotensin I
c) Angiotensin II
d) Angiotensin-converting enzyme
025. Choose the group of antihypertensive drugs which diminishes the metabolism of bradykinin:
85
a) Ganglioblockers
b) Alfa-adrenoblockers
c) Angiotensin-converting enzyme inhibitors
d) Diuretics
026. Hydralazine (a vasodilator) can produce:
a) Seizures, extrapyramidal disturbances
b) Tachycardia, lupus erhythromatosis
c) Acute hepatitis
d) Aplastic anemia
027. Choose the vasodilator which releases NO:
a) Nifedipine
b) Hydralazine
c) Minoxidil
d) Sodium nitroprusside
028. The reason of diuretics administration for hypertension treatment is:
a) Block the adrenergic transmission
b) Diminishing of blood volume and amount of Na+ ions in the vessels endothelium
c) Depression of rennin-angiotensin-aldosterone system
d) Depression of the vasomotor center
029. Tick the diuretic agent – aldosterone antagonist:
a) Furosemide
b) Spironolactone
c) Dichlothiazide
d) Captopril
030. Tick the diuretic agent having a potent and rapid effect:
a) Furosemide
b) Spironolactone
c) Dichlothiazide
d) Indapamide

PART IX Hypertensive (anti-hypotensive) drugs. Drugs influencing cerebral blood flow. Anti-migraine agents
001. The main principle of shock treatment is:
a) To increase the arterial pressure
b) To increase the peripheral vascular resistance
c) To increase the cardiac output
d) To improve the peripheral blood flow
002. Pick out the drug which increases cardiac output:
a) Noradrenalin
b) Methyldopa
c) Phenylephrine
d) Angiotensinamide
003. Tick the synthetic vasoconstrictor having an adrenomimic effect:
a) Noradrenalin
b) Adrenalin
c) Phenylephrine
d) Angiotensinamide
004. Indicate the vasoconstrictor of endogenous origin:
a) Ephedrine
b) Phenylephrine
c) Xylomethazoline
d) Angiotensinamide
005. Which type of receptors can be activated by angiotensinamide:
a) Adrenergic receptors
b) Cholinergic receptors
c) Dopaminergic receptors
d) Angiotensin’s receptors
006. General unwanted effects of vasoconstrictors is:
a) Increase of arterial pressure
b) Increase of cardiac output
c) Decrease of peripheral blood flow
d) Increase of blood volume
86
007. For increasing blood pressure in case of low cardiac output the following agents must be used:
a) Ganglioblockers
b) Vasoconstrictors
c) Positive inotropic drugs
d) Diuretics
008. Tick the positive inotropic drug of glycoside structure:
a) Dopamine
b) Digoxin
c) Dobutamine
d) Adrenalin
009. Tick the positive inotropic drug of non-glycoside structure:
a) Digitoxin
b) Digoxin
c) Dobutamine
d) Strophanthin
010. Dopamine at low doses influences mainly:
a) Alfa-adrenoreceptors (leads to peripheral vasoconstriction)
b) Dopamine receptors (leads to vasodilation of renal and mesenterial vessels)
c) Beta-1 adrenoreceptors (leads to enhanced cardiac output)
d) All of the above
011. Dopamine at medium doses influences mainly:
a) Alfa-adrenoreceptors (leads to peripheral vasoconstriction)
b) Dopamine receptors (leads to vasodilation of renal and mesenterial vessels)
c) Beta-1 adrenoreceptors (leads to enhanced cardiac output)
d) All of the above
012. Dopamine in high doses influences mainly the:
a) Alfa-adrenoreceptors (leads to peripheral vasoconstriction)
b) Dopamine’s receptors (leads to vasodilation of renal and mesenterial vessels)
c) Beta-1 adrenoreceptors (leads to enhancing of cardiac output)
d) All of the above
013. Tick the group of drugs for treatment of shock with hypovolaemia (reduced circulating blood volume):
a) Positive inotropic drugs
b) Vasoconstrictors
c) Plasmoexpanders
d) Analeptics and tonics
014. Tick the group of drugs for chronic hypotension treatment:
a) Positive inotropic drugs
b) Vasoconstrictors
c) Plasmoexpanders
d) Analeptics and tonics
015. Indicate the group of drugs influencing the cerebral flow:
a) Ca-channel blockers
b) Derivatives of GABA
c) Derivatives of Vinca minor plant
d) All the above
016. Tick the drug influencing the blood flow which is related to antiplatelet agents:
a) Heparin
b) Aspirin
c) Pyracetam
d) Tanakan
017. Which of the following drugs is related to anticoagulants and may be useful in disorders of cerebral circulation?
a) Aspirin
b) Cinnarizine
c) Nicergoline
d) Heparin
018. Indicate the drugs which are Ca-channel blockers influencing the brain blood flow:
a) Aminalon, Picamilon
b) Nimodipine, Cinnarizine
c) Heparin, Warfarin
d) Vinpocetine, Nicergoline
019. Indicate the drugs influencing the blood flow in the brain - derivatives of GABA:
87
a) Aminalon, Picamilon
b) Nimodipine, Cinnarizine
c) Heparin, Warfarin
d) Vinpocetine, Nicergoline
020. Indicate the drug - Vinca minor alcaloid:
a) Nicergoline
b) Warfarin
c) Cinnarizine
d) Vinpocetine
021. Tick the drug – a derivative of Ergot:
a) Nicergoline
b) Warfarin
c) Cinnarizine
d) Vinpocetine
022. Indicate the nootropic agent useful in disorders of brain circulation:
a) Aspirin
b) Pyracetam
c) Warfarin
d) All the above
023. What is the main action of GABA derivatives in disorders of brain circulation?
a) Decrease of vessel permeability
b) Stimulation of the metabolic processes in neurons
c) Brain vessel constriction
d) Intracranial pressure increase
024. Choose the appropriate mechanism of vinpocetine action:
a) It dilates cerebral vessels and improves blood supply
b) It constricts cerebral vessels and decreases blood supply
c) It stimulates GABA-receptors and thus increases cerebral metabolic processes
d) It constricts peripheral vessels and increases blood pressure
025. Antiaggregants are used in disorders of brain circulation for:
a) Stimulation of the metabolic processes in neurons
b) Dilation of cerebral vessels
c) Improving the microcirculation in cerebral tissue
d) All the above
026. Migraine is a disorder connected with:
a) Thrombosis of cerebral vessels
b) Brain hemorrhage
c) Dysfunction of regulation of cerebral vessel tonus
d) Malignant growth in brain
027. Main agents for acute migraine attack treatment are Ergot and indol derivatives and NSAID’s. The consideration is:
a) True
b) False
028. The following Indol derivative is used for treatment of acute migraine attack:
a) Paracetamol
b) Sumatriptan
c) Ergotamine
d) Metoclopramide
029. The following Ergot derivative is used for treatment of acute migraine attack:
a) Paracetamol
b) Sumatriptan
c) Ergotamine
d) Metoclopramide
030. The derivative of lysergic acid for migraine attack prevention is:
a) Metoclopramide
b) Methysergide
c) Sumatriptan
d) Ergotamine
88
(5) METABOLIC PROFILE DRUGS

PART I Hypothalamic & Pituitary Hormones, Thyroid & Antithyroid Drugs


001. Hormones are:
a) Products of endocrine gland secretion
b) Mediators of inflammatory process
c) By-products of tissue metabolism
d) Product of exocrine gland secretion
002. Select an endocrine drug which is an amino acid derivative:
a) Insulin
b) Hydrocortisone
c) Calcitonin
d) Thyroxine
003. Select an endocrine drug which is a peptide derivative:
a) Oxitocin
b) Prednisolone
c) Nandrolone
d) Progesterone
004. Select an endocrine drug which is a steroidal derivative:
a) Gonadorelin
b) Insulin
c) Levothyroxine
d) Hydrocortisone
005. Hormone analogues are:
a) Naturally occurring substances but slightly different from hormones
b) Naturally occurring substances but less efficacious than hormones
c) Naturally occurring substances having the same structure but different pharmacological properties than hormones
d) Synthetic compounds, which resemble the naturally occurring hormones
006. Regarding the mechanism of action of hormones, indicate the FALSE statement:
a) Hormones interact with the specific receptors in the wall of the cells
b) Cyclic AMP acts as a second messenger system
c) They stimulate adenylcyclase enzyme
d) Many hormones owe their effect to primary actions on subcellular membrane.
007. Hypothalamic and pituitary hormones (and their synthetic analogs) have pharmacologic applications in three areas,
EXCEPT the following:
a) As replacement therapy for hormone deficiency states
b) As drug therapy for a variety of disorders using pharmacologic doses to elicit a hormonal effect that is not present at
physiologic a blood levels
c) As a diagnostic tool for performing stimulation tests to diagnose hypo- or hyperfunctional endocrine states
d) As food supplements
008. Which of the following hormones is produced by the hypothalamic gland?
a) Growth hormone-releasing hormone (GHRH)
b) Follicle-stimulating hormone (FSH)
c) Aldosterone
d) Estradiol
009. Which of the following hormones is produced by the anterior lobe of the pituitary?
a) Thyrotropin-releasing hormone (TRH)
b) Corticotropin-releasing hormone (CRH)
c) Growth hormone (somatotropin, GH)
d) Growth hormone-releasing hormone (GHRH)
010. The posterior pitutary does NOT secret:
a) Vasopressin
b) Oxytocin
c) Growth hormone
d) All of the above
011. Which of the following organs is a target for prolactin?
a) Liver
b) Adrenal cortex
c) Thyroid
d) Mammary gland
89
012. Which of the following organ hormones is a target for growth hormone (somatotropine, GH)?
a) Glucocorticoids
b) Insulin-like growth factors (IGF, somatomedins)
c) Triiodthyronine
d) Testosterone
013. All of the following statements about growth hormone are true, EXCEPT:
a) It may stimulate the synthesis or release of somatomedins
b) Low levels of insulin-like growth factor (IGF)-1 are associated with dwarfism
c) Hypersecretion can result in acromegaly
d) It is contraindicated in subjects with closed epiphyses
014. Correct statements about adrenocorticotropic hormone (ACTH) include all of the following, EXCEPT:
a) Endogenous ACTH is also called corticotropin
b) ACTH stimulates the synthesis of corticosteroids
c) ACTH is most useful clinically as a diagnostic tool in adrenal insufficiency
d) The oral route is the preferred rout of administration
015. The hypothalamic control exists for the thyroid gland. This consideration is:
a) True
b) False.
016. Indications of bromocriptine are following, EXCEPT:
a) Prolactin-secreting adenomas
b) Amenorrhea-Galactorrhea
c) Prolactin deficiency
d) Acromegaly
017. Currently used dopamine agonists decreasing pituitary prolactin secretion are following:
a) Bromocriptine
b) Cabergoline
c) Pergolide
d) All of the above
018. Indications of oxitocin are following:
a) Labor and augment dysfunctional labor for conditions requiring early vaginal delivery
b) Incompleted abortion
c) For control of pospartum uterine hemorrhage
d) All of the above
019. Indications of vasopressin are following:
a) Diabetes mellitus
b) Hypertension
c) Pituitary diabetes insipidus
d) Incompleted abortion
020. Vasopressin possesses the following:
a) Antidiuretic property
b) Vasodilatation property
c) Release of a thyroid hormone into the plasma
d) Diuretic property
021. Oxytocin produces the following effects:
a) It causes contraction of the uterus
b) It assists the progress of spermatozoa into the uterine cavity
c) It brings about milk ejection from the lactating mammary gland
d) All of the above
022. Vasopressin causes a pressor effect by:
a) Releasing noradrenaline from the nerve terminals
b) Releasing and activating renin-angiotensin system
c) A direct action on smooth muscles of the blood vessels
d) All of the above mechanisms
023. Which of the following statements is true
a) Hypothyroidism is a syndrome resulting from deficiency of thyroid hormones and is manifested largely by a
reversible slowing down of all body functions
b) Hypothyroidism is the clinical syndrome that results when tissues are exposed to high levels of thyroid hormone
024. Which of the following hormones is produced by the thyroid gland?
a) Thyroxine
b) Thyroid-stimulating hormone
c) Thyrotropin-releasing hormone
90
d) Thyroglobulin.
025. Which of the following hormones is produced by the thyroid gland?
a) Thyroid-stimulating hormone
b) Thyrotropin-releasing hormone
c) Triiodothyronine
d) Thyroglobulin.
026. Thyroid stimulating hormone regulates the following:
a) Iodine uptake
b) Biosynthesis of iodothyroglobulin
c) Release of thyroid hormone into the plasma
d) All of the above.
027. Thyroid hormones produce various pharmacological effects. Indicate the wrong statement(s).
a) Decline of the basal metabolic rate in the body
b) Increase in the rate and force of contraction of the heart
c) Increase in the blood cholestrol level
d) Increase in the heat production
028. Synthesis and release of thyroid hormones are controlled by:
a) Anterior pituitary alone
b) Hypothalamus alone
c) Blood levels of thyroid hormones alone
d) All of the above
029. Thyrotrophin stimulates the following processes:
a) Concentration of iodine by thyroid follicles
b) Iodination of thyroglobulin
c) Release of thyroxine and triidothyronine
d) De-iodination of thyroid hormones.
030. The rate of secretion of thyrotropin is controlled by:
a) The amount of iodine in the thyroid gland
b) The amount of thyroid hormones in the thyroid gland
c) The concentration of thyroid hormones in blood
d) The concentration of catecholamines in blood
031. Indications of thyroid hormones are following, EXCEPT:
a) Cretinism
b) Myxoedema
c) Hashimoto's disease
d) For treatment of simple obesity
032. The common side effect of thyroid hormones is following:
a) Increases in basal metabolic rate
b) Angina pectoris
c) Tremors
d) Exopthalmos
033. Currently used antithyroid drugs include the following, EXCEPT:
a) Propylthiouracil (PTU)
b) Diatrizoate sodium (Hypaque)
c) Methimazole (Tapazole)
d) Potassium perchlorate
034. In an area where goitre is endemic, which of the following drugs is used?
a) Iodide 1 part in 100000
b) Propylthiouracil 200 mg daily
c) Methimazole 40 mg daily
d) Any of the above can be used.
035. Iodide preparations can be used in following situations, EXCEPT:
a) In thyroid disorders
b) In granulomatous lesions e.g. Syphilis
c) As an antiseptic
d) In iodism
036. Daily administration of large doses (several milligrammes) of iodides to a thyrotoxic patient causes:
a) Involution of the thyroid which reaches a maximum in two weeks
b) Increased vascularity of the thyroid gland
c) Decreased storage of the colloid in the thyroid gland
d) Thyroid gland growing firm and less vascular
91
037. Radioiodines (I131 and I132) is suitable for:
a) Elderly patients (over 45 years)
b) Pregnant women
c) Nursing mothers
d) Younger patients
038. Radioiodines in the body emit:
a) Mainly β radiations
b) Mainly γ radiations
c) β and γ radiations equally.
d) Do not emit any radiation, therefore, are safe

PART II Pancreatic Hormones & Antidiabetic Drugs


001. Secretory products of pancreatic β-cells are:
a) Glucagon, proglucagon
b) Insulin, C-peptide, proinsulin, islet amyloid polypeptide (IAPP)
c) Somatostatin
d) Pancreatic polypeptide (PP)
002. Insulin is:
a) A glycoprotein with a molecular weight of 6000
b) A small protein with a molecular weight of 5808 having disulphide linkage
c) A fructoolygosaccharide
d) A catecholamine
003. Insulin is a polypeptide hence:
a) It is resistant to destruction by gastric juice
b) It is destroyed by gastric juice
c) It is not a polypeptide
d) It is metabolized immediately by cellular enzymes
004. Bovine insulin is less antigenic than porcine. This consideration is:
a) True
b) False
005. Insulin causes reduction in blood sugar level by the following mechanisms, EXCEPT:
a) Increased glucose uptake in the peripheral tissue
b) Reduction of breakdown of glycogen
c) Diminished gluconeogenesis
d) Decreased glucose absorption from the gut
006. Which of the following is true for glucagon?
a) Stimulates gluconeogenesis in the liver
b) Stimulates the secretion of insulin by beta cells
c) Inhibits glucose utilization by skeletal muscle
d) Inhibits uptake of aminoacids by cells.
007. Insulin can not be administered by:
a) Oral route
b) Intravenous route
c) Subcutaneous route
d) Intramuscular route.
008. Sources of human insulin production are:
a) Recombinant DNA techniques by inserting the proinsulin gene into E. coli or yeast
b) Postmortem insulin extraction from human autopsy pancreas
c) All of the above
d) None of the above
009. The primary reason for a physician to prescribe human insulin is that:
a) It has a faster onset of action than other insulins
b) It has a shorter duration of action than other insulins
c) It can be given to patients who have an allergy to animal insulins
d) It is more effective in preventing the complications of diabetes than animal insulins
010. Correct statements about crystalline zinc (regular) insulin include all of the following, EXCEPT:
a) It can serve as replacement therapy for juvenile-onset diabetes
b) It can be administered intravenously
c) It is a short-acting insulin
d) It can be administered orally
92
011. Diabetic coma is treated by the administration of:
a) Lente insulin
b) Glucose
c) Crystalline insulin
d) Oral anti-diabetic drugs.
012. Sulphonylureas act by:
a) Reducing the absorption of carbohydrate from the gut
b) Increasing the uptake of glucose in peripheral tissues
c) Reducing the hepatic gluconeogenesis
d) Stimulating the beta islet cells of pancreas to produce insulin
013. Sulphonylureas are effective in totally insulin deficient patients. This consideration is:
a) True
b) False
014. Currently used second-generation sulfonylureas include the following, EXCEPT:
a) Glyburide (Glibenclamide)
b) Glipizide (Glydiazinamide)
c) Glimepiride (Amaril)
d) Tolbutamide (Orinase)
015. Currently used oral hypoglycemic thiazolidinediones include the following, EXCEPT
a) Pioglitazone (Actos)
b) Rosiglitazone (Avandia)
c) Troglitazone (Rezulin)
d) All of the above
016. Thiazolidinediones act by:
a) Diminishing insulin resistance by increasing glucose uptake and metabolism in muscle and adipose tissues
b) Reducing the absorption of carbohydrate from the gut
c) Stimulating the beta islet cells of pancreas to produce insulin
d) All of the above
017. Currently used alpha-glucosidase inhibitors include the following, EXCEPT:
a) Pioglitazone (Actos)
b) Acarbose (Precose)
c) Miglitol (Glyset)
d) All of the above
018. Alpha-glucosidase inhibitors act by:
a) Diminishing insulin resistance by increasing glucose uptake and metabolism in muscle and adipose tissues
b) Competitive inhibiting of intestinal alpha-ghucosidases and modulating the postprandial digestion and
absorption of starch and disaccharides
c) Reducing the absorption of carbohydrate from the gut
d) Stimulating the beta islet cells of pancreas to produce insulin
019. Potency of action of
a) Miglitol is six times higher than that of acarbose
b) Acarbose is more than that of miglitol
c) Miglitol and acarbose is equal
d) Oral hypoglycemic agents depend on the severity of hyperglycemia
020. Which of the following oral hypoglycaemic drugs stimulates both synthesis and release of insulin from beta islet cells:
a) Glibenclamide
b) Phenformin
c) Buformine
d) Metformin
021. Currently used oral hypoglycemic biguanides include the following, EXCEPT:
a) Repaglinide (Prandin)
b) Metformin
c) Phenformine
d) Glipizide
022. The action of insulin is potentiated by:
a) Sulphonylureas
b) Glucagon
c) Biguanides
d) None of the above
023. Duration of action of:
a) Tolbutamide is more than that of chlorpropamide
93
b) Chlorpropamide is more than that of tolbutamide
c) Tolbutamide and chlorpropamide is equal
d) Oral hypoglycemic agents depend on the severity of hyperglycemia
024. Side effects of sulphonylureas are less than those of biguanides. This considerations is:
a) True
b) False
025. Biguanides are used in the following conditions, EXCEPT:
a) As a supplement to sulphonylurea, where it is insufficient to give good results
b) In over weight diabetics
c) To reduce insulin requirements
d) In case of hyperglycemic shock
026. Which of the following agents is/are important hormonal antagonists of insulin in the body?
a) Glucagon
b) Adrenal steroids
c) Adrenaline
d) All of the above
027. Glucagon is:
a) A glycoprotein with a molecular weight of 6000
b) A peptide – identical in all mammals – consisting of a single chain of 29 amino acids
c) A a fructoolygosaccharide
d) A small protein with a molecular weight of 5808 having disulphide linkage
028. Which of the following statements is FALSE?
a) Glucagon is synthesized in the A cells of the pancreatic islets of Langerhans.
b) Glucagon is a peptide – identical in all mammals – consisting of a single chain of 29 amino acids
c) Glucagon is extensively degraded in the liver and kidney as well as in plasma, and at its tissue receptor sites.
d) Half-life of glucagon is between 6 and 8 hours, which is similar to that of insulin
029. Glucagon can be used in the following situations, EXCEPT:
a) Severe hypoglycemia
b) Severe hyperglycemia
c) Endocrine diagnosis
d) Beta-blocker poisoning
030. Main complications of insulin therapy include the following:
a) Hypoglycemia
b) Insulin allergy
c) Lipodystrophy at an injection site
d) All of the above

PART III The Gonadal Hormones & Inhibitors


001. The major natural estrogens produced by women are following, EXCEPT:
a) Estradiol (Estradiol-17β, E2)
b) Estron (E1)
c) Ethinyl estradiol
d) Estriol (E3)
002. Which of the following statements about estrogens are True:
a) Estrogens are required for normal sexual maturation and growth of the female
b) Estrogens decrease the rate of resorption of bone
c) Estrogens enhance the coagulability of blood
d) All of the above
003. The major synthetic estrogens are following, EXCEPT:
a) Dienestrol
b) Diethylstilbestrol
c) Benzestrol
d) Estradiol
004. Which of the following statements about estrogens are True:
a) Estradiol binds strongly to an α2-globulin and albumin with lower affinity
b) Estradiol is converted by liver and other tissues to estron and estriol and their 2-hydroxylated derivatives and
conjugated metabolites and excreted in the bile
c) Estrone and estriol have lower affinity for the estrogen receptors than estradiol
d) All of the above
005. Indications of synthetic estrogens are following, EXCEPT:
94
a) Primary hypogonadism
b) Postmenopausal hormonal therapy
c) Hormonal contraception
d) For treatment of simple obesity
006. Main complications of estrogens’ therapy include the following:
a) Postmenopausal uterine bleeding
b) Breast tenderness
c) Hyperpigmentation
d) All of the above
007. Main contraindications of estrogens’ therapy include the following:
a) Estrogen-dependent neoplasmas such as carcinoma of the endometrium or carcinoma of the breast
b) Undiagnosed genital bleeding
c) Liver disease
d) All of the above
008. Tamoxifen is:
a) Antiprogestin
b) Antiandrogen
c) Antiestrogen
d) Androgen
009. Progesterone is secreted by:
a) Ovarian follicles
b) Corpus luteum
c) Granulosa and theca cells
d) All of the above
010. The major natural progestin is:
a) Estradiol
b) Estron
c) Progesterone
d) Estriol
011. Which of the following statements about progestins is True:
a) Progesterone is rapidly absorbed following administration by any route
b) In the liver, progesterone is metabolized to pregnanediol and conjugated with glucuronic acid.
c) Significant amounts of progestins and their metabolites are excreted in the urine
d) All of the above
012. The normal ovary produces small amount of androgens, including testosterone, androstenedione, and
degydroepiandrosterone. This consideration is:
a) True
b) False
013. Noncontraceptive clinical uses of progestins are following:
a) Hormone replacement therapy
b) Dysmenorrhea
c) Endometriosis
d) All of the above
014. Estrogens possess immunomodulator properties, but progestins do immunodepressant ones. Is it TRUE or FALSE?
a) True
b) False
015. Mifepristone (RU-486) is:
a) Antiprogestin
b) Antiandrogen
c) Antiestrogen
d) Androgen
016. Mifepristone (RU-486) is used as a contraceptive and abortifacient. This consideration is:
a) True
b) False
017. Actions of mifepristone (RU-486) include:
a) Inhibition of ovulation during the follicular phase by blocking hypothalamic-pituitary progesterone receptors, which
suppresses midcycle gonadotropin release
b) During the luteal phase, inhibition of progesterone action on the uterus, which induces prostaglandin release from the
endometrium
c) Termination of pregnancy by facilitating luteolysis, menstruation, uterine motility, softening of the cervix, and
detachment of the embryo.
95
d) All of the above
018. All of the following statements about oral contraceptives are true, EXCEPT:
a) The “combination pill” contains both estrogen and progestin
b) Ethinyl estradiol and mestranol are commonly used in oral contraceptives
c) The “minipill” contains progestin alone
d) The “triphasic pill” contains estrogen, progestin, and luteinizing hormine (LH)

PART IV Glucocorticoid, Steroidal & Nonsteroidal Anti-Inflammatory Drugs


001. Glucocorticoids are hormonal steroids:
a) Having an important effect on intermediary metabolism, cardiovascular function, growth, and immunity
b) Having principally salt-retaining activity
c) Having androgenic or estrogenic activity
d) All of the above
002. Inflammation is:
a) A localized protective reaction of a tissue to irritation, injury, or infection, characterized by pain, redness,
swelling, and sometimes loss of function
b) A deficiency of the normal immune response.
c) A reaction resulting from an immune reaction produced by an individual's white blood cells or antibodies acting on the
body's own tissues or extracellular proteins.
d) All of the above
003. An acute, transient phase, of inflammation is characterized by:
a) Local vasodilatation and increased capillary permeability (phase of damage)
b) Infiltration of leucocytes and phagocytic cells (phase of exudation)
c) Tissue degeneration and fibrosis occurrence (phase of proliferation)
d) All of the above
004. A delayed, subacute phase, of inflammation is characterized by:
a) Local vasodilatation and increased capillary permeability (phase of damage)
b) Infiltration of leucocytes and phagocytic cells (phase of exudation)
c) Tissue degeneration and fibrosis occurrence (phase of proliferation)
d) All of the above
005. A chronic, proliferative phase, of inflammation is characterized by:
a) Local vasodilatation and increased capillary permeability (phase of damage)
b) Infiltration of leucocytes and phagocytic cells (phase of exudation)
c) Tissue degeneration and fibrosis occurrence (phase of proliferation)
d) All of the above
006. The following substances are considered to be referred to as eicosanoids:
a) Prostaglandins
b) Leukotrienes
c) Thromboxanes
d) All of the above
007. Correct statements about cortisol (hydrocortisone) include all of the following, EXCEPT:
a) Cortisol is synthesized from cholesterol
b) ACTH governs cortisol secretion
c) Most cortisol is inactivated in the liver
d) The half-life of cortisol in the circulations is normally about 60-90 hours.
008. Correct statements about glucocorticoids include all of the following, EXCEPT:
a) Effects of glucocorticoids are mediated by widely distributed glucocorticoid receptors that are members of the
superfamily of nuclear receptors.
b) Glucocorticoids have dose-related metabolic effects on carbohydrate, protein, and fat metabolism.
c) Glucocorticoids have pro-inflammatory effects.
d) Glucocorticoids have catabolic effects in lymphoid and connective tissue, muscle, fat, and skin.
009. Physiologic doses of glucocorticoid can result in:
a) Increased liver glycogen stores, gluconeogenesis and lipolysis
b) Maintenance of cardiovascular function (by potentiation of norepinephrine) and skeletal muscle function
c) Increased hemoglobin synthesis, resulting in elevated of red blood cell count
d) All of the above
010. Which of the following glucocorticoids is a short- to medium-acting drug?
a) Prednisolon
b) Dexamethasone
c) Triamcinolone
d) All of the above
96
011. Which of the following glucocorticoids is an intermediate-acting drug?
a) Cortisone
b) Triamcinolone
c) Butamethasone
d) All of the above
012. Which of the following glucocorticoids is a long-acting drug?
a) Prednisolon
b) Dexamethasone
c) Triamcinolone
d) All of the above
013. Which of the following glucocorticoids have one fluoride atom in its chemical structure?
a) Prednisolon
b) Fluocinolone
c) Triamcinolone
d) All of the above
014. Which of the following glucocorticoids have two fluoride atoms in its chemical structure?
a) Prednisolon
b) Dexamethasone
c) Fluocinolone
d) Triamcinolone
015. Which of the following glucocorticoids has no fluoride atoms in its chemical structure?
a) Prednisolon
b) Dexamethasone
c) Fluocinolone
d) Triamcinolone
016. Anti-inflammatory effect of glucocorticoids is caused by
a) Reducing the prostaglandin and leukotriene which results from inhibition of phospholipase A2
b) Reducing macrophages migration into the site of inflammation
c) Decreasing capillary permeability
d) All of the above
017. Which of the following statements concerning the anti-inflammatory effect of glucocorticoids is TRUE?
a) Anti-inflammatory effect of glucocorticoids results from inhibition of cyclooxygenase
b) Anti-inflammatory effect of glucocorticoids results from inhibition of phospholipase A2 and reducing
prostaglandin and leukotriene synthesis
c) Induction of cyclooxygenase II expression which results in reducing amount of an enzyme available to produce
prostoglandins
d) All of the above
018. Immunosupressive effect of glucocorticoids is caused by:
a) Reducing concentration of lymphocytes (T and B cells) and inhibiting function of tissue macrophages and
other antigen-presenting cells
b) Suppression of cyclooxygenase II expression which results in reducing amount of an enzyme available to produce
prostoglandins
c) Activation of phospholipase A2 and reducing prostaglandin and leukotriene synthesis.
d) All of the above
019. Which of the following statements concerning the anti-inflammatory effect of NSAIDs are TRUE?
a) Anti-inflammatory effect of NSAIDs results from inhibition of cyclooxygenase
b) Anti-inflammatory effect of NSAIDs results from inhibition of phospholipase A2 and reducing prostaglandin and
leukotriene synthesis
c) Anti-inflammatory effect of NSAIDs results from induction of cyclooxygenase II expression which results in reducing
the amount of an enzyme available to produce prostoglandins
d) All of the above
020. Indication of glucocorticoids is:
a) Chronic (Addison’s disease) and acute adrenocortical insufficiency
b) Organ transplants (prevention and treatment of rejection – immunosuppression)
c) Inflammatory conditions of bones and joints (arthritis, bursitis, tenosynovitis).
d) All of the above
021. Indications of glucocorticoids are following, EXCEPT:
a) Gastrointestinal diseases (inflammatory bowel disease)
b) Postmenopausal hormonal therapy
c) Inflammatory conditions of bones and joints (arthritis, bursitis, tenosynovitis)
d) Skin diseases (atopic dermatitis, dermatoses, localized neurodermatitis)
97
022. Serious side effects of glucocorticoids include the following, EXCEPT:
a) Acute peptic ulcers
b) Iatrogenic Cushing’s syndrome (rounding, puffiness, fat deposition and plethora alter the appearance of the face –
moon faces)
c) Salicylism (vomiting, tinnitus, decreased hearing, and vertigo)
d) Hypomania or acute psychosis
023. Serious side effects of glucocorticoids include the following:
a) Adrenal suppression
b) Insomnia, behavioral changes (primarily hypomania)
c) Rounding, puffiness, fat deposition and plethora alter the appearance of the face – moon faces
d) All of the above
024. Selective COX-2 inhibitors are safer than nonselective COX-1 inhibitors but without loss of efficacy. This consideration is:
a) True
b) False
025. The constitutive COX-2 isoform tends to be homeostatic in function, while COX-1 is induced during inflammation and
tends to facilitate the inflammatory response. The consideration is:
a) True
b) False
026. Which of the following property combinations is peculiar to the majority of NSAIDs?
a) Antihistaminic, antipyretic, analgesic
b) Immunodepressive, anti-inflammatory, analgesic
c) Antipyretic, analgesic, anti-inflammatory
d) Anti-inflammatory, immunodepressive, antihistaminic
027. Which of the following NSAIDs is a propionic acid derivative?
a) Ibuprofen
b) Indomethacin
c) Metamizole (Analgin)
d) Diclofenac
028. Which of the following NSAIDs is an indol derivative?
a) Ibuprofen
b) Indomethacin
c) Meclofenamic acid
d) Diclofenac
029. Which of the following NSAIDs is a pyrazolone derivative?
a) Ibuprofen
b) Indomethacin
c) Metamizole (Analgin)
d) Diclofenac
030. Which of the following NSAIDs is a fenamate derivative?
a) Phenylbutazone
b) Indomethacin
c) Meclofenamic acid
d) Diclofenac
031. Which of the following NSAIDs is an oxicam derivative?
a) Piroxicam
b) Indomethacin
c) Meclofenamic acid
d) Diclofenac
032. Which of the following NSAIDs is a selective COX-2 inhibitor?
a) Piroxicam
b) Indomethacin
c) Celecoxib
d) Diclofenac
033. Which of the following NSAIDs is a nonselective COX inhibitor
a) Piroxicam
b) Rofecoxib
c) Celecoxib
d) All of the above
034. The following statements concerning aspirin are true, EXCEPT:
a) In contrast to most other NSAIDs, aspirin irreversibly inhibits COX
b) Aspirin interferes with the chemical mediators of the kallikrein system
98
c) Aspirin inhibits phospholipase A2
d) Aspirin inhibits tromboxane A2 formation
035. Indication for aspirin administration are the following, EXCEPT:
a) Inflammatory conditions
b) Decreasing the incidence of transient ischemic attack, unstable angina, coronary artery thrombosis with myocardial
infarction, and thrombosis after coronary artery bypass grafting
c) Relieving severe visceral pain, e.g. myocardial infarction, cancer pain condition, renal or biliary colic
d) Reducing elevated body temperature
036. Side effects of aspirin include following:
a) Gastric upset (intolerance)
b) Salicylism (vomiting, tinnitus, decreased hearing, and vertigo)
c) Gastric ulcers and upper gastrointestinal bleeding
d) All of the above
037. Serious side effects of metamizole (analgin) include the following:
a) Agranulocytosis, aplastic anemia
b) Salicylism (vomiting, tinnitus, decreased hearing, and vertigo)
c) Iatrogenic Cushing’s syndrome (rounding, puffiness, fat deposition and plethora alter the appearance of the face –
moon faces)
d) All of the above
038. Side effects of indometacin include the following:
a) Abdominal pain, diarrhea, gastrointestinal hemorrhage and pancreatitis
b) Dizziness, confusion and depression
c) Trombocytopenia
d) All of the above
039. Ketoprofen is a propionic acid derivative that inhibits both cyclooxygenase (nonselectively) and lipoxygenase. This
statement is:
a) True
b) False
040. Ketorolac is an NSAID that is promoted for systemic use as an anti-inflammatory, not as an analgesic drug. This
statement is:
a) True
b) False
041. Which of the following drugs is a 5-lipoxygenase (5-LOG) inhibitor?
a) Ibuprofen
b) Zileuton (Zyflo)
c) Metamizole (Analgin)
d) Diclofenac
042. Which of the following drugs is a leucotreine D4 receptor (LTD4) blocker?
a) Ibuprofen
b) Zileuton (Zyflo)
c) Zafirleukast (Accolate)
d) Diclofenac
043. Which of the following drugs is a thromboxane A2 receptor (TXA2) antagonist?
a) Sulotroban
b) Zileuton (Zyflo)
c) Zafirleukast (Accolate)
d) Diclofenac

PART V Immunotropic & Antiallergic Agents


001. Immune system is the integrated body system of organs, tissues, cells, and cell products that differentiates self from
nonself and neutralizes potentially pathogenic organisms or substances. This consideration is:
a) True
b) False
002. Antigen is any of various substances, including toxins, bacteria, and the cells of transplanted organs, that when
introduced into the body stimulate the production of antibodies. It is also called an allergen or immunogen. This
consideration is:
a) True
b) False
99
003. Antibody is a protein substance produced in the blood or tissues in response to a specific antigen, such as a bacterium
or a toxin that destroys or weakens bacteria and neutralizes organic poisons, thus forming the basis of immunity. This
consideration is:
a) True
b) False
004. Innate immunity refers to:
a) Antigen-nonspecific defense mechanisms that a host uses immediately or within several hours after exposure
to an antigen. This is the immunity one is born with and is the initial response by the body to eliminate
microbes and prevent infection
b) Antigen-specific defense mechanisms that take several days to become protective and are designed to react with and
remove a specific antigen. This is the immunity one develops throughout life
005. Adaptive (acquired) immunity refers to:
a) Antigen-nonspecific defense mechanisms that a host uses immediately or within several hours after exposure to an
antigen. This is the immunity one is born with and is the initial response by the body to eliminate microbes and prevent
infection
b) Antigen-specific defense mechanisms that take several days to become protective and are designed to react
with and remove a specific antigen. This is the immunity one develops throughout life
006. Allergic reaction is:
a) A local or generalized reaction of an organism to internal or external contact with a specific allergen to which
the organism has been previously sensitized
b) A localized protective reaction of tissue to irritation, injury, or infection, characterized by pain, redness, swelling, and
sometimes a loss of function
007. Immediate allergy reaction (type I allergic reaction) is:
a) An allergic or immune response that begins within a period lasting from a few minutes to about an hour after
exposure to an antigen to which the individual has been sensitized
b) An allergic reaction that becomes apparent only hours after contact
c) An allergic reaction that results from the formation of antigen-antibody complexes between a foreign antigen and IgM
or IgG immunoglobulins. (It occurs during blood transfusion reactions and in hemolytic disease of the newborn)
d) An allergic reaction that is due to the presence of elevated levels of antigen-antibody complexes that cause tissue
damage
008. Delayed allergy reaction (type IV allergic reaction) is:
a) An allergic or immune response that begins within a period lasting from a few minutes to about an hour after exposure
to an antigen to which the individual has been sensitized
b) An allergic reaction that becomes apparent only hours after contact
c) An allergic reaction that results from the formation of antigen-antibody complexes between a foreign antigen and IgM
or IgG immunoglobulins. (It occurs during blood transfusion reactions and in hemolytic disease of the newborn)
d) An allergic reaction that is due to the presence of elevated levels of antigen-antibody complexes that cause tissue
damage
009. Immunodeficiency:
a) A localized protective reaction of tissue to irritation, injury, or infection, characterized by pain, redness, swelling, and
sometimes a loss of function
b) A disorder or deficiency of the normal immune response
c) A disease resulting from an immune reaction produced by an individual's white blood cells or antibodies acting on the
body's own tissues or extracellular proteins
d) All of the above
010. Anaphylactic shock is a severe, sometimes fatal allergic reaction characterized by a sharp drop in blood pressure,
urticaria, and breathing difficulties that is caused by exposure to a foreign substance, such as a drug or bee venom,
after preliminary or sensitizing exposure. This consideration is:
a) True
b) False
011. H1 histamine receptor subtype is distributed in:
a) Smooth muscle, endothelium and brain
b) Gastric mucosa, cardiac muscle, mast cells and brain
c) Presynaptically in brain, mesenteric plexus and other neurons
d) All of the above
012. H2 histamine receptor subtype is distributed in:
a) Smooth muscle, endothelium and brain
b) Gastric mucosa, cardiac muscle, mast cells and brain
c) Presynaptically in brain, mesenteric plexus and other neurons
d) All of the above
013. Most tissue histamine is sequestered and bound in:
100
a) Granules in mast cells or basophils
b) Cell bodies of histaminergic neurons
c) Enterochromaffin-like cell of the fondus of the stomach
d) All of the above
014. These categories of histamine H1 antagonists are noted for sedative effects, EXCEPT:
a) Piperidines; i.e. Loratadine, Fexofenadine
b) Ethanolamines (aminoalkyl ethers); i.e. Dimedrol, Clistin
c) Ethylenediamines; i.e. Suprastine
d) Phenothiazines; i.e. Diprazine, Promethazine
015. Which category of histamine H1 antagonists is noted for the best antiemetic action?
a) Alkylamines (propylamines); i.e. Brompheniramine
b) Ethanolamines (aminoalkyl ethers); i.e. Doxylamine
c) Piperazines; i.e. Hydroxyzine, Cyclizine
d) Ethylenediamines; i.e. Suprastine
016. These categories of histamine H1 antagonists are noted for the anticholinergic effect, EXCEPT:
a) Alkylamines (propylamines); i.e. Brompheniramine
b) Piperazines; i.e. Hydroxyzine, Cyclizine
c) Ethylenediamines; i.e. Suprastine
d) Phenothiazines; i.e. Diprazine, Promethazine
017. Which category of histamine H1 antagonists is noted for the alpha-adrenoreceptor-blocking effect?
a) Alkylamines (propylamines); i.e. Brompheniramine
b) Ethanolamines (aminoalkyl ethers); i.e. Doxylamine, Dimedrol
c) Ethylenediamines; i.e. Suprastine
d) Phenothiazines; i.e. Diprazine, Promethazine
018. Which category of histamine H1 antagonists is noted for the highest local anesthetic effect?
a) Alkylamines (propylamines); i.e. Brompheniramine
b) Piperidines; i.e. Loratadine, Fexofenadine
c) Ethylenediamines; i.e. Suprastine
d) Phenothiazines; i.e. Promethazine
019. Which category of histamine H1 antagonists is recognized for as second-generation antihistamines?
a) Alkylamines (propylamines); i.e. Brompheniramine
b) Piperidines; i.e. Loratadine, Fexofenadine
c) Ethylenediamines; i.e. Suprastine
d) Phenothiazines; i.e. Promethazine
020. These histamine H1 antagonists are recognized for as second-generation antihistamines, EXCEPT:
a) Astemizole
b) Loratadine (Claritin)
c) Cetirizine (Zyrtec)
d) Suprastine
021. Which of histamine H1 antagonists is noted for the serotonin-blocking effect?
a) Brompheniramine
b) Cyproheptadine
c) Suprastine
d) Dimedrol
022. Which of the following histamine H1 antagonists is a long-acting (up to 24-48 h) antihistamine drug?
a) Diazoline
b) Diprazine
c) Suprastine
d) Dimedrol
023. Which of histamine H1 antagonists is noted for the ulcerogenic effect?
a) Diazoline
b) Loratadine
c) Suprastine
d) Dimedrol
024. Indication for administration of histamine H1 antagonists is:
a) Prevention or treatment of the symptoms of allergic reactions (rhinitis, urticaria)
b) Motion sickness and vestibular disturbances
c) Nausea and vomiting in pregnancy (“morning sickness”)
d) All of the above
025. Indications for administration of histamine H1 antagonists are the following EXCEPT:
a) Prevention or treatment of the symptoms of allergic reactions (rhinitis, urticaria)
101
b) Management of seizure states
c) Nausea and vomiting in pregnancy (“morning sickness”)
d) Treatment of sleep disorders
026. Side effect of first-generation histamine H1 antagonists is:
a) Aplastic anemia
b) Vomiting, tinnitus, decreased hearing
c) Sedation
d) Gastric ulcers and upper gastrointestinal bleeding
027. For those first-generation histamine H1 antagonists that cause significant sedation, addition of other drugs that cause
central nervous system depression produces additive effects and is contraindicated while driving or operating
machinery. This consideration is:
a) True
b) False
028. Immunosuppressive drug is any of a variety of substances used to prevent production of antibodies, protein produced by
the immune system in response to the presence in the body of antigens: foreign proteins or polysaccharides such as
bacteria, bacterial toxins, viruses, or other cells or proteins. This consideration is:
a) True
b) False
029. Immunosupressive effect of glucocorticoids is caused by
a) Reducing concentration of lymphocytes (T and B cells) and inhibiting function of tissue macrophages and
other antigen-presenting cells
b) Suppression of cyclooxygenase II expression that results in reducing amount of an enzyme available to produce
prostoglandins
c) Activation of phospholipase A2 and reducing prostaglandin and leukotriene synthesis
d) All of the above
030. Antiallergic effect of glucocorticoids is caused by:
a) Suppression of leukocyte migration and stabilizing lysosomal membranes
b) Reverse the capillary permeability associated with histamine release
c) Suppression of the immune response by inhibiting antibody synthesis
d) All of the above
031. The Immunosuppressive agent is:
a) Corticosteroids
b) Cyclosporine
c) Tacrolimus (FK 506)
d) All of the above
032. Class of cyclosporine A is:
a) Interferons
b) Immunosuppressive agents
c) Monoclonal antibodies
d) Immunoglobulins
033. Mechanism of action of cyclosporine A is:
a) Complement-mediated cytolysis of T lymphocytes
b) ADCC towards T lymphocytes
c) Inhibits calcineurin
d) Compete for Fc receptors with autoantibodies
034. Side effect of cyclosporine A is:
a) Tremor
b) GI disturbance
c) Hepatotoxicity
d) All of the above
035. Side effect of cyclosporine A is:
a) Tremor
b) Anorexia
c) Chills
d) Myalgia
036. Side effect of cyclosporine A is:
a) Diarrhea
b) Headache
c) GI disturbance
d) Immunosuppression
037. Indication of cyclosporine A is:
102
a) Secondary immunodeficiency
b) Hairy cell leukemia
c) Primary immunodeficiency
d) Idiopathic nephrotic syndrome
038. Half-life of cyclosporine A is:
a) 25-35 minutes
b) 21 days
c) 4 - 16 hours
d) 19 hours
039. Class of I.V. IgG preparation is:
a) Monoclonal antibodies
b) Immunosuppressive agents
c) Interferons
d) Immunoglobulins
040. Mechanism of action of I.V. IgG preparation is:
a) Inhibits CD3 receptor
b) Inhibits calcineurin
c) Complement-mediated cytolysis of T lymphocytes
d) Compete for Fc receptors with autoantibodies
041. Half-life of I.V. IgG preparation is:
a) 25-35 minutes
b) 19 hours
c) 4 - 16 hours
d) 21 days
042. Indication for I.V. IgG preparation administration is:
a) Kaposi's sarcoma
b) Acute rejection of organ transplant
c) Condyloma acuminatum
d) Prophylaxis of certain infections
043. Cytotoxic agents are the following EXCEPT:
a) Azathioprine
b) Cyclosporine
c) Leflunomide
d) Cyclophosphamide
044. Class of sirolimus (rapamycin) is:
a) Immunoglobulins
b) Interferons
c) Immunosuppressive agents
d) Monoclonal antibodies
045. Mechanism of action of sirolimus (rapamycin) is:
a) Anti-idiotype antibodies against autoantibodies
b) Modulation of CD3 receptor from the cell surface
c) Inhibits calcineurin
d) ADCC towards T lymphocytes
046. Monoclonal antibodies is:
a) Trastuzumab
b) Rituximab
c) OKT-3
d) All of the above
047. Class of OKT-3 is:
a) Immunosuppressive agents
b) Monoclonal antibodies
c) Interferons
d) Immunoglobulins
048. Half-life of OKT-3 is:
a) 18-24 hours
b) 25-35 minutes
c) 4 - 16 hours
d) 21 days
049. The indication for interferon gamma administration is:
a) Idiopathic nephrotic syndrome
103
b) Hepatitis C virus infection
c) Chronic granulomatous disease
d) Hairy cell leukemia
050. The side effect of interferon gamma is:
a) Hypertension
b) Pulmonary edema
c) Nephrotoxicity
d) Fatigue
051. Half-life of interferon gamma is:
a) 21 days
b) 19 hours
c) 4 - 16 hours
d) 25-35 minutes
052. Half-life of interferon alpha is:
a) 18-24 hours
b) 4-16 hours
c) 25-35 minutes
d) 21 days
053. The indication for interferon alpha administration is:
a) Hepatitis C virus infection
b) Kaposi's sarcoma
c) Condyloma acuminatum
d) All of the above
054. Indication for interferon alpha administration is:
a) Autoimmune diseases
b) Rheumatoid arthritis
c) Organ transplantation
d) Hepatitis C virus infection
055. Indication for interferon alpha administration is:
a) Prophylaxis of sensitization by Rh antigen
b) Rheumatoid arthritis
c) Kaposi's sarcoma
d) Chronic granulomatous disease
056. Class of tacrolimus (FK-506) is:
a) Immunoglobulins
b) Immunosuppressive agents
c) Interferons
d) Monoclonal antibodies
057. Mechanism of action of tacrolimus (FK-506) is:
a) Inhibits CD3 receptor
b) Complement-mediated cytolysis of T lymphocytes
c) Substitution for patient's defiecient immunoglobulins
d) Inhibits calcineurin
058. Immunomodulating agent is:
a) Sirolimus (rapamycin)
b) Levamisole
c) Tacrolimus (FK 506)
d) All of the above
059. Immunomodulating agents are the following EXEPT:
a) Cytokines
b) Levamosole
c) BCG (Bacille Calmette-Guérin)
d) Tacrolimus (FK-506)
060. Mechanism of action of levamisole is:
a) Inhibits CD3 receptor
b) Complement-mediated cytolysis of T lymphocytes
c) Substitution for patient's defiecient immunoglobulins
d) Increase the number of T-cells
104
PART VI Vitamins, Vitamin-like Compounds, Antivitamins, Enzymes & Antienzymes
001. Vitamins are:
a) Inorganic nutrients needed in small quantities in the body
b) Organic substances needed in very large quantities in the body
c) Any of various fat-soluble or water-soluble organic substances essential in minute amounts for normal
growth and activity of the body and obtained naturally from plant and animal foods
d) Products of endocrine gland secretion
002. Vitamin-like compounds are:
a) A number of compounds, whose nutritional requirements exist at specific periods of development,
particularly neonatal development, and periods of rapid growth
b) Inorganic nutrients needed in small quantities in body
c) Organic substances needed in very large quantities in body
d) Products of endocrine gland secretion
003. Antivitamins are:
a) Any of various fat-soluble or water-soluble organic substances essential in minute amounts for normal growth and
activity of the body and obtained naturally from plant and animal foods
b) Substances that prevent vitamins from exerting their typical metabolic effects
c) Any of numerous proteins or conjugated proteins produced by living organisms and functioning as specialized
catalysts for biochemical reactions
d) Nonprotein organic substances that usually contain a vitamin or mineral and combine with a specific apoenzyme to
form an active enzyme system
004. Coenzymes are:
a) Any of various fat-soluble or water-soluble organic substances essential in minute amounts for normal growth and
activity of the body and obtained naturally from plant and animal foods
b) Substances that prevent vitamins from exerting their typical metabolic effects
c) Any of numerous proteins or conjugated proteins produced by living organisms and functioning as specialized
catalysts for biochemical reactions
d) Nonprotein organic substances that usually contain a vitamin or mineral and combines with a specific
apoenzyme to form an active enzyme system
005. Antienzymes are:
a) Agents, especially an inhibitory enzymes or an antibodies to enzymes, that retard, inhibit, or destroy enzymic
activity
b) Substances that prevent vitamins from exerting their typical metabolic effects
c) Any of numerous proteins or conjugated proteins produced by living organisms and functioning as specialized
catalysts for biochemical reactions
d) Nonprotein organic substances that usually contain a vitamin or mineral and combines with a specific apoenzyme to
form an active enzyme system
006. Hypovitaminosis is an insufficiency of one or more essential vitamins. This consideration is:
a) True
b) False
007. Hypervitaminosis is any of various abnormal conditions in which the physiological effect of a vitamin is produced to a
pathological degree by excessive intake of the vitamin. This consideration is:
a) True
b) False
008. Select a fat-soluble vitamin:
a) Ascorbic acid
b) Tocopherol
c) Thiamine
d) Riboflavin
009. Select a water-soluble vitamin:
a) Vitamin A
b) Vitamin E
c) Vitamin D
d) Vitamin B1
010. Which of the following vitamins can be also synthesized from a dietary precursor?
a) Vitamin С
b) Vitamin A
c) Vitamin B1
d) Vitamin B6
011. Which of the following vitamins resembles with hormone
a) Vitamin К
105
b) Vitamin A
c) Vitamin D
d) Vitamin E
012. Beri-beri is caused by the deficiency of:
a) Riboflavin
b) Ascorbic acid
c) Nicotinic acid
d) Thiamine
013. Beri-beri is
a) Disease caused by a deficiency of thiamine, endemic in eastern and southern Asia, and characterized by
neurological symptoms, cardiovascular abnormalities, and edema. It is also called endemic neuritis
b) Inflammation at the corners of the mouth caused by a deficiency of riboflavin, associated with a wrinkled or fissured
epithelium that does not involve the mucosa
c) A disorder of the lips often due to riboflavin deficiency and other B-complex vitamin deficiencies and characterized by
fissures, especially in the corners of the mouth
d) All of the above
014. Deficiency symptom of riboflavin is:
a) Cheilitis – inflammation of the lips or of a lip, with redness and the production of fissures radiating from the angles of
the mouth
b) Cheilosis – a disorder of the lips characterized by fissures, especially in the corners of the mouth
c) Angular stomatitis, associated with a wrinkled or fissured epithelium that does not involve the mucosa
d) All of the above
015. All of the following statements concerning vitamin A functions are true EXCEPT:
a) Transmission of light stimuli to the brain, via combination with a specific protein, opsin, to form a visual pigment,
rhodopsin, in the retina of the eye
b) Regulation of cell growth and differentiation in epithelium, connective tissues (including bone and cartilage) and
hematopoietic tissues by retinoic acid, a highly bioactive metabolite of retinol
c) Retinoic acid is especially important during embryogenesis
d) Acts as a hormone involved in regulation of calcium and phosphorus homeostasis
016. Deficiency symptom of vitamin A is:
a) Night blindness – lessened ability to see in dim light
b) Xerophthalmia and keratomalacia
c) Various epithelial tissue defects, leading to decreased resistance to infective diseases, male and female infertility
d) All of the above
017. Xerophthalmia is:
a) Extreme dryness of the conjunctiva resulting from a disease localized in the eye or from systemic deficiency
of vitamin A
b) A condition, usually in children with vitamin A deficiency, characterized by softening and subsequent ulceration and
perforation of the cornea
c) A condition of the eyes in which vision is normal in daylight or other strong light but is abnormally weak or completely
lost at night or in dim light and that results from vitamin A deficiency
d) All of the above
018. Keratomalacia is:
a) Extreme dryness of the conjunctiva resulting from a disease localized in the eye or from systemic deficiency of vitamin
A
b) A condition, usually in children with vitamin A deficiency, characterized by softening and subsequent
ulceration and perforation of the cornea
c) A visual defect marked by the inability to see as clearly in bright light as in dim light
d) All of the above
019. Night blindness (Hemeralopia, Nyctalopia) is
a) Extreme dryness of the conjunctiva resulting from a disease localized in the eye or from systemic deficiency of vitamin
A
b) A condition, usually in children with vitamin A deficiency, characterized by softening and subsequent ulceration and
perforation of the cornea
c) A condition of the eyes in which vision is normal in daylight or other strong light but is abnormally weak or
completely lost at night or in dim light and that results from vitamin A deficiency
d) All of the above
020. All of the following statements concerning vitamin E functions are true, EXCEPT:
a) An extremely important antioxidant, which protects cell membrane lipids from peroxidation by breaking the chain
reaction of free radical formation to which polyunsaturated fatty acids are particularly vulnerable
b) Antisterility and antiabortion factor
c) Specifically required for synthesis of prothrombin and several other clotting factors
106
d) An essential for oxidative processes regulation
021. Which of the following statements concerning vitamin B1 functions are true:
a) An extremely important antioxidant, which protects cell membrane lipids from peroxidation by breaking the chain
reaction of free radical formation to which polyunsaturated fatty acids are particularly vulnerable
b) An essential coenzyme for oxidative decarboxylate of alpha-keto acids, most important being conversion of
pyruvate to acetyl coenzyme A
c) Specifically required for synthesis of prothrombin and several other clotting factors
d) Essential constituent of the flavoproteins, flavin mononucleotide (FMN) and flavin adenine dinucleotide (FAD)
022. All of the following statements concerning vitamin B2 functions are true EXCEPT:
a) Essential constituent of flavoproteins, flavin mononucleotide (FMN) and flavin adenine dinucleotide (FAD)
b) Plays key roles in hydrogen transfer reactions associated with glycolysis, TCA cycle and oxidative phosphorylation
c) An essential coenzyme for oxidative decarboxylate of alpha-keto acids, most important being conversion of
pyruvate to acetyl coenzyme A
d) Deficiency symptoms are cheilitis, cheilosis and angular stomatitis
023. Which of the following statements concerning vitamin PP (B3, niacin) functions are true:
a) Active group of the coenzymes nicotinamide-adenine dinucleotide (NAD) and nicotinamide-adenine
phosphate (NADP)
b) An essential coenzyme for oxidative decarboxylate of alpha-keto acids, most important being conversion of pyruvate to
acetyl coenzyme A
c) Specifically required for synthesis of prothrombin and several other clotting factors
d) Essential constituent of flavoproteins, flavin mononucleotide (FMN) and flavin adenine dinucleotide (FAD)
024. Which of the following statements concerning pyridoxine (vitamin B6) functions are true:
a) Active functional form is pyridoxal phosphate, which is an essential coenzyme for transamination and
decarboxylation of amino acids in more than 50 different enzyme systems
b) Active group of the coenzymes nicotinamide-adenine dinucleotide (NAD) and nicotinamide-adenine phosphate
(NADP)
c) Essential constituent of flavoproteins, flavin mononucleotide (FMN) and flavin adenine dinucleotide (FAD)
d) An extremely important antioxidant, which protects cell membrane lipids from peroxidation by breaking the chain
reaction of free radical formation to which polyunsaturated fatty acids are particularly vulnerable
025. Which of the following statements concerning pantothinic acid functions are true:
a) Active functional form is pyridoxal phosphate, which is an essential coenzyme for transamination and decarboxylation
of amino acids in more than 50 different enzyme systems
b) Essential constituent of coenzyme A, the important coenzyme for acyl transfer in the TCA cycle and de novo
fatty acid synthesis
c) An extremely important antioxidant, which protects cell membrane lipids from peroxidation by breaking the chain
reaction of free radical formation to which polyunsaturated fatty acids are particularly vulnerable
d) Coenzyme for several reactions involving CO2 fixation into various compounds e.g. acetyl CoA to malonyl CoA (acetyl
CoA carboxylase) – initial step in de novo fatty acid synthesis; propionyl CoA to methylmalonyl CoA (propionyl CoA
carboxylase), pyruvate to oxaloacetate (pyruvate carboxylase)
026. Which of the following statements concerning biotin functions are true:
a) Active functional form is pyridoxal phosphate, which is an essential coenzyme for transamination and decarboxylation
of amino acids in more than 50 different enzyme systems
b) Essential constituent of coenzyme A, the important coenzyme for acyl transfer in the TCA cycle and de novo fatty acid
synthesis
c) An extremely important antioxidant, which protects cell membrane lipids from peroxidation by breaking the chain
reaction of free radical formation to which polyunsaturated fatty acids are particularly vulnerable
d) Coenzyme for several reactions involving CO2 fixation into various compounds e.g. acetyl CoA to malonyl
CoA (acetyl CoA carboxylase) – initial step in de novo fatty acid synthesis; propionyl CoA to methylmalonyl
CoA (propionyl CoA carboxylase), pyruvate to oxaloacetate (pyruvate carboxylase)
027. Which of the following statements concerning vitamin B12 (cyanocobalamin) functions are true:
a) Active functional form is pyridoxal phosphate, which is an essential coenzyme for transamination and decarboxylation
of amino acids in more than 50 different enzyme systems
b) Essential constituent of coenzyme A, the important coenzyme for acyl transfer in the TCA cycle and de novo fatty acid
synthesis
c) Coenzyme for numerous metabolic reaction, including transformation of methylamlonyl CoA to succinyl CoA
in the metabolism of propionate; DNA synthesis (acts in concert with folic acid); transmethylation e.g.
methionine synthesis from homocysteine
d) An extremely important antioxidant, which protects cell membrane lipids from peroxidation by breaking the chain
reaction of free radical formation to which polyunsaturated fatty acids are particularly vulnerable
028. Which of the following statements concerning folic acid (folacin) functions are true:
a) Active functional form is pyridoxal phosphate, which is an essential coenzyme for transamination and decarboxylation
of amino acids in more than 50 different enzyme systems
107
b) Essential constituent of coenzyme A, the important coenzyme for acyl transfer in the TCA cycle and de novo fatty acid
synthesis
c) Carrier of one-carbon (e.g. methyl) groups that are added to, or removed from, metabolites such as histidine,
serine, methionine, and purines
d) An extremely important antioxidant, which protects cell membrane lipids from peroxidation by breaking the chain
reaction of free radical formation to which polyunsaturated fatty acids are particularly vulnerable
029. Which of the following statements concerning vitamin C functions are true:
a) Active functional form is pyridoxal phosphate, which is an essential coenzyme for transamination and decarboxylation
of amino acids in more than 50 different enzyme systems
b) Essential constituent of coenzyme A, the important coenzyme for acyl transfer in the TCA cycle and de novo fatty acid
synthesis
c) Carrier of one-carbon (e.g. methyl) groups that are added to, or removed from, metabolites such as histidine, serine,
methionine, and purines
d) Has antioxidant properties and is required for various hydroxylation reactions e.g. proline to hydroxypoline
for collagen synthesis
030. Dermatitis, diarrhoea and dementia are characteristics of:
a) Dry beriberi
b) Pyridoxine deficiency
c) Scurvy
d) Pellagra
031. Pellagra is:
a) A disease caused by a deficiency of niacin in the diet and characterized by skin eruptions, digestive and
nervous system disturbances, and eventual mental deterioration
b) Inflammation of several nerves at one time caused by a deficiency of thiamin, marked by paralysis, pain, and muscle
wasting. Also called multiple neuritis or polyneuritis
c) A severe form of anemia most often affecting elderly adults, caused by a failure of the stomach to absorb vitamin B12
and characterized by abnormally large red blood cells, gastrointestinal disturbances, and lesions of the spinal cord.
Also called pernicious anemia, malignant anemia
d) All of the above
032. Pernicious anemia is:
a) A severe form of anemia most often affecting elderly adults, caused by a failure of the stomach to absorb
vitamin B12 and characterized by abnormally large red blood cells, gastrointestinal disturbances, and lesions
of the spinal cord
b) A form of anemia in which the capacity of the bone marrow to generate red blood cells is defective, caused by a bone
marrow disease or exposure to toxic agents, such as radiation, chemicals, or drugs
c) Anemia characterized by a decrease in the concentration of corpuscular hemoglobin
d) All of the above
033. Rickets is:
a) A deficiency disease resulting from a lack of vitamin D or calcium and from insufficient exposure to sunlight,
characterized by defective bone growth and occurring chiefly in children
b) A disease occurring primarily in adults that results from a deficiency in vitamin D or calcium and is characterized by a
softening of the bones with accompanying pain and weakness
c) A disease characterized by a decrease in bone mass and density, occurring especially in postmenopausal women,
resulting in a predisposition to fractures and bone deformities such as a vertebral collapse
d) All of the above
034. Scurvy is:
a) A disease caused by deficiency of vitamin C and characterized by spongy bleeding gums, bleeding under the
skin, and weakness
b) Extreme dryness of the conjunctiva resulting from a disease localized in the eye or from systemic deficiency of vitamin
A
c) A disease caused by deficiency of niacin in the diet and characterized by skin eruptions, digestive and nervous system
disturbances, and eventual mental deterioration
d) All of the above
035. Which of the following vitamins is given along with isoniazide in treatment of tuberculosis?
a) Nicotinic acid
b) Riboflavin
c) Pyridoxine
d) Ascorbic acid
036. Which of the following vitamins is also known as an antisterility factor?
a) Vitamin E
b) Vitamin B6
c) Vitamin B1
108
d) Vitamin К
037. Mega doses of which vitamin are some time beneficial viral respiratory infections
a) Vitamin С
b) Vitamin A
c) Vitamin К
d) Vitamin PP
038. Which of the following vitamins improves megaloblast anemia but does not protect the neurological manifestations of
pernicious anemia?
a) Vitamin B12
b) Vitamin BC
c) Vitamin PP
d) Vitamin D
039. Vitamin К enhances the anticoagulant property of coumarins. This statement is:
a) True
b) False
040. Loosening of teeth, gingivitis and hemorrhage occur in the deficiency of:
a) Vitamin К
b) Vitamin В1
c) Vitamin B6
d) Vitamin C
041. Ingestion of polar bear liver may cause acute poisoning of:
a) Vitamin D
b) Vitamin E
c) Vitamin A
d) Vitamin C
042. Which of the following antivitamins prevent a vitamin B6 from exerting its typical metabolic effects?
a) Isoniazide
b) Ethanol
c) Carbamazepine
d) All of the above
043. Which of the following antivitamins prevent a vitamin A from exerting its typical metabolic effects?
a) Lipooxidase
b) Oral contraceptives
c) Antibiotics
d) All of the above
044. Which of the following antivitamins prevent a vitamin K from exerting its typical metabolic effects?
a) Cholestiramine
b) Coumarins
c) Antibiotics
d) All of the above
045. Which of the following coenzymes is of vitamin origin?
a) Riboxine
b) Coenzyme Q10
c) Piridixal-5-phosphate
d) Lipoic acid
046. Which of the following coenzymes is not of vitamin origin?
a) Coenzyme Q10
b) Magnesium
c) Carnitine
d) All of the above
047. These substances are vitamin-like compounds, EXСEPT:
a) Choline
b) Vitamin PP
c) Vitamin U (methylmethioninesulfonil chloride)
d) Orotate acid
048. Which of the following substances is a vitamin-like compound?
a) Ascorbic acid
b) Taurine
c) Thiamine
d) Riboflavin
049. Which of the following antienzymes is a proteolysis inhibitor?
109
a) Contrical
b) Sulbactam
c) Aminocaproic acid
d) Disulfiram
050. Which of the following antienzymes is a beta-lactamase inhibitor?
a) Clavulanic acid
b) Sulbactam
c) Tazobactam
d) All of the above
051. Which of the following antienzymes is a fibrinolysis inhibitor?
a) Clavulanic acid
b) Sulbactam
c) Aminocaproic acid
d) Disulfiram
052. Which of the following antienzymes is an aldehyde dehydrogenase inhibitor?
a) Tazobactam
b) Sulbactam
c) Aminocaproic acid
d) Disulfiram
053. Which of the following antienzymes is a cholinesterase inhibitor?
a) Physostigmine
b) Selegiline
c) Aminocaproic acid
d) Disulfiram
054. Which of the following antienzymes is a monoamine oxidase (MAO) inhibitor:
a) Physostigmine
b) Selegiline
c) Acetazolamide
d) Disulfiram
055. Which of the following antienzymes is a carbonic anhydrase inhibitor:
a) Physostigmine
b) Selegiline
c) Aminocaproic acid
d) Acetazolamide
056. Which of the following antienzymes is a xantine oxidase inhibitor?
a) Physostigmine
b) Allopurinol
c) Aminocaproic acid
d) Acetazolamide
057. Which of the following antienzymes is an aromatase inhibitor used in cancer therapy?
a) Physostigmine
b) Allopurinol
c) Aminocaproic acid
d) Aminoglutethimide
058. Which of the following enzymes improves GIT functions (replacement therapy):
a) Pepsin
b) Urokinase
c) L-asparaginase
d) Lydaze
059. Which of the following enzymes has fibrinolytic activity?
a) Pepsin
b) Urokinase
c) L-asparaginase
d) Lydaze
060. Which of the following enzymes is used in cancer therapy?
a) Pepsin
b) Urokinase
c) L-asparaginase
d) Lydaze
061. Which of the following statements concerning nutritional supplement (dietary supplement) are True:
110
a) Nutritional supplements are intended to supplement the diet and bear or contain one or more of the following dietary
ingredients: a vitamin, a mineral, an herb or other botanical, an amino acid, a dietary substance for human use to
supplement the diet by increasing the total daily intake (e.g., enzymes or tissue from organ or glands), a concentrate,
such as a meal replacement or energy bar, a metabolite, constituent, or extract
b) Nutritional supplements are regulated as foods, and not as drugs
c) Nutritional supplements are not pre-approved on their safety and efficacy, unlike drugs
d) All of the above

PART VII Antihyperlipidemic Drugs & Drugs Used In the Treatment of Gout
001. Lipoprotein is:
a) A conjugated protein having a lipid component; the principal means for transporting lipids in the blood
b) Any of various fat-soluble or water-soluble organic substances essential in minute amounts for normal growth and
activity of the body and obtained naturally from plant and animal foods
c) Product of endocrine gland secretion
d) Mediators of inflammatory process
002. Very low density lipoprotein (VLDL) is:
a) A lipoprotein containing a very large proportion of lipids to protein and carrying most cholesterol from the
liver to the tissues
b) A lipoprotein that contains relatively high amounts of cholesterol and is associated with an increased risk of
atherosclerosis and coronary artery disease. It is also called beta-lipoprotein
c) A lipoprotein that contains relatively small amounts of cholesterol and triglycerides and is associated with a decreased
risk of atherosclerosis and coronary artery disease. It is also called alpha-lipoprotein
d) Large lipoprotein particle that is created by the absorptive cells of the small intestine. It transports lipids to adipose
tissue where they are broken down by lipoprotein lipase
003. Low-density lipoprotein (LDL) is:
a) A lipoprotein that contains relatively high amounts of cholesterol and is associated with an increased risk of
atherosclerosis and coronary artery disease. It is also called beta-lipoprotein
b) A lipoprotein that contains relatively small amounts of cholesterol and triglycerides and is associated with a decreased
risk of atherosclerosis and coronary artery disease. It is also called alpha-lipoprotein
c) A lipoprotein containing a very large proportion of lipids to protein and carrying most cholesterol from the liver to the
tissues
d) Large lipoprotein particle that is created by the absorptive cells of the small intestine. It transports lipids to adipose
tissue where they are broken down by lipoprotein lipase
004. High-density lipoprotein (HDL) is:
a) A lipoprotein that contains relatively small amounts of cholesterol and triglycerides and is associated with a
decreased risk of atherosclerosis and coronary artery disease. It is also called alpha-lipoprotein
b) A lipoprotein containing a very large proportion of lipids to protein and carrying most cholesterol from the liver to the
tissues
c) A lipoprotein that contains relatively high amounts of cholesterol and is associated with an increased risk of
atherosclerosis and coronary artery disease. It is also called beta-lipoprotein
d) Large lipoprotein particle that is created by the absorptive cells of the small intestine. It transports lipids to adipose
tissue where they are broken down by lipoprotein lipase
005. Chylomicron is:
a) A lipoprotein that contains relatively small amounts of cholesterol and triglycerides and is associated with a decreased
risk of atherosclerosis and coronary artery disease. It is also called alpha-lipoprotein
b) A lipoprotein containing a very large proportion of lipids to protein and carrying most cholesterol from the liver to the
tissues
c) A lipoprotein that contains relatively high amounts of cholesterol and is associated with an increased risk of
atherosclerosis and coronary artery disease. It is also called beta-lipoprotein
d) Large lipoprotein particle that is created by the absorptive cells of the small intestine. It transports lipids to
adipose tissue where they are broken down by lipoprotein lipase
006. Hyperlipoproteinemia is a condition marked by an abnormally high level of lipoproteins in the blood. This consideration is:
a) True
b) False
007. Hypertriglyceridemia denotes high blood levels of triglycerides. It has been associated with atherosclerosis, even in the
absence of hypercholesterolemia (high cholesterol levels). This consideration is:
a) True
b) False
008. Hypercholesterolemia (or hypercholesteremia) is an abnormally high concentration of cholesterol in the blood. This
consideration is:
a) True
b) False
111
009. Which of the following consideration about type I familial hyperlipoproteinemia is True:
a) Type I familial hyperlipoproteinemia marked by the increased serum concentrations of chylomicrons and
triglycerides, which decrease if the diet becomes fat free, decreased concentrations of high- and low-density
lipoproteins, which increase if the diet is fat free, and decreased tissue lipoprotein lipase activity
b) Type I familial hyperlipoproteinemia characterized by increased serum concentrations of chylomicrons, pre-low-density
lipoproteins, and triglycerides that are considered to be the result of a combination of fat and carbohydrate-induced
hyperlipemia
010. Familial chylomicronemia (type I) is caused by deficiency in lipoprotein lipase activity. This consideration is:
a) True
b) False
011. The Coronary Primary Prevention Trial (CPPT) demonstrated that treatment with a lipid-lowering drug could reduce the
risk of death due to coronary heart disease. This consideration is:
a) True
b) False
012. Women taking probucol (Lorelco) should wait for 6 months after cessation of therapy before becoming pregnant. This
consideration is:
a) True
b) False
013. Nicotinic acid (Niacin) plus a bile acid-binding resin has not proven effective in combating hyperlipidemia. This
consideration is:
a) True
b) False
014. The ideal therapy for patients with elevated levels of cholesterol would lower the serum concentration of LDL-cholesterol
while raising the concentration of HDL-cholesterol. This consideration is:
a) True
b) False
015. Agents, which lower levels of LDL-cholesterol, tend to promote regression of atherosclerotic plaques. This consideration
is:
a) True
b) False
016. Clofibrate (Atromid-S) is the drug of choice for treatment of broad-beta hyperlipidemia (type III). This consideration is:
a) True
b) False
017. One advantage of gemfibrozil (Lopid) is that, in addition to lowering blood levels of most lipids, it raises the level of HDL
cholesterol. This consideration is:
a) True
b) False
018. Probucol (Lorelco) appears to increase clearance of LDL cholesterol by a non-receptor mediated mechanism. This
consideration is:
a) True
b) False
019. All of the following statements concerning cholestyramine (Questran) are true, EXCEPT:
a) It would not be a good choice for treating patients with familial hypertriglyceridemia (type IV)
b) It is not well tolerated by patients
c) It works by directly binding cholesterol in the blood
d) It is an effective drug for treatment of types IIa and IIb hyperlipidemia
020. All of the following statements concerning drugs which inhibit cholesterol synthesis are true, EXCEPT:
a) They work in part by increasing the rate of LDL clearance from the plasma
b) They are the most effective single agents for lowering LDL-cholesterol
c) When used with a bile-acid binding resin, they can lower LDL-cholesterol by 50% or more
d) No special monitoring is required in patients receiving one of them
021. All of the following statements concerning nicotinic acid (Niacin) are true, EXCEPT:
a) It reduces the rate of synthesis of VLDL
b) Sustained-release preparations of this drug are largely free of side effects
c) Almost all patients taking the traditional dosage form of this drug experience uncomfortable flushing
d) It should not be used with antihypertensives
022. All of the following statements concerning drugs which inhibit cholesterol synthesis are true, EXCEPT:
a) When used alone, they are the most effective agents for lowering LDL cholesterol
b) They are often effective in patients in whom a diet, with or without a bile acid-binding resin or niacin, has failed
c) Lovastatin (Mevacor) plus a resin causes regression of coronary lesions in about one third of treated patients
112
d) Members of this drug class are generally not as well tolerated as the older bile acid-binding resins
023. All of the following statements concerning drugs which inhibit cholesterol synthesis are true, EXCEPT:
a) These drugs should not be used in pregnant women or children
b) These drugs often cause myopathy if used in combination with cyclosporine (Sandimmune)
c) Failure to discontinue the drug after myopathy has been detected can cause acute renal failure
d) Several of these drugs tend to lengthen the sleep cycle
024. All of the following statements concerning the fibric acid derivatives are true, EXCEPT:
a) Clofibrate (Atromid-S) is the drug of choice for therapy of Type III hyperlipidemia
b) Gemfibrozil (Lopid) increases HDL cholesterol while lowering LDL cholesterol
c) Gemfibrozil (Lopid) has been shown to reduce mortality associated with a heart disease
d) Gemfibrozil (Lopid) is generally well tolerated
025. All of the following statements concerning the bile acid-binding resins are true, EXCEPT:
a) They decrease total cholesterol and LDL
b) They are contraindicated in patients with hypertriglyceridemia
c) When used alone, they do not slow the progression of atherosclerotic lesions
d) They are the drugs of choice for therapy of type II hyperlipidemia when used either alone or in combination with
selected agents
026. All of the following statements concerning nicotinic acid (Niacin) are true, EXCEPT:
a) Both triglycerides and LDL cholesterol are reduced by this drug
b) The drug acts by directly decreasing the rate of synthesis of apoproteins
c) Doses higher than 3 gm/day are no longer used because of possible disturbances of hepatic or pancreatic functions
d) Most patients taking this drug experience uncomfortable cutaneous flushing, itching, and/or rashes
027. All of the following statements concerning the general principles of therapy with lipid-lowering drugs are true EXCEPT:
a) Therapy with a lipid-lowering drug should be always accompanied by an appropriate diet
b) A lipid-lowering diet should be discontinued if it fails to decrease the levels of plasma LDL cholesterol by at
least 10%
c) Lipid-lowering drugs should only be administered after at least 3 months of prior dietary therapy
d) Some combinations of lipid-lowering drugs are synergistic
028. The cholesterol synthesis inhibitors increase the rate of clearance of LDL cholesterol from the plasma. This
consideration is:
a) True
b) False
029. Lovastatin (Mevacor) plus a bile-acid binding resin causes regression of coronary lesions in about one third of treated
patients. This consideration is:
a) True
b) False
030. The cholesterol synthesis inhibitors are better tolerated than most other lipid-lowering agents. This consideration is:
a) True
b) False
031. Selected liver and muscle enzymes should be monitored during the use of any cholesterol synthesis inhibitors because
of possible toxic effects. This consideration is:
a) True
b) False
032. The bile acid-binding resins act by directly binding cholesterol and facilitating its excretion. This consideration is:
a) True
b) False
033. Nicotinic acid (Niacin) acts by increasing the rate of catabolism of VLDL. This consideration is:
a) True
b) False
034. Gemfibrozil (Lopid) can cause dizziness and syncope when used with antihypertensives. This consideration is:
a) True
b) False
035. Gemfibrozil (Lopid) increases concentrations of HDL cholesterol more than clofibrate (Atromid-S). This consideration is:
a) True
b) False
036. The bile acid-binding resins can bind many drugs and vitamins and reduce their absorption. This consideration is:
a) True
b) False
037. When used alone, the bile acid-binding resins are contraindicated in patients with hypertriglyceridemia. This
consideration is:
113
a) True
b) False
038. Combinations of lipid-lowering drugs are likely to be synergistic if they work at different steps in the same pathway. This
consideration is:
a) True
b) False
039. Reduction in plasma triglycerides and LDL cholesterol concentrations with gemfibrozil treatment is greater than reduction
in plasma cholesterol and LDL cholesterol concentrations with gemfibrozil treatment. This consideration is:
a) True
b) False
040. Patients with homozygous familial hypercholesterolemia (type IIa) lack any functional LDL receptors on their
hepatocytes. This consideration is:
a) True
b) False
041. Effects of drugs in lowering blood cholesterol levels are additive with those of diet. This consideration is:
a) True
b) False
042. HMG-CoA reductase inhibiting drugs can cause muscle breakdown, especially when used in combination with a
cyclosporine. This consideration is:
a) True
b) False
043. Probucol (Lorelco) reduces the risk of atherosclerosis by stimulating the rate of clearance of LDL by receptor-mediated
pathways. This consideration is:
a) True
b) False
044. Clofibrate (Atromid-S) is generally regarded as superior to gemfibrozil.
a) True
b) False
045. Niacin’s most common side effects can be reduced by pretreatment with aspirin and/or by taking the drug at the end of
meals. This consideration is:
a) True
b) False
046. The major side effect of cholestyramine is hepatotoxicity. This consideration is:
a) True
b) False
047. The statins are dependent on the presence of LDL receptors on hepatocytes in order to exert their effect. This
consideration is:
a) True
b) False
048. This drug increases lipoprotein lipase (LPL) activity in adipose tissue:
a) Cholestyramine (Questran)
b) Lovastatin (Mevacor)
c) Nicotinic acid (Niacin)
d) Gemfibrozil (Loprol)
049. This drug both inhibits an enzyme and indirectly enhances clearance of low density lipoproteins (LDL):
a) Cholestyramine (Questran)
b) Lovastatin (Mevacor)
c) Nicotinic acid (niacin)
d) Probucol (Lorelco)
050. This drug binds bile acids in the GI tract:
a) Cholestyramine (Questran)
b) Nicotinic acid (niacin)
c) Gemfibrozil (Loprol)
d) Probucol (Lorelco)
051. This drug may block oxidation of low density lipoproteins (LDL):
a) Lovastatin (Mevacor)
b) Nicotinic acid (niacin)
c) Gemfibrozil (Loprol)
d) Probucol (Lorelco)
052. This drug weakly stimulates synthesis of very low density lipoproteins (VLDL):
114
a) Cholestyramine (Questran)
b) Lovastatin (Mevacor)
c) Gemfibrozil (Loprol)
d) Probucol (Lorelco)
053. Flushing caused by this drug can be reduced by taking it after meals and/or by pretreatment with aspirin:
a) Lovastatin (Mevacor)
b) Nicotinic acid (niacin)
c) Gemfibrozil (Loprol)
d) Probucol (Lorelco)
054. This drug can cause muscle damage, especially when used with any of several drugs including erythromycin:
a) Cholestyramine (Questran)
b) Lovastatin (Mevacor)
c) Gemfibrozil (Loprol)
d) Probucol (Lorelco)
055. This drug decreases blood levels of high density lipoproteins (HDL):
a) Lovastatin (Mevacor)
b) Nicotinic acid (niacin)
c) Gemfibrozil (Loprol)
d) Probucol (Lorelco)
056. This fibric acid derivative increases blood levels of high density lipoproteins (HDL):
a) Cholestyramine (Questran)
b) Lovastatin (Mevacor)
c) Gemfibrozil (Loprol)
d) Probucol (Lorelco)
057. Gout is a familial metabolic disease characterized by recurrent episodes of acute arthritis due to deposits of monosodium
urate in joints and cartilage. This consideration is:
a) True
b) False
058. Probenecid and sulfinpyrazone are uricosuric drugs employed to decrease the body pool of urate in patients with
tophaceous gout or in those with increasingly frequent gouty attacks. This consideration is:
a) True
b) False
059. Which of the following drugs is an uricosuric agent:
a) Allopurinol
b) Sulfinpyrazone
c) Colchicine
d) Indomethacin
060. Uricosuric drugs are the following, EXCEPT:
a) Probenecid
b) Sulfinpyrazone
c) Colchicine
d) Aspirin (at high dosages)
061. Which of the following drugs used in the treatment of gout acts by preventing the migration of granulocytes:
a) Allopurinol
b) Sulfinpyrazone
c) Colchicine
d) Indomethacin
062. Which of the following drugs used in the treatment of gout has as its primary effect the reduction of uric acid synthesis
a) Allopurinol
b) Sulfinpyrazone
c) Colchicine
d) Indomethacin
063. Characteristics of probenecid include all of the following, EXCEPT:
a) It promotes the renal tubular secretion of penicillin
b) It is useful in the treatment of gout
c) At appropriate doses, it promotes the excretion of uric acid
d) The metabolic products of probenecid are uricosuric

PART VIII Agents That Affect Bone Mineral Homeostasis


001. Action of the parathyroid hormone is:
115
a) Increased calcium and phosphate absorption in intestine (by increased 1,25-dihydroxyvitamin D3 production)
b) Decreased calcium excretion and increased phosphate excretion in kidneys
c) In bone, calcium and phosphate resorption increased by high doses. Low doses may increase bone formation.
d) All of the above
002. The parathyroid hormone increases serum calcium and decreases serum phosphate. This consideration is:
a) True
b) False.
003. The following statements about the parathyroid hormone are true, EXCEPT:
a) The parathyroid hormone (PTH) is a single-chain peptide hormone composed of 84 amino acids
b) The parathyroid hormone increases calcium and phosphate absorption in intestine (by increased 1,25-
dihydroxyvitamin D3 production)
c) The parathyroid hormone increases serum calcium and decreases serum phosphate
d) The parathyroid hormone increases calcium excretion and decreases phosphate excretion in kidneys
004. Which of the following statements about calcitonin is true:
a) Calcitonin secreted by parafollicular cells of the mammalian thyroid is a single-chain peptide hormone with 32 amino
acids
b) Effects of calcitonin are to lower serum calcium and phosphate by acting on bones and kidneys.
c) Calcitonin inhibits osteoclastic bone resorption.
d) All of the above
005. Mechanism of action of calcitonin is:
a) Inhibits hydroxyapatite crystal formation, aggregation, and dissolution
b) Raises intracellular cAMP in osteoclasts
c) Activates bone resorption
d) Inhibits macrophages
006. Indications for calcitonin administration are the following, EXCEPT:
a) Hypercalcemia
b) Paget's disease
c) Hypophosphatemia
d) Osteoporosis
007. Side effect of calcitonin is:
a) Hypercalcemia
b) Metastatic calcifications
c) Tetany
d) GI toxicity
008. Side effect of calcitonin is:
a) Pruritus
b) Hypotension
c) Fractures
d) Hypocalcemia
009. Glucocorticoid hormones alter bone mineral homeostasis:
a) By antagonizing vitamin D-stimulated intestinal calcium transport
b) By stimulating renal calcium excretion
c) By increasing parathyroid hormone stimulated bone resorption
d) By all of the above
010. Estrogens can prevent accelerated bone loss during the immediate postmenopausal period and at least transiently
increase bone in the postmenopausal subject. This consideration is:
a) True
b) False
011. Action of vitamin D3 is:
a) Increased calcium and phosphate absorption by 1,25-dihydroxyvitamin D3
b) Calcium and phosphate excretion may be decreased by 25-hydroxyvitamin D3 and 1,25-dihydroxyvitamin D3
c) Increased calcium and phosphate resorption by 1,25-dihydroxyvitamin D3; bone formation may be increased by 25,24-
dihydroxyvitamin D3
d) All of the above
012. Vitamin D3 increases serum calcium and phosphate. This consideration is:
a) True
b) False
013. Route of administration of vitamin D3 is:
a) Subcutaneous
b) Oral
c) Intravenous
116
d) Intranasal
014. Side effect of vitamin D3 is:
a) Defective bone mineralization
b) Metastatic calcifications
c) Hepatic toxicity
d) Nephrolithiasis
015. Indication of vitamin D3 is:
a) Hypercalcemia
b) Paget's disease
c) Hypophosphatemia
d) Osteomalacia
016. 25-hydroxyvitamin D3 (calcifediol) is less effective than 1,25-dihydroxyvitamin D3 (calcitriol) in stimulating intestinal
calcium transport, so that hypercalcemia is less of a problem with calcifediol. This consideration is:
a) True
b) False
017. Route of administration of 25-hydroxyvitamin D3 (calcifediol) is:
a) Oral
b) Subcutaneous
c) Intravenous
d) Intranasal
018. Indication for 25-hydroxyvitamin D3 (calcifediol) administration is:
a) Primary hyperparathyroidism
b) Rickets
c) Hypercalcemia
d) Failure of vitamin D formation in skin
019. Side effect of 25-hydroxyvitamin D3 (calcifediol) is:
a) Hypercalcemia
b) Pruritus
c) GI toxicity
d) All of the above
020. Indications for 1,25-dihydroxyvitamin D3 (calcitriol) administration are the following, EXCEPT:
a) Hypocalcemia in chronic renal failure
b) Vitamin D-dependent rickets
c) Malabsorption of vitamin D from intestine
d) Elevated skeletal turnover
021. Indication for 1,25-dihydroxyvitamin D3 (calcitriol) administration is:
a) Vitamin D resistance
b) Elevated skeletal turnover
c) Hypercalcemia of malignancy
d) Hypophosphatemia
022. The following statement refers to 1,25-dihydroxyvitamin D3 (calcitriol):
a) When rapidity of action is required, 1,25-dihydroxyvitamin D3 (calcitriol), 0.25-1 μg daily, is the vitamin D metabolite of
choice, since it is capable of raising serum calcium within 24-48 hours
b) Calcitriol also raises serum phosphate, though this action is usually not observed early in treatment
c) Undergoes enterohepatic circulation
d) All of the above
023. Which of the following statements refers to 1,25-dihydroxyvitamin D3 (calcitriol):
a) The combined effect of calcitriol and all other vitamin D metabolites and analogs on both calcium and
phosphate makes careful monitoring of the level of these minerals especially important to avoid ectopic
calcification
b) Does not undergo enterohepatic circulation
c) Toxic to osteoclasts
d) Bioavailability increases with the administered dose
024. Route of administration of 1,25-dihydroxyvitamin D3 (calcitriol) is:
a) Subcutaneous
b) Intravenous
c) Intranasal
d) Oral
025. Commercially available analogs of 1,25-dihydroxyvitamin D3 (calcitriol) are:
a) Doxercalciferol (Hectoral)
b) Paricalcitol (Zemplar)
117
c) All of the above
d) None of the above
026. Side effect of dihydrotachysterol is:
a) Hepatic toxicity
b) General malaise
c) Lymphocytopenia
d) Hypertension
027. Route of administration of dihydrotachysterol is:
a) Intravenous
b) Subcutaneous
c) Oral
d) Intranasal
028. Which of the following statements refers to cholecalciferol:
a) Frequent monitoring of both calcium and phosphorus serum levels is necessary in case of intravenous administration
b) Has potent anti-osteoclast activity – mechanism unknown
c) Can usually lower serum calcium levels in 48 hours
d) Mechanism of action: 1. Genomic effects 2. Cytoplasmic effects
029. Indication for cholecalciferol administration is:
a) Hypercalcemia
b) Parathyroid hormone deficiency
c) Primary hyperparathyroidism
d) Malabsorption of vitamin D from intestine
030. Route of administration of cholecalciferol is:
a) Subcutaneous
b) Intranasal
c) Intravenous
d) Oral
031. The unwanted effect of cholecalciferol is:
a) Defective bone mineralization
b) Lymphocytopenia
c) CNS toxicity
d) Metastatic calcifications
032. The unwanted effect of dihydrotachysterol is:
a) Tetany
b) Anorexia
c) CNS toxicity
d) Lymphocytopenia
033. Indication for dihydrotachysterol administration is:
a) Parathyroid hormone resistance
b) Paget's disease
c) Increased osteolysis
d) Hypophosphatemia
034. Conditions associated with hypophosphatemia include:
a) Primary hyperparathyroidism
b) Vitamin D deficiency
c) Idiopathic hypercalciuria
d) All of the above.
035. The long-term effects of hypophosphatemia include proximal muscle weakness and abnormal bone mineralization
(osteomalacia). This consideration is:
a) True
b) False
036. Recommended phosphorus daily allowance is:
a) 900-1200 mg
b) 600-900 g
c) 25 g
d) 1.5-4 mg
037. Interactions with other drugs of phosphorus is:
a) Amiloride: decrease renal excretion
b) Glucocorticoids: decrease absorption
c) Loop diuretics: increase renal excretion
d) Calcitonin: increases renal excretion
118
038. Indication for pamidronate administration is:
a) Failure of vitamin D formation in skin
b) Hypoparathyroidism
c) Elevated skeletal turnover
d) Hypercalcemia
039. Route of administration of pamidronate is:
a) Oral
b) Subcutaneous
c) Intranasal
d) Intravenous
040. Correct statements about pamidronate include all of the following, EXCEPT:
a) Because it causes gastric irritation, pamidronate is not available as an oral preparation
b) Skeletal half-life is 24 h
c) Fever and lymphocytopenia are reversible
d) Can be irritable to the esophagus if not washed promptly to the stomach
041. Route of administration of alendronate is:
a) Intravenous
b) Subcutaneous
c) Oral
d) Intranasal
042. Correct statements about alendronate include all of the following, EXCEPT:
a) Can be irritable to the esophagus if not washed promptly to the stomach
b) 1st generation biphosphonate
c) Reduces osteoclast activity without significantly affecting osteoblasts; useful in the treatment of Paget's disease
d) More potent than EHDP; has a wider therapeutic window
043. Indications of alendronate are the following, EXCEPT:
a) Hypoparathyroidism
b) Glucocorticoid-induced osteoporosis
c) Paget's disease
d) Syndromes of ectopic calcification
044. Indication for etidronate administration is:
a) Malabsorption of vitamin D from intestine
b) Paget's disease
c) Vitamin D deficiency in a diet
d) Hypercalciuria
045. Indications for etidronate administration are the following, EXEPT:
a) Paget's disease
b) Osteoporosis
c) Hypophosphatemia
d) Hypercalcemia
046. Which of the following statements refers to etidronate:
a) Reduces osteoclast activity without significantly affecting osteoblasts; useful in treatment of Paget's disease
b) Serum phosphorus concentrations should be monitored at least daily in case of oral administration
c) 2nd generation biphosphonate (amino-biphosphonate)
d) Bioavailability increases with the administered dose
047. Correct statements about etidronate include all of the following, EXCEPT:
a) Skeletal half-life is hundreds of days
b) Bioavailability increases with the administered dose
c) 2nd generation biphosphonate (amino-biphosphonate)
d) 1st generation biphosphonate.
048. Unwanted effect of etidronate is:
a) Anorexia
b) Defective bone mineralization
c) Hypercalcemia
d) Cardiac arrhythmias
049. The major causes of hypocalcemia in the adult are:
a) Hypoparathyroidism
b) Vitamin D deficiency
c) Renal failure and malabsorption
d) All of the above
050. The major causes of hypercalcemia in the adult are the following, EXCEPT :
119
a) Hyperparathyroidism
b) Cancer with or without bone metastases
c) Renal failure and malabsorption
d) Hypervitaminosis D
051. Which of the following statements refers to calcium:
a) Recommended Ca daily allowance for males: 1. 1-10 years: 800 mg 2. 11-18 years: 1200 mg 3. 19-50 years: 1000 mg
4. > 51 years: 1000 mg
b) Ca chloride is very irritating and can cause necrosis if extravasated
c) In achlorhydric patients calcium carbonate should be given with meals to increase absorption or patient switched to
calcium citrate, which is somewhat better absorbed
d) All of the above
052. Indication for calcium administration is:
a) Failure of formation of vitamin D in skin
b) Malabsorption of vitamin D from intestine
c) Hypercalcemia of malignancy
d) Vitamin D deficiency
053. Which of the calcium preparations is the most preferable for IV injection
a) Calcium gluceptate (0.9 meq calcium/mL)
b) Calcium gluconate (0.45 meq calcium/mL)
c) Calcium chloride (0.68-1.36 meq calcium/mL)
d) All of the above
054. Which of the oral calcium preparations is often the preparation of choice:
a) Calcium carbonate (40% calcium)
b) Calcium lactate (13% calcium)
c) Calcium phosphate (25% calcium)
d) Calcium citrate (17% calcium)
055. Interactions with other drugs of calcium is:
a) Ethanol: decreases absorption
b) Loop diuretics: increase renal excretion
c) Glucocorticoids: stimulate renal excretion
d) All of the above
056. Correct statements about magnesium include all of the following, EXCEPT:
a) Magnesium is mainly an intracellular cation, and is the fourth most abundant cation in the body
b) The recommended dietary amounts of magnesium have been set at 6 mg/kg day (350-400 mg)
c) The most common specific causes encountered in clinical practice are: diet, alcoholism (drinking), diarrhea and
malabsorption, diabetes mellitus, diuretics, and drugs such as aminoglycosides and amphotericin
d) It is a physiological calcium agonist
057. Recommended magnesium daily allowance is:
a) 350-400 mg
b) 6-9 g
c) 25 g
d) 1.5-4 mg
058. The major causes of hypomagnesaemia are:
a) Insufficient dietary intake, e.g. malnutrition
b) Abnormal gastrointestinal loss, e.g. severe diarrhea or chronic alcoholism
c) Abnormal renal loss, e.g. diabetes mellitus or during therapy with some kind of drugs such as amphotericin B,
gentamicin, cisplatin, cardiac glycosides, distal and loop diuretics
d) All of the above
059. Which of the magnesium preparation is the most preferable for I.V. injection
a) Magnesium sulfate
b) Magnesium chloride
c) Magnesium glutamate
d) All of the above
060. Which of the oral magnesium preparations is often the preparation of choice:
a) Magnesium lactate
b) Magnesium oxide
c) MagneB6 (Mg pidolate / Mg lactate + pyridoxine hydrochloride)
d) All of the above.
061. Correct statements about fluoride include all of the following, EXCEPT:
a) Fluoride is effective for the prophylaxis of dental caries
b) Fluoride is accumulated by bone and teeth, where it may stabilize the hydroxyapatite crystal
120
c) Subjects living in areas with naturally fluoridated water (1-2 ppm) had more dental caries and fewer vertebral
compression fractures than subjects living in nonfluoridated water areas
d) Chronic exposure to very high level of fluoride dust in the inspired air results in crippling fluorosis, characterized by
thickening of the cortex of long bones and bony exostoses.
062. Recommended fluoride daily allowance is:
a) 1.5-4 mg
b) 600-900 g
c) 25 g
d) 350-400 mg
063. Which of the following statements refers to gallium nitrate:
a) It is approved by the FDA for the management of hypercalcemia of malignancy
b) This drug acts by inhibiting bone resorption
c) Because of potential nephrotoxicity, patients should be well-hydrated and have good renal output before starting the
infusion
d) All of the above
064. Which of the following statements refers to plicamycin (formerly mithramycin):
a) Duration of action is usually several days
b) Mechanism of cytotoxic action appears to involve its binding to DNA, possibly through an antibiotic-Mg2+ complex.
c) The drug causes plasma calcium levels to decrease, apparently through an action on osteoclasts that is independent
of its action on tumor cells and useful in hypercalcemia.
d) All of the above.
065. Unwanted effects of plicamycin (formerly mithramycin) are the following, EXEPT:
a) Thrombocytopenia
b) GI toxicity
c) Bleeding disorders
d) Fractures
066. Unwanted effect of plicamycin (formerly mithramycin) is:
a) Diarrhea
b) Myelosuppression
c) Nephrolithiasis
d) Metastatic calcifications
067. Indication for plicamycin (formerly mithramycin) administration is:
a) Testicular cancers refractory to standard treatment
b) Paget’s disease
c) Hypercalcemia of malignancy
d) All of the above
068. Route of administration of plicamycin is:
a) Intravenous
b) Subcutaneous
c) Intranasal
d) Oral

PART IX Mineralocorticoid, Mineralocorticoid Antagonists, Diuretics, Plasma Expanders


001. Mineralocorticoid effects cause:
a) Increased catabolism
b) Increased Na retension and К excretion
c) Increased gluconeogenesis
d) Deposition of fat on shoulders, face and abdomen
002. Which of the following synthetic steroids shows predominantly mineralocorticoid action?
a) Hydrocortisone
b) Spironolactone
c) Dexamethasone
d) Fludrocortisone
003. The major mineralocorticoids are the following, EXCEPT:
a) Aldosterone
b) Deoxycorticosterone
c) Fludrocortisone
d) Hydrocortisone
004. Which of the following statements about spironolactone is TRUE?
a) Spironolactone reverses many of the manifestations of aldosteronism
b) Spironilactone is also an androgen antagonist and as such is used in the treatment of hirsutism in wormen
121
c) Spironolactone is useful as a diuretic
d) All of the above
005. All of the following statements regarding diuretics are true, EXCEPT:
a) Carbonic anhydrase inhibition leads to increased reabsorption of NaHCO3
+ - + + -
b) Loop diuretics decrease Na reabsorption at the loop of Henle by competing for the Cl site on the Na /K /2Cl
cotransporter
c) In general, the potency of a diuretic is determined by where it acts in the renal tubule
d) Hydrochlorothiazide decreases urinary calcium excretion
006. The drug inhibits the ubiquitous enzyme carbonic anhydrase:
a) Acetazolamide (Diamox)
b) Furosemide (Lasix)
c) Hydrochlorothiazide (HydroDiuril)
d) Spironolactone (Aldactone)
007. The drug acts by competitively blocking NaCl cotransporters in the distal tubule:
a) Acetazolamide (Diamox)
b) Furosemide (Lasix)
c) Hydrochlorothiazide (HydroDiuril)
d) Spironolactone (Aldactone)
008. The drug acts at the proximal tubule:
a) Acetazolamide (Diamox)
b) Furosemide (Lasix)
c) Hydrochlorothiazide (HydroDiuril)
d) Spironolactone (Aldactone)
009. The drug acts by competing with aldosterone for its cytosolic receptors:
a) Acetazolamide (Diamox)
b) Furosemide (Lasix)
c) Hydrochlorothiazide (HydroDiuril)
d) Spironolactone (Aldactone)
010. The drug is a potassium-sparing diuretic that blocks Na+ channels in the collecting tubules:
a) Acetazolamide (Diamox)
b) Amiloride (Midamor)
c) Furosemide (Lasix)
d) Hydrochlorothiazide (HydroDiuril)
011. Chronic use of this drug can lead to distal tubular hypertrophy, which may reduce its diuretic effect:
a) Acetazolamide (Diamox)
b) Amiloride (Midamor)
c) Furosemide (Lasix)
d) Hydrochlorothiazide (HydroDiuril)
012. The drug has a steroid-like structure which is responsible for its anti-androgenic effect:
a) Amiloride (Midamor)
b) Furosemide (Lasix)
c) Hydrochlorothiazide (HydroDiuril)
d) Spironolactone (Aldactone)
013. Sustained use of this drug results in increased plasma urate concentrations:
a) Furosemide (Lasix)
b) Acetazolamide (Diamox)
c) Both of the above
d) Neither of the above
014. The drug can be used to treat glaucoma:
a) Furosemide (Lasix)
b) Acetazolamide (Diamox)
c) Both of the above
d) Neither of the above
015. The drug can cause ototoxicity:
a) Furosemide (Lasix)
b) Acetazolamide (Diamox)
c) Both of the above
d) Neither of the above
016. The drug acts only on the lumenal side of renal tubules:
a) Furosemide (Lasix)
b) Acetazolamide (Diamox)
122
c) Both of the above
d) Neither of the above
017. The drug can promote sodium loss in patients with low (e.g., 40 ml/min) glomerular filtration rates:
a) Furosemide (Lasix)
b) Acetazolamide (Diamox)
c) Both of the above
d) Neither of the above
018. The drug needs aldosterone present in order to be effective:
a) Hydrochlorothiazide (HydroDiuril)
b) Amiloride (Midamor)
c) Both of the above
d) Neither of the above
019. The drug can be used to treat nephrogenic diabetes insipidus:
a) Hydrochlorothiazide (HydroDiuril)
b) Amiloride (Midamor)
c) Both of the above
d) Neither of the above
020. The drug is sometimes part of fixed-dose combinations used to treat essential hypertension:
a) Hydrochlorothiazide (HydroDiuril)
b) Amiloride (Midamor)
c) Both of the above
d) Neither of the above
021. The drug should never be administered to patients taking potassium supplements:
a) Hydrochlorothiazide (HydroDiuril)
b) Amiloride (Midamor)
c) Furosemide (Lasix)
d) Neither of the above
022. The drug decreases calcium excretion in urine:
a) Hydrochlorothiazide (HydroDiuril)
b) Amiloride (Midamor)
c) Furosemide (Lasix)
d) Acetazolamide (Diamox)
023. The drug acts by competitively blocking the Na+/K+/2Cl- cotransporter:
a) Loop diuretics
b) Thiazide diuretics
c) Potassium-sparing diuretics
d) Carbonic anhydrase inhibitors
024. The drug acts at the proximal tubule:
a) Loop diuretics
b) Thiazide diuretics
c) Potassium-sparing diuretics
d) Carbonic anhydrase inhibitors
025. The drug acts in the distal convoluted tubule:
a) Loop diuretics
b) Thiazide diuretics
c) Potassium-sparing diuretics
d) Carbonic anhydrase inhibitors
026. The drug acts in the collecting tubules:
a) Loop diuretics
b) Thiazide diuretics
c) Potassium-sparing diuretics
d) Carbonic anhydrase inhibitors
027. The drug is the most potent diuretic:
a) Loop diuretics
b) Thiazide diuretics
c) Potassium-sparing diuretics
d) Carbonic anhydrase inhibitors
028. The drug acts by competitively blocking the NaCl cotransporter:
a) Loop diuretics
b) Thiazide diuretics
c) Potassium-sparing diuretics
123
d) Carbonic anhydrase inhibitors
029. The drug inhibits sodium and chloride transport in the cortical thick ascending limb and the early distal tubule:
a) Acetazolamide (Diamox)
b) Furosemide (Lasix)
c) Hydrochlorothiazide (Hydrodiuril)
d) Amiloride (Midamor)
030. The drug can cause ototoxicity:
a) Acetazolamide (Diamox)
b) Furosemide (Lasix)
c) Hydrochlorothiazide (Hydrodiuril)
d) Amiloride (Midamor)
031. The drug blocks the sodium/potassium/chloride cotransporter in the thick ascending loop of Henle:
a) Acetazolamide (Diamox)
b) Furosemide (Lasix)
c) Hydrochlorothiazide (Hydrodiuril)
d) Amiloride (Midamor)
032. The drug is one of the most potent diuretics:
a) Acetazolamide (Diamox)
b) Furosemide (Lasix)
c) Hydrochlorothiazide (Hydrodiuril)
d) Amiloride (Midamor)
033. The drug is usually given in combination with a thiazide diuretic:
a) Acetazolamide (Diamox)
b) Furosemide (Lasix)
c) Hydrochlorothiazide (Hydrodiuril)
d) Amiloride (Midamor)
034. All of the following statements regarding diuretics are true EXCEPT:
a) Furosemide (Lasix) can increase the likelihood of digitalis toxicity
b) Chlorthalidone (Hygroton) can decrease the excretion of lithium
c) Ibuprofen can increase the antihypertensive effect of chlorthalidone
d) Chlorthalidone has a longer duration of action than furosemide
035. The drug is the least potent diuretic:
a) Osmotic diuretics
b) Loop diuretics
c) Thiazide diuretics
d) Potassium-sparing diuretics
036. These agents must be given parenterally because they are not absorbed when given orally:
a) Osmotic diuretics
b) Loop diuretics
c) Thiazide diuretics
d) Potassium-sparing diuretics
037. These drugs may be used in the treatment of recurrent calcium nephrolithiasis:
a) Osmotic diuretics
b) Loop diuretics
c) Thiazide diuretics
d) Potassium-sparing diuretics
038. Furosemide (Lasix) acts at this nephron site:
a) Proximal convoluted tubule
b) Ascending thick limb of the loop of Henle
c) Distal convoluted tubule
d) Collecting duct
039. Metolazone (Mykrox) acts at this nephron site:
a) Proximal convoluted tubule
b) Ascending thick limb of the loop of Henle
c) Distal convoluted tubule
d) Collecting duct
040. Acetazolamide (Diamox) acts at this nephron site:
a) Proximal convoluted tubule
b) Ascending thick limb of the loop of Henle
c) Distal convoluted tubule
d) Collecting duct
124
041. Spironolactone (Aldactone) acts at this nephron site:
a) Proximal convoluted tubule
b) Ascending thick limb of the loop of Henle
c) Distal convoluted tubule
d) Collecting duct
042. Amiloride (Midamone) acts at this nephron site:
a) Proximal convoluted tubule
b) Ascending thick limb of the loop of Henle
c) Distal convoluted tubule
d) Collecting duct
043. The drug competitively blocks chloride channels and prevents movement of sodium, potassium, and chloride into the
renal tubular cells:
a) Furosemide (Lasix)
b) Acetazolamide (Diamox)
c) Triamterene (Dyrenium)
d) Mannitol (Osmitrol)
044. The drug acts by affecting the tubular fluid composition in a non-receptor mediated fashion:
a) Furosemide (Lasix)
b) Acetazolamide (Diamox)
c) Triamterene (Dyrenium)
d) Mannitol (Osmitrol)
045. The drug is a blood substitute having haemodynamical activity:
a) Polyglucinum
b) Haemodesum
c) Sodium chloridum isotonic for injections
d) "Disolum", "Trisolum"
046. This drug is a desintoxicative plasma substitute:
a) Polyglucinum
b) Haemodesum
c) Sodium chloridum isotonic for injections
d) "Disolum", "Trisolum"
047. This drug is a controller of water-salt and acid-basic state:
a) Polyglucinum
b) Haemodesum
c) Glucose isotonic for injections
d) "Disolum", "Trisolum"

(6) CHEMOTHERAPEUTIC DRUGS

PART I ANTIBIOTICS
001. What does the term “antibiotics” mean:
a) Non-organic or synthetic substances that selectively kill or inhibit the growth of other microorganisms
b) Substances produced by some microorganisms and their synthetic analogues that selectively kill or inhibit
the growth of another microorganisms
c) Substances produced by some microorganisms and their synthetic analogues that inhibit the growth of organism cells
d) Synthetic analogues of natural substances that kill protozoa and helminthes
002. General principles of anti-infective therapy are:
a) Clinical judgment of microbiological factors
b) Definitive identification of a bacterial infection and the microorganism’s susceptibility
c) Optimal route of administration, dose, dosing frequency and duration of treatment
d) All of the above
003. Minimal duration of antibacterial treatment usually is:
a) Not less than 1 day
b) Not less than 5 days
c) Not less than 10-14 days
d) Not less than 3 weeks
004. Rational anti-microbial combination is used to:
a) Provide synergism when microorganisms are not effectively eradicated with a single agent alone
b) Provide broad coverage
c) Prevent the emergence of resistance
d) All of the above
125
005. Mechanisms of bacterial resistance to anti-microbial agents are the following, EXCEPT:
a) Active transport out of a microorganism or/and hydrolysis of an agent via enzymes produced by a microorganism
b) Enlarged uptake of the drug by a microorganism
c) Modification of a drug’s target
d) Reduced uptake by a microorganism
006. The statement, that some microorganisms can develop alternative metabolic pathways for rendering reactions inhibited
by the drug, is:
a) True
b) False
007. All of the following drugs are antibiotics, EXCEPT:
a) Streptomycin
b) Penicillin
c) Co-trimoxazole
d) Chloramphenicol
008. Bactericidal effect is:
a) Inhibition of bacterial cell division
b) Inhibition of young bacterial cell growth
c) Destroying of bacterial cells
d) Formation of bacterial L-form
009. Which of the following groups of antibiotics demonstrates a bactericidal effect?
a) Tetracyclines
b) Macrolides
c) Penicillins
d) All of the above
010. Bacteristatic effect is:
a) Inhibition of bacterial cell division
b) Inhibition of young bacterial cells growth
c) Destroying of bacterial cells
d) Formation of bacterial L-form
011. Which of the following groups of antibiotics demonstrates a bacteristatic effect:
a) Carbapenems
b) Macrolides
c) Aminoglycosides
d) Cephalosporins
012. Which of the following antibiotics contains a beta-lactam ring in their chemical structure :
a) Penicillins
b) Cephalosporins
c) Carbapenems and monobactams
d) All groups
013. Tick the drug belonging to antibiotics-macrolides:
a) Neomycin
b) Doxycycline
c) Erythromycin
d) Cefotaxime
014. Tick the drug belonging to antibiotics-carbapenems:
a) Aztreonam
b) Amoxacillin
c) Imipinem
d) Clarithromycin
015. Tick the drug belonging to antibiotics-monobactams:
a) Ampicillin
b) Bicillin-5
c) Aztreonam
d) Imipinem
016. Tick the drug belongs to antibiotics-cephalosporins:
a) Streptomycin
b) Cefaclor
c) Phenoxymethilpenicillin
d) Erythromycin
017. Tick the drug belonging to lincozamides:
a) Erythromycin
126
b) Lincomycin
c) Azithromycin
d) Aztreonam
018. Tick the drug belonging to antibiotics-tetracyclines:
a) Doxycycline
b) Streptomycin
c) Clarithromycin
d) Amoxacillin
019. All of antibiotics are aminoglycosides, EXCEPT:
a) Gentamycin
b) Streptomycin
c) Clindamycin
d) Neomycin
020. Tick the drug belonging to nitrobenzene derivative:
a) Clindamycin
b) Streptomycin
c) Azithromycin
d) Chloramphenicol
021. Tick the drug belonging to glycopeptides:
a) Vancomycin
b) Lincomycin
c) Neomycin
d) Carbenicillin
022. Antibiotics inhibiting the bacterial cell wall synthesis are:
a) Beta-lactam antibiotics
b) Tetracyclines
c) Aminoglycosides
d) Macrolides
023. Antibiotic inhibiting bacterial RNA synthesis is:
a) Erythromycin
b) Rifampin
c) Chloramphenicol
d) Imipinem
024. Antibiotics altering permeability of cell membranes are:
a) Glycopeptides
b) Polymyxins
c) Tetracyclines
d) Cephalosporins
025. All of the following antibiotics inhibit the protein synthesis in bacterial cells, EXCEPT:
a) Macrolides
b) Aminoglycosides
c) Glycopeptides
d) Tetracyclines
026. Biosynthetic penicillins are effective against:
a) Gram-positive and gram-negative cocci, Corynebacterium diphtheria, spirochetes, Clostridium gangrene
b) Corynebacterium diphtheria, mycobacteries
c) Gram positive cocci, viruses
d) Gram negative cocci, Rickettsia, mycotic infections
027. Which of the following drugs is a gastric acid resistant:
a) Penicillin G
b) Penicillin V
c) Carbenicillin
d) Procain penicillin
028. Which of the following drugs is penicillinase resistant:
a) Oxacillin
b) Amoxacillin
c) Bicillin-5
d) Penicillin G
029. All of the following drugs demonstrate a prolonged effect, EXCEPT:
a) Penicillin G
b) Procain penicillin
127
c) Bicillin-1
d) Bicillin-5
030. Mechanism of penicillins’ antibacterial effect is:
a) Inhibition of transpeptidation in the bacterial cell wall
b) Inhibition of beta-lactamase in the bacterial cell
c) Activation of endogenous proteases, that destroy bacterial cell wall
d) Activation of endogenous phospholipases, which leads to alteration of cell membrane permeability
031. Pick out the beta-lactamase inhibitor for co-administration with penicillins:
a) Clavulanic acid
b) Sulbactam
c) Tazobactam
d) All of the above
032. Cephalosporines are drugs of choice for treatment of:
a) Gram-positive microorganism infections
b) Gram-negative microorganism infections
c) Gram-negative and gram-positive microorganism infections, if penicillins have no effect
d) Only bacteroide infections
033. Carbapenems are effective against:
a) Gram-positive microorganisms
b) Gram-negative microorganisms
c) Only bacteroide infections
d) Broad-spectum
034. All of the following antibiotics are macrolides, EXCEPT:
a) Erythromycin
b) Clarithromycin
c) Lincomycin
d) Roxythromycin
035. Tetracyclins have following unwanted effects:
a) Irritation of gastrointestinal mucosa, phototoxicity
b) Hepatotoxicity, anti-anabolic effect
c) Dental hypoplasia, bone deformities
d) All of the above
036. Tick the drug belonging to antibiotics-aminoglycosides:
a) Erythromycin
b) Gentamycin
c) Vancomycin
d) Polymyxin
037. Aminoglycosides are effective against:
a) Gram positive microorganisms, anaerobic microorganisms, spirochetes
b) Broad-spectum, except Pseudomonas aeruginosa
c) Gram negative microorganisms, anaerobic microorganisms
d) Broad-spectum, except anaerobic microorganisms and viruses
038. Aminoglycosides have the following unwanted effects:
a) Pancytopenia
b) Hepatotoxicity
c) Ototoxicity, nephrotoxicity
d) Irritation of gastrointestinal mucosa
039. Choose the characteristics of chloramphenicol:
a) Broad-spectum. Demonstrates a bactericidal effect.
b) Influences the Gram-positive microorganisms. Demonstrates a bactericidal effect.
c) Influences the Gram-negative microorganisms. Demonstrates a bactericidal effect.
d) Broad-spectum. Demonstrates a bacteristatic effect.
040. Chloramphenicol has the following unwanted effects:
a) Nephrotoxicity
b) Pancytopenia
c) Hepatotoxicity
d) Ototoxicity
041. Choose the characteristics of lincozamides:
a) Broad-spectum. Demonstrates a bactericidal effect.
b) Influence mainly the anaerobic organisms, Gram negative cocci.
c) Broad-spectum. Demonstrates a bacteristatic effect.
128
d) Influence mainly the anaerobic organisms, Gram positive cocci.
042. Lincozamides have the following unwanted effect:
a) Nephrotoxicity
b) Cancerogenity
c) Pseudomembranous colitis
d) Irritation of respiratory organs
043. Choose the characteristics of vancomicin:
a) It is a glycopeptide, inhibits cell wall synthesis active only against Gram-negative bacteria
b) It is a glycopeptide, that alters permeability of cell membrane and is active against anaerobic bacteria
c) It is a beta-lactam antibiotic, inhibits cell wall synthesis active only against Pseudomonas aeruginosa
d) It is a glycopeptide, inhibits cell wall synthesis and is active only against Gram-positive bacteria.
044. Vancomicin has the following unwanted effects:
a) Pseudomembranous colitis
b) Hepatotoxicity
c) “Red neck” syndrome, phlebitis
d) All of the above
045. Which of the following drugs is used for systemic and deep mycotic infections treatment:
a) Co-trimoxazol
b) Griseofulvin
c) Amphotericin B
d) Nitrofungin
046. Which of the following drugs is used for dermatomycosis treatment:
a) Nystatin
b) Griseofulvin
c) Amphotericin B
d) Vancomycin
047. Which of the following drugs is used for candidiasis treatment:
a) Griseofulvin
b) Nitrofungin
c) Myconazol
d) Streptomycin
048. All of the following antifungal drugs are antibiotics, EXCEPT:
a) Amphotericin B
b) Nystatin
c) Myconazol
d) Griseofulvin
049. Mechanism of Amphotericin B action is:
a) Inhibition of cell wall synthesis
b) Inhibition of fungal protein synthesis
c) Inhibition of DNA synthesis
d) Alteration of cell membrane permeability
050. Azoles have an antifungal effect because of:
a) Inhibition of cell wall synthesis
b) Inhibition of fungal protein synthesis
c) Reduction of ergosterol synthesis
d) Inhibition of DNA synthesis
051. Which of the following drugs alters permeability of Candida cell membranes:
a) Amphotericin B
b) Ketoconazole
c) Nystatin
d) Terbinafine
052. Amfotericin B has the following unwanted effects:
a) Psychosis
b) Renal impairment, anemia
c) Hypertension, cardiac arrhythmia
d) Bone marrow toxicity
053. Tick the drug belonging to antibiotics having a polyene structure:
a) Nystatin
b) Ketoconazole
c) Griseofulvin
d) All of the above
129
054. All of the following drugs demonstrate a fungicidal effect, EXCEPT:
a) Terbinafin
b) Amfotericin B
c) Ketoconazole
d) Myconazol
055. Characteristics of polyenes are following, except:
a) Alter the structure and functions of cell membranes
b) Broad-spectrum
c) Fungicidal effect
d) Nephrotoxicity, hepatotoxicity
056. Characteristics of Amfotericin B are following, EXCEPT:
a) Used for systemic mycosis treatment
b) Poor absorption from the gastro-intestinal tract
c) Does not demonstrate nephrotoxicity
d) Influences the permeability of fungus cell membrane

PART II SYNTHETIC ANTIBACTERIAL DRUGS


001. Sulfonamides are effective against:
a) Bacteria and Chlamidia
b) Actinomyces
c) Protozoa
d) All of the above
002. Mechanism of sulfonamides’ antibacterial effect is:
a) Inhibition of dihydropteroate reductase
b) Inhibition of dihydropteroate synthase
c) Inhibition of cyclooxygenase
d) Activation of DNA gyrase
003. Combination of sulfonamides with trimethoprim:
a) Decreases the unwanted effects of sulfonamides
b) Increases the antimicrobial activity
c) Decreases the antimicrobial activity
d) Increases the elimination of sulfonamides
004. Sulfonamide potency is decreased in case of co-administration with:
a) Oral hypoglycemic agents
b) Local anesthetics – derivatives of paraaminobenzoic acid
c) Local anesthetics – derivatives of benzoic acid
d) Non-narcotic analgesics
005. The following measures are necessary for prevention of sulfonamide precipitation and crystalluria:
a) Taking of drinks with acid pH
b) Taking of drinks with alkaline pH
c) Taking of saline drinks
d) Restriction of drinking
006. Resorptive sulfonamides have the following unwanted effects on blood system:
a) Hemolytic anemia
b) Thrombocytopenia
c) Granulocytopenia
d) All of the above
007. Mechanism of Trimethoprim’ action is:
a) Inhibition of cyclooxygenase
b) Inhibition of dihydropteroate reductase
c) Inhibition of dihydropteroate synthase
d) Inhibition of DNA gyrase
008. Sulfonamides have the following unwanted effects:
a) Hematopoietic disturbances
b) Crystalluria
c) Nausea, vomiting and diarrhea
d) All of the above
009. Tick the drug, which is effective against mycobacteria only:
a) Isoniazid
b) Streptomycin
c) Rifampin
130
d) Kanamycin
010. Tick the antimycobacterial drug belonging to first-line agents:
a) PAS
b) Isoniazid
c) Kanamycin
d) Pyrazinamide
011. Tick the antimycobacterial drug, belonging to second-line agents:
a) Isoniazid
b) PAS
c) Rifampin
d) Streptomycin
012. Tick the antimycobacterial drug, belonging to antibiotics:
a) Isoniazid
b) PAS
c) Ethambutol
d) Rifampin
013. Tick the antimycobacterial drug – hydrazide of isonicotinic acid:
a) Rifampin
b) Isoniazid
c) Ethambutol
d) Pyrazinamide
014. Mechanism of Izoniazid action is:
a) Inhibition of protein synthesis
b) Inhibition of mycolic acids synthesis
c) Inhibition of RNA synthesis
d) Inhibition of ADP synthesis
015. Mechanism of Rifampin action is:
a) Inhibition of mycolic acids synthesis
b) Inhibition of DNA dependent RNA polymerase
c) Inhibition of topoisomerase II
d) Inhibition of cAMP synthesis
016. Mechanism of Cycloserine action is:
a) Inhibition of mycolic acids synthesis
b) Inhibition of RNA synthesis
c) Inhibition of cell wall synthesis
d) Inhibition of pyridoxalphosphate synthesis
017. Mechanism of Streptomycin action is:
a) Inhibition of cell wall synthesis
b) Inhibition of protein synthesis
c) Inhibition of RNA and DNA synthesis
d) Inhibition of cell membranes permeability
018. Rifampin has the following unwanted effect:
a) Dizziness, headache
b) Loss of hair
c) Flu-like syndrome, tubular necrosis
d) Hepatotoxicity
019. Isoniazid has following unwanted effect:
a) Cardiotoxicity
b) Hepatotoxicity, peripheral neuropathy
c) Loss of hair
d) Immunotoxicity
020. Ethambutol has the following unwanted effect:
a) Cardiotoxicity
b) Immunetoxicity
c) Retrobulbar neuritis with red-green color blindness
d) Hepatotoxicity
021. Streptomycin has the following unwanted effect:
a) Cardiotoxicity
b) Hepatotoxicity
c) Retrobulbar neuritis with red-green color blindness
d) Ototoxicity, nephrotoxicity
131
022. Mechanism of aminosalicylic acid action is:
a) Inhibition of mycolic acids synthesis
b) Inhibition of folate synthesis
c) Inhibition of DNA dependent RNA polymerase
d) Inhibition of DNA gyrase
023. All of the following agents are the first-line antimycobacterial drugs, EXCEPT:
a) Rifampin
b) Pyrazinamide
c) Isoniazid
d) Streptomycin
024. All of the following antimycobacterial drugs have a bactericidal effect, EXCEPT:
a) Pyrazinamide
b) Streptomycin
c) Rifampin
d) Isoniazid
025. Combined chemotherapy of tuberculosis is used to:
a) Decrease mycobacterium drug-resistance
b) Increase mycobacterium drug-resistance
c) Decrease the antimicrobal activity
d) Decrease the onset of antimycobacterial drugs biotransformation:
026. Tick the antibacterial drug – a nitrofurane derivative:
a) Nitrofurantoin
b) Trimethoprim
c) Ciprofloxacin
d) Nystatin
027. Tick the antibacterial drug – a nitroimidazole derivative:
a) Clavulanic acid
b) Metronidazole
c) Nitrofurantoin
d) Doxycycline
028. Tick the antibacterial drug – a quinolone derivative:
a) Nitrofurantoin
b) Nalidixic acid
c) Streptomycin
d) Metronidazole
029. Tick the antibacterial drug – a fluoroquinolone derivative:
a) Chloramphenicol
b) Nitrofurantoin
c) Nalidixic acid
d) Ciprofloxacin
030. Tick the indications for nitrofuranes:
a) Infections of respiratory tract
b) Infections of urinary and gastro-intestinal tracts
c) Syphilis
d) Tuberculosis
031. Tick the unwanted effects of nitrofuranes:
a) Nausea, vomiting
b) Allergic reactions
c) Hemolytic anemia
d) All of the above
032. Tick the indications for Metronidazole:
a) Intra-abdominal infections, vaginitis, enterocolitis
b) Pneumonia
c) As a disinfectant
d) Influenza
033. Tick the unwanted effects of Metronidazole:
a) Nausea, vomiting, diarrhea, stomatitis
b) Hypertension
c) Disturbances of peripheral blood circulation
d) All of the above
034. The mechanism of fluoroquinolones’ action is:
132
a) Inhibition of phospholipase C
b) Inhibition of DNA gyrase
c) Inhibition of bacterial cell synthesis
d) Alteration of cell membrane permeability
035. Fluoroquinolones are active against:
a) Gram negative microorganisms only
b) Mycoplasmas and Chlamidiae only
c) Gram positive microorganisms only
d) Variety of Gram-negative and positive microorganisms, including Mycoplasmas and Chlamidiae
036. Tick the unwanted effects of fluoroquinolones:
a) Hallucinations
b) Headache, dizziness, insomnia
c) Hypertension
d) Immunetoxicity
037. Tick the indications for fluoroquinolones:
a) Infections of the urinary tract
b) Bacterial diarrhea
c) Infections of the urinary and respiratory tract, bacterial diarrhea
d) Respiratory tract infections
038. The drug of choice for syphilis treatment is:
a) Gentamycin
b) Penicillin
c) Chloramphenicol
d) Doxycycline

PART III ANTIPROTOZOAL AND ANTHELMINTIC DRUGS


001. Tick the drug used for malaria chemoprophylaxis and treatment:
a) Chloroquine
b) Quinidine
c) Quinine
d) Sulfonamides
002. Tick the drug used for amoebiasis treatment:
a) Nitrofurantoin
b) Iodoquinol
c) Pyrazinamide
d) Mefloquine
003. Tick the drug used for trichomoniasis treatment:
a) Metronidazole
b) Suramin
c) Pyrimethamine
d) Tetracycline
004. Tick the drug used for toxoplasmosis treatment:
a) Chloroquine
b) Tetracyclin
c) Suramin
d) Pyrimethamine
005. Tick the drug used for balantidiasis treatment::
a) Azitromycin
b) Tetracycline
c) Quinine
d) Trimethoprim
006. Tick the drug used for leishmaniasis treatment:
a) Pyrimethamine
b) Albendazole
c) Sodium stibogluconate
d) Tinidazole
007. Tick the antimalarial drug belonging to 8-aminoquinoline derivatives:
a) Doxycycline
b) Quinidine
c) Primaquine
d) Chloroquine
133
008. All of the following antimalarial drugs are 4-quinoline derivatives, EXCEPT:
a) Chloroquine
b) Mefloquine
c) Primaquine
d) Amodiaquine
009. Tick the antimalarial drug belonging to pyrimidine derivatives:
a) Mefloquine
b) Pyrimethamine
c) Quinidine
d) Chloroquine
010. Tick the drug used for trypanosomosis treatment:
a) Melarsoprol
b) Metronidazole
c) Tetracyclin
d) Quinidine
011. Tick the antimalarial drug having a gametocidal effect:
a) Mefloquine
b) Primaquine
c) Doxycycline
d) Sulfonamides
012. All of the following antimalarial drugs influence blood schizonts, EXCEPT:
a) Mefloquine
b) Chloroquine
c) Primaquine
d) Quinidine
013. Tick the antimalarial drug influencing tissue schisonts:
a) Mefloquine
b) Chloroquine
c) Quinidine
d) Primaquine
014. Tick the group of antibiotics having an antimalarial effect:
a) Aminoglycosides
b) Tetracyclins
c) Carbapenems
d) Penicillins
015. Tick the amebecide drug for the treatment of an asymptomatic intestinal form of amebiasis:
a) Chloroquine
b) Diloxanide
c) Emetine
d) Doxycycline
016. Tick the drugs for the treatment of an intestinal form of amebiasis:
a) Metronidazole and diloxanide
b) Diloxanide and streptomycin
c) Diloxanide and Iodoquinol
d) Emetine and metronidazole
017. Tick the drug for the treatment of a hepatic form of amebiasis:
a) Diloxanide or iodoquinol
b) Tetracycline or doxycycline
c) Metronidazole or emetine
d) Erythromycin or azitromycin
018. Tick the luminal amebecide drug:
a) Metronidazole
b) Emetine
c) Doxycycline
d) Diloxanide
019. Tick the drug of choice for the treatment of extraluminal amebiasis:
a) Iodoquinol
b) Metronidazole
c) Diloxanide
d) Tetracycline
020. Tick the drug, blocking acetylcholine transmission at the myoneural junction of helminthes:
134
a) Levamisole
b) Mebendazole
c) Piperazine
d) Niclosamide
021. Tick niclosamide mechanism of action:
a) Increasing cell membrane permeability for calcium, resulting in paralysis, dislodgement and death of helminthes
b) Blocking acetylcholine transmission at the myoneural junction and paralysis of helminthes
c) Inhibiting microtubule synthesis in helminthes and irreversible impairment of glucose uptake
d) Inhibiting oxidative phosphorylation in some species of helminthes
022. Tick praziquantel mechanism of action:
a) Blocking acetylcholine transmission at the myoneural junction and paralysis of helminthes
b) Inhibiting microtubule synthesis in helminthes and irreversible impairment of glucose uptake
c) Increasing cell membrane permeability for calcium, resulting in paralysis, dislodgement and death of
helminthes
d) Inhibiting oxidative phosphorylation in some species of helminthes
023. Tick piperazine mechanism of action:
a) Inhibiting microtubule synthesis in helminthes and irreversible impairment of glucose uptake
b) Blocking acetylcholine transmission at the myoneural junction and paralysis of helminthes
c) Inhibiting oxidative phosphorylation in some species of helminthes
d) Increasing cell membrane permeability for calcium, resulting in paralysis, dislodgement and death of helminthes
024. Tick the drug, a salicylamide derivative:
a) Praziquantel
b) Piperazine
c) Mebendazole
d) Niclosamide
025. Tick mebendazole mechanism of action:
a) Inhibiting oxidative phosphorylation in some species of helminthes
b) Increasing cell membrane permeability for calcium, resulting in paralysis, dislodgement and death of helminthes
c) Inhibiting microtubule synthesis in helminthes and irreversible impairment of glucose uptake
d) Blocking acetylcholine transmission at the myoneural junction and paralysis of helminthes
026. Tick the drug, inhibiting oxidative phosphorylation in some species of helminthes:
a) Niclosamide
b) Piperazine
c) Praziquantel
d) Mebendazole
027. Tick the drug for neurocysticercosis treatment:
a) Praziquantel
b) Pyrantel
c) Piperazine
d) Bithionol
028. Tick the drug for nematodosis (roundworm invasion) treatment:
a) Niclosamide
b) Praziquantel
c) Bithionol
d) Pyrantel
029. Tick the drug for cestodosis (tapeworm invasion) treatment:
a) Piperazine
b) Praziquantel
c) Pyrantel
d) Ivermectin
030. Tick the drug for trematodosis (fluke invasion) treatment:
a) Bithionol
b) Ivermectin
c) Pyrantel
d) Metronidazole
031. Tick the drug, a benzimidazole derivative:
a) Praziquantel
b) Mebendazole
c) Suramin
d) Pyrantel
032. Tick the broad spectrum drug for cestodosis, trematodosis and cycticercosis treatment:
135
a) Piperazine
b) Ivermectine
c) Praziquantel
d) Pyrantel
033. Tick the drug for ascaridosis and enterobiosis treatment:
a) Bithionol
b) Pyrantel
c) Praziquantel
d) Suramin
034. Tick the drug for strongiloidosis treatment:
a) Niclosamide
b) Praziquantel
c) Bithionol
d) Ivermectin
035. Tick the drug for echinococcosis treatment:
a) Suramin
b) Mebendazole or Albendazole
c) Piperazine
d) Iodoquinol

PART IV ANTIVIRAL AGENTS. AGENTS FOR CHEMOTHERAPY OF CANCER


001. All of the following antiviral drugs are the analogs of nucleosides, EXCEPT:
a) Acyclovir
b) Zidovudine
c) Saquinavir
d) Didanozine
002. Tick the drug, a derivative of adamantane:
a) Didanozine
b) Rimantadine
c) Gancyclovir
d) Foscarnet
003. Tick the drug, a derivative of pyrophosphate:
a) Foscarnet
b) Zidovudine
c) Vidarabine
d) Acyclovir
004. Tick the drug, inhibiting viral DNA synthesis:
a) Interferon
b) Saquinavir
c) Amantadine
d) Acyclovir
005. Tick the drug, inhibiting uncoating of the viral RNA:
a) Vidarabine
b) Rimantadine
c) Acyclovir
d) Didanozine
006. Tick the drug, inhibiting viral reverse transcriptase:
a) Zidovudine
b) Vidarabine
c) Rimantadine
d) Gancyclovir
007. Tick the drug, inhibiting viral proteases:
a) Rimantadine
b) Acyclovir
c) Saquinavir
d) Zalcitabine
008. Tick the drug of choice for herpes and cytomegalovirus infection treatment:
a) Saquinavir
b) Interferon alfa
c) Didanozine
d) Acyclovir
136
009. Tick the drug which belongs to nonnucleoside reverse transcriptase inhibitors:
a) Zidovudine
b) Vidarabine
c) Nevirapine
d) Gancyclovir
010. All of the following antiviral drugs are antiretroviral agents, EXCEPT:
a) Acyclovir
b) Zidovudine
c) Zalcitabine
d) Didanozine
011. Tick the drug used for influenza A prevention:
a) Acyclovir
b) Rimantadine
c) Saquinavir
d) Foscarnet
012. Tick the drug used for HIV infection treatment, a derivative of nucleosides:
a) Acyclovir
b) Zidovudine
c) Gancyclovir
d) Trifluridine
013. Tick the antiviral drug which belongs to endogenous proteins:
a) Amantadine
b) Saquinavir
c) Interferon alfa
d) Pencyclovir
014. Tick the drug which belongs to nucleoside reverse transcriptase inhibitors:
a) Didanosine
b) Gancyclovir
c) Nevirapine
d) Vidarabine
015. All of the following antiviral drugs are anti-influenza agents, EXCEPT:
a) Acyclovir
b) Amantadine
c) Interferons
d) Rimantadine
016. Tick the unwanted effects of zidovudine:
a) Hallucinations, dizziness
b) Anemia, neutropenia, nausea, insomnia
c) Hypertension, vomiting
d) Peripheral neuropathy
017. Tick the unwanted effects of intravenous acyclovir infusion:
a) Renal insufficiency, tremors, delerium
b) Rash, diarrhea, nausea
c) Neuropathy, abdominal pain
d) Anemia, neutropenia, nausea, insomnia
018. Tick the drug that can induce peripheral neuropathy and oral ulceration:
a) Acyclovire
b) Zalcitabine
c) Zidovudine
d) Saquinavir
019. Tick the unwanted effects of didanozine:
a) Hallucinations, dizziness, insomnia
b) Anemia, neutropenia, nausea
c) Hypertension, vomiting, diarrhea
d) Peripheral neuropathy, pancreatitis, diarrhea, hyperuricemia
020. Tick the unwanted effects of indinavir:
a) Hypotension, vomiting, dizziness
b) Nephrolithiasis, nausea, hepatotoxicity
c) Peripheral neuropathy, pancreatitis, hyperuricemia
d) Anemia, neutropenia, nausea
021. Tick the drug that can induce nausea, diarrhea, abdominal pain and rhinitis:
137
a) Acyclovire
b) Zalcitabine
c) Zidovudine
d) Saquinavir
022. All of the following effects are disadvantages of anticancer drugs, EXCEPT:
a) Low selectivity to cancer cells
b) Depression of bone marrow
c) Depression of angiogenesis
d) Depression of immune system
023. Rational combination of anticancer drugs is used to:
a) Provide synergism resulting from the use of anticancer drugs with different mechanisms combination
b) Provide synergism resulting from the use of anticancer drugs with the same mechanisms combination
c) Provide stimulation of immune system
d) Provide stimulation of cell proliferation
024. Tick the anticancer alkylating drug, a derivative of chloroethylamine:
a) Methotrexate
b) Cisplatin
c) Cyclophosphamide
d) Carmustine
025. Tick the anticancer alkylating drug, a derivative of ethylenimine:
a) Mercaptopurine
b) Thiotepa
c) Chlorambucil
d) Procarbazine
026. Tick the group of hormonal drugs used for cancer treatment:
a) Mineralocorticoids and glucocorticoids
b) Glucocorticoids and gonadal hormones
c) Gonadal hormones and somatotropin
d) Insulin
027. Tick the anticancer alkylating drug, a derivative of alkylsulfonate:
a) Fluorouracil
b) Carboplatin
c) Vinblastine
d) Busulfan
028. Tick the anticancer drug of plant origin:
a) Dactinomycin
b) Vincristine
c) Methotrexate
d) Procarbazine
029. Action mechanism of alkylating agents is:
a) Producing carbonium ions altering protein structure
b) Producing carbonium ions altering DNA structure
c) Structural antagonism against purine and pyrimidine
d) Inhibition of DNA-dependent RNA synthesis
030. Tick the anticancer drug, a pyrimidine antagonist:
a) Fluorouracil
b) Mercaptopurine
c) Thioguanine
d) Methotrexate
031. Methotrexate is:
a) A purine antagonist
b) A folic acid antagonist
c) An antibiotic
d) An alkylating agent
032. Tick the antibiotic for cancer chemotherapy:
a) Cytarabine
b) Doxorubicin
c) Gentamycin
d) Etoposide
033. Fluorouracil belongs to:
a) Antibiotics
138
b) Antimetabolites
c) Plant alkaloids
d) Bone marrow growth factor
034. Tick the action mechanism of anticancer drugs belonging to plant alkaloids:
a) Inhibition of DNA-dependent RNA synthesis
b) Cross-linking of DNA
c) Mitotic arrest at a metaphase
d) Nonselective inhibition of aromatases
035. General contraindications for anticancer drugs are:
a) Depression of bone marrow
b) Acute infections
c) Severe hepatic and/or renal insufficiency
d) All of the above
036. Action mechanism of methotrexate is:
a) Inhibition of dihydrofolate reductase
b) Activation of cell differentiation
c) Catabolic depletion of serum asparagine
d) All of the above
037. Tick the anticancer drug belonging to inorganic metal complexes:
a) Dacarbazine
b) Cisplatin
c) Methotrexate
d) Vincristine
038. Tick the indication for estrogens in oncological practice:
a) Leukemia
b) Cancer of prostate
c) Endometrial cancer
d) Brain tumors
039. Enzyme drug used for acute leukemia treatment:
a) Dihydrofolate reductase
b) Asparaginase
c) Aromatase
d) DNA gyrase
040. All of the following drugs are derivatives of nitrosoureas, EXCEPT:
a) Carmustine
b) Vincristine
c) Lomustine
d) Semustine
041. Tick the group of drugs used as subsidiary medicines in cancer treatment:
a) Cytoprotectors
b) Bone marrow growth factors
c) Antimetastatic agents
d) All of the above
042. Tick the estrogen inhibitor:
a) Leuprolide
b) Tamoxifen
c) Flutamide
d) Anastrozole
043. Tick the antiandrogen drug:
a) Flutamide
b) Aminoglutethimide
c) Tamoxifen
d) Testosterone
044. Tick the drug belonging to aromatase inhibitors:
a) Octreotide
b) Anastrozole
c) Flutamide
d) Tamoxifen
045. Tick the drug belonging to gonadotropin-releasing hormone agonists:
a) Leuprolide
b) Tamoxifen
139
c) Flutamide
d) Anastrozole
4/20/2020 Student Area :: TesT.EDU.kg

Module:

1. The drug acts by competitively blocking the NaCl cotransporter:

Loop diuretics

Thiazide diuretics

Potassium-sparing diuretics

Carbonic anhydrase inhibitors

2. Choose the drug which is a H2-receptor antagonist:

Omeprazole

Pirenzepine

Carbenoxolone

Ranitidine

3. Select the side-effect characteristic for non-selective beta2-adrenomimics:

Depression of the breathing center

Tachycardia

Peripheral vasoconstriction

Dry mouth

4. All of the following statements regarding cardiac glycosides are true EXCEPT:

They inhibit the Na+/K+-ATPase and thereby increase intracellular Ca++ in myocardial cells

They cause a decrease in vagal tone

Children tolerate higher doses of digitalis than do adults

The most frequent cause of digitalis intoxication is concurrent administration of diuretics that
deplete K

5. All of the following drugs are proton pump inhibitors EXCEPT:

Pantoprozole

Omeprazole

Famotidine

test.edu.kg/s/modules/ 1/8
4/20/2020 Student Area :: TesT.EDU.kg

Rabeprazole

6. Tick out the drug belonging to non-narcotic antitussives:

Libexine

Tusuprex

Codeine

Aethylmorphine hydrochloride

7. Which of the following M-cholinoblocking agents is used especially as an anti-asthmatic?

Atropine

Ipratropium

Platiphylline

Metacin

8. This drug is a directly acting vasodilator:

Labetalol

Clonidine

Enalapril

Nifedipine

9. The drug acts at the proximal tubule:

Loop diuretics

Thiazide diuretics

Potassium-sparing diuretics

Carbonic anhydrase inhibitors

10. Tolerance to this inotropic drug develops after a few days:

Amrinone

Amiodarone

Dobutamine

Adenosine

11. Pick out the bronchodilator drug related to xanthine:

Atropine
test.edu.kg/s/modules/ 2/8
4/20/2020 Student Area :: TesT.EDU.kg

Orciprenaline

Adrenaline

Theophylline

12. Which of the following nitrates and nitrite drugs is a short-acting drug?

Nitroglycerin, 2% ointment (Nitrol)

Nitroglycerin, oral sustained-release (Nitrong)

Amyl nitrite, inhalant (Aspirols, Vaporole)

Sustac

13. All of the following statements concerning the use of angiotensin-converting enzyme (ACE)
inhibitors in the treatment of heart failure are true EXCEPT:

They improve hemodynamics by decreasing afterload

They can increase plasma cholesterol levels

They may slow the progression of heart failure by preventing myocardial and vascular remodeling

They are effective rst-line agents in the treatment of chronic heart failure

14. Sustained use of this drug results in increased plasma urate concentrations:

Furosemide (Lasix)

Acetazolamide (Diamox)

Both of the above

Neither of the above

15. Indicate the drug belonging to antitussives of narcotic type of action:

Glaucine hydrochloride

Aethylmorphine hydrochloride

Tusuprex

Libexine

16. Most of drugs are antacids EXCEPT:

Misoprostol

Maalox

Mylanta

test.edu.kg/s/modules/ 3/8
4/20/2020 Student Area :: TesT.EDU.kg

Almagel

17. This drug is an inhibitor of renin synthesis:

Propranolol

Enalapril

Diazoxide

Losartan

18. Which of the following statements concerning nitrate mechanism of action is True?

Therapeutically active agents in this group are capable of releasing nitric oxide (NO) in to
vascular smooth muscle target tissues

Nitric oxide (NO) is an effective activator of soluble guanylyl cyclase and probably acts mainly
through this mechanism

Nitrates useful in angina decrease myocardial oxygen requirement (by decreasing the
determinations of oxygen demand) and increase myocardial oxygen delivery (by reversing
coronary arterial spasm)

All of the above

19. Angina pectoris is:

Severe constricting chest pain, often radiating from the precordium to the left shoulder and down
the arm, due to insu cient blood supply to the heart that is usually caused by coronary disease

An often fatal form of arrhythmia characterized by rapid, irregular brillar twitching of the
ventricles of the heart instead of normal contractions, resulting in a loss of pulse

The cardiovascular condition in which the heart ability to pump blood weakens

All of the above

20. Select the emetic agent having a re ex action:

Ipecacuanha derivatives

Apomorphine hydroclorid

Chlorpromazine

Metoclopramide

21. All of the following statements regarding cardiac glycoside-induced ventricular


tachyarrhythmias are true EXCEPT:

Lidocaine is a drug of choice in treatment

Digibind should be used in life-threatening cases

They occur more frequently in patients with hyperkalemia than in those with hypokalemia

test.edu.kg/s/modules/ 4/8
4/20/2020 Student Area :: TesT.EDU.kg

They are more likely to occur in patients with a severely damaged heart

22. Tick the drug belonging to non-selective beta2-adrenomimics:

Salbutamol

Isoprenaline

Salmeterol

Terbutaline

23. Select the drug which inhibits peristalsis:

Castor oil

Bisacodyl

Loperamide

Sorbitol

24. The drug acts by competitively blocking NaCl cotransporters in the distal tubule:

Acetazolamide (Diamox)

Furosemide (Lasix)

Hydrochlorothiazide (HydroDiuril)

Spironolactone (Aldactone)

25. The following agents are cardioselective beta1-adrenoceptor-blocking drugs labeled for use in
angina, EXCEPT:

Metoprolol

Talinolol

Atenolol

Propranolol

26. Choose the unwanted effects of clonidine:

Parkinson’s syndrome

Sedative and hypnotic effects

Agranulocytosis and aplastic anemia

Dry cough and respiratory depression

27. This drug inhibits the angiotensin-converting enzyme:

test.edu.kg/s/modules/ 5/8
4/20/2020 Student Area :: TesT.EDU.kg

Captopril

Enalapril

Ramipril

All of the above

28. This drug inhibits breakdown of cAMP in vascular smooth muscle:

Digoxin

Dobutamine

Amrinone

Dopamine

29. Indicate the expectorant with the re ex mechanism:

Sodium benzoate

Derivatives of Ipecacucnha and Thermopsis

Trypsin

Ambroxol

30. All of the following drugs are antiemetics EXCEPT:

Metoclopramide

Ondansetron

Chlorpromazine

Apomorphine hydrochloride

31. Choose the drug that causes constipation:

Sodium bicarbonate

Aluminium hydroxide

Calcium carbonate

Magnesium oxide

32. Which of the following antianginal agents refers to re ex coronary dilators:

Dipyridamole

Validol

Atenolol

test.edu.kg/s/modules/ 6/8
4/20/2020 Student Area :: TesT.EDU.kg

Alinidine

Total questions: 40; Correctly answered: 27; correct incorrect skipped


Percentage of correct: 67.5%; Skipped question: 0;
33. All of the following drugs destroy disul de bonds ofanswer answer
proteoglycans, which causes question
Assessment: fair and reduction of viscosity of sputum, EXCEPT:
depolymerization

Acetylcysteine

Ambroxol

Desoxiribonuclease

Bromhexin

34. Pick out the sympatholythic drug:

Labetalol

Prazosin

Guanethidine

Clonidine

35. The drug should never be administered to patients taking potassium supplements:

Hydrochlorothiazide (HydroDiuril)

Amiloride (Midamor)

Furosemide (Lasix)

Neither of the above

36. Tick the antitussive agent with a peripheral effect:

Codeine

Tusuprex

Libexine

Glaucine hydrochloride

37. Choose the vasodilator which releases NO:

Nifedipine

Hydralazine

Minoxidil

Sodium nitroprusside

38. This drug group useful in angina increase myocardial oxygen delivery (by reversing coronary
arterial spasm) and does not decrease myocardial oxygen requirement (by decreasing the

test.edu.kg/s/modules/ 7/8
4/20/2020 Student Area :: TesT.EDU.kg

determinations of oxygen demand):

Beta-adrenoceptor-blocking drugs (Atenolol, Metoprolol)

Myotropic coronary dilators (Dipyridamole)

Calcium channel blockers (Nifedipine, Nimodipine)

Potassium channel openers (Minoxidil)

39. All of the following statements regarding verapamil are true EXCEPT:

It blocks L-type calcium channels

It increases heart rate

It relaxes coronary artery smooth muscle

It depresses cardiac contractility

40. Which of the following antianginal agents is a calcium channel blocker?

Nitroglycerin

Dipyridamole

Minoxidil

Nifedipine

Test System1

test.edu.kg/s/modules/ 8/8
GENERAL PHARMACOLOGY

1. All of the following are general mechanisms of drug permeation Except

A. Aqueous diffusion

B. Aqueous hydrolysis+

C. Lipid diffusion

D. Pinocytosis or endocytosis

2. If the plasma concentration of a drug declines with “first-order kinetics”, this means that

A. There is only one metabolic path for drug disposition

B. The half-life is the same regardless of the plasma concentration+

C. The drug is largely metabolized in the liver after oral administration and has low
bioavailability elimination

D. The rate of elimination is proportionate to the rate of administration at all times

3. Regarding termination of drug action

A. Drug must be exerted from the body to terminate their action

B. Metabolism of drugs always increases their water solubility

C. Metabolism of drugs always abolishes their pharmacologic activity

D. Hepatic metabolism and renal excretion are the two most important mechanisms involved +

4. Distribution of drugs to specific tissues

A. Is independent of blood flow to the organ

B. Is independent of the solubility of the drug in that tissue

C. Depends on the unbound drug concentration gradient between blood and tissue+

D. Is increased for drugs that are strongly bound to plasma proteins


5. A physical process by which a weak acid becomes less water-soluble and more lipid-
soluble at low ph is

A. Distribution

B Elimination

C. First pass- effect

D. Protonation+

6. Dose-response curves are used for drug evaluation in the animal laboratory and in the
clinic, Quantal dose-response curves are often

A. Used for determining the therapeutic index of a drug+

B. Used for determining the maximal efficacy of a drug

C. Invalid in the presence of inhibitors of the drug being studied

D. Obtained from the study of intact subject but not from isolated tissue preparations

7. The following are excreted faster in basic urine

A. Weak acids+

B. Strong acids

C. Weak Bases

D. None of the above

8. Which of the following statements about spare receptors is most correct ?

A. Spare receptors, in the absence of drug, are sequestered in the cytoplasm

B. Spare receptors will be detected if the intracellular effect of drug-receptor interaction lasts
longer than the drug-receptor interaction itself +

C. Spare receptors influence the maximal efficacy of the drug-receptor system

D. Spare receptors activate the effector machinery of the cell without the need for a drug
9. Which of the following terms best describes an antagonist that interacts directly with the
agonist and not at all or only incidentally, with the receptor

A. Pharmacological antagonist

B. Partial agonist

C. Physiological antagonist

D. Chemical antagonist+

10. Which of the following terms best describes a drug that blocks the action of epinephrine
at its receptors by occupying those receptors without activating them

A. Pharmacological antagonist+

B. Partial agonist

C. Physiological antagonist

D. Chemical antagonist

11. Which of the following provides information about the variation in sensitivity of the
drug within the population studied

A. Maximal efficacy

B. Therapeutic index

C. Drug potency

D. Quantal dose-response curve +

12. Which of the following most accurately describes the transmembrane signaling process
involved in steroid hormone action

A. Action on a membrane spanning tyrosine kinase

B. Activation of a G protein which activates or inhibits adenyl cyclase

C. Diffusion into the cytoplasm and binding to an intracellular receptor +

D. Diffusion of “STAT” molecules across the Membrane


13. Which of the following is a phase II drug metabolizing reaction

A. Acetylation +

B. Deamination

C. Hydrolysis

D. Oxidation

14. Which of the following drugs may inhibit the hepatic microsomal P450 responsible for
warfarin metabolism

A. Cimetidine +

B. Ethanol

C. Phenobarbital

D. Procainamide

15. With regard to clinical trials of new drugs, which of the following is most correct

A. Phase I involves the study of a small number of normal volunteers by highly trained clinical
pharmacologists +

B. Phase II involves the use of the new drug in a large number of patients (100-5000) who have
the disease to be treated

C. Phase III involves the determination of the drug’s therapeutic index by the cautious induction
of toxicity

D. Chemical antagonist

16. Animal testing of potential new therapeutic agents

A. Extends over a time period of at least 3 years in order to discover late toxicities +

B. Requires the use of at least two primate species, e.g. Monkey and baboon

C. Requires the submission of histopathologic slides and specimens to the FDA for government
evaluation
D. Has good predictability for drug allergy-type reactions

17. The “dominant lethal” test involves the treatment of a male adult animal with a
chemical before mating; the pregnant female is later examined for fetal death and
abnormalities. The dominant lethal test therefore is a test of

A. Teratogenicity

B. Mutagenicity +

C. Carcinogenicity

D. All of the above

18. The Ames test is a method for detecting

A. Carcinogenesis in rodents

B. Carcinogenesis in primates

C. Teratogenesis in any mammalian species

D. Mutagenesis in bacteria +

19. “Nicotinic” sites include all of the following except

A. Bronchial smooth muscle +

B. Adrenal medullary cells

C. Parasympathetic ganglia

D. Skeletal muscle

20. A good example of chemical antagonism

A. Heparin & Protamine +

B. Protamine & Zinc

C. Heparin & Prothrombin

D. All the above


21. Which of the following agents is a prodrug that is much less toxic in mammals than in
insects

A. Acetylcholine +

B. Bethanechol

C. Physostigmine

D. Pilocarpine

22. Phenylephrine causes

A. Constriction of vessels in the nasal mucosa +

B. Increased gastric secretion and motility

C. Increased skin temperature

D. Miosis

23. Pretreatment with propranolol will block which one of the following

A. Methacholine-induced tachycardia +

B. Nicotine-induced hypertension

C. Norepinephrine-induced bradycardia

D. Phenylephrine-induced mydriasis

24. Most drug receptors are

A. Small molecules with a molecular weight between 100 and 1000

B. Lipids arranged in a bilayer configuration

C. Proteins located on cell membranes or in the cytosol +

D. DNA molecules

25. With regard to distribution of a drug from the blood into tissues

A. Blood flow to the tissue is an important determinant


B. Solubility of the drug in the tissue is an important determinant

C. Concentration of the drug in the blood is an important determinant

D. All of the above are important determinants +

26. The pH value is calculated mathematically as the

A. Log of the hydroxyl ion (OH–) concentration

B. Negative log of the OH– concentration

C. Log of the hydrogen ion (H+) concentration

D. Negative log of the H+ concentration+

27. Which property is classified as colligative?

A. Solubility of a solute

B. Osmotic pressure +

C. Hydrogen ion (H+) concentration

D. Dissociation of a solute

28. The colligative properties of a solution are related to the

A. pH of the solution

B. Number of ions in the solution

C. Total number of solute particles in the solution +

D. Number of unionized molecules in the solution

29. The pH of a buffer system can be calculated with the

A. Noyes – Whitney equation

B. Henderson – Hasselbalch equation +

C. Michaelis – Menten equation

D. Yong equation
30. Which mechanism is most often responsible for chemical degradation?

A. Racemization

B. Photolysis

C. Hydrolysis +

D. Decarboxylation

31. Which equation is used to predict the stability of a drug product at room temperature
from experiments at accelerated temperature?

A. The stokes equation

B. The Yong equation

C. The Arrhenius equation +

D. The Michaelis – Menten equation

32. Based on the relation between the degree of ionization and the solubility of a weak acid,
the drug aspirin (pKa 3.49) will be most soluble at

A. pH 1.0

B. pH 2.0

C. pH 3.0

D. pH 6.0 +

33. The particle size of the dispersed solid in a suspension is usually greater than

A. 0.5 µm +

B. 0.4 µm

C. 0.3 µm

D. 0.2 µm

34. In the extemporaneous preparation of a suspension, levigating is used to


A. Reduce the zeta potential

B. Avoid bacterial growth

C. Reduce particle size +

D. Enhance viscosity

35. Active transport differs from facilitated transport in following ways, except

A. Carrier is involved +

B. It is against concentration gradient

C. Energy is required

D. All of the above

36. Vanishing cream is an ointment that may be classified as

A. A water –soluble base

B. An oleaginous base

C. An absorption base

D. An emulsion base +

37. Rectal suppositories intended for adult use usually weigh approximately

A. 1g

B. 2g +

C. 3g

D. 4g

38. In the fusion method of making cocoa butter suppositories which substance is most
likely to be used to lubricate the mold?

A. Mineral oil +

B. Propylene glycol
C. Cetyl alcohol

D. Stearic acid

39. A very fine powdered chemical is defined as one that

A. Completely passes through a # 80 sieve

B. Completely passes through a # 120 sieve +

C. Completely passes through a # 20 sieve

D. Passes through a # 60 sieve and not more than 40% through a # 100 sieve

40. Which technique is typically used to mill camphor?

A. Trituration

B. Levigation

C. Pulverization and intervention +

D. Geometric dilution

41. Which type of paper best protects a divided hygroscopic powder?

A. Waxed paper +

B. Glassine

C. White bond

D. Blue bond

42. Which capsule size has the smallest capacity?

A. 5 +

B. 4

C. 1

43. The shells of soft gelatin capsules may be made elastic or plastic–like by the addition of

A. Sorbitol +
B. Povidone

C. Polyethylene glycol

D. Lactose

44. Nonionic surface-active agents used as synthetic emulsifiers include

A. Tragacanth +

B. Sodium lauryl sulphate

C. Sorbitan esters(spans)

45. A ceramic mortar may be preferable to a glass mortar when

A. A volatile oil is added to a powder mixture

B. Colored substances (dyes) are mixed into a powder

C. Comminution is desired in addition to mixing +

46. Divided powders may be dispensed in

A. Individual-dose packets +

B. A bulk container

C. A perforated, sifter –type container

47. Agents that may be used to coat enteric coated tablets include

A. Hydroxypropyl methyl cellulose


B. Carboxymethyl cellulose
C. Cellulose acetate phthalate +
48. Active transport of a substance across biological membrane has the following
characteristics except
A. It is specific
B It is pH dependent +
C It is saturable
D It requires metabolic energy
49. Bioavailability of drug refers to
A. Percentage of administered dose that reaches systemic circulation in the unchanged form +
B. Ratio of oral to parental dose
C. Ratio of orally administered drug to that excreted in the feces
D. Ratio of drug excreted unchanged in urine to that excreted as metabolites

50. Factor which can affect the absorption of drug is


A. Dissolution rate
B. Particle Size
C. Lipid Solubility
D. All the above +

51. Excipients are

A. Pharmacologically inert substances


B. Used to mask an unpleasant taste
C. Used to increase solubility or stability to the agent
D. Employed to add bulk to the active agent used in small quantities +

52. All the below mentioned drugs cause enzyme inhibition in man except one

A. Acetazolamide

B. Allopurinol

C. Meprobamate +

D. Disulfiram
53. All the below mentioned drugs cause enzyme induction in man except one

(a) Phenytoin
(c) Griseofulvine

(b) Phenobarbitone
(d) Enalapril +
54. A common oral problem caused by herpes simplex type I virus (HSV-1) is
A. Aphthous ulcers
B. Canker sores
C. Aphthous stomatitis
D. Fever blisters +
55. The definition of a surfactant (an ingredient in toothpaste) can best be described by
which of the following statements
A. Prevents drying of the preparation
B. Removes debris by its detergent action and causes foaming, which is usually desired by the
patient +
C. Physically removes plaque and debris
D. Determines the texture, dispersiveness, and appearance of the product

56. Which is not a risk factor for hyperphosphatemia and death from sodium phosphate
enemas when used in children?
A. Renal insufficiency
B. Hirschsprung’s disease
C. Anorectal malformations
D. Children between the ages of 6 and 12 years +

57. Which of the following factors is associated with an increased risk of noncompliance in
the elderly?
A. Polypharmacy +
B. Hypertension
C. Male gender
D. Living with a spouse in an isolated environment
E. Expensive medications

58. The principal difference between competitive and non-competitive inhibition is


A. Extent of receptor site blocking
B. Whether inhibition occurs
C. Extent of enzyme3 inhibition +
D. Degree of agonism
59. Drug administrated through which of the following routes is mot likely to be subjected
to first-pass metabolism:
A. Oral +
B. Sublingual
C. Subcutaneous
D. Rectal
60. Many receptors use distinct hetero _________ GTPä-binding regulatory proteins
A. Tetrameric
B. Trimeric +
C. Dimeric
D. Monomeric

61. Alkalization of urine hastens the excretion of


A. Weakly basic drugs
B. Weakly acidic drugs +
C. Strong electrolytes
D. Nonpolar drugs

62. Majority of drugs cross biological membranes primarily by


A. Weakly basic drugs +
B. Weakly acidic drugs
C. Strong electrolytes
D. Nonpolar drugs
63. The most important factor which governs diffusion of drugs across capillaries other
than those in the brain is
A. Blood flow through the capillary +
B. Lipid solubility of the drug
C. pKa value of the drug
D. pH of the medium
64. Active transport of a substance across biological membrane has the following
characteristics except
A. It is specific
B. It is pH dependent +
C. It is saturable
65. Bioavailability differences among oral formulations of a drug are most likely to occur if
the drug
A. Is freely water soluble
B. Is completely absorbed
C. Is incompletely absorbed +
D. Undergoes little first-pass metabolism

66. An antagonist has


(a) Intrinsic activity and no affinity
(b) Only intrinsic activity and no affinity
(c) No intrinsic activity and no affinity
(d) Affinity same as agonist and devoid of intrinsic activity +

67. The most important factor governing absorption of a drug from intact skin is
A. Molecular weight of the drug
B. Site of application
C. Lipid solubility of the drug +
D. Nature of the base used in the formulation

68. Redistribution is a feature of


A. Highly plasma protein bound drugs
B. Depot preparations
C. Poorly lipid soluble drugs
D. Highly lipid soluble drugs +
69. Weakly acidic drugs
A. Are bound primarily to a1 acid glycoprotein in plasma
B. Are excreted faster in alkaline urine +
C. Are highly ionized in the gastric juice
D. Do not cross blood—brain barrier
70. High plasma protein binding
A. Increases the volume of distribution of the drug
B. Facilitates glomerular filtrtion of the drug
B. Minimizes drug interactions
D. Generally makes the drug long acting +

71. Biotransformation of drugs is primarily directed to


A. Activate the drug
B. Inactivate the drug
C. Convert lipid soluble drugs into nonlipid soluble metabolites+
D. Convert nonlipid soluble drugs into lipid soluble metabolites

72. drug may be best administered by:


(a) Oral route
(b) Inhalation +
(c) Sublingual route
(d) Intrathecal route

73. Which of the following cytochrome P450 isoenzymes is involved in the metabolism of a
large number of drugs in human beings and has been implicated in some dangerous drug
interactions:
A. CYP 3A4 +
B. CYP 2C9
C. CYP 2E1
D. CYP 1A2

74. The most commonly occurring conjugation reaction for drugs and their metabolites is
A. Glucuronidation +
B. Acetylation
C. Methylation
D. Glutathione conjugation
75. G-protein coupled receptors span the plasma membrane as a bundle of _____ alpha
helices
A. One
B. Three
C. Seven +
D. Ten
76. Which of the following drug metabolizing reactions is entirely nonmicrosomal
A. Glucuronide conjugation
B. Acetylation +
C. Oxidation
D. Reduction
77. Induction of drug metabolizing enzymes involves
A. A conformational change in the enzyme protein to favor binding of substrate molecules
B. Expression of enzyme molecules on the surface of hepatocytes
C. Enhanced transport of substrate molecules into hepatocytes
D. Increased synthesis of enzyme protein +
78. Drugs which undergo high degree of First-pass metabolism in liver
A. Have oral bioavailability
B. Are excreted primarily in bile
C. Are contraindicated in liver disease
D. Exhibit zero order kinetics of elimination +
79. Glomerular filtration of a drug is affected by its
A. Lipid solubility
B. Plasma protein binding +
C. Degree of ionization
D. Rate of tubular secretion
80. If a drug undergoes net tubular secretion, its renal clearance will be
A. More than the glomerular filtration rate +
B. Equal to the glomerular filtration rate
C. Less than the glomerular filtration rate
81. Which of the following is not a primary/ fundamental, but a derived pharmacokinetic
parameter
A. Bio-availability
B. Volume of distribution
C. Clearance
D. Plasma half life +
82. If a drug is eliminated by first order Kinetics
A. constant amount of the drug will be eliminated per unit time
B. Its clearance value will remain constant +
C. Its elimination half-life will increase with dose
D. It will be completely eliminated from the body in 2 x half-life period

83. If a drug has a constant bio-availability and first order elimination, its maintenance
dose rate will be directly proportional to its
A. Volume of distribution
B. Plasma protein binding
C. Lipid solubility
D. Total body clearance +

84. The following dose of a drug is governed by its


A. Aqueous diffusion
B. Aqueous hydrolysis
C. Lipid diffusion +
D. Pinocytosis or endocytosis

85. Monitoring plasma drug concentration is useful while using


A. Antihypertensive drugs
B. Levodopa
C. Lithium carbonate +
D. MAO inhibitors
86. Microsomal enzyme induction has one of the following features
A. Takes about one week to develop
B. Results in increased affinity of the enzyme for the substrate +
C. It is irreversible
D. Can be used to treat acute drug poisonings
87. Which of the following is a competitive type of enzyme inhibitor
A. Acetazolamide
B. Disulfiram
C. Physostigmine +
D. Theophylline
88. What is true in relation to drug receptors
A. All drugs act through specific receptors
B. All drug receptors are located on the surface of the target cells
C. Agonists induce a conformational change in the receptor +
D. Partial agonists have low affinity for the Receptor

89. A partial agonist can antagonize the effects of a full agonist because it has
A. High affinity but low intrinsic activity +
B. Low affinity but high intrinsic activity
C. No affinity and low intrinsic activity
D. High affinity but no intrinsic activity

90. Receptor agonists possess


A. Result in increased smooth endoplasmic reticulum
B. Result in increased rough endoplasmic reticulum
C. Result in decreased enzymes in the soluble cytoplasmic fraction +
E. Require 3–4 months to reach completion

91. Agonists affect the receptor molecule in the following manner

A. Alter its amino acid sequence

B. Denature the receptor protein


C. Alter its folding or alignment of subunits +

D. Induce covalent bond formation

92. Receptors perform the following function/Functions

A. Ligand recognition

B. Signal transduction

C. Both ligand recognition and signal transduction n+

D. Disposal of agonists and antagonists

93. Which of the following receptor types has 7 helical membrane, spanning amino acid
segments with 3 extracellular and 3 intracellular loops

A. Tyrosine protein kinase receptor

B. Gene expression regulating receptor

C. Intrinsic ion channel containing receptor

D. G protein coupled receptor +

94. Which of the following is a G protein coupled receptor

A. Muscarinic cholinergic receptor +

B. Nicotinic cholinergic receptor

C. Glucocorticoid receptor

D. Insulin receptor

95. Placebo effects result presumably from the

A. Physician-patient relationship

B. Mental set up imparted by the therapeutic settings

C. Mental set up imparted by the physician


D. All of the above +

96. All of the following sub serve as intracellular second messengers in receptor mediated
signal transduction except

A. Cyclic AMP

B. Inositol trisphosphate

C. Diacyl glycerol

D. G protein +

97. A receptor which itself has enzymatic property is

A. Insulin receptor +

B. Progesterone receptor

C. Thyroxine receptor

D. Glucagon receptor

98. Down regulation of receptors can occur as a consequence of

A. Continuous use of agonists +

B. Continuous use of antagonists

C. Chronic use of CNS depressants

D. Denervation

99. When therapeutic effects decline both below and above a narrow range of doses, a drug
is said to exhibit

A. Ceiling effect

B. Desensitization

C. Therapeutic window phenomenon +

D. Nonreceptor mediated action


100. ‘Drug efficacy’ refers to

A. The range of diseases in which the drug is beneficial

B. The maximal intensity of response that can be produced by the drug +

C. The therapeutic dose range of the drug

D. The therapeutic index of the drug

101. Tachyphylaxis is

A. A drug interaction between two similar types of drugs

B. Rapidly developing tolerance+

C. A synergism between two types of drugs

D. None of the above

102.Drug A in a dose of 10 mg produces same response as with 100 mg of drug B

A. Drug A is 10 times more potent than drug B +

B. Drug B is 10 times more potent than drug A

C. Drug A is 10 times more efficacious than drug B

D. Both are equally potent

103. Teratogenicity is

A. The acute reaction to drugs

B. Intolerance to drugs

C. Tumour forming action of the drugs

D. Malformation of the fetus +

104. The chances of foetal malformation with a teratogenic drug is maximum

A. During first trimester of pregnancy +

B. During second trimester of pregnancy


C. During third trimester of pregnancy

D. When given just prior to the labor

105. Phocomelia is a known teratogenic effect of

A. Anticancer drugs

B. Antiviral drugs

C. Antiepileptic drugs

D. Thalidomide +

106. Which of the following drugs are known to cause toxic cataract?

A. Chloroquine

B. Ergot

C. Phenothiazine

D. All of the above +

107. The passage of drugs into the foetus from placenta

A. Is by active transport

B. Is by passive diffusion +

C. Is by carrier mediated transport

D. By any of the above methods

108. Idiosyncrasy reaction of a drug is

A. A type of hypersensitivity reaction

B. A type of drug antagonism

C. Unpredictable, inherent, qualitatively abnormal reaction of a drug +

D. Quantitatively exaggerated response


109. Two drugs having similar effects are termed as

A. Heterergic drugs

B. Isomer drugs +

C. Homergic drugs

D. Antagonistic drugs

110. AntagonisticAntagonism between barbiturate and amphetamine is termed as

A. Non-competitive antagonism

B. Physiological antagonism +

C. Competitive antagonism

D. Synergism

111. Which one of the following is an example of physical or chemical interaction?

A. Warfarin plus salicylates–prolongation of anticoagulant effect and bleeding tendency

B. Methotrexate plus sulfonamides–pancytopenia

C. Heparin plus protamine–reversal of heparin effect +

D. Sulfonamides plus salicylate–sulfa toxicity

112. First order kinetics of the drugs is called when

A. A constant fraction of the drug is removed in per unit time +

B. A constant amount of the drug is removed in per unit time

C. Total amount of the drug is removed in one hour

D. Total amount of the drug is removed in first passage through the kidneys

113. For the drugs which follow first order kinetics, after 4 half life the elimination will be
approximately

A. 40%
B. 94% +

C. 25%

D. 4%

114. Passive diffusion of a drug across cell membrane is low when its molecular mass is
greater than

A. 50–100 Da

B. 100–200 Da +

C. 200–300 Da

D 300–400 Da

115. Passage of drug across most capillary endothelial membranes is dependent upon

A. Lipid solubility +

B. pH gradient

C. Blood flow

D. All of the above

116. Following receptors are membrane proteins, except

A. Receptors for fast neurotransmitters, coupled directly to an ion channel

B. Receptors for many hormones and slow transmitters, coupled to effector system

C. Receptor for insulin and various growth factors, which are directly linked to tyrosine kinase

D. Receptors for steroid hormone +

117. pH difference between extracellular and intracellular fluid is

A. Nil

B. 0.2

C. 0.4 +
D. 0.8
DRUGS ACTING ON CENTRAL NERVOUS SYSTEM

1. Substance secreted into the blood by a neuron is


A. Neurohormone +
B. Neuromodulator
C. Neuromediator
D. Neurotransmitter
2. Which of the following is a leukotriene receptor blocker?
A. Alprostadil
B. Aspirin
C. Ibuprofen
D. Zafirlukast +
3. The primary endogenous substrate for nitric oxide synthase is
A. Acetylcholine
B. Angiotensinogen
C. Arginine +
D. Citruline
4. Which one of the following chemicals does not satisfy the criteria for a
neurotransmitter role in the CNS?
A. Acetylcholine
B. Dopamine
C. Glycine
D. Nitric Oxide +
5. Neurotransmitters may
A. Increase chloride conductance to cause inhibition +
B. Increase potassium conductance to cause excitation
C. Increase sodium conductance to cause inhibition
D. Increase calcium conductance to cause inhibition
6. Which of the following chemicals is most likely to function as a neurotransmitter in
hierarchical systems?
A. Dopamine
B. Glutamate +
C. Metenkephalin
D. Norepinephrine
7. Suramin is an antagonist of ________ receptors
A. Purine +
B. Somatostanin
C. Neuropeptide Y
D. Neurotensin
8. Induction of various forms of synaptic plasticity is more closely associated with
_____ receptors
A. AMPA
B. Kainate
C. NMDA +
D. All of the above
9. Which one of the following drugs may increase anticoagulant effects by
displacement of warfarin from plasma protein binding sites and is inactive until
converted in the body to an active metabolite?
A. Buspirone
B. Chloral hydrate +
C. Clorazepate
D. Secobarbital
10. This hypnotic drug facilitates the inhibitory actions of GABA, but it lacks
anticonvulsant or muscle relaxing properties and has minimal effect on sleep
architecture.
A. Buspirone
B. Diazepam
C. Flurazepam
D. Zaleplon +
11. Following is GABAA agonist
A. Muscimol +
B. Baclofen
C. Bicuculline
D. None of the above
12. Chronic use of ethanol is reported to increase
A. Alcohol dehydrogenase
B. Aldehyde dehydrogenase
C. Microsomal ethanol-oxidizing system activity +
D. Monoamine oxidase
13. Following is glycine antagonists
A. Quisqualate
B. Taurine
C. Strychnine +
D. â-alanine
14. Withdrawal of antiseizure drugs can cause increased seizure frequency and
severity. Withdrawal is least likely to be a problem with
A. Clonazepam
B. Diazepam
C. Ethosuximide +
D. Phenobarbital
15. With chronic use in seizure states, the adverse effects of this drug include coarsening
of facial features, hirsutism, gingival hyperplasia, and osteomalacia.
A. Carbamazepine
B. Ethosuximide
C. Gabapentin
D. Phenytoin +
16. The most likely mechanism involved in the antiseizure activity of carbamazepine is
A. Block of sodium ion channels +
B. Block of calcium ion channels
C. Facilitation of GABA actions on chloride ion channels
D. Glutamate receptor antagonism
17. Which one of the following statements
about phenytoin is accurate?
A. Displaces sulfonamides from plasma proteins
B. Drug of choice in myoclonic seizures
C. Half-life is increased if used with phenobarbital
D. Toxicity may occur with only small increments in dose +
18. Following is GABAB agonist
A. Muscimol
B. Baclofen +
C. Picrotoxin
D. Bicuculline
19. Following is GABAA antagonist
A. Muscimol
B. Bicuculline +
C. Strychnine
D. Baclofen
20. The inhalation anaesthetic with the fastest onset of action is
A. Enflurane
B. Isoflurane
C. Nitric oxide
D. Nitrous oxide +
21. Which of the following was the first compound to be identified Pharmacologically
as a transmitter in the CNS?
A. Glycine
B. Glutamate
C. Acetylcholine +
D. Norepinephrine
22. You have a vial containing 4 mL of a 2% solution of lidocaine. How much lidocaine
is present in 1 mL?
A. 2 mg
B. 8 mg
C. 20 mg +
D. 80 mg
23. Respiratory depression following use of this agent may be reversed by
administration of flumazenil
A Desflurane
B. Fentanyl
C. Ketamine
D. Midazolam +
24. Use of this agent is associated with a high incidence of disorientation, sensory and
perceptual illusions, and vivid dreams during recovery from anaesthesia
A. Diazepam
B. Fentanyl
C. Ketamine +
D. Midazolam
25. Epinephrine added to a solution of lidocaine for a peripheral nerve block will
A. Increase the risk of convulsions
B. Increase the duration of anaesthetic action of the local anaesthetic +
C. Both (A) and (B)
D. Neither (A) nor (B)
26. Characteristics of nondepolarizing neuromuscular blockade include which one of
the following?
A. Block of post tetanic potentiation
B. Histamine blocking action
C. Poorly sustained titanic tension +
D. Significant muscle fasciculations during onset of block
27. Which of the following does not cause skeletal muscle contractions or twitching
A. Acetylcholine
B. Nicotine
C. Strychnine
D.Vecuronium +
28. Which one of the following is most effective in the management of malignant
hyperthermia?
A. Baclofen
B. Dantrolene +
C. Haloperidol
D. Succinylcholine
29. Following is a G-protein coupled receptor
A. AMPA
B. Kainate
C. NMDA
D. Metabotropic +
30. Which one of the following drugs has caused hyperkalaemia leading to cardiac
arrest in patients with neurologic disorders?
A. Baclofen
B. Dantrolene
C. Succinylcholine +
D. Tubocurarine
31. Following is an excitatory amino acid receptor antagonist?
A. Phencyclidine +
B. Quisqualate
C. Homocysteate
D. Kainate
32. Which one of the following drugs has spasmolytic activity and could also be used in
the management of seizures caused by overdose of a local anaesthetic?
A. Baclofen
B. Cyclobenzaprine
C. Dantrolene
D. Diazepam +
33. Which one of the following drugs given preoperatively will prevent postoperative
pain caused by succinylcholine?
A. Baclofen
B. Dantrolene
C. Diazepam
D. Tubocurarine +
34.The major reason why carbidopa is of value in parkinsonism is that the compound
A. Crosses the blood-brain barrier
B. Inhibits monoamine oxidase type A
C. Inhibits aromatic L-amino acid decarboxylase +
D. Is converted to the false neurotransmitter carbidopamine
35.Great caution must be exercised in the use of this drug (or drugs from the same
class) in parkinsonian patients who have prostatic hypertrophy or obstructive
gastrointestinal disease
A. Benztropine +
B. Carbidopa
C. Levodopa
D. Ropinirole
36. Which of the following statements about pramipexole is accurate?
A. Activates dopamine D2 receptors
B. Commonly a first-line therapy for Parkinson’s disease
C. May cause postural hypotension
D. All of the above +
37. Which one of the following statements about the action of phenothiazines is
accurate?
A. They activate muscarinic receptors
B. They are antiemetic +
C. They decrease serum prolactin levels
D. They elevate the seizure threshold
38. Choose the correct statement from the following?
A. Muscimol is GABAB selective
B. Bicuculline is GABAA agonist
C. Picrotoxin blocks chloride channels associated with GABAA receptors +
D. Baclofen is GABAA agonist
39.Following is the main inhibitory transmitter in the brain
A. Dopamine
B. Norepinephrine
C. Glycine
D. GABA +
40. Established clinical uses of this drug include enuresis and chronic pain
A. Bupropion
B. Fluvoxamine
C. Imipramine +
D. Phenelzine
41. Which one of the following drugs is most likely to increase plasma levels of
alprazolam, theophylline, and warfarin
A. Desipramine
B. Fluvoxamine +
C. Imipramine
D. Nefazodone
42. Which one of the following actions of opioid analgesics is mediated via activation of
kappa receptors?
A. Cerebral Vascular dilation
B. Decreased uterine tone
C. Euphoria
D. Sedation +
43. ______ antagonists are known to attenuate some of the actions of alcohol.
A. GABAA +
B. GABAB
C. NMDA
D. Glycine
44. Which one of the following effects is unlikely to occur during treatment with
amitriptyline?
A. Alpha adrenoceptor blockade
B. Elevation of the seizure threshold +
C. Mydriasis
D. Sedation
E. Urinary retention
45.Which one of the following drugs is most likely to be of value in obsessive compulsive
disorders (OCD)
A. Amitriptyline
B. Bupropion
C. Clomipramine +
D. Desipramine
46. Compared with other antidepressant drugs, mirtazapine has the distinctive ability to
act as an antagonist of
A. Alpha2 adrenoceptors +
B. Beta adrenoceptors
C. D2 receptors
D. NE transporters
47.Which one of the following signs or symptoms is likely to occur with marijuana
A. Bradycardia
B. Conjunctival reddening +
C. Hypertension
D. Increased psychomotor performance
48. The antihypertensive effects of captopril can be antagonized (reduced) by
A. Angiotensin II receptor blockers
B. Loop diuretics
C. NSAIDS +
D. Sulfonylurea hypoglycemics
49. The primary site of action of tyramine is
A. Ganglionic receptors
B. Gut and liver catechol–O–methyltransferase
C. Postganglionic sympathetic nerve terminals +
D. Preganglionic sympathetic nerve terminals
50. Which one of the following drugs has been used in ophthalmology, but causes
mydriasis and cycloplegia lasting more than 24 hours?
A. Atropine +
B. Echothiophate
C. Edrophonium
D. Ephedrine
E. Tropicamide
51. The antihypertensive drug most likely to aggravate angina pectoris is
A. Clonidine
B. Guanethidine
C. Hydralazine +
D. Methyldopa
52. This agent is currently a first-choice drug in the management of absence seizures as
well as partial, primary generalized, and tonic-colonic seizures.
A. Carbamazepine
B. Clonazepam
C. Ethosuximide
D. Valproic acid +
53.The phenothiazines have a variety of actions at different receptor types. However,
they do not appear to interact with receptors for
A. Dopamine
B. Histamine
C. Nicotine +
D. Norepinephrine
54. A psychiatric patient taking medications develops a tremor, thyroid enlargement,
and leucocytosis. The drug he is taking is most likely to be
A. Clomipramine
B. Haloperidol
C. Imipramine
D. Lithium +
55. Diuretics that increase the delivery of poorly absorbed solute to the thick ascending
limb of the nephron include
A. Furosemide
B. Indapamide
C. Mannitol +
D. Spironolactone
56. Following is used in methanol poisoning
A. Disulfiram
B. Naltrexone
C. Ethanol +
D. None of the above
57. A drug that is used in the treatment of parkinsonism and will also attenuate
reversible extrapyramidal side effects of neuroleptics is
A. Amantadine
B. Levodopa
C. Pergolide
D. Trihexyphenidyl +
58.Which of the following is a common effect of muscarinic stimulant drugs?
A. Decreased peristalsis
B. Decreased secretion by salivary glands
C. Hypertension
D. Miosis +
59. Four stages of general anaesthesia are distinctly seen with the use of
A. Halothane
B. Diethyl ether +
C. Nitrous oxide
D. Enflurane
60. Infusion of phentolamine into the cerebrospinal fluid of an experimental animal will
prevent the blood pressure – lowering action of
A. Clonidine +
B. Enalapril
C. Guanethidine
D. Reserpine
61. A drug suitable for producing a brief (5 to 15 minute) increase in cardiac vagal tone
is
A. Digoxin
B. Edrophonium +
C. Ergotamine
D. Pralidoxime
62. Propranolol and hydralazine have which of the following effects in common?
A. Decreased cardiac force
B. Decreased cardiac output
C. Decreased mean arterial blood pressure +
D. Increased systemic vascular resistance
63. Toxicities of local anaesthetics do not include
A. Cardiovascular arrhythmias and collapse (bupivacaine)
B. Convulsions (lidocaine)
C. Dizziness, sedation (lidocaine)
D. Hypertensive emergencies, strokes (procaine) +
64. Benzodiazepines are least effective in
A. Alcohol withdrawal syndromes
B Balanced anaesthesia regimens
C. Initial management of phencyclidine overdose
D. Obsessive – compulsive disorders +
65. Which one of the following drugs exerts its anticonvulsant effects by blocking
sodium channels in neuronal membranes
A. Acetazolamide
B. Carbamazepine +
C. Diazepam
D. Gabapentin
66. Cocaine intoxication has become a common problem in hospital emergency rooms.
Which one of the following drugs is not likely to be of any value in the management of
cocaine overdose?
A. Dantrolene
B. Diazepam
C. Lidocaine
D. Naltrexone +
67. Which one of the following agents used in hypertension is a prodrug that is
converted to its active form in the brain?
A. Clonidine
B. Doxazosin
C. Methyldopa +
D. Nitroprusside
68. The introduction of this drug may represent a novel approach to the treatment of
major depressive disorders since it appears to act as an antagonist at alpha2
adrenoceptors in the CNS
A. Amoxapine
B. Bupropion
C. Citalopram
D. Mirtazapine +
69. Which one of the following pairs of drugs: indication is accurate
A. Amphetamine: Alzheimer’s dementia
B. Bupropion: Acute anxiety
C. Fluoxetine: Insomnia
D. Ropinirole: Parkinson’s disease +
70. Following general anaesthetic is not metabolized
A. Halothane
B. Ether
C. Isoflurane
D. Nitrous oxide +
71. The appropriate chemical classification for meperidine is
A. Phenylpropylamines
B. Piperazines
C. 4-phenylpiperidines +
72. Which of the following neuromuscular blocking agents can cause muscarinic
responses such as bradycardia and increased glandular secretions?
A. Tubocurarine
B. Succinycholine
C. Pancuronium
D. Decamethonium +
73. Which of the following drugs is a volatile substance that is administered by
inhalation?
A. Thiopental
B. Halothane +
C. Alprazolam
D. Buspirone
74. The brief duration of action of an ultrashort acting barbiturate is due to a
A. Slow rate of metabolism in the liver
B. Low lipid solubility, resulting in a minimal concentration in the brain
C. High degree of binding to plasma proteins
D. Slow rate of excretion by the kidneys +
75. Which of the following agents would not be an alternative to Phenobarbital in the
treatment of partial seizures?
A. Trimethadione +
B. Gabapentin
C. Felbamate
D. Lamotrigine
E. None of the above
76. Which of the following diuretics is most similar in chemical structure to the
antihypertensive agent diazoxide?
A. Furosemide
B. Spironolactone
C. Mannitol
D. Chlorothiazide +
78. Following is an example of ideal anaesthetic
A. Ether
B. Halothane
C. Thiopental
D. None of the above +
79. An action common to most general anaesthetic
A. Increase in the cellular threshold of firing +
B. Potentiation of spontaneous and evoked activity of neurons
C. Inhibition of effects of glutamate
D. None of the above
80. All of the following adverse effects are associated with the use of levodopa except
A. Sialorrhea
B. Orthostatic hypotension
C. Delusions, confusion, and depression
D. Livedo reticularis +
81. The activity of which of the following drugs is dependent on a p-phenyl – N –
alkylpiperidine moiety
A. Phenobarbital
B. Chlorpromazine
C. Diazepam
D. Meperidine +
82. Parenteral calcium is used as an antidote for which of the following situations
A. Verapamil overdoses
B. Hyperkalaemia
C. Cocaine intoxication
D. Verapamil overdoses and hyperkalaemia +
83.Which of the following benzodiazepines is expected to cause the least amount of
adverse effects in the elderly?
A. Chlordiazepoxide
B. Diazepam
C. Flurazepam
D. Oxazepam +
84. The use of morphine in the patient who has had a myocardial infarction (MI)
centres around three distinct pharmacologic properties. Which of the following choices
includes these properties
A. Relief of pain, relief of anxiety, and increased oxygen supply
B. Relief of anxiety, after load reduction, increased preload
C. Relief of anxiety, preload reduction, and relief of pain +
D. Vagomimetic effect, relief of anxiety, respiratory depression
85. Ophthalmic agents contraindicated in glaucoma patients include which of the
following substances?
A. Antioxidants
B. Antipruritic
C. Decongestants +
D. Emollients
86.. Which of the following drugs is excepted to cause anticholinergic adverse effects in
the elderly?
A. Propoxyphene
B. Ciprofloxacin
C. Amitriptyline +
D. Propranolol
87.Reflex tachycardia, headache, and postural hypotension are adverse effects that limit
the use of which of the following antihypertensive agents?
A. Prazosin
B. Captopril
C. Methyldopa
D. Hydralazine +
88. Following is inverse agonist of benzodiazepine receptors
A. Flumazenil
B. β-Carbolines +
C. Chlordiazepoxide
D. Glutethimide
89. Which anticonvulsant drug requires therapeutic monitoring of Phenobarbital serum
levels as well as its own serum levels
A. Phenytoin
B. Primidone +
C. Clonazepam
D. Ethotoin
90. Zolpidem belongs to following class of drugs
A. Benzodiazepines
B. Carbamates
C. Imidazopyridines +
D. None of the above
91. What are the most common adverse effects of anticonvulsive drugs?
A. headache and dizziness
B. Gastrointestinal symptoms
C. Alternation of cognition and mentation
D. All of the above +
92. What are the important elements in considering treatment strategies for Parkinson’s
patients?
A. Age
B. Clinical presentation of disease
C. Sex
D. Etiology of the disease +
93. Which of the following anticonvulsants is contraindicated in patients with a
history of hypersensitivity to tricyclic antidepressants?
A. Phenytoin
B. Ethosuximide
C. Acetazolamide
D. Carbamazepine +
94. Which of the following agents should not be used concurrently with levodopa?
A. Diphenhydramine
B. Benztropine
C. Amantadine
D. Monoamine oxidase (MAO) inhibitors +
95. A patient with major depression should receive antidepressant therapy for at least
A. 2 weeks
B. 6 weeks
C. 2 months
D. 6 months +
96. Which of the following atypical antipsychotics would be the least sedating?
A. Quetiapine
B. Risperidone +
C. Olanzapine
D. Clozapine
97. Which of the following mood stabilizers would be most appropriate in a patient with
liver disease?
A. Lithium +
B. Valproic acid
C. Carbamazepine
D. None of the above
98. Ethinamate is _________ derivative
A. Phenothiazine
B. Urethane +
C. Piperidine
D. Tertiary alcohol
99. Which of the following factors may reduce lithium concentrations?
A. Dehydration
B. Pregnancy +
C. Reduced sodium intake
D. Nonsteroidal anti – inflammatory drugs
100. Morphine stimulates
A. Biliary and pancreatic secretions
B. Nonpropulsive rhythmic contractions of small intestine of man +
C. Propulsive contractions in small intestine of man
D. Propulsive peristaltic waves in colon
101. A drug that produces increased contraction of the sphincter iridis by local
application is a
A. Parasympathimimetic drug +
B. Parasympatholytic drug
C. Sympathomimetic drug
D. Sympatholytic drug
102. Which of the following should not be administered to a patient with myasthenia
gravis
A. Prostigmine
B. Digitalis
C. Atropine
D. Curare +
103. The toxicity of methanol is due to its conversion in the body to
A. Acetaldehyde
B. Formic acid and formaldehyde +
C. Ethyl alcohol
D. Carbonic acid
104. One of epinephrine’s actions on the heart is
A. Acceleration due to depression of the vagus
B. Increase in refractory period
C. Depression of the SA node
D. Increase in the concentration of the enzyme phosphorylase A +
105. Epinephrine HCl has little or no effect on
A. Unbroken skin
B. Conjunctiva
C. Precapillary sphincter
D. Nasal mucosa +
106. Acetylcholine has both muscarinic and nicotinic actions. The muscarinic action can
be blocked by
A. Epinephrine
B. Atropine +
C. Nicotine
D. Curare
107. Which of the following is the fastest acting anticoagulant
A. Warfarin (Coumadin)
B. Heparin +
C. Ouabain
D. Protamine sulfate
108. Phencyclidine is a non-competitive antagonist of
A. Kainate
B. AMPA
C. NMDA +
D. All of the above
109. Parkinsonism is probably due to
A. Too little dopamine in the brain +
B. Too little levodopa in the brain
C. Too little acetylcholine in the brain
D. Too much levodopa in the brain
110. A prominent toxic effect of local anaesthetics is
A. CNS stimulation +
B. CNS depression
C. Tachycardia
D. Local ischemia
111. Which of the following could be used as an antidote for curare poisoning
A. Neostigmine +
B. Atropine
C. Homatropine
D. Hexamethonium
112. Cigarette smoking increases the side effects of
A. Narcotic analgesics
B. Analeptics
C. Antidepressants
D. Oral contraceptives +
113. Which of the following may increase seizure activity in epileptic patients
A. Ethotoin
B. Phenobarbital
C. Trihexyphenidyl
D. Amantadine +
114. Trazodone (Desyrel) is used primarily as a (an)
A. Analgesic
B. Cardiotonic
C. Tranquilizer
D. Antidepressant +
115. Propofol
A. Enhances the neuromuscular blockade of tubocurarine
B. Causes marked postoperative confusion
C. Can produce involuntary movements during its use +
D. Has a delayed recovery after induction
116. A common side effect of ephedrine is
A. Rashes
B. Nervousness +
C. Blood dyscrasia
D. Drowsiness
117. Thiazide diuretics may produce an increase in blood levels of uric acid and
A. Potassium
B. BUN
C. Urea
D. Glucose +
118. Symptoms of schizophrenia are produced by following agents, except
A. Amphetamine
B. Apomorphine
C. Bromocriptine
D. Reserpine +
119. Select the anticholinesterase drug that is being used to afford symptomatic
improvement in Alzheimer’s disease
A. Echothiophate
B. Tacrine +
C. Demecarium
D. Ambenonium
120. Pilocarpine is used in
A. Glaucoma +
B. Paralytic ileus
C. Urinary retention
D. All of the above
121. Select the longer acting ocular betablocker
A. Timolol
B. Betaxolol
C. Carteolol
D. Levobunolol +
122.Which of the following is a prodrug of adrenaline used topically in glaucoma
A. Phenylephrine
B. Dipivefrine +
C. Phenylpropanolamine
D. Dorzolamide
123. Which of the following diuretics is most effective in acute congestive glaucoma
A. Indapamide
B. Amiloride
C. Mannitol +
D. Furosemide
124. Local anaesthetics block nerve conduction by
A. Blocking all cation channels in the neuronal membrane
B. Hyperpolarizing the neuronal membrane
C. Interfering with depolarization of the neuronal membrane +
D. Both (b) and (c)
125. Which of the following diseases is worsened by propranolol
A. Glaucoma
B. Raynaud’s disease +
C. Benign prostatic hypertrophy
D. Parkinsonism
126. Diazepam is used as a muscle relaxant for
A. Deep intra-abdominal operation
B. Tracheal intubation
C. Tetanus +
D. Diagnosis of myasthenia gravis
127. Which sensation is blocked first by low concentrations of a local anaesthetic
A. Temperature
B. Pain +
C. Touch
D. Deep pressure
128. Following are the MAO-inhibitors, except
A.Tranylcypromine
B. Nomifensine +
C. Moclobemide
D. Isocarboxid
129. Which of the following is a poor surface anaesthetic
A. Procaine +
B. Lidocaine
C. Tetracaine
D. Benoxinate
130. The local anaesthetic having high cardiotoxic and arrhythmogenic potential is
(a) Lidocaine
(b) Procaine
(c) Bupivacine +
(d) Chloroprocaine
131. The calcium channel blocker used for prophylaxis of migraine but not for angina
pectoris is
A. Verapamil
B. Diltiazem
C. Flunarizine +
D. Amlodipine
132. In which of the following techniques the concentration of the local anaesthetic
required is the lowest
A. Infiltration anaesthesia +
B. Field block anaesthesia
C. Nerve block anaesthesia
D. Spinal anaesthesia
133. Captopril produces greater fall in blood pressure in
A. Diuretic treated patients +
B. Patients having low plasma rennin activity
C. Sodium replete normotensive individuals
D. Untreated CHF patients
134. Enalapril differs from captopril in that
A. It blocks angiotensin II receptors
B. It does not produce cough as a side effect
C. It is less liable to cause abrupt first dose hypotension +
D. It has a shorter duration of action
135. Spinal anaesthesia is not suitable for
A. Vaginal delivery
B. Lower segment caesarean section
C. Prostatectomy
D. Operations on mentally ill patients +
136. Following is a MAO-inhibitor and also has anti-tubercular activity
A. Iproniazid +
B. Phenelzine
C. Moclobemide
D Isocarboxid
137. Which of the following is true of nitrous oxide?
A. It irritates the respiratory mucosa
B. It has poor analgesic action
C. It is primarily used as a carrier and adjuvant to other anaesthetics +
D. It frequently induces post anaesthetic nausea and retching
138. The factor that effects rate of elimination of general anaesthetics is
A. Pulmonary ventilation
B. Blood flow
C. Solubility in blood and tissue
D. All of the above +
139. Which of the following is a sign of deep anaesthesia?
A. Appearance of tears in eyes
B. Resistance to passive inflation of lungs
C. Fall in blood pressure +
D. Patient makes swallowing movements
140. Which inhalational general anaesthetic is metabolized in the body to a significant
extent
A. Sevoflurane
B. Isoflurane
C. Ether
D. Halothane +
141. The drug/drugs used mainly for induction
of general anaesthesia is/are
A. Thiopentone sodium +
B. Fentanyl + droperidol
C. Ketamine
D. All of the above
142. Following agent potentiates actions of
GABA
A. Volatile anaesthetics
B. Barbiturates +
C. Propofol
D. All of the above
143. Which of the following drugs is routinely used in preanesthetic medication for
prolonged operations
A. Atropine
B. Morphine
C. Promethazine
D. Ranitidine +
144. Hypnotic benzodiazepines increase the period of time spent in the following stage
of sleep
A. Stage II +
B. Stage III
C. Stage IV
D. REM stage
145. The primary mechanism of action of benzodiazepines is
A. Dopamine antagonism
B. Adenosine antagonism
C. Opening of neuronal chloride channels
D. Facilitation of GABA–mediated chloride influx +
146. The following drug is used to reverse the CNS depression produced by diazepam
A. Nikethamide
B. Doxapram
C. Physostigmine
D. Flumazenil +
147. Which of the following is not a CNS depressant but increases the tendency to
fall asleep at night?
A. Pyridoxine
B. Methaqualone
C. Melatonin +
D. Ethanol
148. The following antiepileptic drug is also effective in manic-depressive illness
A. Ethosuccimide
B. Primidone
C. Phenobarbitone
D. Carbamazepine +
149. The preferred drug for status epilepticus is
A. Intravenous diazepam +
B. Intravenous phenytoin sodium
C. Intramuscular phenobarbitone
D. Rectal paraldehyde
150. The most effective single drug in parkinsonism is
A. Bromocriptine
B. Selegiline
C. Levodopa D. +
D. Biperiden
DRUGS ACTING ON AUTONOMOUS NERVOUS SYSTEM

1. In the autonomic regulation of blood pressure


A. Cardiac output is maintained constant at the expense of other hemodynamic variables
B. Elevation of blood pressure results in elevated aldosterone secretion
C. Baroreceptor nerve fibres decrease firing rate when arterial pressure increases
D. Stroke volume and mean arterial blood pressure are the primary direct determinants of
cardiac output +

2. The autonomic nervous system is also known as


A. Visceral
B. Vegetative
C. Involuntary nervous system +
D. All of the above
3. Full activation of the sympathetic nervous system, as in maximal exercise, can
produce all of the following responses except
A. Bronchial relaxation
B. Decreases intestinal motility
C. Increased renal blood flow +
D. Mydriasis
4. The neurotransmitter agent that is normally released in the sinoatrial node of the
heart in response to a blood pressure increase is
A. Acetylcholine +
B. Dopamine
C. Epinephrine
D. Glutamate
5. A direct-acting cholinomimetic that is lipid-soluble and often used in the treatment of
glaucoma is
A. Acetylcholine
B. Bethanechol
C. Physostigmine
D. Pilocarpine +
6.Which of the following agents is a prodrug that is much less toxic in mammals than in
insects
A. Acetylcholine
B. Bethanechol
C. Physostigmine C +
D. Pilocarpine
7. Atropine overdose may cause which one of the following
A. Gastrointestinal smooth muscle cramping
B. Increased cardiac rate +
C. Increased gastric secretion
D. Pupillary constriction
8.The synaptic pre-ganglionic neurons have their cell bodies in the lateral horn of the
grey matter of the
A.Thoracic segment only
B. Lumbar segment only
C. Both (a) and (b) +
D. None of the above
9. Which one of the following can be blocked by atropine?
A. Decreased blood pressure caused by hexamethonium
B. Increased blood pressure caused by nicotine
C. Increased skeleton muscle strength caused by neostigmine
D. Tachycardia caused by infusion of acetylcholine +
10. ‘Fight or flight’ state can be described as
A.Sympathetic active; para-sympathetic quiescent +
B. Sympathetic active; para-sympathetic active
C. Sympathetic quiescent; para-sympathetic active
11. In male sex organs _______ system is responsible for erection.
A. Sympathetic
B. Parasympathetic +
C. Norepinephrine
D. None of the above
12. Rest and digest’ state can be described as
A. Sympathetic active; para-sympathetic quiescent
B. Sympathetic active; para-sympathetic active
C. Sympathetic quiescent; para-sympathetic active +
13. Reserpine, an alkaloid derived from the root of Rauwolfia serpentine
A. Can be used to control hyperglycaemia
B. Can cause severe depression of mood +
C. Can decrease gastrointestinal secretion and motility
D. Has no cardiac effects
14. Following enzyme is involved in the synthesis of acetylcholine

A. Choline acetyl transferase +


B. Acetyl cholinesterase
C. Both (a) and (b)
D. None of the above

15. Following is an antagonist of ganglion type nicotinic receptors


A. Tubercurarine
B. αbungarotoxin
C. Trimethaphan +
D. All of the above
16. Oxytremorine is a selective agonist of muscarinic _______ receptors
A. M1 +
B. M2
C. M3
D. M4
17. A drug that is useful in glaucoma and high-altitude sickness is
A. Acetazolamide +
B. Amiloride
C. Demeclocycline
E. Desmopressin
18. Cromolyn has as its major action
A. Block of calcium channels in lymphocytes
B. Block of mediator release from mast cells +
C. Block of phosphodiesterase in mast cells and basophils
D. Smooth muscle relaxation in the bronchi
19. Which one of the following is least likely to be useful in the therapy of hypercalcemia
A. Calcitonin
B. Glucocorticoids
C. Plicamycin
D. Thiazide diuretics +
20. Characteristics of Vitamin D and its metabolites include which one of the following
A. Act to decrease serum levels of calcium
B. Activation of their Vitamin D receptors increases cellular CAMP
C. Calcitriol is the major derivative responsible for increasing intestinal absorption of
phosphate +
D. Metabolites of Vitamin D increase renal excretion of calcium
21. Action of norepinephrine and epinephrine are terminated by
A. Reuptake into nerve terminal
B. Dilution by diffusion and uptake at extraneuronal site
C. Metabolic transformation
D. All of the above +
22. Typical results of beta-receptor activation include which one of the following
A. Hypoglycaemia
B. Lipolysis +
C. Glycogen synthesis
D. Decreased skeletal muscle tremor
23. Drugs that block the α receptor on effector cells at adrenergic nerve endings
A. Antagonize the effects of isoproterenol on the heart rate
B. Antagonize some of the effects of epinephrine on the blood pressure +
C. Antagonize the effects of epinephrine on adenylyl cyclase
D. Cause mydriasis
24. Yohimbine is an antagonist of ______ receptors.
A. α1
B. α2 +
C. both (a) and (b)
D. None of the above
25. Which of the following organs is innervated only by parasympathetic nerves
A. Iris muscles
B. Ciliary muscle +
C. Sweat glands
D. Splenic capsule
26. α1 – receptors are coupled with _______ G protein.
A. Gs
B. Gi
C. Gq +
D. Go
27. Hyoscine differs from atropine in that it
A. Exerts depressant effects on the CNS at relatively low doses +
B. Exerts more potent effects on the heart than on the eye
C. Is longer acting
D. Has weaker antimotion sickness activity
28. Which of the following anticholinergic drugs is primarily used in preanesthetic
medication and during surgery
A. Glycopyrrolate +
B. Pipenzolate methyl bromide
C. Isopropamide
D. Dicyclomine
29. Glycopyrrolate is the preferred antimuscarinic drug for use before and during
surgery because
A. It is potent and fast acting
B. It has no central action
C. It has antisecretory and vagolytic actions
D. All of the above +
30. The cardiac muscarinic receptors
A. Are of the M1 subtype
B. Are of the M2 subtype +
C. Are selectively blocked by pirenzepine
D. Function through the PIP2 → IP3/DAG pathway

31. The most suitable mydriatic for a patient of corneal ulcer is


A. Atropine sulphate +
B. Homatropine
C. Cyclopentolate
D. Tropicamide
32. The most effective antidote for belladonna poisoning is
A. Neostigmine
B. Physostigmine +
C. Pilocarpine
D. Methacholine
33. Atropine is contraindicated in
A. Cyclic AMP
B. Inositol trisphosphate
C. Diacyl glycerol
D. G protein +
34. Select the correct statement from the following about α 1 – adrenergic receptor
agonists
A. Isoproterenol > epinephrine ≥ norepinephrine +
B. Epinephrine > isoproterenol > norepinephrine
C. Isoproterenol = epinephrine ≥ norepinephrine
D. Epinephrine ≥ norepinephrine > isoproterenol
35. The most efficacious inhibitor of catecholamine synthesis in the body is
A. α - methyl – p – tyrosine +
B. α methyldopa
C. α methyl - norepinephrine
D. Pyrogallol
36. Adrenaline raises blood glucose level by the following actions except
A. Inducing hepatic glycogenolysis
B. Inhibiting insulin secretion from pancreatic β cells
C. Augmenting glucagons secretion from pancreatic α cells
D. Inhibiting peripheral glucose utilization +
37. Noradrenaline is administered by
A. Subcutaneous injection
B. Intramuscular injection
C. Slow intravenous infusion +
D. All of the above routes
38. Dobutamine differs from dopamine in that
A. It does not activate peripheral dopaminergic receptors +
B. It does not activate adrenergic β receptors
C. It causes pronounced tachycardia
D. It has good blood-brain barrier penetrability
39. Ephedrine is similar to adrenaline in the following feature
A.Potency
B. Inability to penetrate blood-brain barrier
C. Duration of action
D. Producing both α and β adrenergic effects +
40.Choose the correct statement from the following about α1 – adrenergic receptor
agonists
A. Norepinephrine > isoproterenol > epinephrine
B Norepinephrine < epinephrine > isoproterenol
C Epinephrine≥ norepinephrine >> isoproterenol +
D Epinephrine > isoproterenol > norepinephrine
41. A sympathomimetic amine that acts almost exclusively by releasing noradrenaline
from the nerve endings is
A. Ephedrine
B. Dopamine
C. Isoprenaline
D. Tyramine +
42. Low doses of adrenaline dilate the following vascular bed
A. Cutaneous
C. Renal
B. Mucosal
D. Skeletal muscle +
43. α2–adrenergic receptors are associated with following except
A. Increase in phospholipase C activity
B. Increase in potassium channel conductance
C. Decrease in calcium channel conductance
D. Increase in adenylyl cyclase activity +
44. While undergoing a surgical procedure a patient develops hypotension. Which of the
following drugs can be injected intramuscularly to raise his BP
A. Noradrenaline
B. Isoprenaline
C. Mephentermine +
D. Isoxsuprine
45. Vasoconstrictors should not be used in
A. Neurogenic shock
B. Haemorrhagic shock
C. Secondary shock +
D. Hypotension due to spinal anaesthesia
46. The drug which produces vasoconstriction despite being an α adrenergic blocker is
A. Phenoxybenzamine
B. Ergotamine +
C. Dihydroergotoxine
D. Tolazoline
47. Prazosin is an effective antihypertensive while nonselective α adrenergic blockers
are not because
A. It is the only orally active α blocker
B. It improves plasma lipid profile
C. It does not concurrently enhance noradrenaline release +
D. It improves urine flow in males with prostatic hypertrophy
48. Phenylephrine
A. Mimics the transmitter at post-synaptic receptors +
B. Displaces transmitter from axonal terminal
C. Inhibits synthesis of transmitter
D. None of the above
49. A β adrenergic blocker that is primarily eliminated unchanged by renal excretion is
A. Propranolol
B. Metoprolol
C. Alprenolol
D. Atenolol +
50. In a patient of myocardial infarction, βadrenergic blockers are used with the
following aim/aims:
A. To reduce the incidence of reinfarction
B. To prevent cardiac arrhythmias
C. Myocardial salvage
D. All of the above +
51. The β-adrenergic blocker which possesses both β-1 selectivity as well as intrinsic
sympathomimetic activity is
A. Alprenolol
B. Atenolol
C. Acebutolol +
D. Metoprolol
52. Propranolol can be used to allay anxiety associated with
A. Chronic neurotic disorder
B. Schizophrenia
C. Short-term stressful situation +
D. Endogenous depression
53. Propranolol does not block the following action of adrenaline
A. Bronchodilation
B. Lipolysis
C. Muscle tremor
D. Mydriasis +
54. The site of action of d-tubocurarine is
A. Spinal internuncial neurone
B. Motor nerve ending
C. Muscle end-plate +
D. Sodium channels in the muscle fibre
55. The fall in blood pressure caused by d- tubocuranine is due to
A. Reduced venous return.
B. Ganglionic blockade
C. Histamine release
D. All of the above +
56. The neuromuscular blocker having prominent antivagal action is
A. Pancuronium
B. Vecuronium
C. Atracurium
D. Gallamine triethiodide +
57. Neuromuscular blocking drugs do not produce central actions because
A. Nicotinic receptors are not present in the brain
B. They do not cross the blood-brain barrier +
C. They are sequestrated in the periphery by tight binding to the skeletal muscles
D. They do not ionize at the brain pH
58. Pancuronium differs from tubocurarine in that
A. It is a depolarizing blocker
B. Its action is not reversed by neostigmine
C. It can cause rise in BP on rapid I.V. injection +
D. It causes marked histamine release
59. Muscarinic receptors are G-protein coupled receptors, causing
A. Inactivation of phospholipase C
B. Activation of adenylyl cyclase
C. Activation of potassium or inhibition of calcium channels +
D. All of the above
60. Postoperative muscle soreness may be side effect of the following neuromuscular
blocker
A. d-tubocurarine
B. Succinylcholine +
C. Pancuronium
D. Atracurium
61. Following drug enhances the evoked release of acetylcholine
A. 4-aminopyridine +
B. Vesamicol
C. Magnesium ion
D. None of the above
62. Select the ergot compound which is primarily used for dementia
A. Bromocriptine
B. Ergotamine
C. Codergocrine +
D. Methysergide
63. Following muscarinic agonist is susceptible to hydrolysis by cholinesterase
A. Carbachol
B. Methacholine +
C. Bethanechol
D. Muscarine
64. Stimulation of exocrine glands by muscarinic agonist leads to
A. Sweating
B. Salivation
C. Bronchial secretions
D. All of the above +
65. β blockers are contraindicated in
A. Prophylaxis of anxiety states
B. Hyperthyroidism
C. Hypertrophic obstructive cardiomyopathy
D. Peripheral vascular disease +
66. β blockers are better avoided in
A. Bronchial asthma +
B. Pheochromocytoma
C. Myocardial infarction
D. Migraine
67. Preferential beta 1 adrenergic blocking agent is

A. Propranolol
B. Atenolol +
C. Penbutolol
D. Oxprenolol
68. Prazosin
A. Produces preferential alpha1 adrenergic receptor blockade +
B. Has a half-life of 24 hours
C. Can worsen lipid profile in hypertensive patients
D. Does not depress the baroreceptor function
69. The drug that produces neuromuscular blockade by persistent depolarization is
A. D-tubocurarine
B. Gallamine
C. Pancuronium
D. Decamethonium +
70. Agents that block neuromuscular transmission of acetyl choline acts via
A. Inhibiting acetyl choline synthesis
B. Inhibiting acetyl choline release
C. Inhibiting post-synaptic action of acetyl choline
D. All of the above +
71. The skeletal muscle relaxant causing significant release of histamine is
A. Pancuronium
B. Atracurium
C. Gallamine
D. D-tubocurarine +
72. β blockers
A. Relieve symptoms of thyrotoxicosis +
B. Should not be used along with carbimazole
C. Reduce the synthesis of hormones in the gland
D. Should not be given before thyroid surgery
73.The antimuscarinic agent preferred in the management of motion sickness is
A. Atropine methonitrate
B. Scopolamine +
C. Homatropine methyl bromide
D. Ipratropium bromide

74. Side effect which is observed only with the use of β blockers and not with other
sympathetic blocking agents
A. Sedation and drowsiness +
B. Congestive heart failure
C. Bronchoconstriction
D. Reflex tachycardia
75. Following agent inhibits the release of acetyl choline
A. Magnesium ion +
B. Triethylcholine
C. Vesamicol
D. All of the above
76. Which of the following drugs has highest α agonist activity
A. Epinephrine +
B. Norepinephrine
C. Ephedrine
D. Amphetamine
77. Atropine is antagonist to which type of muscarinic
A. M1 +
B. M2
C. M3
D. M4
78. Following are irreversible anti-cholinesterase, except
A. Parathion
B. Pyridostigmine +
C. Dyflos
D. Ecothiopate
79.The ciliary muscle of iris has predominant
A. Sympathetic innervation
B. Parasympathetic innervation +
C. Somatic innervation
D. Mixture of all the above
80. Following is a short-acting anti-cholinesterase
A. Neostigmine
B. Pyridostigmine
C. Edrophonium +
D. Physostigmine

81. The antibiotic that may accentuate the neuromuscular blockade produced by d-
tubocurarine is
(a) Pencillin G

(b) Erythromycin

(c) Streptomycin +

(d) Chloramphenicol
82. Centrally acting skeletal muscle relaxant is
(a) Carisoprodol +
(b) Dantrolene
(c) Gallamine
(d) Succinylcholine
83. The organ extremely sensitive to the action of atropine is
(a) Gastric glands
(b) Salivary glands +
(c) Urinary bladder muscle
(d) Heart
1) Beta blocker which has peripheral vasodilation effect is:
a) atenolol
b) acebutolol
c) metoprolol
d) labitolol
e) +carvedilol

2) T-half of a drug can determine all the following:


a) +closing interval
b) elimination time
c) +therapeutic dosage
d) steady plasma conc.

3) Phototoxicity is commonest with:


a) +doxycycline
b) minocycline
c) prostacycline
d) oxytetracycline

4) Gout is precipitate by all except:


a) frusemide
b) thiazide
c) PYZ
d) +sulfinpyrazone

5) Effect of parasympathetic (muscarinic ) on heart is:


a) refractory period is decresed
b) +conduction is decresed
c) forse of attrial contraction is increased
d) P_R interval is increased

6) Call of the following antibiotics act on cell wall except:


a) ampicilinc
b) +griseofulvin
c) bacitracin
d) none

7) CNS stimulant action of nikethamide has .target cells mainly in:


a) +medulla
b) mid brain
c) pons
d) cerebral cortex

8) Nicotinic receptor are seen in:


a) +sk. Muscle
b) cardiac mus.
c) Visceral smooth mus.
d) Salivary gland

9) Sparfloxacin and tarfinadine can cause


a) nephropathy
b) +ventricular arrhythmia
c) myopathy
d) all

1
10) Dapsone is used in:
a) erythema
b) SLE
c) Lichen planus
d) +Dermatitis herpetiformis

11) Cyclosporine A inhibites:


a) +T cells
b) CD8
c) CD4
d) В cells

12) Increase renal blood flow is caused by:


a) +dopamine
b) dobutamine
c) ]adrenaline
d) Non adrenaline

13) True abt. Lente insulin:


a) +made of 3 parts zinc suspension (amorphorous) and 7 parts zinc suspension (crystalline)
b) duration of action is 36 hrs.
c) clear alkaline solution
d) none

14) Which of the following is fluroquinolone:


a) +pefloxacin
b) cefotaxime
c) urodopenicilline
d) none

15) Metoprolol is the member of:


a) a1 selective group
b) a2- selective group
c) +β1-selective group
d) β2- selective group

16) The principal adverse effect of phentolamine are related to:


a) Gastric secretion
b) Brochi constriction
c) Smooth muscle dilation
d) +Cardiac stimulation

17) Adverse effect of neuromuscular blocking drugs:


a) Hypokalemia
b) +Hyperkalemia
c) Hypomagnemia
d) Hypermagnemia

18) The crossing the blood-brain barrier of trimethaphan is:


a) +Absent
b) Poor
c) Good
d) Excellent

2
19. Direct action of physostigmine on cholinoceptors is:
a. +Absent
b. Poor
c. Good
d. Excellent

19) Lipid soluble agents of anticholiesterase have more marked effect on:
a) Skeletal muscles
b) Nicotinic
c) Eye
d) +CNS

20) High doses of parasympathomimetic drugs may produce:


a) Constriction of arteries and dilation of veins
b) Constriction of veins and dilation of arterials
c) Constriction of veins and arterials
d) +Dilation of veins and arterials

21) Topical instillation of atropine causes:


a) +miosis
b) mydriases
c) abolition of light reflex
d) cycloplegia

22) Remission inducing drug for Rh arthritis include:


a) aspirin
b) +gold thiol
c) ibuprofen
d) all

23) Not used in H. pylori gastritis is a:


a) +cisapride
b) clarithromycin
c) metronidazole
d) colloidal bismuth

24) Protein binding of a drug helps in:


a) distribution
b) +prolonged half life
c) limiting metabolism
d) all

25) Аll the drug can be given in pt. with bronchial asthma except:
a) timolol eyedrop
b) acetaminophen
c) ketorolac
d) +nefopam

26) Opioids act by:


a) cellular hyper polarization
b) reduced neurotransmitter reuptake
c) +reduced level of subst. P
d) evoking glutamate release

3
27) Storage of Fe is:
a) heamoglobine
b) myoglobone
c) +feritinine
d) heamosiderin

28) Thiazides is:


a) Torsemide
b) dichlorphenamide
c) Acetazolamide
d) +Chlorthalidone

29) Isoproterenol has duration of action:


a. 30-60 minutes
b. +60-90 minute
c. 90-120 minute
d. 120-150 minute

30) Pre/post ganglionic fibre ratio of sympathetic nervous system:


a) +1:1-1:2
b) 1:2-1:3
c) 1:3-1:4
d) 1:4 —1:4

31) Muscarinic receptors have been divided into


a) 3 subtypes
b) 4 subtypes
c) +5 subtypes
d) 6 subtypes

32) M-cholinomimetics:
a) Galantamine
b) +Aceclydine
c) Pentamine
d) Carbacholine

33) Response of eyes to the parasympathomimetic drugs:


a) Mydriasis
b) +Miosis
c) Increases of intraocular tension
d) Relax of accommodation

34) Irreversible anticholiesterases:


a) Physostigmine
b) +Phosphacol
c) Neostigmine
d) Edrophonium

35) Which of the following is an osmotic diuretics:


a) +urea
b) metolazone
c) bumetanide
d) d-acetazolamide

4
36) Therapeutic index of a drag is a indicator of:
a) potency
b) safety
c) +toxicity
d) efficacy

37) Tachyphylaxis is a:
a) +acute tolerance
b) chronic tolerance
c) both
d) none

38) Drug used in anaphylaxis is:


a) nor epinephrine
b) +epinephrine
c) dopamine
d) antihistaminic

39) Breast feeding is not contraindicated when the mother is taking the following medication except:
a) theophyline
b) cloromphenicol
c) metronidazole
d) cimetidine

40) Antacids are:


a) physical antagonist
b) chemical antagonist
c) +reversible antagonist
d) irreversible antagonist

41) The toxic effects of a large dose of nicotine is:


a) Salivation
b) Nausea
c) +Convulsions
d) Cutaneous vasodilation

42) Excessive doses of lidocaine cause:


a) Paresthesias
b) +Hypotension
c) Hypertension
d) Disorientation

43) The CNS effect of nicotine:


a) +Tremors
b) Diarrhea
c) Hypotension
d) Decrease of hear rate

44) In what stage of anesthesia may occur the regular respiration and relaxation of the skeletal muscle:
a) Analgesia
b) Excitement
c) +Surgical anesthesia
d) Medullary paralysis

5
45) Heparin therapy is monitored by:
a. ВТ
b. CT
c. +PT
d. All of the above

46) Which of the following drug is useful in dissolving gall bladder stone:
a) +chenodoxycholic acid
b) lactulose
c) clofibrate
d) none

47) Tetracycline can cause all except:


a) +gray baby syndrome
b) hepatotoxicity
c) pain on injection
d) fanconis anaemia

48) Sulfonamide derivates is:


a) +Torsemide
b) dichlorphenamide
c) Acetazolamide
d) Chlorthalidone

49) Methacholine usually could be used in treatment of:


a. +Glaucoma
b. Bronchial constriction
c. Hypertension
d. Hypotension

50. Neostigmine CNS effect is:


a. +Absent
b. Poor
c. Good
d. Excellent

51. Atropine is highly selective for:


a. a-receptors
b. β-receptors
c. +m-receptors
d. n-receptors

52. Good administration for trimethaphan is:


a. Oral
b. Subcutaneous
c. Intramuscular
d. +Intravenous

53. The duration of action of succinyicnoline is


a. + 5-10 minutes
b. 10-20 minutes
c. 20-30 minutes
d. 30-40 minutes

6
54. Location of β-1 receptors:
a. Bronchi
b. Urinary tract
c. Uterus
d. +Heart

55. Irreversible antagonist drug is:


a. +Phenoxybenzamine
b. Prazosine
c. Labetalol
d. Phentolamine

56. Botulinus toxin influence on the release of acetylcholine


a. Induce
b. Increase
c. +Inhibit
d. Doesn’t influence

57. Nicotinic receptors have been divided into


a. +2 subtypes
b. 3 subtypes
c. 4 subtypes
d. 5 subtypes

58. Physostigmine is well absorbed from


a. Oral administration
b. Intravenous
c. Intramuscular
d. +All sites

59. Effect of atropine on the iris persists:


a. 12-24 hours
b. 24-48 hours
c. 48-72 hours
d. +More than 72 hours

60. The crossing the blood-brain barrier of trimethaphan is:


a. +Absent
a. Poor
b. Good
c. Excellent

61. Selective antagonist for the β receptors


a. Yohimbine
b. Clonidine
c. Methoxamine
d. +Prazosin

62. The principal adverse effect of phentolamine are related to


a. Gastric secretion
b. Brochi constriction
c. Smooth muscle dilation
d. +Cardiac stimulation

7
63. A half-life of esmolol is:
a. +5-10 minutes
b. 10-30 minutes
c. 30-40 minutes
d. 40-50 minutes

64. Metoprolol is the member of:


a. a1-selective group
b. a2- selective group
c. + β1 selective group
d. β2 selective group

65. Uses of a-blockers is not:


a. Pheochromocytoma
b. Diabetes
c. Thyrotoxicosis
d. +Cardiac arrhythmias

66. Clinical pharmacology includes


a) Pharmacokinetics and pharfnacodynamics
b) Treatment of systemic infection
c) Science of compounding and dispending drug
d) +Application of pharmacological information

67. Activates a receptor to produce an effect similar to physiological signal:


a. Antagonist
b. +Agonist
c. Inverse agonist
d. Partial agonist

68. Cross the placenta barrier


a) Chlordiazepoxide
b) Diazepam
c) Prazepam
d) +All

69. ADH is located in:


a. Spleen
b. Liver
c. +Kidney
d. All

70. Opioid drugs include:


a. Full agonist
b. Partial agonist
c. Antagonist
d. +All

71. Most opioid analgetics are well absorbed:


a. Subcutaneous
b. Intramuscular
c. Oral
d. +All

8
72. Opioids localize in higher concentration in tissues except:
a. +Brain
b.Liver
c. Muscle
d. Lungs

73. Action of opioids is:


a. +Decrease of alveolar PCO2
b. Suppression of cough
c. Mydriasis
d. Diarrhea

74. Adverse effect of opioids is except:


a. Respiratory depression
b. Nausea
c. Vomiting
d. +Decrease intracranial pressure

75. Clinical use of opioid analgesics:


a. Constipation
b. Arrhythmia
c. +Cough
d. All

76. The maximal blood levels of local anesthetics


a. +Intercostals
b. Caudal
c. Epidural
d. Brachial plexus

77. The primary mechanism of action of local anesthetics is blockade of:


a. Ion channels
b. Potassium channels
c. Chlorium channels
d. +Sodium channels

78. Undergo extensive first pass metabolism in liver except:


a. +Nitroglycerine
b. Isosorbide mononitrate
c. Isosorbide dinitrate
d. Eiythrityl tetranitrate

79. The local anesthetics are converted in:


a. Kidney
b. +Liver
c. Lungs
d. Brain

80. Nitrates may be used except:


a. Angina pectoris
b. Biliaric colic
c. Esophageal spasm
d. +Bradicardia

9
81. Doses of dilthiazem:
a. 10-30 mg
b. +30-60 mg
c. 60-90 mg
d. 90-120 mg

82. The highest blood levels in the following order:


a. +Intercostals
a. Caudal
b. Branchial plexus
b. Sciatic nerve

83. Antiadrenergic agent is:


a. Amiadarone
b. Verapamile
c. +Propranolol
d. Propafenon

84. Agent widening AP is:


a. +Amiadarone
b. Verapamil
c. Propranolol
d. Propafenon

85. Calcium channel blocker is:


a. Amiadarone
b. +Verapamile
c. Propranolol
d. Propafenon

86. Membrane stabilizing agent is:


a) Amiadarone
b) +Verapamile
c) Propranolol
d) Propafenon

87. Excessive doses of lidocaine cause:


a. Paresthesias
b. +Hypotension
c. Hypertension
d. Disorientation

88. How many percents of mexiletine metabolized in liver:


a. 30%
b. 60%
c. +90%
d. 100%

89. The targets for benzodiazepine action is:


a. a-aminobutyric acid
b. β-aminobutyric acid
c. + γ-aminobutyric acid
d. 5-aminobutyric acid

10
90. Intermediate-acting benzodiazepines:
a. Clorazepam
b. +Alprazolam
c. Oxazepam
d. Flurazepam

91. Mechanism of action of methylxanthines


+Translocation of extracellular calcium
Translocation of intracellular calcium
Increase in cAMP
Decrease in cAMP
Decrease in GMP

92. The methylxanthines are well absorbed


a. +Orally
b. Subcutaneously
c. Intramuscular
d. Intravenously

93. The lowest blood levels in the following order:


a. Intercostals
b. Caudal
c. Branchial plexus
d. +Sciatic nerve

94. The highest blood levels in the following order:


a. +Intercostals
b. Caudal
c. Branchial plexus
d. Sciatic nerve

95. May occur the regular respiration and relaxation of the skeletal muscles:
a. Analgesia
b. Excitement
c. + Surgical anesthesia
d. Medullary paralysis

96. SGRIs block the up-take of serotonin leading to :


a. decrease concentration
b.+ Increase concentration
c. Inhibit the receptor
d. is not influence

97. Tick drus is forming a physical barrier to HCL :


a. Ranitidine
b. +Sucralfate
c. Omeprazole
d. Pirenzepine

98. Iindicate the drug that cause metabolic alkalosis:


a. +Sodium bicarbonate
b. Cimetidine
c. Pepto-Bismol
d. Carbenoxolone

11
99. The drug can be used to treat glaucoma:
a. Furosemide (Lasix)
b. +Acetazolamide (Diamox)
c. Both of the above
d. Neither of the above

100. Bactericidal effect is:


a) Inhibition of bacterial cell division
b) Inhibition of young bacterial cell growth
c) +Destroying of bacterial cells
d) Formation of bacterial L-form

101. Tick the drug belonging to antibiotics-tetracycline:


a. +Doxycycline
b. Streptomycin
c. Clarithromycin
d. Amoxacilhn

102. Tick the drug belongs to antibiotics-cephalosporin:


a. Streptomycin
b. +Cefaclor
c. Phenoxymethylpenicillin
d. Erythromycin

103. This drug reduces blood pressure by acting on vasomotor centers in the CNS:
a. Labetalol
b. +Clonidine
c. Enalapril
d. Nifedipine

104. Pick out the drug an alpha- and beta-adrenoreceptors blocker:


a. +Labetalol
b. Verapamil
c. Nifedipine
d. Metoprolol

105. This drug is converted to an active metabolite after absorption:


a) Labetalol
b) Clonidine
c) +Enalapril
d) Nifedipine

106. Indicate an agent, which is related to direct-acting drugs:


a) Edrophonium
b) Physostigmine
c) +Carbachol
d) Isoflurophate

107. All of these drugs reduce intestinal peristalsis EXCEPT:


a. Loperamide
b. +Cisapride
c. Methyl cellulose
d. Magnesium aluminium silicate

12
108. This drug acts by inhibiting type III cyclic nucleotide phosphodiesterase:
a. Amiodarone
b. +Milrinone
c. Propanolol
d. Enalapril

109. Drug is favored for digitalis-induced arrhythmias:


a. Verapamil
b. Amiodarone
c. + Lidocaine
d. Propanolol

110. Tick the drug belonging to non-selective beta-2 adrenomimetics:


a. Salbutamol
b. +Isoprenaline
c. Salmeterol
d. Terbutaline

111. Which of the following M-choiinoblocking agents is used especially as an anti-asthmatic?


a. Atropine
b. +Ipratropium
c. Platiphylline
d. metacin

112. A teratogenic action is:


a. Toxic action on the liver
b. +Negative action on the fetus causing fetal malformation
c. Toxic action on blood system
d. Toxic action on kidneys

113. What term is used to describe a decrease in responsiveness to a drug which develops in a few
minutes?
a. Refractoriness
b. Cumulative effect
c. Tolerance
d. +Tachyphylaxis

114. If an agonist can produce sub maximal effects and has moderate efficacy it's called:
a. +Partial agonist
b. Antagonist
c. Agonist-antagonist
d. Full agonist

115. Tachyphylaxis is:


A drug interaction between two similar types of drugs:
a. +Very rapidly developing tolerance
b. A decrease m responsiveness to a drug, taking days or weeks to develop
c. None of the above

116. Idiosyncratic reaction of a drug is:


a. A type of hypersensitivity reaction
b. A type of drug antagonism
c. +Unpredictable, inherent, qualitatively abnormal reaction to a drug
d. Quantitatively exaggerated response

13
117. Tick the second messenger of G-protein-coupled (metabotropic) receptor:
a. Adenylyl cyclase
b. Sodium ions
c. Phospholipase С
d. +cAMP

119. What does the term “potentiation” mean?


a. Cumulative ability of a drug
b. Hypersensitivity to a drug
c. Fast tolerance developing
d. +intensive increase of drug effects due to their combination

120. If an agonist can produce sub maximal effects and has high efficacy it's called:
a. Partial agonist
b. Antagonist
c. Agonist-antagonist
d. +Full agonist

121. What kind of substances can't permeate membranes by passive diffusion?


a. Lipid-soluble
b. Non-ionized substances
c. Hydrophobic substances
d. +Hydrophilic substances

122. Which of the following is more potent:


a) cimetidine
b) ranitidine
c) nizatidine
d) +famotidine

123. Indicate the laxative drug belonging to osmotic laxatives:


a. Docusate sodium
b. Bisacodyl
c. Phenolpnthalein
d. +Sodium phosphate

124. The mechanism of methylxanthines action is:


a) +Inhibition of the enzyme phosphodiesterase
b) Beta-2 adrenoreceptor stimulation
c) Inhibition of the production of inflammatory cytokines
d) Inhibition or M-cholinorcceptors

125. The drug is the most potent diuretic:


a. +Loop diuretics
b. Thiazide diuretics
c. Potassium-sparing diuretics
d. Carbonic anhydrase inhibitors

126. The drugs is used in the treatment of recurrent Ca nephrolithiasis:


a. Osmotic diuretics
b. +Loop diuretics
c. Thiazide diuretics
d. Potassium-sparing diuretics

14
127. Duration of nitroglycerin action (sublingual) is:
a) +10-30 minutes
b) 6-8 hours
c) 3-5 minutes
d) 1.5-2 hours

128. All of the following agents are cardiac glycosides EXCEPT:


a. Digoxin
b. Strophantin К
c. +Amrinone
d. Digitoxin

129. This drug inhibits breakdown of cAMP in vascular smooth muscle:


a. Digoxin
b. Dobutamine
c. +Amrinone
d. Dopamine

130. Catecholamine includes following EXCEPT:


a. +Ephedrine
b. Epinephrine
c. Isoprcnalinc
d. Norepinephrine

131. Indicate the beta-1 selective agonist:


a. Isoproterenol
b. +Dobutamine
c. Metaproterenol
d. Epinephrine

132. Which of the following is preferable for the emergency therapy of cardiogenic shock?
a. Epinephrine
b. +dobutamine
c. Isoproterenol
d. Mcthoxamine

133. Which of the following drugs is an nonselective beta receptor antagonist?


a) Metoprolol
b) Atenolol
c) +Propranolol
d) Acebutolol

134. Which of the following drugs is useful in the treatment of pheochromocytoma?


a. Phenylephrine
b. Propranolol
c. +Phentolamine
d. Epinephrine

135. All of the following drugs are antibiotics, EXCEPT:


a. Streptomycin
b. Penicillin
c. +Co-trimoxazole
d. Chloramphenicol

15
1. Which of the following acts indirectly ?
a. Epinephrine
b. Norepinephrine
c. +Ephedrine
d. d) Methoxamine

2. Indicate the alfa 2-selective agonist:


a. +Xylometazoline
b. Epinephrine
c. Dobutamine
d. Methoxamine

3. Epinephrine produces all of the following effects except:


a. Bronchodilation
b. +Decrease in oxvgen consumption
c. Hvperglvcemia
d. Mydrias'is

4. Indicate the drug, which may be useful in the emergency management of cardiac arrest:
a. ethoxamine
b. Phenylephrine
c. +Epinephrine
d. Xylometazoline

5. Indicate the beta-l selective antagonist:


a. Propranolol
b. +Metoprolol
c. Carvedilol
d. Sotalol

6. Non selective alfa-receptor antagonists are most useful in the treatment of:
a. Asthma
b. Cardiac arrhythmias
c. +Pheochromocvtoma
d. Chronic hypertension

7.This drug is a Class 1C antiarrhythmic drug:


a. +Flecainide
b. Sotalol
c. Lidocaine
d. Verapamil

8. This drug is associated with Torsades de pointes.


a. Flecainide
b. +Sotalol
c. Lidocaine
d. Verapamil

9. This drug is used intravenously to terminate supraventricular tachycardias:


a. Nifedipine
b. +Verapamil
c. Both of the above
d. None of the above

10. Bactericidal effect is:


a. Inhibition of young bacterial cell growth
1
b. Inhibition of bacterial cell division
c. +Destroying of bacterial cells
d. Formation of bacterial L-form

11. Tick the drug belonging to antibiotics-macrolides:


a. Neomycin
b. Doxicycline
c. +Erythromycin
d. Cefotaxime

12. Tick the drug belonging to antibiotics-tetracyclmes:


a. +Doxycycline
b. Streptomycin
c. Clarithromycin
d. Amoxacillin

13. Antibiotic inhibiting bacterial RNA synthesis is:


a. Erythromycin
b. +Rifampin
c. Chloramphenicol
d. Imipinem

14. Which of the following agents is related to tricyclic antidepressants ?


a. Nefazolan
b. +Amitriptyline
c. Fluoxetine
d. Isocarboxazid

15. Indicate an effective antidepressant with minimal autonomic toxicity:


a. Amitntyline
b. +Fluoxetine
c. Imipramine
d. Doxepin

16. A ganglioblocking drug for hypertension treatment is:


a. Hydralazine
b. Tubocurarin
c. +Trimethaphan
d. Metoprolol

17. This drug inhibits the angiotensin-converting enzyme:


a. Captopril
b. Enalapril
c. Ramipril
d. +All of the above

18. This drug is an inhibitor of rennin synthesis:


a. +Propranolol
b. Enalapril
c. Diazoxide
d. Losartan
19. This drug routinely produces some tachycardia:
a. +Nifedipine
b. Clonidin
c. Enalapril
d. Propranolol

2
20. Which of the following agents is the precursor of dopamine ?
a) Bromocriptine
b) +Levodopa
c) Selegiline
d) Amantadine

21. Indicate a selective inhibitor of monoamine oxidase B:


a) Levodopa
b) Amantadine
c) Tolcapone
d) +Selegiline

22. Which of the following benzodiazepines has the shortest duration of action?
a) +Triazolam
b) Clorazepate
c) Prazepam
d) Clordiazepoxide

23. Which of the following anxiolytics has minimal abuse liability ?


a) Oxazepam
b) +Buspirone
cj Flumazenil
d) Alprazolam

24. Which of the following direct-acting is mainly muscarinic in action ?


a) +Bethanechol
b) Carbacbol
c) Acetylcholine
d) None of the above

25. Which of the following cholinesterase inhibitors is irreversible?


a) Physostigmine
b) Edrophonium
c) Neostigmine
d) +Isoflurophate

26. Chronic long-term therapy of myasthenia is usually accomplished with:


a) Edrophonium
b) +Neostigmine
c) Echothiophate
d) Carbachol

27. Indicate the skeletal muscle relaxant, which is a depolarizing agent:


a) Vencuronium
b) Scopolamine
c) +Succinylcholine
d) Hexamethonium
28. Which of the following drugs is a selective M l blocker ?
a. Atropine
b. Scopolamine
c. +Pirenzepme
d. Homatropine

29. Indicate a drug, which is effective in the treatment of mushroom poising:


a)Pralldoxime

3
b) Pilocarpine
c) Homatropine
d) +Atropine

30. Which of the following drug is an intermediate-duration muscle relaxant?


a) +Vecuronium
b) Tubocurarine
c) Pancuronium
d) Rapacuronium

31. Which of the following muscular relaxants causes hypotension and bronchospasm ?
a. Vecuronium
b. Succinvlcholine
c. +Tubocurarine
d. Rapacuronium

32. The drug is a K+ -sparing diuretic that blocks Na+ channels in the collecting tubules:
a. Acetazolamide (Diamox}
b. +Amiloride (Midamor)
c. Furosemide (Lasix)
d. Hydrochlorothiazide (HydroDiuril)

33. The drug acts only on the luminal side of renal tubules:
a. +Furosemide (Lasix)
b. Acetazolamide (Dianiox)
c. Both of the above
d. Neither of the above

34. The drug acts at the proximal tubule:


a. Loop diuretics
b. Thiazide diuretics
c. Potassium-sparing diuretics
d. +Carbonic anhvdrase inhibitors

35. The drug is one of the most potent diuretics:


a. Acetazolamide (Diamox)
b. +Furosemide (Lasix)
c. Hydrochlorothiazide (Hydrodiuril)
d. Aniiloride (Midamor)

36. Furosemide acts at the nephron site:


a. Proximal convoluted tubule
b. +Ascending thick limb of the loop of Henle
c. Distal convoluted tubule
d. Collecting duct

37. Which of the following nitrates are long-acting?


a. Nitroglycerin
b. Isosoroide dinitrat
c. Amyl nitrite
d. +Sustac

38. The following agents are cardioselective beta-1 blocking drugs except:
a. +Propranolol

4
b. Talinolol
c. Atenolol
d. Metaprolol

39. Drug is used in digitalis intoxication:


a) Lidocaine
b) +Digibind (Digoxin immune fab)
c) Oral potassium supplementation
d) Reducing the dose ol the drug

40. All of the following drugs are used in CHF


a. Digoxin
b. Dobutamine
c. +Verapamil
d. Dopamine

41. Which of the following inhaled anesthetics is an induction agent of choice in patient
with airway problems ?
a. Desfurane
b. Nitrous oxide
c. +Halothane
d. None of the above

42. Which of the following intravenous anesthetics has antiemetic actions?


a. Thiopental
b. +Propofol
c. Ketamine
d. Fentanyl

43. Pick out the bronchodilator drug related to xanthine:


a) Atropine
b) Orciprenaline
b) Adrenaline
d) +Theophylline

44. All of the following drugs are inhaled glucocorticoids except:


a) Triamcinolone
b) Beclometazone
c) +Sodium cromoglycate
dj Budesonide

45. Indicate the drug belonging to proton pump inhibitors:


a) Pirenzepme
b) Ranitidine
c) +Omenprazole
d) T rimethaphan

46. Which of the following drugs may cause reversible gynecomastia?


a. Omeprazole
b. Pirenzepine
c. +Cimetidine
d. Sucralfate

5
47. Choose the drug that causes constipation:
a) Sodium bicarbonate
b) +Aluminium hvdroxide
c) Calcium carbonate
d) Magnesium oxide

48. Indicate an antiemetic agent which is related to neuroleptics:


a. Metoclopramide
b. Nabilone
c. Tropisetron
d. +Prochlorperazine

49. Which of the following is not an adverse effect of ranitidine:


a) headache
b) +gynecomastia
c) dizziness
d) confusion

50. What does “pharmacokinetics" include?


a. Complications of drug therapj
b. +Drug biotransformation in the organism
c. Influence of drugs on metabolism processes
d. Influence of drugs on genes

51. All of the following agents are beta-receptor agonists except:


a. Epinephrine
b. Isoproterenol
c. +Methoxamine
d. Dobutamine

52. Which of the following is a beta-2 selective agonist?


a. +Terbutaline
b. Xylometazoline
c. Isoproterenol
d. Dobutamine

53. Isoproterenol produces all of the following effects except:


a) Increase in cardiac output
b) Fall in diastolic and mean arterial pressure
c) +Bronchoconstriction
d) Tachycardia

54.1ndicate the alfa-1 selective antagonist:


a. Phentolamine
b. Dihydroergotamine
c. +Prazosin
d. Labetalol

55. Indicate the beta antagonist, which has partial beta-agonist activity:
a. Prooranolol
b. Meioprolol
c. +Pindolol
d. Betaxolol

6
56. The adverse effects of phentolamine include all of the following EXCEPT:
a. Diarrhea
b. +Bradycardia
c. Arrhythmias
d. Myocardial ischemia

57. This drug is a Class II antiarrhythmic drug:


a. Flecainide
b. +Propranolol
c. Lidocaine
d. Verapamil

58. This drug is used in treating supraventricular tachycardias:


a)+Digoxin
b) Dobutamine
c) Amiinone
d) Dopamine

59. This is a drug of choice for acute treatment of ventricular tachycardias:


a) Flecainide
b) Sotalol
c) +Lidocaine
d) Verapamil

60. All of the following are common adverse effects of Ca channe blockers EXCEPT:
a. +Skeletal muscle weakness
b. Dizziness
c. Headache
d. Flushing

61. Bacteriostatic effect is:


a) +Inhibition of bacterial cell division
b) Inhibition of young bacterial cells growth
c) Destroying of bacterial cells
d) Formation of bacterial L-form

62. Tick the drug belonging to antibiotics-monobactams:


a) Ampicillin
b) Bicillin-5
c) +Aztreonam
d) Imipinem

63. Tick the drug belonging to nitrobenzene derivative:


a. Clindamycin
b. Streptomycin
c. Azithromycin
d. +Chloramphenicol

64. Antibiotics altering permeability of cell membranes are:


a. Glvcopeptides
b. +Polymyxins
c. Tetracvclines
d. Cephalosporins

7
65. Which of the following atuidepressants is a selective serotonin reuptake inhibitor?
a) Phenelzine
b) Desipramine
c) Maprotiline
d) +Fluoxetine

66. Which of the following antidepressants is used for treatment of eating disorders,
especially buliemia ?
a. Amitriptvline
b. +Fluoxetine
c. Imipramihe
d. Tranylcypromine

67. Pick out the sympatholytic drug:


a. aj Labetalol
b. Prazosin
c. + Guanethidine
d. Clonidine

68. Choose the selective blocker of beta-1 adrenoreceptors:


a. Labetalol
b. Prazosin
c. +Atenolol
d. Propranolol

69. This drug blocks alpha-1 adrenergic receptors:


a. +Prazosin
b. Clonidine
c. Enalapril
d. Nifedipine

70. This drug is contraindicated in patients with bronchial asthma:


a. +Propranolol
b. Clonidine
c. Enalapril
d. Nifedipme

71. Tick the diuretic agent - aldosterone antagonist:


a. Furosemide
b. +Spironolactone
c. Dichloiiiiazide
d. Captopril

72. Indicate the agents of choice in the treatment of most anxiety states:
a. Barbiturates
b. +Benzodiazepines
c. Lithium salts
d. Phenothiazines

73. Which of the following anxiolytics is preferred in patient with limited hepatic function?
a. +Buspirone
b. Quazepam
c. Diazepam
d. Chlordiazep oxide

8
74. Muscarinic receptors are located in:
a. Autonomic ganglia
b. Skeletal muscle neuromuscular junctions
c. +Autonomic effector cells
d. Sensorv carotid sinus baroreceptor zone

75. Which of the following direct-acting drug has the shortest duration of action?
a. +Acetylcholine
b. Metnacholine
c. Carbachol
d. Bethanechol

76. Indicate cholinesterase activator:


a. +Pralidoxime
b. Edrophonium
c. Pilocarpine
d. Isoflurophate

77. Indicate the reversible cholinesterase inhibitor, which penetrates the blood-brain barrier:
a. +Physostigmine
b. Edrophonium
c. Neostigmine
d. Piridostigmine

78. Which of the following agents is a ganglion-blocking drug?


a. Homatropine
b. +Hexametnonium
c. Rapacuronium
d. Edrophonium

79. Atropine is used prior to premedication to reduce:


a. Muscle tone
b. +Secretions
c. Nausea and vomiting
d. All of the above

80. Which of the following agents is used as an inhalation drug in asthma?


a. Atropine
b. +Ipratropium
c. Lobelme
d. Homatronine

81. Indicate the long-acting neuromuscular blocking agent:


a. Rapacuronium
b. Mivacurium
c. +Tubocurarine
d. Rocuronium

82. Which of the following neuromuscular blockers causes transient muscle fasciculations?
a. Mivacurium
b. Pancuronium
c. +Succinvlcholine
d. Tubocurarine

9
83. The drug acts at the proximal tubule:
a. +Acetazolamiae (Diamoxj
b. Furosemide (Lasix)
c. Hydrochlorothiazide (HydroDiuril)
d. Spironolactone (Aldactone)

84. The drug can be used to treat glaucoma:


a. Furosemide (Lasix)
b. +Acetazolamide (Dianiox)
c. Both of the above
d. Neither of the above

85. The drug decreases calcium excretion in urine:


a. +Hydrochlorothiazide (HydroDiuril)
b. Amiloride (Midamor)
c. Furosemide (Lasix)
d. Acetazolamide (Diamox)

86. The drug acts in the collecting tubules:


a. Loop diuretics
b. Thiazide diuretics
c. +Potassium-sparing diuretics
d. Carbonic annydrase inhibitors

87. The drug is the least potent diuretic:


a. Osmotic diuretics
b. Loop diuretics
c. Thiazide diuretics
d. +Potassium-sparing diuretics

88. Acetazolamide acts at this nephron site


a. +Proximal convoluted tubule
b. Ascending thick limb of the loop Henle
c. Distal convoluted tubule
d. Collecting duct

89. Which of the following is a Ca channel blocker ?


a. Nitroglycerin
b. Dipyndamole
c. Minoxidil
d.+ Nifedipine

90. All of the following are used in CHF except:


a. Reduced salt intake
b. +Verapamil
c. ACE inhibitors
d. Diuretics

91. This drug is a selective beta-1 agonist:


a. Digoxin
b. +Dobutamine
c. Amrinone
d. Dopamine

92. Indicate drug, which is an inhibitor of NMDA glutamate receptors:


a. Thiopental

10
b. Halothane
c. +Ketamine
d. Sevoflurane

93. Which of the following inhaled anesthetic can produce hepatic necrosis?
a. Soveflurane
b. Desfluane
c. +Halothane
d. Nitrous oxide

94. Indicate the intravenous anesthetic, which is a benzodiazepine derivative:


a. +Midazolam
b. Thiopental
c. Ketamin
d. Propofol

95. Pick out the bronchodilator drug belonging to sympathomimics:


a. Isonrenaline
b. +Epnedrine
c. Atropine
d. Salbutamol

96. Choose the drug belonging to membrane stabilizing agents:


a. Zileutin
b. +Sodium cromoglycate
c. Zafirlucast
d. Montelucast

97. All of the following agents intensify the secretion of gastric glands ЕХСЕРT.
a. +Pepsin
b. Gastrin
c. Histamine
d. Carbonate mineral waters

98. Tick the drug forming a physical barrier HC1 and pepsin:
a. Ranitidine
b. +Sucralfate
c. Omeprazole
d. Pirenzepine

99. Indicate the drug that cause metabolic alkalosis:


a. +Sodium bicarbonate
b. Cimetidine
c. Pepto-Bismol
d. Carbenoxolone

100. Indicate the beta-1 selective antagonist:


a) Propranolol
b) +Metoprolol
c) Garvedilol
d) Sotalol
101. This drug is an inhibitor of renin synthesis:
a.+Propranolol
b. Enalapril
c. Diazoxide
d. Losartan

11
102. This drug routinely produces some tachycardia:
a. Propranolol
b. Clonidine
c. Enalapril
d. +Nifedipine

103. Which drugs is a selective Ml blocker?


a. Atropine
b. +Scopolamine
c. Pircnzepine
d. Homatropine

104. Indicate a drug, which is effective in the treatment of mushroom poising:


a. Pralidoxime
b. Pilocarpine
c. Homatropine
d. +Atropine

105. The drug is a K+-sparing diuretic that blocks Na+ channels inlhe collecting tubules:
a. Acetazolamide (Diamox)
b. +Amiloride (Midamor)
c. Furosemide (Lasix)
d. Hydrochlorothiazide (HydroDiuril)

106. Indicate the drug belonging to proton pump inhibitors:


a) Pircnzepine
b) Ranitidine
c) +Omeprazole
d) Trimcthaphan

107. Which of the following drugs may cause reversible gynecomastia?


a) Omeprazole
b) Pircnzepine
c) +Cimetitfine
d) Sucralfate

108. Choose the drug that causes constipation:


a) Sodium bicarbonate
b) +Aluminium hydroxide
c) Calcium carbonate
d) Magnesium oxide

109. Furosemide acts at the nephron site:


a. Proximal convoluted tubule
b. +Ascending thick limb of the loop of Henle
c. Distal convoluted tubule
d. collecting duct

110. A bronchial smooth muscle contains:


a. Alfa-1 receptor
b. Alfa-2 receptor
c. Beta-1 receptor
d. + Beta-2 reccptor

12
111. Which drug is a non selective β-receptor agonist?
a. Norepinephrine
b. Terbutaline
c. +Isoproterenol
d. Dobutamine

112. Sustained use of this drug results in increased plasma urate concentrations:
a) +Furosemide (Lasix)
b) Acetazolamide (Diamox)
c) Both of the above
d) Neither of the above

113. This drug is a Class IB antiarrhythmic drug:


a. Flecainide
b. Sotalol
c. +Lidocaine
d. Verapamil

114. Tick the drug belongs to antibiotics-cephalosporins:


a. Streptomycin
b. +Ceficlor
c. Phenoxymcthilpenicillin
d. Erythromycin

115. Which of the following drugs is a nonselective alfa receptor antagonist?


a. Prazosin
b. +Phentolamine
c. Metoprolol
d. Reserpine

116. Which of the following groups of antibiotics shows a bacteristatic effect:


a. Carbapenems
b. +Macrolides
c. Aminoglycosides
d. Cephafos'porins

117. All of the following agents are cardiac glycosides EXCEPT:


a. Digoxin
b. Strophantin К
c. +Amrinone
d. Digitoxin

118. Amiloride acts ai this nephron site:


a. Proximal convoluted tubule
b. Ascending thick limb of the loop of Henle
c. Distal convoluted tubule
d. +Collecting duct

119. Indicate the drug which is a leucotrienc receptor antagonist:


a. Sodium cromoglycatc
b. +Zafirlucast
c. Zileutin
d. Triamcinolone

13
120. Choose the drug which is a H2-receptor antagonist:
a. Omeprazole
b. Pirenzepme
c. Carbenoxolonc
d. +Ranitidine

121. Which drug is an analog of prostaglandin El?


a. +Misoprostole
b. De-nol
c. Sucralfate
d. Omeprazole

122. Tick the mechanism of metoclopramide antiemetic action:


a. HI and H2-receptor blocking effect
b. M-cholinoreceptor stimulating effect
c. +D2-dopaniine and 5-HT3-serotonin receptor blockina effect
d. M-choiinoblocking effect

123. All of these drugs reduce intestinal peristalsis EXCEPT:


a. Loperamide
b. +Cisapride
c. Methyl cellulose
d. Magnesium aluminium silicate

124. All drugs inhibit cell wall synthesis, except:


a. Carbapenems.
b. Monobactams.
c. Cephamycins.
d. +Nitrofurantoin

125. This drug is a Class III antianhythmic drug:


a. Flecainide
b. +Sotalol
c. docaine
d. Verapamil

126. Minimal duration of antibacterial treatment is:


a. Not less than 1 day
b. +Not less than 5 days
c. Not less than 10-I4 days
d. Not less than 3 weeks

127. Which antibiotics contains a β-lactam ring in their chemical structure:


a. Penicillins
b. Cephalosporins
c. Carbapenems and monobactams
d. +All groups

128. The mode of action may be:


a) +specific and nonspecific
b) selective and nonselective
c) reversible and irreversible
d) metabolic and fimctional

14
129. All of the following antibiotics inhibit the protein synthesis in bacterial cells, EXCEPT:
a. Macrolides
b. Aminoglycosides
c. +Glycopcptides
d. Tetracyclincs

130. All are true about digoxin except:


a.+Causes bradycardia
b.Acts by inhibiting Na+K+ ATPase
c. It is 95 % plasma protein bound
d. Primarily excreted unchanged by glomerular filtration acting

131. All of the following are central antihypertensive drugs EXCEPT:


a. Methyldopa
b. Clonidine
c.Moxonidine
d. +Minoxidil

132. This drug is a directly acting vasodilator:


a. Labetalol
b. Clonidine
c. Enalapril
d. +Nifedipine

133. This drug is a potassium channel activator:


a) Nifedipine
b) Saralasin
c) +Diazoxide
d) Losartan

134. Which of the following is commonly used in the treatment of glaucoma?


a. +Pilocarpine
b.Lobeline
c. Acethylcholine
d. Neostigmine

135. Indicate muscles, which are more resistant to block and recover more rapidly:
a. Hand
b. Leg
c. Neck
d. +Diaphragm

136. Tick the drug, inhibiting viral DNA synthesis:


a) Interferon
b) Saquinavir
c) Amantadine
d)+Acyclovir

137. The drug interaction with different cellular and subcellular structures can result in their
a)stimulation
b)+depression
c)decreasing
d)stabilization

15
138. Tick the anticancer drug, a pyrimidine antagonist:
a) Fluorouracil
b) +Mercaptopurine
c) Thioguanine
d) Methotrexate

139. Geriatric pharmacology studies the peculiarities of action and clinical use of drugs in
a) children
b) pregnant women
c) nursing women
d) +elderly

140. Tick the second messenger of G-protein-coupled (metallotropic) receptor:


a. Adenylyl cyclase
b. Sodium ions
c. Phosnholipase С
d. +сАМР

150. Which drug is a proton pump inhibitor?


a. Cimetidme
b. Ranitidine
c. Pirenzcpin
d. +Omeprazole

151. Choose an emetic drug of central action:


a. +Ipecacuanha derivatives
b. Promethazine
c. Tropisetron
d. Apomorphine hydrochloride

152. Therapeutic index is indicator of:


a) Safety
b) +Efficacy
c) Potency
d) Toxicity

153. Morphine causes all, EXCEPT:


a) Peripheral vasodilatation
b) Decrease intracranial tension
c) Nausea and vomiting
d) +Decrease in gastrointestinal secretion

154. What kind of substances can’t permeate membranes by passive diffusion?


a. Lipid-soluble
b. Non-ionized substances
c. Hydrophobic substances
d. +Hydrophilic substances

155. Tachyphylaxis is:


a. A drug interaction between two similar types of drugs
b. +Very rapidly developing tolerance
c. A decrease in responsiveness to a drug, taking days or weeks to develop
d. None of the above

16
156. Pharmacology is a science about
a)chemistry
b)physiology.
c)+medicinal preparations
d)pharmaceutical business

157. A teratogenic action is:


a. Toxic action on the liver
b. +Negative action on the fetus causing fetal malformation
c. Toxic action on blood system
d. Toxic action on kidneys

158. Which of the following agents is an selective agents is an alfa-l selective agonist?
a. Norepinephrine
b. +Methoxamine
c. Ritodrine
d. Ephedrine

159. Indicate the indirect-acting agent:


a. Epinephrine
b. Phenylephrine
c. +Ephedrine
d. Isoproterenol

160. Which of the following is related to short acting topical decongestant agents ?
a. Xylometazoline
b. Terbutaline
c. +Phenylephrine
b. Norepinephrine

161. Which of the following agents is an alfa 2-selective antagonist ?


a. +Yohimbine
b. Tamsulosin
c. Tolazoline
d. Prazosin

162. Which of the following drugs is a reversible nonselective alfa-, beta-antagonist?


a. + Labetalol
b. Phentolamine
c. Metoorolol
d. Propranolol

163. This drug is a Class 1A antiarrhythmic drug:


a. Sotalol
b. Propranolol
c. Verapamil
d. +Quinidine

164. This drug prolongs repolarization:


a. Flecainide
b. +Sotalol
c. Lidocaine
d. Verapamil

17
165. This drug has beta-adrenergic blocking activity:
a. Flecainide
b. +Sotalol
c. Lidocaine
d. Verapamil

166. This drug acts to inhibiting slow Ca channels in the SA and AV nodes:
a. Quinidine
b. Adenosine
c. Fleeainide
d. +Diltiazem

167. Which of the following groups of antibiotics shows a bactericidal effect?


a. Tetracyclines
b. Macrolides
c. +Penicillins
d. All of the above

168. Tick the drug belonging to antibiotics - carbapenems:


a. Aztreonam
b. Amoxacillin
c. +Imipinem
d. Clarithromycin

169. Tick the drug belonging to lincozamides:


a. Erythromycin
b. +Lincomvcm
c. Azithromycin
d. Aztreonam

170. Tick the drug belonging to glycopeptides:


a. +Vancomycin
b. Lincomycin
c. Neomycm
d. Carbenicillin

171. Which of the following drugs is a gastric acid resistant:


a. Penicillin G
b. +Penicillin V
c. Carbenicillin
d. Procain penicillin

I72. Indicate the antidepressant, which blocks the reuptake pumps for serotonin and norepinephrine:
a. +Amitriptyline
b. Fluoxetine
c. Maprotiline
d. Phenelzine

173. A highly selective serotonine reuptake inhibitor is:


a. Sertaline
b. Paroxetine
c. Fluoxetine.
d. +All of the above

18
174. Tick the drug with nonselective beta-adrenoblocking activity:
a. Atenolol
b. +Propranolol
c. Metoprolol
d. Nebivolol
175. Pick out the diuretic agent for hvpertension treatment:
a. Losartan
b. +Dichlothiazide
c. Captcpril
d. Prazosin
176. This drug is a non-peptide angiotensin II receptor antagonist:
a. Clonidine
b. Captopril
c. +Losartan
d. Diazoxide

177. Choose the vasodilator which releases NO:


a. Nifedipine
b. Hydralazine
c. Minoxidil
d. +Sodium nitroprusside

178. Indicate a peripheral dopa decarboxvlase inhibitor:


a. Tolcapone
b. Clozapine
c. + Carbiaopa
d. Selegiline

179. Indicate D2 receptor agonist with antiparkinsonian activity:


a. Sinemet
b. Levodopa
c. +Bromocriptine
d. Selegiline

180. Which of the following anxylotics is a benzodiazepine derivative:


a. Buspirone
b. +Clordiazepoxide
c. Meprobamate
d. Chloral hydrate

181. Indicate the competitive antagonist of β-2 receptors:


a. +Flumazenil
b. Buspirone
c. Picrotoxin
d. Diazenam

182. Which of the following activates both muscarinic and nicotinic receptors ?
a. Lobeline
b. Pilocarpine
c. Nicotine
d. +Bethanechol

19
183. The M-cholinimimetic agent is:
a. Carbachol
b. +Pilocarpine
c. Acetylcholine
d. Betfianechol

184. Isofluorophate increases all of the following effects except:


a. Lacrimation
b. +Bronchodilation
c. Muscle twitching
d. Salivation

185. Which of the following cholinomimetics is used in the treatment of atropine intoxication?
a. Neostigmine
b. Carbochol
c. +Physostigmine
d. Lobeline

186. Which of the following drugs is effective in motion sickness?


a. Atropine
b. Ipratropium
c. +Scopolamine
d. Homatropine

187. Which of the following drugs is useful in the treatment of uterine spasms?
a. Carbachol
b. Vecuronium
c. +Atropine
d. Edrophonium

188. Antimuscarinics are used in the treatment of the following disorders EXCEPT:
a. Motion sickness
b. +Glaucoma
c. Hyperhidrosis
d. Asthma

189. Indicate the nondepolarizing agent, which has short duration of action:
a. Succinylcholine
b. Tubocurarine
c. +Mivacurium
d. Pancuronium

190. The drug inhibits the ubiquitous enzyme carbonic anhydrase:


a. +Acetazolamide (Diamoxj
b. Furosemide (Lasix]
c. Hydrochlorothiazide (HydroDiuril)
d. Spironolactone (Aldactone)

191. The drug acts by competing with aldosterone for its cytosotic receptors:
a. Acetazolamide (Diamoxj
b. Furosemide (Lasix)
c. Hydrochlorothiazide (HydroDiuril)
d. +Spironolactone (Aldactone)

20
192. The drug can cause ototoxicity:
a. +Furosemide (Lasix)
b. Acetazolamide (Diamox)
c. Both of the above
d. Neither of the above

193. The drug acts by competitively blocking the Na+/K+/2Cl- cotransporter:


a. +Loop diuretics
b. Thiazide diuretics
c. cl Potassium-sparing diuretics
d. Carbonic anhydrase inhibitors

194. The drug acts by competitively blocking the NaCI cotransporter:


a. Loop diuretics
b. +Thiazide diuretics
c. Potassium-sparing diuretics
d. Carbonic anhydrase inhibitors

195. These agents must be given parenterally because they are not absorbed w'hen given
orally:
a. + Osmotic diuretics
b. Loon diuretics
c. Thiazide diuretics
d. Potassium-sparing diuretics

196. Spironolactone acts at this nephron site:


a. Proximal convoluted tubule
b. Ascending thick limb of the loop of Henle
c. Distal convoluted tubule
d. +Collecting duct

197. Main clinical use of Ca channel blockers is:


a. Angina pectoris
b. Hypertension
c. Supraventricular tachyarrhythmia
d. +All of the above

198. The non-glycoside positive inotropic drug is:


a. Digoxin
b. Strophantin К
c. +Dobutamine
d. Digitoxin

199. Tolerance to this drug develops after a few days:


a. Amrinone
b. Amiodarone
c. +Dobutamine
d. Adenosine

200. Which of the following general anesthetics belongs to inhalants ?


a. Thiopental
b. +Desfluran
c. Ketamine
d. Propoftol

21
201. Indicated the inhaled anesthetic, which may cause nephrotoxicity:
a. Halothane
b. +Soveflurane
c. Nitrous oxide
d. Diethvl ether

202. Which of these groups of drugs is used for asthma treatment ?


a. Methylxanthines
b. M-cholinoblocking agents
c. Beta-2 stimulants
d. +All of above

203. Select the side-effect characteristic for non-selective beta-2 adrenomimetics:


a. Depression of the breathing centre
b. +Tachycardia
c. Peripheral vasoconstriction
d. Dry mouth

204. Indicate the side effect of theophylline:


a. Bradycardia
b. +Increased myocardial demands for oxygen
c. Depression of respiratory centre
d. Elevation of the arterial blood pressure

205. Gastric acid secretion is under the control the following agents EXCEPT:
a. Histamine
b. Acetylcholine
c. +Serotonin
d. Gastrin

206. All of the following drugs are proton pump inhibitors except:
a. Pantoprozole
b. Omeprazole
c. +Famotidine
d. Rabeprazole

207. Most of drugs are antacids EXCEPT:


a. +Misoprostol
b. Maalox
c. Mylanta
d. Almagel

208. This drug is related to short-acting topical decongestant:


a. Xylometazoline
b. Terbutaline
c. +Phenylephrine
d. Norepinephrine

209. This drug is a Class IA antiarrhythmic drug:


a. Sotalol
b. Propranolol
c. Verapamil
d. +Quinidine

22
210. This drug acts by inhibiting slow Ca channels in the SA and AV nodes:
a. Quinidine
b. Adenosine
c. Fleeainide
d. +Diltiazem

211. Which of the following groups of antibiotics shows a bactericidal effect?


a. Tetracyclines
b. Macrolides
c. Penicillins
d. +All of the above

212. Most of drugs are antacids EXCEPT:


a. +Misoprostol
b. Maalox
c. Mvlanta
d. Almagel

213. Side effects of anticancer drugs are all, EXCEPT:


a. Low selectivity to cancer cells
b. Depression of bone marrow
c. Depression of angiogenesis
d. +Depression of immune system

214. Main clinical use of Ca channel blockers is:


a. Angina pectoris
b. Hypertension
c. Supraventricular tachyarrhythmias
d. +All of the above

215. Which drug is a H2-receptor antagonist:


a. Omeprazole
b. Pirenzepine
c. Carbenoxolone
d. +Ranitidine

23
DRUGS ACTING ON AUTONOMOUS NERVOUS SYSTEM

1. In the autonomic regulation of blood pressure


A. Cardiac output is maintained constant at the expense of other hemodynamic variables
B. Elevation of blood pressure results in elevated aldosterone secretion
C. Baroreceptor nerve fibres decrease firing rate when arterial pressure increases
D. Stroke volume and mean arterial blood pressure are the primary direct determinants of
cardiac output +

2. The autonomic nervous system is also known as


A. Visceral
B. Vegetative
C. Involuntary nervous system +
D. All of the above
3. Full activation of the sympathetic nervous system, as in maximal exercise, can
produce all of the following responses except
A. Bronchial relaxation
B. Decreases intestinal motility
C. Increased renal blood flow +
D. Mydriasis
4. The neurotransmitter agent that is normally released in the sinoatrial node of the
heart in response to a blood pressure increase is
A. Acetylcholine +
B. Dopamine
C. Epinephrine
D. Glutamate
5. A direct-acting cholinomimetic that is lipid-soluble and often used in the treatment of
glaucoma is
A. Acetylcholine
B. Bethanechol
C. Physostigmine
D. Pilocarpine +
6.Which of the following agents is a prodrug that is much less toxic in mammals than in
insects
A. Acetylcholine
B. Bethanechol
C. Physostigmine C +
D. Pilocarpine
7. Atropine overdose may cause which one of the following
A. Gastrointestinal smooth muscle cramping
B. Increased cardiac rate +
C. Increased gastric secretion
D. Pupillary constriction
8.The synaptic pre-ganglionic neurons have their cell bodies in the lateral horn of the
grey matter of the
A.Thoracic segment only
B. Lumbar segment only
C. Both (a) and (b) +
D. None of the above
9. Which one of the following can be blocked by atropine?
A. Decreased blood pressure caused by hexamethonium
B. Increased blood pressure caused by nicotine
C. Increased skeleton muscle strength caused by neostigmine
D. Tachycardia caused by infusion of acetylcholine +
10. ‘Fight or flight’ state can be described as
A.Sympathetic active; para-sympathetic quiescent +
B. Sympathetic active; para-sympathetic active
C. Sympathetic quiescent; para-sympathetic active
11. In male sex organs _______ system is responsible for erection.
A. Sympathetic
B. Parasympathetic +
C. Norepinephrine
D. None of the above
12. Rest and digest’ state can be described as
A. Sympathetic active; para-sympathetic quiescent
B. Sympathetic active; para-sympathetic active
C. Sympathetic quiescent; para-sympathetic active +
13. Reserpine, an alkaloid derived from the root of Rauwolfia serpentine
A. Can be used to control hyperglycaemia
B. Can cause severe depression of mood +
C. Can decrease gastrointestinal secretion and motility
D. Has no cardiac effects
14. Following enzyme is involved in the synthesis of acetylcholine

A. Choline acetyl transferase +


B. Acetyl cholinesterase
C. Both (a) and (b)
D. None of the above

15. Following is an antagonist of ganglion type nicotinic receptors


A. Tubercurarine
B. αbungarotoxin
C. Trimethaphan +
D. All of the above
16. Oxytremorine is a selective agonist of muscarinic _______ receptors
A. M1 +
B. M2
C. M3
D. M4
17. A drug that is useful in glaucoma and high-altitude sickness is
A. Acetazolamide +
B. Amiloride
C. Demeclocycline
E. Desmopressin
18. Cromolyn has as its major action
A. Block of calcium channels in lymphocytes
B. Block of mediator release from mast cells +
C. Block of phosphodiesterase in mast cells and basophils
D. Smooth muscle relaxation in the bronchi
19. Which one of the following is least likely to be useful in the therapy of hypercalcemia
A. Calcitonin
B. Glucocorticoids
C. Plicamycin
D. Thiazide diuretics +
20. Characteristics of Vitamin D and its metabolites include which one of the following
A. Act to decrease serum levels of calcium
B. Activation of their Vitamin D receptors increases cellular CAMP
C. Calcitriol is the major derivative responsible for increasing intestinal absorption of
phosphate +
D. Metabolites of Vitamin D increase renal excretion of calcium
21. Action of norepinephrine and epinephrine are terminated by
A. Reuptake into nerve terminal
B. Dilution by diffusion and uptake at extraneuronal site
C. Metabolic transformation
D. All of the above +
22. Typical results of beta-receptor activation include which one of the following
A. Hypoglycaemia
B. Lipolysis +
C. Glycogen synthesis
D. Decreased skeletal muscle tremor
23. Drugs that block the α receptor on effector cells at adrenergic nerve endings
A. Antagonize the effects of isoproterenol on the heart rate
B. Antagonize some of the effects of epinephrine on the blood pressure +
C. Antagonize the effects of epinephrine on adenylyl cyclase
D. Cause mydriasis
24. Yohimbine is an antagonist of ______ receptors.
A. α1
B. α2 +
C. both (a) and (b)
D. None of the above
25. Which of the following organs is innervated only by parasympathetic nerves
A. Iris muscles
B. Ciliary muscle +
C. Sweat glands
D. Splenic capsule
26. α1 – receptors are coupled with _______ G protein.
A. Gs
B. Gi
C. Gq +
D. Go
27. Hyoscine differs from atropine in that it
A. Exerts depressant effects on the CNS at relatively low doses +
B. Exerts more potent effects on the heart than on the eye
C. Is longer acting
D. Has weaker antimotion sickness activity
28. Which of the following anticholinergic drugs is primarily used in preanesthetic
medication and during surgery
A. Glycopyrrolate +
B. Pipenzolate methyl bromide
C. Isopropamide
D. Dicyclomine
29. Glycopyrrolate is the preferred antimuscarinic drug for use before and during
surgery because
A. It is potent and fast acting
B. It has no central action
C. It has antisecretory and vagolytic actions
D. All of the above +
30. The cardiac muscarinic receptors
A. Are of the M1 subtype
B. Are of the M2 subtype +
C. Are selectively blocked by pirenzepine
D. Function through the PIP2 → IP3/DAG pathway

31. The most suitable mydriatic for a patient of corneal ulcer is


A. Atropine sulphate +
B. Homatropine
C. Cyclopentolate
D. Tropicamide
32. The most effective antidote for belladonna poisoning is
A. Neostigmine
B. Physostigmine +
C. Pilocarpine
D. Methacholine
33. Atropine is contraindicated in
A. Cyclic AMP
B. Inositol trisphosphate
C. Diacyl glycerol
D. G protein +
34. Select the correct statement from the following about α 1 – adrenergic receptor
agonists
A. Isoproterenol > epinephrine ≥ norepinephrine +
B. Epinephrine > isoproterenol > norepinephrine
C. Isoproterenol = epinephrine ≥ norepinephrine
D. Epinephrine ≥ norepinephrine > isoproterenol
35. The most efficacious inhibitor of catecholamine synthesis in the body is
A. α - methyl – p – tyrosine +
B. α methyldopa
C. α methyl - norepinephrine
D. Pyrogallol
36. Adrenaline raises blood glucose level by the following actions except
A. Inducing hepatic glycogenolysis
B. Inhibiting insulin secretion from pancreatic β cells
C. Augmenting glucagons secretion from pancreatic α cells
D. Inhibiting peripheral glucose utilization +
37. Noradrenaline is administered by
A. Subcutaneous injection
B. Intramuscular injection
C. Slow intravenous infusion +
D. All of the above routes
38. Dobutamine differs from dopamine in that
A. It does not activate peripheral dopaminergic receptors +
B. It does not activate adrenergic β receptors
C. It causes pronounced tachycardia
D. It has good blood-brain barrier penetrability
39. Ephedrine is similar to adrenaline in the following feature
A.Potency
B. Inability to penetrate blood-brain barrier
C. Duration of action
D. Producing both α and β adrenergic effects +
40.Choose the correct statement from the following about α1 – adrenergic receptor
agonists
A. Norepinephrine > isoproterenol > epinephrine
B Norepinephrine < epinephrine > isoproterenol
C Epinephrine≥ norepinephrine >> isoproterenol +
D Epinephrine > isoproterenol > norepinephrine
41. A sympathomimetic amine that acts almost exclusively by releasing noradrenaline
from the nerve endings is
A. Ephedrine
B. Dopamine
C. Isoprenaline
D. Tyramine +
42. Low doses of adrenaline dilate the following vascular bed
A. Cutaneous
C. Renal
B. Mucosal
D. Skeletal muscle +
43. α2–adrenergic receptors are associated with following except
A. Increase in phospholipase C activity
B. Increase in potassium channel conductance
C. Decrease in calcium channel conductance
D. Increase in adenylyl cyclase activity +
44. While undergoing a surgical procedure a patient develops hypotension. Which of the
following drugs can be injected intramuscularly to raise his BP
A. Noradrenaline
B. Isoprenaline
C. Mephentermine +
D. Isoxsuprine
45. Vasoconstrictors should not be used in
A. Neurogenic shock
B. Haemorrhagic shock
C. Secondary shock +
D. Hypotension due to spinal anaesthesia
46. The drug which produces vasoconstriction despite being an α adrenergic blocker is
A. Phenoxybenzamine
B. Ergotamine +
C. Dihydroergotoxine
D. Tolazoline
47. Prazosin is an effective antihypertensive while nonselective α adrenergic blockers
are not because
A. It is the only orally active α blocker
B. It improves plasma lipid profile
C. It does not concurrently enhance noradrenaline release +
D. It improves urine flow in males with prostatic hypertrophy
48. Phenylephrine
A. Mimics the transmitter at post-synaptic receptors +
B. Displaces transmitter from axonal terminal
C. Inhibits synthesis of transmitter
D. None of the above
49. A β adrenergic blocker that is primarily eliminated unchanged by renal excretion is
A. Propranolol
B. Metoprolol
C. Alprenolol
D. Atenolol +
50. In a patient of myocardial infarction, βadrenergic blockers are used with the
following aim/aims:
A. To reduce the incidence of reinfarction
B. To prevent cardiac arrhythmias
C. Myocardial salvage
D. All of the above +
51. The β-adrenergic blocker which possesses both β-1 selectivity as well as intrinsic
sympathomimetic activity is
A. Alprenolol
B. Atenolol
C. Acebutolol +
D. Metoprolol
52. Propranolol can be used to allay anxiety associated with
A. Chronic neurotic disorder
B. Schizophrenia
C. Short-term stressful situation +
D. Endogenous depression
53. Propranolol does not block the following action of adrenaline
A. Bronchodilation
B. Lipolysis
C. Muscle tremor
D. Mydriasis +
54. The site of action of d-tubocurarine is
A. Spinal internuncial neurone
B. Motor nerve ending
C. Muscle end-plate +
D. Sodium channels in the muscle fibre
55. The fall in blood pressure caused by d- tubocuranine is due to
A. Reduced venous return.
B. Ganglionic blockade
C. Histamine release
D. All of the above +
56. The neuromuscular blocker having prominent antivagal action is
A. Pancuronium
B. Vecuronium
C. Atracurium
D. Gallamine triethiodide +
57. Neuromuscular blocking drugs do not produce central actions because
A. Nicotinic receptors are not present in the brain
B. They do not cross the blood-brain barrier +
C. They are sequestrated in the periphery by tight binding to the skeletal muscles
D. They do not ionize at the brain pH
58. Pancuronium differs from tubocurarine in that
A. It is a depolarizing blocker
B. Its action is not reversed by neostigmine
C. It can cause rise in BP on rapid I.V. injection +
D. It causes marked histamine release
59. Muscarinic receptors are G-protein coupled receptors, causing
A. Inactivation of phospholipase C
B. Activation of adenylyl cyclase
C. Activation of potassium or inhibition of calcium channels +
D. All of the above
60. Postoperative muscle soreness may be side effect of the following neuromuscular
blocker
A. d-tubocurarine
B. Succinylcholine +
C. Pancuronium
D. Atracurium
61. Following drug enhances the evoked release of acetylcholine
A. 4-aminopyridine +
B. Vesamicol
C. Magnesium ion
D. None of the above
62. Select the ergot compound which is primarily used for dementia
A. Bromocriptine
B. Ergotamine
C. Codergocrine +
D. Methysergide
63. Following muscarinic agonist is susceptible to hydrolysis by cholinesterase
A. Carbachol
B. Methacholine +
C. Bethanechol
D. Muscarine
64. Stimulation of exocrine glands by muscarinic agonist leads to
A. Sweating
B. Salivation
C. Bronchial secretions
D. All of the above +
65. β blockers are contraindicated in
A. Prophylaxis of anxiety states
B. Hyperthyroidism
C. Hypertrophic obstructive cardiomyopathy
D. Peripheral vascular disease +
66. β blockers are better avoided in
A. Bronchial asthma +
B. Pheochromocytoma
C. Myocardial infarction
D. Migraine
67. Preferential beta 1 adrenergic blocking agent is

A. Propranolol
B. Atenolol +
C. Penbutolol
D. Oxprenolol
68. Prazosin
A. Produces preferential alpha1 adrenergic receptor blockade +
B. Has a half-life of 24 hours
C. Can worsen lipid profile in hypertensive patients
D. Does not depress the baroreceptor function
69. The drug that produces neuromuscular blockade by persistent depolarization is
A. D-tubocurarine
B. Gallamine
C. Pancuronium
D. Decamethonium +
70. Agents that block neuromuscular transmission of acetyl choline acts via
A. Inhibiting acetyl choline synthesis
B. Inhibiting acetyl choline release
C. Inhibiting post-synaptic action of acetyl choline
D. All of the above +
71. The skeletal muscle relaxant causing significant release of histamine is
A. Pancuronium
B. Atracurium
C. Gallamine
D. D-tubocurarine +
72. β blockers
A. Relieve symptoms of thyrotoxicosis +
B. Should not be used along with carbimazole
C. Reduce the synthesis of hormones in the gland
D. Should not be given before thyroid surgery
73.The antimuscarinic agent preferred in the management of motion sickness is
A. Atropine methonitrate
B. Scopolamine +
C. Homatropine methyl bromide
D. Ipratropium bromide

74. Side effect which is observed only with the use of β blockers and not with other
sympathetic blocking agents
A. Sedation and drowsiness +
B. Congestive heart failure
C. Bronchoconstriction
D. Reflex tachycardia
75. Following agent inhibits the release of acetyl choline
A. Magnesium ion +
B. Triethylcholine
C. Vesamicol
D. All of the above
76. Which of the following drugs has highest α agonist activity
A. Epinephrine +
B. Norepinephrine
C. Ephedrine
D. Amphetamine
77. Atropine is antagonist to which type of muscarinic
A. M1 +
B. M2
C. M3
D. M4
78. Following are irreversible anti-cholinesterase, except
A. Parathion
B. Pyridostigmine +
C. Dyflos
D. Ecothiopate
79.The ciliary muscle of iris has predominant
A. Sympathetic innervation
B. Parasympathetic innervation +
C. Somatic innervation
D. Mixture of all the above
80. Following is a short-acting anti-cholinesterase
A. Neostigmine
B. Pyridostigmine
C. Edrophonium +
D. Physostigmine

81. The antibiotic that may accentuate the neuromuscular blockade produced by d-
tubocurarine is
(a) Pencillin G

(b) Erythromycin

(c) Streptomycin +

(d) Chloramphenicol
82. Centrally acting skeletal muscle relaxant is
(a) Carisoprodol +
(b) Dantrolene
(c) Gallamine
(d) Succinylcholine
83. The organ extremely sensitive to the action of atropine is
(a) Gastric glands
(b) Salivary glands +
(c) Urinary bladder muscle
(d) Heart
DRUGS ACTING ON CENTRAL NERVOUS SYSTEM

1. Substance secreted into the blood by a neuron is


A. Neurohormone +
B. Neuromodulator
C. Neuromediator
D. Neurotransmitter
2. Which of the following is a leukotriene receptor blocker?
A. Alprostadil
B. Aspirin
C. Ibuprofen
D. Zafirlukast +
3. The primary endogenous substrate for nitric oxide synthase is
A. Acetylcholine
B. Angiotensinogen
C. Arginine +
D. Citruline
4. Which one of the following chemicals does not satisfy the criteria for a
neurotransmitter role in the CNS?
A. Acetylcholine
B. Dopamine
C. Glycine
D. Nitric Oxide +
5. Neurotransmitters may
A. Increase chloride conductance to cause inhibition +
B. Increase potassium conductance to cause excitation
C. Increase sodium conductance to cause inhibition
D. Increase calcium conductance to cause inhibition
6. Which of the following chemicals is most likely to function as a neurotransmitter in
hierarchical systems?
A. Dopamine
B. Glutamate +
C. Metenkephalin
D. Norepinephrine
7. Suramin is an antagonist of ________ receptors
A. Purine +
B. Somatostanin
C. Neuropeptide Y
D. Neurotensin
8. Induction of various forms of synaptic plasticity is more closely associated with
_____ receptors
A. AMPA
B. Kainate
C. NMDA +
D. All of the above
9. Which one of the following drugs may increase anticoagulant effects by
displacement of warfarin from plasma protein binding sites and is inactive until
converted in the body to an active metabolite?
A. Buspirone
B. Chloral hydrate +
C. Clorazepate
D. Secobarbital
10. This hypnotic drug facilitates the inhibitory actions of GABA, but it lacks
anticonvulsant or muscle relaxing properties and has minimal effect on sleep
architecture.
A. Buspirone
B. Diazepam
C. Flurazepam
D. Zaleplon +
11. Following is GABAA agonist
A. Muscimol +
B. Baclofen
C. Bicuculline
D. None of the above
12. Chronic use of ethanol is reported to increase
A. Alcohol dehydrogenase
B. Aldehyde dehydrogenase
C. Microsomal ethanol-oxidizing system activity +
D. Monoamine oxidase
13. Following is glycine antagonists
A. Quisqualate
B. Taurine
C. Strychnine +
D. â-alanine
14. Withdrawal of antiseizure drugs can cause increased seizure frequency and
severity. Withdrawal is least likely to be a problem with
A. Clonazepam
B. Diazepam
C. Ethosuximide +
D. Phenobarbital
15. With chronic use in seizure states, the adverse effects of this drug include coarsening
of facial features, hirsutism, gingival hyperplasia, and osteomalacia.
A. Carbamazepine
B. Ethosuximide
C. Gabapentin
D. Phenytoin +
16. The most likely mechanism involved in the antiseizure activity of carbamazepine is
A. Block of sodium ion channels +
B. Block of calcium ion channels
C. Facilitation of GABA actions on chloride ion channels
D. Glutamate receptor antagonism
17. Which one of the following statements
about phenytoin is accurate?
A. Displaces sulfonamides from plasma proteins
B. Drug of choice in myoclonic seizures
C. Half-life is increased if used with phenobarbital
D. Toxicity may occur with only small increments in dose +
18. Following is GABAB agonist
A. Muscimol
B. Baclofen +
C. Picrotoxin
D. Bicuculline
19. Following is GABAA antagonist
A. Muscimol
B. Bicuculline +
C. Strychnine
D. Baclofen
20. The inhalation anaesthetic with the fastest onset of action is
A. Enflurane
B. Isoflurane
C. Nitric oxide
D. Nitrous oxide +
21. Which of the following was the first compound to be identified Pharmacologically
as a transmitter in the CNS?
A. Glycine
B. Glutamate
C. Acetylcholine +
D. Norepinephrine
22. You have a vial containing 4 mL of a 2% solution of lidocaine. How much lidocaine
is present in 1 mL?
A. 2 mg
B. 8 mg
C. 20 mg +
D. 80 mg
23. Respiratory depression following use of this agent may be reversed by
administration of flumazenil
A Desflurane
B. Fentanyl
C. Ketamine
D. Midazolam +
24. Use of this agent is associated with a high incidence of disorientation, sensory and
perceptual illusions, and vivid dreams during recovery from anaesthesia
A. Diazepam
B. Fentanyl
C. Ketamine +
D. Midazolam
25. Epinephrine added to a solution of lidocaine for a peripheral nerve block will
A. Increase the risk of convulsions
B. Increase the duration of anaesthetic action of the local anaesthetic +
C. Both (A) and (B)
D. Neither (A) nor (B)
26. Characteristics of nondepolarizing neuromuscular blockade include which one of
the following?
A. Block of post tetanic potentiation
B. Histamine blocking action
C. Poorly sustained titanic tension +
D. Significant muscle fasciculations during onset of block
27. Which of the following does not cause skeletal muscle contractions or twitching
A. Acetylcholine
B. Nicotine
C. Strychnine
D.Vecuronium +
28. Which one of the following is most effective in the management of malignant
hyperthermia?
A. Baclofen
B. Dantrolene +
C. Haloperidol
D. Succinylcholine
29. Following is a G-protein coupled receptor
A. AMPA
B. Kainate
C. NMDA
D. Metabotropic +
30. Which one of the following drugs has caused hyperkalaemia leading to cardiac
arrest in patients with neurologic disorders?
A. Baclofen
B. Dantrolene
C. Succinylcholine +
D. Tubocurarine
31. Following is an excitatory amino acid receptor antagonist?
A. Phencyclidine +
B. Quisqualate
C. Homocysteate
D. Kainate
32. Which one of the following drugs has spasmolytic activity and could also be used in
the management of seizures caused by overdose of a local anaesthetic?
A. Baclofen
B. Cyclobenzaprine
C. Dantrolene
D. Diazepam +
33. Which one of the following drugs given preoperatively will prevent postoperative
pain caused by succinylcholine?
A. Baclofen
B. Dantrolene
C. Diazepam
D. Tubocurarine +
34.The major reason why carbidopa is of value in parkinsonism is that the compound
A. Crosses the blood-brain barrier
B. Inhibits monoamine oxidase type A
C. Inhibits aromatic L-amino acid decarboxylase +
D. Is converted to the false neurotransmitter carbidopamine
35.Great caution must be exercised in the use of this drug (or drugs from the same
class) in parkinsonian patients who have prostatic hypertrophy or obstructive
gastrointestinal disease
A. Benztropine +
B. Carbidopa
C. Levodopa
D. Ropinirole
36. Which of the following statements about pramipexole is accurate?
A. Activates dopamine D2 receptors
B. Commonly a first-line therapy for Parkinson’s disease
C. May cause postural hypotension
D. All of the above +
37. Which one of the following statements about the action of phenothiazines is
accurate?
A. They activate muscarinic receptors
B. They are antiemetic +
C. They decrease serum prolactin levels
D. They elevate the seizure threshold
38. Choose the correct statement from the following?
A. Muscimol is GABAB selective
B. Bicuculline is GABAA agonist
C. Picrotoxin blocks chloride channels associated with GABAA receptors +
D. Baclofen is GABAA agonist
39.Following is the main inhibitory transmitter in the brain
A. Dopamine
B. Norepinephrine
C. Glycine
D. GABA +
40. Established clinical uses of this drug include enuresis and chronic pain
A. Bupropion
B. Fluvoxamine
C. Imipramine +
D. Phenelzine
41. Which one of the following drugs is most likely to increase plasma levels of
alprazolam, theophylline, and warfarin
A. Desipramine
B. Fluvoxamine +
C. Imipramine
D. Nefazodone
42. Which one of the following actions of opioid analgesics is mediated via activation of
kappa receptors?
A. Cerebral Vascular dilation
B. Decreased uterine tone
C. Euphoria
D. Sedation +
43. ______ antagonists are known to attenuate some of the actions of alcohol.
A. GABAA +
B. GABAB
C. NMDA
D. Glycine
44. Which one of the following effects is unlikely to occur during treatment with
amitriptyline?
A. Alpha adrenoceptor blockade
B. Elevation of the seizure threshold +
C. Mydriasis
D. Sedation
E. Urinary retention
45.Which one of the following drugs is most likely to be of value in obsessive compulsive
disorders (OCD)
A. Amitriptyline
B. Bupropion
C. Clomipramine +
D. Desipramine
46. Compared with other antidepressant drugs, mirtazapine has the distinctive ability to
act as an antagonist of
A. Alpha2 adrenoceptors +
B. Beta adrenoceptors
C. D2 receptors
D. NE transporters
47.Which one of the following signs or symptoms is likely to occur with marijuana
A. Bradycardia
B. Conjunctival reddening +
C. Hypertension
D. Increased psychomotor performance
48. The antihypertensive effects of captopril can be antagonized (reduced) by
A. Angiotensin II receptor blockers
B. Loop diuretics
C. NSAIDS +
D. Sulfonylurea hypoglycemics
49. The primary site of action of tyramine is
A. Ganglionic receptors
B. Gut and liver catechol–O–methyltransferase
C. Postganglionic sympathetic nerve terminals +
D. Preganglionic sympathetic nerve terminals
50. Which one of the following drugs has been used in ophthalmology, but causes
mydriasis and cycloplegia lasting more than 24 hours?
A. Atropine +
B. Echothiophate
C. Edrophonium
D. Ephedrine
E. Tropicamide
51. The antihypertensive drug most likely to aggravate angina pectoris is
A. Clonidine
B. Guanethidine
C. Hydralazine +
D. Methyldopa
52. This agent is currently a first-choice drug in the management of absence seizures as
well as partial, primary generalized, and tonic-colonic seizures.
A. Carbamazepine
B. Clonazepam
C. Ethosuximide
D. Valproic acid +
53.The phenothiazines have a variety of actions at different receptor types. However,
they do not appear to interact with receptors for
A. Dopamine
B. Histamine
C. Nicotine +
D. Norepinephrine
54. A psychiatric patient taking medications develops a tremor, thyroid enlargement,
and leucocytosis. The drug he is taking is most likely to be
A. Clomipramine
B. Haloperidol
C. Imipramine
D. Lithium +
55. Diuretics that increase the delivery of poorly absorbed solute to the thick ascending
limb of the nephron include
A. Furosemide
B. Indapamide
C. Mannitol +
D. Spironolactone
56. Following is used in methanol poisoning
A. Disulfiram
B. Naltrexone
C. Ethanol +
D. None of the above
57. A drug that is used in the treatment of parkinsonism and will also attenuate
reversible extrapyramidal side effects of neuroleptics is
A. Amantadine
B. Levodopa
C. Pergolide
D. Trihexyphenidyl +
58.Which of the following is a common effect of muscarinic stimulant drugs?
A. Decreased peristalsis
B. Decreased secretion by salivary glands
C. Hypertension
D. Miosis +
59. Four stages of general anaesthesia are distinctly seen with the use of
A. Halothane
B. Diethyl ether +
C. Nitrous oxide
D. Enflurane
60. Infusion of phentolamine into the cerebrospinal fluid of an experimental animal will
prevent the blood pressure – lowering action of
A. Clonidine +
B. Enalapril
C. Guanethidine
D. Reserpine
61. A drug suitable for producing a brief (5 to 15 minute) increase in cardiac vagal tone
is
A. Digoxin
B. Edrophonium +
C. Ergotamine
D. Pralidoxime
62. Propranolol and hydralazine have which of the following effects in common?
A. Decreased cardiac force
B. Decreased cardiac output
C. Decreased mean arterial blood pressure +
D. Increased systemic vascular resistance
63. Toxicities of local anaesthetics do not include
A. Cardiovascular arrhythmias and collapse (bupivacaine)
B. Convulsions (lidocaine)
C. Dizziness, sedation (lidocaine)
D. Hypertensive emergencies, strokes (procaine) +
64. Benzodiazepines are least effective in
A. Alcohol withdrawal syndromes
B Balanced anaesthesia regimens
C. Initial management of phencyclidine overdose
D. Obsessive – compulsive disorders +
65. Which one of the following drugs exerts its anticonvulsant effects by blocking
sodium channels in neuronal membranes
A. Acetazolamide
B. Carbamazepine +
C. Diazepam
D. Gabapentin
66. Cocaine intoxication has become a common problem in hospital emergency rooms.
Which one of the following drugs is not likely to be of any value in the management of
cocaine overdose?
A. Dantrolene
B. Diazepam
C. Lidocaine
D. Naltrexone +
67. Which one of the following agents used in hypertension is a prodrug that is
converted to its active form in the brain?
A. Clonidine
B. Doxazosin
C. Methyldopa +
D. Nitroprusside
68. The introduction of this drug may represent a novel approach to the treatment of
major depressive disorders since it appears to act as an antagonist at alpha2
adrenoceptors in the CNS
A. Amoxapine
B. Bupropion
C. Citalopram
D. Mirtazapine +
69. Which one of the following pairs of drugs: indication is accurate
A. Amphetamine: Alzheimer’s dementia
B. Bupropion: Acute anxiety
C. Fluoxetine: Insomnia
D. Ropinirole: Parkinson’s disease +
70. Following general anaesthetic is not metabolized
A. Halothane
B. Ether
C. Isoflurane
D. Nitrous oxide +
71. The appropriate chemical classification for meperidine is
A. Phenylpropylamines
B. Piperazines
C. 4-phenylpiperidines +
72. Which of the following neuromuscular blocking agents can cause muscarinic
responses such as bradycardia and increased glandular secretions?
A. Tubocurarine
B. Succinycholine
C. Pancuronium
D. Decamethonium +
73. Which of the following drugs is a volatile substance that is administered by
inhalation?
A. Thiopental
B. Halothane +
C. Alprazolam
D. Buspirone
74. The brief duration of action of an ultrashort acting barbiturate is due to a
A. Slow rate of metabolism in the liver
B. Low lipid solubility, resulting in a minimal concentration in the brain
C. High degree of binding to plasma proteins
D. Slow rate of excretion by the kidneys +
75. Which of the following agents would not be an alternative to Phenobarbital in the
treatment of partial seizures?
A. Trimethadione +
B. Gabapentin
C. Felbamate
D. Lamotrigine
E. None of the above
76. Which of the following diuretics is most similar in chemical structure to the
antihypertensive agent diazoxide?
A. Furosemide
B. Spironolactone
C. Mannitol
D. Chlorothiazide +
78. Following is an example of ideal anaesthetic
A. Ether
B. Halothane
C. Thiopental
D. None of the above +
79. An action common to most general anaesthetic
A. Increase in the cellular threshold of firing +
B. Potentiation of spontaneous and evoked activity of neurons
C. Inhibition of effects of glutamate
D. None of the above
80. All of the following adverse effects are associated with the use of levodopa except
A. Sialorrhea
B. Orthostatic hypotension
C. Delusions, confusion, and depression
D. Livedo reticularis +
81. The activity of which of the following drugs is dependent on a p-phenyl – N –
alkylpiperidine moiety
A. Phenobarbital
B. Chlorpromazine
C. Diazepam
D. Meperidine +
82. Parenteral calcium is used as an antidote for which of the following situations
A. Verapamil overdoses
B. Hyperkalaemia
C. Cocaine intoxication
D. Verapamil overdoses and hyperkalaemia +
83.Which of the following benzodiazepines is expected to cause the least amount of
adverse effects in the elderly?
A. Chlordiazepoxide
B. Diazepam
C. Flurazepam
D. Oxazepam +
84. The use of morphine in the patient who has had a myocardial infarction (MI)
centres around three distinct pharmacologic properties. Which of the following choices
includes these properties
A. Relief of pain, relief of anxiety, and increased oxygen supply
B. Relief of anxiety, after load reduction, increased preload
C. Relief of anxiety, preload reduction, and relief of pain +
D. Vagomimetic effect, relief of anxiety, respiratory depression
85. Ophthalmic agents contraindicated in glaucoma patients include which of the
following substances?
A. Antioxidants
B. Antipruritic
C. Decongestants +
D. Emollients
86.. Which of the following drugs is excepted to cause anticholinergic adverse effects in
the elderly?
A. Propoxyphene
B. Ciprofloxacin
C. Amitriptyline +
D. Propranolol
87.Reflex tachycardia, headache, and postural hypotension are adverse effects that limit
the use of which of the following antihypertensive agents?
A. Prazosin
B. Captopril
C. Methyldopa
D. Hydralazine +
88. Following is inverse agonist of benzodiazepine receptors
A. Flumazenil
B. β-Carbolines +
C. Chlordiazepoxide
D. Glutethimide
89. Which anticonvulsant drug requires therapeutic monitoring of Phenobarbital serum
levels as well as its own serum levels
A. Phenytoin
B. Primidone +
C. Clonazepam
D. Ethotoin
90. Zolpidem belongs to following class of drugs
A. Benzodiazepines
B. Carbamates
C. Imidazopyridines +
D. None of the above
91. What are the most common adverse effects of anticonvulsive drugs?
A. headache and dizziness
B. Gastrointestinal symptoms
C. Alternation of cognition and mentation
D. All of the above +
92. What are the important elements in considering treatment strategies for Parkinson’s
patients?
A. Age
B. Clinical presentation of disease
C. Sex
D. Etiology of the disease +
93. Which of the following anticonvulsants is contraindicated in patients with a
history of hypersensitivity to tricyclic antidepressants?
A. Phenytoin
B. Ethosuximide
C. Acetazolamide
D. Carbamazepine +
94. Which of the following agents should not be used concurrently with levodopa?
A. Diphenhydramine
B. Benztropine
C. Amantadine
D. Monoamine oxidase (MAO) inhibitors +
95. A patient with major depression should receive antidepressant therapy for at least
A. 2 weeks
B. 6 weeks
C. 2 months
D. 6 months +
96. Which of the following atypical antipsychotics would be the least sedating?
A. Quetiapine
B. Risperidone +
C. Olanzapine
D. Clozapine
97. Which of the following mood stabilizers would be most appropriate in a patient with
liver disease?
A. Lithium +
B. Valproic acid
C. Carbamazepine
D. None of the above
98. Ethinamate is _________ derivative
A. Phenothiazine
B. Urethane +
C. Piperidine
D. Tertiary alcohol
99. Which of the following factors may reduce lithium concentrations?
A. Dehydration
B. Pregnancy +
C. Reduced sodium intake
D. Nonsteroidal anti – inflammatory drugs
100. Morphine stimulates
A. Biliary and pancreatic secretions
B. Nonpropulsive rhythmic contractions of small intestine of man +
C. Propulsive contractions in small intestine of man
D. Propulsive peristaltic waves in colon
101. A drug that produces increased contraction of the sphincter iridis by local
application is a
A. Parasympathimimetic drug +
B. Parasympatholytic drug
C. Sympathomimetic drug
D. Sympatholytic drug
102. Which of the following should not be administered to a patient with myasthenia
gravis
A. Prostigmine
B. Digitalis
C. Atropine
D. Curare +
103. The toxicity of methanol is due to its conversion in the body to
A. Acetaldehyde
B. Formic acid and formaldehyde +
C. Ethyl alcohol
D. Carbonic acid
104. One of epinephrine’s actions on the heart is
A. Acceleration due to depression of the vagus
B. Increase in refractory period
C. Depression of the SA node
D. Increase in the concentration of the enzyme phosphorylase A +
105. Epinephrine HCl has little or no effect on
A. Unbroken skin
B. Conjunctiva
C. Precapillary sphincter
D. Nasal mucosa +
106. Acetylcholine has both muscarinic and nicotinic actions. The muscarinic action can
be blocked by
A. Epinephrine
B. Atropine +
C. Nicotine
D. Curare
107. Which of the following is the fastest acting anticoagulant
A. Warfarin (Coumadin)
B. Heparin +
C. Ouabain
D. Protamine sulfate
108. Phencyclidine is a non-competitive antagonist of
A. Kainate
B. AMPA
C. NMDA +
D. All of the above
109. Parkinsonism is probably due to
A. Too little dopamine in the brain +
B. Too little levodopa in the brain
C. Too little acetylcholine in the brain
D. Too much levodopa in the brain
110. A prominent toxic effect of local anaesthetics is
A. CNS stimulation +
B. CNS depression
C. Tachycardia
D. Local ischemia
111. Which of the following could be used as an antidote for curare poisoning
A. Neostigmine +
B. Atropine
C. Homatropine
D. Hexamethonium
112. Cigarette smoking increases the side effects of
A. Narcotic analgesics
B. Analeptics
C. Antidepressants
D. Oral contraceptives +
113. Which of the following may increase seizure activity in epileptic patients
A. Ethotoin
B. Phenobarbital
C. Trihexyphenidyl
D. Amantadine +
114. Trazodone (Desyrel) is used primarily as a (an)
A. Analgesic
B. Cardiotonic
C. Tranquilizer
D. Antidepressant +
115. Propofol
A. Enhances the neuromuscular blockade of tubocurarine
B. Causes marked postoperative confusion
C. Can produce involuntary movements during its use +
D. Has a delayed recovery after induction
116. A common side effect of ephedrine is
A. Rashes
B. Nervousness +
C. Blood dyscrasia
D. Drowsiness
117. Thiazide diuretics may produce an increase in blood levels of uric acid and
A. Potassium
B. BUN
C. Urea
D. Glucose +
118. Symptoms of schizophrenia are produced by following agents, except
A. Amphetamine
B. Apomorphine
C. Bromocriptine
D. Reserpine +
119. Select the anticholinesterase drug that is being used to afford symptomatic
improvement in Alzheimer’s disease
A. Echothiophate
B. Tacrine +
C. Demecarium
D. Ambenonium
120. Pilocarpine is used in
A. Glaucoma +
B. Paralytic ileus
C. Urinary retention
D. All of the above
121. Select the longer acting ocular betablocker
A. Timolol
B. Betaxolol
C. Carteolol
D. Levobunolol +
122.Which of the following is a prodrug of adrenaline used topically in glaucoma
A. Phenylephrine
B. Dipivefrine +
C. Phenylpropanolamine
D. Dorzolamide
123. Which of the following diuretics is most effective in acute congestive glaucoma
A. Indapamide
B. Amiloride
C. Mannitol +
D. Furosemide
124. Local anaesthetics block nerve conduction by
A. Blocking all cation channels in the neuronal membrane
B. Hyperpolarizing the neuronal membrane
C. Interfering with depolarization of the neuronal membrane +
D. Both (b) and (c)
125. Which of the following diseases is worsened by propranolol
A. Glaucoma
B. Raynaud’s disease +
C. Benign prostatic hypertrophy
D. Parkinsonism
126. Diazepam is used as a muscle relaxant for
A. Deep intra-abdominal operation
B. Tracheal intubation
C. Tetanus +
D. Diagnosis of myasthenia gravis
127. Which sensation is blocked first by low concentrations of a local anaesthetic
A. Temperature
B. Pain +
C. Touch
D. Deep pressure
128. Following are the MAO-inhibitors, except
A.Tranylcypromine
B. Nomifensine +
C. Moclobemide
D. Isocarboxid
129. Which of the following is a poor surface anaesthetic
A. Procaine +
B. Lidocaine
C. Tetracaine
D. Benoxinate
130. The local anaesthetic having high cardiotoxic and arrhythmogenic potential is
(a) Lidocaine
(b) Procaine
(c) Bupivacine +
(d) Chloroprocaine
131. The calcium channel blocker used for prophylaxis of migraine but not for angina
pectoris is
A. Verapamil
B. Diltiazem
C. Flunarizine +
D. Amlodipine
132. In which of the following techniques the concentration of the local anaesthetic
required is the lowest
A. Infiltration anaesthesia +
B. Field block anaesthesia
C. Nerve block anaesthesia
D. Spinal anaesthesia
133. Captopril produces greater fall in blood pressure in
A. Diuretic treated patients +
B. Patients having low plasma rennin activity
C. Sodium replete normotensive individuals
D. Untreated CHF patients
134. Enalapril differs from captopril in that
A. It blocks angiotensin II receptors
B. It does not produce cough as a side effect
C. It is less liable to cause abrupt first dose hypotension +
D. It has a shorter duration of action
135. Spinal anaesthesia is not suitable for
A. Vaginal delivery
B. Lower segment caesarean section
C. Prostatectomy
D. Operations on mentally ill patients +
136. Following is a MAO-inhibitor and also has anti-tubercular activity
A. Iproniazid +
B. Phenelzine
C. Moclobemide
D Isocarboxid
137. Which of the following is true of nitrous oxide?
A. It irritates the respiratory mucosa
B. It has poor analgesic action
C. It is primarily used as a carrier and adjuvant to other anaesthetics +
D. It frequently induces post anaesthetic nausea and retching
138. The factor that effects rate of elimination of general anaesthetics is
A. Pulmonary ventilation
B. Blood flow
C. Solubility in blood and tissue
D. All of the above +
139. Which of the following is a sign of deep anaesthesia?
A. Appearance of tears in eyes
B. Resistance to passive inflation of lungs
C. Fall in blood pressure +
D. Patient makes swallowing movements
140. Which inhalational general anaesthetic is metabolized in the body to a significant
extent
A. Sevoflurane
B. Isoflurane
C. Ether
D. Halothane +
141. The drug/drugs used mainly for induction
of general anaesthesia is/are
A. Thiopentone sodium +
B. Fentanyl + droperidol
C. Ketamine
D. All of the above
142. Following agent potentiates actions of
GABA
A. Volatile anaesthetics
B. Barbiturates +
C. Propofol
D. All of the above
143. Which of the following drugs is routinely used in preanesthetic medication for
prolonged operations
A. Atropine
B. Morphine
C. Promethazine
D. Ranitidine +
144. Hypnotic benzodiazepines increase the period of time spent in the following stage
of sleep
A. Stage II +
B. Stage III
C. Stage IV
D. REM stage
145. The primary mechanism of action of benzodiazepines is
A. Dopamine antagonism
B. Adenosine antagonism
C. Opening of neuronal chloride channels
D. Facilitation of GABA–mediated chloride influx +
146. The following drug is used to reverse the CNS depression produced by diazepam
A. Nikethamide
B. Doxapram
C. Physostigmine
D. Flumazenil +
147. Which of the following is not a CNS depressant but increases the tendency to
fall asleep at night?
A. Pyridoxine
B. Methaqualone
C. Melatonin +
D. Ethanol
148. The following antiepileptic drug is also effective in manic-depressive illness
A. Ethosuccimide
B. Primidone
C. Phenobarbitone
D. Carbamazepine +
149. The preferred drug for status epilepticus is
A. Intravenous diazepam +
B. Intravenous phenytoin sodium
C. Intramuscular phenobarbitone
D. Rectal paraldehyde
150. The most effective single drug in parkinsonism is
A. Bromocriptine
B. Selegiline
C. Levodopa D. +
D. Biperiden
GENERAL PHARMACOLOGY

1. All of the following are general mechanisms of drug permeation Except

A. Aqueous diffusion

B. Aqueous hydrolysis+

C. Lipid diffusion

D. Pinocytosis or endocytosis

2. If the plasma concentration of a drug declines with “first-order kinetics”, this means that

A. There is only one metabolic path for drug disposition

B. The half-life is the same regardless of the plasma concentration+

C. The drug is largely metabolized in the liver after oral administration and has low
bioavailability elimination

D. The rate of elimination is proportionate to the rate of administration at all times

3. Regarding termination of drug action

A. Drug must be exerted from the body to terminate their action

B. Metabolism of drugs always increases their water solubility

C. Metabolism of drugs always abolishes their pharmacologic activity

D. Hepatic metabolism and renal excretion are the two most important mechanisms involved +

4. Distribution of drugs to specific tissues

A. Is independent of blood flow to the organ

B. Is independent of the solubility of the drug in that tissue

C. Depends on the unbound drug concentration gradient between blood and tissue+

D. Is increased for drugs that are strongly bound to plasma proteins


5. A physical process by which a weak acid becomes less water-soluble and more lipid-
soluble at low ph is

A. Distribution

B Elimination

C. First pass- effect

D. Protonation+

6. Dose-response curves are used for drug evaluation in the animal laboratory and in the
clinic, Quantal dose-response curves are often

A. Used for determining the therapeutic index of a drug+

B. Used for determining the maximal efficacy of a drug

C. Invalid in the presence of inhibitors of the drug being studied

D. Obtained from the study of intact subject but not from isolated tissue preparations

7. The following are excreted faster in basic urine

A. Weak acids+

B. Strong acids

C. Weak Bases

D. None of the above

8. Which of the following statements about spare receptors is most correct ?

A. Spare receptors, in the absence of drug, are sequestered in the cytoplasm

B. Spare receptors will be detected if the intracellular effect of drug-receptor interaction lasts
longer than the drug-receptor interaction itself +

C. Spare receptors influence the maximal efficacy of the drug-receptor system

D. Spare receptors activate the effector machinery of the cell without the need for a drug
9. Which of the following terms best describes an antagonist that interacts directly with the
agonist and not at all or only incidentally, with the receptor

A. Pharmacological antagonist

B. Partial agonist

C. Physiological antagonist

D. Chemical antagonist+

10. Which of the following terms best describes a drug that blocks the action of epinephrine
at its receptors by occupying those receptors without activating them

A. Pharmacological antagonist+

B. Partial agonist

C. Physiological antagonist

D. Chemical antagonist

11. Which of the following provides information about the variation in sensitivity of the
drug within the population studied

A. Maximal efficacy

B. Therapeutic index

C. Drug potency

D. Quantal dose-response curve +

12. Which of the following most accurately describes the transmembrane signaling process
involved in steroid hormone action

A. Action on a membrane spanning tyrosine kinase

B. Activation of a G protein which activates or inhibits adenyl cyclase

C. Diffusion into the cytoplasm and binding to an intracellular receptor +

D. Diffusion of “STAT” molecules across the Membrane


13. Which of the following is a phase II drug metabolizing reaction

A. Acetylation +

B. Deamination

C. Hydrolysis

D. Oxidation

14. Which of the following drugs may inhibit the hepatic microsomal P450 responsible for
warfarin metabolism

A. Cimetidine +

B. Ethanol

C. Phenobarbital

D. Procainamide

15. With regard to clinical trials of new drugs, which of the following is most correct

A. Phase I involves the study of a small number of normal volunteers by highly trained clinical
pharmacologists +

B. Phase II involves the use of the new drug in a large number of patients (100-5000) who have
the disease to be treated

C. Phase III involves the determination of the drug’s therapeutic index by the cautious induction
of toxicity

D. Chemical antagonist

16. Animal testing of potential new therapeutic agents

A. Extends over a time period of at least 3 years in order to discover late toxicities +

B. Requires the use of at least two primate species, e.g. Monkey and baboon

C. Requires the submission of histopathologic slides and specimens to the FDA for government
evaluation
D. Has good predictability for drug allergy-type reactions

17. The “dominant lethal” test involves the treatment of a male adult animal with a
chemical before mating; the pregnant female is later examined for fetal death and
abnormalities. The dominant lethal test therefore is a test of

A. Teratogenicity

B. Mutagenicity +

C. Carcinogenicity

D. All of the above

18. The Ames test is a method for detecting

A. Carcinogenesis in rodents

B. Carcinogenesis in primates

C. Teratogenesis in any mammalian species

D. Mutagenesis in bacteria +

19. “Nicotinic” sites include all of the following except

A. Bronchial smooth muscle +

B. Adrenal medullary cells

C. Parasympathetic ganglia

D. Skeletal muscle

20. A good example of chemical antagonism

A. Heparin & Protamine +

B. Protamine & Zinc

C. Heparin & Prothrombin

D. All the above


21. Which of the following agents is a prodrug that is much less toxic in mammals than in
insects

A. Acetylcholine +

B. Bethanechol

C. Physostigmine

D. Pilocarpine

22. Phenylephrine causes

A. Constriction of vessels in the nasal mucosa +

B. Increased gastric secretion and motility

C. Increased skin temperature

D. Miosis

23. Pretreatment with propranolol will block which one of the following

A. Methacholine-induced tachycardia +

B. Nicotine-induced hypertension

C. Norepinephrine-induced bradycardia

D. Phenylephrine-induced mydriasis

24. Most drug receptors are

A. Small molecules with a molecular weight between 100 and 1000

B. Lipids arranged in a bilayer configuration

C. Proteins located on cell membranes or in the cytosol +

D. DNA molecules

25. With regard to distribution of a drug from the blood into tissues

A. Blood flow to the tissue is an important determinant


B. Solubility of the drug in the tissue is an important determinant

C. Concentration of the drug in the blood is an important determinant

D. All of the above are important determinants +

26. The pH value is calculated mathematically as the

A. Log of the hydroxyl ion (OH–) concentration

B. Negative log of the OH– concentration

C. Log of the hydrogen ion (H+) concentration

D. Negative log of the H+ concentration+

27. Which property is classified as colligative?

A. Solubility of a solute

B. Osmotic pressure +

C. Hydrogen ion (H+) concentration

D. Dissociation of a solute

28. The colligative properties of a solution are related to the

A. pH of the solution

B. Number of ions in the solution

C. Total number of solute particles in the solution +

D. Number of unionized molecules in the solution

29. The pH of a buffer system can be calculated with the

A. Noyes – Whitney equation

B. Henderson – Hasselbalch equation +

C. Michaelis – Menten equation

D. Yong equation
30. Which mechanism is most often responsible for chemical degradation?

A. Racemization

B. Photolysis

C. Hydrolysis +

D. Decarboxylation

31. Which equation is used to predict the stability of a drug product at room temperature
from experiments at accelerated temperature?

A. The stokes equation

B. The Yong equation

C. The Arrhenius equation +

D. The Michaelis – Menten equation

32. Based on the relation between the degree of ionization and the solubility of a weak acid,
the drug aspirin (pKa 3.49) will be most soluble at

A. pH 1.0

B. pH 2.0

C. pH 3.0

D. pH 6.0 +

33. The particle size of the dispersed solid in a suspension is usually greater than

A. 0.5 µm +

B. 0.4 µm

C. 0.3 µm

D. 0.2 µm

34. In the extemporaneous preparation of a suspension, levigating is used to


A. Reduce the zeta potential

B. Avoid bacterial growth

C. Reduce particle size +

D. Enhance viscosity

35. Active transport differs from facilitated transport in following ways, except

A. Carrier is involved +

B. It is against concentration gradient

C. Energy is required

D. All of the above

36. Vanishing cream is an ointment that may be classified as

A. A water –soluble base

B. An oleaginous base

C. An absorption base

D. An emulsion base +

37. Rectal suppositories intended for adult use usually weigh approximately

A. 1g

B. 2g +

C. 3g

D. 4g

38. In the fusion method of making cocoa butter suppositories which substance is most
likely to be used to lubricate the mold?

A. Mineral oil +

B. Propylene glycol
C. Cetyl alcohol

D. Stearic acid

39. A very fine powdered chemical is defined as one that

A. Completely passes through a # 80 sieve

B. Completely passes through a # 120 sieve +

C. Completely passes through a # 20 sieve

D. Passes through a # 60 sieve and not more than 40% through a # 100 sieve

40. Which technique is typically used to mill camphor?

A. Trituration

B. Levigation

C. Pulverization and intervention +

D. Geometric dilution

41. Which type of paper best protects a divided hygroscopic powder?

A. Waxed paper +

B. Glassine

C. White bond

D. Blue bond

42. Which capsule size has the smallest capacity?

A. 5 +

B. 4

C. 1

43. The shells of soft gelatin capsules may be made elastic or plastic–like by the addition of

A. Sorbitol +
B. Povidone

C. Polyethylene glycol

D. Lactose

44. Nonionic surface-active agents used as synthetic emulsifiers include

A. Tragacanth +

B. Sodium lauryl sulphate

C. Sorbitan esters(spans)

45. A ceramic mortar may be preferable to a glass mortar when

A. A volatile oil is added to a powder mixture

B. Colored substances (dyes) are mixed into a powder

C. Comminution is desired in addition to mixing +

46. Divided powders may be dispensed in

A. Individual-dose packets +

B. A bulk container

C. A perforated, sifter –type container

47. Agents that may be used to coat enteric coated tablets include

A. Hydroxypropyl methyl cellulose


B. Carboxymethyl cellulose
C. Cellulose acetate phthalate +
48. Active transport of a substance across biological membrane has the following
characteristics except
A. It is specific
B It is pH dependent +
C It is saturable
D It requires metabolic energy
49. Bioavailability of drug refers to
A. Percentage of administered dose that reaches systemic circulation in the unchanged form +
B. Ratio of oral to parental dose
C. Ratio of orally administered drug to that excreted in the feces
D. Ratio of drug excreted unchanged in urine to that excreted as metabolites

50. Factor which can affect the absorption of drug is


A. Dissolution rate
B. Particle Size
C. Lipid Solubility
D. All the above +

51. Excipients are

A. Pharmacologically inert substances


B. Used to mask an unpleasant taste
C. Used to increase solubility or stability to the agent
D. Employed to add bulk to the active agent used in small quantities +

52. All the below mentioned drugs cause enzyme inhibition in man except one

A. Acetazolamide

B. Allopurinol

C. Meprobamate +

D. Disulfiram
53. All the below mentioned drugs cause enzyme induction in man except one

(a) Phenytoin
(c) Griseofulvine

(b) Phenobarbitone
(d) Enalapril +
54. A common oral problem caused by herpes simplex type I virus (HSV-1) is
A. Aphthous ulcers
B. Canker sores
C. Aphthous stomatitis
D. Fever blisters +
55. The definition of a surfactant (an ingredient in toothpaste) can best be described by
which of the following statements
A. Prevents drying of the preparation
B. Removes debris by its detergent action and causes foaming, which is usually desired by the
patient +
C. Physically removes plaque and debris
D. Determines the texture, dispersiveness, and appearance of the product

56. Which is not a risk factor for hyperphosphatemia and death from sodium phosphate
enemas when used in children?
A. Renal insufficiency
B. Hirschsprung’s disease
C. Anorectal malformations
D. Children between the ages of 6 and 12 years +

57. Which of the following factors is associated with an increased risk of noncompliance in
the elderly?
A. Polypharmacy +
B. Hypertension
C. Male gender
D. Living with a spouse in an isolated environment
E. Expensive medications

58. The principal difference between competitive and non-competitive inhibition is


A. Extent of receptor site blocking
B. Whether inhibition occurs
C. Extent of enzyme3 inhibition +
D. Degree of agonism
59. Drug administrated through which of the following routes is mot likely to be subjected
to first-pass metabolism:
A. Oral +
B. Sublingual
C. Subcutaneous
D. Rectal
60. Many receptors use distinct hetero _________ GTPä-binding regulatory proteins
A. Tetrameric
B. Trimeric +
C. Dimeric
D. Monomeric

61. Alkalization of urine hastens the excretion of


A. Weakly basic drugs
B. Weakly acidic drugs +
C. Strong electrolytes
D. Nonpolar drugs

62. Majority of drugs cross biological membranes primarily by


A. Weakly basic drugs +
B. Weakly acidic drugs
C. Strong electrolytes
D. Nonpolar drugs
63. The most important factor which governs diffusion of drugs across capillaries other
than those in the brain is
A. Blood flow through the capillary +
B. Lipid solubility of the drug
C. pKa value of the drug
D. pH of the medium
64. Active transport of a substance across biological membrane has the following
characteristics except
A. It is specific
B. It is pH dependent +
C. It is saturable
65. Bioavailability differences among oral formulations of a drug are most likely to occur if
the drug
A. Is freely water soluble
B. Is completely absorbed
C. Is incompletely absorbed +
D. Undergoes little first-pass metabolism

66. An antagonist has


(a) Intrinsic activity and no affinity
(b) Only intrinsic activity and no affinity
(c) No intrinsic activity and no affinity
(d) Affinity same as agonist and devoid of intrinsic activity +

67. The most important factor governing absorption of a drug from intact skin is
A. Molecular weight of the drug
B. Site of application
C. Lipid solubility of the drug +
D. Nature of the base used in the formulation

68. Redistribution is a feature of


A. Highly plasma protein bound drugs
B. Depot preparations
C. Poorly lipid soluble drugs
D. Highly lipid soluble drugs +
69. Weakly acidic drugs
A. Are bound primarily to a1 acid glycoprotein in plasma
B. Are excreted faster in alkaline urine +
C. Are highly ionized in the gastric juice
D. Do not cross blood—brain barrier
70. High plasma protein binding
A. Increases the volume of distribution of the drug
B. Facilitates glomerular filtrtion of the drug
B. Minimizes drug interactions
D. Generally makes the drug long acting +

71. Biotransformation of drugs is primarily directed to


A. Activate the drug
B. Inactivate the drug
C. Convert lipid soluble drugs into nonlipid soluble metabolites+
D. Convert nonlipid soluble drugs into lipid soluble metabolites

72. drug may be best administered by:


(a) Oral route
(b) Inhalation +
(c) Sublingual route
(d) Intrathecal route

73. Which of the following cytochrome P450 isoenzymes is involved in the metabolism of a
large number of drugs in human beings and has been implicated in some dangerous drug
interactions:
A. CYP 3A4 +
B. CYP 2C9
C. CYP 2E1
D. CYP 1A2

74. The most commonly occurring conjugation reaction for drugs and their metabolites is
A. Glucuronidation +
B. Acetylation
C. Methylation
D. Glutathione conjugation
75. G-protein coupled receptors span the plasma membrane as a bundle of _____ alpha
helices
A. One
B. Three
C. Seven +
D. Ten
76. Which of the following drug metabolizing reactions is entirely nonmicrosomal
A. Glucuronide conjugation
B. Acetylation +
C. Oxidation
D. Reduction
77. Induction of drug metabolizing enzymes involves
A. A conformational change in the enzyme protein to favor binding of substrate molecules
B. Expression of enzyme molecules on the surface of hepatocytes
C. Enhanced transport of substrate molecules into hepatocytes
D. Increased synthesis of enzyme protein +
78. Drugs which undergo high degree of First-pass metabolism in liver
A. Have oral bioavailability
B. Are excreted primarily in bile
C. Are contraindicated in liver disease
D. Exhibit zero order kinetics of elimination +
79. Glomerular filtration of a drug is affected by its
A. Lipid solubility
B. Plasma protein binding +
C. Degree of ionization
D. Rate of tubular secretion
80. If a drug undergoes net tubular secretion, its renal clearance will be
A. More than the glomerular filtration rate +
B. Equal to the glomerular filtration rate
C. Less than the glomerular filtration rate
81. Which of the following is not a primary/ fundamental, but a derived pharmacokinetic
parameter
A. Bio-availability
B. Volume of distribution
C. Clearance
D. Plasma half life +
82. If a drug is eliminated by first order Kinetics
A. constant amount of the drug will be eliminated per unit time
B. Its clearance value will remain constant +
C. Its elimination half-life will increase with dose
D. It will be completely eliminated from the body in 2 x half-life period

83. If a drug has a constant bio-availability and first order elimination, its maintenance
dose rate will be directly proportional to its
A. Volume of distribution
B. Plasma protein binding
C. Lipid solubility
D. Total body clearance +

84. The following dose of a drug is governed by its


A. Aqueous diffusion
B. Aqueous hydrolysis
C. Lipid diffusion +
D. Pinocytosis or endocytosis

85. Monitoring plasma drug concentration is useful while using


A. Antihypertensive drugs
B. Levodopa
C. Lithium carbonate +
D. MAO inhibitors
86. Microsomal enzyme induction has one of the following features
A. Takes about one week to develop
B. Results in increased affinity of the enzyme for the substrate +
C. It is irreversible
D. Can be used to treat acute drug poisonings
87. Which of the following is a competitive type of enzyme inhibitor
A. Acetazolamide
B. Disulfiram
C. Physostigmine +
D. Theophylline
88. What is true in relation to drug receptors
A. All drugs act through specific receptors
B. All drug receptors are located on the surface of the target cells
C. Agonists induce a conformational change in the receptor +
D. Partial agonists have low affinity for the Receptor

89. A partial agonist can antagonize the effects of a full agonist because it has
A. High affinity but low intrinsic activity +
B. Low affinity but high intrinsic activity
C. No affinity and low intrinsic activity
D. High affinity but no intrinsic activity

90. Receptor agonists possess


A. Result in increased smooth endoplasmic reticulum
B. Result in increased rough endoplasmic reticulum
C. Result in decreased enzymes in the soluble cytoplasmic fraction +
E. Require 3–4 months to reach completion

91. Agonists affect the receptor molecule in the following manner

A. Alter its amino acid sequence

B. Denature the receptor protein


C. Alter its folding or alignment of subunits +

D. Induce covalent bond formation

92. Receptors perform the following function/Functions

A. Ligand recognition

B. Signal transduction

C. Both ligand recognition and signal transduction n+

D. Disposal of agonists and antagonists

93. Which of the following receptor types has 7 helical membrane, spanning amino acid
segments with 3 extracellular and 3 intracellular loops

A. Tyrosine protein kinase receptor

B. Gene expression regulating receptor

C. Intrinsic ion channel containing receptor

D. G protein coupled receptor +

94. Which of the following is a G protein coupled receptor

A. Muscarinic cholinergic receptor +

B. Nicotinic cholinergic receptor

C. Glucocorticoid receptor

D. Insulin receptor

95. Placebo effects result presumably from the

A. Physician-patient relationship

B. Mental set up imparted by the therapeutic settings

C. Mental set up imparted by the physician


D. All of the above +

96. All of the following sub serve as intracellular second messengers in receptor mediated
signal transduction except

A. Cyclic AMP

B. Inositol trisphosphate

C. Diacyl glycerol

D. G protein +

97. A receptor which itself has enzymatic property is

A. Insulin receptor +

B. Progesterone receptor

C. Thyroxine receptor

D. Glucagon receptor

98. Down regulation of receptors can occur as a consequence of

A. Continuous use of agonists +

B. Continuous use of antagonists

C. Chronic use of CNS depressants

D. Denervation

99. When therapeutic effects decline both below and above a narrow range of doses, a drug
is said to exhibit

A. Ceiling effect

B. Desensitization

C. Therapeutic window phenomenon +

D. Nonreceptor mediated action


100. ‘Drug efficacy’ refers to

A. The range of diseases in which the drug is beneficial

B. The maximal intensity of response that can be produced by the drug +

C. The therapeutic dose range of the drug

D. The therapeutic index of the drug

101. Tachyphylaxis is

A. A drug interaction between two similar types of drugs

B. Rapidly developing tolerance+

C. A synergism between two types of drugs

D. None of the above

102.Drug A in a dose of 10 mg produces same response as with 100 mg of drug B

A. Drug A is 10 times more potent than drug B +

B. Drug B is 10 times more potent than drug A

C. Drug A is 10 times more efficacious than drug B

D. Both are equally potent

103. Teratogenicity is

A. The acute reaction to drugs

B. Intolerance to drugs

C. Tumour forming action of the drugs

D. Malformation of the fetus +

104. The chances of foetal malformation with a teratogenic drug is maximum

A. During first trimester of pregnancy +

B. During second trimester of pregnancy


C. During third trimester of pregnancy

D. When given just prior to the labor

105. Phocomelia is a known teratogenic effect of

A. Anticancer drugs

B. Antiviral drugs

C. Antiepileptic drugs

D. Thalidomide +

106. Which of the following drugs are known to cause toxic cataract?

A. Chloroquine

B. Ergot

C. Phenothiazine

D. All of the above +

107. The passage of drugs into the foetus from placenta

A. Is by active transport

B. Is by passive diffusion +

C. Is by carrier mediated transport

D. By any of the above methods

108. Idiosyncrasy reaction of a drug is

A. A type of hypersensitivity reaction

B. A type of drug antagonism

C. Unpredictable, inherent, qualitatively abnormal reaction of a drug +

D. Quantitatively exaggerated response


109. Two drugs having similar effects are termed as

A. Heterergic drugs

B. Isomer drugs +

C. Homergic drugs

D. Antagonistic drugs

110. AntagonisticAntagonism between barbiturate and amphetamine is termed as

A. Non-competitive antagonism

B. Physiological antagonism +

C. Competitive antagonism

D. Synergism

111. Which one of the following is an example of physical or chemical interaction?

A. Warfarin plus salicylates–prolongation of anticoagulant effect and bleeding tendency

B. Methotrexate plus sulfonamides–pancytopenia

C. Heparin plus protamine–reversal of heparin effect +

D. Sulfonamides plus salicylate–sulfa toxicity

112. First order kinetics of the drugs is called when

A. A constant fraction of the drug is removed in per unit time +

B. A constant amount of the drug is removed in per unit time

C. Total amount of the drug is removed in one hour

D. Total amount of the drug is removed in first passage through the kidneys

113. For the drugs which follow first order kinetics, after 4 half life the elimination will be
approximately

A. 40%
B. 94% +

C. 25%

D. 4%

114. Passive diffusion of a drug across cell membrane is low when its molecular mass is
greater than

A. 50–100 Da

B. 100–200 Da +

C. 200–300 Da

D 300–400 Da

115. Passage of drug across most capillary endothelial membranes is dependent upon

A. Lipid solubility +

B. pH gradient

C. Blood flow

D. All of the above

116. Following receptors are membrane proteins, except

A. Receptors for fast neurotransmitters, coupled directly to an ion channel

B. Receptors for many hormones and slow transmitters, coupled to effector system

C. Receptor for insulin and various growth factors, which are directly linked to tyrosine kinase

D. Receptors for steroid hormone +

117. pH difference between extracellular and intracellular fluid is

A. Nil

B. 0.2

C. 0.4 +
D. 0.8
1) Beta blocker which has peripheral vasodilation effect is:
a) atenolol
b) acebutolol
c) metoprolol
d) labitolol
e) +carvedilol

2) T-half of a drug can determine all the following:


a) +closing interval
b) elimination time
c) +therapeutic dosage
d) steady plasma conc.

3) Phototoxicity is commonest with:


a) +doxycycline
b) minocycline
c) prostacycline
d) oxytetracycline

4) Gout is precipitate by all except:


a) frusemide
b) thiazide
c) PYZ
d) +sulfinpyrazone

5) Effect of parasympathetic (muscarinic ) on heart is:


a) refractory period is decresed
b) +conduction is decresed
c) forse of attrial contraction is increased
d) P_R interval is increased

6) Call of the following antibiotics act on cell wall except:


a) ampicilinc
b) +griseofulvin
c) bacitracin
d) none

7) CNS stimulant action of nikethamide has .target cells mainly in:


a) +medulla
b) mid brain
c) pons
d) cerebral cortex

8) Nicotinic receptor are seen in:


a) +sk. Muscle
b) cardiac mus.
c) Visceral smooth mus.
d) Salivary gland

9) Sparfloxacin and tarfinadine can cause


a) nephropathy
b) +ventricular arrhythmia
c) myopathy
d) all

1
10) Dapsone is used in:
a) erythema
b) SLE
c) Lichen planus
d) +Dermatitis herpetiformis

11) Cyclosporine A inhibites:


a) +T cells
b) CD8
c) CD4
d) В cells

12) Increase renal blood flow is caused by:


a) +dopamine
b) dobutamine
c) ]adrenaline
d) Non adrenaline

13) True abt. Lente insulin:


a) +made of 3 parts zinc suspension (amorphorous) and 7 parts zinc suspension (crystalline)
b) duration of action is 36 hrs.
c) clear alkaline solution
d) none

14) Which of the following is fluroquinolone:


a) +pefloxacin
b) cefotaxime
c) urodopenicilline
d) none

15) Metoprolol is the member of:


a) a1 selective group
b) a2- selective group
c) +β1-selective group
d) β2- selective group

16) The principal adverse effect of phentolamine are related to:


a) Gastric secretion
b) Brochi constriction
c) Smooth muscle dilation
d) +Cardiac stimulation

17) Adverse effect of neuromuscular blocking drugs:


a) Hypokalemia
b) +Hyperkalemia
c) Hypomagnemia
d) Hypermagnemia

18) The crossing the blood-brain barrier of trimethaphan is:


a) +Absent
b) Poor
c) Good
d) Excellent

2
19. Direct action of physostigmine on cholinoceptors is:
a. +Absent
b. Poor
c. Good
d. Excellent

19) Lipid soluble agents of anticholiesterase have more marked effect on:
a) Skeletal muscles
b) Nicotinic
c) Eye
d) +CNS

20) High doses of parasympathomimetic drugs may produce:


a) Constriction of arteries and dilation of veins
b) Constriction of veins and dilation of arterials
c) Constriction of veins and arterials
d) +Dilation of veins and arterials

21) Topical instillation of atropine causes:


a) +miosis
b) mydriases
c) abolition of light reflex
d) cycloplegia

22) Remission inducing drug for Rh arthritis include:


a) aspirin
b) +gold thiol
c) ibuprofen
d) all

23) Not used in H. pylori gastritis is a:


a) +cisapride
b) clarithromycin
c) metronidazole
d) colloidal bismuth

24) Protein binding of a drug helps in:


a) distribution
b) +prolonged half life
c) limiting metabolism
d) all

25) Аll the drug can be given in pt. with bronchial asthma except:
a) timolol eyedrop
b) acetaminophen
c) ketorolac
d) +nefopam

26) Opioids act by:


a) cellular hyper polarization
b) reduced neurotransmitter reuptake
c) +reduced level of subst. P
d) evoking glutamate release

3
27) Storage of Fe is:
a) heamoglobine
b) myoglobone
c) +feritinine
d) heamosiderin

28) Thiazides is:


a) Torsemide
b) dichlorphenamide
c) Acetazolamide
d) +Chlorthalidone

29) Isoproterenol has duration of action:


a. 30-60 minutes
b. +60-90 minute
c. 90-120 minute
d. 120-150 minute

30) Pre/post ganglionic fibre ratio of sympathetic nervous system:


a) +1:1-1:2
b) 1:2-1:3
c) 1:3-1:4
d) 1:4 —1:4

31) Muscarinic receptors have been divided into


a) 3 subtypes
b) 4 subtypes
c) +5 subtypes
d) 6 subtypes

32) M-cholinomimetics:
a) Galantamine
b) +Aceclydine
c) Pentamine
d) Carbacholine

33) Response of eyes to the parasympathomimetic drugs:


a) Mydriasis
b) +Miosis
c) Increases of intraocular tension
d) Relax of accommodation

34) Irreversible anticholiesterases:


a) Physostigmine
b) +Phosphacol
c) Neostigmine
d) Edrophonium

35) Which of the following is an osmotic diuretics:


a) +urea
b) metolazone
c) bumetanide
d) d-acetazolamide

4
36) Therapeutic index of a drag is a indicator of:
a) potency
b) safety
c) +toxicity
d) efficacy

37) Tachyphylaxis is a:
a) +acute tolerance
b) chronic tolerance
c) both
d) none

38) Drug used in anaphylaxis is:


a) nor epinephrine
b) +epinephrine
c) dopamine
d) antihistaminic

39) Breast feeding is not contraindicated when the mother is taking the following medication except:
a) theophyline
b) cloromphenicol
c) metronidazole
d) cimetidine

40) Antacids are:


a) physical antagonist
b) chemical antagonist
c) +reversible antagonist
d) irreversible antagonist

41) The toxic effects of a large dose of nicotine is:


a) Salivation
b) Nausea
c) +Convulsions
d) Cutaneous vasodilation

42) Excessive doses of lidocaine cause:


a) Paresthesias
b) +Hypotension
c) Hypertension
d) Disorientation

43) The CNS effect of nicotine:


a) +Tremors
b) Diarrhea
c) Hypotension
d) Decrease of hear rate

44) In what stage of anesthesia may occur the regular respiration and relaxation of the skeletal muscle:
a) Analgesia
b) Excitement
c) +Surgical anesthesia
d) Medullary paralysis

5
45) Heparin therapy is monitored by:
a. ВТ
b. CT
c. +PT
d. All of the above

46) Which of the following drug is useful in dissolving gall bladder stone:
a) +chenodoxycholic acid
b) lactulose
c) clofibrate
d) none

47) Tetracycline can cause all except:


a) +gray baby syndrome
b) hepatotoxicity
c) pain on injection
d) fanconis anaemia

48) Sulfonamide derivates is:


a) +Torsemide
b) dichlorphenamide
c) Acetazolamide
d) Chlorthalidone

49) Methacholine usually could be used in treatment of:


a. +Glaucoma
b. Bronchial constriction
c. Hypertension
d. Hypotension

50. Neostigmine CNS effect is:


a. +Absent
b. Poor
c. Good
d. Excellent

51. Atropine is highly selective for:


a. a-receptors
b. β-receptors
c. +m-receptors
d. n-receptors

52. Good administration for trimethaphan is:


a. Oral
b. Subcutaneous
c. Intramuscular
d. +Intravenous

53. The duration of action of succinyicnoline is


a. + 5-10 minutes
b. 10-20 minutes
c. 20-30 minutes
d. 30-40 minutes

6
54. Location of β-1 receptors:
a. Bronchi
b. Urinary tract
c. Uterus
d. +Heart

55. Irreversible antagonist drug is:


a. +Phenoxybenzamine
b. Prazosine
c. Labetalol
d. Phentolamine

56. Botulinus toxin influence on the release of acetylcholine


a. Induce
b. Increase
c. +Inhibit
d. Doesn’t influence

57. Nicotinic receptors have been divided into


a. +2 subtypes
b. 3 subtypes
c. 4 subtypes
d. 5 subtypes

58. Physostigmine is well absorbed from


a. Oral administration
b. Intravenous
c. Intramuscular
d. +All sites

59. Effect of atropine on the iris persists:


a. 12-24 hours
b. 24-48 hours
c. 48-72 hours
d. +More than 72 hours

60. The crossing the blood-brain barrier of trimethaphan is:


a. +Absent
a. Poor
b. Good
c. Excellent

61. Selective antagonist for the β receptors


a. Yohimbine
b. Clonidine
c. Methoxamine
d. +Prazosin

62. The principal adverse effect of phentolamine are related to


a. Gastric secretion
b. Brochi constriction
c. Smooth muscle dilation
d. +Cardiac stimulation

7
63. A half-life of esmolol is:
a. +5-10 minutes
b. 10-30 minutes
c. 30-40 minutes
d. 40-50 minutes

64. Metoprolol is the member of:


a. a1-selective group
b. a2- selective group
c. + β1 selective group
d. β2 selective group

65. Uses of a-blockers is not:


a. Pheochromocytoma
b. Diabetes
c. Thyrotoxicosis
d. +Cardiac arrhythmias

66. Clinical pharmacology includes


a) Pharmacokinetics and pharfnacodynamics
b) Treatment of systemic infection
c) Science of compounding and dispending drug
d) +Application of pharmacological information

67. Activates a receptor to produce an effect similar to physiological signal:


a. Antagonist
b. +Agonist
c. Inverse agonist
d. Partial agonist

68. Cross the placenta barrier


a) Chlordiazepoxide
b) Diazepam
c) Prazepam
d) +All

69. ADH is located in:


a. Spleen
b. Liver
c. +Kidney
d. All

70. Opioid drugs include:


a. Full agonist
b. Partial agonist
c. Antagonist
d. +All

71. Most opioid analgetics are well absorbed:


a. Subcutaneous
b. Intramuscular
c. Oral
d. +All

8
72. Opioids localize in higher concentration in tissues except:
a. +Brain
b.Liver
c. Muscle
d. Lungs

73. Action of opioids is:


a. +Decrease of alveolar PCO2
b. Suppression of cough
c. Mydriasis
d. Diarrhea

74. Adverse effect of opioids is except:


a. Respiratory depression
b. Nausea
c. Vomiting
d. +Decrease intracranial pressure

75. Clinical use of opioid analgesics:


a. Constipation
b. Arrhythmia
c. +Cough
d. All

76. The maximal blood levels of local anesthetics


a. +Intercostals
b. Caudal
c. Epidural
d. Brachial plexus

77. The primary mechanism of action of local anesthetics is blockade of:


a. Ion channels
b. Potassium channels
c. Chlorium channels
d. +Sodium channels

78. Undergo extensive first pass metabolism in liver except:


a. +Nitroglycerine
b. Isosorbide mononitrate
c. Isosorbide dinitrate
d. Eiythrityl tetranitrate

79. The local anesthetics are converted in:


a. Kidney
b. +Liver
c. Lungs
d. Brain

80. Nitrates may be used except:


a. Angina pectoris
b. Biliaric colic
c. Esophageal spasm
d. +Bradicardia

9
81. Doses of dilthiazem:
a. 10-30 mg
b. +30-60 mg
c. 60-90 mg
d. 90-120 mg

82. The highest blood levels in the following order:


a. +Intercostals
a. Caudal
b. Branchial plexus
b. Sciatic nerve

83. Antiadrenergic agent is:


a. Amiadarone
b. Verapamile
c. +Propranolol
d. Propafenon

84. Agent widening AP is:


a. +Amiadarone
b. Verapamil
c. Propranolol
d. Propafenon

85. Calcium channel blocker is:


a. Amiadarone
b. +Verapamile
c. Propranolol
d. Propafenon

86. Membrane stabilizing agent is:


a) Amiadarone
b) +Verapamile
c) Propranolol
d) Propafenon

87. Excessive doses of lidocaine cause:


a. Paresthesias
b. +Hypotension
c. Hypertension
d. Disorientation

88. How many percents of mexiletine metabolized in liver:


a. 30%
b. 60%
c. +90%
d. 100%

89. The targets for benzodiazepine action is:


a. a-aminobutyric acid
b. β-aminobutyric acid
c. + γ-aminobutyric acid
d. 5-aminobutyric acid

10
90. Intermediate-acting benzodiazepines:
a. Clorazepam
b. +Alprazolam
c. Oxazepam
d. Flurazepam

91. Mechanism of action of methylxanthines


+Translocation of extracellular calcium
Translocation of intracellular calcium
Increase in cAMP
Decrease in cAMP
Decrease in GMP

92. The methylxanthines are well absorbed


a. +Orally
b. Subcutaneously
c. Intramuscular
d. Intravenously

93. The lowest blood levels in the following order:


a. Intercostals
b. Caudal
c. Branchial plexus
d. +Sciatic nerve

94. The highest blood levels in the following order:


a. +Intercostals
b. Caudal
c. Branchial plexus
d. Sciatic nerve

95. May occur the regular respiration and relaxation of the skeletal muscles:
a. Analgesia
b. Excitement
c. + Surgical anesthesia
d. Medullary paralysis

96. SGRIs block the up-take of serotonin leading to :


a. decrease concentration
b.+ Increase concentration
c. Inhibit the receptor
d. is not influence

97. Tick drus is forming a physical barrier to HCL :


a. Ranitidine
b. +Sucralfate
c. Omeprazole
d. Pirenzepine

98. Iindicate the drug that cause metabolic alkalosis:


a. +Sodium bicarbonate
b. Cimetidine
c. Pepto-Bismol
d. Carbenoxolone

11
99. The drug can be used to treat glaucoma:
a. Furosemide (Lasix)
b. +Acetazolamide (Diamox)
c. Both of the above
d. Neither of the above

100. Bactericidal effect is:


a) Inhibition of bacterial cell division
b) Inhibition of young bacterial cell growth
c) +Destroying of bacterial cells
d) Formation of bacterial L-form

101. Tick the drug belonging to antibiotics-tetracycline:


a. +Doxycycline
b. Streptomycin
c. Clarithromycin
d. Amoxacilhn

102. Tick the drug belongs to antibiotics-cephalosporin:


a. Streptomycin
b. +Cefaclor
c. Phenoxymethylpenicillin
d. Erythromycin

103. This drug reduces blood pressure by acting on vasomotor centers in the CNS:
a. Labetalol
b. +Clonidine
c. Enalapril
d. Nifedipine

104. Pick out the drug an alpha- and beta-adrenoreceptors blocker:


a. +Labetalol
b. Verapamil
c. Nifedipine
d. Metoprolol

105. This drug is converted to an active metabolite after absorption:


a) Labetalol
b) Clonidine
c) +Enalapril
d) Nifedipine

106. Indicate an agent, which is related to direct-acting drugs:


a) Edrophonium
b) Physostigmine
c) +Carbachol
d) Isoflurophate

107. All of these drugs reduce intestinal peristalsis EXCEPT:


a. Loperamide
b. +Cisapride
c. Methyl cellulose
d. Magnesium aluminium silicate

12
108. This drug acts by inhibiting type III cyclic nucleotide phosphodiesterase:
a. Amiodarone
b. +Milrinone
c. Propanolol
d. Enalapril

109. Drug is favored for digitalis-induced arrhythmias:


a. Verapamil
b. Amiodarone
c. + Lidocaine
d. Propanolol

110. Tick the drug belonging to non-selective beta-2 adrenomimetics:


a. Salbutamol
b. +Isoprenaline
c. Salmeterol
d. Terbutaline

111. Which of the following M-choiinoblocking agents is used especially as an anti-asthmatic?


a. Atropine
b. +Ipratropium
c. Platiphylline
d. metacin

112. A teratogenic action is:


a. Toxic action on the liver
b. +Negative action on the fetus causing fetal malformation
c. Toxic action on blood system
d. Toxic action on kidneys

113. What term is used to describe a decrease in responsiveness to a drug which develops in a few
minutes?
a. Refractoriness
b. Cumulative effect
c. Tolerance
d. +Tachyphylaxis

114. If an agonist can produce sub maximal effects and has moderate efficacy it's called:
a. +Partial agonist
b. Antagonist
c. Agonist-antagonist
d. Full agonist

115. Tachyphylaxis is:


A drug interaction between two similar types of drugs:
a. +Very rapidly developing tolerance
b. A decrease m responsiveness to a drug, taking days or weeks to develop
c. None of the above

116. Idiosyncratic reaction of a drug is:


a. A type of hypersensitivity reaction
b. A type of drug antagonism
c. +Unpredictable, inherent, qualitatively abnormal reaction to a drug
d. Quantitatively exaggerated response

13
117. Tick the second messenger of G-protein-coupled (metabotropic) receptor:
a. Adenylyl cyclase
b. Sodium ions
c. Phospholipase С
d. +cAMP

119. What does the term “potentiation” mean?


a. Cumulative ability of a drug
b. Hypersensitivity to a drug
c. Fast tolerance developing
d. +intensive increase of drug effects due to their combination

120. If an agonist can produce sub maximal effects and has high efficacy it's called:
a. Partial agonist
b. Antagonist
c. Agonist-antagonist
d. +Full agonist

121. What kind of substances can't permeate membranes by passive diffusion?


a. Lipid-soluble
b. Non-ionized substances
c. Hydrophobic substances
d. +Hydrophilic substances

122. Which of the following is more potent:


a) cimetidine
b) ranitidine
c) nizatidine
d) +famotidine

123. Indicate the laxative drug belonging to osmotic laxatives:


a. Docusate sodium
b. Bisacodyl
c. Phenolpnthalein
d. +Sodium phosphate

124. The mechanism of methylxanthines action is:


a) +Inhibition of the enzyme phosphodiesterase
b) Beta-2 adrenoreceptor stimulation
c) Inhibition of the production of inflammatory cytokines
d) Inhibition or M-cholinorcceptors

125. The drug is the most potent diuretic:


a. +Loop diuretics
b. Thiazide diuretics
c. Potassium-sparing diuretics
d. Carbonic anhydrase inhibitors

126. The drugs is used in the treatment of recurrent Ca nephrolithiasis:


a. Osmotic diuretics
b. +Loop diuretics
c. Thiazide diuretics
d. Potassium-sparing diuretics

14
127. Duration of nitroglycerin action (sublingual) is:
a) +10-30 minutes
b) 6-8 hours
c) 3-5 minutes
d) 1.5-2 hours

128. All of the following agents are cardiac glycosides EXCEPT:


a. Digoxin
b. Strophantin К
c. +Amrinone
d. Digitoxin

129. This drug inhibits breakdown of cAMP in vascular smooth muscle:


a. Digoxin
b. Dobutamine
c. +Amrinone
d. Dopamine

130. Catecholamine includes following EXCEPT:


a. +Ephedrine
b. Epinephrine
c. Isoprcnalinc
d. Norepinephrine

131. Indicate the beta-1 selective agonist:


a. Isoproterenol
b. +Dobutamine
c. Metaproterenol
d. Epinephrine

132. Which of the following is preferable for the emergency therapy of cardiogenic shock?
a. Epinephrine
b. +dobutamine
c. Isoproterenol
d. Mcthoxamine

133. Which of the following drugs is an nonselective beta receptor antagonist?


a) Metoprolol
b) Atenolol
c) +Propranolol
d) Acebutolol

134. Which of the following drugs is useful in the treatment of pheochromocytoma?


a. Phenylephrine
b. Propranolol
c. +Phentolamine
d. Epinephrine

135. All of the following drugs are antibiotics, EXCEPT:


a. Streptomycin
b. Penicillin
c. +Co-trimoxazole
d. Chloramphenicol

15
1. Which of the following acts indirectly ?
a. Epinephrine
b. Norepinephrine
c. +Ephedrine
d. d) Methoxamine

2. Indicate the alfa 2-selective agonist:


a. +Xylometazoline
b. Epinephrine
c. Dobutamine
d. Methoxamine

3. Epinephrine produces all of the following effects except:


a. Bronchodilation
b. +Decrease in oxvgen consumption
c. Hvperglvcemia
d. Mydrias'is

4. Indicate the drug, which may be useful in the emergency management of cardiac arrest:
a. ethoxamine
b. Phenylephrine
c. +Epinephrine
d. Xylometazoline

5. Indicate the beta-l selective antagonist:


a. Propranolol
b. +Metoprolol
c. Carvedilol
d. Sotalol

6. Non selective alfa-receptor antagonists are most useful in the treatment of:
a. Asthma
b. Cardiac arrhythmias
c. +Pheochromocvtoma
d. Chronic hypertension

7.This drug is a Class 1C antiarrhythmic drug:


a. +Flecainide
b. Sotalol
c. Lidocaine
d. Verapamil

8. This drug is associated with Torsades de pointes.


a. Flecainide
b. +Sotalol
c. Lidocaine
d. Verapamil

9. This drug is used intravenously to terminate supraventricular tachycardias:


a. Nifedipine
b. +Verapamil
c. Both of the above
d. None of the above

10. Bactericidal effect is:


a. Inhibition of young bacterial cell growth

1
b. Inhibition of bacterial cell division
c. +Destroying of bacterial cells
d. Formation of bacterial L-form

11. Tick the drug belonging to antibiotics-macrolides:


a. Neomycin
b. Doxicycline
c. +Erythromycin
d. Cefotaxime

12. Tick the drug belonging to antibiotics-tetracyclmes:


a. +Doxycycline
b. Streptomycin
c. Clarithromycin
d. Amoxacillin

13. Antibiotic inhibiting bacterial RNA synthesis is:


a. Erythromycin
b. +Rifampin
c. Chloramphenicol
d. Imipinem

14. Which of the following agents is related to tricyclic antidepressants ?


a. Nefazolan
b. +Amitriptyline
c. Fluoxetine
d. Isocarboxazid

15. Indicate an effective antidepressant with minimal autonomic toxicity:


a. Amitntyline
b. +Fluoxetine
c. Imipramine
d. Doxepin

16. A ganglioblocking drug for hypertension treatment is:


a. Hydralazine
b. Tubocurarin
c. +Trimethaphan
d. Metoprolol

17. This drug inhibits the angiotensin-converting enzyme:


a. Captopril
b. Enalapril
c. Ramipril
d. +All of the above

18. This drug is an inhibitor of rennin synthesis:


a. +Propranolol
b. Enalapril
c. Diazoxide
d. Losartan
19. This drug routinely produces some tachycardia:
a. +Nifedipine
b. Clonidin
c. Enalapril
d. Propranolol

2
20. Which of the following agents is the precursor of dopamine ?
a) Bromocriptine
b) +Levodopa
c) Selegiline
d) Amantadine

21. Indicate a selective inhibitor of monoamine oxidase B:


a) Levodopa
b) Amantadine
c) Tolcapone
d) +Selegiline

22. Which of the following benzodiazepines has the shortest duration of action?
a) +Triazolam
b) Clorazepate
c) Prazepam
d) Clordiazepoxide

23. Which of the following anxiolytics has minimal abuse liability ?


a) Oxazepam
b) +Buspirone
cj Flumazenil
d) Alprazolam

24. Which of the following direct-acting is mainly muscarinic in action ?


a) +Bethanechol
b) Carbacbol
c) Acetylcholine
d) None of the above

25. Which of the following cholinesterase inhibitors is irreversible?


a) Physostigmine
b) Edrophonium
c) Neostigmine
d) +Isoflurophate

26. Chronic long-term therapy of myasthenia is usually accomplished with:


a) Edrophonium
b) +Neostigmine
c) Echothiophate
d) Carbachol

27. Indicate the skeletal muscle relaxant, which is a depolarizing agent:


a) Vencuronium
b) Scopolamine
c) +Succinylcholine
d) Hexamethonium
28. Which of the following drugs is a selective M l blocker ?
a. Atropine
b. Scopolamine
c. +Pirenzepme
d. Homatropine

29. Indicate a drug, which is effective in the treatment of mushroom poising:


a) Pralldoxime

3
b) Pilocarpine
c) Homatropine
d) +Atropine

30. Which of the following drug is an intermediate-duration muscle relaxant?


a) +Vecuronium
b) Tubocurarine
c) Pancuronium
d) Rapacuronium

31. Which of the following muscular relaxants causes hypotension and bronchospasm ?
a. Vecuronium
b. Succinvlcholine
c. +Tubocurarine
d. Rapacuronium

32. The drug is a K+ -sparing diuretic that blocks Na+ channels in the collecting tubules:
a. Acetazolamide (Diamox}
b. +Amiloride (Midamor)
c. Furosemide (Lasix)
d. Hydrochlorothiazide (HydroDiuril)

33. The drug acts only on the luminal side of renal tubules:
a. +Furosemide (Lasix)
b. Acetazolamide (Dianiox)
c. Both of the above
d. Neither of the above

34. The drug acts at the proximal tubule:


a. Loop diuretics
b. Thiazide diuretics
c. Potassium-sparing diuretics
d. +Carbonic anhvdrase inhibitors

35. The drug is one of the most potent diuretics:


a. Acetazolamide (Diamox)
b. +Furosemide (Lasix)
c. Hydrochlorothiazide (Hydrodiuril)
d. Aniiloride (Midamor)

36. Furosemide acts at the nephron site:


a. Proximal convoluted tubule
b. +Ascending thick limb of the loop of Henle
c. Distal convoluted tubule
d. Collecting duct

37. Which of the following nitrates are long-acting?


a. Nitroglycerin
b. Isosoroide dinitrat
c. Amyl nitrite
d. +Sustac

38. The following agents are cardioselective beta-1 blocking drugs except:
a. +Propranolol

4
b. Talinolol
c. Atenolol
d. Metaprolol

39. Drug is used in digitalis intoxication:


a) Lidocaine
b) +Digibind (Digoxin immune fab)
c) Oral potassium supplementation
d) Reducing the dose ol the drug

40. All of the following drugs are used in CHF


a. Digoxin
b. Dobutamine
c. +Verapamil
d. Dopamine

41. Which of the following inhaled anesthetics is an induction agent of choice in patient with
airway problems ?
a. Desfurane
b. Nitrous oxide
c. +Halothane
d. None of the above

42. Which of the following intravenous anesthetics has antiemetic actions?


a. Thiopental
b. +Propofol
c. Ketamine
d. Fentanyl

43. Pick out the bronchodilator drug related to xanthine:


a) Atropine
b) Orciprenaline
b) Adrenaline
d) +Theophylline

44. All of the following drugs are inhaled glucocorticoids except:


a) Triamcinolone
b) Beclometazone
c) +Sodium cromoglycate
dj Budesonide

45. Indicate the drug belonging to proton pump inhibitors:


a) Pirenzepme
b) Ranitidine
c) +Omenprazole
d) T rimethaphan

46. Which of the following drugs may cause reversible gynecomastia?


a. Omeprazole
b. Pirenzepine
c. +Cimetidine
d. Sucralfate

5
47. Choose the drug that causes constipation:
a) Sodium bicarbonate
b) +Aluminium hvdroxide
c) Calcium carbonate
d) Magnesium oxide

48. Indicate an antiemetic agent which is related to neuroleptics:


a. Metoclopramide
b. Nabilone
c. Tropisetron
d. +Prochlorperazine

49. Which of the following is not an adverse effect of ranitidine:


a) headache
b) +gynecomastia
c) dizziness
d) confusion

50. What does “pharmacokinetics" include?


a. Complications of drug therapj
b. +Drug biotransformation in the organism
c. Influence of drugs on metabolism processes
d. Influence of drugs on genes

51. All of the following agents are beta-receptor agonists except:


a. Epinephrine
b. Isoproterenol
c. +Methoxamine
d. Dobutamine

52. Which of the following is a beta-2 selective agonist?


a. +Terbutaline
b. Xylometazoline
c. Isoproterenol
d. Dobutamine

53. Isoproterenol produces all of the following effects except:


a) Increase in cardiac output
b) Fall in diastolic and mean arterial pressure
c) +Bronchoconstriction
d) Tachycardia

54.1ndicate the alfa-1 selective antagonist:


a. Phentolamine
b. Dihydroergotamine
c. +Prazosin
d. Labetalol

55. Indicate the beta antagonist, which has partial beta-agonist activity:
a. Prooranolol
b. Meioprolol
c. +Pindolol
d. Betaxolol

6
56. The adverse effects of phentolamine include all of the following EXCEPT:
a. Diarrhea
b. +Bradycardia
c. Arrhythmias
d. Myocardial ischemia

57. This drug is a Class II antiarrhythmic drug:


a. Flecainide
b. +Propranolol
c. Lidocaine
d. Verapamil

58. This drug is used in treating supraventricular tachycardias:


a)+Digoxin
b) Dobutamine
c) Amiinone
d) Dopamine

59. This is a drug of choice for acute treatment of ventricular tachycardias:


a) Flecainide
b) Sotalol
c) +Lidocaine
d) Verapamil

60. All of the following are common adverse effects of Ca channe blockers EXCEPT:
a. +Skeletal muscle weakness
b. Dizziness
c. Headache
d. Flushing

61. Bacteriostatic effect is:


a) +Inhibition of bacterial cell division
b) Inhibition of young bacterial cells growth
c) Destroying of bacterial cells
d) Formation of bacterial L-form

62. Tick the drug belonging to antibiotics-monobactams:


a) Ampicillin
b) Bicillin-5
c) +Aztreonam
d) Imipinem

63. Tick the drug belonging to nitrobenzene derivative:


a. Clindamycin
b. Streptomycin
c. Azithromycin
d. +Chloramphenicol

64. Antibiotics altering permeability of cell membranes are:


a. Glvcopeptides
b. +Polymyxins
c. Tetracvclines
d. Cephalosporins

7
65. Which of the following atuidepressants is a selective serotonin reuptake inhibitor?
a) Phenelzine
b) Desipramine
c) Maprotiline
d) +Fluoxetine

66. Which of the following antidepressants is used for treatment of eating disorders, especially
buliemia ?
a. Amitriptvline
b. +Fluoxetine
c. Imipramihe
d. Tranylcypromine

67. Pick out the sympatholytic drug:


a. aj Labetalol
b. Prazosin
c. + Guanethidine
d. Clonidine

68. Choose the selective blocker of beta-1 adrenoreceptors:


a. Labetalol
b. Prazosin
c. +Atenolol
d. Propranolol

69. This drug blocks alpha-1 adrenergic receptors:


a. +Prazosin
b. Clonidine
c. Enalapril
d. Nifedipine

70. This drug is contraindicated in patients with bronchial asthma:


a. +Propranolol
b. Clonidine
c. Enalapril
d. Nifedipme

71. Tick the diuretic agent - aldosterone antagonist:


a. Furosemide
b. +Spironolactone
c. Dichloiiiiazide
d. Captopril

72. Indicate the agents of choice in the treatment of most anxiety states:
a. Barbiturates
b. +Benzodiazepines
c. Lithium salts
d. Phenothiazines

73. Which of the following anxiolytics is preferred in patient with limited hepatic function?
a. +Buspirone
b. Quazepam
c. Diazepam
d. Chlordiazep oxide

8
74. Muscarinic receptors are located in:
a. Autonomic ganglia
b. Skeletal muscle neuromuscular junctions
c. +Autonomic effector cells
d. Sensorv carotid sinus baroreceptor zone

75. Which of the following direct-acting drug has the shortest duration of action?
a. +Acetylcholine
b. Metnacholine
c. Carbachol
d. Bethanechol

76. Indicate cholinesterase activator:


a. +Pralidoxime
b. Edrophonium
c. Pilocarpine
d. Isoflurophate

77. Indicate the reversible cholinesterase inhibitor, which penetrates the blood-brain barrier:
a. +Physostigmine
b. Edrophonium
c. Neostigmine
d. Piridostigmine

78. Which of the following agents is a ganglion-blocking drug?


a. Homatropine
b. +Hexametnonium
c. Rapacuronium
d. Edrophonium

79. Atropine is used prior to premedication to reduce:


a. Muscle tone
b. +Secretions
c. Nausea and vomiting
d. All of the above

80. Which of the following agents is used as an inhalation drug in asthma?


a. Atropine
b. +Ipratropium
c. Lobelme
d. Homatronine

81. Indicate the long-acting neuromuscular blocking agent:


a. Rapacuronium
b. Mivacurium
c. +Tubocurarine
d. Rocuronium

82. Which of the following neuromuscular blockers causes transient muscle fasciculations?
a. Mivacurium
b. Pancuronium
c. +Succinvlcholine
d. Tubocurarine

9
83. The drug acts at the proximal tubule:
a. +Acetazolamiae (Diamoxj
b. Furosemide (Lasix)
c. Hydrochlorothiazide (HydroDiuril)
d. Spironolactone (Aldactone)

84. The drug can be used to treat glaucoma:


a. Furosemide (Lasix)
b. +Acetazolamide (Dianiox)
c. Both of the above
d. Neither of the above

85. The drug decreases calcium excretion in urine:


a. +Hydrochlorothiazide (HydroDiuril)
b. Amiloride (Midamor)
c. Furosemide (Lasix)
d. Acetazolamide (Diamox)

86. The drug acts in the collecting tubules:


a. Loop diuretics
b. Thiazide diuretics
c. +Potassium-sparing diuretics
d. Carbonic annydrase inhibitors

87. The drug is the least potent diuretic:


a. Osmotic diuretics
b. Loop diuretics
c. Thiazide diuretics
d. +Potassium-sparing diuretics

88. Acetazolamide acts at this nephron site


a. +Proximal convoluted tubule
b. Ascending thick limb of the loop Henle
c. Distal convoluted tubule
d. Collecting duct

89. Which of the following is a Ca channel blocker ?


a. Nitroglycerin
b. Dipyndamole
c. Minoxidil
d.+ Nifedipine

90. All of the following are used in CHF except:


a. Reduced salt intake
b. +Verapamil
c. ACE inhibitors
d. Diuretics

91. This drug is a selective beta-1 agonist:


a. Digoxin
b. +Dobutamine
c. Amrinone
d. Dopamine

92. Indicate drug, which is an inhibitor of NMDA glutamate receptors:


a. Thiopental

10
b. Halothane
c. +Ketamine
d. Sevoflurane

93. Which of the following inhaled anesthetic can produce hepatic necrosis?
a. Soveflurane
b. Desfluane
c. +Halothane
d. Nitrous oxide

94. Indicate the intravenous anesthetic, which is a benzodiazepine derivative:


a. +Midazolam
b. Thiopental
c. Ketamin
d. Propofol

95. Pick out the bronchodilator drug belonging to sympathomimics:


a. Isonrenaline
b. +Epnedrine
c. Atropine
d. Salbutamol

96. Choose the drug belonging to membrane stabilizing agents:


a. Zileutin
b. +Sodium cromoglycate
c. Zafirlucast
d. Montelucast

97. All of the following agents intensify the secretion of gastric glands ЕХСЕРT.
a. +Pepsin
b. Gastrin
c. Histamine
d. Carbonate mineral waters

98. Tick the drug forming a physical barrier HC1 and pepsin:
a. Ranitidine
b. +Sucralfate
c. Omeprazole
d. Pirenzepine

99. Indicate the drug that cause metabolic alkalosis:


a. +Sodium bicarbonate
b. Cimetidine
c. Pepto-Bismol
d. Carbenoxolone

100. Indicate the beta-1 selective antagonist:


a) Propranolol
b) +Metoprolol
c) Garvedilol
d) Sotalol
101. This drug is an inhibitor of renin synthesis:
a. +Propranolol
b. Enalapril
c. Diazoxide
d. Losartan

11
102. This drug routinely produces some tachycardia:
a. Propranolol
b. Clonidine
c. Enalapril
d. +Nifedipine

103. Which drugs is a selective Ml blocker?


a. Atropine
b. +Scopolamine
c. Pircnzepine
d. Homatropine

104. Indicate a drug, which is effective in the treatment of mushroom poising:


a. Pralidoxime
b. Pilocarpine
c. Homatropine
d. +Atropine

105. The drug is a K+-sparing diuretic that blocks Na+ channels inlhe collecting tubules:
a. Acetazolamide (Diamox)
b. +Amiloride (Midamor)
c. Furosemide (Lasix)
d. Hydrochlorothiazide (HydroDiuril)

106. Indicate the drug belonging to proton pump inhibitors:


a) Pircnzepine
b) Ranitidine
c) +Omeprazole
d) Trimcthaphan

107. Which of the following drugs may cause reversible gynecomastia?


a) Omeprazole
b) Pircnzepine
c) +Cimetitfine
d) Sucralfate

108. Choose the drug that causes constipation:


a) Sodium bicarbonate
b) +Aluminium hydroxide
c) Calcium carbonate
d) Magnesium oxide

109. Furosemide acts at the nephron site:


a. Proximal convoluted tubule
b. +Ascending thick limb of the loop of Henle
c. Distal convoluted tubule
d. collecting duct

110. A bronchial smooth muscle contains:


a. Alfa-1 receptor
b. Alfa-2 receptor
c. Beta-1 receptor
d. + Beta-2 reccptor

12
111. Which drug is a non selective β-receptor agonist?
a. Norepinephrine
b. Terbutaline
c. +Isoproterenol
d. Dobutamine

112. Sustained use of this drug results in increased plasma urate concentrations:
a) +Furosemide (Lasix)
b) Acetazolamide (Diamox)
c) Both of the above
d) Neither of the above

113. This drug is a Class IB antiarrhythmic drug:


a. Flecainide
b. Sotalol
c. +Lidocaine
d. Verapamil

114. Tick the drug belongs to antibiotics-cephalosporins:


a. Streptomycin
b. +Ceficlor
c. Phenoxymcthilpenicillin
d. Erythromycin

115. Which of the following drugs is a nonselective alfa receptor antagonist?


a. Prazosin
b. +Phentolamine
c. Metoprolol
d. Reserpine

116. Which of the following groups of antibiotics shows a bacteristatic effect:


a. Carbapenems
b. +Macrolides
c. Aminoglycosides
d. Cephafos'porins

117. All of the following agents are cardiac glycosides EXCEPT:


a. Digoxin
b. Strophantin К
c. +Amrinone
d. Digitoxin

118. Amiloride acts ai this nephron site:


a. Proximal convoluted tubule
b. Ascending thick limb of the loop of Henle
c. Distal convoluted tubule
d. +Collecting duct

119. Indicate the drug which is a leucotrienc receptor antagonist:


a. Sodium cromoglycatc
b. +Zafirlucast
c. Zileutin
d. Triamcinolone

13
120. Choose the drug which is a H2-receptor antagonist:
a. Omeprazole
b. Pirenzepme
c. Carbenoxolonc
d. +Ranitidine

121. Which drug is an analog of prostaglandin El?


a. +Misoprostole
b. De-nol
c. Sucralfate
d. Omeprazole

122. Tick the mechanism of metoclopramide antiemetic action:


a. HI and H2-receptor blocking effect
b. M-cholinoreceptor stimulating effect
c. +D2-dopaniine and 5-HT3-serotonin receptor blockina effect
d. M-choiinoblocking effect

123. All of these drugs reduce intestinal peristalsis EXCEPT:


a. Loperamide
b. +Cisapride
c. Methyl cellulose
d. Magnesium aluminium silicate

124. All drugs inhibit cell wall synthesis, except:


a. Carbapenems.
b. Monobactams.
c. Cephamycins.
d. +Nitrofurantoin

125. This drug is a Class III antianhythmic drug:


a. Flecainide
b. +Sotalol
c. docaine
d. Verapamil

126. Minimal duration of antibacterial treatment is:


a. Not less than 1 day
b. +Not less than 5 days
c. Not less than 10-I4 days
d. Not less than 3 weeks

127. Which antibiotics contains a β-lactam ring in their chemical structure:


a. Penicillins
b. Cephalosporins
c. Carbapenems and monobactams
d. +All groups

128. The mode of action may be:


a) +specific and nonspecific
b) selective and nonselective
c) reversible and irreversible
d) metabolic and fimctional

14
129. All of the following antibiotics inhibit the protein synthesis in bacterial cells, EXCEPT:
a. Macrolides
b. Aminoglycosides
c. +Glycopcptides
d. Tetracyclincs

130. All are true about digoxin except:


a. +Causes bradycardia
b.Acts by inhibiting Na+K+ ATPase
c. It is 95 % plasma protein bound
d. Primarily excreted unchanged by glomerular filtration acting

131. All of the following are central antihypertensive drugs EXCEPT:


a. Methyldopa
b. Clonidine
c.Moxonidine
d. +Minoxidil

132. This drug is a directly acting vasodilator:


a. Labetalol
b. Clonidine
c. Enalapril
d. +Nifedipine

133. This drug is a potassium channel activator:


a) Nifedipine
b) Saralasin
c) +Diazoxide
d) Losartan

134. Which of the following is commonly used in the treatment of glaucoma?


a. +Pilocarpine
b. Lobeline
c. Acethylcholine
d. Neostigmine

135. Indicate muscles, which are more resistant to block and recover more rapidly:
a. Hand
b. Leg
c. Neck
d. +Diaphragm

136. Tick the drug, inhibiting viral DNA synthesis:


a) Interferon
b) Saquinavir
c) Amantadine
d) +Acyclovir

137. The drug interaction with different cellular and subcellular structures can result in their
a) stimulation
b) +depression
c) decreasing
d) stabilization

15
138. Tick the anticancer drug, a pyrimidine antagonist:
a) Fluorouracil
b) +Mercaptopurine
c) Thioguanine
d) Methotrexate

139. Geriatric pharmacology studies the peculiarities of action and clinical use of drugs in
a) children
b) pregnant women
c) nursing women
d) +elderly

140. Tick the second messenger of G-protein-coupled (metallotropic) receptor:


a. Adenylyl cyclase
b. Sodium ions
c. Phosnholipase С
d. +сАМР

150. Which drug is a proton pump inhibitor?


a. Cimetidme
b. Ranitidine
c. Pirenzcpin
d. +Omeprazole

151. Choose an emetic drug of central action:


a. +Ipecacuanha derivatives
b. Promethazine
c. Tropisetron
d. Apomorphine hydrochloride

152. Therapeutic index is indicator of:


a) Safety
b) +Efficacy
c) Potency
d) Toxicity

153. Morphine causes all, EXCEPT:


a) Peripheral vasodilatation
b) Decrease intracranial tension
c) Nausea and vomiting
d) +Decrease in gastrointestinal secretion

154. What kind of substances can’t permeate membranes by passive diffusion?


a. Lipid-soluble
b. Non-ionized substances
c. Hydrophobic substances
d. +Hydrophilic substances

155. Tachyphylaxis is:


a. A drug interaction between two similar types of drugs
b. +Very rapidly developing tolerance
c. A decrease in responsiveness to a drug, taking days or weeks to develop
d. None of the above

16
156. Pharmacology is a science about
a) chemistry
b) physiology.
c) +medicinal preparations
d) pharmaceutical business

157. A teratogenic action is:


a. Toxic action on the liver
b. +Negative action on the fetus causing fetal malformation
c. Toxic action on blood system
d. Toxic action on kidneys

158. Which of the following agents is an selective agents is an alfa-l selective agonist?
a. Norepinephrine
b. +Methoxamine
c. Ritodrine
d. Ephedrine

159. Indicate the indirect-acting agent:


a. Epinephrine
b. Phenylephrine
c. +Ephedrine
d. Isoproterenol

160. Which of the following is related to short acting topical decongestant agents ?
a. Xylometazoline
b. Terbutaline
c. +Phenylephrine
b. Norepinephrine

161. Which of the following agents is an alfa 2-selective antagonist ?


a. +Yohimbine
b. Tamsulosin
c. Tolazoline
d. Prazosin

162. Which of the following drugs is a reversible nonselective alfa-, beta-antagonist?


a. + Labetalol
b. Phentolamine
c. Metoorolol
d. Propranolol

163. This drug is a Class 1A antiarrhythmic drug:


a. Sotalol
b. Propranolol
c. Verapamil
d. +Quinidine

164. This drug prolongs repolarization:


a. Flecainide
b. +Sotalol
c. Lidocaine
d. Verapamil

17
165. This drug has beta-adrenergic blocking activity:
a. Flecainide
b. +Sotalol
c. Lidocaine
d. Verapamil

166. This drug acts to inhibiting slow Ca channels in the SA and AV nodes:
a. Quinidine
b. Adenosine
c. Fleeainide
d. +Diltiazem

167. Which of the following groups of antibiotics shows a bactericidal effect?


a. Tetracyclines
b. Macrolides
c. +Penicillins
d. All of the above

168. Tick the drug belonging to antibiotics - carbapenems:


a. Aztreonam
b. Amoxacillin
c. +Imipinem
d. Clarithromycin

169. Tick the drug belonging to lincozamides:


a. Erythromycin
b. +Lincomvcm
c. Azithromycin
d. Aztreonam

170. Tick the drug belonging to glycopeptides:


a. +Vancomycin
b. Lincomycin
c. Neomycm
d. Carbenicillin

171. Which of the following drugs is a gastric acid resistant:


a. Penicillin G
b. +Penicillin V
c. Carbenicillin
d. Procain penicillin

I72. Indicate the antidepressant, which blocks the reuptake pumps for serotonin and norepinephrine:
a. +Amitriptyline
b. Fluoxetine
c. Maprotiline
d. Phenelzine

173. A highly selective serotonine reuptake inhibitor is:


a. Sertaline
b. Paroxetine
c. Fluoxetine.
d. +All of the above

18
174. Tick the drug with nonselective beta-adrenoblocking activity:
a. Atenolol
b. +Propranolol
c. Metoprolol
d. Nebivolol
175. Pick out the diuretic agent for hvpertension treatment:
a. Losartan
b. +Dichlothiazide
c. Captcpril
d. Prazosin
176. This drug is a non-peptide angiotensin II receptor antagonist:
a. Clonidine
b. Captopril
c. +Losartan
d. Diazoxide

177. Choose the vasodilator which releases NO:


a. Nifedipine
b. Hydralazine
c. Minoxidil
d. +Sodium nitroprusside

178. Indicate a peripheral dopa decarboxvlase inhibitor:


a. Tolcapone
b. Clozapine
c. + Carbiaopa
d. Selegiline

179. Indicate D2 receptor agonist with antiparkinsonian activity:


a. Sinemet
b. Levodopa
c. +Bromocriptine
d. Selegiline

180. Which of the following anxylotics is a benzodiazepine derivative:


a. Buspirone
b. +Clordiazepoxide
c. Meprobamate
d. Chloral hydrate

181. Indicate the competitive antagonist of β-2 receptors:


a. +Flumazenil
b. Buspirone
c. Picrotoxin
d. Diazenam

182. Which of the following activates both muscarinic and nicotinic receptors ?
a. Lobeline
b. Pilocarpine
c. Nicotine
d. +Bethanechol

19
183. The M-cholinimimetic agent is:
a. Carbachol
b. +Pilocarpine
c. Acetylcholine
d. Betfianechol

184. Isofluorophate increases all of the following effects except:


a. Lacrimation
b. +Bronchodilation
c. Muscle twitching
d. Salivation

185. Which of the following cholinomimetics is used in the treatment of atropine intoxication?
a. Neostigmine
b. Carbochol
c. +Physostigmine
d. Lobeline

186. Which of the following drugs is effective in motion sickness?


a. Atropine
b. Ipratropium
c. +Scopolamine
d. Homatropine

187. Which of the following drugs is useful in the treatment of uterine spasms?
a. Carbachol
b. Vecuronium
c. +Atropine
d. Edrophonium

188. Antimuscarinics are used in the treatment of the following disorders EXCEPT:
a. Motion sickness
b. +Glaucoma
c. Hyperhidrosis
d. Asthma

189. Indicate the nondepolarizing agent, which has short duration of action:
a. Succinylcholine
b. Tubocurarine
c. +Mivacurium
d. Pancuronium

190. The drug inhibits the ubiquitous enzyme carbonic anhydrase:


a. +Acetazolamide (Diamoxj
b. Furosemide (Lasix]
c. Hydrochlorothiazide (HydroDiuril)
d. Spironolactone (Aldactone)

191. The drug acts by competing with aldosterone for its cytosotic receptors:
a. Acetazolamide (Diamoxj
b. Furosemide (Lasix)
c. Hydrochlorothiazide (HydroDiuril)
d. +Spironolactone (Aldactone)

20
192. The drug can cause ototoxicity:
a. +Furosemide (Lasix)
b. Acetazolamide (Diamox)
c. Both of the above
d. Neither of the above

193. The drug acts by competitively blocking the Na+/K+/2Cl- cotransporter:


a. +Loop diuretics
b. Thiazide diuretics
c. cl Potassium-sparing diuretics
d. Carbonic anhydrase inhibitors

194. The drug acts by competitively blocking the NaCI cotransporter:


a. Loop diuretics
b. +Thiazide diuretics
c. Potassium-sparing diuretics
d. Carbonic anhydrase inhibitors

195. These agents must be given parenterally because they are not absorbed w'hen given
orally:
a. + Osmotic diuretics
b. Loon diuretics
c. Thiazide diuretics
d. Potassium-sparing diuretics

196. Spironolactone acts at this nephron site:


a. Proximal convoluted tubule
b. Ascending thick limb of the loop of Henle
c. Distal convoluted tubule
d. +Collecting duct

197. Main clinical use of Ca channel blockers is:


a. Angina pectoris
b. Hypertension
c. Supraventricular tachyarrhythmia
d. +All of the above

198. The non-glycoside positive inotropic drug is:


a. Digoxin
b. Strophantin К
c. +Dobutamine
d. Digitoxin

199. Tolerance to this drug develops after a few days:


a. Amrinone
b. Amiodarone
c. +Dobutamine
d. Adenosine

200. Which of the following general anesthetics belongs to inhalants ?


a. Thiopental
b. +Desfluran
c. Ketamine
d. Propoftol

21
201. Indicated the inhaled anesthetic, which may cause nephrotoxicity:
a. Halothane
b. +Soveflurane
c. Nitrous oxide
d. Diethvl ether

202. Which of these groups of drugs is used for asthma treatment ?


a. Methylxanthines
b. M-cholinoblocking agents
c. Beta-2 stimulants
d. +All of above

203. Select the side-effect characteristic for non-selective beta-2 adrenomimetics:


a. Depression of the breathing centre
b. +Tachycardia
c. Peripheral vasoconstriction
d. Dry mouth

204. Indicate the side effect of theophylline:


a. Bradycardia
b. +Increased myocardial demands for oxygen
c. Depression of respiratory centre
d. Elevation of the arterial blood pressure

205. Gastric acid secretion is under the control the following agents EXCEPT:
a. Histamine
b. Acetylcholine
c. +Serotonin
d. Gastrin

206. All of the following drugs are proton pump inhibitors except:
a. Pantoprozole
b. Omeprazole
c. +Famotidine
d. Rabeprazole

207. Most of drugs are antacids EXCEPT:


a. +Misoprostol
b. Maalox
c. Mylanta
d. Almagel

208. This drug is related to short-acting topical decongestant:


a. Xylometazoline
b. Terbutaline
c. +Phenylephrine
d. Norepinephrine

209. This drug is a Class IA antiarrhythmic drug:


a. Sotalol
b. Propranolol
c. Verapamil
d. +Quinidine

22
210. This drug acts by inhibiting slow Ca channels in the SA and AV nodes:
a. Quinidine
b. Adenosine
c. Fleeainide
d. +Diltiazem

211. Which of the following groups of antibiotics shows a bactericidal effect?


a. Tetracyclines
b. Macrolides
c. Penicillins
d. +All of the above

212. Most of drugs are antacids EXCEPT:


a. +Misoprostol
b. Maalox
c. Mvlanta
d. Almagel

213. Side effects of anticancer drugs are all, EXCEPT:


a. Low selectivity to cancer cells
b. Depression of bone marrow
c. Depression of angiogenesis
d. +Depression of immune system

214. Main clinical use of Ca channel blockers is:


a. Angina pectoris
b. Hypertension
c. Supraventricular tachyarrhythmias
d. +All of the above

215. Which drug is a H2-receptor antagonist:


a. Omeprazole
b. Pirenzepine
c. Carbenoxolone
d. +Ranitidine

23
This page
intentionally left
blank
Copyright © 2008, New Age International (P) Ltd., Publishers
Published by New Age International (P) Ltd., Publishers

All rights reserved.


No part of this ebook may be reproduced in any form, by photostat, microfilm,
xerography, or any other means, or incorporated into any information retrieval
system, electronic or mechanical, without the written permission of the publisher.
All inquiries should be emailed to rights@newagepublishers.com

ISBN (13) : 978-81-224-2926-8

PUBLISHING FOR ONE WORLD


NEW AGE INTERNATIONAL (P) LIMITED, PUBLISHERS
4835/24, Ansari Road, Daryaganj, New Delhi - 110002
Visit us at www.newagepublishers.com
This book is dedicated to
My Parents, Late Shri G. C. Naidu and
Parents,
Smt. G. Lalitha
For their untiring efforts and hardwork in bringing me up to what all I
am today.

Prof. Dr. G. Vidya Sagar


This page
intentionally left
blank
SNIPPETS

Very useful book for students preparing for GATE & USMLE. Recommended reading.
Dr. Sanjay Pai
Al-Ameen college of Pharmacy
Bangalore.

I found the book absorbing, questions are well framed.


Dr. Gabhe
C.U Shah College of Pharmacy,
Mumbai.

Good Reference for PG medical entrance. Recommended reading.


Dr. Kaushik Shah
M.S.(Gen. Surgery)
Rana Hospital, Mandvi

This book will be of good use for students appearing for Competitive exams.
Presentation of the matter is good.
Prof. Y. Madhusudhan Rao
Univ. College of Pharmaceutical Sciences
Kakatiya University, Warangal.
FOREWORD

Medical & Pharmacy are fast growing professions with a wide range of opportunities open to the students after a basic
degree. These professions play a vital role in health care management.
This book will be of immense value for students to develop themselves as the meritorious & motivated candidates for
admission to post graduate courses like M.D., M.S. & M.Pharm.
I compliment the author for his pains-taking efforts in mobilizing a very large number of good MCQs from a vast
subject like Pharmacology. Adequate coverage of all topics is done.
I feel that this book will be a very useful companion for professional PG entrance examinations. I strongly recom-
mend this book to college library collection

PROF. DR. KANTIBHAI GOR


Vice Chancellor
K. S. K. V. Kachchh University
Bhuj - Kutch.
ACKNOWLEDGEMENTS

I acknowledge the help rendered by the following well wishers during the preparation of the manuscript.
• Dr. Ananta Naik Nagappa
Professor, Pharmacy practice, Manipal College of Pharm. Sciences, Manipal, Karnataka.
• Dr. K. R. Mahadik
Principal, Bharati Vidyapeeth Deemed University’s College of Pharmacy, Pune.
• Prof. Vijay Raghunath Patil
Principal, Tapi Valley Education Society’s College of Pharmacy
Faizpur, Maharashtra.
• Dr. Havigiray R. Chitme
Professor of Pharmacology, Oman medical College, Oman
• Dr. Chandrakant S. Magdum
Vice Principal, Shri Appasaheb Birnale College of Pharmacy,
Sangli, Maharashtra.
• Dr. B. P. Nagori
Director, Lachchoo Memorial College of Pharmacy,
Jodhpur.
My sincere thanks are due to Mr. Ojas M. Suroo, Lecturer, Computer Science of my institute for his meticulous
typing the manuscript and the final format of the book.
Finally, I would like to place on record the generous help rendered by New age International (P) Limited,
Publishers in bringing out this book.

Prof. Dr. G.Vidya Sagar.


This page
intentionally left
blank
This page
intentionally left
blank
CONTENTS

Preface (xi)

CHAPTER 1 1
General Pharmacology

CHAPTER 2 29
Drugs Acting on Central Nervous System

CHAPTER 3 85
Drugs Acting on Autonomous Nervous System

CHAPTER 4 103
Vitamins & Minerals

CHAPTER 5 107
Analgesics & Antipyretics

CHAPTER 6 121
Cardiovascular Drugs

CHAPTER 7 135
Drugs Used in Respiratory Disorders

CHAPTER 8 143
Antibiotics

CHAPTER 9 157
Drugs Used in Gastrointestinal Tract Disorders

CHAPTER 10 165
Oxytocics & Uterine Muscle Relaxants

CHAPTER 11 167
Chemotherapy
xiv

CHAPTER 12 193
Drugs used in Endocrine disorders (Hormones)

CHAPTER 13 207
ANTIDIABETICS

CHAPTER 14 211
Anticoagulants

CHAPTER 15 215
Antihyperlipedemic agents

CHAPTER 16 217
Antacids

CHAPTER 17 221
Antiemetics

CHAPTER 18 223
Match the Following
CHAPTER 1
GENERAL PHARMACOLOGY

1. All of the following are general (e) Distribution of a drug out of the bloodstream
mechanisms of drug permeation Except terminates the drug’s effect
(a) Aqueous diffusion 4. Distribution of drugs to specific tissues
(b) Aqueous hydrolysis
(a) Is independent of blood flow to the organ
(c) Lipid diffusion
(b) Is independent of the solubility of the drug in
(d) Pinocytosis or endocytosis that tissue
(e) Special carrier transport (c) Depends on the unbound drug concentration
2. If the plasma concentration of a drug gradient between blood and tissue
declines with “first-order kinetics”, this (d) Is increased for drugs that are strongly bound
means that to plasma proteins
(a) There is only one metabolic path for drug (e) Has no effect on the half-life of the drug
disposition
5. A physical process by which a weak acid
(b) The half-life is the same regardless of the becomes less water-soluble and more
plasma concentration lipid-soluble at low pH is
(c) The drug is largely metabolized in the liver
(a) Distribution (b) Elimination
after oral administration and has low
bioavailability elimination (c) First-pass effect (d) Permeation
(d) The rate of climination is proportionate to (e) Protonation
the rate of administration at all times 6. Dose-response curves are used for drug
(e) The drug is not distributed outside the evaluation in the animal laboratory and
vascular system in the clinic, Quantal dose-response
curves are often
3. Regarding termination of drug action
(a) Used for determining the therapeutic index
(a) Drug must be exerted from the body to
terminate their action of a drug
(b) Metabolism of drugs always increases their (b) Used for determining the maximal efficacy
water solubility of a drug
(c) Metabolism of drugs always abolishes their (c) Invalid in the presence of inhibitors of the
pharmacologic activity drug being studied
(d) Hepatic metabolism and renal excretion are (d) Obtained from the study of intact subject but
the two most important mechanisms involved not from isolated tissue preparations
2 MCQs IN PHARMACOLOGY

(e) Used to determine the statistical variation (a) Maximal efficacy


(standard deviation) of the maximal response (b) Therapeutic index
to the drug. (c) Drug potency
7. The following are excreted faster in basic (d) Grade dose-response curve
urine (e) Quantal dose-response curve
(a) Weak acids (b) Strong acids 12. Which of the following most accurately
(c) Weak Bases (d) None of the above describes the transmembrane signaling
process involved in steroid hormone
8. Which of the following statements about action ?
spare receptors is most correct ?
(a) Action on a membrane spanning tyrosine
(a) Spare receptors, in the absence of drug, are kinase
sequestered in the cytoplasm
(b) Activation of a G protein which activates or
(b) Spare receptors will be detected if the inhibits adenyl cyclase
intracellular effect of drug-receptor interaction (c) Diffusion into the cytoplasm and binding to
lasts longer than the drug-receptor interaction an intracellular receptor
itself
(d) Diffusion of “STAT” molecules across the
(c) Spare receptors influence the maximal membrane
efficacy of the drug-receptor system
(e) Opening of transmembrane ion channels
(d) Spare receptors activate the effector
machinery of the cell without the need for a 13. Which of the following is a phase II drug-
drug metabolizing reaction ?
(e) Spare receptors may be detected by the (a) Acetylation (b) Deamination
finding that the EC50 is greater than the Kd (c) Hydrolysis (d) Oxidation
for the agonist (e) Reduction
9. Which of the following terms best 14. Which of the following drugs may inhibit
describes an antagonist that interacts the hepatic microsomal P450 responsible
directly with the agonist and not at all or for warfarin metabolism
only incidentally, with the receptor ? (a) Cimetidine (b) Ethanol
(a) Pharmacological antagonist (c) Phenobarbital (d) Procainamide
(b) Partial agonist (e) Rifampin
(c) Physiological antagonist
15. With regard to clinical trials of new drugs,
(d) Chemical antagonist which of the following is most correct ?
(e) Noncompetitive antagonist
(a) Phase I involves the study of a small number
10. Which of the following terms best of normal volunteers by highly trained clinical
describes a drug that blocks the action of pharmacologists
epinephrine at its receptors by occupying (b) Phase II involves the use of the new drug in a
those receptors without activating them ? large number of patients (100-5000) who
have the disease to be treated
(a) Pharmacological antagonist
(c) Phase III involves the determination of the
(b) Partial agonist
drug’s therapeutic index by the cautious
(c) Physiological antagonist induction of toxicity
(d) Chemical antagonist (d) Chemical antagonist
(e) Noncompetitive antagonist (e) Phase II requires the use of a positive control
11. Which of the following provides information (a known effective drug) and a placebo
about the variation in sensitivity of the drug 16. Animal testing of potential new
within the population studied ? therapeutic agents
GENERAL PHARMACOLOGY 3

(a) Extends over a time period of at least 3 years (c) Physostigmine (d) Pilocarpine
in order to discover late toxicities (e) Neostigmine
(b) Requires the use of at least two primate 22. Phenylephrine causes
species, eg. Monkey and baboon
(a) Constriction of vessels in the nasal mucosa
(c) Requires the submission of histopathologic
(b) Increased gastric secretion and motility
slides and specimens to the FDA for
government evaluation (c) Increased skin temperature
(d) Has good predictability for drug allergy-type (d) Miosis
reactions (e) All of the above
(e) May be abbreviated in the case of some very 23. Pretreatment with propranolol will block
toxic agents used in cancer which one of the following ?

17. The “dominant lethal” test involves the (a) Methacholine-induced tachycardia
treatment of a male adult animal with a (b) Nicotine-induced hypertension
chemical before mating; the pregnant (c) Norepinephrine-induced bradycardia
female is later examined for fetal death (d) Phenylephrine-induced mydriasis
and abnormalities. The dominant lethal
24. Most drug receptors are
test therefore is a test of
(a) Small molecules with a molecular weight
(a) Teratogenicity between 100 and 1000
(b) Mutagenicity (b) Lipids arranged in a bilayer configuration
(c) Carcinogenicity (c) Proteins located on cell membranes or in the
(d) All of the above cytosol
(e) None of the above (d) DNA molecules
(e) RNA molecules
18. The Ames test is a method for detecting
25. With regard to distribution of a drug from
(a) Carcinogenesis in rodents
the blood into tissues
(b) Carcinogenesis in primates
(a) Blood flow to the tissue is an important
(c) Teratogenesis in any mammalian species
determinant
(d) Teratogenesis in primates
(b) Solubility of the drug in the tissue is an
(e) Mutagenesis in bacteria
important determinant
19. “Nicotinic” sites include all of the (c) Concentration of the drug in the blood is an
following except important determinant
(a) Bronchial smooth muscle (d) Size (volume) of the tissue is an important
(b) Adrenal medullary cells determinant
(c) Parasympathetic ganglia (e) All of the above are important determinants
(d) Skeletal muscle
26. The pH value is calculated mathematically
(e) Sympathetic ganglia
as the
20. A good example of chemical antagonism (a)

Log of the hydroxyl ion (OH ) concentration
(a) Heparin & Protamine (b)

Negative log of the OH concentration
(b) Protamine & Zinc (c)
+
Log of the hydrogen ion (H ) concentration
(c) Heparin & Prothrombin +
(d) Negative log of the H concentration
(d) All the above + –
(e) Ratio of H /OH concentration
21. Which of the following agents is a pro-
drug that is much less toxic in mammals 27. Which property is classified as colligative?
than in insects ? (a) Solubility of a solute
(a) Acetylcholine (b) Bethanechol (b) Osmotic pressure
4 MCQs IN PHARMACOLOGY

(c) Hydrogen ion (H+) concentration (c) 0.3 µm (d) 0.2 µm


(d) Dissociation of a solute (e) 0.1 µm
(e) Miscibility of the liquids
34. In the extemporaneous preparation of a
28. The colligative properties of a solution are suspension, levigation is used to
related to the (a) Reduce the zeta potential
(a) pH of the solution (b) Avoid bacterial growth
(b) Number of ions in the solution (c) Reduce particle size
(c) Total number of solute particles in the solution (d) Enhance viscosity
(d) Number of unionized molecules in the (e) Reduce viscosity
solution
35. Active transport differs from facilitated
(e) pKa of the solution
transport in following ways, except
29. The pH of a buffer system can be (a) Carrier is involved
calculated with the (b) It is against concentration gradient
(a) Noyes – Whitney equation (c) Energy is required
(b) Henderson – Hasselbalch equation (d) All of the above
(c) Michaelis – Menten equation
36. Vanishing cream is an ointment that may
(d) Yong equation
be classified as
(e) Stokes equation
(a) A water –soluble base
30. Which mechanism is most often (b) An oleaginous base
responsible for chemical degradation? (c) An absorption base
(a) Racemization (b) Photolysis (d) An emulsion base
(c) Hydrolysis (d) Decarboxylation (e) An oleic base
(e) Oxidation
37. Rectal suppositories intended for adult
31. Which equation is used to predict the use usually weigh approximately
stability of a drug product at room (a) 1g (b) 2g
temperature from experiments at
(c) 3g (d) 4g
accelerated temperature?
(e) 5g
(a) The stokes equation
(b) The Yong equation 38. In the fusion method of making cocoa
butter suppositories,which substance is
(c) The Arrhenius equation
most likely to be used to lubricate the mold?
(d) The Michaelis – Menten equation
(e) The Hixson – Crowell equation (a) Mineral oil (b) Propylene glycol
(c) Cetyl alcohol (d) Stearic acid
32. Based on the relation between the degree
(e) Magnesium silicate
of ionization and the solubility of a weak
acid, the drug aspirin (pKa 3.49) will be 39. A very fine powdered chemical is defined
most soluble at as one that
(a) pH 1.0 (b) pH 2.0 (a) Completely passes through a # 80 sieve
(c) pH 3.0 (d) pH 4.0 (b) Completely passes through a # 120 sieve
(e) pH 6.0 (c) Completely passes through a # 20 sieve
33. The particle size of the dispersed solid in (d) Passes through a # 60 sieve and not more
a suspension is usually greater than than 40% through a # 100 sieve
(e) Passes through a # 40 sieve and not more
(a) 0.5 µm (b) 0.4 µm
than 60% through a # 60 sieve
GENERAL PHARMACOLOGY 5

40. Which technique is typically used to mill (b) Carboxymethyl cellulose


camphor? (c) Cellulose acetate phthalate
(a) Trituration
48. For each tablet processing problem listed
(b) Levigation
below, select the most likely reason for
(c) Pulverization and intervention the condition
(d) Geometric dilution
(a) Excessive moisture (1) Picking
(e) Attrition
in the granulation
41. Which type of paper best protects a (b) Entrapment of air (2) Mottling
divided hygroscopic powder? (c) Tablet friability (3) Capping
(a) Waxed paper (b) Glassine (d) Degraded drug (4) Sticking
(c) White bond (d) Blue bond (e) Tablet hardness
(e) Vegetable parchment
49. For each description of a comminution
42. Which capsule size has the smallest procedure below, select the process that
capacity? it best describes
(a) 5 (b) 4
(a) Trituration
(c) 1 (d) 0
(b) Spatulation
(e) 000
(c) Levigation
43. The shells of soft gelatin capsules may (d) Pulverization by intervention
be made elastic or plastic–like by the
(e) Tumbling
addition of
(1) Rubbing or grinding a substance in a
(a) Sorbitol
mortar that has a rough inner surface
(b) Povidone
(2) Reducing and subdividing a substance
(c) Polyethylene glycol by adding an easily removable solvent
(d) Lactose
(3) adding a suitable agent to form a paste
(e) pKa of the solution and then rubbing or grinding the paste
44. Nonionic surface-active agents used as in mortar
synthetic emulsifiers include
50. Match the drug product below with the
(a) Tragacanth type of controlled-release dosage form
(b) Sodium lauryl sulphate that it represents
(c) Sorbitan esters(spans)
(a) Matrix device (1) Biphenamine
45. A ceramic mortar may be preferable to a Capsules
glass mortar when (b) Ion-exchange (2) Thorazine
(a) A volatile oil is added to a powder mixture resin complex Spansule Capsules
(b) Colored substances (dyes) are mixed into a (c) Hydrocolloid (3) Valrelease
powder system
(c) Comminution is desired in addition to mixing (d) Osmotic system (4) Slow - K
46. Divided powders may be dispensed in (e) Coated granules
(a) Individual-dose packets 51. The route of drug administration that
(b) A bulk container gives the most rapid onset of the phar-
(c) A perforated, sifter –type container macological effect is
47. Agents that may be used to coat enteric (a) Intramuscular injection
coated tablets include (b) Intravenous injection
(a) Hydroxypropyl methyl cellulose (c) Intradermal injection
6 MCQs IN PHARMACOLOGY

(d) Peroral administration (b) Glomerular filtration rate (GFR)


(e) Subcutaneous injection (c) Active renal secretion
52. Acidic drugs mainly bind to plasma (d) Passive renal absorption
(e) Drug metabolism rate
(a) Albumin
(b) á1 – acid glycoprotein 59. The earliest evidence that a drug is stored
(c) Both (a) and (b) in tissue is
(d) None of the above (a) An increase in plasma protein binding
53. After peroral administration, drugs
generally are absorbed best from the (b) A large apparent volume of distribution (VD)
(a) Buccal cavity (b) Stomach (c) A decrease in the rate of formation of
metabolites by the liver
(c) Duodenum (d) Ileum
(e) Rectum (d) An increase in the number of side effects
produced by the drug
54. The passage of drug molecules from a (e) A decrease in the amount of free drug
region of high drug concentration to a excreted in the urine
region of low drug concentration is
known as 60. The intensity of the pharmacologic action
(a) Active transport (b) Bioavailability of a drug is most dependent on the
(c) Biopharmaceutics (d) Simple diffusion (a) Concentration of the drug at the receptor site
(e) Pinocytosis (b) Elimination half-life (t½) of the drug
55. What equation describes the rate of drug (c) Onset time of the drug after oral
dissolution from a tablet? administration
(d) Minimum toxic concentration (MTC) of the
(a) Fick’s law
drug in plasma
(b) Henderson – Hasselbach equation
(e) Minimum effective concentration (MEC)
(c) Law of mass action
of the drug in the body
(d) Michaelis – Menten equation
(e) Noyes Whitney equation 61. Drug that show nonlinear pharmacoki-
netics have which property?
56. Dose dumping is a problem in the
formulation of (a) A constant ratio of drug metabolites is formed
as the administered dose increases
(a) Compressed tablets
(b) The elimination half-life (t½) increases as the
(b) Modified- release drug products
administered dose increases
(c) Hard gelatin capsules
(c) The area under the plasma drug
(d) Soft gelatin capsules concentration versus time curve (AUC)
(e) Suppositories increases in direct proportion to an increase
57. The rate of drug bioavailability is most in the administered dose
rapid when the drug is formulated as a (d) Both low and high doses follow first-order
(a) Controlled – release product elimination kinetics
(b) Hard gelatin capsule (e) The steady-state drug concentration increases
in direct proportion to the dosing rate
(c) Compressed tablet (d) Solution
(e) Suspension 62. The loading dose (DL) of a drug is usually
based on the
58. Creatinine clearance is used as a mea-
surement of (a) Total body clearance (ClT) of the drug
(a) Renal excretion rate (b) Percentage of drug bound to plasma proteins
GENERAL PHARMACOLOGY 7

(c) Fraction of drug excreted unchanged in the urine 68. The principle of superposition in designing
(d) Apparent volume of distribution (VD) and multiple-dose regimens assumes that
desired drug concentration in plasma (a) Each dose affects the next subsequent dose
(e) Area under the plasma drug concentration causing nonlinear elimination
versus time curve (AUC) (b) Each dose of drug is eliminated by zero-order
63. The renal clearance of insulin is used as elimination
a measurement of (c) Steady-state plasma drug concentration are
(a) Effective renal blood flow reached at approximately 10 half-lives
(b) Rate of renal drug excretion
(d) Early doses of drug do not affect subsequent
(c) Intrinsic enzyme activity
doses
(d) Active renal secretion
(e) The fraction of drug absorbed is equal to
(e) Glomerular filtration rate (GFR) the fraction of drug eliminated
64. All of the following statements about 69. Which equation is true for a zero-order
plasma protein binding of a drug are true reaction rate of drug ?
except
(a) dA/dt = - k (b) t½= 0.693/k
(a) Displacement of a drug from plasma protein
binding sites results in a transient increased (c) A = A0e -kt
volume of distribution (VD) 70. Which of the following functional groups
(b) Displacement of a drug from plasma protein is most susceptible to hydrolysis ?
binding sites makes more free drug available
(a) R – CO – R (b) R – COOR
for glomerular filtration
(c) R – O - R (d) R – NH- CH3
(c) Displacement of a potent drug that is normally
more than 95% bound may cause toxicity (e) R – COOH
(d) Albumin is the major protein involved in 71. Monomer units of proteins are known as
protein binding of drugs
(a) Monosaccharides (b) Prosthetic groups
(e) Drugs that are highly bound to plasma
(c) Amino acids (d) Purines
proteins generally have a greater V D
compared with drugs that are highly bound (e) Nucleosides
to tissue proteins 72. Glucose is a carbohydrate that cannot be
65. _______ is expressed in both the intestinal hydrolyzed into a simpler substance. It is
epithelium and the kidney. best described as
(a) CYP2D6 (b) CYP1A1/2 (a) A sugar (b) A monosaccharide
(c) CYP3A4 (d) CYP2E1 (c) A disaccharide (d) A polysaccharide
(e) An oligosaccharide
66. The initial distribution of a drug into tissue
is determined chiefly by the 73. All of the following carbohydrates are
(a) Rate of blood flow to tissue considered to be polysaccharides except
(b) Glomerular filtration rate (GFR) (a) Heparin (b) Starch
(c) Stomach emptying time (c) Glycogen (d) Maltose
(d) Affinity of the drug for tissue (e) Cellulose
(e) Plasma protein binding of the drug
74. Which of the following compounds are
67. Which tissue has the greatest capacity to considered the building blocks of nucleic
bio-transform drugs? acids ?
(a) Brain (b) Kidney (a) Nucleotides (b) Nucleosides
(c) Liver (d) Lung (c) Monosaccharides (d) Purines
(e) Skin (e) Amino acids
8 MCQs IN PHARMACOLOGY

75. Which of the following terms best 83. Which class of antibody has the longest
describes a co-factor that is firmly bound serum half-life and opsonizes antigens for
to an apoenzyme? phagocytosis through two different
(a) Holoenzyme (b) Prosthetic group pathways?
(c) Coenzyme (d) Transferase (a) Immunoglobulin G (IgG)
(e) Heteropolysaccharide (b) Immunoglobulin M(IgM)
76. Enzymes that uncouple peptide linkages (c) Immunoglobulin A (IgA)
are best classified as (d) Immunoglobulin E (IgE)
(a) Hydrolases (b) Ligases 84. Urticaria that appears rapidly after the
(c) Oxidoreductases (d) Transferases ingestion of food usually indicates which
(e) Isomaerases type of hypersensitivity reaction?

77. The sugar that is inherent in the nucleic (a) Type I (b) Type II
acids RNA and DNA is (c) Type III (d) Type IV
(a) Glucose (b) Sucrose 85. A patient receives long-term, high-dose
(c) Ribose (d) Digitoxose therapy with a sulfonamide. After
(e) Maltose approximately 3 weeks of therapy, the
patient has a low-grade fever, rash, and
78. N-oxidation will be involved with the muscle and joint pain. Which type of
metabolism of following drugs, except hypersensitivity accounts for these
(a) Dapsone (b) Meperidine symptoms?
(c) Phenytoin (d) Chlorpheniramine (a) Type I (b) Type II
79. Which of the following statements (c) Type III (d) Type IV
describes plasmids? They
86. CD4+T cells specifically recognize antigens
(a) Are single – stranded DNA molecules in which form?
(b) Carry optional gene(s) (a) Bound to major histocompatibility (MHC)
(c) Carry genes essential for growth class I molecules on the surface of any body
(d) Are always found in linear form cell
80. Bacteria that grow at temperatures as (b) In free, soluble form in extracellular fluids
high as 550C are known as (c) Bound to MHC class II molecules on the
surface of special antigen-presenting cells
(a) Psychrophiles (b) Thermophiles
(APCs)
(c) Mesophiles (d) Auxotrophs
87. Which of the following statements
81. Which of the following organisms can use
concerning a drug receptor is true?
only molecular oxygen as the final
acceptor? (a) It mediates the nonspecific action of volatile
anesthetics
(a) Obligate anaerobes
(b) Facultative anaerobes (b) Its expression is induced only by exogenously
added drugs
(c) Obligate aerobes
(c) It can bind endogenous ligand to produce
(d) Strict anaerobes
physiological activity
82. A declining growth rate occurs during (d) It mediates the cathartic activity of
which of the following phases of bacterial magnesium citrate
cell growth? (e) Down-regulation of receptor level can lead
(a) Lag phase (b) Exponential phase to sensitization of the target cell to the
(c) Stationary phase (d) Death phase receptor agonist.
GENERAL PHARMACOLOGY 9

88. Which of the following acids has the (c) All drugs are excreted more rapidly in an
highest degree of ionization in an alkaline urine.
aqueous solution?
94. Which of the following drugs is
(a) Aspirin pKa = 3.5 considered to be the agent of choice for
(b) Indomethacin pKa = 4.5 anaphylactic reactions?
(c) Warfarin pKa = 5.1 (a) Edrophonium (b) Ipratropium
(d) Ibuprofen pKa = 5.2 (c) Ambenonium (d) Propantheline
(e) Phenobarbital pKa = 7.4 (e) Homatropine
89. Which of the following salts forms an 95. Which of the following drugs is considred
aqueous solution that is alkaline to to be the agent of choice for anaphylactic
litmus? reactions?
(a) Sodium chloride (a) Clonidine (b) Isoproterenol
(b) Benzalkonium chloride (c) Epinephrine (d) Phenylephrine
(c) Meperidine hydrochloride (e) Terbutaline
(d) Cefazolin sodium
96. Which of the following emissions from the
(e) Chlordiazepoxide hydrochloride decay of radionuclides is most commonly
90. Hydrolysis reaction are involved with the used in nuclear medicine imaging?
metabolism of following drugs, except (a) X-ray (b) Beta
(a) Procaine (b) Diazepam (c) Alpha (d) Gamma
(c) Aspirin (d) Lidocaine (e) Positron

91. Flurazepam has pKa of 8.2. What 97. Which of the following radionuclides is
percentage of flurazepam will be ionized mot commonly used in nuclear pharmacy
at a urine pH of 5.2? practice?

(a) 0.1% (b) 1% (a) 67


GA (b) 201
TI
(c) 50% (d) 99% (c) 99m
TC (d) 123
I
(e) 99.9% (e) 133
Xe

92. Precipitation may occur when mixing 98. Which of the following radionuclides is
aqueous solutions of meperidine hydro- generator produced?
chloride with which of the following (a) 99m
TC (b) 201
TI
solutions? (c) 67
GA (d) 133
Xe
(a) Sodium bicarbonate injection (e) 123
I
(b) Atropine sulfate injection 99. Abrasives, ingredients in dentifrices, are
(c) Sodium chloride injection noted for which of the following actions?
93. The excretion of a weakly acidic drug (a) Providing flavor
generally is more rapid in alkaline urine (b) Cleansing via a foaming detergent action
than in acidic urine. This process occurs (c) Removing plaque and debris
because (d) Preventing dental caries
(a) A weak acid in alkaline media will exist (e) Adding thickness to the product
primarily in its ionized form, which cannot
be reabsorbed easily 100. The appropriate pH range for ophthalmic
products is
(b) A weak acid in alkaline media will exist in
its lipophilic form, which cannot be (a) 2.0 – 3.0 (b) 4.0 – 6.0
reabsorbed easily. (c) 6.0 – 8.0 (d) 8.0 – 10.0
10 MCQs IN PHARMACOLOGY

101. Which type of contact lens can most easily (a) Renal insufficiency
be ruined by the absorption of chemicals? (b) Hirschsprung’s disease
(a) Hard lenses (b) Soft lenses (c) Anorectal malformations
(c) Gas-permeable lenses (d) Children between the ages of 6 and 12 years
102. All of the following desensitizing agents 107. Which of the following factors is as-
are recommended for sensitive teeth sociated with an increased risk of non-
except compliance in the elderly?
(a) 10% carbamide peroxide (a) Polypharmacy
(b) 5% potassium nitrate (b) Hypertension
(c) Dibasic sodium citrate (c) Male gender
(d) 10% strontium chloride hexahydrate (d) Living with a spouse in an isolated environment
(e) Expensive medications
103. Carbamide peroxide appears to soften
earwax by 108. The principal difference between compet-
(a) Causing oxygen to be released, which itive and non-competitive inhibition is
loosens the wax (a) Extent of receptor site blocking
(b) Stimulating fluid secretion in the ear canal (b) Whether inhibition occurs
(c) Actually dissolving the ear wax (c) Extent of enzyme inhibition
(d) Decreasing lipid content of the wax (d) Degree of agonism
(e) None of the above (e) None of the above
104. A common oral problem caused by 109. Drug administrated through which of the
herpes simplex type I virus (HSV-1) is following routes is mot likely to be
(a) Aphthous ulcers subjected to first-pass metabolism:
(b) Canker sores (a) Oral (b) Sublingual
(c) Aphthous stomatitis (c) Subcutaneous (d) Rectal
(d) Fever blisters
110. Many receptors use distinct hetero
(e) Thrush _________ GTPä-binding regulatory
105. The definition of a sur factant (an proteins
ingredient in toothpaste) can best be (a) Tetrameric (b) Trimeric
described by which of the following (c) Dimeric (d) Monomeric
statements? Surfactant
111. Alkalinization of urine hastens the
(a) Prevents drying of the preparation
excretion of
(b) Removes debris by its detergent action and
(a) Weakly basic drugs
causes foaming, which is usually desired by
the patient (b) Weakly acidic drugs
(c) Physically removes plaque and debris (c) Strong electrolytes
(d) Determines the texture, dispersiveness, and (d) Nonpolar drugs
appearance of the product 112. Majority of drugs cross biological
(e) Adds flavor to the preparation which makes membranes primarily by
it more appealing to the patient
(a) Weakly basic drugs
106. Which is not a risk factor for hyper- (b) Weakly acidic drugs
phosphatemia and death from sodium (c) Strong electrolytes
phosphate enemas when used in
(d) Nonpolar drugs
children?
GENERAL PHARMACOLOGY 11

113. The most important factor which governs (a) Are bound primarily to a1 acid glycoprotein
diffusion of drugs across capillaries other in plasma
than those in the brain is (b) Are excreted faster in alkaline urine
(a) Blood flow through the capillary (c) Are highly ionized in the gastric juice
(b) Lipid solubility of the drug (d) Do not cross blood—brain barrier
(c) pKa value of the drug 120. High plasma protein binding
(d) pH of the medium (a) Increases the volume of distribution of the
114. Active transport of a substance across drug
biological membrane has the following (b) Facilitates glomerular filtrtion of the drug
characteristics except (c) Minimizes drug interactions
(a) It is specific (d) Generally makes the drug long acting
(b) It is pH dependent 121. Biotransformation of drugs is primarily
(c) It is saturable directed to
(d) It requires metabolic energy (a) Activate the drug
115. Bioavailability differences among oral (b) Inactivate the drug
formulations of a drug are most likely to (c) Convert lipid soluble drugs into nonlipid
occur if the drug soluble metabolites
(d) Convert nonlipid soluble drugs into lipid
(a) Is freely water soluble
soluble metabolites
(b) Is completely absorbed
(c) Is incompletely absorbed 122. A prodrg is
(d) Undergoes little first-pass metabolism (a) The prototype member of a class of drugs
(b) The oldest member of a class of drugs
116. Bioavailability of drug refers to
(c) An inactive drug that is transformed in the
(a) Perecentage of administered dose that body to an active metabolite
reaches systemic circulation in the (d) A drug that is stored in body tissues and is
unchanged form then gradually released in the circulation
(b) Rario of oral to parental dose
123. Which of the following cytochrome P450
(c) Ratio of orally administered drug to that
isoenzymes is involved in the metabolism
excreted in the faeces
of a large number of drugs in human
(d) Ratio of drug excreted unchanged in urine beings and has been implicated in some
to that excreted as metabolites dangerous drug interactions:
117. The most important factor governing (a) CYP 3A4 (b) CYP 2C9
absorption of a drug from intact skin is (c) CYP 2E1 (d) CYP 1A2
(a) Molecular weight of the drug 124. The most commonly occurring conjugation
(b) Site of application reaction for drugs and their metabolites is
(c) Lipid solubility of the drug (a) Glucuronidation
(d) Nature of the base used in the formulation (b) Acetylation
118. Redistribution is a feature of (c) Methylation
(d) Glutathione conjugation
(a) Highly plasma protein bound drugs
(b) Depot preparations 125. G-protein coupled receptors span the
(c) Poorly lipid soluble drugs plasma membrane as a bundle of _____
alpha helices
(d) Highly lipid soluble drugs
(a) One (b) Three
119. Weakly acidic drugs
(c) Seven (d) Ten
12 MCQs IN PHARMACOLOGY

126. Which of the following drug metabolizing (a) A constant amount of the drug will be
reactions is entirely nonmicrosomal eliminated per unit time
(a) Glucuronide conjugation (b) Its clearance value will remain constant
(b) Acetylation (c) Its elimination half-life will increase with dose
(c) Oxidation (d) It will be completely eliminated from the body
(d) Reduction in 2 x half-life period

127. Induction of drug metabolizing enzymes 133. If a drug has a constant bio-availability
involves and first order elimination, its main-
tainance dose rate will be directly
(a) A conformational change in the enzyme proportional to its
protein to favor binding of substrate
molecules (a) Volume of distribution
(b) Expression of enzyme molecules on the (b) Plasma protein binding
surface of hepatocytes (c) Lipid solubility
(c) Enhanced transport of substrate molecules (d) Total body clearance
into hepatocytes
134. The following dose of a drug is governed
(d) Increased synthesis of enzyme protein by its
128. Drugs which undergo high degree of first- (a) Aqueous diffusion
pass metabolism in liver (b) Aqueous hydrolysis
(a) Have oral bioavailability (c) Lipid diffusion
(b) Are excreted primarily in bile (d) Pinocytosis or endocytosis
(c) Are contraindicated in liver disease
135. Monitoring plasma drug concentration is
(d) Exhibit zero order kinetics of elimination useful while using
129. Glomerular filtration of a drug is affected (a) Antihypertensive drugs
by its (b) Levodopa
(a) Lipid solubility (c) Lithium carbonate
(b) Plasma protein binding (d) MAO inhibitors
(c) Degree of ionization
136. Microsomal enzyme induction has one of
(d) Rate of tubular secretion the following features
130. If a drug undergoes net tubular secretion, (a) Takes about one week to develop
its renal clearance will be (b) Results in increased affinity of the enzyme
(a) More than the glomerular filtration rate for the substrate
(b) Equal to the glomerular filtration rate (c) It is irreversible
(c) Less than the glomerular filtration rate (d) Can be used to treat acute drug poisonings
(d) Equal to the rate of urine formation 137. Which of the following is a competitive
131. Which of the following is not a primary/ type of enzyme inhibitor
fundamental, but a derived pharmaco- (a) Acetazolamide (b) Disulfiram
kinetic parameter (c) Physostigmine (d) Theophyline
(a) Bio-availability
138. What is true in relation to drug receptors
(b) Volume of distribution
(a) All drugs act through specific receptors
(c) Clearance
(b) All drug receptors are located on the surface
(d) Plasma half life
of the target cells
132. If a drug is eliminated by first order (c) Agonists induce a conformational change in
kinetics the receptor
GENERAL PHARMACOLOGY 13

(d) Partial agonists have low affinity for the 145. Placebo effects result presumably from
receptor the
139. A partial agonist can antagonize the (a) Physician-patient relationship
effects of a full agonist because it has (b) Mental set up imparted by the therapeutic
(a) High affinity but low intrinsic activity settings
(b) Low affinity but high intrinsic activity (c) Mental set up imparted by the physician
(c) No affinity and low intrinsic activity (d) All of the above
(d) High affinity but no intrinsic activity 146. All of the following subserve as
140. Receptor agonists possess intracellular second messengers in
receptor mediated signal transduction
(a) Result in increased smooth endoplasmic except
reticulum
(a) Cyclic AMP
(b) Result in increased rough endoplasmic reticulum
(b) Inositol trisphosphate
(c) Result in decreased enzymes in the soluble
cytoplasmic fraction (c) Diacyl glycerols
(d) Require 3–4 months to reach completion (d) G protein

141. Agonists affect the receptor molecule in 147. A receptor which itself has enzymatic
the following manner property is
(a) Alter its amino acid sequence (a) Insulin receptor
(b) Denature the receptor protein (b) Progesterone receptor
(c) Alter its folding or alignment of subunits (c) Thyroxine receptor
(d) Induce covalent bond formation (d) Glucagon receptor

142. Receptors perform the following function/ 148. Down regulation of receptors can occur
functions as a consequence of
(a) Ligand recognition (a) Continuous use of agonists
(b) Signal transduction (b) Continuous use of antagonists
(c) Both ligand recognition and signal (c) Chronic use of CNS depressants
transduction (d) Denervation
(d) Disposal of agonists and antagonists
149. When therapeutic effects decline both
143. Which of the following receptor types has below and above a narrow range of
7 helical membrane, spanning amino acid doses, a drug is said to exhibit
segments with 3 extracellular and 3
(a) Ceiling effect
intracellular loops
(b) Desensitization
(a) Tyrosine protein kinase receptor
(c) Therapeutic window phenomenon
(b) Gene expression regulating receptor
(d) Nonreceptor mediated action
(c) Intrinsic ion channel containing receptor
(d) G protein coupled receptor 150. ‘Drug efficacy’ refers to

144. Which of the following is a G protein (a) The range of diseases in which the drug is
coupled receptor beneficial
(b) The maximal intensity of response that can
(a) Muscarinic cholinergic receptor
be produced by the drug
(b) Nicotinic cholinergic receptor
(c) The therapeutic dose range of the drug
(c) Glucocorticoid receptor
(d) The therapeutic index of the drug
(d) Insulin receptor
14 MCQs IN PHARMACOLOGY

151. If the dose-response curves of a drug for 158. A fixed dose combination preparation
producing different actions are widely meant for internal use must not contain
separated on the dose axis, the drug is the following class of drug
(a) Highly potent (b) Highly efficacious (a) Thiazide diuretic
(c) Highly toxic (d) Highly selective (b) Fluoroquinolone antimicrobial
152. The therapeutic index of a drug is a (c) Corticosteroid
measure of its (d) H2 blocker
(a) Safety (b) Potency 159. Interindividual variations in equieffective
(c) Efficacy (d) Dose variability doses of a drug are most marked if it is
153. If the effect of combination of two drugs disposed by
is equal to the sum of their individual (a) glomerular filtration
effects, the two drugs are exhibiting (b) Tubular secretion
(a) Potentiation (b) Synergism (c) Both (a) and (b)
(c) Cross tolerance (d) Antagonism (d) Hepatic metabolism
154. The antagonism between adrenaline and 160. The pharmacokinetics of drugs in the
histamine is called ‘Physiological anta- neonate differs from that in adults,
gonism’ because because their
(a) Both are physiologically present in the body (a) Intestinal transit is fast
(b) They act on physiological receptors (b) Glomerular filtration rate is high
(c) Both affect many physiological processes (c) Tubular transport mechanisms are not well
(d) They have opposite physiological effects developed
155. A drug ‘R’ producing no response by itself (d) Drug metabolizing enzymes are overactive
causes the log dose-response curve of
161. Pharmacodynamic tolerance may involve
another drug ‘S’ to shift to the right in a
changes in _____of drug receptors
parallel manner without decreasing the
maximal response : Drug ‘R’ is a (a) Number (b) Affinity
(a) Partial agonist (c) Function (d) All the above
(b) Inverse agonist 162. Drug metabolism can be induced by the
(c) Competitive antagonist following factors except
(d) Noncompetitive antagonist (a) Cigarette smoking
156. A drug which does not produce any (b) Acute alcohol ingestion
action by itself but decreases the slope (c) Exposure to insecticides
of the log dose-response curve and
(d) Consumption of characoal boiled meat
suppresses the maximal response to
another drug is a 163. Which of the following is true of placebos
(a) Physiological antagonist (a) Placebo is a dummy medication
(b) Competitive antagonist (b) Placebo is the inert material added to the
(c) Noncompetitive antagonist drug for making tablets
(d) Partial agonist (c) Placebos do not produce any effect
157. A drug which is generally administered (d) All patients respond to placebos
in standard doses without the need for 164. In patients of hepatic cirrhosis
dose individualization is
(a) The extent of change in pharmacokinetics of
(a) Insulin (b) Mebendazole drugs can be predicted from the values of
(c) Prednisolone (d) Digoxin liver function tests
GENERAL PHARMACOLOGY 15

(b) High doses of furosemide can be safely used (a) Chloroquine (b) Warfarin sodium
(c) Metformin is the preferred oral hypoglycaemic (c) Dicyclomine (d) Methyldopa
(d) Disposition of atenolol is not significantly
171. Select the drug which has been found to
affected
be a strong human teratogen
165. An undersireable effect of a drug that (a) Isoniazid (b) Isotretinoin
occurs at therapeutic doses and can be (c) Hydralazine (d) Propylthiouracil
predicted from its pharmacological
actions is called 172. Oral route
(a) Side effect (b) Toxic effect (a) Yields better absorption than parenteral
(c) Allergic reaction (d) Idiosyncrasy administration for majority of drugs
(b) Should be avoided in the recumbent position
166. Which of the following is a type II
(c) Has no demerit
(unpredictable) adverse drug reaction
(d) Produces quick onset of action
(a) Side effect
(b) Toxic effect 173. Intramuscular route
(c) Idiosyncrasy (a) Always produces faster absorption than oral
(d) Physical dependence route
(b) Can be used to inject mild irritants
167. A ‘toxic effect’ differs from a ‘side effect’
(c) In a child is made into the gluteus maximus
in that
muscle
(a) It is not a pharmacological effect of the drug (d) Can be used to inject a volume of 15 ml
(b) It is a more intense pharmacological effect
that occurs at high dose or after prolonged 174. Advantage of sublingual route include the
medication following except
(c) It must involve drug induced cellular injury (a) It has a rapid onset of action
(d) It involves host defence mechanisms (b) Spitting out the tablet can terminate its action
(c) Its usefulness is limited to treat local conditions
168. Which of the following statements is true
in relation to ‘drug toxicity’ and ‘poisoning’ (d) It avoids first pass hepatic metabolism

(a) The two terms are synonymous 175. First pass metabolism
(b) When a toxic effect requires specific (a) Can increase the oral bio-availability of the
treatment, it is called poisoning drug
(c) A toxic effect which endangers life by (b) Occurs only in the liver
markedly affecting vital functions is called (c) Is higher on intravenous administration
poisoning
(d) Necessitates high oral dose for certain drugs
(d) Toxicity is caused by drugs while poisoning
is caused by other harmful chemicals 176. Bio-transformation

169. The most vulnerable period of pregnancy (a) Renders the drug more lipid soluble
for the causation of foetal malformations (b) Can be altered by drugs
due to drugs is (c) Is necessary for all drugs for their elimination
(a) 18–55 days of gestation (d) Takes place only in the liver
(b) 56–84 days of gestation 177. Entry of a drug in the central nervous
(c) Second trimester system is enhanced if the drug is
(d) 36 weeks onwards (a) Ionized
170. Which of the following is a proven human (b) More lipid soluble
teratogen (c) Given intravenously
(d) Highly plasma protein bound
16 MCQs IN PHARMACOLOGY

178. Kinetic processes of elimination for a (a) In aqueous solution


large number of drugs is (b) In suspension
(a) First order (c) In oily solution
(b) First order followed by zero order (d) In solid form
(c) Zero order followed by first order 185. Alcohol is rapidly absorbed from the
(d) Zero order intestine because
179. A drug is said to be potent when (a) It is lipid soluble and non-electrolyte
(a) It produces maximal response (b) It is lipid soluble and highly ionised
(b) The amount needed to produce a certain (c) It is absorbed by active transport
response is less (d) It is not absorbed quickly
(c) It produces minimal/no side effects 186. After intramuscular injection, the drugs
(d) It has a rapid onset of action
(a) In oily solution are more rapidly absorbed
180. Spare receptors are often found among (b) In aqueous solution are more rapidly
drugs that elicit absorbed
(a) Smooth muscle contraction (c) Suspended in various repository vehicles are
more rapidly absorbed
(b) Smooth muscle relaxation
(d) All solutions are rapidly absorbed
(c) Secretion
(d) Cardiac stimulation 187. Bio-availability of a drug is

181. What is the best criterian for judging the (a) The percentage of drug released from a
therapeutic superiority of a drug over its formulation that becomes available for
congeners? biological effect
(b) The percentage of drug that is ionized from
(a) Potency
a formulation
(b) Wide range of activity
(c) The net amount of actual therapeutic agent
(c) Efficacy present in the formulation
(d) Variability (d) The dose of a drug by which 50% animals
182. pKa of a compound show signs of toxicity

(a) Is the pH of solution at which the compound 188. The absorption time of a drug can be
is 50% ionized reduced by
(b) Is the pH of compound at which it is 50% (a) Making a more soluble salt – for oral
ionized (b) By using hyaluronidase – for injection
(c) Is the time in which the compound is ionized (c) By using vesoconstrictor substances
(d) Is the time in which total compound is ionized (d) By giving combination of drugs
183. Pharmacokinetics is 189. Reduction of heavy metal toxicity by
(a) The study of absorption, distribution, dimercaprol is an exmple of
metabolism and excretion of drugs (a) Chemical antagonist
(b) The study of biological and therapeutic effects (b) Physiological antagonist
of the drugs (c) Pharmacokinetic antagonist
(c) The method of development of new (d) Antagonism by receptor block
pharmacological agent
190. Which of the following drugs is primarily
(d) The study of carcinogenic activity of a new
stored in the natural fat of the body
drug
(a) Primaquine (b) Acetyl salicylic acid
184. In which form the drug is absorbed more
(c) Thiopentone (d) Vitamin C
rapidly?
GENERAL PHARMACOLOGY 17

191. Bio-transformation of the drugs is to 198. The receptor concept was first introduced
render them by
(a) Less lipid soluble (a) J.N. Langley in 1878
(b) More protein bound (b) Paul Ehrlich in 1926
(c) Less ionized (c) Somonis in 1964
(d) Less protein bound (d) Wakesman in 1826
192. Drug metabolism occurs chiefly in 199. True tolerance develops because of
(a) Liver (b) Brain (a) Diminition in absorption
(c) Spleen (d) Kidneys (b) Rapid excretion of the drug
193. For renal excretion the factors important (c) Both of the above
are (d) None of the above
(a) Extent of plasma protein binding of drugs 200. Two drugs binding to the same receptors
(b) Glomerular filtration rate is
(c) Active renal tubular reabsorption (a) Chemical antagonism
(d) All of the above (b) Pharmacokinetic antagonism
194. If tubular urine is made more acid (c) Competitive antagonism
(d) Non-competitive antagonism
(a) Excretion of weak acid is reduced
(b) Excretion of weak acid is increased 201. Tachyphylaxis is
(c) Excretion of weak base is increased (a) A drug interaction between two similar types
(d) Excretion of weak base is reduced of drugs
(b) Rapidly developing tolerance
195. Simultaneous administration of two
drugs may – (select the false statement) (c) A synergism between two types of drugs
(a) Show an additive effect (d) None of the above
(b) Produce synergism 202. Drug A in a dose of 10 mg produces same
(c) Result in antagonism response as with 100 mg of drug B
(d) Produce any of above phenomena (a) Drug A is 10 times more potent than drug B
196. The advantages of bio-assay over (b) Drug B is 10 times more potent than drug A
chemical assay include – (select false (c) Drug A is 10 times more efficacious than
statement) drug B
(a) It is cheaper (d) Both are equally potent
(b) The active principal does not have to be (e) Both are equally efficacious
known 203. Teratogenicity is
(c) The active principal does not have to be in a
(a) The acute reaction to drugs
pure state
(b) Intolerance to drugs
(d) The sensitivity may be greater
(c) Tumour forming action of the drugs
197. In presence of competitive antagonist (d) Malformation of the foetus
(a) The maximum response of agonist can never
204. The chances of foetal malformation with
be achieved
a teratogenic drug is maximum
(b) The maximum can be achieved by increasing
the concentration activity (a) During first trimester of pregnancy
(c) Maximum can be achieved only if the (b) During second trimester of pregnancy
antagonist is having intrinsic activity (c) During third trimester of pregnancy
(d) None of the above (d) When given just prior to the labor
18 MCQs IN PHARMACOLOGY

205. Phocomelia is a known teratogenic effect 212. Which one of the following is an example
of of physical or chemical interaction?
(a) Anticancer drugs (a) Warfarin plus salicylates–prolongation of
(b) Antiviral drugs anticoagulant effect and bleeding tendency
(c) Antiepileptic drugs (b) Methotrexate plus sulfonamides–pancytopenia
(d) Thalidomide (c) Heparin plus protamine–reversal of heparin
effect
206. Which of the following drugs are known
(d) Sulfonamides plus salicylate–sulfa toxicity
to cause toxic cataract?
(a) Chloroquine (b) Ergot 213. First order kinetics of the drugs is called
when
(c) Phenothiazine (d) Naphthalene
(e) All of the above (a) A constant fraction of the drug is removed in
per unit time
207. The passage of drugs into the foetus from (b) A constant amount of the drug is removed in
placenta per unit time
(a) Is by active transport (c) Total amount of the drug is removed in one
(b) Is by passive diffusion hour
(c) Is by carrier mediated transport (d) Total amount of the drug is removed in first
(d) By any of the above methods passage through the kidneys

208. Idiosyncrasy reaction of a drug is 214. For the drugs which follow first order
kinetics, after 4 half life the elimination
(a) A type of hypersensitivity reaction
will be approximately
(b) A type of drug antagonism
(a) 40% (b) 94%
(c) Unpredictable, inherent, qualitatively
abnormal reaction of a drug (c) 25% (d) 4%
(d) Quantitatively exaggerated response 215. Passive diffusion of a drug across cell
209. Two drugs having similar effects are membrane is low when its molecular
termed as mass is greater than

(a) Heterergic drugs (a) 50–100 Da (b) 100–200 Da


(b) Isomer drugs (c) 200–300 Da (d) 300–400 Da
(c) Homergic drugs 216. Passage of drug across most capillary
(d) Antagonistic drugs endothelial membranes is dependent
upon
210. If the combined effect of two drugs acting
by the same mechanism is equal to the (a) Lipid solubility (b) pH gradient
allegebric sum of their individual effect, (c) Blood flow (d) All of the above
it is called as
217. Following receptors are membrane
(a) Antagonism (b) Additive effect proteins, except
(c) Potentiation (d) None of the above (a) Receptors for fast neurotransmitters, coupled
211. Antagonism between barbiturate and directly to an ion channel
amphetamine is termed as (b) Receptors for many hormones and slow
transmitters, coupled to effector system
(a) Non-competitive antagonism
(c) Receptor for insulin and various growth
(b) Physiological antagonism
factors, which are directly linked linked to
(c) Competitive antagonism
tyrosine kinase
(d) Synergism
(d) Receptors for steroid hormone
GENERAL PHARMACOLOGY 19

218. pH difference between extracellular and 227. The hepatic enzyme inducer naringenin
intracellular fluid is is present in
(a) Nil (b) 0.2 (a) Tobacco smoke (b) Grape juice
(c) 0.4 (d) 0.8 (c) Alcohol (d) Apple Juice
219. Which type of drugs penetrate CNS better 228. In gene tranfer which metal particle is
(a) Lipid soluble (b) Weak acids often used
(c) Weak bases (d) All equally (a) Iron (b) Gold
220. Acidic drugs mostly bind to plasma (c) Platinum (d) Mobybdenum
(a) Albumin (b) Globulin 229. Slow reacting substance of anaphylaxis
(c) Glycoprotein (d) None of the above refers to
221. The number of P 450 gene families (a) LTC4 (b) LTD4
identified in human being is (c) Both of the above (d) None of the above
(a) 4 (b) 8 230. Drugs producing allergic reaction
(c) 12 (d) 16 generally act as
222. The majority of drug biotransformation (a) Complete antigens
occurs by which cytochrome family (b) Haptens
(a) CYP1 (b) CYP2 (c) Antibodies
(c) CYP3 (d) None of the above (d) Mediators
223. Nonlinearity in pharmacokinetics of a 231. An addicting drug which produces little
drug is due to saturation of or no physical dependence is
(a) Protein binding (a) Amphetamine (b) Methadone
(b) Hepatic metabolism (c) Phenobarbitone (d) Diazepam
(c) Active renal transport
(d) All of the above 232. Which of the following statements
regarding therapeutic window is correct ?
224. Which of the following disease is due to
(a) The ratio of LD50 to the ED50
G protein receptor malfunction
(b) The Dosage range between the minimum
(a) Precocious puberty effective therapeutic concentration and the
(b) Retinitis pigmentosa minimum toxic concentration
(c) Malignant hyperthyroidism (c) Both the above
(d) All of the above (d) None of the above
225. The pharmacokinetic alternations in 233. Which of the following statements
elderly are due to
regarding acid –base balance is not
(a) Reduction in lean body mass and total body correct ?
water
(a) it is an essential balance between the amount
(b) Increase in percentage of body fat of carbonic acid and biocarbonate in blood.
(c) Reduced cytochrome P450 enzymes
(b) It must be kept constant so that the hydrogen
(d) All of the above in concentration in the blood plasma is in
226. The effect of enzyme induction is greatest turn kept constant
when the drug is given (c) Any deviation in the balance can have a
(a) Digoxin (b) Furosemide profound effect on physiological function
(c) Enalapril (d) Amrinone (d) All the above
(e) None of the above
20 MCQs IN PHARMACOLOGY

234. During liver disease the metabolism and (b) Bioavailabiligy


elimination of which of the following (c) Bioequivalence
drugs is decreased
(d) Fixed Dose combination
(a) Morphine (e) All the above
(b) Pentobarbitone (f) None of the above
(c) Propanolol
(d) All the above 240. Following statement is true about
receptors linked directly to ion channels
(e) None of the above
(a) These receptors are involved mainly in fast
235. In celiac disease oral absorption of which synaptic transmission
of the following drugs is decreased
(b) These are monomeric proteins containing
(a) Amoxycillin one transmembrane segment
(b) Cephalexin (c) Ligand binding and channel opening occur
(c) Cotrimoxazole on a minute time-scale
(d) All the above (d) All of the above
236. Which people are said to be slowest 241. Route of administration suitable for
acetylators because they metabolize emergency and permits titration of the
isoniazid by the process of acetylation dosage as well is
very slowly
(a) Oral (b) Intravenous
(a) Canadian Eskimos
(c) Intramuscular (d) Subcutaneous
(b) Indians
(e) All the above (f) None of the above
(c) Asiatic Jews
(d) Chinese 242. The disadvantage of oral route is
(e) Europeans (a) Vomiting as a result of gastrointestinal
(f) all the above irritation
(b) Destruction of some drugs by the digestive
237. Which people are said to be fastest
acetylators because they metabolize enzyme and non-favorable gastric pH
isoniazid by the process of acetylation (c) Irregularities in the absorption in the presence
very quickly of the food and other drugs
(a) Canadian Eskimos (d) Patient non-compliance
(b) Indians (e) All the above
(c) Asiatic Jews (f) None of the above
(d) Chinese 243. Drugs which have a tendency to accumu-
(e) Europeans late in the body fat mostly have
(f) all the above (a) Extremely high lipid-water partition co-
238. Clearance of which of the following drugs efficient
is reduced parallel to decrease in the (b) Extremely low lipid-water partition co-efficient
creatine clearance (c) None of the above
(a) Aminoglycosides (b) Digoxin
244. Once the drug enters the blood, the rate
(c) Phenobarbitone (d) All the above
at which it subsequently penetrates the
(e) None of the above tissues and other body fluids depends on
239. Pharmaceutical factors which can modify (a) Capillary permeability
the effect of drug are (b) Extent of plasma protein and tissue binding
(a) Formulation type (c) Transport mechanism
GENERAL PHARMACOLOGY 21

(d) All the above (a) Oxidation (b) Reduction


(e) None of the above (c) Acetylation (d) All the above
245. Factor which can effect the absorption of 252. The metabolic reaction which makes the
drug is metabolite of codeine more Pharmaco-
(a) Dissolution rate (b) Particle Size logically potent analgesic is
(c) Lipid Solubility (d) All the above (a) Dealkylation (b) Oxidation
(e) None of the above (c) Deamination (d) All the above
246. The movement of drug molecules across 253. Prodrug
the cell membrane is by (a) Facilitates absorption and distribution of
(a) Diffusion through the lipid drugs with poor lipid solubility
(b) Diffusing through aqueous pores that traverse (b) Increases the duration of action of drugs that
the lipid are rapidly eliminated
(c) Combination with a carrier molecule which (c) Promotes Site specific delivery of drugs
acts as a catalyst (d) All the above
(d) Pinocytosis
(e) All the above 254. The removal of oxygen or an alteration
in a drug which leads to a decrease in
(f) None of the above the proportion of oxygen in the drug
247. Which of the following responses develops compound is known as
to drugs due to antigen-antibody reaction ? (a) Oxidation (b) Reduction
(a) Toxic response (c) Hydrolysis (d) All the above
(b) Idiosyncratic response
255. Excipients are
(c) Allergic response
(a) Pharmacologically inert substances
(d) None of the above
(b) Used to mask an unpleasant taste
248. The microsomal oxidation of drugs is (c) Used to increase solubility or stability to the
carried out through a larger or smaller agent
extent in
(d) Employed to add bulk to the active agent
(a) Liver (b) Kidney used in small quantities
(c) Lung (d) small intestine (e) All the aboves
(e) all of the above
256. Bio-assay is used to
249. The mechanism of Biotransformation of
(a) Determine the relationship between the dose
Aspirin to Salicylic acid and Acetic acid is
administered and the magnitude of response
(a) Oxidation (b) Reduction
(b) Determine the potency of a new agent
(c) Hydrolysis (d) None of the above compared with that of an established drug
250. Which one of the statements regarding (c) Determine the relationship between the doses
microsomal enzymes is not correct producing a desired effect and those eliciting
(a) They lack specificity undesirable or toxic effect
(b) Capable of metabolizing substances of (d) All the above
different structure (e) None of the above
(c) Only catalyze reaction of compounds which 257. Which of the following endogenous
are lipid insoluble compounds undergo methylation ?
(d) All the above
(a) Histamine (b) Estradiol
251. Which one of the following is a Phase II – (c) Thyroxine (d) All the above
Drug metabolizing reaction (e) None of the above
22 MCQs IN PHARMACOLOGY

258. Which one of the following statements structure of drugs and the effects that
regarding glucuronide conjugation is not they produce
correct (a) Paracelsus (b) William Harvey
(a) The most frequently occurring reactions (c) James Blake (d) Rudolf Buccheim
(b) The conjugating agent is glucuronic acid, (e) None of the above
C6H10O6
264. The great scientist who explained the
(c) The glucuronide are rapidly eliminated in the
circulation of the blood to the world which
urine
signaled of the beginning of the scientific
(d) The glucuronides are also secreted in the bile, study of the medical sciences was
but this does not always lead to their
elimination in the feces (a) Paracelsus (b) William Harvey
(e) All the above (c) James Blake (d) Rudolf Buccheim
(e) None of the above
(f) None of the above
265. All the below mentioned drugs cause
259. Which of the following therapeutic sys-
enzyme inhibition in man except one
tems provides continuous, unattended,
controlled drug input for a long period (a) Acetazolamide (b) Allopurinol
without gastrointestinal or hepatic drug (c) Meprobamate (d) Disulfiram
inactivation prior to systemic circulation ? (e) Selegiline
(a) Parenteral (b) Oral 266. All the below mentioned drugs cause
(c) Transdermal (d) All the above enzyme induction in man except one
(c) None of the above (a) Phenytoin (b) Phenobarbitone
260. The translocation of a solute from one (c) Griseofulvine (d) Enalapril
side of a biologic barrier to the other side (e) Rifampicin
and the transferred solute appearing in
the same form on both side of the 267. Cimetidine potentiates the action of
biological barrier is known as propranol, theophylline, warfarin and
phenytoin because
(a) Bioavailability (b) Biotransport
(a) It causes deficiency of G - 6–PD
(c) Bioequivalence (d) None of the
(b) It blocks the H2 – histaminergic receptors
261. Guanine nucleotides bind to the ______ (c) It is an inhibitor of microsomal P–450
subunit of G-proteins
(d) None of the above
(a) α (b) β
268. The incidence of adverse drug reaction
(c) γ (d) All the above
rises with age in the adult, especially after
262. Pharmacodynamics include 65 years because of
(a) The biological effects produced by chemicals (a) The increasing number of drugs they need
(b) The site/s and mechanism by which the to take because they tend to have multiple
biological effects are produced diseases
(c) The factors that effect the safety and (b) Poor compliance with dosing regimens
effectiveness of the agent (c) Bodily changes of aging that require
(d) All the above modification of dosage regimens
(e) None of the above (d) All the above
(e) None of the above
263. The person who set forth the principles
that drugs are effective only after 269. Agranulocytosis is
reaching a responsive tissue and that (a) Virtual absence from the blood of white cells
there is a relationship between the known as neutrophils
GENERAL PHARMACOLOGY 23

(b) It is a life-threatening conditions that results (b) Drugs with low safety margin
from toxic damage to the bone-marrow by (c) Drugs activated in the body
some drugs (d) Hit and run drugs
(c) Can be treated with antibiotics and
sometimes transfusion of white blood cells 275. During competitive equilibrium type of
antagonism
(d) All the above
(a) Equilibrium constant (Km) is unchanged, but
270. Characteristically following oral admin- total number of binding sites (Vmax) is reduced
istration to a man, drugs (b) Equilibrium constant (Km) is increased and
(a) Are absorbed readily when in the unionized total number of binding sites (Vmax) is reduced
rather than in the ionized form (c) Equilibrium constant (Km) is increased and
(b) Are absorbed primarily in the intestine total number of binding sited (Vmax) remain
(c) Cross membranes mainly by simple diffusion unchanged
(d) With high lipid solubility readily penetrate (d) None of the above
into CNS 276. Addition of oxygen or negatively charged
(e) All the above radical and removal of hydrogen or
positively charged radical is known as
271. Competitive antagonists
(a) Reduction (b) Oxidation
(a) Dissociate from receptors faster than their
(c) Acetylation (d) Hydrolysis
respective agonists
(e) None of the above
(b) Alter the shape of the log dose response
curve of an agonist 277. Pharmacopoeia includes list of esta-
(c) According to the rate theory have low blished drugs and medicinal preparation
dissociation rate constants with the description of their
(d) Initiate the opposite cellular response to (a) Physical properties
receptor occupancy to that obtained by the (b) Identification test
agonist (c) Purification test
(e) All the above (d) Potency test
272. A non-competitive antagonist (e) All the above
(a) Alters the mechanism of action of an agonist 278. Claude Bernard is known as
(b) Alters the potency of an agonist (a) Father of Medicine
(c) Shifts the dose-response curve of an agonist (b) Father of Pharmacognosy
to the right (c) Father of Polypharmacy
(d) Decreases the maximum response to an (d) Father of Pharmacology
agonist
(e) Father of Experimental Medicine
(e) None of the above
279. Oswald Schmiedeberg is known as
273. Repeated administration of a drug may
(a) Father of Medicine
(a) Increase its own metabolism
(b) Father of Pharmacognosy
(b) Increase the metabolism of other drugs (c) Father of Polypharmacy
(c) Increase the metabolism of endogenous (d) Father of Pharmacology
compounds
(e) Father of Experimental Medicine
(d) All the above
(e) None of the above 280. Catalytic conversion of GTP to GDP is
carried out by _____ subunit of G-proteins
274. Plasma concentration is useful for
(a) α (b) β
(a) Drugs with high safety margin (c) γ (d) All the above
24 MCQs IN PHARMACOLOGY

281. Theophrastus is known as 288. Acetylcholine and atropine action on the


(a) Father of Medicine muscarinic receptors is a classical exam-
ple of
(b) Father of Pharmacognosy
(a) Competitive antagonism
(c) Father of Polypharmacy
(b) Non-competitive antagonism
(d) Father of Pharmacology
(c) Non-equilibrium antagonism
(e) Father of Experimental Medicine
(d) Physiological antagonism
282. Hippocrates is known as (e) Chemical antagonism
(a) Father of Medicine
289. Biological half-life of is calculated as
(b) Father of Pharmacognosy
(a) t ½ = 0.693 x CL/Vd
(c) Father of Polypharmacy
(b) t ½ = 0.693 x K/ln2
(d) Father of Pharmacology
(c) t ½ = 0.693 x Vd/CL
(e) Father of Experimental Medicine
(d) t ½ = 0.693 x ln2/K
283. When a drug lacking effect of its own (e) None of the above
increases the effects of the second active
drug [0 +1= >1]; such an effect is called 290. Following statement is true regarding
therapeutic index
(a) Additive effect (b) Synergism
(a) It is based on animal toxicity data
(c) Potentiation
(b) It reflects forms of toxicity that are important
284. When two drugs with the same effect clinically
produce an effect greater than the sum (c) It takes into account idiosyncratic toxic reaction
of the effects of individual drugs [1 + 1>2]; (d) All the above
such an effect is called
291. Ocuserts are
(a) Additive effect (b) Synergism
(c) Potentiation (a) Placed under the eyelid
(b) Intrauterine contraceptives
285. Weak acids ionize more in (c) Monoclonal antibodies
(a) Alkaline pH (b) Acidic pH (d) None of the aboves
(c) Not dependent of pH
292. Nitroglycerin is given in angina pectoris
286. Theophylline has shorter half life in by sublingual route because:
infants and children because (a) Liver is by-passed
(a) Renal tubular secretor y mechanism is (b) Can be spat after desired effect
deficient (c) Rapid absorption
(b) Immaturity of the nephron (d) Non irritant and lipid soluble drug
(c) Immaturity of the blood-brain barrier (e) All the above
(d) Reduced plasma - protein binding
293. Volatile drug may be best administered
(e) Liver size larger on a body weight basis by:
287. An antagonist has (a) Oral route (b) Inhalation
(a) Intrinsic activity and no affinity (c) Sublingual route (d) Intrathecal route
(b) Only intrinsic activity and no affinity (e) Rectal route
(c) No intrinsic activity and no affinity 294. A desired clinical response may be
(d) Affinity same as agonist and devoid of delayed altered or blocked by:
intrinsic activity (a) A drug that does not go into solution
(e) None of the above (b) A drug that does not get to its site of action
GENERAL PHARMACOLOGY 25

(c) Abnormal pharmacogenetics (a) Topical (b) Oral


(d) Lack of absorption from site of administration (c) Parenteral
(e) All the above
302. Parenteral administration of a drug refers
295. Following compartment constitutes the to the giving of a preparation, except
largest percentage (a) Orally
(a) Plasma (b) Intestinal fluid (b) Intradermally
(c) Intracellular fluid (d) Fat (c) Intramuscularly
296. The duration of action of a drug is de- (d) Intravenously
pendent of its 303. Pharmacokinetics consIders
(a) Plasma and tissue binding
(a) The way in which the body affects a drug by
(b) Metabolism the process of absorption, distribution,
(c) Tubular filtration and secretion metabolism and excretion
(d) All the above (b) The effects of the drug on the body and the
mode of drug action.
297. When a drug has a low therapeutic
index, that drug should be (c) The proportion of administered drug that is
available to have an effect.
(a) Used mostly orally
(d) The removal of the drug by the liver before it
(b) Used mostly intravenously
has become available
(c) Considered a potentially toxic substance
(d) Given only in submilligram doses 304. If gut motility is increased then
(a) Drug absorption is decreased
298. A drug is being metabolized by zero-
order kinetics. This indicates that (b) Drug absorption is increased
(c) Drug absorption is not effected
(a) A constant amount of drug is being
catabolised each hour 305. The rate of drug absorption is greatest in
(b) A constant fraction of drug is being
(a) The small intestine
catabolised each hour
(b) The large intestine
(c) A drug is water soluble substance
(c) The stomach
(d) The time required to catabolise half of the
drug is independent of the initial con- 306. Drug distribution may depend on tissue
centration of the drug perfusion
299. Drugs interact with their receptors sites (a) Highly vascular organs rapidly acquire a
by forming drug
(a) Ionic bonds (b) Highly vascular organs acquire a drug slowly
(b) Hydrogen bonds (c) Levels of a drug in bone may rise quickly
due to its high vascularity
(c) Van der Waals bond
(d) All the above (d) Levels of a drug in bone may rise slowly due
to its reduced
300. The main route of administration of a
drug to produce a local effect is 307. Most drugs and metabolites are excreted
by
(a) Topical (b) Oral
(a) The kidneys
(c) Parenteral
(b) The bile
301. The main routes of administration of a (c) The lungs
drug is to provide a systemic effect.
(d) Perspiration, saliva and tears
Identify the wrong answer
26 MCQs IN PHARMACOLOGY

308. Pharmacodynamics considers: (a) Steroids (b) Vitamin D


(a) The way in which the body affects the drug (c) Retinoids (d) all the above
(b) The effects of the drug on the body and the 311. Factors affecting a client’s response to a
mode of drug action
drug include all the following except:
(c) Drug metabolism
(a) A body weight i.e. the larger the individual,
309. A drug that binds to a cell receptor and the larger the area for drug distribution
causes a response is called an:
(b) Body fat –A loss of body fat stores means
(a) Agonist less drug available for activity
(b) Antagonist (c) Body fat–An increase in body fat means
(c) Receptor blocker greater sequestering in body fat and less drug
310. Receptors for _________ are DNA-binding activity
proteins (d) The presence of certain foods

ANSWERS

1. b 2. b 3. d 4. c 5. e 6. a
7. a 8. b 9. d 10. a 11. e 12. c
13. a 14. a 15. a 16. a 17. b 18. e
19. a 20. a 21. a 22. a 23. a 24. c
25. e 26. d 27. b 28. c 29. b 30. c
31. c 32. e 33. a 34. c 35. a 36. d
37. b 38. a 39. b 40. c 41. a 42. a
43. a 44. a 45. c 46. a 47. c
48. 1. a, 2. d, 3. b, 4. a 49. 1. a, 2. d, 3. c 50. 1. b, 2. e, 3. c, 4. a
51. b 52. b 53. c 54. d 55. d 56. b
57. d 58. b 59. b 60. a 61. b 62. d
63. e 64. e 65. c 66. a 67. c 68. d
69. a 70. b 71. c 72. b 73. d 74. a
75. b 76. a 77. c 78. c 79. b 80. b
81. c 82. d 83. a 84. a 85. c 86. c
87. c 88. a 89. d 90. b 91. e 92. a
93. a 94. b 95. c 96. d 97. c 98. a
99. c 100. c 101. b 102. a 103. a 104. d
105. b 106. d 107. a 108. c 109. a 110. b
111. b 112. a 113. a 114. b 115. c 116. a
117. c 118. d 119. b 120. d 121. c 122. c
123. a 124. a 125. c 126. b 127. d 128. a
GENERAL PHARMACOLOGY 27

129. b 130. a 131. d 132. b 133. d 134. c


135. c 136. b 137. c 138. c 139. a 140. c
141. c 142. c 143. d 144. a 145. d 146. d
147. a 148. a 149. c 150. b 151. d 152. a
153. b 154. d 155. c 156. c 157. b 158. c
159. d 160. c 161. d 162. b 163. a 164. d
165. a 166. c 167. b 168. c 169. a 170. b
171. b 172. b 173. b 174. c 175. d 176. b
177. b 178. a 179. b 180. a 181. b 182. a
183. a 184. a 185. a 186. b 187. a 188. a
189. a 190. b 191. a 192. a 193. d 194. a
195. d 196. a 197. b 198. a 199. d 200. c
201. b 202. a 203. d 204. a 205. d 206. d
207. b 208. c 209. b 210. b 211. b 212. c
213. a 214. b 215. b 216. c 217. d 218. c
219. a 220. a 221. c 222. c 223. d 224. d
225. d 226. a 227. b 228. b 229. c 230. b
231. a 232. b 233. e 234. d 235. a 236. e
237. a 238. d 239. e 240. a 241. b 242. d
243. a 244. d 245. d 246. 5 247. c 248. e
249. c 250. c 251. c 252. a 253. d 254. b
255. d 256. b 257. d 258. f 259. c 260. b
261. a 262. d 263. c 264. b 265. c 266. d
267. c 268. d 269. d 270. e 271. c 272. d
273. d 274. b 275. c 276. b 277. e 278. e
279. d 280. a 281. b 282. a 283. c 284. b
285. a 286. e 287. d 288. a 289. c 290. a
291. a 292. e 293. b 294. e 295. c 296. d
297. c 298. a 299. d 300. a 301. a 302. a
303. a 304. a 305. a 306. a & d 307. a 308. b
309. a 310. d 311. a & d
28 MCQs IN PHARMACOLOGY

EXPLANATIONS FOR ANSWERS

7. a because of enhanced disassociation weak acids 180. a Spare receptors are generally found whenever
are excreted faster in basic (not acidic) urine. a receptor acts catalytically and not
20. a Chemical antagonism does not involve any stoichiometrically. Hence, they are generally
receptors. Protamine is a positively charged found among drugs, which elicit smooth muscle
protein at physiological pH and thus contraction.
antagonizes the effects of Heparin which is 189. a Dimercaprol interacts directly with heavy metals
negatively charged at physiological pH. in blood and tissue fluids and prevents binding
35. a Facilitated diffusion differs from active transport of heavy metals to cellular constituents.
in that it does not require energy source and it Dimercaprol forms an inactive complex with
carries the transport in the direction of heavy metals.
electrochemical gradient. 200. c Two drugs binding to the same receptor is an
52. a Glycoprotein binds to basic drugs and albumin example of competitive antagonism and effect
binds to many acidic drugs and a small number of one drug can be decreased by increasing
of basic drugs. the concentration of other drug.
65. c CYP3A4 is involved in the metabolism of 217. d Receptors for steroid hormones are intracellular
majority of drugs and is also expressed DNA – binding proteins, which regulate gene
extrahepatically. In fact, poor bioavailability of transcription.
many drugs is attributed to the drug metabolism 240. a Receptors linked directly to ion channels are
by CYP3A4 in gastrointestinal tract. involved in the fast synaptic transmission e.g.
78. c N-oxidation is involved in the metabolism of nicotinic acetylcholine receptor. This receptor is
dapsone, meperidine and chloramphenicol. a pentamer i.e. it is made up of five poly-peptide
Metabolism of phenytoin involves oxidative N- subunits.
dealkylation. 261. a In G-protein coupled receptors, agonist-receptor
110. b Many receptors use heterotrimeric GTP-binding complex enhances GTP binding to the α subunit,
regulatory proteins. The subunits are designated mainly by dissociating already bound GDP.
as β, α and α and their classification is based 280. a The termination of agonist-receptor coupling in
on á subunits. a G-protein-coupled receptor is because of
125. c G-protein coupled receptors are hydrophobic conversion of GTP to GDP by a GTPase that is
proteins and span the plasma membrane in intrinsic to α subunit.
seven á-helical segments. 290. a Therapeutic index is the range between minimal
145. d Placebo effect is a desired response to a therapy and maximal concentration required to elicit
without drug. Placebo effect is mainly because desired response.
of various psychological reasons e.g. physician- 295. c intracellular fluid comprises 35%, whereas
patient relationship, mental set up affected by contribution of other compartments is far less
surroundings and positive mental set up because e.g. plasma – 5%; interstitial fluid – 16%; fat –
of known and trusted physician. 20% and transcellular fluid – 2%.
161. d Pharmacodynamic tolerance involves multiple 310. d Receptors for steroids, vitamin D, retinoids and
mechanism such as number, affinity and function also thyroid hormones are DNA-binding
of receptors. proteins, which regulate transcription of genes.
DRUGS ACTING ON CENTRAL NERVOUS SYSTEM 29

CHAPTER 2
DRUGS ACTING ON CENTRAL
NERVOUS SYSTEM

1. A-50-year-old man has a history of (a) Acetylcholine (b) Citruline


frequent episodes of renal colic with high- (c) Isoproterenol (d) Nitroglycerin
calcium renal stones. The most useful
(e) Nitroprusside
agent in the treatment of recurrent
calcium stones is 6. The primary endogenous substrate for
(a) Mannitol (b) Furosemide nitric oxide synthase is
(c) Spironolactone (d) Hydrochlorothiazide (a) Acetylcholine (b) Angiotensinogen
(e) Acetazolamide (c) Arginine (d) Citruline
2. Which of the following drugs is correctly (e) Heme
associated with its site of action and
maximal diuretic efficacy? 7. Which of the following is a recognized
effect of nitric oxide?
(a) Thiazides–distal convoluted tubute–10% of
filtered Na+ (a) Arrhythmia
(b) Spironolactone–proximal convoluted tubule (b) Bronchoconstriction
–40% (c) Constipation
(c) Bumetanide–thick ascending limb–15% (d) Inhibition of acute graft rejection
(d) Metolazone–collecting tubule–2% (e) Pulmonary vasodilation
(e) All of the above
8. cAMP is an example of
3. Substance secreted into the blood by a
neuron is (a) Neurohormone (b) Neuromodulator
(a) Neurohormone (b) Neuromodulator (c) Neuromediator (d) Neurotransmitter
(c) Neuromediator (d) Neurotransmitter 9. One effect that theophylline, nitroglycerin,
4. Which of the following is a leukotriene isoproterenol, and histamine have in
receptor blocker? common is

(a) Alprostadil (b) Aspirin (a) Direct stimulation of cardiac contractile force
(c) Ibuprofen (d) LTC4 (b) Tachycardia
(e) Zafirlukast (c) Increased gastric acid secretion
5. A molecule that stimulates nitric oxide (d) Postural hypotension
synthase, especially the eNOS isoform, is (e) Throbbing headache
30 MCQs IN PHARMACOLOGY

10. Which one of the following chemicals 16. Suramin is an antagonists of ________
does not satisfy the criteria for a receptors
neurotransmitter role in the CNS? (a) Purine (b) Somatostanin
(a) Acetylcholine (b) Dopamine (c) Neuropeptide Y (d) Neurotensin
(c) Glycine (d) Nitric Oxide
17. Which one of the following statements
(e) Substance P
best describes the mechanism of action
11. Neurotransmitters may of benzodiazepines?
(a) Increase chloride conductance to cause (a) Benzodiazepines activate GABAB receptors
inhibition in the spinal cord
(b) Increase potassium conductance to cause (b) Their inhibition of GABA transminase leads
excitation to increased levels of GABA
(c) Increse sodium conductance to cause (c) Benzodiazepines block glutamate receptors
inhibition in hierarchical neuronal pathways in the
(d) Increase calcium conductance to cause brain
inhibition (d) They increase the frequency of opening of
(e) Exert all of the above actions chloride ion channels that are coupled to
GABAA receptors
12. Which of the following chemicals is most (e) They are direct-acting GABA receptor
likely to function as a neurotransmitter agonists in the CNS
in hierarchical systems?
18. Which one of the following statements
(a) Dopamine (b) Glutamate
about the use of triazolam in this elderly
(c) Metenkephalin (d) Norepinephrine patient is accurate?
(e) Serotonin
(a) Ambulatory dysfunction does not occur in
13. Activation of metabotropic receptors elderly patients taking one-half of the
located presynaptically causes inhibition conventional adult dose
by decreasing the inward flux of (b) Hypertension is a common adverse effects
(a) Calcium (b) Chloride of benzodiazepines in patients over 70 years
of age
(c) Potassium (d) Sodium
(c) Over-the-counter cold medications may
(e) None of the above
antagonize the hypnotic effects of the drug
14. This compound decrease the functional
activities of several CNS neurotransmit- (d) She may experience amnesia, especially if
ters, including dopamine, norepineph- she also drinks alcoholic beverages
rine, and serotonin. At high doses it may (e) Triazolam is distinctive in that it does not
cause parkinsonism-like extrapyramidal cause rebound insomnia on abrupt
system dysfunction. discontinuance
(a) Amphetamine (b) Baclofen
19. The most likely explanation for the
(c) Diazepam (d) Ketamine increased sensitivity of elderly patients
(e) Reserpine to a single dose of triazolam and other
sedative-hypnotic drugs is
15. This amine neurotransmitter is found in
high concentration in cell bodies in the (a) Changes in brain function that accompany
pons and brain stem; at some sites, the aging process
release of transmitter is autoregulated via (b) Decreased renal function
presynaptic inhibition. (c) Increased cerebral blood flow
(a) Acetylcholine (b) Dopamine (d) Decreased hepatic metabolism of lipid-
(c) Glutamate (d) Norepinephrine soluble drugs
(e) Substance P (e) Changes in plasma protein binding
DRUGS ACTING ON CENTRAL NERVOUS SYSTEM 31

20. Induction of various forms of synaptic (a) Additive CNS depression


plasticity is more closely associated with (b) Antagonism of sedative or hypnotic actions
______ receptors
(c) Competition for plasma protein binding
(a) AMPA (b) Kainate (d) Induction of liver drug-metabolizing enzymes
(c) NMDA (d) All of the above (e) Inhibition of liver drug-metabolizing enzymes
21. Which one of the following drugs may 26. A 42-year-old man with a history of
increase anticoagulant effects by alcoholism is brought to the emergency
displacement of warfarin from plasma
room in a confused and delirious state. He
protein binding sites and is inactive until
has truncal ataxia and ophthalmoplegia.
converted in the body to an active
The most appropriate immediate course of
metabolite?
action is to administer.
(a) Buspirone (b) Chloral hydrate
(a) Chlordiazepoxide (b) Disulfiram
(c) Clorazepate (d) Secobarbital
(c) Folic acid (d) Lorazepam
(e) Zaleplon
(e) Thiamine
22. Which one of the following drugs has
been used in the management of alcohol 27. Which one of the following statements
withdrawal states and in maintenance about the bio-disposition of ethanol is
treatment of patient with tonic-clonic or accurate?
partial seizure states? Its chronic use may (a) Ethanol is absorbed at all levels of the
lead to an increased metabolism of gastrointestinal tract
warfarin and phenytoin. (b) Acetic acid is the initial product of ethanol
(a) Chlordiazepoxide (b) Meprobamate metabolism
(c) Phenobarbital (d) Triazolam (c) After an intravenous dose, plasma levels of
(e) Zolpidem ethanol are lower in women than in men
(d) The elimination of ethanol follows first-order
23. A 40-year-old patient with liver
kinetics
dysfunction is scheduled for a surgical
procedure. Lorazepam can be used for (e) Alcohol dehydrogenase exhibits genetic
preanesthetic sedation in this patient variability
without concern for excessive CNS
28. Following is GABAA agonist
depression because the drug is
(a) Muscimol (b) Baclofen
(a) A selective anxiolytic like buspirone
(c) Bicuculline (d) None of the above
(b) Actively secreted in the renal proximal tubule
(c) Conjugated extrahepatically 29. Chronic use of ethanol is reported to
(d) Eliminated via the lungs increase
(e) Reversible by administration of naloxone (a) Alcohol dehydrogenase
24. This hypnotic drug facilitates the (b) Aldehyde dehydrogenase
inhibitory actions of GABA, but it lacks (c) Microsomal ethanol-oxidizing system activity
anticonvulsant or muscle relaxing (d) Monoamine oxidase
properties and has minimal effect on (e) NADH dehydrogenase
sleep architecture.
30. The chronic abuse of alcohol predisposes
(a) Buspirone (b) Diazepam
to hepatic damage following overdose of
(c) Flurazepam (d) Phenobarbital
acetaminophen because ethanol
(e) Zaleplon
(a) Blocks acetaminophen metabolism
25. The most frequent type of drug interaction (b) Causes thiamine deficiency
that occurs in patients using drugs of the
(c) Displaces acetaminophen from plasma
sedative hypnotic class is
proteins
32 MCQs IN PHARMACOLOGY

(d) Induces liver drug-metabolizing enzymes 36. Withdrawal of antiseizure drugs can
(e) Inhibits renal clearance of acetaminophen cause increased seizure frequency and
severity. Withdrawal is least likely to be
31. The activity of this enzyme is specifically a problem with
decreased in the Wernicke-Korsakoff
syndrome (a) Clonazepam (b) Diazepam
(c) Ethosuximide (d) Phenobarbital
(a) Alcohol dehydrogenase
(e) Phenytoin
(b) Cytochrome P450
(c) L-Aromatic amino acid decarboxylase 37. A young femal patient who suffers from
(d) NADH dehydrogenase bipolar affective disorder (BAD) has been
(e) Pyruvate dehydrogenase managed with lithium. If she becomes
pregnant, which one of the following
32. Following is glycine antagonists drugs is likely to be effective in bipolar
(a) Quisqualate (b) Taurine affective disorder with minimal risk of
(c) Strychnine (d) â-alanine teratogenicity?
(a) Carbamazepine (b) Clonazepam
33. Which one of the following statements
concerning the pharmacokinetics of (c) Phenytoin (d) Valproic acid
antiseizure drugs is accurate? (e) None of the above
(a) At high doses, phenytoin elimination follows 38. The most likely mechanism involved in
first-order kinetics the antiseizure activity of carbamazepine
(b) Valporic acid may increase the activity of is
hepatic ALA synthase and the synthesis of (a) Block of sodium ion channels
porphyrins
(b) Block of calcium ion channels
(c) The administration of phenytoin to patients
in methadone maintenance programs has led (c) Facilitation of GABA actions on chloride ion
to symptoms of opioid overdose, including channels
respiratory depression (d) Glutamate receptor antagonism
(d) Although ethosuximide has a half-life of (e) Inhibition of GABA transaminase
approximately 40 hours, the drug is usually
39. Which one of the following statements
taken twice a day
about phenytoin is accurate?
(e) Treatment with vigabatrin may reduce the
effectiveness of oral contraceptives (a) Displaces sulfonamides from plasma proteins
(b) Drug of choice in myoclonic seizures
34. With chronic use in seizure states, the
(c) Half-life is increased if used with pheno-
adverse effects of this drug include
barbital
coarsening of facial features, hirsutism,
gingival hyperplasia, and osteomalacia. (d) Isoniazid (INH) decreases steady state blood
levels of phenytoin
(a) Carbamazepine (b) Ethosuximide
(e) Toxicity may occur with only small increments
(c) Gabapentin (d) Phenytoin
in dose
(e) Valproic acid
40. Which one of the following statements
35. Which one of the following statements concerning nitrous oxide is accurate?
about vigabatrin is accurate?
(a) It continues to be a useful component of
(a) Blocks neuronal reuptake of GABA anesthesia protocols because of its lack of
(b) Drug of choice in absence seizures cardiovascular depression
(c) Is established to be teratogenic in humans (b) Megaloblastic anemia is a common adverse
(d) Life-threatening skin disorders may occur effect in patients exposed to nitrous oxide
(e) Visual field defects occur in up to one-third for periods longer than 2 hours
of patients (c) It is the most potent of the inhaled
DRUGS ACTING ON CENTRAL NERVOUS SYSTEM 33

anaesthetics receive other anaesthetics


(d) There is a direct association between the use (a) Enflurane (b) Ketamine
of nitrous oxide and malignant hyperthermia (c) Morphine (d) Propofol
(e) More than 30% of nitrous oxide is eliminated (e) Remifentanil
via hepatic metabolism
48. The pKa of lidocaine is 7.9. In infected
41. Following is GABAB agonist tissue at pH 6.9, the fraction in the ionized
(a) Muscimol (b) Baclofen form will be
(c) Picrotoxin (d) Bicuculline (a) 1% (b) 10%
42. Following is GABAA antagonist (c) 50% (d) 90%
(e) 99%
(a) Muscimol (b) Bicuculline
(c) Strychnine (d) Baclofen 49. Which of the following statements about
nerve blockade with local anaesthetics
43. The inhalation anesthetic with the fastest
is most correct?
onset of action is
(a) Block is faster in onset in infected tissues
(a) Enflurane (b) Isoflurane
(b) Block is faster in onset in unmyelinated fibers
(c) Nitric oxide (d) Nitrogen dioxide
(c) Block is slower in onset in hypocalcemia
(e) Nitrous oxide
(d) Block is faster in onset in hyperkalemia
44. An intravenous bolus dose of thiopental (e) Block is slower in onset in the periphery of a
usually leads to loss of onsciousness nerve bundle than in the center of a bundle
within 10–15 seconds. If no further drugs
are administered, the patient will regain 50. Which of the following was the first com-
consciousness in just a few minutes. The pound to be identified Pharmacologically
reason for this, that thiopental is as a transmitter in the CNS ?
(a) A good substrate for renal tubular secretion (a) Glycine (b) Glutamate
(b) Exhaled rapidly (c) Acetulcholine (d) Norepinephrine
(c) Rapidly metabolized by hepatic enzymes 51. You have a vial containing 4 mL of a 2%
(d) Redistributed from brain to other body tissues solution of lidocaine. How much lidocaine
(e) Secreted in the bile is present in 1 mL?
(a) 2 mg (b) 8 mg
45. Respiratory depression following use of
this agent may be reversed by adminis- (c) 20 mg (d) 80 mg
tration of flumazenil (e) 200 mg
(a) Desflurane (b) Fentanyl 52. Which one of the following statements
(c) Ketamine (d) Midazolam about the toxicity of local anaesthetics is
(e) Propofol most correct?

46. Use of this agent is associated with a high (a) Serious cardiovascular reactions are more likely
incidence of disorientation, sensory and to occur with tetracaine than with bupivacaine
perceptual illusions, and vivid dreams (b) Cyanosis may occur following injection of
during recovery from anesthesia large doses of lidocaine, especially in
patients with pulmonary disease
(a) Diazepam (b) Fentanyl
(c) Intravenous injection of local anaesthetics may
(c) Ketamine (d) Midazolam
stimulate ectopic cardiac pacemaker activity
(e) Thiopental
(d) In overdosage, hyper ventilation (with
47. Postoperative vomiting is uncommon oxygen) is helpful to correct acidosis and
with this intravenous agent; patients are lower extracellular potassium
able to ambulate sooner than those who (e) Most local anaesthetics cause vasoconstriction
34 MCQs IN PHARMACOLOGY

53. Epinephrine added to a solution of (a) Diazepam (b) Pancuronium


lidocaine for a peripheral nerve block will (c) Tizanidine (d) Tubocurarine
(a) Increase the risk of convulsions (e) Vecuronium
(b) Increase the duration of anesthetic action of 60. Which one of the following drugs has
the local anesthetic caused hyperkalemia leading to cardiac
(c) Both (A) and (B) arrest in patients with neurologic
(d) Neither (A) nor (B) disorders?
(e) None of the above (a) Baclofen (b) Dantrolene
54. A child requires multiple minor surgical (c) Succinylcholine (d) Tubocurarine
procedures in the nasopharynx. Which (e) Vecuronium
of the following drugs has high surface 61. Following is an excitatory amino acid
activity and vasoconstrictor actions that receptor antagonist?
reduce bleeding in mucous membrane?
(a) Phencyclidine (b) Quisqualate
(a) Benzocaine (b) Bupivacaine
(c) Homocysteate (d) Kainate
(c) Cocaine (d) Lidocaine
(e) Procaine 62. Which one of the following drugs has
spasmolytic activity and could also be
55. Characteristics of nondepolarizing used in the management of seizures
neuromuscular blockade include which caused by overdose of a local anesthetic?
one of the following? (a) Baclofen (b) Cyclobenzaprine
(a) Block of posttetanic potentiation (c) Dantrolene (d) Diazepam
(b) Histamine blocking action (e) Tizanidine
(c) Poorly sustained titanic tension
63. Which one of the following drugs given
(d) Significant muscle fasciculations during onset preoperatively will prevent postoperative
of block pain caused by succinylcholine?
(e) Stimulation of autonomic ganglia (a) Baclofen (b) Dantrolene
56. Which of the following does not cause (c) Diazepam (d) Lidocaine
skeletal muscle contactions or twitching? (e) Tubocurarine
(a) Acetylcholine (b) Nicotine 64. As the physician, you could tell the patient
(c) Strychnine (d) Succinylcholine (and close family members) all of the
(e) Vecuronium following things about levodopa except?

57. Which one of the following is most (a) Taking the drug in divided doses will
effective in the management of malignant decrease nausea and vomiting
hyperthermia? (b) He should be careful when he stands up
because he may get dizzy
(a) Baclofen (b) Dantrolene
(c) Uncontrollable muscle jerks may occur
(c) Haloperidol (d) Succinylcholine
(d) A net-like reddish to blue discoloration of
(e) Vecuronium the skin is a likely side effect of the medication
58. Following is a G-protein coupled receptor (e) The drug will probably improve his symptoms
for a period of time but not indefinitely
(a) AMPA (b) Kainate
(c) NMDA (d) Metabotropic 65. As the physician who is prescribing
levodopa, you will note that the drug
59. Which one of the following drugs is most
often associated with hypotension caused (a) Causes less severe behavioral side effects if
by histamine release? given with carbidopa
DRUGS ACTING ON CENTRAL NERVOUS SYSTEM 35

(b) Fluctuates in its effectiveness with increasing (c) The primar y therapeutic benefit of
frequency as treatment continues antimuscarinic drugs in parkinsonism is their
(c) Prevents extrapyramidal adverse effects of ability to reliever bradykinesia
antipsychotic drugs (d) Dopamine receptor against should not be
(d) Protects against cancer in patients with used in Parkinson’s disease prior to a trial of
melanoma levodopa
(e) Has toxic effects that include pulmonary (e) The concomitant use of selegiline may
infiltrates increase the peripheral adverse effects of
levodopa
66. The major reason why carbidopa is of
value in parkinsonism is that the 70. A previously healthy 50-year-old woman
compound begins to suffer from slowed mentation
(a) Crosses the blood-brain barrier and develops writhing movement of her
tongue and hands. In addition, she has
(b) Inhibits monoamine oxidase type A
delusions of being persecuted. The
(c) Inhibits aromatic L-amino acid decarboxylase woman has no past history of psychiatric
(d) Is converted to the false neurotransmitter or neurologic disorders. The most
carbidopamine appropriate drug for treatment is
(e) Inhibits monoamine oxidase type B
(a) Amantadine (b) Bromocriptine
67. Which one of the following statements (c) Haloperidol (d) Levodopa
about bromocriptine is accurate? (e) Trihexyphenidyl
(a) It should not be administered to patients
71. Great caution must be exercised in the
taking antimuscarinic drugs
use of this drug (or drugs from the same
(b) Effectiveness in Parkinson’s disease requires class) in parkinsonian patients who have
its metabolic conversion to an active prostatic hypertrophy or obstructive
metabolite gastrointestinal disease
(c) The drug is contraindicated in patients with
(a) Benztropine (b) Carbidopa
a history of psychosis
(c) Levodopa (d) Ropinirole
(d) The drug should not be administered to
patients already taking levodopa (e) Selegiline
(e) Mental disturbances occur more commonly 72. Which of the following statements about
with levodopa than with bromocriptine pramipexole is accurate?
68. A 72-year-old patient with parkinsonism (a) Activates dopamine D2 receptors
presents with swollen feet. They are red, (b) Commonly a first-line therapy for Parkinson’s
tender, and very painful. You could clear disease
up these symptoms within a few days if (c) May cause postural hypotension
you told the patient to stop taking
(d) Not an ergot derivative
(a) Amantadine (b) Benztropine
(e) All of the above
(c) Bromocriptine (d) Levodopa
(e) Selegiline 73. Tolcapone may be of value in patient
being treated with levodopa–carbidopa
69. Concerning the drugs used in because it
parkinsonism, which of the following
(a) Activates catechol – O – methyltransferease
statements is accurate?
(b) Decreases formation of 3 – O – methyldopa
(a) Levodopa causes mydriasis and can
(c) Inhibits monoamine oxidase type B
precipitate an attack of acute glaucoma
(d) Inhibits dopamine reuptake
(b) Useful therapeutic effects of amantadine
continue for several years (e) Releases dopamine from nerve endings
36 MCQs IN PHARMACOLOGY

74. Concerning hypotheses for the patho- (c) Give oral diphenydramine
physiologic basis of schizophrenia, which (d) Switch the patient to fluphenazine
one of the following statements is accu-
(e) Inject benztropine
rate?
(a) Positron emission tomography has shown 78. Which one of the following statements
decreased dopamine receptors in the brains about the action of phenothiazines is
of both untreated and drug-treated accurate?
schizophrenics (a) They activate muscarinic receptors
(b) Drugs that block dopamine receptors are (b) They are antiemetic
useful for alleviating psychotic symptoms in (c) They decrease serum prolactin levels
parkinsoniam patients
(d) They elevate the seizure threshold
(c) The clinical potency of many antipsychotic
(e) They raise blood pressure
drugs correlates well with their beta
adrenoceptor-blocking actions 79. Within days of starting haloperidol treat-
(d) Drug-induced psychosis can occur without ment for a psychatric disorder, a young
activation of brain dopamine receptors male patient developed severe general-
(e) All effective antipsychotic drugs have high ized muscle rigidity and a high fever. In
affinity for dopamine D2 receptors the emergency room he was incoherent,
with increased heart rate, hypotension,
75. Choose the correct statement from the and diaphoresis, Laboratory studies in-
following? dicated acidosis, leukocytosis, and in-
(a) Muscimol is GABAB selective creased creatine kinase. The most likely
(b) Bicuculline is GABAA agonist reason for these symptoms is that the pa-
(c) Picrotoxin blocks chloride channels tient was suffering from
associated with GABAA receptors (a) Agranulocytosis
(d) Baclofen is GABAA agonist (b) A severe bacterial infection
76. A 30-year-old male patient is on drug (c) Neuroleptic malignant syndrome
therapy for a psychiatric problem. He (d) Spastic retrocollis
complains that he feels “flat” and that (e) Tardive dyskinesia
he gets confused at times. He has been
gaining weight and has lost his sex drive. 80. Following is the main inhibitor y
As he moves his had, you notice a slight transmitter in the brain
tremor. He tells you that since he has been (a) Dopamine (b) Norepinephrine
on medication he is always thirsty and (c) Glycine (d) GABA
frequently has to urinate. The drug he is
most likely to be taking is 81. Concerning the proposed mechanisms of
(a) Clonazepam (b) Clozapine action of antidepressant drugs, which one
of the following statements is accurate?
(c) Haloperidol (d) Lithium
(e) Trifluoperazine (a) Bupropion is an effective inhibitor of NE and
5-HT transporters
77. A young male patient diagnosed as (b) Chronic treatment with an antidepressant often
schizophrenic develops severe muscle leads to the up-regulation of adrenoceptors
cramps with torticollis a short time after
(c) Elevation in amine metabolites in cere-
drug therapy is intiated with haloperidol.
brospinal fluid is characteristic of most
The best course of action would be to
depressed patients prior to drug therapy
(a) Add clozapine to the drug regimen
(d) MAO inhibitors used as antidepressants
(b) Discontinue haloperidol and observe the selectively decrease the metabolism of
patient norepinephrine
DRUGS ACTING ON CENTRAL NERVOUS SYSTEM 37

(e) The acute effect of most tricyclics is to block 87. Established clinical uses of this drug
the neuronal reuptake of both norepinephrine include enuresis and chronic pain
and serotonin in the CNS (a) Bupropion (b) Fluvoxamine
82. Which one of the following effects is (c) Imipramine (d) Phenelzine
unlikely to occur during treatment with (e) Selegiline
amitriptyline?
88. Which one of the following drugs is most
(a) Alpha adrenoceptor blockade likely to increase plasma levels of
(b) Elevation of the seizure threshold alprazolam, the ophyline, and warfarin
(c) Mydriasis (a) Desipramine (b) Fluvoxamine
(d) Sedation (c) Imipramine (d) Nefazodone
(e) Urinary retention (e) Venlafaxine
83. A 54-year-old male patient was using 89. Which one of the following actions of
fluoxetine for depression but decided to opioid analgesics is mediated via
stop taking the drug. When questioned, activation of kappa receptors?
he said that it affected his sexual
(a) Cerebral Vascular dilation
performance and that “he wasn’t getting
(b) Decreased uterine tone
any younger.” You notice that he is a used
of tobacco products. If you decide to (c) Euphoria
reinstitute drug therapy in this patient, (d) Sedation
the best choice would be (e) Psychologic dependence
(a) Amoxapine (b) Bupropion 90. ______ antagonists are known to
(c) Imipramine (d) Sertraline attenuate some of the actions of alcohol.
(e) Venlafaxine (a) GABAA (b) GABAB
84. Regarding maprotiline, which one of the (c) NMDA (d) Glycine
following statements is accurate?
91. Which one of the following statements
(a) Blocks serotonin reuptake selectively about propoxyphene is accurate?
(b) Cause hypertension (a) Analgesia equivalent to oxycodone
(c) Raises the seizure threshold (b) Antagonist at mu receptors
(d) Sedation occurs commonly (c) Causes dose-limiting diarrhea
(e) Has a tricyclic structure (d) Highly effective cough suppressant
85. Which one of the following drugs is most (e) Seizures have occurred in overdose
likely to be of value in obsessive 92. A young male patient is brought to the
compulsive disorders (OCD)? emergency room of a hospital suffering
(a) Amitriptyline (b) Bupropion from an overdose of cocaine following
(c) Clomipramine (d) Desipramine intravenous administration. His symp-
toms are unlikely to include
(e) Mirtazapine
(a) Agitation (b) Bradycardia
86. Compared with other antidepressant
(c) Hyperthermia (d) Myocardial infarct
drugs, mirtazapine has the distinctive
(e) Seizures
ability to act as an antagonist of
(a) Alpha2 adrenoceptors 93. Which one of the following statements
(b) Beta adrenoceptors about hallucinogens is accurate?
(c) D2 receptors (a) Mescaline and related hallucinogens are
thought to exert their CNS actions through
(d) NE transporters
dopaminergic systems in the brain
(e) 5-HT transporters
38 MCQs IN PHARMACOLOGY

(b) Teratogenic effects are known to occur with (c) Hyperuricemia


the use of LSD during pregnancy (d) Liver damage
(c) Scopolamine is unique among hallucinogens (e) Severe cardiac arrhythmia
in that animals will self-administer it
98. Alcohol drinking is associated with which
(d) Dilated pupils, tachycardia, tremor and
of the following changes in serum lipid
increased alertness are characteristic effects
concentrations?
of psilocybin
(e) Phencyclidine can be anticipated to cause (a) Decreased HDL cholesterol
dry mouth and urinary retention (b) Decreased IDL cholesterol
(c) Decreased VLDL cholesterol
94. Which one of the following signs or symp-
toms is likely to occur with marijuana? (d) Increased LDL cholesterol
(e) Increased triglyceride
(a) Bradycardia
(b) Conjunctival reddening 99. A patient suffering from a depressive
(c) Hypertension disorder is being treted with imipramine.
If he uses diphenhydramine for allergic
(d) Increased psychomotor performance
rhinitis, a drug interaction is likely to
(e) Mydriasis
occur because
95. This agent has sedative and amnestic (a) Diphehydramine inhibits imipramine
properties. Small doses added to metabolism
alcoholic beverages are not readily (b) Both drugs block reuptake of norepinephrine
detected by taste and have been used in released from sympathetic nerve endings
“date rape” attacks. The drug is
(c) Imipramine inhibits the metabolism of
chemically related to a brain inhibitory
diphenydramine
neurotransmitter. Which one of the
following most closely resembles the (d) Both drugs block muscarinic receptors
description given? (e) The drugs compete with each other for renal
elimination
(a) Amyl nitrite
(b) Flunitrazepam 100. If phenelzine is administered to a patient
(c) Gamma-hydroxybutyrate taking fluoxetine, the most likely result
is
(d) Hashish
(e) Metcathinone (a) Antagonism of the antidepressant action of
fluoxetine
96. The patient is started on gemfibrozil. The (b) A decrease in the plasma levels of fluoxetine
major mechanism of action of gemfibrozil
(c) Hypertensive crisis
is
(d) Priapism
(a) Increased excretion of bile acid salts
(e) Agitation, muscle rigidity, hyperthermia,
(b) Increased expression of high-affinity LDL seizures
receptors
(c) Increased lipid hydrolysis by lipoprotein 101. The antihypertensive effects of captopril
lipase can be antagonized (reduced) by
(d) Inhibition of secretion of VLDL by the liver (a) Angiotensin II receptor blockers
(e) Reduction of secretion of HDL by the liver (b) Loop diureties
(c) NSAIDS
97. When used as monotherapy, a major
toxicity of gemfibrozil is increased risk of (d) Sulfonylurea hypoglycemics
(e) Thiazides
(a) Bloating and constipation
(b) Cholelithiasis 102. Which one of the following drugs most
resembles the psychoactive constituent(s)
DRUGS ACTING ON CENTRAL NERVOUS SYSTEM 39

of St. John’s wort in terms of proposed 109. Recreational use of drugs sometimes
mechanism of action? leads to dependence. Which of the
(a) Alprazolam (b) Fluoxetine following is least likely to cause physical
dependence?
(c) Levodopa (d) Methylphenidate
(e) Selegine (a) Alcohol is excitatory in nature
(b) Alcohol has a direct stimulatory effect on
103. The primary site of action of tyramine is sexual responsiveness
(a) Ganglionic receptors (c) Alcohol is teratogenic
(b) Gut and liver catechol–O–methyltransferase (d) Alcohol increases ADA production
(c) Postganglionic sympathetic nerve terminals
110. This agent is currently a first-choice drug
(d) Preganglionic sympathetic nerve terminals in the management of absence seizures
(e) Vascular smooth muscle cell receptors as well as partial, primary generalized,
and tonic-clonic seizures.
104. Which one of the following drugs has
been used in ophthalmology, but causes (a) Carbamazepine (b) Clonazepam
mydriasis and cycloplegia lasting more (c) Ethosuximide (d) Phenytoin
than 24 hours? (e) Valproic acid
(a) Atropine (b) Echothiophate 111. If one patient is taking amitriptyline and
(c) Edrophonium (d) Ephedrine another patient is taking chlorpromazine,
(e) Tropicamide they are both likely to experience
105. The antihypertensive drug most likely to (a) Excessive salivation
aggravate angina pectoris is (b) Extrapyramidal dysfunction
(a) Clonidine (b) Guanethidine (c) Gynecomastia
(c) Hydralazine (d) Methyldopa (d) Increased gastrointestinal motility
(e) Propranolol (e) Postural hypotension

106. After an intravenous bolus injection of 112. The phenothiazines have a variety of
lidocaine, the major factors determining actions at different receptor types.
the initial plasma concentration are However, they do not appear to interact
with receptors for
(a) Dose and clearance
(a) Dopammine (b) Histamine
(b) Dose and apparent volume of distribution
(c) Nicotine (d) Norepinephrine
(c) Apparent volume of distribution and
(e) Muscarine
clearance
(d) Clearance and half-life 113. A psychiatric patient taking medications
(e) Half-life and dose develops a tremor, thyroid enlargement,
and leukocytosis. The drug he is taking
107. Diuretics that increase the delivery of is most likely to be
poorly absorbed solute to the thick
(a) Clomipramine (b) Haloperidol
ascending limb of the nephron include
(c) Imipramine (d) Lithium
(a) Furosemide (b) Indapamide
(e) Sertraline
(c) Mannitol (d) Spironolactone
(e) All of the above 114. The mechanism of action of benzodiaz-
epines is
108. Following is used in methanol poisoning (a) Activation of GABAB receptors
(a) Disulfiram (b) Naltrexone (b) Antagonism of glycine receptors in the spinal
(c) Ethanol (d) None of the above cord
40 MCQs IN PHARMACOLOGY

(c) Blockade of the action of glutamic acid 120. the consumption of shellfish harvested
(d) Increased GABA mediated chloride ion during a “red tide” (due to a large
conductance population of a dinoflagellate species) is
(e) Inhibition of GABA aminotranferase not recommended. This is because the
shellfish are likely to contain
115. A drug that is used in the treatment of
(a) Arsenic (b) Botulinum toxins
parkinsonism and will also attenuate
(c) Cyanide (d) Saxitoxin
reversible extrapyramidal side effects of
neuroleptics is (e) Tetrodotoxin
(a) Amantadine (b) Levodopa 121. A 35-year-old female who has never
(c) Pergolide (d) Selegiline been pregnant suffers each month from
(e) Trihexyphenidyl pain, discomfort, and mood depression
at the time of menses. She may benefit
116. The characteristics of one-daily dosing from the use of this selective inhibitor of
with aminoglycosides compared with the reuptake of serotonin.
conventional dosing protocols (every 6-12
(a) Amitriptyline (b) Bupropion
hours) include
(c) Mirtazapine (d) Paroxetine
(a) Decreased drug uptake into the renal cortex
(e) Trazodone
(b) Higher peak serum drug levels to MIC ratios
(c) Postantibiotic actions 122. Which of the following is a common effect
of muscarinic stimulant drugs?
(d) All of the above
(e) None of the above (a) Decreased peristalsis
(b) Decreased secretion by salivary glands
117. A young mother is breast-feeding her 2-
(c) Hypertension
month-old infant. Which one of the
(d) Inhibition of sweat glands
following drug situations involving the
mother is most likely to be safe for the (e) Miosis
nursing infant? 123. Four stages of general anesthesia are
(a) Doxycycline, for Lyme disease distinctly seen with the use of
(b) Metronidazole, for trichomoniasis (a) Halothane (b) Diethyl ether
(c) Nystatin, for a yeast infection (c) Nitrous oxide (d) Enflurane
(d) Phentermine, used for weight reduction
124 Infusion of phenotolamine into the
(e) Triazolam, used as a sleeping pill
cerebrospinal fluid of an experimental
118. Cocaine intoxication has become a animal will prevent the bood pressure –
common problem in hospital emergency lowering action of
rooms. Which one of the following drugs (a) Clonidine (b) Enalapril
is not likely to be of any value in the (c) Guanethidine (d) Reserpine
management of cocaine overdose?
(e) Trimethaphan
(a) Dantrolene (b) Diazepam
125. A drug suitable for producing a brief (5
(c) Lidocaine (d) Naltrexone
to 15 minute) increase in cardiac vagal
(e) Nitroprusside
tone is
119. Which one of the following agents used (a) Digoxin (b) Edrophonium
in hypertension is a prodrug that is (c) Ergotamine (d) Pralidoxime
converted to its active form in the brain?
(e) Pyridostigmine
(a) Clonidine (b) Doxazosin
126. Propranolol and hydralazine have which
(c) Methyldopa (d) Nitroprusside
of the following effects in common?
(e) Verapamil
DRUGS ACTING ON CENTRAL NERVOUS SYSTEM 41

(a) Decreased cardiac force thought processes, suffers from paranoia,


(b) Decreased cardiac output and “hears voices”. The drug most likely
(c) Decreased mean arterial blood pressure to be responsible for her condition is
(d) Increased systemic vascular resistance (a) Gamma-hydroxybutyrate (GHB)
(e) Tachycardia (b) Hashish
(c) Heroin
127. Toxicities of local anaesthetics do not
(d) Marijuana
include
(e) Methamphetamine
(a) Cardiovascular arrhythmias and collapse
(bupivacaine) 132. Mental retardation, microcephaly, and
(b) Convulsions (lidocaine) underdevelopment of the mid face region
(c) Dizziness, sedation (lidocaine) in an infant is associated with chronic
maternal abuse of
(d) Hypertensive emergencies, strokes (procaine)
(e) Methemoglobinemia (a) Amphetamine (b) Cocaine
(c) Ethanol (d) Mescaline
128. Benzodiazepines are least effective in
(e) Phencyclidine
(a) Alcohol withdrawal syndromes
133. After ingestion of a meal that included
(b) Balanced anesthesia regimens
sardines, cheese, and red wine, a patient
(c) Initial management of phencyclidine taking an antidepressant drug experi-
overdose ences a hypertensive crisis. The drug most
(d) Obsessive – compulsive disorders likely to be responsible is
(e) Social phobias (a) Bupropion (b) Fluoxetine
129. Which one of the following drugs exerts (c) Imipramine (d) Phenelzine
its anticonvulsant effects by blocking (e) Trazodone
sodium channels in neuronal membranes?
134. A woman taking haloperidol develops a
(a) Acetazolamide (b) Carbamazepine spectrum of adverse effects that include
(c) Diazepam (d) Gabapentin the amenorrheagalactorrhea syndrome
(e) Vigabatrin and extrapyramidal dysfunction,
including bradykinesia, muscle rigidity,
130. Regarding the pharmacodynamic actions
and tremor at rest. Her psychiatrist
of local anaesthetics, which one of the
prescribes a newer antipsychotic drug
following statements is most accurate?
that improves both positive and negative
(a) All local anaesthetics with ester bonds are symptoms of schizopherenia with few of
vasodilators the side effects that result from dopamine
(b) Amides cause a high incidence of receptor blockade. Since weekly blood
hypersensitivity reactions tests are not deemed necessary the drug
(c) Protonated forms of such drugs readily prescribed by the psychiatrist is probably
penetrate bio-membranes (a) Bupropion (b) Clozapine
(d) The ionized forms of local anaesthetics cause (c) Nefazodone (d) Olanzapine
a use-dependent blockade of sodium ion (e) Sertraline
channels
(e) Type A alpha nerve fibers are highly sensitive 135. A 24-year-old man with a history of
to blockade partial seizures has been treated with
standard anticonvulsants for several
131. A patient is brought to the emergency years. He is currently taking valproic acid,
room suffering from an overdose of an which is not fully effective and his neu-
illicit drug. She is agitated, has disordered rologist prescribes a new drug approved
42 MCQs IN PHARMACOLOGY

for adjunctive use in partial seizures. (a) Clopidogrel (b) Enoxaparin


Unfortunately, the patient develops toxic (c) Heparin (d) Ticlopidine
epidermal necrolysis. The new drug pre- (e) Warfarin
scribed was
141. Following general anesthetic is not
(a) Felbamate (b) Gabapentin
metabolized
(c) Lamotrigine (d) Tiagabine
(a) Halothane (b) Ether
(e) Vigabatrin
(c) Isoflurane (d) Nitrous oxide
136. The introduction of this drug may
represent a novel approach to the 142. Which of the following statements
treatment of major depressive disorders concerning morphine and hydromorphone
since it appears to act as an antagonist is true?
at alpha2 adrenoceptors in the CNS (a) Hydromorphone is a more effective analgesic
(a) Amoxapine (b) Bupropion because it has a smaller ED50 than morphine.
(c) Citalopram (d) Mirtazapine (b) Morphine and hydromorphone are equally
potent because they have the same Emax
(e) Paroxetine
(c) Morphine has a greater ED50 and is thus a
137. Which one of the following pairs of drug: less effective analgesic than hydromorphone
indication is accurate? (d) Hydromorphone is a more potent analgesic
(a) Amphetamine:Alzheimer’s dementia because it has a greater Emax than morphine
(b) Bupropion:Acute anxiety (e) Hydromorphone has a smaller ED50 and is
(c) Fluoxetine:Insomnia thus a more potent analgesic than morphine
(d) Ropinirole:Parkinson’s disease 143. Meperidine is classified as a
(e) Trazodone:Attention deficit disorder
(a) Weak acid (b) Salt
138. The dose of this immunosuppressive (c) Weak base
prodrug must be significantly reduced in
144. Assuming that meperidine is absorbed
patients who are also taking the xanthine
after oral administration and that a large
oxidase inhibitor allopurinol.
percentage of the dose is excreted
(a) Azathioprine unchanged, the effect of alkalinization of
(b) Cyclosporine the urine will increase its
(c) Hydroxychloroquine (a) Duration of action
(d) Methotrexate (b) Rate of excretion
(e) Tacrolimum (c) Ionization in the glomerular filtrate for growth
139. Which one of the following drugs is most 145. The appropriate chemical classification
likely to cause hypoglycemia when used for meperidine is
as monotherapy in the treatment of type
(a) Phenylpropylamines
2 diabetes?
(b) Piperazines
(a) Acarbose (b) Glipizide
(c) 4-phenylpiperidines
(c) Metformin (d) Miglitol
(e) Rosiglitazone 146. Which of the following neuromuscular
blocking agents can cause muscarinic
140. Anticoagulation is needed immediately in responses such as bradycardia and
a patient with pulmonary embolism. increased glandular secretions?
Since there is some concern about a
(a) Tubocurarine (b) Succinycholine
possible drug-induced thrombocytopenia,
the most appropriate drug for parenteral (c) Pancuronium (d) Decamethonium
administration in this patient is (e) Gallamine
DRUGS ACTING ON CENTRAL NERVOUS SYSTEM 43

147. Which of the following drugs is a volatile 152. Which of the following agents would not
substance that is administered by be an alternative to Phenobarbital in the
inhalation? treatment of partial seizures?
(a) Thiopental (b) Halothane (a) Trimethadione (b) Gabapentin
(c) Alprazolam (d) Buspirone (c) Felbamate (d) Lamotrigine
(e) Phenytoin (e) None of the above

148. The brief duration of action of an ultra- 153. A 32-year-old forklift operator with a
short acting barbiturate is due to a past history of cardiac arrhythmias is
suffering from seasonal rhinitis. Which of
(a) Slow rate of metabolism in the liver
the following choices is the best recom-
(b) Low lipid solubility, resulting in a minimal mendation for this patient?
concentration in the brain
(a) Diphenhydramine (b) Meclizine
(c) High degree of binding to plasma proteins
(c) Astemizole (d) Fexofenadine
(d) Rapid rate of redistribution from the brain
due to its high liposolubility (e) Famotidine
(e) Slow rate of excretion by the kidneys 154. Which of the following diuretics is most
similar in chemical structure to the
149. Which of the following mechanisms of antihypertensive agent diazoxide?
action most likely contributes to the
treatment of parkinsonism? (a) Furosemide (b) Spironolactone
(c) Mannitol (d) Acetazolamide
(a) The direct-acting dopaminergic agonist
amantadine mimics the activity of striatal (e) Chlorothiazide
dopamine 155. Following is an example of ideal anesthetic
(b) The antimuscarinic activity of dophen-
(a) Ether (b) Halothane
hydramine contributes to the restoration of
(c) Thiopental (d) None of the above
striatal dopaminergic-cholinergic neuro-
transmitter balance 156. An action common to most general
(c) Striatal H1 – receptors are blocked by the anesthetic
antihistaminic trihexyphenidyl (a) Increase in the cellular threshold of firing
(d) The ergoline bromocriptine stimulates the (b) Potentiation of spontaneous and evoked
release of striatal dopamine from intact activity of neurons
terminals. (c) Inhibition of effects of glutamate
(e) One of the above (d) None of the abvoe
150. All of the following adverse effects are 157. An unconscious patient is brought to the
associated with the use of levodopa emergency department with a history of
except an unknown drug overdose. Which of the
(a) Sialorrhea following actions should the physician
(b) Orthostatic hypotension perform?
(c) Delusions, confusion, and depression (a) Administer 50ml of 50% dextrose, thiamine
(d) Dyskinesia and dystonia 100 mg IV push, and naloxone 0.8 mg IV
push, and naloxone 0.8 mg IV push
(e) Livedo reticularis
(b) Protect the patient’s airway and ensure that
151. The activity of which of the following vital signs are stable
drugs is dependent on a p-phenyl – N – (c) Perform gastric lavage
alkylpiperidine moiety?
(d) Order the following laboratory tests: CBC,
(a) Phenobarbital (b) Chlorpromazine electrolytes, and a toxicology screen
(c) Diazepam (d) Imipramine (e) All of the above
(e) Meperidine
44 MCQs IN PHARMACOLOGY

158. Ethyl alcohol (EtOH) is administered to (a) Amlodipine 5 mg every day


patients who have ingested either (b) Atenolol 25 mg every day
ethylene glycol or methanol because (c) Benazepril 10 mg every day
EtOH
(d) Hydrochlorothiazide 25 mg every day
(a) To call EMS and have the child taken to the
(e) Methyldopa 250 mg three times a day
hospital emergency department
(b) Administer 1 g/kg of activated charcoal with 164. Which of the following benzodiazepines
sorbitol is expected to cause the least amount of
(c) Administer syrup of ipecac 15 ml by mouth adverse effects in the elderly?
to induce vomiting (a) Chlordiazepoxide (b) Diazepam
(d) Suggest that the child receive emergency (c) Fluazepam (d) Oxazepam
hemodialysis (e) Temazepam
(e) Suggest that the child receive acid diuresis
with ammonium chloride 165. The use of morphine in the patient who
has had a myocardial infarction (MI) cen-
159. Parenteral calcium is used as an antidote ters around three distinct pharmacologic
for which of the following situations? properties. Which of the following choices
(a) Verapamil overdoses includes these properties?
(b) Hyperkalemia (a) Relief of pain, relief of anxiety, and increased
(c) Cocaine intoxication oxygen supply
(d) Verapamil overdoses and hyperkalemia (b) Relief of anxiety, after load reduction,
increased preload
160. An overdose victim presents to the
(c) Relief of anxiety, preload reduction, and relief
emergency department with an elevated
of pain
heart rate, decreased blood pressure,
dilated pupils, and lethargy, upon arrival (d) Vagomimetic effect, relief of anxiety,
to the ICU, she has a generalized tonic- respiratory depression
clonic seizure that is treated with IV (e) Bradycardia, preload reduction, and
diazepam and forsphenytoin. Which of increased after load
the following is the most likely intoxicant?
166. Which of the following agents represents
(a) Ethyl alcohol (b) Methanol a relatively new class of drugs used in
(c) Acetaminophen (d) Oxycodone treating hypertension?
(e) Amitriptyline (a) Trandolapril (b) Carvedilol
161. Opthalmic agents contriaindicated in (c) Lorsartan (d) Moexipril
glaucoma patients include which of the (e) Nitrendipine
following substances?
167. Reflex tachycardia, headache, and
(a) Antioxidants (b) Antipuritics postural hypotension are adverse effects
(c) Decongestants (d) Emollients that limit the use of which of the following
antihypertensive agents?
162. Which of the following drugs is excepted
to cause anticholinergic adverse effects (a) Prazosin (b) Captopril
in the elderly? (c) Methyldopa (d) Guanethidine
(a) Propoxyphene (b) Ciprofloxacin (e) Hydralazine
(c) Amitriptyline (d) Propranolol
168. Following is inverse agonist of
(e) Cimetidine benzodiazepine receptors
163. Which of the following antihypertensive (a) Flumazenil (b) β-Carbolines
agents should be avoided in elderly (c) Chlordiazepoxide (d) Glutethimide
patients?
DRUGS ACTING ON CENTRAL NERVOUS SYSTEM 45

169. Match the adverse effects with the 173. Which anticonvulsant drug requires
antihypertensive agent that is most likely therapeutic monitoring of Phenobarbital
to cause them. serum levels as well as it’s own serum
(a) Trandolapril (b) Methyldopa levels?
(c) Nitroprusside (d) Terazosin (a) Phenytoin (b) Primidone
(e) Propranolol (c) Clonazepam (d) Ethotoin
(1) Thiocyanate intoxication, hypotension, (e) Carbamazepine
and convulsions 174. Zolpidem belongs to following class of
(2) Bradycardia, bronchospasm, and drugs
cardiac decompensation
(a) Benzodiazepines
(3) Postural, hypotension, fever, and a
(b) Carbamates
positive Coombs’ test
(c) Imidazopyridines
(4) First–dose syncope, postural hypo-
tension, and palpitations (d) None of the above

170. Match each description of a β -blocker 175. What are the most common adverse
with the most appropriate β -adrener- effects of anticonvulsive drugs?
gicblocking agent. (a) headache and dizziness
(a) Esmolol (b) Labetalol (b) Gastrointestinal symptoms
(c) Bisoprolol (d) Nadolol (c) Alternation of cognition and mentation
(e) Pindolol (d) Adverse effects on appetite and body weight
(1) A β-blocker with intrinsic sympathomi- (e) All of the above
metic activity 176. What are the important elements in
(2) A β-blocker that also blocks a-adrenergic considering treatment strategies for
receptors Parkinson’s patients?
(3) A β-blocker with an ultrashort duration (a) Age
of action
(b) Clinical presentation of disease
(4) A β-blocker with a long duration of
(c) Sex
action and nonselective blocking activity
(d) Etiology of the disease
(5) A b-blokcer with relative cardioselective
blocking activity (e) All of the above

171. Phenytoin is effective for the treatment 177. Which of the following have liver failure in
of all of the following types of seizures their side effect profile and how often the
except liver function tests should be monitored?
(a) Ropinirole
(a) Generalized tonic-clonic
(b) Pramipexole
(b) Simple partial
(c) Levodopa/carbidopa
(c) Complex partial
(d) Tolcapone
(d) Absence
(e) Both (a) and (b)
(e) Grand mal
178. Which of the following agents should not
172. Which of the following anticonvulsants
be used concurrently with levodopa?
is contraindicated in patients with a
history of hypersensitivity to tricyclic (a) Diphenhydramine
antidepressants? (b) Benztropine
(a) Phenytoin (b) Ethosuximide (c) Amantadine
(c) Acetazolamide (d) Carbamazepine (d) Monoamine oxidase (MAO) inhibitors
(e) Phenobarbital (e) Carbidopa
46 MCQs IN PHARMACOLOGY

179. Amantadine has which of the following (a) Donepezil (Aricept)


advantages over levodopa? (b) Trihexyphenidyl (Artane)
(a) More rapid relief of symptoms (c) Diphenhydramin (Benadryl)
(b) Higher success rate (d) Benztropine (Cogentin)
(c) Better long-term effects
185. Which of the following statements about
180. A patient that has been treated with depression is true?
haloperidol for 3 weeks presents with (a) The incidence of depression is greater in men
muscle stiffness, tremor, and shuffling than in women
gait. This is most likely which type of (b) Approximately 5% of institutionalized elders
extrapyramidal side effect? develop depression
(a) Akathisia (c) Depression has no genetic link
(b) Tardive dyskinesia (d) Depression is diagnosed using the DSM-IV
(c) Pseudoparkinsonism criteria
(d) Acute dystonia
186. A patient with major depression should
181. The atypical antipsychotics differ from the receive antidepressant therapy for at
typical agents in various ways that define least
them as atypical. Which of the following (a) 2 weeks (b) 6 weeks
is not a defining property of the atypical (c) 2 months (d) 6 months
antipsychotics?
187. Which of the following patients is mot
(a) Sustained hyperprolactinemia
likely to require maintenance antidepres-
(b) Improved efficacy in treating the negative
sant therapy?
symptoms
(a) A 22-year–old woman depressed about the
(c) Lower risk for extrapyramidal symptoms (EPS)
loss of a parent
(d) Greater serotonin receptor blockade than
(b) A 33-year-old man presenting with his second
dopamine blockade
episode of depression
182. Which of the following atypical (c) A67-year-old man experiencing his first
antipsychotics would be the least episode of depression
sedating? (d) A34-year-old woman experiencing
(a) Quetiapine (b) Risperidone postpartum depression
(c) Olanzapine (d) Clozapine
188. Which of the following medications would
183. The typical antipsychotic agents are most likely exacerbate a preexisting
classified into high and low potency seizure disorder?
agents. Which of the following statements (a) Venlafaxine (b) Trazodone
best defines high and low potency? (c) Bupropion (d) Paroxetine
(a) High potency = more weight gain; low
189. A patient receiving astemizole for
potency = weight loss
seasonal allergies is diagnosed with
(b) High potency = less extrapyramidal
major depression. Which of the following
symptoms (EPS); low potency = high EPS
antidepressants would most likely
(c) High potency = high EPS; low potency = low interact with the current regiment?
EPS
(a) Nefazodone (b) Amitriptyline
(d) High potency = high dose; low potency =
(c) Sertraline (d) Venlafaxine
low dose
190. Which of the following medications would
184. Which of the following agents is not used
be considered first-line monotherapy for
to treat extrapyramidal symptoms?
the treatment of bipolar disorder?
DRUGS ACTING ON CENTRAL NERVOUS SYSTEM 47

(a) Gabapentin (b) Lithium conditions limits the effectiveness of


(c) Risperidone (d) Lamotrigine erythropoietin?
(a) A patient’s allergy to erythropoietin
191. Which of the following is the appropriate
therapeutic range for lithium in the (b) Depletion of iron stores, requiring oral or
treatment of acute mania? parenteral supplementation
(a) 0.6–1.0 mEq/L (b) 0.6–1.5 mEq/L (c) The ineffectiveness of erythropoietin, as 30%
of patients do not respond
(c) 0.6–0.8 mEq/L (d) 0.8–1.2 mEq/L
(d) The anemia of chronic renal failure is not
192. Which of the following mood stabilizers due to a lack of er ythropoietin, so
would be most appropriate in a patient erythropoietin will not ameliorate
with liver disease?
198. Morphine stimulates
(a) Lithium (b) Valproic acid
(a) Billary and pancreatic secretions
(c) Carbamazepine (d) None of the above
(b) Nonpropulsive rhythmic contractions of small
193. Ethinamate is _________ derivative intestine of man
(a) Phenothiazine (b) Urethane (c) Propulsive contractions in small intestine of
(c) Piperidine (d) Tertiary alcohol man
(d) Propulsive peristaltic waves in colon
194. Which of the following factos may reduce
lithium concentrations? (e) Human uterus at full term

(a) Dehydration 199. The brain-stem centers affected most


(b) Pregnancy strongly by barbiturates are the
(c) Reduced sodium intake (a) Respiratory centers
(d) Nonsteroidal anti – inflammatory drugs (b) Vasomotor centers
(c) Cardioinhibitory centers
195. Match the description with the appropri-
ate agent. (d) Cardioacceleratory centers
(e) None of the above
(a) Cimetidine (1) Decrease theophylline
clearance 200. Following statement is correct about
(b) Albuterol (2) Has anticholinergic criteria for neurotransmitter
activity with few side
(a) Must be shown to be present in the
effects
presynaptic terminals
(c) Impratropium (3) Has high β2 – adrenergic
(b) Must be released from the presynaptic
selectivity bromide
terminal concomitanly with presynaptic
(d) Epinephrine activity
(e) Atropine (c) When applied experimentally, effects must
196. The diuretic of choice for the initial be identical to the effects of stimulating the
treatment of a patient with either acute or presynaptic pathway
chronic renal failure (ARF, CRF) whose (d) All of the above
creatinine clearance is below 25 ml/min is
201. A drug that produces increased
(a) Hydrochlorothiazide contraction of the sphincter iridis by local
(b) Bumetanide application is a
(c) Furosemide (a) Parasympathimimetic drug
(d) Ethacrynic acid (b) Parasympatholytic drug
197. Erythropoietin is used commonly to treat (c) Sympathomimetic drug
the anemia associated with chronic renal (d) Sympatholytic drug
failure (CRF). Which of the following (e) None of the above
48 MCQs IN PHARMACOLOGY

202. BAL (British Anti-Lewiside is used to (a) Edema


counter the toxic effects of (b) Congestive heart failure
(a) Atropine (b) Mercury (c) Symptoms of anxiety
(c) Morphine (d) Barbiturates (d) Epilepsy
(e) Digitalis (e) Muscle spasms
203. Which of the following should not be ad- 209. Isoetharine is a drug which stimulated
ministered to a patient with myasthenia primarily which of the following
gravis? receptors?
(a) Prostigmine (b) Digitalis (a) Alpha (b) Histamine
(c) Atropine (d) Curare (c) Beta 1 (d) Beta 2
(e) Insulin (e) None of the above
204. The toxicity of methanol is due to its 210. The mechanism explaining the clinical
conversion in the body to picture observed in carbon monoxide
(a) Acetaldehyde poisoning is
(b) Formic acid and formaldehyde (a) Hemolysis of red blood cells
(c) Ethyl alcohol (b) A chemical union of the carbon monoxide
(d) Carbonic acid with the hemoglobin of the red blood cells
(e) Methane (c) Transformation of the carbon monoxide to
carbon dioxide in the blood
205. One of epinephrine’s actions on the heart is
(d) Arrest of oxidation in the tissues by enzyme
(a) Acceleration due to depression of the vagus
interference
(b) Increase in refractory period
(e) None of the above
(c) Depression of the SA node
(d) Increase in the concentration of the enzyme 211. Acetylcholine has both muscarinic and
phosphorylase A nicotinic actions. The muscarinic action
can be blocked by
(e) None of the above
(a) Epinephrine (b) Atropine
206. Epinephrine HCl has little or no effect on
(c) Nicotine (d) Curare
(a) Unbroken skin
(e) Nicotine and curare
(b) Conjunctiva
(c) Precapillary sphincter 212. Which of the following is the fastest acting
anticoagulant?
(d) Nasal mucosa
(e) Pupil (a) Warfarin (Coumadin)
(b) Heparin
207. Epinephrine is often included in the ad-
(c) Ouabain
minstration of local anaesthetics
because it (d) Protamine sulfate
(e) Vitamin K
(a) Enhances analgesic effect
(b) Neutralizes irritant action 213. Phencyclidine is a non-competitive an-
(c) Delays diffusion of the anesthetic from the tagonist of
site of injection (a) Kainate (b) AMPA
(d) Increases diffusion of the anesthetic (c) NMDA (d) All of the above
(e) Increases blood levels of the anesthetic
214. Which of the following would be the drug
208. Paxipam (halazepam) is used primarily oc choice for treatment of hypertensive
to treat crisis?
DRUGS ACTING ON CENTRAL NERVOUS SYSTEM 49

(a) Diazoxide (Hyperstat) 222. Which of the following may increase


(b) Clonidine (Catapres) seizure activity in epileptic patients?
(c) Propranolol (Inderal) (a) Ethotoin (b) Phenobarbital
(d) Guanethidine (Ismelin) (c) Trihexyphenidyl (d) Amantadine
(e) Proserpine (Serpasil) (e) L - Dopa

215. Parkingsonism is probably due to 223. Trazodone (Desyrel) is used primarily as


a (an)
(a) Too little dopamine in the brain
(a) Analgesic (b) Cardiotonic
(b) Too little levodopa in the brain
(c) Tranquilizer (d) Antidepressant
(c) Too little acetylcholine in the brain
(e) Antihypertensive
(d) Too much levodopa in the brain
(e) Too much dopamine in the brain 224. In addition to their marked potency, the
loop diuretics Lasix and Edecrin have an
216. A prominent toxic effect of local anaes- added advantage of
thetics is
(a) Being completely free of side effects
(a) CNS stimulation (b) CNS depression (b) Causing hypokalemia
(c) Tachycardia (d) Local ischemia (c) Not causing any electrolyte imbalance
(e) None of the above (d) Requiring only once weekly dosing
217. Which of the following could be used as (e) Being effective when given orally
an antidote for curare poisoning?
225. Neuronal death in response to high
(a) Neostigmine (b) Atropine glutamate concentration is mediated by
(c) Homatropine (d) Hexamethonium (a) Kainate (b) AMPA
(e) None of the above (c) NMDA (d) All of the above
218. Cigarette smoking increases the side 226. A common side effect of ephedrine is
effects of
(a) Rashes (b) Nervousness
(a) Narcotic analgesics (c) Blood dyscrasia (d) Drowsiness
(b) Analeptics (e) Ulcers
(c) Antidepressants
227. Thiazide diuretics may produce an
(d) Oral contraceptives increase in blood levels of uric acid and
(e) Cardiac glycosides
(a) Potassium (b) BUN
219. Fluoxetine is a________ transport blocker (c) Urea (d) Lithium
(a) Serotonin (b) Norepinephrine (e) Glucose
(c) Dopamine (d) GABA 228. A group of drugs used widely to treat
mild hypertension is
220. Vivactil (protriptyline) is used mainly as
a (an) (a) Synaptic blockers
(a) Analgesic (b) Muscle relaxant (b) Diuretics
(c) Cardiotonic (d) Antidepressant (c) Ganglionic blockers
(e) Antispasmodic (d) MAO inhibitors
(e) None of the above
221. Which is the drug of choice for trigeminal
neuralgia? 229. What is the advantage of levodopa over
dopamine in treating parkinsonism?
(a) Carbamazepine (b) Phenytoin
(a) It is more stable
(c) Flurazepam (d) Diazepam
(b) It is less toxic
(e) Trimethadione
(c) It is more readily available
50 MCQs IN PHARMACOLOGY

(d) It is considerably less expensive (c) It has additional direct agonistic action on
(e) It is more effective nicotinc receptors at the muscle end plate
(d) It penetrates blood-brain barrier
230. In an anaesthetized dog, repeated
intravenous injection of ephedrine shows 237. The mechanism by which neostigmine
the phenomenon of improves contraction of myasthenic
(a) Anaphylaxis (b) Tachyphylaxis muscle involves
(c) Idiosyncrasy (d) Drug resistance (a) Repetitive binding of the acetylcholine
231. The cotransmitter may ser ve the molecules to the same receptors at the muscle
following function/functions end plate
(a) Regulate the release of the primary transmitter (b) Diffusion of acetylcholine released from
from the nerve ending motor ner ve endings to a wider area
activating neighbouring receptors
(b) Alter postjunctional action of the primary
transmitter (c) Activation of motor end-plate receptors by
(c) Itself act as an alternative transmitter neostigmine molecules themselves
(d) All of the above (d) All of the above

232. Pseudocholinesterase differs from the 238. Symptoms of schizophrenia are produced
cholinesterase in that by following agents, except
(a) It does not hydrolyse acetylcholine (a) Amphetamine (b) Apomorphine
(b) It hydrolyses acetylcholine at a slower rate (c) Bromocriptine (d) Reserpine
(c) It is more susceptible to inhibition by
239. Select the anticholinesterase drug that is
physostigmine
being used to afford symptomatic
(d) It is the only form of circulating cholinesterase improvement in Alzheimer’s disease
233. Acetylcholine has no therapeutic appli- (a) Echothiophate (b) Tacrine
cation because
(c) Demecarium (d) Ambenonium
(a) None of its actions are beneficial in any
condition 240. Pilocarpine reduces intraocular tension in
open angle glaucoma by
(b) Its effects are transient
(c) It produces wide spread actions affecting (a) Contracting sphincter pupillae
many organs (b) Increasing tone of ciliary muscle
(d) Both ‘B’ and ‘C’ are correct (c) Reducing aqueous formation
234. Pilocarpine is used in (d) Enhancing u/veo-scleral outflow
(a) Glaucoma (b) Paralytic ileus 241. Timolol eye drops are preferred over
(c) Urinary retention (d) All of the above pilocarpine eye drops by many glaucoma
patients because
235. Which of the following inhibitors binds
only to the anionic site of the choline- (a) Timolol is more effective than pilocarpine
sterase enzyme ? (b) Timolol acts by enhancing uveo-scleral
(a) Neostigmine (b) Physostigmine outflow
(c) Edrophonium (d) Dyflos (c) Timolol produces less ocular side effects
(d) There are no contraindications to timolol
236. Neostigmine is preferred over physostig-
mine for treating myasthenia gravis be- 242. Select the longer acting ocular beta
cause blocker
(a) It is better absorbed orally (a) Timolol (b) Betaxolol
(b) It has longer duration of action (c) Cartilol (d) Levobunolol
DRUGS ACTING ON CENTRAL NERVOUS SYSTEM 51

243. Which of the following is a prodrug of 250. Local anaesthetics block nerve conduction
adrenaline used topically in glaucoma ? by
(a) Phenylephrine (a) Blocking all cation channels in the neuronal
(b) Dipivefrine membrane
(c) Phenylpropanolamine (b) Hyperpolarizing the neuronal membrane
(d) Dorzolamide (c) Interfering with depolarization of the
neuronal membrane
244. Which of the following diuretics is most (d) Both (b) and (c)
effective in acute congestive glaucoma ?
251. Sensitivity of a nerve fibre to blockade
(a) Indapamide (b) Amiloride by lidocaine depends on
(c) Mannitol (d) Furosemide
(a) Whether the fibre is sensory or motor
245. Neostigmine is beneficial in cobra en- (b) Whether the fibre is myelinated or
venomation because nonmyelinated
(a) It binds to and inactivates cobra toxin (c) Internodal distances in the fibre
(b) It reverses coma due to cobra toxin (d) Both (b) and (c)
(c) It counteracts the cardio-depressant action 252. Which sensation is blocked first by low
of cobra toxin concentrations of a local anaesthetic ?
(d) It antagonizes the paralyzing action of cobra (a) Temperature (b) Pain
toxin
(c) Touch (d) Deep pressure
246. Which is the most important drug in the 253. Following are the MAO-inhibitors, except
treatment of organophosphate poisoning ?
(a) Tranylcypromine (b) Nomifensine
(a) Atropine sulfate (b) Pralidoxime
(c) Moclobemide (d) Isocarboxid
(c) Diazepam (d) Adrenaline
254. The local anaethetic with the longest
247. Which of the following diseases is wors- duration of action is
ened by propranolol ?
(a) Procaine (b) Chloroproacine
(a) Glaucoma (c) Lidocaine (d) Dibucaine
(b) Raynaud’s disease
255. Which of the following is a poor surface
(c) Benign prostatic hypertrophy
anaesthetic ?
(d) Parkinsonism
(a) Procaine (b) Lidocaine
248. Diazepam is used as a muscle relaxant for (c) Tetracaine (d) Benoxinate
(a) Deep intra-abdominal operation
256. The local anaesthetic having high car-
(b) Trachial intubation diotoxic and arrhythmogenic potential is
(c) Tetanus (a) Lidocaine (b) Procaine
(d) Diagonosis of myasthenia gravis (c) Bupivacine (d) Chloroprocaine
249. The local anaesthetics having amide 257. Low concentration of bupivacaine is
linkage differ from those having ester preferred for spinal/epidural obstetric
linkage in that the amide linked local analgesia because
anaesthetics
(a) It has a longer duration of action
(a) Are not surface anaesthetics (b) It can produce sensory blockade without
(b) Have a shorter duration of action paralyzing abdominal muscles
(c) Are degraded in the plasma (c) It distributes more in maternal tissues so that
(d) Do not show cross-sensitivity with ester linked less reaches the foetus
local anaesthetics (d) All of the above are correct
52 MCQs IN PHARMACOLOGY

258. Which of the following local anaesthetics 264. The calcium channel blocker used for
is poorly water soluble PABA derivative prophylaxis of migraine but not for
and primarily used for anorectal lesions, angina pectoris is
wounds and ulcers ? (a) Verapamil (b) Diltiazem
(a) Benzocaine (b) Dibucaine (c) Flunarizine (d) Amlodipine
(c) Procaine (d) Benoxinate
265. Select the nonapeptide which can be
259. Oxethazaine is used for anaesthetizing generated from plasma globulin by
gastric mucosa because snake venom enzymes and causes fall in
BP and intense pain when applied to
(a) It is not absorbed from the gastrointestinal
blister base
tract
(b) It remains largely unionized in acidic medium (a) Kallidin (b) Bradykinin
(c) It is highly ionized in acidic medium (c) Angiotensin II (d) Angiotensin III
(d) It produces no systemic effects even at high 266. Captopril produces greater fall in blood
doses pressure in
260. In which of the following techiniques the (a) Diuretic treated patients
concentration of the local anaesthetic (b) Patients having low plasma rennin activity
required is the lowest ? (c) Sodium replete normotensive individuals
(a) Infiltration anaesthesia (d) Untreated CHF patients
(b) Field block anaesthesia
267. Enalapril differs from captopril in that
(c) Nerve block anaesthesia
(a) It blocks angiotensin II receptors
(d) Spinal anaesthesia
(b) It does not produce cough as a side effect
261. The segmental level of spinal anaesthesia (c) It is less liable to cause abrupt first dose
depends on hypotension
(a) Volume of the local anaesthetic injected (d) It has a shorter duration of action
(b) Specific gravity of the local anaesthetic 268. Which of the following drugs increases
solution cardiac output in congestive heart failure
(c) Posture of the patient without having any direct myocardial
(d) All of the above action ?
262. Spinal anaesthesia is not suitable for (a) Digoxin (b) Captopril
(c) Amrinone (d) Dobutamine
(a) Vaginal delivery
(b) Lower segment caesarian section 269. Which of the following statements most
(c) Prostatectomy closely describes the current status of
(d) Operations on mentally ill patients angiotensin converting enzyme inhibitors
in congestive heart failure ?
263. Epidural anaesthesia differs from spinal (a) They are the first choice drugs unless
anaesthesia in that contraindicated
(a) Epidural anaesthesia produces less (b) They are used when diuretics alone fail
cardiovascular complications (c) They are a substitute for digitalis
(b) Headache is more common after epidural (d) They are to be used as adjuncts only in
anaesthesia resistant cases
(c) Blood concentrations of the local anaesthetic
are lower after epidural anaesthesia 270. Following is a MAO-inhibitor and also has
anit-tubercular activity
(d) Greater separation between sensory and
motor blockade can be obtained with (a) Iproniazid (b) Phenelzine
epidural anaesthesia (c) Moclobemide (d) Isocarboxid
DRUGS ACTING ON CENTRAL NERVOUS SYSTEM 53

271. Losartan is a 278. Which of the following is true of nitrous


(a) Selective AT1 receptor antagonist oxide ?
(b) Selective AT2 receptor antagonist (a) It irritates the respiratory mucosa
(c) Nonselective AT1 + AT2 receptor antagonist (b) It has poor analgesic action
(d) AT1 receptor partial agonist (c) It is primarily used as a carrier and adjuvant
to other anaesthetics
272. Losartan differs from analapril in the
(d) It frequently induces post anaesthetic nausea
following respect
and retching
(a) It does not potentiate bradykinin
279. Ether is widely used as a general anaes-
(b) If depresses cardiovascular reflexes
thetic in India, specially in peripheral
(c) It impairs carbohydrate tolerance hospitals because
(d) It does not have fetopathic potentisl
(a) It is nonexplosive
273. Bisphosphonates are indicated in the (b) It is pleasant smelling and nonirritating
following condition (c) It induces anaesthesia rapidly
(a) Organophosphate poisoning (d) It is cheap and can be administered without
(b) Dementia anaesthetic machine
(c) Steven’s Johnson syndrome 280. The factor that effects rate of elimination
(d) Postmenopausal osteoporosis of general anaesthetics is
274. The most likely mechanism by which (a) Pulmonary ventilation
general anaesthetics produce their action (b) Blood flow
is (c) Solubility in blood and tissue
(a) Affecting receptor operated ion channels in (d) All of the above
cerebral neurones
281. Which of the following general anaesthet-
(b) Blocking voltage sensitive Na+ channels in
ics has the most marked uterine relax-
neuronal membrane
ant action ?
(c) Depressing metabolic activity of cerebral
neurons (a) Propofol (b) Halothane
(d) Blocking production of high energy (c) Nitrous oxide (d) Ether
phosphates in the brain 282. The distinguishing feature of enflurane
275. Which of the following is a sign of deep compared to halothane is its
anaesthesia ? (a) Pleasant and nonirritating nature
(a) Appearance of tears in eyes (b) Propensity to precipitate seizures
(b) Resistane to passive inflation of lungs (c) Propensity to cause hypotension
(c) Fall in blood pressure (d) Bronchodilator action
(d) Patient makes swallowing movements 283. The drug/drugs used mainly for induction
276. Which inhalational general anaesthetic is of general anaesthesia is/are
metabolized in the body to a significant (a) Thiopentone sodium
extent (b) Fentanyl + droperidol
(a) Sevoflurane (b) Isoflurane (c) Ketamine
(c) Ether (d) Halothane (d) All of the above
277. Which of the following general anaesthet- 284. The anaesthetic action of thiopentone
ics has poor muscle relaxant action ? sodium is characterized by
(a) Ether (b) Nitrous oxide (a) Good muscle relaxation
(c) Halothane (d) Isoflurane (b) Poor analgesia
54 MCQs IN PHARMACOLOGY

(c) Sensitization of heart to adrenaline 291. Which of the following drugs has been
(d) No postoperative residual CNS depression found to reduce alcohol craving and
chances of resumed heavy drinking by
285. Dissociative anaesthesia is produced by alcoholics after they have undergone a
(a) Ketamine detoxification programme
(b) Fentanyl + droperidol (a) Chlordiazepoxide (b) Chlorpromazine
(c) Propofol (c) Methadone (d) Naltrexone
(d) Both (a) and (b)
292. Following agent potentiates actions of
286. Use of morphine in preanaesthetic GABA
medication (a) Volatile anaesthetics
(a) Is routine except in the presence of (b) Barbiturates
contraindications (c) Propofol
(b) Is restricted to patients being anaesthetized (d) All of the above
with ether
293. Ethanol is used in methanol poisoning
(c) Should be made only in combination with
because it
atropine
(d) Is restricted mostly to patients in pain (a) Antagonises the actions of methanol
preoperatively (b) Stimulates the metabolism of methanol and
reduces its blood level
287. Which of the following drugs is routinely
(c) Inhibits the metabolism of methanol and
used in preanaesthetic medication for
generation of toxic metabolite
prolonged operations
(d) Replenishes the folate stores depleted by
(a) Atropine (b) Morphine methanol
(c) Promethazine (d) Ranitidine
294. The mechanism of action of barbiturates
288. Patients treated with the following drugs differs from that of benzodiazepines in
should be cautioned not to consume that they
alcoholic beverages (a) Do not affect the GABA–benzodiazepine
(a) Mebendazole (b) Metronidazole receptor–chloride channel complex
(c) Methimazole (d) Metamizol (b) Act as inverse agonists at the benzodiaze-
pine receptor
289. Consumption of alcoholic beverages,
even in moderate amounts, is contrain- (c) Increase the frequency of chloride channel
dicated for the following category of opening without affecting its life time
subjects except (d) Have both GABA–facilitatory as well as
GABA–mimetic actions
(a) Epileptics
(b) Patients with history of myocardial infarction 295. Which one of the following processes
(c) Gastroesophageal reflux patients plays the major role in terminating the
action of phenobarbitone
(d) Pregnant women
(a) Billary excretion
290. What is considered to be the safe limit of (b) Renal excretion
daily alcohol consumption by an adult
(c) Hepatic metabolism
man in the absence of contraindications
and interacting drugs (d) Redistribution

(a) 20 – 40 ml of whisky 296. Currently barbiturates are primarily used


(b) 50 – 100 ml of whisky as
(c) 120 – 180 ml of whisky (a) Hypnotic
(d) 200 – 300 ml of whisky (b) Sedative
DRUGS ACTING ON CENTRAL NERVOUS SYSTEM 55

(c) Antiepileptic 304. Phenytoin appears to derive its anticon-


(d) Preanaesthetic medicant vulsant action from

297. Hypnotic benzodiazepines increase the (a) Selective inhibition of high frequency
period of time spent in the following stage neuronal discharges
of sleep (b) Selective inhibition of epileptic focus
(a) Stage II (b) Stage III (c) Selective inhibition T-type Ca2+ current in
brain cells
(c) Stage IV (d) REM stage
(d) Selective enhancement of inhibitor y
298. The primary mechanism of action of transmission in the brain
benzodiazepines is
305. The characterstics feature of phenytoin
(a) Dopamine antagonism
pharmacokinetics is
(b) Adenosine antagonism
(a) High first pass metabolism
(c) Opening of neuronal chloride channels
(b) Nonsaturation kinetics of metabolism
(d) Facilitation of GABA–mediated chloride
influx (c) Capacity limited metabolism saturating at
higher therapeutic concentration range
299. At a single hypnotic dose, the pharma- (d) Extrahepatic metabolism
cokinetics of diazepam is characterized
by 306. Which of the following drugs displaces
plasma protein bound phenytion as well
(a) Slow elimination and little redistribution
as decreases its metabolism
(b) Slow elimination with marked redistribution
(a) Carbamazepine (b) Sodium valproate
(c) Rapid elimination and marked redistribution
(c) Cimetidine (d) Chloramphenicol
(d) Ultra rapid elimination
307. The following antiepileptic drug is also
300. The following drug is used to reverse the
CNS depression produced by diazepam effective in manic-depressive illness

(a) Nikethamide (b) Doxapram (a) Ethosuccimide (b) Primidone


(c) Physostigmine (d) Flumazenil (c) Phenobarbitone (d) Carbamazepine

301. Which of the following is not a CNS 308. The drug of choice for trigeminal neuralgia
depressant but increases the tendency to is
fall asleep at night ? (a) Aspirin (b) Imipramine
(a) Pyridoxine (b) Methaqualone (c) Carbamazepine (d) Valproic acid
(c) Melatonin (d) Ethanol 309. Which of the following is a narrow
302. The barbiturate having higher anticonvul- spectrum antiepileptic drug effective only
sant sedative activity ratio is in absence seizures
(a) Pentobarbitone (b) Phenobarbitone (a) Phenacemide (b) Ethosuccimide
(c) Bultabarbitone (d) Thiopentone (c) Sodium valproate (d) Primidone

303. The most probable mechanism of 310. Sodium valproate has been shown to
anticonvulsant action of phenytoin is (a) Prolong neuronal Na+ channel inactivation
(a) Facilitation of GABA ergic inhibitory (b) Attenuate ‘T’ type Ca2+ current in neurones
transmission (c) Inhibit degradation of GABA by GABA –
(b) Hyperpolarization of neurones transaminase
(c) Interaction with Ca2+ channels to rpmote Ca2+ (d) All of the above
influx
311. Sodium valproate should be used with
(d) Prolongation of voltage sensitive neuronal
caution in young children because they
Na+ channel inactivation
56 MCQs IN PHARMACOLOGY

are particularly at risk of developing the 319. Which of the following is the most suitable
following adverse effect drug for a 6-year-old girl suffering from
(a) Hepatitis (b) Loss of hair absence seizures with occasional
generalized tonic-clonic seizures ?
(c) Anorexia (d) Tremor
(a) Ethosuccimide (b) Sodium valproate
312. 2-bromo-2-chloro-1,1,1 trifluroethane is
(c) Carbamazepine (d) Phenytoin
(a) Halothane (b) Enflurane
320. The preferred drug for status spilepticus
(c) Isoflurane (d) Desflurane
is
313. Clobazam is a benzodiazepine used as (a) Intravenous diazepam
(a) Hypnotic (b) Muscle relaxant (b) Intravenous phenytoin sodium
(c) Anxiolytic (d) Antiepileptic (c) Intramuscular phenobarbitone
314. Which of the following is a recently (d) Rectal paraldehyde
developed antiepileptic drug used mainly 321. The most effective single drug in parkin-
as add-on therapy of refractory partial sonism is
seizures ?
(a) Bromocriptine (b) Selegiline
(a) Viloxazine (b) Lamotrigine
(c) Levodopa (d) Biperiden
(c) Bupropion (d) Clozapine
322. The dopamine D2 receptor has the fol-
315. Gabapentin acts lowing feature.
(a) As GABAA agonist (a) It is excitatory in nature
(b) As precursor of GABA (b) It is negatively coupled to adenlyl cyclase
(c) By enhancing GABA release (c) It is selectively blocked by chlorpromazine
(d) By GABA independent mechanism (d) It is not blocked by metoclopramide
316. Which of the following is a GABA – 323. The usual cardiovascular effect of
transaminase inhibitor ? levodopa is
(a) Gabapentin (b) Vigabatrin (a) Bradycardia due to increased vagal tone
(c) Lamotrigine (d) Clobazam (b) Rise in blood pressure due to increased
317. Which of the following factors indicates noradrenaline content of adrenergic nerve
that withdrawal of the successfully used endings
antiepileptic medication is likely to result (c) Fall in blood pressure due to decrease in
in recurrence of seizures ? sympathetic tone
(a) Childhood epilepsy (d) Both (a) and (b)
(b) Partial seizures 324. Which of the following adverse effects of
(c) Treatment started soon aftrer seizure onset levodopa has a delayed onset and
(d) Absence of EEG abnormality increases in severity with continued
therapy ?
318. An epileptic woman controlled by pheny-
(a) Nausea and vomiting
toin therapy conceives. Which of the fol-
lowing measures is most appropriate ? (b) Postural hypotension
(c) Cardiac arrhythmia
(a) Mecal termination of pregnancy
(d) Abnormal movements
(b) Withdraw phenytoin therapy
(c) Gradually reduce phenytoin dose to the 325. The drug which abolishes the therapeu-
lowest effective level tic effect of levodopa in parkinsonism, but
(d) Stubstitute phenytoin with a combination of not that of levodopacarbidopa combina-
carbamazepine and sodium valproate tion is
DRUGS ACTING ON CENTRAL NERVOUS SYSTEM 57

(a) Metoclopramide (b) Pyridoxine 332. Which of the following drugs is added to
(c) Chlorpromazine (d) Isoniazid levodopa therapy of parkinsonism to
attenuate wearing off effect ?
326. Use of carbidopa along with levodopa in
(a) Selegiline (b) Trihexiphenidyl
the treatment of parkinsonism
(c) Amantadine (d) Any of the above
(a) Inhibits development of involuntar y
movements 333. The following drug is effective in
(b) Minimises ‘on–off’ effect chlorpromazine induced parkinsonism.
(c) Inhibits occurrence of behavioral abnormal (a) Trihexyphenidyl
ities (b) Selegiline
(d) Accentuates nausea and vomiting (c) Bromocriptine
327. Which of the following adverse effects of (d) Levodopa + carbidopa
levodopa is not minimized by combining 334. For majority of patients of parkinsonism
it with carbidopa the standard drug therapy is
(a) Involuntary movements (a) Levodopa
(b) Nausea and vomiting (b) Levodopa + carbidopa
(c) Cardiac arrhythmia (c) Levodopa + trihexiphenidyl
(d) ‘On–off ‘ effect (d) Bromocriptine
328. Though bromocriptine acts directly on 335. Compared to other antipsychotic drugs,
dopamine receptors, it is used in the distinctive feature of penfluridol is
parkinsonism only as a supplement to
levodopa because (a) Very long duration of action
(b) Weak dopamine D2 blocking activity
(a) It has low efficacy
(c) Lack of extrapyramidal side effects
(b) It produces ‘first dose hypotension’
(d) Additional 5–HT2 receptor blocking activity
(c) Used alone, its effective doses produce
intolerable side effects 336. Which of the following adverse effects of
(d) Its therapeutic effect takes long time to develop neuroleptic drugs is positively correlated
to the antipsychotic potency of the
329. Nitrous oxide is different compounds ?
(a) Non-flammable (a) Sedation
(b) Non-irritant (b) Extrapyramidal motor distubances
(c) Potent analgesis (c) Postural hypotension
(d) All of the above (d) Lowering of seizure threshold
330. The antiparkinsonian drug which acts by 337. Chlorpromazine therapy increases the
inhibiting the degradation of dopamine secretion of the following hormone.
in the brain is
(a) Prolactin (b) Gonadotropin
(a) Carbidopa (b) Amantadine
(c) Corticotropin (d) Antidiuretic hormone
(c) Selegiline (d) Bromocriptine
338. What is true of risperidone ?
331. Tolerance to the antiparkinsonian action
develops most rapidly in the case of (a) It is an atypical neuroleptic which produces
few extrapyramidal side effects
(a) Levodopa
(b) It has combined dopamine D2 and 5-HT2
(b) Levodopa + carbidopa receptor blocking activity
(c) Amantadine (c) It is highly sedative
(d) Bromocriptine (d) Both (a) and (b)
58 MCQs IN PHARMACOLOGY

339. Which of the following is a high potency 346. The most important reason for the
antipsychotic drug having minimal unpopularity of nonselective MAO
sedative and autonomic effects and no inhibitors as antidepressants is their
propensity to cause weight gain ?
(a) Low antidepressant efficacy
(a) Chlorpromazine (b) Triflupromazine (b) Organ toxicity
(c) Haloperidol (d) Reserpine (c) Potential to interact with many foods and
340. The psychotic symptoms most benefited drugs
by neuroleptic drugs are (d) Propensity to precipitate hypomania in
(a) Judgement and memory impairment depressed patients
(b) Loss of insight and voltion 347. The antidepressant which selectively
(c) Hallucinations, delusions and aggressive blocks 5-hydroxytryptamine uptake is
(d) Apathy and social withdrawl (a) Desipramine (b) Amoxapine
341. A manic patient has been brought to the (c) Fluoxetine (d) Dothiepin
hospital with nonstop talking, singing, 348. Adaptive changes in brain monoamine
uncontrollable behavior and apparent turnover due to blockade of noradre-
loss of contact with reality. Which of the naline/5-HT reuptake is credited with the
following is the most appropriate drug following effect.
for rapid control of his symptoms ?
(a) Antipsychotic (b) Antianxiety
(a) Lithium carbonate (b) Diazepam
(c) Antiparkinsonian (d) Antidepressant
(c) Haloperidol (d) Chloral hydrate
349. A 65-year-old man was brought to the
342. The drug which should not be used to hospital with complaints of pain in lower
treat neurotic anxiety and tension abdomen and not having passed urine
syndromes despite having antianxiety for 16 hours. The bladder was found to
action is be full. His son informed that he was
(a) Buspirone (b) Chlorpromazine depressed for the last 2 years and only
(c) Diazepam (d) Alprazolam the day before a doctor had given him
some medicine. Which of the following
343. Which of the following is a nonsedative drugs is he most likely to have received ?
anxiolytic ?
(a) Alprazolam (b) Haloperidol
(a) Meprobamate (b) Buspirone (c) Imipramine (d) Trazodone
(c) Hydroxyzine (d) Alprazolam
350. Following analgesics are frequently
344. Which of the following statements in employed as supplemental drugs during
correct about buspirone ? general anesthesia.
(a) It interacts with benzodiazepine receptor as (a) Meperidine (b) Fentanyl
an inverse agonist (c) Alfentanil (d) All of the above
(b) It is a rapidly acting anxiolytic, good for
panic states 351. Prolonged painful erection of penis has
(c) It produces physical dependence and been noted particularly as a side effect
suppresses barbiturate withdrawl syndrome of
(d) It has anxiolytic but no anticonvulsant or (a) Doxepin (b) Trimipramine
muscle relaxant propery (c) Mianserin (d) Trazodone
345. Which of the following selective MAO–B 352. A 30-year old woman suffering from
inhibitor ? endogenous depression improved after
(a) Selegiline (b) Chlorgiline one month of treatment with amitripty-
line. How long the drug should be
(c) Moclobemide (d) Tranylcypromine
continued
DRUGS ACTING ON CENTRAL NERVOUS SYSTEM 59

(a) Selegiline (b) Chlorgiline 360. The drug of choice for hyperkinetic children
(c) Moclobemide (d) Tranylcypromine is

353. Which of the following statements about (a) Methylphenidate (b) Nikethamide
lithum is not correct (c) Caffeine (d) Clonazepam
(a) It has a sedative action in normal individuals 361. The neurotransmitter system in the brain
(b) It controls mania, but takes 1–2 weeks to most affected in Alzheimer’s disease is
produce the effect (a) Glutaminergic (b) Gabaergic
(c) It has prophylactic effect in recurrent unipolar (c) Dopaminergic (d) Cholinergic
depression
(d) It can be combined with tricyclic antidepres- 362. Which of the following drugs is claimed
sants for refractory cases of major depres- to have a therapeutic effect in senile
sion dementia and has adrenergic blocking
activity
354. For therapeutic effect in manic depressive
(a) Digoxin (b) Furosemide
illness, stady-state serum lithium concen-
tration should be maintained between (c) Enalapril (d) Amrinone

(a) 0.2–0.4 mEq/L (b) 0.5–0.8 mEq/L 363. Which of the following drugs improves
(c) 1.0–1.3 mEq/L (d) 1.5–2.5 mEq/L some symptoms in Alzheimer’s dementia
by increasing brain acelylcholine levels
355. Renal excretion of lithium is enhanced by
(a) Pemoline (b) Tacrine
(a) Furosemide (b) Mannitol (c) Nicergoline (d) Piribedil
(c) Indomethacin (d) Both (a) and (b)
364. Pyritinol (pyrithioxine) is used as
356. Which of the following drugs can be used
(a) Analeptic drug (b) Cerebroactive drug
as an alternative to lithium in mania and
bipolar illness (c) Antiepileptic drug (d) Antidepressant drug

(a) Diethyl carbamazine 365. Extract of the following plant has platelet
(b) Haloperidol activating factor antagonistic activity and
is claimed to benefit congnitive disorders
(c) Clomipramine
due to cerebral ischaemia
(d) Carbamazepine
(a) Ginkgo biloba (b) Claviceps purpurea
357. The constellation of side effects consist-
(c) Amanita muscaria (d) Artemisia annua
ing of thirst, polyuria, looseness of stools
and fine tremors is characteristically as- 366. Digitalis increases the force of contraction
sociated with the following psychotropic of ventricles by
drug (a) Increasing the duration of systole
(a) Amitriptyline (b) Buspirone (b) Increasing the rate of contraction without
(c) Lorazepam (d) Lithium carbonate affecting the duration of systole
358. Prolonged lithium therapy can cause (c) Increasing the rate of contraction, but
reducing the duration of systole
(a) Diabetes mellitus (b) Goiter
(d) Increasing both the rate of contraction as
(c) Parkinsonism (d) Gout well as the duration of systole
359. Strychnine produces convulsions by 367. Digitalis induced increase in refractory
(a) Stimulating NMDA receptors period of myocardial fibres is most
(b) Facilitating the excitatory transmitter glutamate consistent and pronounced in the
(c) Blocking the inhibitory transmitter GABA (a) Atria (b) Ventricles
(d) Blocking the inhibitory transmitter glycine (c) A-V node (d) Purkinje fibres
60 MCQs IN PHARMACOLOGY

368. Following general anaesthetic is oil at (c) Depression of A-V conduction


room temperature (d) Enhanced Purkinje fibre automaticity
(a) Propofol (b) Ketamine
375. What is the usual response to digoxin in
(c) Droperidol (d) Diazepam
a patient of atrial fibrillation ?
369. The most important channel of elimina- (a) Restoration of normal sinus rhythm
tion of digoxin is
(b) Conversion of atrial fibrillation to atrial flutter
(a) Glomerular filtration
(c) Increase in atrial fibrillation frequency, but
(b) Tubular secretion decrease in ventricular rate
(c) Hepatic metabolism (d) Decrease in atrial fibrillation frequency, but
(d) Excretion in bile increase in ventricular rate
370. Digitalis is most suitable for treatment of 376. The preferred diuretic for mobilizing
CHF when it is due to
edema fluid in CHF is
(a) Digoxin (b) Furosemide
(a) Polythiazide (b) Furosemide
(c) Enalapril (d) Amrinone
(c) Metolazone (d) Amiloride
371. The dose of digoxin in congestive heart
failure is adjusted by monitoring 377. Infusion of potassium chloride is indicated
in digitalis toxicity when the manifestation(s)
(a) Electrocardiogram is/are
(b) Heart rate and symptoms of CHF
(a) Vomiting, hyperapnoea and visual disturbance
(c) Blood pressure
(b) Pulsus bigeminus with heart rate 110/min
(d) Plasma digoxin levels
in a patient on maintenance digoxin therapy
372. Digoxin affords the following benefit/ (c) Ventricular tachycardia in a child who has
benefits in CHF accidentally ingested 10 digoxin tablets
(a) Restores cardiac compensation and relieves (d) 2:1 A-V block with occasional ventricular
symptoms extrasystoles
(b) Reverses the pathological changes of CHF
378. Which of the following is the most suitable
(c) Prolongs survival of CHF patients
antiarrhythmic drug for counteracting
(d) Both (a) and (b)
ventricular extrasystoles due to digoxin
373. A patient of CHF was treated with toxicity ?
furosemide and digoxin. He became
(a) Lidocaine (b) Quinidine
symptom-free and is stable for the last 3
months with resting heart rate 68/min (c) Verapamil (d) Amiodarone
in sinus rhythm but left ventricular 379. The apparent stimulation of CNS by local
ejection fraction is low. Which of the anesthetic is due to
following lines of action is warranted.
(a) Direct excitatory action
(a) Stop above medication and start an ACE
inhibitor (b) Depression of inhibitory neurons
(b) Continue all medication as before (c) Non specific action
(c) Continue the diuretic but stop digoxin (d) All of the above
(d) Continue digoxin but stop the diuretic 380. The following drug is used for short-term
374. Which of the following actions of digoxin control of emergency heart failure but not
is responsible for beneficial effect in for long-term treatment of congestive
auricular fibrillation ? heart failure
(a) Increased myocardial contractility (a) Digoxin (b) Enalapril
(b) Suppression of SA node (c) Dobutamine (d) Theophylline
DRUGS ACTING ON CENTRAL NERVOUS SYSTEM 61

381. Select the drug which is an ‘inodilator’ (a) It has selective antihypertensive action at
beneficial in refractory congestive heart doses which cause little diuresis
failure (b) It is a more efficacious antihypertensive
(a) Astemizole (b) Amiodarone (c) Its antihypertensive action develops more
(c) Amrinone (d) Amiloride rapidly
(d) All of the above
382. The principal action common to all class I
antiarrhythmic drugs is 389. In the treatment of hypertension the beta
adrenergic blockers have the following
(a) Na+ channel blockade
advantage
(b) K+ channel openine
(a) They have minimal effect on work capacity,
(c) Depression of impulse conduction
sleep quality and libido
(d) Prolongation of effective refractory period
(b) They do not cause postural hypotension
383. Following is an amide linked local anaesthetic (c) Used alone, they have high ceiling
(a) Bupivacaine (b) Procaine antihypertensive efficacy
(c) Tetracaine (d) Proparacaine (d) They can be used in combination with any
other antihypertensive drug
384. Persistent dry cough may occur as a side
effect of the following antihypertensive 390. The following antihypertensive drug has
drug a favourable effect on plasma lipid profile
(a) Enalapril (b) Atenolol (a) Prazosin
(c) Diltiazem (d) Methyldopa (b) Propranolol
(c) Hydrochlorothiazide
385. Loss of taste sensation can be a side effect
(d) Furosemide
of the following antihypertensive drug
(a) Clonidine (b) Captopril 391. The following drug has been found to
improve urine flow in elderly males with
(c) Verapamil (d) Prazosin
benign prostatic hypertrophy
386. Furosemide is to be preferred over (a) Nifedipine (b) Prazosin
hydrochlorothiazide when hypertension
(c) Disopyramide (d) Imipramine
is accompanied by
392. Following is an ester linked local anaesthetic
(a) Asthma
(b) Hyperuricaemia (a) Mepivacaine (b) Ropivacaine
(c) Benzocaine (d) Prilocaine
(c) Diabetes
(d) Congestive heart failure 393. The following antihypertensive is used
topically to treat alopacia areata
387. Thiazide diuretics are the preferred first
(a) Hydralazine (b) Prazosin
line antihypertensives from the following
category of patients (c) Minoxidil (d) Guanethidine

(a) Young hypertensives 394. Diazoxide is an effective hypotensive, but


it is not used in the long-term treatment
(b) Physically and sexually active male
of hypertension because
hypertensives
(a) It impairs glucose tolerance by inhibiting
(c) Elderly obese hypertensives
insulin release
(d) Diabetic hypertensives
(b) It inhibits uric acid excretion
388. Indapamide differs from other diuretics (c) It causes marked Na+ and water reterntion
in that (d) All of the above
62 MCQs IN PHARMACOLOGY

395. The following antihypertensive drug (a) Furosemide (b) Methyldopa


tends to lower plasma rennin activity (c) Propranolol (d) Captopril
(a) Clonidine (b) Hydralazine
401. Secretion of K+ in the late distal tubule
(c) Nifedipine (d) Captopril and collecting ducts of kidney depends
396. Methyldopa lowers BP by on

(a) Inhibiting dopa decarboxylase in adrenergic (a) Intracellular K+ content


nerve endings (b) Unabsorbed Na+ load presented to the distal
(b) Generating α-methyl noradrenaline in brain segment
which reduces sympathetic tone (c) Aldosterone
(c) Generating α-methyl noradrenaline which (d) All of the above
acts as a false transmitter in peripheral
402. Which of the following diuretics is orally
adrenergic nerve endings
active, efficacious in acidosis as well as
(d) Activating vascular dopamine receptors alkalosis, causes diuresis even in moderately
397. Injection of local anaesthetic directly into severe renal failure and has additional
tissue without taking into consideration carbonic anhydrase inhibitory action
the course of cutaneous nerves is (a) Mannitol
(a) Infiltration anesthesia (b) Bendroflumethiazide
(b) Field block anesthesia (c) Mersalyl
(c) Nerve block anesthesia (d) Furosemide
(d) All of the above 403. The following diuretic abolishes the
398. Select the correct statement about corticomedullary osmotic gradient in
combining antihypertensive drugs the kidney

(a) Antihypertensive combinations should (a) Acetazolamide (b) Furosemide


always be preferred over single drugs (c) Benzthiazide (d) Spironolactone
(b) Combinations of antihypertensives with 404. Intravenous furosemide promptly miti-
similar pattern of haemodynamic action are gates dyspnoea in acute left ventricular
superior to those with dissimilar pattern failure by
(c) Only those antihypertensives which act on
(a) Producing bronchodilatation
different regulatory systems maintaining
(b) Causing rapid diuresis and reducing
blood pressure should be combined
circulating blood volume
(d) A diuretic must be included whenever
(c) Increasing venous capacitance and reducing
antihypertensives are combined
cardiac preload
399. Angiotensin converting enzyme inhibitors (d) Stimulating left ventricular contractility
are contraindicated in
405. Parenteral furosemide is an alternative
(a) High rennin hypertensives
diuretic to mannitol in the following
(b) Diabetics condition
(c) Congestive heart failure patients
(a) Pulmonary edema (b) Cirrhotic edema
(d) Pregnant women
(c) Cerebral edema (d) Cardiac edema
400. A woman in the 28th week of pregnancy
406. Though ethacrynic acid is also a high
has been found to have pregnancy in-
ceiling diuretic, it is rarely used compared
duced hypertension with a blood pres-
to furosemide because
sure reading of 150/95 mm Hg. Select
the most appropriate antihypertensive (a) It is more ototoxic
drug for her (b) It causes diarrhoea and gut bleeding
DRUGS ACTING ON CENTRAL NERVOUS SYSTEM 63

(c) Its response increases steeply over a narrow 412. A patient of congestive heart failure was
dose range being treated with furosemide and
(d) All of the above digoxin. He developed urinary tract
infection. Which of the following
+
407. Thiazide diuretics enhance K elimination antimicrobials should not be prescribed
in urine primarily by (a) Ampicillin (b) Gentamicin
+
(a) Inhibiting proximal tubular K reabsorption (c) Norfloxacin (d) Cotrimoxazole

(b) Inhibiting Na+ K+–2C1 cotransport in the
413. Which of the following has a ‘self limiting
ascending limb of of loop of Henle
diuretic’ (action of the drug itself causing
(c) Increasing the availability of Na+ in the distal changes which limit further diuresis)
tubular fluid to exchange with interstitial K+ action ?
(d) Potentiating the action of aldosterone (a) Indapamide (b) Spironolactone
408. The primary site of action of thiazide (c) Theophylline (d) Acetazolamide
diuretics is
414. The most important therapeutic indication
(a) Proximal tubule of acetazolamide is
(b) Ascending limb of loop of Henle (a) Congestive heart failure
(c) Cortical diluting segment (b) Renal insufficiency
(d) Collecting ducts (c) Cirrhosis of liver
409. The most important reason for the (d) Glaucoma
thiazides being only moderately effica- 415. Aldosterone increases Na+ reabsorption
cious diuretics is and K+ excretion in the renal collecting
(a) FSH + LH obtained from urine of menstruating duct cells by
women (a) Inducing synthesis of Na+ K+ ATP ase
(b) LH obtained from urine of pregnant women (b) Inducing synthesis of amiloride sensitive Na+
(c) FSH + LH obtained from urine of menopausal channels
women (c) Translocating Na+ channels from cytosolic
(d) LH obtained from serum of pregnant mare site to luminal membrane
(d) All of the above
410. Combined tablets of thiazide or high
ceiling duretics with potassium chloride 416. Select the diuretic that can cause gynae-
are not recommended because comastia, hirsutism and menstrual distur-
bance as a side effect on long-term use.
(a) Potassium absorbed while diuresis is
occurring is largely excreted out (a) Amiloride (b) Spironolactone
(b) Potassium administered concurrently (c) Metolazone (d) Acetazolamide
diminishes the diuretic action 417. Which of the following is a potassium
(c) Potassium chloride in tablet formulation is retaining diuretic ?
likely to cause gut ulceration (a) Trimethoprim (b) Triamterene
(d) Both (a) and (c) (c) Trimethaphan (d) Trimethadione
411. _______ technique relies on using the 418. Triam
vasculature to bring the local anesthetic
solution to the nerve trunks and endings (a) It has greater natriuretic action
(b) Its K+ retaining action is not dependent on
(a) Spinal anesthesia
presence of aldosterone
(b) Nerve block anesthesia
(c) It acts from the luminal membrane side of
(c) Intravenous regional anesthesia the distal tubular cells
(d) All of the above (d) Both (b) and (c)
64 MCQs IN PHARMACOLOGY

419. Potassium sparing diuretics should not be (b) Produces reflex tachycardia
coadministered with (c) Is contraindicated in patients with renal
(a) Furosemide insufficiency
(b) Hydrochlorothiazide (d) Decreases plasma renin activity
(c) Captopril 426. Diazoxide
(d) Verapamil
(a) Causes marked sodium and water retention
420. The primar y mechanism by which (b) Can be used in hypertensive emergencies
antidiuretic hormone reduces urine
(c) Can produce alopecia
volume is
(d) Lowers blood sugar levels
(a) Decrease in glomerular filtration rate
(b) Decreased renal blood flow 427. Hydroxyethyl starches
(c) Decreased water permeability of descending (a) Are allergenic
limb of loop of Henle (b) Do not cause coagulation disturbances
(d) Increased water permeability of collecting (c) Can precipitate renal failure
duct cells (d) Are hydrolysed by amylase
421. Which of the following tissues is most
428. Normal saline
sensitive to vasopressin ?
(a) Has a long duration of action
(a) Vascular smooth muscle
(b) Is commonly used as a vehicle for
(b) Intestinal smooth muscle
noradrenaline drip
(c) Renal collecting ducts
(c) Is 5% sodium chloride
(d) Uterus
(d) Is a commonly used agent to raise effective
422. Which of the following drugs reduces blood volume in emergencies
urine volume in both pituitary origin as
well as renal diabetes insipidus and is 429. The sympathomimetic amine preferred in
orally active ? shock due to myocardial infarction is

(a) Vasopressin (a) Dopamine (b) Isoprenaline


(b) Hydrochlorothiazide (c) Epinephrine (d) Norepinephrine
(c) Chlorpropamide 430. Phenytoin sodium
(d) Carbmazepine
(a) Depresses ventricular automaticity
423. Alpha methyldopa (b) Decreases AV nodal conduction
(a) Is ineffective when given orally (c) Is given by constant intravenous infusion
(b) Decreases serum prolactin levels (d) Decreases the conduction velocity in the
(c) Is valuable in patients with compromised ventricular fibers
renal function 431. Dale’s vasomotor reversal phenomenon
(d) Causes troublesome postural hypotension is exhibited by
424. Fetal alcohol syndrome is characterized by (a) Adrenaline (b) Noradrenaline
(a) Normal CNS function (c) Isoprenaline (d) Both (a) and (b)
(b) Characteristic cluster of facial abnormalities 432. Phenylephrine instilled in the eye can
(c) Normal growth produce
(d) All of the above (a) Mydriasis but no cycloplegia
425. Prazosin (b) Mydriasis and cycloplegia
(c) Miosis and cycloplegia
(a) Is relatively ineffective in blocking presynaptic
alpha2 receptors (d) Miosis without cycloplegia
DRUGS ACTING ON CENTRAL NERVOUS SYSTEM 65

433. Synthesis of norepinephrine occurs from (a) Rapid hepatic metabolism


(a) Tyramine (b) Tyrptophan (b) Rapid renal clearance
(c) Tyrosine (d) Tetracaine (c) Extensive binding to plasma proteins
434. The drug of choice in anaphylactic shock (d) Redistribution to body tissues outside the CNS
is 443. The drug of choice for myoclonic epilepsy
(a) Isoprenaline (b) Adrenaline (a) Carbamazepine (b) Phenobarbitone
(c) Dobutamine (d) Noradrenaline (c) Phenytoin sodium (d) Valproic acid
435. Phenoxybenzamine is less commonly used 444. Peripheral conversion of levodopa to
to treat chronic essential hypertension,
dopamine is facilitated by
because it
(a) Thiamine (b) Riboflavin
(a) Has a transient action of 2 hours
(c) Pyridoxine (d) Cyanocobalamin
(b) Has to be given parenterally
(c) Also blocks the vasodilatory beta2 adrenergic 445. Which of the following does not produce
receptors hypertensive crisis when coadministered
(d) Causes tachycardia and postural hypotension with levodopa?

436. The anti-cholinesterase with the shortest (a) Phenelzine (b) Selegiline
duration of action is (c) Isocarboxazid (d) Pargyline
(a) Neostigmine (b) Physostigmine 446. Nitrous oxide
(c) Edrophonium (d) Ambenonium
(a) Provides slow induction
437. The agent that produces cycloplegia for (b) Produces adequate muscle relaxation
a short time is (c) By itself is not a potent anaesthetic
(a) Cyclopentolate (b) Homatropine (d) Is a poor analogesic
(c) Tropicamide (d) Atropine
447. Propofol
438. The drug of choice in the treatment of
(a) Enhances the neuromuscular blockade of d-
organophosphorus poisoning is
tubocurarine
(a) Neostigmine (b) Atropine (b) Causes marked postoperative confusion
(c) Pralidoxime (d) Acetylcholine
(c) Can produce involuntary movements during
439. The average rate of ethanol metabolism its use
in one hour in normal adults is between (d) Has a delayed recovery after induction
(a) 1 to 2 ml (b) 8 to 12 ml 448. Neurolept analgesia
(c) 30 to 50 ml (d) 100 to 150 ml
(a) Is used in patients with Parkinson’s disease
440. Following barbiturate is significantly (b) Precludes the use of succinylcholine
excreted as unchanged drug (c) Induces unconsciousness
(a) Phenobarbital (b) Pentobarbital (d) Is useful for minor surgical procedures
(c) Secobarbital (d) All of the above 449. The local anaesthetic with a short dura-
441. Acute opium poisoning is treated by tion of action is
intravenous administration of (a) Lignocaine (b) Procaine
(a) Naloxone (b) Pethidine (c) Tetracaine (d) Mepivacaine
(c) Nalbuphine (d) Noscapine 450. The local anaesthetic with significant
442. Ultrashort duration of effct of thiopentone vasoconstriction effect is
sodium given intravenously is due to (a) Cocaine (b) Procaine
(c) Bupivacaine (d) Tetracaine
66 MCQs IN PHARMACOLOGY

451. The local anaesthetic recommended as an (c) They are strong bases and are highly ionized
antiarrhythmic is at gastric pH
(a) Lignocaine (b) Procaine (d) They are weak acids and are ionized at
(c) Medivacuene (d) Tetracaine gastric pH

452. A H 1 receptor antagonist has limited 458. Zolpidem interacts with


beneficial effect in (a) BZ1 receptors
(a) Combating motion sickness (b) BZ2 receptors
(b) Systemic anaphylaxis (c) Both (a) and
(c) Allergic conjunctivitis (d) None of the above
(d) Reversing extrapyramidal side effects caused
by phenothazines 459. Phenobarbitone reduces the incidence of
severe hyperbilirubinaemia in the
453. The H1 receptor antagonist with high neonates because
anticholinergic activity is
(a) It prevents the synthesis of bilirubin
(a) Diphenhydramine (b) Terfenadine
(b) It conjugates with the bilirubin
(c) Mepyramine (d) Clemastine
(c) It stimulates hepatic glucuronyl tranferase
454. Magnesium sulfate (d) None of the above mechanism
(a) Is used to control eclamptic seizures during
460. The methanol is toxic to optic nerve, it is
pregnancy
because of
(b) Is not given in patients in whom b2 adrenergic
receptor (a) Methanol itself
(c) Is used to arrest post partum hemorrhage (b) Oxidative metabolites of methanol
(d) Can arrest labor even when cervix is 7 cm (c) Conjugated product of methanol
dilated (d) Idiosyncratic reaction
455. Colchicine 461. A patient has resulted in acute
(a) Is used for treatment and prevention of acute hypertensive episode after ingestion of
gouty arthritis certain cheeses or wine. This interaction
(b) Causes cellular proliferation is because he is on
(c) Is an effective analgesic in osteoarthritis (a) Tetracyclines (b) Tranyleypromine
(d) Enhances uric acid excretion (c) Aspirin (d) Digitalis
456. Probenecid 462. Which of the following drugs increases
(a) Increases excretion of uric acid by blocking the metabolism of bishydroxycoumarin
its tubular reabsorption by induction of hepatic microsomal
(b) Is used in combination with salicylates enzymes?
(c) Is useful in gouty patients with nephrolithiasis (a) Digoxin (b) Furosemide
(d) Reduces the plasma concentrations of (c) Enalapril (d) Amrinone
rifampicin
463. Antihistamines, phenothiazines, tricyclic
457. Salicylates and barbiturates are more antidepressants and antiparkinsonian
readily absorbed from the stomach agents have a common property that
because
(a) All produces CNS stimulation
(a) They are weak bases and are ionized at (b) All have adrenergic action
gastric pH
(c) All have some atropine like action
(b) They are weak acids and predominantly non-
ionized in gastric pH (d) All have long plasma half-life
DRUGS ACTING ON CENTRAL NERVOUS SYSTEM 67

464. Regarding the eardiac and therapeutic 469. Hemicholinium modifies Ach metabolism
effect of phenytoin, which of the following by
is true? (a) Interfering with destruction of Ach
(a) It increases the action potential duration in (b) Interfering with relese of Ach
purkinje fibres (c) Interfering with synthesis of Ach
(b) It has a brief duration of action because (d) Promoting release of Ach
of t½ of 2 hours
(c) It is probably the drug of choice in the 470. Action of acetylcholine is terminated by
treatmet of arrhythmias caused by digitalis (a) Diffusion away from the site of action
intoxication (b) Deamination
(d) The drug markedly depresses conduction (c) Hydrolysis
velocity in the A-v node and intraventricular
(d) Temporary adsorption on plasma proteins
conduction system
471. Norepinephrine in the synaptic cleft can
465. Phenytoin
activate presynaptic alpha 2 receptors
(a) Is useful only in the treatment of epilepsy which results in
(b) Has an antiarrhythmic action that is quite (a) Increased release of exocytotic norepinephrine
different from that of quinidine
(b) Displacement of norepinephrine from the
(c) Has antiarrhythmic action against many types storage site
of arrhythmias
(c) Further inhibition of exocytotic release of
(d) Increases excitability of atrial or ventricular norepinephrine
muscle
(d) Increased synthesis of norephinephrine
466. The drug of choice in the treatment of
472. Which of the following barbiturates is
hypertensive emergency is
preferred as sedative agent in a patient
(a) Guanethidine (b) Reserpine having liver disease ?
(c) Diazoxide (d) Furosemide (a) Allobarbitone (b) Cyclobarbitone
467. Which of the following statements (c) Phenobarbitone (d) Pentobarbitone
concerning hypertension is true?
473. One of the way to classify regions of the
(a) It is easier to obtain salt depletion in cortex is
hypertensive patients with thiazide diuretics
(a) By the modality of information processed
than by limitation of salt intake
(b) By the physiological position
(b) I.V. guanethidine is very useful in controlling
hypertensive emergencies (c) By the chemical relationship between cell
types in the major cortical layers
(c) In essential hypertension the excretion of
catecholamine in the urine is abnormally high (d) All of the above
(d) Aldosterone stimulates the secretion of 474. Peak serum level of orally administered
angiotensin leading to certain types of lithium is usually achieved in
hypertension (a) 5 to 10 min (b) 1 to 2 days
468. The duration of action of methacholine is (c) 2 to 4 hours (d) 8 to 12 hours
longer than that of acetylcholine because
475. For induction of anaesthesia nitrous oxide
(a) Acetylcholine is inactivated by active is superior to ether because
transport into schwan cell
(a) Nitrous oxide is not explosive, while ether
(b) Acetylcholine is inactivated by active vapours are flammable in air
transport into nerve cell
(b) Iuduction with nitrous oxide is very rapid
(c) Acetylcholine is metabolized more rapidly while with ether it is slow and unplea sant to
(d) Acetylcholine is excreted more rapidly patient
68 MCQs IN PHARMACOLOGY

(c) Ether is irritant to respiratory tract and may 482. Which of the following neuromuscular
lead to coughing, laryngeal spasm and blocking agent has short duration of
increased mucus secretion while nitrous oxide action
does not (a) Atracurium (b) Mivacurium
(d) All of the above (c) Pancuronium (d) Doxacurium
476. Succinylcholine is preferred over d- 483. Botulinium toxin causes muscle palsy by
tubocurarine for endotrachial intubation, blocking
because of the following reasons except
(a) Nerve action potential
(a) With succinylcholine total paralysis lasts upto (b) Acetylcholine release
4 min. with 50% recovery at about 10 min
(c) Permeability to sodium and potassium
(b) Allergic reactions due to release of histamine
(d) Muscle action potential
are not seen with succinylcholine
(c) In addition to neuromuscular blocking effect 484. Dopamine is preferred over nore-
succinylcholine blocks autonomic ganglia pinephrine in cardiogenic and hypo-
volemic shock because
(d) Bronchospasm is less common with
succinylcholine (a) Better cardiac stimulation
(b) Potent renal vasodilator
477. The difference between autonomic and
somatic nerves are all except (c) Potent cerebral vasodilation
(d) All of the above
(a) Somatic nerves do not contain peripheral
ganglia unlike autonomic nerves 485. What is the mechanism of action of
(b) Motor nerves to skeletal muscles are clonidine
myelinated while postganglionic autonomic (a) α1 antagonist (b) α2 agonist
nerves are unmyelinated (c) α1 agonist (d) β1 agonist
(c) The presynaptic transmitted in both the cases
is different 486. What is true of methylphenidate
(d) None of the above (a) Mild CNS stimulant
(b) Abuse potential as of amphetamine
478. In endothelium nitric oxide is synthezised
(c) Effective in narcolepsy and attention deficit-
from
disorder
(a) Arginine (b) Valine (d) All of the above
(c) Tyrosine (d) All of the above
487. The longest half-life is of which of the
479. Ipratropium bromide is principally following betablockers
effective in (a) Esmolol (b) Nadolol
(a) Asthma (b) Bronchitis (c) Pindolol (d) Acebutolol
(c) COPD (d) All of the above
488. Which of the following is both alpha and
480. What are obidoxine and pralidoxime beta blocker
(a) Acetylcholine reactivators (a) Propranolol (b) Labetalol
(b) Choline esterase reactivators (c) Esmolol (d) Metoprolol
(c) Choline esterase inhibitors 489. The rate limiting enzyme in synthesis of
(d) Cellular poisons melatonin and serotonin is
481. Which of the following cholinergic drugs (a) Tryptophan hydroxylase
is used in Alzheimer’s disese (b) 5 HT N-acetylase
(a) Neostigmine (b) Pyridostigmine (c) Aldehyde dehydrogenase
(c) Tacrine (d) None of the above (d) Aldehyde reductase
DRUGS ACTING ON CENTRAL NERVOUS SYSTEM 69

490. Which of the following is a function of (c) Isoflurane (d) Desflurane


melatonin
499. The barbiturate of choice for induction of
(a) Darkening of skin colour anaesthesis is
(b) Suppression of ovarian function
(a) Pentothal (b) Thiamylal
(c) Regulation of biorhythm
(c) Methohexital (d) Etomidate
(d) All of the above
500. Ketamine injection produces
491. Which neurotransmitter has maximum
(a) Amnesia (b) Analgesia
number of receptor subtypes
(c) Sedation (d) Immobility
(a) GABA (b) Glutamate
(e) All of the above
(c) Serotonin (d) Substance P
501. Local anaesthetics prevent or relieve pain
492. The cell bodies of 5HT neurons in CNS are
by
principally located in
(a) Inhibit substance P
(a) Hypothalamus (b) Frontal lobe
(b) Interrupt nerve conduction
(c) Raphenuclei (d) Caudate nucleus
(c) Modify neuronal metabolism
493. In GI tract enterochromaffin cells (d) All of the above
maximum in
502. What is true about sensitivity of nerve
(a) Stomach (b) Duodenum
fibres to local anaesthetics ?
(c) Jejunum (d) Colol
(a) Small fibres are more sensitive
494. Tryptophan is essential for synthesis of (b) Large fibres are more sensitive
(a) Niacin (b) Serotonin (c) Both small and large fibres are equally
(c) Melatonin (d) All of the above senstive
(d) Small fibres are resistant
495. Which of the following is 5HT antagonists
except 503. Which function is lost last after local
(a) Sumatriptan (b) Ondansetron anaesthetic injection
(c) Ketanserin (d) None of the above (a) Pain (b) Touch
(c) Muscle tone (d) Deep pressure
496. The rate at which inhalational anaesthetic
passes into tissues depends on 504. A vasoconstrictor is added to local
(a) Tissue: Blood partition coefficient anaesthetic for
(b) The rate at which gas is delivered to the (a) Prolongation of anaesthetic action
tissues (b) Tackling anaphylaxis
(c) The partial pressure of the gas in the arterial (c) To prevent bleeding
blood (d) All of the above
(d) All of the above
505. The local anaesthetic of choice during
497. General anaesthesis for surgery requires labor is
(a) Unconsciousness (a) Procaine (b) Bupivacaine
(b) Analgesia (c) Cocaine (d) Lignocaine
(c) Suppression of visceral reflexes
506. Bier’s block refers to
(d) Muscle relaxation
(a) Field block anaesthesia
(e) All of the above
(b) Nerve block anaesthesia
498. The most ideal inhalation anaesthetic is (c) Intravenous regional anaesthesia
(a) Halothane (b) Enflurane (d) Infiltration anaesthesia
70 MCQs IN PHARMACOLOGY

507. Which of the following is the most trouble 516. Chemically the phenothiazines have
shooter in spinal anaesthesia (a) One ring (b) Two rings
(a) Sympathetic block (c) Three rings (d) Five rings
(b) Parasympathetic block
517. Following are examples of generalized
(c) Contracted intestines seizures, except
(d) Paralysed intercostals muscles
(a) Tonic-clonic seizures
508. A local anaesthetic in its lowest concen- (b) Infantile spasms
tration will block (c) Simple partial seizures
(a) Somatic motor (d) Atonic seizures
(b) Somatic sensory
518. The D3 receptors are preferentially
(c) Preganglionic sympathetic present in
(d) All of the above
(a) Negative inotropy
509. Benzodiazepines act on which receptors (b) QT prolongation
(a) GABAA (b) GABAB (c) P-R prolongation
(c) Both of the above (d) None of the above (d) ST depression
(e) All of the above
510. Which of the following benzodiazepines
has shortest half-life 519. What is rabit syndrome
(a) Alprazolam (b) Oxazepam (a) Tardive dyskinesia
(c) Clonazepam (d) Midazolam (b) Akathisia
(c) Perioral tremor
511. Which of the following barbiturate has
(d) None of the above
shortest half-life
(a) Secobarbital (b) Pentobarbital 520. The maximum daily dose of Alprazolam is
(c) Methohexital (d) Amobarbital (a) Anxiolytic (b) Sedative
(c) Antidepressant (d) All of the above
512. Barbiturate induced enzyme induction
facilitates metabolism of 521. Atypical antidepressant is
(a) Steroid (b) Ethanol (a) Selegiline (b) Trazodone
(c) Vit K and D (d) All of the above (c) Desipramine (d) Doxeptan
513. Most barbiturates loose biological action 522. Setraline belongs to which class of com-
after oxidation of radicals at position pounds
(a) Digoxin (b) Furosemide (a) MAO inhibitor
(c) Enalapril (d) Amrinone (b) Serotonin reuptake inhibitor
514. The metabolism of ethanol is directly (c) Nor epinephrine reuptake inhibitor
proportional to (d) None of the above
(a) Digoxin (b) Furosemide 523. Following is an irreversible inhibitor of
(c) Enalapril (d) Amrinone GABA amino transferase
(a) Pyrimidine (b) Vigabatrin
515. Disulfiram–ethanol interaction often
produces hypertension due to (c) Lamotrigine (d) Gabapentin

(a) Increased concentration of ethanol 524. Lithium has also been used for which of
(b) Inhibition of sympathetic tone the following
(c) Inhibition of dopamine hydroxylase (a) Gout (b) Salt substitute
(d) None of the above (c) Anticonvulsant (d) All of the above
DRUGS ACTING ON CENTRAL NERVOUS SYSTEM 71

525. Which lithium salt is used for manic- 534. Which of the following is 5 HT receptor
depressive disorders antagonist
(a) Carbonate (b) Chloride (a) Methysergide (b) Plzotifen
(c) Bromide (d) All of the above (c) Minaserin (d) All of the above
526. MAO inhibitors in general raise the brain 535. Fenton reaction refers to the formation of
level of
(a) Free radical (b) Hydrogen peroxide
(a) Dopamine (b) Serotonin
(c) Nitric oxide (d) None of the above
(c) Nor adrenaline (d) All of the above
536. How many types of dopamine receptors
527. Which of the following is not MAO-A are present in brain
inhibitor
(a) 3 (b) 5
(a) Selegiline (b) Brofaromine
(c) 7 (d) 9
(c) Clorgyline (d) Moclobemide
537. The term endorphin refers to which of the
528. Which of the recently introduced
following
antiepileptics is useful in absence seizure
(a) Encephalins (b) Dynorphins
(a) Absence attack
(b) Partial seizure (simple) (c) b endrophin (d) All of the above
(c) Clonic tonic seizure 538. How many types of opioid receptor are
(d) Complex partial seizure present in brain

529. Gabapentin acts by (a) 3 (b) 4


(c) 5 (d) 6
(a) Binds to GABAA receptors
(b) Binds to GABAB receptors 539. The agent that has strong affinity for µ, δ
(c) Inhibits GABA destruction and kappa receptor is
(d) Promotes GABA release (a) Morphine (b) Etorphine
(e) All of the above (c) Fentanyl (d) Sufentanil
530. IV administration of phenytoin in status 540. Anti-epileptic, chemically derived from
epilepticus carries the danger of the tricyclic antidepressant is
(a) Arrhythmia (b) Cerebellar atrophy (a) Carbamazepine (b) Ethosuximide
(c) Renal failure (d) Jaundice (c) Phenytoin (d) None of the above
(e) All of the above
541. The triad of coma, pinpoint pupil and
531. For seizure control, therapeutic plasma decreased respiration points toward
phenobarbitone should be poisoning with
(a) 2 –5 µg/ml (b) 10 – 20 µg/ml (a) Organophosphorus
(c) 50 – 60 µg/ml (d) 80 – 90 µg/ml (b) Morphine
532. The drug of choice for status epilepticus (c) Mushroom
is (d) Belladona
(a) Clonazepam (b) Diazepam
542. Which of the following is not a feature of
(c) Lorazepam (d) Alprazolam alcohol withdrawal syndrome
533. The maximum dose of ergotamine per (a) Seizure
week in treatment of migraine is (b) Hypertension
(a) 5 mg (b) 10 mg (c) Constricted pupils
(c) 20 mg (d) 40 mg (d) Delirium tremens
72 MCQs IN PHARMACOLOGY

543. Angel dust refers to 552. Quinidine is contraindicated in


(a) LSD (b) MDMA (a) Atrial flutter (b) Atrial fibrillation
(c) Phencyclidine (d) MDA (c) PSVT (d) Digoxin toxicity
544. Acetazolamide inhibits which form of 553. Nalorphine is not used as antidote for
carbonic anhydrase (a) Morphine (b) Pethidine
(a) Membrane bound (b) Cytosolic (c) Pentazocine (d) Heroin
(c) Both of the above (d) None of the above
554. Carbonic anhydrase inhibitor is often
545. The site of action of loop diuretics is useful in
(a) Proximal convoluted tubule (a) Thyrotoxicosis (b) Petit mal epilepsy
(b) Proximal straight tubule (c) Myasthenia gravis (d) Bronchial asthma
(c) Thick ascending limb 555. Which of the following drug is
(d) Distal convoluted tubule contraindicated in myasthenia gravis ?
546. Which loop diuretic is more likely to have (a) Neostigmine (b) Pyridostigmine
toto toxicity Ethacryhia (c) Quinine (d) Ephedrine
(a) Furosemide (b) Bumetamide 556. Petit mal epilepsy is treated by
(c) Ethacrynic acid (d) Torsemide
(a) Phenobarbitone (b) Ethosuximide
547. Among thiazide diuretics the relative (c) Diazepam (d) Phenytoin
potency is highest for
557. Lithium toxicity may manifest as
(a) Hydrochlorthiazide
(a) Polyuria (b) Nephrotic syndrome
(b) Benzthiazide
(c) Renal calculi (d) Acute renal failure
(c) Polythiazide
(d) Indapamide 558. In Parkinsonism levodopa exer ts
following effects except
548. The anti diuretic action of vasopressin is
(a) Improves tremor
mediated via
(b) Improves akinesia
(a) V1 receptor (b) V2 receptor
(c) Improves muscle power
(c) Both of the above (d) None of the above
(d) Improves dystonia
549. Hyoscine differs from atropine in that
559. The primary mechanism of action of
(a) Its duration of action is longer benzodiazepine is
(b) Anticholinergic action is more on heart (a) Dopamine antagonism
(c) Has depressant action on CNS in low doses (b) Adenosine antagonism
(d) Slowly absorbed from GI tract (c) Opening up of chloride channel
(d) Facilitation of GABA mediated chloride influx
550. All are side effects of bromocriptine
except 560. Which of the following general anaes-
thetic has poor muscle relaxation ?
(a) Hallucination
(b) Tremor (a) Ether (b) Nitrous oxide
(c) Halothane (d) Enflurane
(c) Suppression of lactation
(d) Postural hypotension 561. Which of the following secretions is not
stimulated by acetylcholine ?
551. ETDA is useful in poisoning due to all except
(a) Tear (b) Bile
(a) Lead (b) Zinc (c) Sweat (d) Pancreatic juice
(c) Iron (d) Mercury
DRUGS ACTING ON CENTRAL NERVOUS SYSTEM 73

562. Which of the following drugs is not 569. Huntington’s chorea is hereditary
effective in acute congestive glaucoma disorder where there is
(a) Acetazolamide (b) Pilocarpine (a) Loss of GABA neurons in corpus striatum
(c) Timolol (d) Mannitol (b) An over-activity in dopaminergic nigro-striatal
pathway
563. The anticholinergic used only as prean-
(c) Loss of some cholinergic neurons in the
aesthetic medication is
corpus striatum
(a) Atropine (b) Glycopyrrolate (d) All the above
(c) Isopropamide (d) Dicyclomine (e) None of the above
564. Ephedrine is similar to adrenaline in 570. Which of the following MAO inhibitors
respect to prevents the 1-methyl 4-Phenyl 1,2,3,6
(a) Duration of action tetrahydropyridine’s (MPTP) selective
(b) Potency action in destroying the dopaminergic
(c) Penetration of blood brain barrier nigro-striatal neurons
(d) Both α and β adrenergic effects2106. (a) Tranylcypromine
(b) Phenelzine
565. The neurotransmitter released at the end
of the sympathetic nerve fibre is (c) Selegiline
(d) All of the above
(a) Epinephrine (b) Furosemide
(e) None of the above
(c) Enalapril (d) Amrinone
571. Concurrent use ow which of the following
566. Valproic acid is shown to be effective
drugs may reverse the antiparkinsonian
against
effect of levodopa
(a) Absence seizures
(a) Carbidopa (b) Selegiline
(b) Partial seizures
(c) Bromocriptine (d) Pyridoxine
(c) Generalized seizures
(e) All of the above (f) None of the above
(d) All of the above
572. Selegiline is a
567. In which of the following conditions
centrally acting anticholinergic drugs (a) MAO – A inhibitor
cannot be used for treatment (b) MAO – B inhibitor
(a) Parkinsonian disease (c) Drug that releases dopamine
(b) Akathisia (d) All of the above
(c) Tardive dyskinesia (e) None of the above
(d) All the above 573. Schizophrenia has following clinical
(e) None of the above feature

568. Levodopa and carbidopa combination is (a) Delusions


used to treat parkinsonian disease (b) Hallucinations
because carbidopa (c) Thought disorders
(a) Selectively stimulates dopa decarboxylase (d) All of the above
(b) Enters the CNS and increases the dopamine 574. Which of the following is an ergot
in the nigro-striatum derivative used in Parkinson’s disease as
(c) Reduces levodopa – induced dyskinesias a dopamine agonist
(d) Reduces levodopa induced vomiting and (a) Levodopa (b) Carbidopa
nausea (c) Bromocriptine (d) Selegiline
(e) All of the above (e) Amantadine (f) All of the above
(f) None of the above (g) None of the abvoe
74 MCQs IN PHARMACOLOGY

575. Which of the following drugs used in 582. Droperidol, a neuroleptic, belongs to
Parkinson’s disease is a peripherally following class of drugs.
acting dopa decarboxylase inhibitor ?
(a) Phenothiazines
(a) Levodopa (b) Carbidopa (b) Thioxanthines
(c) Bromocriptine (d) Selegiline (c) Butyrophenone
(e) Amantadine (f) All of the above (d) Benzamides
(g) None of the above
583. Which of the following opioid analgesics
576. Which of the following opioid antagonists is used as a preferred and rational
is used as an adjunct to prevent relapse antitussive agent ?
in addicts following detoxification
because of its long half-life ? (a) Morphine (b) Codeine
(c) Meperidine (d) Methadone
(a) Nalorphine (b) Naloxone
(c) Naltrexone (d) All of the above 584. Which of the following opioid analgesics
(e) None of the above has very low protein binding ?
(a) Morphine (b) Codeine
577. Which of the following opioid antagonists
is not a derivative of oxymorphone ? (c) Meperidine (d) Methadone
(e) Propoxyphene (f) All of the above
(a) Nalorphine (b) Naloxone
(c) Naltrexone (d) All of the above (g) None of the above
(e) None of the above 585. Which of the following group of analgesic
drugs act on the CNS ?
578. Which of the following opioid antagonists
is a competitive antagonist of all the (a) Morphine – like drugs
effects of morphine at opioid receptor in (b) NSAIDS (Aspirin and related drugs)
a low dose and in higher dose mimics (c) Local Anaesthetic
effects of morphine (such as analgesic
(d) All of the above
effect) ?
(e) None of the above
(a) Nalorphine (b) Naloxone
(c) Naltrexone (d) All of the above 586. Which of the following statements
(e) None of the above regarding lithium is not correct ?
(a) It is a monovalent cation
579. Morphine administration leads to
(b) Salts of lithium share some chemical
(a) Physical dependence characteristics with salts or sodium and
(b) Psychological dependence potassium
(c) Physical as well as Psychological dependence (c) The half life of lithium is more in adult and
(d) None of the above adolescent patient of bipolar as compared
to an elderly patient of bipolar disorder
580. Naloxone and naltrexone are
(d) Chances of neonatal goiter and congenital
(a) Opioid agonists cardiovascular malformation if administered
(b) Opioid agonists/antagonists to a pregnant women
(c) Opioid antagonists (e) All of the above
(d) None of the above (f) None of the above
581. Opioid analgesics inhibit the release of 587. Which of the following anticonvulsant
which of the following hormones. drugs is used for prophylactic treatment
(a) Anti-diuretic hormone of mood swings (i.e. manic-depression
(b) Luteinizing hormone illness) in patients unresponsive to lithium
carbonate ?
(c) Prolactin
(d) Somatotropin (a) Phenobarbitone (b) Phenytoin
DRUGS ACTING ON CENTRAL NERVOUS SYSTEM 75

(c) Ethosuximide (d) Carbamazepine (c) Mitochondrial membrane


(e) All of the above (f) None of the above (d) All of the above
588. Which of the following thioxanthene 594. Which of the following drugs is used in
derivatives may be used as an anti- the childhood enuresis ?
emetic and sedative ? (a) Imipramine hydrochloride
(a) Chlorprothixene (b) Flupenthixol (b) Imipramine pamoate
(c) Thiothixene (d) All of the above (c) Both the above
(e) None of the above (d) None of the above
589. Haloperidol is used in Huntington’s chorea 595. Which of the following tricyclic antide-
as a pressant drugs has the least anticholin-
(a) Anti-psychotic drug ergic effect and sedation ?
(b) Anti-dyskinetic drug (a) Amitriptyline (b) Amoxapine
(c) Anti-emetic (c) Clomipramine (d) Desipramine
(d) None of the above (e) All the above (f) None of the above
590. Chlorpromazine is a phenothiazine deriv- 596. The tricyclic antidepresent drugs produce
ative having a their effect by
(a) Aliphatic side chain (a) Increasing the synaptic concentration of
(b) Piperazine side chain noradrenaline in the CNS
(c) Peperazine side chain (b) Increasing the synaptic concentration of 5-
HT in the CNS
591. Which of the following extra-pyramidal
(c) By causing inhibition of noradrenaline and
effects seen with antipsychotic agents is
5-HT re-uptake by the pre-synaptic neuronal
not due to dopamine receptor blockade,
but due to super-sensitivity of dopamine membrane
receptors ? (d) All the above
(a) Acute dystonia reaction 597. Which of the following tricyclic antide-
(b) Parkinsonian syndrome pressant drugs is a preferred drug in the
(c) Tardive dyskinesia treatment of obsessive – compulsive
dirorder ?
(d) Akathisia
(e) All the above (a) Amitriptyline (b) Amoxapine
(f) None of the above (c) Clomipramine (d) Doxepin
(e) All the above (f) None of the above
592. Which of the following extra-pyramidal
effects seen with antipsychotic agents is 598. MAO – A enzyme has no substrate
also known as “rabbit syndrome” ? preference for
(a) Acute dystonia reaction (a) Noradrenaline (b) Dopamine
(b) Parkinsonian syndrome (c) 5–HT (d) Phenylethylamine
(c) Tardive dyskinesia (e) All the above (f) None of the above
(d) Perioral tremon 599. Risk with acute overdose is less with which
(e) Akathisia(f) None of the above of the following class of antidepressants
593. MAO is localized in (a) Tricyclic antidepressants
(a) Cell membrane (b) MAO inhibitors
(b) Plasma (c) 5-HT uptake inhibitors
(d) Atypical antidepressand drugs
76 MCQs IN PHARMACOLOGY

600. Pergolide is. (a) Tonic – clonic seizures


(a) Dopamine precursor (b) Absence seizures
(b) Dopamine agonist (c) Atonic seizures
(c) Dopamine releasing agent (d) All of the above
(d) Dopamine antagonist (e) None of the above

601. Phenelzine and tranylcypromine belong 607. Which of the following barbiturates is a
to which one of the following group long acting barbiturate (i.e. has a longer
(class) of antidepressants onset and duration of action)
(a) Tricyclic antidepressants (a) Phenobarbital (b) Amobarbital
(b) MAO inhibitors (c) Secobarbital (d) All the above
(c) 5-HT uptake inhibitors (e) None of the above
(d) Atypical antidepressant drugs 608. The half life of ethosuximide in adult is
(e) None of the above
(a) 10–20 hours (b) 30–40 hours
602. Which one of the following drugs is not (c) 50–60 hours (d) 60–70 hours
5-HT uptake inhibitor but used as an (e) None of the above
antidepressant ?
609. Concurrent use of carbamazepine and
(a) Fluoxetine (b) Maprotiline
phenyton will lead to
(c) Paroxetine (d) Sertraline
(a) Increased metabolism of phenyton
(e) All of the above (f) None of the above
(b) Decreased metabolism of phenytoin
603. Following endogenous opioid peptide is (c) No change in the metabolism of phenytoin
a pentapeptide
610. Zolpidem is used as
(a) Enkephalin (b) Dynorphin A
(a) Anticonvulsant drug
(c) Endorphin (d) Dynorphin B
(b) Anti-anxiety drug
604. Concurrent use of which of the following (c) Sedative and hypnotic drug
may lead to similar effects seen in
(d) myorelaxant drug
patients who are administered disulfiram
after the ingestion of alcohol (e) All the above
(f) None of the above
(a) Oral hypoglycemic drug
(b) Coumarin type antigoagulants 611. Which of the benzodiazepines is used
(c) Salicylates only as a sedative and hypnotic
(d) All of the above (a) Chlordiazepoxide (b) Clorazepam
(e) None of the abvoe (c) Diazepam (d) Flurazepam
(e) All of the above (f) None of the above
605. Wernicke’s encephalopathy and Korsa-
koff’s psychosis occur often in alcoholic’s 612. Which of the following benzodiazepines
due to is not in active form and behaves as a
(a) Malnutrition prodrug or drug prercursor
(b) Vitamin deficiencies (a) Chlordiazepoxide (b) Diazepam
(c) Faulty gastrointestinal deficiencies (c) Parzepam (d) Clorazepate
(d) All of the above (e) All of the above (f) None of the above
(e) None of the above 613. The dynorphins are endogenous ligands
606. Phenobarbitone is beneficial in the for
treatment of which of the following (a) µ (b) δ
generalized seizures (c) κ (d) σ
DRUGS ACTING ON CENTRAL NERVOUS SYSTEM 77

614. A drug that produces a mild degree of (e) All of the above (f) None of the above
nonselective depression of the central
620. In small intestine, morphine particularly
nervous system by decreasing the
affects
response of an individual to all sensory
modalities is known as (a) Duodenum
(b) Ileum
(a) Sedative (b) Hypnotic
(c) Both (a) and (b)
(c) None of the above
(d) Does not affect small intestine
615. Lidocaine is used as
621. Local anaesthetics affect membrane
(a) Mucosal local anaesthetic excitability by inhibiting
(b) Parenteral local anaesthetic agent (a) Sodium (Na+) Channel inactivation
(c) Mucosal as well as parenteral local anaesthetic (b) Increased (Na+) conductance
agent
(c) Increased potassium (K+) conductance
(d) None of the above (d) All of the above
616. Prilocaine, a local anaesthetic is used as (e) None of the above
(a) Dental anaesthesia 622. The order of loss of sensory function
(b) Low spinal anaesthesia for vaginal delivery when a local anaesthetic is used is
(c) Abdominal surgical anaesthesia (a) Temperature, pain, touch and deep pressure
(d) Caudal anaesthesia (b) Pain, touch, deep pressure and termperture
(e) All the above (c) Pain, temperature, touch and deep pressure
(f) None of the above (d) None of the above

617. Which of the following statements 623. Ester type local anaesthetics are hydrolyzed
regarding procaine, a local anaesthetic by
agent is not correct. (a) True cholinesterase
(a) It has a very low lipid solubility at pH 7.4 (b) Pseudo cholinesterase
(b) It is least protein bound (c) Liver microsomal enzymes
(c) it has a short duration of action (d) All of the above
(d) It is an amide – type local anaesthetic (e) None of the above
(e) All of the above 624. When an amide is acted upon by an
amidase
618. Which of the following parenteral local
anaesthetics has a very high protein (a) Alcohol and acid is formed
binding and may produce analgesia for (b) Amine and acid is formed
more than 10 hours via its nerve block (c) Alcohol and amine is formed
action (d) None of the above
(a) Chloroprocaine (b) Bupivacaine 625. Which one of the following is not an ester
(c) Lidocaine (d) Prilocaine local anaesthetic ?
(e) All of the above (f) None of the above (a) Cocaine (b) Procaine
(c) Lidocaine (d) Benzocaine
619. Which of the following parenteral local
(e) All of the above (f) None of the above
anaesthetics has a short duration of
action (i.e. between 30 to 60 minutes) 626. Which of the following local anaesthetics
cause arteriolar vasoconstriction ?
(a) Chloroprocaine (b) Bupivacaine
(c) Lidocaine (d) Prilocaine (a) Lidocaine (b) Procaine
(c) Cocaine (d) Prilocaine
78 MCQs IN PHARMACOLOGY

(e) All of the above 633. Post – anaesthesia emergency delirium


is most frequent side effect of
627. Local anaesthetics are drugs that produce
(a) Enflurane (b) Halothane
(a) Irreversible loss of sensitivity to pain in the
(c) Isoflurane (d) Methoxyflurane
restricted area to which they are applied
(e) Nitrous oxide (f) All the above
(b) Reversible loss of sensitivity to pain in the
restricted area to which they are applied. (g) None of the above
(c) No loss of sensitivity to pain in the area to 634. Which of the following anaesthetic agents
which they are applied is not indicated in low doses to provide
(d) None of the above analgesia for procedures not requiring
loss of consciousness
628. Local anaesthetic molecule consists of
(a) Enflurane (b) Halothane
(a) Lipophilic group linked by an ester or amide
(c) Isoflurane (d) Methoxyflurane
chain
(e) Nitrous oxide (f) All the above
(b) Lipophilic group linked by an ester or amide
chain to an amine substitute (g) None of the above
(c) Lipophilic group linked to an amine substitute 635. Blood pressure is thought to generally
(d) None of the above remain unchanged with which one of the
following anaesthetic agent
629. Ketamine, a general anaesthetic agent
can be administered by (a) Enflurane (b) Halothane
(c) Isoflurane (d) Nitrous oxide
(a) Intravenous route
(e) All the above (f) None of the above
(b) Intramuscular route
(c) Intravenous or Intramuscular route 636. Which of the following anaesthetic agents
(d) None of the above has the best (most) analgesic effect in low
concentration
630. In the following benzodiazepines used as
(a) Enflurane (b) Halothane
intravenous general anaesthetic which
one has a more rapid onset and shorter (c) Isoflurane (d) Nitrous oxide
half life (e) All the above (f) None of the above
(a) Diazepam (b) Lorazepam 637. Opioid angalgesics provide symptomatic
(c) Midazolam relief of
631. Which of the following is a intravenous (a) Pain (b) Cough
anaesthetic (c) Diarrhea (d) All of the above
(a) Thiopental (b) Ethomidate 638. Opioid receptors act via
(c) Propofol (d) Ketamine (a) Opening of potassium channels
(e) All of the above (f) None of the above (b) Inhibition of calcium channels
632. Which of the following intravenous (c) Both (a) and (b)
anaesthetic agents is useful to produce (d) Opening of sodium channels
sedation prior to anaesthesia and is also
helpful in reducing the amount of 639. The main factor that determines the speed
inhalation anaesthetic and recovery from inhalation of anaes-
thetics is
(a) Ultra–short acting barbiturates
(b) Benzodiazepines (a) Blood : Gas partition coefficient
(c) Opioid analgesic anaesthesia (b) Oil : Gas partition coefficient
(d) Propofol (c) Minimum alveolar concentration
(e) All of the above (d) All of the above
(f) None of the above (e) None of the above
DRUGS ACTING ON CENTRAL NERVOUS SYSTEM 79

640. Which of the following inhalation general (a) Excitatory causing decrease in potassium
anaesthetic is not a volatile liquid conductance
(a) Enflurane (b) Methoxyflurane (b) Inhibitory causing decrease in potassium
(c) Ether (d) Isoflurane conductance
(e) Nitrous oxide (f) All of the above (c) Excitor y causing increase in cation
conductance
(g) None of the above
(d) All of the above
641. Which of the following general anaesthetics (e) None of the above
is not administered intravenously
646. Which of the following CNS Neurotrans-
(a) Thiopental (b) Diazepam
mitters is excitatory on the individual
(c) Ketamine (d) Halothane neurons and causes decrease in potassi-
(e) All of the above um conductance
642. The use of nitrous oxide to relieve the (a) Dopamine (b) Acetylcholine
pain of surgery (c) GABA (d) Glycine
(a) Horace Wells (b) Humphry Davy (d) All of the above (e) None of the above
(c) William Morton (d) None of the above 647. Which of the following amino acids is an
643. Which of the following is designated as inhibitory transmitter and increases the
“The Vital Node” membrane permeability to chloride ions,
and thus causing inhibitory post synaptic
(a) Mid brain potential (IPSP) in the CNS
(b) Pons
(a) Glycine (b) Glutamate
(c) Medulla oblongata
(c) Aspartate (d) All of the above
(d) All of the above
(e) None of the above
(e) None of the above
648. Which of the following is a neurotrans-
644. CNS stimulant agents belong to
mitter in the central nervous system (CNS)
(a) Respiratory stimulants
(a) Acetylcholine
(b) Psychomotor stimulants
(b) Noradrenaline
(c) Psychomimetic agents
(c) Dopamine
(d) All of the above
(d) 5–Hydroxytryptamine
645. Acetylcholine action at the nicotinic (e) All of the above
receptor in the motoneuron – Renshaw
cell synapse is

ANSWERS
1. d 2. a 3. a 4. e 5. a 6. c
7. e 8. b 9. b 10. d 11. a 12. b
13. a 14. e 15. d 16. a 17. d 18. d
19. a 20. c 21. b 22. c 23. c 24. e
25. a 26. e 27. a 28. a 29. c 30. d
31. e 32. c 33. d 34. d 35. e 36. c
37. b 38. a 39. e 40. a 41. b 42. b
43. e 44. d 45. d 46. c 47. d 48. d
80 MCQs IN PHARMACOLOGY

49. d 50. c 51. c 52. d 53. b 54. c


55. c 56. e 57. b 58. d 59. d 60. c
61. a 62. d 63. e 64. d 65. b 66. c
67. c 68. c 69. a 70. c 71. a 72. e
73. b 74. d 75. c 76. d 77. e 78. b
79. c 80. d 81. e 82. b 83. b 84. d
85. c 86. a 87. c 88. b 89. d 90. a
91. e 92. b 93. d 94. b 95. c 96. c
97. b 98. e 99. d 100. e 101. c 102. b
103. c 104. a 105. c 106. b 107. c 108. c
109. c 110. e 111. e 112. c 113. d 114. d
115. e 116. d 117. c 118. d 119. c 120. d
121. d 122. e 123. b 124. a 125. b 126. c
127. d 128. d 129. b 130. d 131. e 132. c
133. d 134. d 135. c 136. d 137. d 138. a
139. c 140. b 141. d 142. e 143. c 144. a
145. c 146. d 147. b 148. e 149. b 150. e
151. e 152. a 153. d 154. e 155. d 156. a
157. e 158. c 159. d 160. e 161. c 162. c
163. e 164. d 165. c 166. c 167. e 168. b
169. 1. c, 2. e3. a, 4. b 170. 1. e, 2. b3. a, 4. d5. c 171. d 172. d
173. b 174. c 175. e 176. d 177. d 178. d
179. a 180. c 181. a 182. b 183. c 184. a
185. d 186. d 187. c 188. c 189. a 190. b
191. d 192. a 193. b 194. b 195. 1. a, 2. c3. b
196. c 197. b 198. b 199. a 200. d 201. a
202. b 203. d 204. b 205. d 206. d 207. c
208. c 209. d 210. b 211. b 212. b 213. c
214. a 215. a 216. a 217. a 218. d 219. a
220. d 221. a 222. d 223. d 224. e 225. c
226. b 227. e 228. b 229. e 230. b 231. d
232. b 233. d 234. a 235. c 236. c 237. d
238. d 239. b 240. b 241. c 242. d 243. b
244. c 245. d 246. a 247. b 248. c 249. d
250. c 251. d 252. b 253. b 254. d 255. a
256. c 257. d 258. a 259. b 260. a 261. d
262. d 263. d 264. c 265. b 266. a 267. c
268. b 269. a 270. a 271. a 272. a 273. d
274. a 275. c 276. d 277. b 278. c 279. d
280. d 281. b 282. b 283. a 284. b 285. a
286. d 287. d 288. c 289. b 290. b 291. d
292. b 293. c 294. d 295. b 296. c 297. a
298. d 299. b 300. d 301. c 302. b 303. d
DRUGS ACTING ON CENTRAL NERVOUS SYSTEM 81

304. a 305. c 306. b 307. d 308. c 309. b


310. d 311. a 312. a 313. d 314. b 315. c
316. b 317. b 318. c 319. b 320. a 321. c
322. b 323. c 324. d 325. b 326. b 327. a
328. c 329. d 330. c 331. c 332. a 333. a
334. b 335. a 336. b 337. a 338. d 339. c
340. c 341. c 342. b 343. b 344. d 345. a
346. c 347. c 348. d 249. c 350. d 351. d
352. b 353. a 354. b 355. b 356. d 357. d
358. b 359. d 360. a 361. d 362. c 363. b
364. b 365. a 366. c 367. c 368. a 369. a
370. d 371. b 372. a 373. a 374. c 375. c
376. b 377. b 378. a 379. b 380. c 381. c
382. a 383. a 384. a 385. b 386. d 387. c
388. a 389. b 390. a 391. b 392. c 393. c
394. d 395. a 396. b 397. a 398. c 399. d
400. b 401. d 402. d 403. b 404. c 405. c
406. d 407. c 408. c 409. a 410. d 411. c
412. b 413. d 414. d 415. d 416. b 417. b
418. d 419. c 420. d 421. c 422. b 423. c
424. b 425. a 426. b 427. b 428. d 429. a
430. a 431. a 432. a 433. c 434. b 435. d
436. c 437. c 438. b 439. b 440. a 441. a
442. d 443. d 444. c 445. b 446. c 447. c
448. d 449. b 450. a 451. a 452. b 453. a
454. a 455. a 456. a 457. b 458. a 459. c
460. b 461. b 462. a 463. b 464. c 465. b
466. c 467. a 468. c 469. c 470. c 471. c
472. b 473. a 474. c 475. d 476. c 477. c
478. a 479. c 480. b 481. c 482. b 483. b
484. b 485. a 486. d 487. b 488. b 489. a
490. d 491. c 492. c 493. b 494. d 495. b
496. d 497. e 498. c 499. b 500. e 501. b
502. a 503. c 504. a 505. b 506. c 507. a
508. c 509. a 510. d 511. c 512. d 513. b
514. c 515. c 516. c 517. c 518. c 519. c
520. b 521. b 522. b 523. b 524. d 525. a
526. d 527. a 528. b 529. d 530. e 531. b
532. b 533. b 534. d 535. a 536. b 537. d
538. a 539. b 540. a 541. b 542. c 543. c
544. c 545. c 546. c 547. c 548. b 549. d
550. b 551. d 552. d 553. c 554. b 555. c
556. b 557. a 558. c 559. d 560. b 561. b
562. c 563. b 564. b 565. a 566. d 567. c
82 MCQs IN PHARMACOLOGY

568. d 569. d 570. c 571. d 572. b 573. d


574. c 575. b 576. c 577. a 578. a 579. c
580. c 581. b 582. c 583. b 584. b 585. a
586. c 587. d 588. a 589. b 590. a 591. c
592. d 593. c 594. a 595. d 596. d 597. c
598. d 599. d 600. b 601. b 602. b 603. a
604. a 605. d 606. a 607. a 608. c 609. a
610. c 611. d 612. d 613. c 614. a 615. c
616. a 617. d 618. b 619. a 620. a 621. a
622. d 623. b 624. b 625. c 626. c 627. b
628. b 629. c 630. c 631. d 632. b 633. b
634. b 635. d 636. d 637. d 638. c 639. a
640. e 641. d 642. b 643. c 644. d 645. c
646. b 647. a 648. e

EXPLANATIONS FOR ANSWERS


3.a Neurotransmitter is a substance present in 50. c In CNS, acetylcholine was the first neurotrans-
neuron and is secreted by neurons to transmit mitter to be identified pharmacologically. In
signals to its postsynaptic targets. 1950s Eccles showed that excitation of Renshaw
8. b An example of neuromodulator is cAMP. It acts cells by motor axon collaterals was blocked by
as a second messenger of synaptic transmission. nicotinic antagonists.
16. a Suramin is an antagonist of purinergic receptors, 58. d Metabotropic receptors of glutamate are
specifically P2x and P2y coupled with G-proteins. AMPA, kinate and
NMDA receptors are coupled with ion channels.
20. c NMDA receptors are closely associated with
the induction of synaptic plasticity. AMPA and 61. a Phencyclidine blocks NMDA receptors and thus
kainite receptors are involved in mediation of antagonizes effects of glutamate, an excitatory
fast depolarization. amino acid neurotransmitter.
28. a Muscimol is GABAA agonist. 75. c Picrotoxin is an antagonist of GABAB receptors
and blocks chloride channels associated with
Baclofen is a GABAB agonist.
it. Muscimol is GABAA selective agonist.
Bicuculline is a GABAA antagonist. Bicuculline is GABAA antagonist. Baclofen is
32. c Strychnine is an antagonist of glycine receptors. GABAB agonist.
Taurine and â-analine are agonists of glycine 80. d In the brain, GABA is the main inhibitory
receptors. Quisqualate is an agonist of AMPA neurotransmitter, whereas glycine plays an
receptors. important inhibitory role in brain stem and spinal
41. b Baclofen is an agonist of GABAB receptor. cord.
Muscimol is an agonist of GABAA receptor. 90. a A direct interaction is believed to exist between
Picrotoxin and bicuculline are antagonists of GABAA receptors and various drugs e.g. alcohol,
GABAA receptors. volatile anaesthetics etc. Hence, GABA A
antagonists attenuate some of the action of alcohol.
DRUGS ACTING ON CENTRAL NERVOUS SYSTEM 83

108. c As ethanol competes for alcohol dehydroge- 270. a Iproniazid is an antitubercular agent, an
nase, a 10% ethanol solution is generally used inhibitor of MAO and has antidepressant effect.
in methanol poisoning. 280. d The major factors, which affect elimination of a
109. c Alcohol is teratogenic and is associated with general anesthetic are, pulmonary ventilation,
fetal alcohol syndrome. blood flow and solubility in blood and tissues.
123. b Diethyl ether has high solubility in blood and 292. b Barbiturates bind to GABA receptors (GABAA)
has slow onset of central action and hence all but on a different site than benzodiazepines and
the four stages of general anesthesia can be enhance the action of GABA.
observed with this drug. 312. a Chemical name of halothane is 2-bromo-2-
141. d Metabolism is not an important route of chloro-1,1,1-trifluoroethane.
elimination of inhalational anaesthetics. However, 329. d Nitric oxide is a colorless gas without significant
except nitrous oxide, other anaesthetics such as odor or taste and is non-flammable, non-irritant
halothane, ether and isoflurane do undergo some and potent analgesic.
degree of metabolism.
350. d Surgical pain is often severe and hence opioid
155. d An ideal anesthetic should possess wide margin
analgesics such as meperidine, fentanyl,
of safety with no adverse effect. But no single
alfentanil, sufentanil and morphine are generally
anaesthetic can achieve all the desirable effects
used as supplemental agents during general
of general anaesthetics without some adverse
anaesthesia.
effects.
368. a Propofol is an intravenous anesthetic agent. It
156. a An action common to most general anaesthetics
is an oil at room temperature and is supplied as
is to increase the cellular threshold of firing and
1% emulsion.
thus decreasing neuronal activity.
379. b Local anaesthetics produce stimulation of CNS,
168. b β-Carbolines are inverse agonists of
restlessness and tremors, which may proceed
benzodiazepines.
to convulsions. Selective depression of inhibitory
174. c Zolpidem is a nonbenzodiazepine sedative- neurons is responsible for the CNS excitation.
hypnotic agent and belongs to imidazopyridine
class of drugs. 383. a Bupivacaine is an amide-linked local anaesthetic.
Procaine, tetracaine and proparacaine have ester
193. b Ethinamate is a sedative-hypnotic agent and is
linkage in their structure.
a urethane derivative. Ethinamate has rapid
onset and short duration of action. 392. c Benzocaine has ester linkage in its structure.
Mepivacaine, ropivacaine and priocaine have
200. d For substance to be classified as neuro-transmitter,
amide linkage in their structure.
it has to fulfill all the criteria as stated in A, B and
C. 397. a In infiltration anesthesia, local anaesthetic is
directly injected into tissue without taking into
213. c Phencyclidine, also called as angel-dust, is a
consideration the course of cutaneous nerves.
non-competitive antagonist of NMDA receptors
and it produces various psychotic symptoms e.g. 411. c Intravenous regional anesthesia or Bier’s block
hallucinations, thought disorders etc. relies on using the vasculature to bring the local
anaesthetic to the nerve trunks and endings.
219. a Fluoxetine is a serotonin transport blocker and
it is an antidepressant agent. 424. b Fetal alcohol syndrome is characterized by
225. c Neuronal death in response to high glutamate cluster of facial abnormalities viz. short nose,
concentration is mediated by excessive hypoplastic upper lip and short palpebral
activation of NMDA receptors and is associated fissures. In addition, it is associated with CNS
with influx of calcium. dysfunction and slow growth.
238. d Drugs that increases dopaminergic activity in 440. a Except Phenobarbital, only insignificant
CNS can produce or aggravate symptoms of quantities of the barbiturates are excreted
schizophrenia unchanged.
253. b Tranylcypromine, moclobemide and isocarboxid 458. a Zolpidem is a nonbenzodiazepine sedative-
are MAO inhibitors. Nomifensine is a dopamine hypnotic that interacts with benzodiazepine BZ1
transport blocker. receptors to produce its effects.
84 MCQs IN PHARMACOLOGY

473. a Regions of the cortex can be classified in three characterized by disordered thinking, delusions,
different ways: auditory hallucinations and emotional
(i) By the modality of the information processed. withdrawal.
(ii) By the cytoarchitectonic classification i.e. the 582. c Droperidol like haloperidol belongs to
geometrical relationship between cell types in butyrophenone neuroleptic class of drug.
the major cortical layers. 593. c MAO is a flavin-containing enzyme, which is
(iii) By the anatomical position. localized in mitochondrial membranes in nerve
496. d The rate at which inhalational anaesthetics terminals, liver and other organs.
passes into tissues depends on various factors; 600. b Pergolide, an ergot derivative, is a dopamine
tissue-blood partition coefficient i.e. solubility agonist and it interacts with both D1 and D2
of anaesthetics in tissues and blood, the rate of receptors.
delivery of anesthetic to the tissue. 603. a Enkephalin is a pentapeptide i.e. it has 5 amino
517. c Generalized seizure can be of various types – acids in its structure.
tonic- clonic seizures, absence seizures, tonic 613. c Dynorphin, specifically dynorphin A, is the
seizures, atonic seizures, clonic and myoclonic
endogenous ligand for ê1 receptors that are
seizures and infantile spasms.
responsible for spinal analgesia.
523. b Vigabatrin is an irreversible inhibitor of GABA
620. a In small intestine, morphine particularly affects
aminotransferase, the enzyme responsible for
upper part of small intestine, especially the
the degradation of GABA and thus increases
duodenum compared to ileum.
GABA at synaptic sites.
637. d Opioid analgesics provide symptomatic relief
540. a Carbamazepine is a derivative of iminostilbene
of pain, cough and diarrhea but the disease
and is chemically related to tricyclic
antidepressants. remains unaffected.
566. d Although valproic acid has been widely used 638. c All the opioid receptors are coupled to pertussis
for the treatment of absence seizures, it is also toxin-sensitive GTP binding proteins. These
effective against wide variety of partial and receptors produce opening of potassium
generalized seizures. channels and inactivation of calcium channels.
573. d Schizophrenia is an idiopathic psychoses 644. d CNS stimulants belong to all three categories.
DRUGS ACTING ON AUTONOMOUS NERVOUS SYSTEM 85

CHAPTER 3
DRUGS ACTING ON
AUTONOMOUS NERVOUS SYSTEM

1. In the autonomic regulation of blood 4. Several children at a summer camp were


pressure hospitalized with symptoms thought to
be due to ingestion of food containing
(a) Cardiac output is maintained constant at the
botulinum toxins. The effects of botulinum
expense of other hemodynamic variables
toxin are likely to include
(b) Elevation of blood pressure results in elevated
aldosterone secretion (a) Bronchospasm
(c) Baroreceptor nerve fibers docrease firing rate (b) Cycloplegia
when arterial pressure increases (c) Diarrhea
(d) Stroke volume and mean arterial blood (d) Skeletal muscle spasms
pressure are the primary direct determinants (e) Hyperventilation
of cardiac output
5. The neurotransmitter agent that is
(e) A condition that reduces the sensitivity of the normally released in the sinoatrial node
sensory baroreceptor nerve endings might of the heart in response to a blood
cause an increase in sympathetic discharge pressure increase is
2. The autonomic nervous system is also (a) Acetylcholine (b) Dopamine
known as (c) Epinephrine (d) Glutamate
(a) Visceral (e) Norepinephrine
(b) Vegetative 6. A direct-acting cholinomimetic that is
(c) Involuntary nervous system lipid-soluble and often used in the
(d) All of the above treatment of glaucoma is

3. Full activation of the sympathetic nervous (a) Acetylcholine (b) Bethanechol


system, as in maximal exercise, can (c) Physostigmine (d) Pilocarpine
produce all of the following responses (e) Neostigmine
except
7. Which of the following agents is a pro-
(a) Bronchial relaxation drug that is much less toxic in mammals
(b) Decreases intestinal motility than in insects ?
(c) Increased renal blood flow (a) Acetylcholine (b) Bethanechol
(d) Mydriasis (c) Physostigmine (d) Pilocarpine
(e) Increased heart rate (tachycardia) (e) Neostigmine
86 MCQs IN PHARMACOLOGY

8. Atropine overdose may cause which one 14. A 7-years-old boy has a significant bed-
of the following wetting problem. A long-acting indirect
(a) Gastrointestinal smooth muscle crampine sympathomimetic agent sometimes used
(b) Increased cardiac rate by the oral route for this and other
indications is
(c) Increased gastric secretion
(d) Pupillary constriction (a) Dobutamine (b) Ephedrine
(e) Urinary frequency (c) Epinephrine (d) Isoproterenol
(e) Phenylephrine
9. The synaptic pre-ganglionic neurons have
their cell bodies in the lateral horn of the 15. When pupillary dilation-but not cyclo-
grey matter of the plegia-is desired, a good choice is
(a) Thoracic segment only (a) Homatropine (b) Isoproterenol
(b) Lumbar segment only (c) Phenylephrine (d) Pilocarpine
(c) Both (a) and (b) (e) Tropicamide
(d) None of the above
16. ‘Fight or flight’ state can be described as
10. Which one of the following can be
(a) Sympathetic active; para-sympathetic
blocked by atropine?
quiescent
(a) Decreased blood pressure caused by (b) Sympathetic active; para-sympathetic active
hexamethonium
(c) Sympathetic quiescent; para-sympathetic
(b) Increased blood pressure caused by nicotine
active
(c) Increased skeleton muscle strength caused
by neostigmine 17. Your patient is to receive a selective β2
(d) Tachycardia caused by exercise stimulant drug β2 selective stimulant are
often effective in
(e) Tachycardia caused by infusion of acetylcholine
(a) Angina due to coronary insufficiency
11. Which of the following best describes the
(b) Asthma
mechanism of action of scopolamine ?
(c) Chronic heart failure
(a) Irreversible antagonist at nicotinic receptors
(d) Delayed or insufficiently strong labor
(b) Irreversible antagonist at muscarinic
receptors (e) All of the above
(c) Physiologic antagonist at muscarinic receptors 18. Which of the following drugs is the drug
(d) Reversible antagonist at muscarinic receptors of choice in anaphylaxis associated with
(e) Reversible antagonist at nicotinic receptors bronchospasm and hypotension ?

12. Accepted therapeutic indications for the (a) Cortisone (b) Epinephrine
use of antimuscarinic drugs include all of (c) Isoproterenol (d) Norepinephrine
the following except (e) Phenylephrine
(a) Hypertension
19. Which of the following effects of epineph-
(b) Motion sickness rine would be blocked by phentolamine
(c) Parkinson’s disease but not by metoprolol ?
(d) Postoperative bladder spasm (a) Cardiac stimulation
(e) Traveler’s diarrhea (b) Contractionof radial smooth muscle in the iris
13. In male sex organs _______ system is (c) Increase of camp in fat
responsible for erection. (d) Relaxation of bronchial smooth muscle
(a) Sympathetic (b) Parasympathetic (e) Relaxation of the uterus
(c) Norepinephrine (d) None of the above
DRUGS ACTING ON AUTONOMOUS NERVOUS SYSTEM 87

20. Propranolol is useful in all of the following (d) β1 – selective receptor blockers
except (e) Nonselective β - blockers
(a) Angina 25. A visitor from another city comes to your
(b) Familial tremor office complaining of incessant cough. He
(c) Hypertension has diabetes and hypertension and has
(d) Idiopathic hypertrophic subaortic cardiomy- recently started taking a different
opathy antihypertensive medication. The most
likely cause of his cough is
(e) Partial atrioverntricular heart block
(a) Enalapril (b) Losartan
21. ‘Rest and digest’ state can be described
(c) Minoxidil (d) Propranolol
as
(e) Verapamil
(a) Sympathetic active; para-sympathetic
quiescent 26. Reserprine, an alkaloid derived from the
(b) Sympathetic active; para-sympathetic active root of Rauwolfia serpentina
(c) Sympathetic quiescent; para-sympathetic (a) Can be used to control hyperglycemia
active (b) Can cause severe depression of mood
(c) Can decrease gastrointestinal secretion and
22. A friend has very severe hypertension and
motility
asks about a drug her doctor wishes to
prescribe. Her physician has explained that (d) Has no cardiac effects
this drug is associated with tachycardia (e) A spare receptor agonist
and fluid retention (which may be marked)
27. Nitroglycerin, either directly or through
and increased hair growth. Which of the
reflexes, results in which one of the
following is most likely to produce the
following effects
effects that your friend has described?
(a) Decreased heart rate
(a) Captopril (b) Guanethidine
(b) Decreased venous capacitance
(c) Minoxidil (d) Prazosin
(c) Increased afterload
(e) Propranolol
(d) Increased cardiac force
23. Which one of the following is character- (e) Increased diastolic intramyocardial fiber
istic of captopril and enalapril ? tension
(a) Competitively blocks angiotensim II at its 28. The antianginal effect of propranolol may
receptor be attributed to which one of the following
(b) Decreases angiotensin II concentration in the
(a) Block of exercise – induced tachycardia
blood
(b) Decreased end – diastolic ventricular volume
(c) Decreases rennin concentration in the blood
(c) Dilation of consticted coronary vessels
(d) Increases sodium and decreases potassium
(d) Increased cardiac force
in the blood
(e) Increased resting heart rate
(e) Decreases sodium and increases potassium
in the urine 29. The major common determinant of myo-
cardial oxygen consumption is
24. Postural hypotension is a common ad-
verse effect of which one of the follow- (a) Blood volume
ing types of drugs ? (b) Cardiac output
(a) ACE inhibitors (c) Diastolic blood pressure
(b) Alpha-receptor blockers (d) Heart rate
(c) Arteriolar dilators (e) Myocardial fiber tension
88 MCQs IN PHARMACOLOGY

30. Choose the incorrect statement from the (b) An increase in ATP synthesis
following (c) A modification of the actin molecule
(a) Sympathetic system increases rate of SA node (d) An increase in systolic intracellular calcium
(b) Sympathetic system causes constriction of levels
coronary arteries (e) A block of sodium/calcium exchange
(c) Sympathetic system causes increased motility
36. A patient has been taking digoxin for
of GIT smooth muscles
several years for chronic heart failure is
(d) Sympathetic system causes constriction of about to receive atropine for another
sphincter in GIT condition. A common effect of digoxin (at
31. Denervation super-sensitivity includes therapeutic blood levels) that can be
almost entirely blocked by atropine is
(a) Proliferation of receptors
(a) Decreased appetite
(b) Loss of mechanism for transmitter removal
(b) Increased atrial contractility
(c) Increased post-junctional responsiveness
(c) Increased PR interval on the ECG
(d) All of the above
(d) Headaches
32. Which of the following is approved for (e) Tachycardia
the treatment of hemorrhagic stroke ?
37. In a patient given a cardiac glycoside,
(a) Amyl nitrite
important effects of the drug on the heart
(b) Hydralazine include which of the following?
(c) Isosorbide moninitrate
(a) Decreased atrioventricular conduction
(d) Nifedipine velocity
(e) Nimodipine (b) Decreased ejection time
33. Which of the following drugs used for the (c) Increased ectopic automaticity
treatment of angina by inhalation has a (d) Increased ectopic automaticity
very rapid onset and a brief duration of (e) All of the above
effect ?
38. Which of the following situations consti-
(a) Amyl nitrite
tutes an added risk of digoxin toxicity ?
(b) Hydralazine
(a) Starting administration of captopril
(c) Isosorbide mononitrate
(b) Starting administration of quinidine
(d) Nifedipine
(c) Hyperkalemia
(e) Nimodipine
(d) Hypermagnesemia
34. Which of the following is a vasodilator (e) Hypocalcemia
drug used for hypertension that lacks a
direct effect on autonomic receptors but 39. At rest, the interior of the typical
may provoke anginal attacks? mammalian neuronal axon potential is
approximately ________ mV
(a) Amyl nitrite
(b) Hydralazine (a) 0 (b) –70
(c) Isosorbide mononitrate (c) –100 (d) –20
(d) Nifedipine 40. Following enzyme is involved in the
(e) Nimodipine synthesis of actylcholine

35. The biochemical mechanism of action of (a) Choline acetyl transferase


digitalis is associated with (b) Acetyl cholinesterase
(a) A decrease in calcium uptake by the (c) Both (a) and (b)
sarcoplasmic reticulum (d) None of the above
DRUGS ACTING ON AUTONOMOUS NERVOUS SYSTEM 89

41. Which of the following has been shown (a) Adenosine (b) Amiodarone
to prolong life in patient with chronic (c) Disopyramide (d) Esmolol
congestive failure but has a negative (e) Mexiletine
inotropic effect on cardiac contractility?
(a) Carvedilol (b) Digitoxin 48. Which of the following has the longest
half-life of all antiarrhythmic drugs?
(c) Digoxin (d) Dobutamine
(e) Enalapril (a) Adenosine (b) Amiodarone
(c) Disopyramide (d) Esmolol
42. Which of the following is the drug of
(e) Flecainide
choice in treating suicidal overdose of
digitoxin ? 49. Which of the following diuretics would
(a) Digoxin antibodies be most useful in a patient with cerebral
edema?
(b) Lidocaine
(c) Magnesium (a) Acetazolamide (b) Amiloride
(d) Potassium (c) Ethacrynic acid (d) Furosemide
(e) Quinidine (e) Mannitol

43. When used as an antiarrhythmic drug, 50. Which of the following therapies would
Lidocaine typically be most useful in the management of
severe hypercalcemia?
(a) Increases action potential duration
(b) Increases contractility (a) Amiloride plus saline infusion
(c) Increses PR interval (b) Furosemide plus saline infusion
(d) Reduces abnormal automaticity (c) Hydrochlorothiazide plus saline infusion
(e) Reduces resting potential (d) Mannitol plus saline infusion
(e) Spironolactone plus saline infusion
44. A 16-year-old-girl is found to have
paroxysmal attacks of rapid heart rate. 51. A 55-year-old patient with severe post-
The antiarrhythmic of choice in most cases hepatitis cirrhosis is started on a diuretic
of acute AV nodal tachycardia is for another condition. Two days later
(a) Adenosine (b) Amiodarone he is found in a coma. The drug most
likely to cause coma in a patient with
(c) Flecainide (d) Propranolol
cirrhosis is
(e) Quinidine
(a) Acetazolamide
45. Recognized adverse effects of quinidine (b) Amiloride
include which one of the following?
(c) Furosemide
(a) Cinchonism (d) Hydrochlorothiazide
(b) Constipation (e) Spironolactone
(c) Lupus erythematosus
(d) Increase in digoxin clearance 52. Oxytremorine is a selective agonist of
muscarinic _______ receptors
(e) Precipitation of hyperthyroidism
(a) M1 (b) M2
46. Following is an antagonist of ganglion
(c) M3 (d) M4
type nicotinic receptors
(a) Tubercurarine (b) α-bungarotoxin 53. A drug that is useful in glaucoma and
high-altitude sickness is
(c) Trimethaphan (d) All of the above
(a) Acetazolamide (b) Amiloride
47. Which of the following is an orally active
(c) Demeclocycline (d) Desmopressin
drug that blocks sodium channels and
decreases action potential duration? (e) Ethacrynic acid
90 MCQs IN PHARMACOLOGY

54. Cromolyn has as its major action (a) Chronic renal failure
(a) Block of calcium channels in lymphocytes (b) Hyperparathyroidism
(b) Block of mediator release from mast cells (c) Intestinal osteodystrophy
(c) Block of phosphodiesterase in mast cells and (d) Nutritional rickets
basophils (e) Osteoporosis
(d) Smooth muscle relaxation in the bronchi 60. Which one of the following drugs, when
(e) Stimulation of cortisol release by the adrenals used chronically, is associated with the
development of bone pain and mineral-
55. Following events occur in the cytoplasm
ization defects such as osteomalacia?
and not inside storage vesicles, except
(a) Calcitonin
(a) Conversion of tyrosine to dopa
(b) Dihydrotachysterol
(b) Conversion of dopa to dopamine
(c) Ergocalciferol
(c) Conversion of dopamine to norepinephrine
(d) Etidronate
(d) Conversion of dopamine to 3,4 dihydroxy
phenyl acetic acid (e) Norgestrel

56. Which one of the following is lest likely 61. A 70-year-old man has severe urinary
to be useful in the therapy of hyper- hesitancy associated with benign
calcemia? prostatic hyperplasia. He has tried alpha
blockers with little relief. His physician
(a) Calcitonin recommends a drug that blocks 5α α -
(b) Glucocorticoids reductase in the prostate and writes a
(c) Plicamycin prescription for
(d) Parenteral infusion of phosphate (a) Atropine (b) Clonidine
(e) Thiazide diuretics (c) Hydralazine (d) Neostigmine
57. Characteristics of Vitamin D and its (e) Propranolol
metabolites include which one of the 62. Action of norepinephrine and epinephrine
following? are terminated by
(a) Act to decrease serum levels of calcium (a) Reuptake into nerve terminal
(b) Activation of their Vitamin D receptors (b) Dilution by diffusion and uptake at extra-
increases cellular CAMP neuronal site
(c) Calcitriol is the major derivative responsible (c) Metabolic transformation
for increasing intestinal absorption of
(d) All of the above
phosphate
(d) Metabolites of Vitamin D increase renal 63. Typical results of beta-receptor activation
excretion of calcium include which one of the following?
(e) Vitamin D deficiency results in Pager’s (a) Hypoglycemia
disease (b) Lipolysis
58. Which of the following conditions is an (c) Glycogen synthesis
indication for the use of calcitonin? (d) Decreased skeletal muscle tremor
(a) Chronic renal failure (e) Decreased rennin secretion
(b) Hypoparathyroidism 64. A patient is admitted to the emergency
(c) Intestinal osteodystrophy room with orthostatic hypotension and
(d) Pager’s disease evidence of marked GI bleeding. Which
of the following most accurately describes
(e) Rickets
the probable autonomic response to this
59. Clinical uses of Vitamin D do not include bleeding?
DRUGS ACTING ON AUTONOMOUS NERVOUS SYSTEM 91

(a) Slow heart rate, dilated pupils, damp skin (a) Intestinal (b) Ureteric
(b) Rapid heart rate, dilated pupils, damp skin (c) Bronchial (d) Layngeal
(c) Slow heart rate, dry skin, increased bowel 71. α 1 – receptors are coupled with _______
sounds G protein.
(d) Rapid heart rate, constricted pupils,
(a) Gs (b) Gi
increased bowel sounds
(c) Gq (d) Go
(e) Rapid heart rate, constricted pupils, warm
skin 72. Hyoscine differs from atropine in that it
65. Drugs that block the α receptor on effector (a) Exerts depressant effects on the CNS at
cells at adrenergic nerve endings relatively low doses
(b) Exerts more potent effects on the heart than
(a) Antagonize the effects of isoproterenol on
on the eye
the heart rate
(c) Is longer acting
(b) Antagonize some of the effects of
epinephrine on the blood pressure (d) Has weaker antimotion sickness activity
(c) Antagonize the effects of epinephrine on 73. Which of the following anticholinergic
adenylyl cyclase drugs is primarily used in preanaesthetic
(d) Cause mydriasis medication and during surgery
(e) Decreases blood glucose levels (a) Glycopyrrolate
(b) Pipenzolate methyl bromide
66. Yohimbine is an antagonist of ______
receptors. (c) Isopropamide
(d) Dicyclomine
(a) α1 (b) α2
(c) both (a) and (b) (d) None of the above 74. Glycopyrrolate is the preferred antimus-
cannic drug for use before and during
67. Which of the following organs is surgery because
innervated only by parasympathetic
(a) It is potent and fast acting
nerves ?
(b) It has no central action
(a) Iris muscles (b) Ciliary muscle
(c) It has antisecretory and vagolytic actions
(c) Sweat glands (d) Splenic capsule
(d) All of the above
68. Muscarinic cholinergic receptors
75. Which of the following mydriatics has the
(a) Are located only on parasympathetically fastest and briefest action ?
innervated effector cells (a) Atropine (b) Homatropine
(b) Mediate responses by opening an instrinsic (c) Tropicamide (d) Cyclopentolate
Na+ ion channel
(c) Are present on vascular endothelium which 76. The most suitable mydratic for a patient
has no cholinergic nerve supply of corneal ulcer is
(d) Predominate in the autonomic ganglia (a) Atropine sulfate (b) Homatropine
(c) Cyclopentolate (d) Tropicamide
69. The cardiac muscarinic receptors
(a) Are of the M1 subtype 77. The most effective antidote for belladon-
na poisoning is
(b) Are of the M2 subtype
(a) Neostigmine (b) Physostigmine
(c) Are selectively blocked by pirenzepine
(c) Pilocarpine (d) Methacholine
(d) Function through the PIP 2 → IP 3/DAG
pathway 78. Atropine is contraindicated in
70. Atropine does not exert relaxant/ antis- (a) Cyclic AMP (b) Inositol trisphosphate
pasmodic effect on the following muscle (c) Diacyl glycerols (d) G protein
92 MCQs IN PHARMACOLOGY

79. Select the correct statement from the 86. Adrenaline raises blood glucose level by
following about α1 – adrenergic receptor the following actions except
agonists (a) Inducing hepatic glycogenolysis
(a) Isoproterenol > epinephrine ≥ norepinephrine (b) Inhibiting insulin secretion from pancreatic
(b) Epinephrine > isoproterenol > norepinephrine β cells
(c) Isoproterenol = epinephrine ≥ norepinephrine (c) Augmenting glucagons secretion from
pancreatic α cells
(d) Epinephrine ≥ norepinephrine > isoproterenol
(d) Inhibiting peripheral glucose utilization
80. The most efficacious inhibitor of cate-
cholamine synthesis in the body is 87. Noradrenaline is administered by

(a) α - methyl – p - tyrosine (a) Subcutaneous injection


(b) Intramuscular injection
(b) α - methyldopa
(c) Slow intravenous infusion
(c) α - methyl - norepinephrine
(d) All of the above routes
(d) Pyrogallol
88. Dobutamine differs from dopamine in that
81. Tyramine induces release of noradrenaline
from adrenergic nerve endings (a) It does not activate peripheral dopaminergic
receptors
(a) By depolarizing the axonal membrane
(b) It does not activate adrenergic β receptors
(b) By mobilizing Ca2+
(c) It causes pronounced tachycardia
(c) By a nonexocytotic process
(d) It has good blood-brain barrier penetrability
(d) Only in the presence of MAO inhibitors
89. Ephedrine is similar to adrenaline in the
82. Choose the correct statement from the following feature
following about α1 – adrenergic receptor
(a) Potency
agonists
(b) Inability to penetrate blood-brain barrier
(a) Norepinephrine > isoproterenol > epinephrine
(c) Duration of action
(b) Norepinephrine < epinephrine > isoproterenol (d) Producing both α and β adrenergic effects
(c) Epinephrine ≥ norepinephrine >> isoproterenol
90. Continuous exposure of ctecholamine-
(d) Epinephrine > isoproterenol > norepinephrine
sensitive cells and tissues to adrenergic
83. A sympathomimetic amine that acts almost agonists causes a progressive diminition
exclusively by releasing noradrenaline in their capacity to respond, this
from the nerve endings is phenomenon is called as
(a) Ephedrine (b) Dopamine (a) Refractoriness (b) Desensitization
(c) Isoprenaline (d) Tyramine (c) Tachyphylaxis (d) All of the above

84. Low doses of adrenaline dilate the 91. While undergoing a surgical procedure
following vascular bed a patient develops hypotension. Which
of the following drugs can be injected
(a) Cutaneous (b) Mucosal intramuscularly to raise his BP
(c) Renal (d) Skeletal muscle
(a) Noradrenaline (b) Isoprenaline
85. α 2–adrenergic receptors are associated (c) Mephentermine (d) Isoxsuprine
with following except
92. Vasoconstrictors should not be used in
(a) Increase in phospholipase C activity
(a) Neurogenic shock
(b) Increase in potassium channel conductance
(b) Haemorrhagic shock
(c) Decrease in calcium channel conductance (c) Secondary shock
(d) Increase in adenylyl cyclase activity (d) Hypotension due to spinal anaesthesia
DRUGS ACTING ON AUTONOMOUS NERVOUS SYSTEM 93

93. Guanethidine inhibits 100. Phenylephrine


(a) Synthesis of transmitter (a) Mimics the transmitter at post-synaptic
(b) Metabolism of transmitter receptors
(c) Release of transmitter (b) Displaces transmitter from axonal terminal
(d) Displacement of transmitter from axonal (c) Inhibits synthesis of transmitter
terminal (d) None of the above
94. The drug which produces vasoconstriction 101. A β adrenergic blocker that is primarily
despite being an α adrenergic blocker is eliminated unchaged by renal excretion is
(a) Phenoxybenzamine (a) Propranolol
(b) Ergotamine (b) Metoprolol
(c) Dihydroergotoxine (c) Alprenolol
(d) Tolazoline (d) Atenolol
95. Prazosin is an effective antihypertensive 102. In a patient of myocardial infarction, β
while nonselective α adrenergic blockers adrenergic blockers are used with the
are not because following aim/aims:
(a) It is the only orally active α blocker (a) To reduce the incidence of reinfarction
(b) It improves plasma lipid profile (b) To prevent cardiac arrhythmias
(c) It does not concurrently enhance noradrena- (c) Myocardial salvage
line release (d) All of the above
(d) It improves urine flow in males with prostatic
103. Congential dopamine-β β -hydroxylase
hypertrophy
deficiency is characterized by
96. Which of the following is true of sildenafil (a) Absence of norepinephrine and epinephrine
(a) It enhances sexul enjoyment in normal men (b) Increased concentration of dopamine
(b) It delays ejaculation (c) Intact cholinergic innervation
(c) It is indicated only for treatment of erectile (d) All of the above
dysfunction in men
104. The site of action of d-tubocurarine is
(d) It blocks cavernosal α2 adrenoceptors
(a) Spinal internuncial neurone
97. The β-adrenergic blocker which possesses
(b) Motor nerve ending
both β 1 selectivity as well as intrinsic
sympathomimetic activity is (c) Muscle end-plate
(d) Sodium channels in the muscle fiber
(a) Alprenolol (b) Atenolol
(c) Acebutolol (d) Metoprolol 105. The fall in blood pressure caused by d-
tubocuranine is due to
98. Propanolol can be used to allay anxiety
associated with (a) Reduced venous return.
(b) Ganglionic blockade
(a) Chronic neurotic disorer
(c) Histamine release
(b) Schizopherenia
(d) All of the above
(c) Short-term stressful situation
(d) Endogenous depression 106. The neuromuscular blocker having
prominent antivagal action is
99. Propranolol does not block the following
action of adrenaline (a) Pancuronium
(b) Vecuronium
(a) Bronchodilation (b) Lipolysis
(c) Atracurium
(c) Muscle tremor (d) Mydriasis
(d) Gallamine triethiodide
94 MCQs IN PHARMACOLOGY

107. Neuromuscular blocking drugs do not (a) It is a more potent oxytocic


produce central actions because (b) It has antiemetic property
(a) Nicotinic receptors are not present in the (c) It is a less potent a adrenergic blocker but
brain more potent vasoconstrictor
(b) They do not cross the blood-brain barrier (d) It is a more potent a adrenergic blocker and
(c) They are sequestrated in the periphery by less potent vasoconstrictor
tight binding to the skeletal muscles
115. Select the ergot compound which is
(d) They do not ionize at the brain pH primarily used for dementia
108. Pancuronium differs from tubocurarine in (a) Bromocriptine (b) Ergotamine
that (c) Codergocrine (d) Methysergide
(a) It is a depolarizing blocker
116. The ‘amine’ ergot alkaloid differs from
(b) Its action is not reversed by neostigmine ‘amino acid’ ergot alkaloids in that it has
(c) It can cause rise in BP on rapid I.V. injection
(a) High oral bioavailability
(d) It causes marked histamine release
(b) Better CNS penetrability
109. Muscarinic receptors are G-protein (c) Weaker oxytocic action
coupled receptors, causing (d) Strong anti – 5 – HT action
(a) Inactivation of phospholipase C
117. Select the correct statement in relation to
(b) Activation of adenylyl cyclase drug therapy of migraine
(c) Activation of potassium or inhibition of
(a) Simple analgesics like paracetamol are
calcium channels
ineffective in migraine
(d) All of the above
(b) Ergot alkaloids are used for the prophylaxis
110. Postoperative muscle soreness may be as well as treatment of migraine attacks
side effect of the following neuromuscular (c) Use of ergot alkaloids is restricted to severe
blocker or resistant cases
(a) d-tubocurarine (b) Succinylcholine (d) Ergot alkaloids should be given till 24 hours
(c) Pancuronium (d) Atracurium after an attack has subsided

111. Following drug enhances the evoked 118. Ergotamine relieves migraine by
release of acetylcholine (a) Blocking vascular a adrenergic receptors
(a) 4-aminopyridine (b) Vesamicol (b) Blocking vascular 5-HT2 receptors
(c) Magnesium ion (d) None of the above (c) Dilating cranial arterio-venous shunt channels
(d) Constricting cranial vessels and reducing
112. Which of the following muscle relaxants
perivascular neurogenic inflammation
can be used to control spasticity associated
with upper motor neurone paralysis ? 119. Which of the following drugs is most com-
(a) d-tubocurarine (b) Succinylcholine monly used for prophylaxis of migraine ?
(c) Mephenesin (d) Baclofen (a) Ergotamine (b) Propranolol
(c) Methysergide (d) Sumatripitan
113. Following muscarinic agonist is suscepti-
ble to hydrolysis by cholinesterase 120. Stimulation of exocrine glands by
(a) Carbachol (b) Methacholine muscarinic agonist leads to
(c) Bethanechol (d) Muscarine (a) Sweating
(b) Salivation
114. Dihydroergotamine differs from ergota-
(c) Bronchial secretions
mine in the following respect
(d) All of the abvoe
DRUGS ACTING ON AUTONOMOUS NERVOUS SYSTEM 95

121. Datura stramonium (thorn apple) 129. The drug that produces neuromuscular
mainly contains following muscarinic blockade by persistent depolarization is
antagonist (a) D-tubocurarine (b) Gallamine
(a) Atropine (b) Hyoscine (c) Pancuronium (d) Decamethonium
(c) Homatropine (d) Dicyclomine
130. Agents that block neuromuscular
122. β-blockers are contraindicated in transmission of acetyl choline acts via
(a) Prophylaxis of anxiety states (a) Inhibiting acetyl choline synthesis
(b) Hyperthyroidism (b) Inhibiting acetyl choline release
(c) Hypertrophic obstructive cardiomyopathy (c) Inhibiting post-synaptic action of acetyl
(d) Peripheral vascular disease choline
(d) All of the above
123. β-blockers are better avoided in
(a) Bronchial astham 131. The skeletal muscle relaxant causing
significant release of histamine is
(b) Pheochromocytoma
(c) Myocardial infarction (a) Pancuronium (b) Atracurium
(d) Migraine (c) Gallamine (d) D-tubocurarine

124. Preferential beta1 adrenergic blocking 132. The antibiotic that may accentuate the
agent is neuromuscular blockade produced by d-
tubocurarine is
(a) Propranolol (b) Atenolol
(a) Pencillin G (b) Erythromycin
(c) Penbutolol (d) Oxprenolol
(c) Streptomycin (d) Chloramphenicol
125. Prazosin
133. Centrally acting skeletal muscle relaxant
(a) Produces preferential alpha1 adrenergic is
receptor blockade
(a) Carisoprodol (b) Dantrolene
(b) Has a half-life of 24 hours
(c) Gallamine (d) Succinylcholine
(c) Can worsen lipid profile in hypertensive
patients 134. β-blockers
(d) Does not depress the baroreceptor function (a) Relieve symptoms of thyrotoxicosis
126. The organ extremely sensitive to the (b) Should not be used along with carbimiazole
action of atropine is (c) Reduce the synthesis of hormones in the gland
(a) Gastric glands (d) Should not be given before thyroid surgery
(b) Salivary glands 135. Which of the following alpha adrenocep-
(c) Urinary bladder muscle tor blockers is powerful and also has
(d) Heart longer duration of action ?

127. The antimuscarinic agent preferred in the (a) Tolazoline (b) Phenoxybenzamine
management of motion sickness is (c) Phentolamine (d) Chlorpromazine
(a) Atropine methonitrate 136. Heart has beta as sympathetic and va-
(b) Scopolamine gus as parasympathetic drive. If simul-
(c) Homatropine methyl bromide taneously both are blocked by beta
blocker and stropine, the heart will
(d) Ipratropium bromide
(a) Stop
128. Belladonna poisoning is best treated with
(b) Beat with very slow rate
(a) Neostigmine (b) Physostigmine (c) Beat with very fast rate
(c) Acetylcholine (d) Adrenaline (d) Beat at its intrinsic rate, about 100/min
96 MCQs IN PHARMACOLOGY

137. Side effect which is observed only with (c) α1 (d) α1


the use of β blockers and not with other (e) All of the above
sympathetic blocking agents
146. Which of the following drugs has highest
(a) Sedation and drowsiness
alfa agonist activity ?
(b) Congestive heart failure
(a) Epinephrine (b) Norepinephrine
(c) Bronochoconstriction
(c) Ephedrine (d) Amphetamine
(d) Reflex tachycardia
147. Atropine is antagonist to which type of
138. Following agent inhibits the release of
muscarinic
acetyl choline
(a) M1 (b) M2
(a) Magnesium ion (b) Triethylcholine
(c) M3 (d) M4
(c) Vesamicol (d) All of the above
148. Following are irreversible anti-cholinest-
139. What is composition of vagus nerve ?
erases, except
(a) Entirely sensory
(a) Parathion (b) Pyridostigmine
(b) Purely motor
(c) Dyflos (d) Ecothiopate
(c) ¾ sensory ¼ motor
(d) ¼ sensory ¾ motor 149. The order of paralysis with d-tubocuraine
is
140. The ciliary muscle of iris has predominant
(a) Fingers, Neck, Eyes, Limbs, Trunk Respiratory
(a) Sympathetic inervation (b) Neck, Limbs, Face, Eyes, Pharynx, Trunk
(b) Parasympathetic inervation Respiratory
(c) Somatic inervation (c) Fingers, Eyes, Limbs, Neck, Trunk Respiratory
(d) Mixture of all the above (d) Face, Eyes, Fingers, Neck, Limbs, Trunk
Respiratory
141. Following is a short-acting anti-cholinest-
erase (e) None of the above

(a) Neostigmine (b) Pyridostigmine 150. Following is a naturally occurring tertiary


(c) Edrophonium (d) Physostigmine amine anti-cholinesterase
(a) Edrophonium (b) Neostigmine
142. Catecholamine D methyl tranferase is
found in all except (c) Pyridostigmine (d) Physostigmine

(a) Adrenergic neurone 151. Which of the following sympathomimetic


(b) Brain drugs is a non –catecholamine ?
(c) Kidney (a) Adrenaline (b) Noradrenaline
(d) None of the above (c) Isoprenaline (d) Ephedrine
(e) All of the above (f) None of the above
143. β3 receptor is present in
(a) Adipose tissue (b) Smooth muscle 152. Which of the following β – adrenoceptor
blocker is the least lipid soluble ?
(c) Heart (d) All of the above
(a) Propanolol (b) Atenolol
144. b1 receptors are present in
(c) Oxprenolol (d) Metoprolol
(a) Liver (b) Kidney (e) All of the above (f) None of the above
(c) Brain (d) None of the above
153. Propanolol is contraindicated / not
145. Activity of which adrenergic receptor is beneficial in
linked to activation of G proteins
(a) Parkinsonism tremors
(a) β1 (b) β
DRUGS ACTING ON AUTONOMOUS NERVOUS SYSTEM 97

(b) Bronchial asthma (e) All of the above (f) None of the above
(c) Insulin – treated diabetes
161. Following agent reduces the transmitter
(d) All of the above release by blocking nerve terminal
(e) None of the above calcium channels
154. Which of the following adrenoreceptor (a) Botulinum toxin (b) β-bungarotoxin
antagonists has partial agonist activity (c) ω-conotoxin (d) All of the above
and also membrane stabilizing activity
162. Atropine in doses of 2.0mg to 5.0mg may
(a) Propanolol (b) Atenolol cause
(c) Oxprenolol (d) Metoprolol
(a) Increased heart rate
(e) All of the above (f) None of the above
(b) dryness of mouth
155. Which of the following indirectly acting (c) Dilated pupils
sympathomimetics is an uptake-1 (d) Difficulty in micturition
inhibitor ? (e) Reduced intestinal peristalsis
(a) Tyramine (b) Amphetamine (f) All of the above
(c) Ephedrine (d) All of the above (g) None of the above
(e) None of the above
163. Chemically following anti-cholinesterase
156. Which of the following drugs is used in is an alcohol
cardiogenic shock ? (a) Ambenonium (b) Edrophonium
(a) Noradrenaline (b) Adrenaline (c) Ecothiophate (d) None of the above
(c) Isoprealine (d) Dobutamine
164. Which of the following â-adrenoceptor
(e) Salbutamol
blockers has the minimal hepatic
157. Pseudo-cholinesterase is present in biotransformation ?
(a) Membrane (b) Vesicles (a) Atenolol (b) Labetalol
(c) Synaptic cleft (d) Plasma and tissue (c) Metoprolol (d) Propranolol
(e) Sotalol
158. Which of the following drugs is used as
inhalation as a bronchodilator and may 165. Which of the following β –adrenoceptor
be used when cough is a pronounced blockers is the drug of choice for
symptom in asthmatic patient ? treatment of essential tremors ?
(a) Atropine (b) Homtropine (a) Propranolol (b) Nadolol
(c) Ipratropium (d) Tropicamide (c) Timolol (d) Atenolol
(e) All of the above (f) None of the above (e) All of the above (f) None of the above
159. Activation of sympathetic system leads 166. Which of the following â-adrenoceptor
to _________ except blockers is used as anti-glaucoma agent
(a) Contraction of bronchial smooth muscles (a) Propranolol (b) Nadolol
(b) Gluconeogenesis (c) Timolol (d) Atenolol
(c) Glycogenolysis (e) All of the above (f) None of the above
(d) Contraction of radial smooth muscle of iris
167. Concurrent use of atropine will interfere
160. Which of the following antimuscarinic with the anti-glaucoma action of
drugs is short acting mydriatic ? (a) Pilocarpine (b) Physostigmine
(a) Atropine (b) Homatropine (c) Carbachol (d) All of the above
(c) Ipratropium (d) Tropicamide (e) None of the above
98 MCQs IN PHARMACOLOGY

168. To reduce the muscarinic side effects of (c) Without membrane stabilizing effect and
oral forms of cholinesterase inhibitors without partial agonist effect
they should be administered (d) With membrane stabilizing effect, partial
(a) On empty stomach agonist effect and direct vasodilator activity
(b) With food 175. Phenylephrine is useful as
(c) With milk (a) Mydriatic to facilitate examination of retina
(d) One hour after food (b) Decongestant for minor allergic hyperemia
(e) With food or milk of conjunctival membranes
169. Which of the following cholinesterase (c) A drug in localizing the lesion in Horner’s
inhibitors is said to produce more severe Syndrome
muscarinic side effects (d) All of the above
(a) Pyridostigmine (b) Neostigmine (e) None of the above
(c) Ambenonium (d) All of the above 176. d-Amphetamine is not used in
(e) None of the above (a) Narcolepsy
170. Which of the following cholinesterase (b) Hyperkinetic child (specilly attention deficit
inhibitors is preferred in patients disorder)
hypersensitive to the bromide ion ? (c) Migraine
(a) Pyridostigmine (b) Neostigmine (d) Child appetite suppressant
(c) Ambenonium (d) All of the above 177. Following is a tertiary amine anti-mus-
(e) None of the above carinic drug used for Parkinson’s disease

171. Following is a synthetic analogue of (a) Benztropine (b) Dicyclomine


atropine, used as an inhalational drug in (c) Pirenzepine (d) Tropicamide
asthma 178. d-Amphetamine has most prominent
(a) Benztropine (b) Pirenzepine (a) Beta1 effects
(c) Ipratropium (d) All of the above (b) Beta2 effects
172. All the cholinomimetic alkaloids except (c) Metabolic effects
one which is a choline ester (d) CNS excitatory effects
(a) Pilocarpine (b) Carbachol (d) None of the above
(c) Muscarine (d) Arecoline 179. Phenylephrine has most prominent
173. Which one of the following β – blockers (a) Alpha1 effects
has reduced risk of bronchospasm ? (b) Beta1 effects
(a) Propranolol (b) Atenolol (c) Beta2 effects
(c) Pindolol (d) None of the above (d) Metabolic effects
(e) CNS excitatory effects
174. Metoprolol is a cardioselective beta block-
er (relatively selective beta1 – blocker) 180. Following is a tertiary amine anti-mus-
(a) With membrane stabilizing effect and with carinic drug mainly used for mydriatic or
partial agonist effect cycloplegic action
(b) Without membrane stabilizing effect and with (a) Benztropine (b) Dicyclomine
partial agonist effect (c) Pirenzepine (d) Trpicamide
DRUGS ACTING ON AUTONOMOUS NERVOUS SYSTEM 99

ANSWERS
1. d 2. c 3. c 4. b 5. a 6. d
7. c 8. b 9. c 10. e 11. d 12. a
13. b 14. b 15. c 16. a 17. b 18. b
19. b 20. e 21. c 22. c 23. b 24. b
25. a 26. b 27. d 28. a 29. e 30. c
31. d 32. e 33. a 34. b 35. d 36. c
37. e 38. b 39. b 40. a 41. a 42. a
43. d 44. a 45. a 46. c 47. e 48. b
49. e 50. b 51. a 52. a 53. a 54. b
55. c 56. e 57. c 58. d 59. b 60. d
61. b 62. d 63. b 64. b 65. b 66. b
67. b 68. c 69. b 70. d 71. c 72. a
73. a 74. d 75. c 76. a 77. b 78. d
79. a 80. a 81. c 82. c 83. d 84. d
85. d 86. d 87. c 88. a 89. d 90. d
91. c 92. c 93. c 94. b 95. c 96. c
97. c 98. c 99. d 100. a 101. d 102. d
103. d 104. c 105. d 106. d 107. b 108. c
109. c 110. b 111. a 112. d 113. b 114. d
115. c 116. a 117. c 118. d 119. b 120. d
121. b 122. d 123. a 124. b 125. a 126. b
127. b 128. b 129. d 130. d 131. d 132. c
133. a 134. a 135. b 136. d 137. a 138. a
139. c 140. b 141. c 142. a 143. a 144. b
145. e 146. a 147. a 148. b 149. c 150. d
151. d 152. b 153. d 154. c 155. b 156. d
157. d 158. c 159. a 160. d 161. c 162. f
163. b 164. a 165. a 166. c 167. d 168. e
169. b 170. c 171. c 172. b 173. b 174. c
175. d 176. c 177. a 178. d 179. a 180. d
100 MCQs IN PHARMACOLOGY

EXPLANATIONS FOR THE ANSWERS

2. d The autonomic nervous system consists of cholineacetyltransferase.


sympathetic, parasympathetic and enteric Acetylcholinesterase splites the acetylcholine into
nervous systems. Hence, the ANS is also called choline and acetate.
as visceral, vegetative or involuntary nervous 46. c There are three types of nicotinic receptors –
system. muscle type, ganglion type and CNS type.
9. c The sympathetic preganglionic neurons have Trimethaphan is an antagonist of ganglion type
their cell bodies in the lateral horn of the grey nicotinic receptors.
matter of thoracic and lumbar segments of the
Tubocurarine is an antagonist of muscle type
spinal cord. The fibers leave the spinal cord in
and α-bungarotoxin is an antagonist of CNS
the spinal nerve as thoracolumbar sympathetic
type nicotinic receptor.
outflow.
52. a Oxytremorine is a selective agonist of M1
13. b In male reproductive system, the sympathetic
muscarinic receptors present in autonomic
nervous system is responsible for ejaculation
ganglia.
(α-receptors), and parasympathetic nervous
system is responsible for erection via. Muscarinic Pirenzepine is an antagonist of these receptors.
receptors. 55. c Coversion of tyrosine to dopa, dopa to dopamine
16. a Sympathetic system is active and parasympathetic and dopamine to 3, 4 dihydorxyphenyl acetic
system is quiescent in ‘fight or flight’ response acid occurs in the cytoplasm.
i.e. stress. Dopamine α hydorxylase, which converts
Parasympathetic system predominates during dopamine to norepinephrine, is present in
satiation and repose i.e. ‘rest and digest’ storage vesicles. Hence conversion of dopamine
response. to norepinephrine occurs in storage vesicles and
not cytoplasm.
21. c See explanation of Q. 16
62. d Actions of norepinephrine and epinephrine are
30. c Sympathetic system increases rate of SA node,
terminated by:
cause contraction of coronary arteries and
constriction of sphincters in GIT. • Reuptake into nerve terminals.
Sympathetic system relaxes the smooth muscles • Dilution by diffusion out of the junctional cleft
of GIT, thus decreases the motility. and uptake by extraneuronal sites.
31. d If a nerve is cut and its terminal are allowed to • Metabolic transformation by monoamine oxide
degenerate, the organ or tissue supplied by it and catechol –O-methyltransferase.
becomes supersensitive to the neurotransmitter 66. b Yohimbine is an antagonist of α2-receptors
released by the nerve terminal. This is called as present in nerve terminals, platelets and
denervation super-sensitivity and it involves pancreatic β-cells. Clonidine is an agonist of
various mechanisms such as proliferation of these receptors.
receptors, loss of mechanism of transmitter 71. c α1 – Receptors are coupled with Gq G- proteins
removal and increased post-junctional and the biochemical affect is increased in either
responsiveness. phospholipase C, D or A2 activity and/or
39. b At rest, the interior of the typical mammalian opening of calcium channels.
neuronal axon potential is approximately – 70 79. a The affinity of β1 – adrenergic receptors can be
mV. It is essentially a diffusion potential based shown as :
on high potassium inside and high sodium and Isoproterenol > Epinephrine = Norepinephrine
chloride outside the axon. Axonal membrane is
The specific agonist of this receptor is
relatively more permeable to potassium than
dobutamine and specific antagonist is
sodium and chloride ions.
metoprolol.
40. a Acetylcholine is synthesized from acetyl-CoA 82. c The affinity of α1 – adrenergic receptors can be
and choline through the catalytic action of shown as:
DRUGS ACTING ON AUTONOMOUS NERVOUS SYSTEM 101

Epinephrine ≥ Norepinephrine >> Isoproterenol resistant than acetylcholine, is susceptible to


Phenylephrine is a specific agonist and prazocin hydrolysis by cholinesterase.
is a specific antagonist of this receptor. Carbachol and bethanechol are completely
85. d Activation of α 2 – adrenergic receptors is resistant to hydrolysis by cholinesterase.
associated with: Muscarine is not metabolized by cholinesterase.
• Increase in phospholipase activity. 120. d Muscarinic agonists stimulate the secretory
• Increase in potassium channel conductance. activity of sweat, lacrimal, salivary and
• Decrease in calcium channel conductance nasopharyngeal glands and hence lead to
increased sweating, salivation and bronchial
• Decrease (and not increase in adenylyl cyclase
secretions.
activity.
121. b Datura stramonium mainly contains atropine.
90. d Refractoriness, desensitization or tachchyphylaxis
describes the same phenomenon. Because of this, Hyoscine (scopolamine)is mainly found in
the duration of action and therapeutic efficacy Hyoscymus niger.
of catecholamines and other agents is Homatropine is a semisynthetuic derivative.
significantly limited. Dicyclomine is a synthetic derivative.
93. c Guanethidine inhibits the release of transmitter. 130. d Blockers of neuromuscular transmission of
Another agent, which also inhibits the release acetylcholineact via various mechanisms:
of transmitter, is bretylium. • Inhibition of acetylcholine synthesis.
Synthesis of transmitter is inhibited by α-methyl • Inhibition of acetylcholine release.
tyrosine. • Inhibition of postsynaptic action of acetylcholine.
Metabolism of transmitter is inhibited by 138. a Magnesium inhibits the entry of calcium into
pargyline and tranylcypromine. Amphetamine the nerve terminal and thus inhibits the
and tyramine cause displacement of transmitter exocytosis and hence release of acetylcholine.
from axon terminals.
Triethylcholine inhibits the transport of choline
100. a Phenylephrine is an agonist of α1-adrenergic into the nerve terminal.
receptor and it mimics the transmitter at post-
Vesamicol inhibits the transport of acetylcholine
synaptic receptors, producing sympathomimetic
into synaptic vesicles.
effect.
141. c Edrophonium is a short-acting anticholinest-
103. d Congenital dopamine β hydroxylase deficiency
erase. It is mainly used in the diagnosis of my-
is characterized by absence of norepinephrine
asthenia gravis.
and epinephrine, increased concentration of
dopamine and intact cholinergic innervation. Neostigmine, pyridostigmine and physostigmine
These patients suffer from severe postural are medium duration anticholinesterase.
hypotension and other symptoms. 148. b Pyridostigmine is a medium duration anticho-
109. c Muscarinic receptors are G protein coupled linesterase. Parathion, dyflos and ecothiopate
receptors, causing: are irreversible anticholinesterase.
• Activation of potassium or inhibition of calcium 150. d Physiostigmine is a naturally occurring tertiary
channels. amine anticholinesterase.
• Activation of phospholipase C Edrophonium, neostigmine and pyridostigmine
are quaternary ammonium compounds.
• Inhibition of adenylyl cyclase
157. d Pseudocholinesterase (or butyrylcholinesterase)
111. a 4-Aminopyridine enhances the evoked release
is widely distributed in tissues e.g. liver, skin, brain
of acetylcholine.
and gastrointestinal smooth muscles. It is also
Vesamicol blocks the transport of acetylcholine present in soluble form in plasma. However, it is
into storage vesicles. not present in synaptic cleft, vesicles or membrane.
Magnesium ions block the calcium entry and 159. a Activation of sympathetic system lead to;
thus inhibit the release of acetylcholine. • Relaxation (and not contraction) of bronchial
113. b Methacholine, although three times more smooth muscles.
102 MCQs in Pharmacology

• Gluconeogenesis. and is used as an inhalational drug in the


• Glycogenolysis treatment of asthma.
• Contraction of radial smooth muscle of iris. Benzotropine is efficacious in preventing
161. c ω-Conotoxin is an inhibitor of nerve terminal dystonia or parkinsonian symptoms.
calcium channels and thus reduces the Pirenzepine is used as an anticular agent.
transmitter release. 177. a Benzotropine is the only antimuscarinic drug
Botulinum toxin and β-bungarotoxin reduce with profound CNS effects and hence it is used
transmitter release by cleaving specific proteins for the treatment of the Parkinson’s disease.
involved in the exocytosis.
180. d Tropicamide is mainly used for mydriatic or
163. b Edrophonium is an anticholinesterase and is cycloplegic action.
chemically an alcohol. It has - OH group
attached to a benzene ring. Ambenonium is a Dicyclomine is antispasmodic.
bis-quaternary compound and ecothiophate is Pirenzepine is anticular agent.
an organophosphate anti-cholinesterase. Benzetropine is used in the treatment of
171. c Ipratropium is a synthetic analogue of atropine Parkinson’s disease.
VITAMINS & MINERALS 103

CHAPTER 4
VITAMINS & MINERALS

1. Thiamine is vitamin (a) Erythropoietin : Macrocytic anemia


(a) B1 (b) B2 (b) Filgrastim : Thrombocytopenia due to
(c) B6 (d) B12 myelocytic leukemia
(c) Iron dextran : Severe macrocytic anemia
2. The iron stored in intestinal mucosal cells
(d) Ferrous sulfate : Microcytic anemia of
is complexed to
pregnancy
(a) Ferritin (b) Intrinsic factor (e) Folic acid : Hemochromatosis
(c) Oprelvekin (d) Transcobalamin II
7. Conversion of methionine to cysteine
(e) Transferrin
depends on vitamin
3. Which of the following is most likely to (a) B1 (b) B2
be required by a 5-year-old boy with
(c) B6 (d) B12
chronic renal in sufficiency?
(a) Erythropoietin (b) G-CSF 8. Avidin, a protein found in egg white is
an antagonist of
(c) Interleukin - 11 (d) Stem cell factor
(e) Thrombopoietin (a) Biotin (b) Pantothenic acid
(c) Choline (d) Pyridoxal
4. in adults, approximately ______ mg of
thiamine per day is completely degraded 9. All of the following are important
by the tissue functions of magnesium (Mg)
except
(a) 0.01 (b) 0.1
(c) 1 (d) 10 (a) Nerve conduction
(b) Phospholipid synthesis
5. The drug of choice for the management
(c) Muscle contractility
of osteoporosis caused by high-dose use
(d) Carbohydrate, fat, and electrolyte metabolism
of glucocorticoids is
(a) Alendronate (b) Calcitonin 10. Factors likely to cause an increase in the
(c) Mestranol (d) Oxandrolone blood urea nitrogen (BUN) level include
(e) Vitamin D (a) Intramuscular (IM)injection of diazepam
(Valium)
6. Which of the following drugs is correctly
(b) Severe liver disease
associated with its clinical application?
(c) Chronic kidney disease
104 MCQs IN PHARMACOLOGY

11. Physiologically carnitine plays following 18. The gut controls the entry of ingested iron
role in the body of
(a) Important for oxidation of fatty acids (a) Regulating the availability of apoferritin
(b) Decreases aerobic metabolism of carbohy- which acts as the carrier of iron across the
drates mucosal cell
(c) Decreases rate of oxidative phosphorylation (b) Regulating the turnover of apoferritin-ferritin
(d) All of the above interconversion in the mucosal cell
(c) Complexing excess iron to form ferritin which
12. Patients receiving iron therapy should be remains stored in the mucosal cell and is shed
warned about
off
(a) Dizziness (d) Regulating the number of transferring
(b) Ringing in the ears receptors on the mucosal cell
(c) Danger of sunlight
19. The percentage of elemental iron
(d) Blackening of the stool hydrated ferrous sulfate is
(e) Paresthesia
(a) 5% (b) 10%
13. Therapeutically vitamin B1 has been (c) 20% (d) 33%
employed most successfully in the
treatment of 20. In isolated fibroblast or epithelial cells,
retinoids enhance the synthesis of
(a) Microcytic anemia
following protein
(b) Pellagra
(a) Fibronectin
(c) Scurvy
(b) Collagenase
(d) Beriberi
(c) Certain species of keratin
(e) Macrocytic anemia
(d) All of the above
14. Magnesium ion is necessary in
21. The side effect which primarily limits
(a) Stimulating enzyme systems acceptability of oral iron therapy is
(b) Muscular contraction
(a) Epigastric pain and bowel upset
(c) Nerve conduction
(b) Black stools
(d) All of the above
(c) Staining of teeth
(e) None of the above
(d) Metallic taste
15. The following derivatives of retinal shows
the greatest biological potency than 22. Iron sorbitol-citric acid differs from iron
others dextran in that

(a) 9-Cis-retinoic acid (b) All-trans-retinoic acid (a) It cannot be injected i.v.
(c) All-trans-retinol (d) 11-Cis-retinal (b) It is not excreted in urine
(c) It is not bound to transferritin in plasma
16. The drug used for controlling tetany is
(d) It produces fewer side effects
(a) Intravenous diazepam
23. Which of the following is true about iron
(b) Intramuscular vitamin D
therapy ?
(c) Intravenous calcium gluconate
(a) Haemoglobin response to intramuscular iron
(d) Intravenous calcitonin
is faster than with oral iron therapy
17. Absorption of oral iron preparations can (b) Iron must be given orally except in pernicious
be facilitated by coadministering anaemia
(a) Antacids (b) Tetracyclines (c) Prophylactic iron therapy must be given
(c) Phosphates (d) Ascorbic acid during pregnancy
VITAMINS & MINERALS 105

(d) Infants on breast feeding do not require (c) Conversion of homocysteine to methionine
medicinal iron (d) All of the above
24. Concentrations of retinal in plasma in 31. Vitamin K is indicated for the treatment
excess of _____ µ g/dl usually are
of bleeding occurring in patients
diagnostic of hypervitaminosis A
(a) Being treated with heparin
(a) 10 (b) 50
(b) Being treated with streptokinase
(c) 100 (d) 200
(c) Of obstructive jaundice
25. Megaloblastic anaemia occurs in
(d) Of peptic ulcer
(a) Vitamin B12 but not folic acid deficiency
32. Menadione (Vitamin K3)
(b) Folic acid but not Vitamin B12 deficiency
(c) Either Vitamin B12 or folic acid deficiency (a) Can cause hemolysis in patients with G-6-PD
(d) Only combined Vitamin B12 + folic acid deficiency
deficiency (b) Is given in large doses in patients with severe
liver disease
26. The daily dietary requirement of Vitamin
B12 by an adult is (c) Is useful to prevent haemorrhagic disease of
the newborn
(a) 1–3 µg (b) 50–100 µg
(d) Is the preparation of choice to antagonize
(c) 0.1–0.5 µg (d) 1–3 µg
the effect of warfarin overdose
27. Which of the following factor(s) is/are
33. Vitamin K promotes the hepatic biosynthesis
required for the absorption of Vitamin B12
ingested in physiological amounts ? of following blood clotting factor

(a) Gastric acid (a) Factor I (b) Factor II


(b) Gastric intrinsic factor (c) Factor VIII (d) All of the above
(c) Transcobalamine 34. folinic acid is principally used
(d) Both (a) and (b)
(a) In pernicious anaemia
28. Hydroxocobalamine differs from cyano- (b) In megaloblastic anaemia secondary to
cobalamine in that Vitamin B12
(a) It is more protein bound and better retained (c) Along with methotrexate therapy
(b) It is beneficial in tobacco amblyopia (d) In treatment of folic acid deficiency
(c) It benefits haematological but not neurological
manifestations of Vit B12 deficiencey 35. Penicillamine
(d) Both (a) and (b) (a) Is effective orally
(b) Can cause anaphylactic reactions in patients
29. Megaloblastic anemia is caused by
allergic to penicillin
deficiency of
(c) Is safe in pregnancy
(a) Iron (b) Vitamin B12
(d) Is not effective in lead poisoning
(c) Vitamin C (d) All of the above
30. Vitamin B12 is a required co-factor for 36. Succimer
the following reaction (a) Can significantly mobilize essential metals
(a) Conversion of methylmalonyl-CoA to succinyl- (b) Produces less toxicity than Dimercaprol
CoA (c) Is ineffective orally
(b) Conversion of 5-CH3-H4-folate to H4-folate (d) Is contraindicated in children
106 MCQs IN PHARMACOLOGY

Answer
1. c 2. a 3. a 4. c 5. a 6. d
7. c 8. a 9. b 10. c 11. a 12. d
13. d 14. d 15. c 16. c 17. d 18. c
19. c 20. a 21. a 22. a 23. c 24. c
25. c 26. a 27. d 28. d 29. b 30. d
31. c 32. a 33. b 34. c 35. a 36. b

EXPLANATIONS FOR THE ANSWERS


1. a Thiamine is vitamin B1 and was the first member and reduce the synthesis of collagen and certain
of vitamin B complex to be identified. Vitamin B6 species of keratin. These effects are mediated
– Pyridoxine, pyridoxal and pyridoxamine. Medi- by change in the nuclear transcription.
cally used vitamin B12 is hydroxycobalamine. Retinoic acid is more potent than retinal in
Vitamin B2 is riboflavin mediating these effects.
4. c In adults, approximately 1 mg of thiamine per 24. c Concentrations of retinal in plasma in excess of
dayis completely degraded by the tissues and 100µg/dl. usually are diagnostic of hypervitminosis
1mg is roughly the minimal daily requirement A. Such hypervitaminosis is generally seen during
of thiamine. the therapeutic use of retinoids in the treatment of
7. c Conversion of methionine to cysteine depends skin disorders.
on vitamin B6. Vitamin B1, B2 and B12 do not 29. b Megaloblastic anemia is caused by deficiency
play any role in this conversion. Vitamin B6 is of vitamin B12. It is characterized by macocytic
also involved in various metabolic transforma- anemia, mild to moderate leukopenia and/or
tions of amino acids e.g. decarboxylation, tran- thrombocytopenia, hypercellular bone marrow
samination and racemization. with megaloblastic maturation or erythroid and
8. a Avidin, a protein found in egg white, is an other precursor cells.
antagonist of biotin. Avidin is a glycoprotein 30. d Vitamin B12 is a cofactor for various biochemical
and it binds with biotin with great affinity and
reactions:
thus prevents its absorption.
• Conversion of methylmalonyl CoA to succinyl CoA.
11. a Carnitine has several physiological roles:
This reaction requires deoxyadenosylcobolamin
• It is important for oxidation of fatty acids. as a cofactor.
• It increases aerobic metabolism of carbohydrates. • Conversion of 5-methyl tetrahydrofolate to
• It increases rate of oxidative phosphorylation. tetrahydro-olate and conversion of homocysteine
• It enhances the excretion of certain organic to methionine. These two reactions use
acids. methylcobolamine as a cofactor.
15. c Of all known derivatives of retinal, all-trans- 33. b Vitamin K promotes the hepatic biosynthesis of
retinol (and its aldehyde, retinal) has the greatest factor II (prothrombin)and also factors VII, IX
biological potency. and X.
20. a In isolated fibroblasts and epithelial cells, Vitamin K does not play important role in the
retinoids enhance the synthesis of fibronectin biosynthesis of factors I and VIII.
ANALGESICS & ANTIPYRETICS 107

CHAPTER 5
ANALGESICS & ANTIPYRETICS

1. â-Aminoethylimidazole is? (d) Slow-reacting substance of anaplylaxis


(a) 2-methyl histamine (SRS-A)
(b) Histamine (e) Thromboxane A2 or its analogs
(c) 2-pyridyl ethylamine 5. Which of the following is a reversible
(d) 4-methyl histamine inhibitor of platelet cyclooxygenase?

2. Agents that often cause vasoconstriction (a) Alprostadil (b) Aspirin


include all of the following except (c) Ibuprofen (d) LTC4
(a) Angiotensin II (b) Methysergide (e) Misoprostol
(c) PGF2α (d) Prostacyclin 6. Vasodilation by prostaglandins involves
(e) Thromboxane (a) Arterioles
3. Inflammation is a complex tissue reaction (b) Precapillary sphincters
that includes the release of cytokines, (c) Postcapillary venules
leukotrienes, prostaglandins, and (d) All of the above
peptides. Prostaglandins involved in
inflammatory processes are produced 7. Fentanyl transdermal patches have been
from arachidonic acid by used postoperatively to provide trans-
dermal analgesia. The most dangerous
(a) Cyclooxygenase 1
adverse effect of this mode of adminis-
(b) Cyclooxygenase 2 tration is
(c) Glutathione – S – transferase
(a) Cutaneous reactions
(d) Lipoxygenase
(b) Diarrhea
(e) Phospholipase A2
(c) Hypertension
4. A 60-year-old woman has glaucoma (d) Relaxation of skeletal muscle
following cataract surgery. Which of the (e) Respiratory depression
following can be used to reduce
intraocular pressure? 8. Opioid analgesics are either contraindicated
or must be used with extreme caution in
(a) Leukotriene LTD4 or its analogs
several clinical situations. For morphine,
(b) Prostaglandin E2 or its analogs such situations do not include
(c) Prostaglandin F2α or its analogs
(a) Aqueous diffusion
108 MCQs IN PHARMACOLOGY

(b) Aqueous hydrolysis 15. The main advantage of ketorolac over


(c) Lipid diffusion aspirin is that ketorolac
(d) Pinocytosis or endocytosis (a) Can be combined more safely with an opioid
(e) Special carrier transport such as codeine
(b) Can be obtained as an over-the-counter agent
9. Following is an example of paraami-
(c) Does not prolong the bleeding time
nophenol NSAID
(d) Is available in a parenteral formulation
(a) Diclofenac (b) Acetaminophen that can be injected intramuscularly or
(c) Piroxicam (d) Celecoxib intravenously
10. This drug, which does not activate opioid (e) Is less likely to cause acute renal failure in
receptors, has been proposed as a patients with some preexisting degree of
maintenance drug in treatment programs renal impairment
for opioid addicts; a singly oral dose will 16. A 45-year-old surgeon has developed
block the effects of injected heroin for up symmetric early morning stiffness in her
to 48 hours hands. She wishes to take a nonsteroidal
(a) Amphetamine (b) Buprenorphine anti-inflammatory drug to relieve these
(c) Naloxone (d) Naltrexone symptoms and wants to avoid gastrointes-
tinal side effects. Which one of the follow-
(e) Propoxyphene
ing drugs is most appropriate?
11. Which one of the following statements (a) Aspirin (b) Celecoxib
about dextromethorphan is accurate? (c) Ibuprofen (d) Indomethacin
(a) Activates kappa receptors (e) Piroxicam
(b) Analgesia equivalent to pentazocine
17. Following is an example of preformed
(c) Highly effective antiemetic and not lipid derived mast cell mediator
(d) Less constipation than codeine of inflammatory process
(e) Use requires a prescription (a) LTC4 (b) PGD2
12. Which one of the following effects does (c) PAF (d) Histamine
not occur in salicylate intoxication ?
18. The toxicity spectrum of aspirin does not
(a) Hyperventilation include
(b) Hypothemia (a) Increased risk of encephalopathy in children
(c) Metabolic acidosis with viral infections
(d) Respiratory alkalosis (b) Increased risk of peptic ulcers
(e) Tinnitus (c) Hyperprothrombinemia
13. Which one of the following drugs is not (d) Metabolic acidosis
useful in dysmenorrhea? (e) Respiratory alkalosis
(a) Aspirin (b) Colchicine 19. Accidental poisonings are common with
(c) Ibuprofen (d) Rofecoxib both aspirin and ibuprofen, two OC drugs
(e) Naproxen available in tasty chewable tablets. In
cases of overdose, aspirin is more likely
14. Following gold compound is generally than ibuprofen to cause
administered orally (a) Autonomic Instability
(a) Aurothioglucose (b) Hepatic necrosis
(b) Auronafin (c) Metabolic acidosis
(c) Gold sodium thiomalate (d) Thrombocytopenia
(d) All of the above (e) Ventricular arrhythmias
ANALGESICS & ANTIPYRETICS 109

20. A drug that decreases blood pressure and (a) Oxidation of the ester
has analgesic and spasmolytic effects (b) Reduction of the carboxylic acid
when given intrathecally is (c) Hydrolysis of the ester
(a) Atenolol (b) Clonidine 27. Diamprit is an agonist of ______ receptors,
(c) Morphine (d) Nitroprusside except
(e) Prazosin (a) H1 (b) H2
21. Cyclooxygenase-1 and –2 are responsible (c) H3 (d) All of the above
for 28. Which of the following enzymes is
(a) The synthesis of prostaglandins from ultimately responsible for the production
arachidonate of prostaglandins associated with
(b) The synthesis of leukotrienes from arachidonate inflammatory reactions?
(c) The conversion of ATP to cAMP (a) Phospholipase
(d) The metabolic degradation of cAMP (b) Lipoxygenase
(e) The conversion of GTP to cGMP (c) Cyclooxygenase-I
(d) Cyclooxygenase II
22. Following agent is generally used in (e) Xanthine oxidase
allergic rhinitis
29. Which of the following prostaglandin
(a) Beclomethasone (b) Fluticasone
analogs is used specifically for the
(c) Triamcinolone (d) All of the above treatment of NSAID induced
gastrointestinal ulcertation?
23. The primary objective for designing drugs
that selectively inhibit COX – 2 is to (a) Alprostadil (b) Misoprostol
(a) Decrease the risk of nephrotoxicity (c) Carboprost (d) Dinoprostone
(e) Epoprostenol
(b) Improve anti-inflammatory effectiveness
(c) Lower the risk of gastrointestinal toxicity 30. Which of the following compounds is
(d) Reduce the cost of treatment of rheumatoid most likely to lower circulating levels of
arthritis leukotrienes?
(e) Selectively decrease thromboxane A2 without (a) Zileuton (b) Montelukast
effects on other eicosanoids (c) Carprofen (d) Aspirin
(e) Allopurinol
24. A newborn was diagnosed as having a
congenital abnormality that resulted in 31. The action of aspirin that results in its
transposition of her great arteries. While greater efficacy as an antithrombotic
preparing the infant for surgery, the (anti-platelet) drug is its ability to
medical team needed to keep the ductus (a) Inhibit lipoxygenase as well as cyclooxygenase
arteriosus open. They did this by infusing (b) Selectively inhibit cyclooxygenase I
(a) Cortisol (b) Indomethacin (c) Inhibit leukocyte migration
(c) Ketorolac (d) Misoprostol (d) Promote uric acid excretion
(e) Tacrolimus (e) Acetylate cyclooxygenase

25. Acetyl salicylic acid is soluble in 32. Which of the following drugs may be
effective in the treatment of gouty
(a) An aqueous base (b) Water
arthritis by acting by two separate and
(c) An aqueous acid distinct mechanisms?
26. Decomposition of the acetyl salicylic acid (a) Allopurinol (b) Probenecid
at room temperature most likely would (c) Colchicine (d) Indomethacin
occur by (e) Sulfinpyrazone
110 MCQs IN PHARMACOLOGY

33. Acute or chronic colchicine toxicity may (a) Contraction of gastrointestinal muscles
be identified by which of the following (b) Decreased muscle tone
signs/symptoms? (c) Decreased peristalsis
(a) Alopecia (d) All of the above
(b) Blood dyscrasias
39. A FDA – approved ingr edient for
(c) Severe gastrointestinal upset protection against painful sensitivity of
(d) All of the above the teeth due to cold, heat, acids, sweets
(e) None of the above or contact is

34. Patients taking chronic doses of non- (a) Dicalcium phosphate


selective nonsteroidal anti-inflammatory (b) Sodium lauryl sulfate
drugs (NSAIDs) should periodically be (c) 5% potassium nitrate
screened for which of the following (d) Zinc chloride
toxicities? (e) Calcium carbonate
(a) Nephrotoxicity
40. Which local anesthetic should be used to
(b) Peripheral neuropathy treat symptoms of pain, itching, burning,
(c) Cardiotoxicity and disconfort in patients with an
(d) All of the above established lidocaine allergy?
(e) None of the above (a) Tetracaine (b) Dibucaine
35. Which of the following medications would (c) Pramoxine (d) Benzocaine
represent arthritis therapy that is least 41. What is the most commom sign/symptom
likely to cause gastric ulceration? of hemorrhoids?
(a) Aspirin (b) Acetaminophen (a) Bleeding (b) Pain
(c) Piroxicam (d) Meclofenamate (c) Seepage (d) Pruritus
(e) Rofecoxib
42. Which of the following agents is
36. In addition to their ability to decrease designated as a safe and effective
inflammatory prostaglandin synthesis, analgesic, anesthetic and antipruritic by
some non-steroidal anti-inflammatory the Food and Drug Administration?
drugs (NSAIDs) may owe part of their (a) Witch hazel (b) Juniper tar
effects to their ability to
(c) Hydrocortisone (d) Phenylephrine
(a) Inhibit leukocyte migration
43. A 65-year-old is interested in taking gink-
(b) Inhibit leukotriene synthesis
go. Which of the following statements is
(c) Stabilize lysosmal membranes correct regarding ginkgo?
(d) All of the above
(a) Ginkgo is contraindicated in diabetes and
(e) None of the above pregnancy
37. The termination of heparin activity by (b) There is a drug-herb interaction between
protamine sulfate is due to ginkgo and aspirin
(a) A chelating action (c) Toxic effects include hypertension and
cardiac arrest
(b) The inhibition of gastrointestinal absorption
(d) There is a drug-herb interaction between
of heparin
ginkgo and phenelzine
(c) The displacement of heparin-plasma protein
(e) Ginkgo is contraindicated in patients with
binding
gallstone pain
(d) An acid-base interaction
(e) The prothrombin-like activity of protamine 44. All of the following medications should
not be used routinely in pregnant patients
38. In gastrointestinal tract, serotonin causes during the third trimester except
ANALGESICS & ANTIPYRETICS 111

(a) Acetaminophen (4) Aspirin may slow this drug’s rate of excretion
(b) Nonsteroidal anti-inflammatory drugs (5) Vision should be monitored every 3–6
(c) Warfarin months
(d) Lithium 48. All of the following statements concerning
(e) Aspirin an acute gouty arthritis attack are correct
except
45. Which of the following statements best
describes the usual course of rheumatoid (a) The diagonosis of gout is assured by a good
arthritis? therapeutic response to colchicines because
no other form of arthritis responds to this drug
(a) It is an acute exacerbation of joint pain treated
(b) To be assured of the diagnosis, monosodium
with short-term anti-inflammatory therapy
urate crystals must be identified in the
(b) It is a chronic disease characterized by acute synovial fluid of the affected joint
changes within nonsynovial joints
(c) Attacks frequently occur in the middle of the
(c) It is an acute disease that is characterized by night
rapid synovial changes due to inflammation
(d) An untreated attack may last up to 2 weeks
(d) It is a chronic disease characterized by acute (e) The first attack usually involves only one joint,
exacerbations followed by remissions with most frequently the big toe (first metatar-
consequences associated with chronic sophalangeal joint)
inflammatory changes
(e) It is a joint disease characterized by a αis an example of eicosanoids
49. TNF-α
marked loss of calcium from the bones and (a) Interleukins
a resultant thinning of the bones (b) Cytotoxic factors
46. Match the drug characteristic with the (c) Interferons
appropriate agent. (d) Colony stimulating factors
(a) Corticosteroids (1) Persistent platelet 50. Potential adverse effects associated with
function effect aspirin include all of the following except
(b) Lbuprofen (2) Oral form of gold (a) Gastrointestinal ulceration
(c) Aspirin (3) Given on an empty (b) Renal dysfunction
stomach
(c) Enhanced methotrexate toxicity
(d) Aurancfin (4) May be used intra-
(d) Cardiac arrhythmias
articularly
(e) Hypersensitivity asthma
(e) Penicillamine (5) May cause drowsiness
51. All of the following facts are true about
47. Match the phrase below with the appro-
non-steroidal anti-inflammatory drugs
priate agent used to treat rheumatoid
(NSAIDs) except
arthritis.
(a) They are antipyretic
(a) Indomethacin
(b) There is a celling effect to their analgesia
(b) Aspirin
(c) They can cause tolerance
(c) Hydroxychloroquine
(d) They do not cause dependence
(d) Methotrexate
(e) They are anti-inflammatory
(e) Cyclophosphamide
(1) May cause hemorrhagic cystitis 52. Which of the following narcotics has the
longest duration of effect?
(2) May cause more severe central nervous
system (CNS)adverse effects than other (a) Methadone
nonsteroidal anti-inflammatory drugs (b) Controlled-release morphine
(NSAIDS) (c) Levorphanol
(3) Enteric – coated form may be useful in (d) Transdermal fentanyl
treating some patients (e) Dihydromorphone
112 MCQs IN PHARMACOLOGY

53. Cylcoocygenase-II specific inhibitors block 60. Eicosanoids are a group of _____ carbon
the following unsaturated fatty acids
(a) Production of cytoprotective prostaglandins (a) 20 (b) 10
(b) Tumor necrosis factor - α (c) 25 (d) 35
(c) Production of prostaglandins responsible for
61. Which of the following durgs is a
pain and inflammation
monamine oxidase inhibitor, but is used
54. The emetic action of morphine is due to to treat hypertension?
(a) Irritation of gastrointestinal tract (a) Tranylcypromine (Parnate)
(b) Stimulation of cerebral cortex (b) Reserpine
(c) Stimulation of medullary vomiting center (c) Propranolol (Inderal)
(d) Stimulation of emetic chemoreceptor trigger (d) Pargyline (Eutonyl)
zone (e) Imipramine (Tofranil)
(e) None of the above
62. Autacoids differ from hormones in that
55. Colchicine is used mainly to treat (a) Autacoids are involved only in the causation
(a) Gout (b) Arthiritis of pathological states
(c) Diabetes (d) Carcinomas (b) Autacoids do not have a specific cell/ tissue
(e) High blood pressure of origin
(c) Autacoids generally act locally at the site of
56. Which type of patient is most likely to generation and release
hypersensitive to aspirin?
(d) Both (b) and (c)
(a) Intrinsic asthamatic
63. Which of the following eicosanoids is
(b) Extrinsic asthamatic
generated through the lipozygenase
(c) Chronic bronchitic pathway ?
(d) Patient with viral injection
(a) Prostaglandin E2 (b) Thromboxane A2
(e) Both (c) and (d)
(c) Prostacyclin (d) Leukotriene C4
57. Codeine acts as a cough sedative by
64. The cyclooxygenase isoenzymes COX-1
(a) Producing mild nausea and COX-2 differ from each other in that
(b) Depressing bronchiolar secretions (a) They catalyse different pathways in prostanoid
(c) Depressing pulmonary action biosynthesis
(d) Depressing cough center (b) COX–1 is inhibited by aspirin but not COX–2
(e) Paralyzing sensory nerves of bronchi (c) COX–2 is inhibited by ibuprofen but not
COX–1
58. The greatest threat from morphine
poisoning is (d) COX–1 is constitutive while COX–2 is
inducible
(a) Renal shutdown
(b) Paralysis of spinal cord 65. Which of the following is an irreversible
(c) Respiratory depression inhibitor of cyclooxygenase ?
(d) Cardiovascular collapse (a) Aspirin (b) Phenylbutazone
(e) None of the above (c) Indomethacin (d) Piroxicam

59. A very common side effect of morphine 66. The prostanoid that consistently constricts
is blood vessels is
(a) Allergic response (b) Blood dyscrasias (a) Prostaglandin E2 (b) Prostaglandin F2a
(c) Constipation (d) Liver damage (c) Thromboxane A2 (d) Prostacyclin
(e) Visceral pain
ANALGESICS & ANTIPYRETICS 113

67. The following prostanoid is a potent 74. Corticosteroid therapy is practically


inducer of platelet aggregation mandatory in the following condition.
(a) Prostacyclin (b) Prostaglandin E2 (a) Septic shock
(c) Prostaglandin D2 (d) Thromboxane A2 (b) Renal transplant
(c) Rheumatoid arthritis
68. Aspirin in low doses produces longlasting
(d) Ulcerative colitis
inhibition
(a) Platelets contain low quantity of COX 75. For limiting cerebral edema due to brain
tumour, the preferred corticosteroids are
(b) Platelets cannot synthesize fresh COX
betamenthasone/dexamethasone
molecules because
(c) Platelets bind aspirin with high affinity
(a) They do not cause Na+ and water retention
(d) Platelet COX is inducible
(b) They are more potent
69. The early pregnancy uterus is sensitive (c) They can be administered intravenously
to the following oxytocic (d) They inhibit brain tumours
(a) Oxytocin 76. Systemic corticosteroid therapy is not
(b) Methylergometrine used routinely and is reserved only for
(c) Prostaglandin F2α severe cases of
(d) Both (a) and (b) (a) Exfoliative dermatitis
(b) Posterior uveitis
70. Cervical priming with prostaglandin
results in (c) Acute rheumatic fever
(d) Hodgkin’s disease
(a) Facilitation of sperm movement through
cervical canal 77. The following adverse effect of corticos-
(b) Increased cervical tone teroids is due to their mineralocorticoid
action
(c) Softening of cervix
(d) Increased cervical secretions (a) Osteoporosis
(b) Rise in blood pressure
71. Corticosteroids exert anti-inflammatory (c) Moon face
action by inhibiting the following enzyme
(d) Increased susceptibility to infection
(a) Cycloxygenase
78. Which of the following bones is affected
(b) Lipozygenase
more by glucocorticoid induced os-
(c) Phospholipase - A teoporosis ?
(d) Phosphodiesterase (a) Femur (b) Humerus
72. Main effect of sulphinpyrazone in (c) Radius (d) Lumber vertebra
hyperuricaemia is 79. Morphine produces analgesia by acting at
(a) Suppress the symptoms (a) Peripheral pain receptors
(b) Promote the elimination of urate (b) A spinal site
(c) prevent urate synthesis (c) Suspraspinal sites
(d) All of the above (d) Both (b) and (c)
73. Which of the following glucocorticoids has 80. In man sedation caused by morphine is
significant mineralocorticoid activity also ? characterized by
(a) Hydrocortisone (a) Initial excitement
(b) Triamcinolone (b) Little or no motor incoordination
(c) Dexamethasone (c) Rise in seizure threshold
(d) Betamethasone (d) All of the above
114 MCQs IN PHARMACOLOGY

81. Instead of depressing, morphine stimulates (a) Tramadol


(a) Vasomotor centre (b) Ethoheptazine
(b) Edinger westphal nucleus (c) Dextropropoxyphene
(c) Temperature regulating centre (d) Alfentanil
(d) Cough centre 89. An opioid analgesic is preferred over
82. In a comatose patient suspected of aspirin like analgesic in the following
poisoning, which of the following findings condition.
would be against the drug being morphine (a) Acute gout (b) Burn
(a) Selegiline (b) Chlorgiline (c) Toothache (d) Neuralgia
(c) Moclobemide (d) Tranylcypromine 90. Morphine has high affinity for the
83. Instead of being effective in hyperuricae- following opioid receptor(s).
mia following drug is contradictred in the (a) µ (Mu) (b) k (Kappa)
treatment of gout (c) d (Delta) (d) All of the above
(a) Indomethacin (b) Diclofenac 91. Which of the following is an agonist-
(c) Piroxicam (d) Aspirin antagonist type of opioid analgesic
84. Morphine dependence is characterized by (a) Pethidine (b) Pentazocine
(a) Marked drug seeking behavior (c) Fentanyl (d) Buprenorphine
(b) Physical dependence without psychic 92. Pentazocine differs from morphine in that
dependence
(a) It is inactive by the oral route
(c) Physical as well as psychic dependence
(b) It does not produce physical dependence
(d) Both (a) and (c)
(c) It has a lower ceiling of analgesic effect
85. Morphine is contraindicated in head (d) Its action is not blocked by naloxone
injury because
93. Which action of morphine is incompletely
(a) It does not relieve the pain of head injury reversed by naloxone ?
(b) It can raise intracranial tension
(a) Analgesia
(c) It can cause constipation
(b) Respiratory depression
(d) It is liable to cause addiction
(c) Sedation
86. Which of the following opioids is more (d) Miosis
potent than morphine ?
94. Lower dose of naloxone is required to
(a) Pethidine
(a) Antagonise the actions of nalorphine
(b) Fentanyl
(b) Antagonise the actions of pentazocine
(c) Dextropropoxyphene
(c) Precipitate withdrawal in mildly morphine
(d) Tramadol dependent subjects
87. Which of the following opioid analgesics (d) Precipitate withdrawal in highly morphine
is similar to codeine in pharmacological dependent subjects
profile but is less constipating ?
95. Following mediators are involved in acute
(a) Methadone inflammation except
(b) Buprenorphine (a) Histamine (b) Leukotrienes
(c) Butorphanol (c) Interferons (d) Bradykinin
(d) Dextropropoxyphene
96. The distinctive feature of the isoenzyme
88. Select the analgesic which acts through cyclooxygenase-2 is
opioids as well as additional spinal
(a) It is not inhibited by indomethacin
monoaminergic mechanisms.
ANALGESICS & ANTIPYRETICS 115

(b) It is inducible roidal anti-inflammatory drugs (NSAIDs)


(c) It generates cytoprotective prostagladins in because
gastric mucosa (a) It has lower anti-inflammatory efficacy than
(d) It is found only in foetal tissues other NSAIDs
97. Aspirin produces analgesia by (b) It has potential to cause agranulocytosis
(a) Preventing sensitization of peripheral pain (c) It has weak analgesic action
receptors (d) It alters the protein binding and metabolism
(b) Affecting gating of pain impulses at spinal of many drugs
level
104. The non-steroidal anti-inflammatory drug
(c) Raising pain threshold at subcortical level which is contraindicated in drivers and
(d) Both (a) and (b) machine operators is
98. Aspirin reduces fever by (a) Phenylbutazone (b) Indomethacin
(a) Decreasing heat production in the body (c) Naproxen (d) Diclofenac sodium
(b) Enhancing cutaneous blood flow 105. In overall assessment, which non-steroi-
(c) Inducing sweating dal anti-inflammatory drug has been con-
(d) Both (b) and (c) sidered to be the safest
99. In the treatment of chronic inflammatory (a) Aspirin (b) Naproxen
diseases, the most important limitation (c) Ibuprofen (d) Piroxicam
of aspirin is
106. The constellation of adverse effects
(a) Acid – base and electrolytc disturbances
associated with non-steroidal anti-inflam-
(b) Hypersensitivity and idiosyncratic reactions
matory drugs does not include the following
(c) Gastric mucosal damage
(a) Sedation (b) Gastric irritation
(d) Salicylism
(c) Fluid retention (d) Rashes
100. Generally the earliest manifestation of
salicylism is 107. Histamine induced edema results from
the action on _____ receptors
(a) Visual disturbance (b) Excitement
(c) Hyperventillation (d) Tinnitus (a) H3 (b) H2
(c) H1 (d) All of the above
101. Aspirin is contraindicated in children
suffering from influenza or similar viral 108. The following nonsteroidal anti-inflam-
infection because of increased risk of matory drug is a relatively selective cy-
(a) Gastric bleeding cloodygenase-2 inhibitor
(b) Thrombocytopenia (a) Tenoxicam (b) Meloxicam
(c) Fancony syndrome (c) Diclofenac sod (d) Ketoprofen
(d) Reye’s syndrome
109. What is the true of nimesulide
102. Aspirin is contraindicated in pregnant
(a) It exerts anti-inflammatory action by several
women near term because
mechanisms in addition to cyclooxygenase
(a) Labour may be delayed and prolonged inhibition
(b) Blood loss during delivery may be more (b) It is preferred for long-term use in rheumatoid
(c) Foetus may suffer premature closure of ductus arthritis
arteriosus
(c) It is contraindicated in aspirin intolerant
(d) All of the above risks asthma patients
103. Phenylbutazone should be used only in (d) All of the above
patients not responding to other nonste-
116 MCQs IN PHARMACOLOGY

110. The distinctive feature of nimesulide is 116. Sulfasalazine is used in the following
disease(s)
(a) It does not inhibit prostaglandin synthesis
(b) It does not cause gastric irritation (a) Bacillary dysentery
(c) It is well tolerated by aspirin intolerant (b) Ulcerative colitis
asthama patients (c) Rheumatoid arthritis
(d) It is not bound to plasma proteins (d) Both (b) and (c)

111. 5-HT produces contraction of smooth 117. Which component of sulfasalazine is


responsible for the therapeutic effect in
muscle and platelet aggregation via
rheumatoid arthritis ?
_______ receptors
(a) Sulfapyridine
(a) 5-HT1 (b) 5-HT2
(b) 5–aminosalicylic acid
(c) 5-HT3 (d) 5-HT4
(c) Both (a) and (b)
112. N-acetyl cysteine is beneficial in acute (d) Intact sulfasalazine molecule
paracetamol poisoning because
118. Among the rheumatoid arthritis disease
(a) It reacts with paracetamol to form a nontoxic modifying drugs, fastest symptom relief
complex is obtained with
(b) It inhibits the generation of the toxic (a) Auranofin (b) Penicillamine
metabosite of paracetamol (c) Sulfasalazine (d) Methotrexate
(c) It is a free readical scavenger
119. Strong nonsteroidal anti-inflammatory
(d) It replenishes hepatic glutathione which in turn
drugs are more commonly used than
binds the toxic metabolite of paracetamol
colchicines in acute gout because
113. For a patient of peptic ulcer, the safest (a) They are more effective
nonopioid analgesic is (b) They act more rapidly
(a) Ketorolac (b) Diclofenac sodium (c) They have additional uricosutic action
(c) Paracetamol (d) Ibuprofen (d) They are better tolerated

114. Which of the following anti-inflammatory 120. Select the drug which is neither analgesic,
analgesics has been cleared for pediatric nor antiinflammatroy nor uricosuric, but
use is highly efficacious in acute gout
(a) Indomethacin (b) Ibuprofen (a) Prednisolone (b) Colchicine
(c) Ketorolac (d) Piroxicam (c) Naproxen (d) Sulfinpyrazone

115. Which of the following statements is 121. The most important dose-limiting adverse
effect of colchicines is
correct about gold therapy of rheumatoid
arthiritis (a) Sedation (b) Kidney damage
(a) It is indicated only in rapidly progressing (c) Diarrhoea (d) Muscle paralysis
disease, not controlled by nonsteroidal anti- 122. Probenecid has the following action(s)
inflammatory drugs
(a) Uricosuric (b) Analgesic
(b) It is indicated only in severe cases after both
(c) Antiinflammatory (d) Both (a) and (c)
nonsteroidal anti-inflammatory drugs and
corticosteroids have failed 123. Vasodilation mediated by 5-HT1 re-
(c) When gold therapy is started, nonsteroidal ceptors involve following mechanism
anti-inflammator y drugs should be (a) Direct relaxant effect on smooth muscle
discontinued (b) Inhibition of norepinephrine
(d) Intramuscular gold is the most rapidly acting (c) Nitric oxide release from endothelial cells
drug in severe rheumatoid arthritis (d) All of the above
ANALGESICS & ANTIPYRETICS 117

124. Allopurinol decreases the plasma concen- (c) Advanced disease


tration of (d) As first line therapy
(a) Hypoxanthine (b) Xanthine 131. Following 5-HT receptors is a ligand gated
(c) Uric acid (d) All of the above ion channel
125. Allopurinol is indicated in the following (a) 5-HT1A (b) 5-HT2A
category of chronic gout patients (c) 5-HT3 (d) 5-HT4
(a) Over producers of uric acid 132. In a person suffering from hepatic disease,
(b) Under excretors of uric acid the dose of pethidine should be
(c) Those with tophi and/or renal urate stones (a) Reduced because it shows idiosyncratic
(d) All of the above reaction
(b) Reduced because the clearance is reduced
126. Nonsteroidal anti-inflammatory drugs
(c) Increased because the clearance is increased
reduce the diuretic action of furosemide
by (d) Increased because patient becomes resistant
to pethidine
(a) Preventing prostaglandin mediated intrarenal
haemodynamic actions 133. Kinins play a role in which of the following
(b) Blocking the action in ascending limb of loop (a) Pain (b) Inflammation
of Henle (c) Asthma (d) Vasodilatation
(c) Enhancing salt and water reabsorption in (e) All of the above
distal tubule
134. The term eicosanoid refers to which of
(d) Increasing aldosterone secretion the following compounds
127. The glucocorticoid described as highly (a) Prostaglandins (b) Leukotrienes
potent anti-inflammatory agent is (c) Interleukins (d) A + B
(a) Methyl prednisolone
135. Which of the following is a side effect of
(b) Cortisone NSAID ?
(c) Triamcinolone (a) GI ulceration
(d) Dexamethasone (b) Blockade of platelet agregation
128. The corticosteroid with a longer duration (c) Inhibition of uterine motility
of action is (d) Renal vasoconstriction
(a) Cortisone (b) Prednisone (e) All of the above
(c) Betamethasone (d) Fludrocortisone 136. Which of the following has minimal anti-
inflammatory action ?
129. Gold in rheumatoid arthritis
(a) Piroxicam (b) Oxaprozin
(a) Can cause regression of degenerative lesions
of this disease (c) Enalapril (d) Amrinone
(b) Can reduce the concentrations of rheumatoid 137. Probenecid increases excretion of
factor (a) Digoxin (b) Furosemide
(c) Does not halt the progress of the disease (c) Enalapril (d) Amrinone
(d) Is employed as first line therapy
138. Which of the following is involved in
130. Use of gold compounds in rheumatoid pathobiology of inflammatory process ?
arthritis is beneficial in (a) Cytokines
(a) Early active disease not responding to (b) Cell adhension molecules
NSAIDs (c) Phospholipoase A2
(b) Mild disease (d) All of the above
118 MCQs IN PHARMACOLOGY

139. Following is the effect of histamine on (d) All of the above


small vessels
140. Earliest sign of aspirin toxicity is
(a) Outward passage of plasma protein and fluid
(a) Tinnitus
into the extracellular spaces
(b) Metabolic acidosis
(b) Increase in the flow of lymph and its protein
content (c) Reye syndrome
(c) Formation of edema (d) Respiratory depression

ANSWERS
1. b 2. d 3. b 4. c 5. c 6. d
7. e 8. e 9. b 10. d 11. d 12. b
13. b 14. b 15. d 16. b 17. d 18. c
19. c 20. b 21. a 22. d 23. c 24. d
25. a 26. c 27. a 28. d 29. b 30. a
31. e 32. d 33. d 34. a 35. e 36. d
37. d 38. a 39. c 40. c 41. a 42. b
43. b 44. a 45. d
46. 1.c, 2.d, 3.c, 4.a, 5.b 47.1.e, 2.a, 3.b, 4.d, 5.c 48. a 49. b
50. e 51. c 52. d 53. c 54. d 55. a
56. a 57. d 58. c 59. c 60. a 61. d
62. d 63. d 64. d 65. a 66. c 67. d
68. b 69. c 70. c 71. c 72. b 73. a
74. b 75. a 76. c 77. b 78. d 79. b
80. b 81. b 82. a 83. d 84. d 85. b
86. b 87. d 88. a 89. b 90. a 91. b
92. c 93. c 94. d 95. c 96. b 97. d
98. d 99. c 100. d 101. d 102. d 103. b
104. b 105. c 106. a 107. c 108. b 109. a
110. c 111. b 112. d 113. c 114. d 115. a
116. d 117. a 118. d 119. d 120. b 121. c
122. a 123. d 124. c 125. d 126. a 127. d
128. c 129. b 130. a 131. c 132. b 133. e
134. d 135. e 136. c 137. c 138. d 139. d
140. a
ANALGESICS & ANTIPYRETICS 119

EXPLANATIONS FOR THE ANSWERS


1. b β-Aminoethylimidazole is a chemical name of reabsorption of uric acid and thus promotes the
histamine elimination of urate.
6. d Prostaglandins are potent vasodilators and this 83. d Aspirin (in general salicylates) are contraindi-
action involves arterioles, precapillary sphincters cated in the treatement of gout as they elevate
and postcapillary venules. Prostaglandins have uric acid levels and they also antagonize the
no effect on large veins. actions of probenecid and sulfinpyrazone.
9. b Acetaminophen or paracetamol is a para- Indomethacin, diclofenac and piroxicam have no
aminophenol derivative. such effects and hence are not contraindicated.
Diclofenac is a phenylacetic acid derivative. 95. c Histamine, leukotrienes and bradykinin are
Piroxicam is an oxicam derivative. mediators of acute inflammation.
Celecoxib is a benzenesulphonamide derivative. Interferons are involved in chronic inflammatory
conditions.
9. b Auronofin is more hydrophilic gold compound
107. c Histamine induced edema results from the action
than aurothioglucose and gold sodium thiomalate
of H1 receptors. This H1 receptor effect results in
and hence is administered orally.
outward passage of plasma protein and fluid from
Aurothioglucose and gold sodium thiomalate capillaries into extracellular spaces and increase
show erratic absorption when administered orally in the flow of lymph and its protein content.
and are generally administered intramuscularly.
H2 and H3 receptors are not involved in these
17. d Histamine is an example of preformed mast cell effects.
mediator of inflammatory process. It produces
111. b 5-HT2 receptors are involved in the 5-HT
vasodilation, vasopermeability, itch, cough and
mediated smooth muscle contraction and platelet
bronchoconstriction. PAF, LTC4 and PGD2 are aggregation.
lipid – derived mediators.
123. d Vasodilation mediated by 5-HT 1 receptors
22. d Topical glucocorticoids e.g. beclomethasone, involve various mechanisms:
fluticasone, triamcinolone, budesonide,
• Direct relaxant effect on smooth muscles
flunisolide or cromolyn are quite effective in
allergic rhinitis. • Release of NO from endothelial cells.
• Inhibition of norepinephrine from sympathetic
27. a Dimaprit is a selective agonist of H2 and H3
nerve terminals.
receptors but not H1 receptors.
131. c 5-HT3 receptors are ligand gated ion channels.
38. a In gastrointestinal tract, serotonin causes:
5-HT1A, 5-HT2A and 5-HT4 receptors are coupled
• Contraction of GI smooth muscles.
to second messenger system as follows:
• Increase in muscle tone.
5-HT1A : Decrease in cAMP
• Increase in peristalsis.
5-HT2A : Increase in IP3/DAG
49. b TNF-α is a cytotoxicfactor and plays important
5-HT4 : Increase in cAMP.
role in the inflammatory response.
139. d In general, histamine dilates blood vessels in
60. a Eicosa means twenty and hence the genral name
humans. It produces variety of effects on small
eicosanoids for prostaglandins, thromboxanes blood vessels such as outward passage of
and leukotrienes which contains 18, 20 and plasma protein and fluid into the extracellular
22 carbon skeleton. spaces, increase in the flow of lymph and its
72. b Sulphinpyrazone inhibits the renal tubular protein content and finally formation of edema.
This page
intentionally left
blank
CHAPTER 6
CARDIOVASCULAR DRUGS

1. Relationship between arterial blood (c) Lovastatin


pressure (BP), cardiac outpur (CO) and (d) Niacin
peripheral vascular resistance (PVR) (e) Simvastatin
can be described as
5. After being counseled about lifestyle and
(a) BP = COxPVR (b) BP = CO/PVR
dietary changes, the patient was started
(c) BP = PVR/CO (d) None of the above
on atorvastatin. During his treatment with
2. If a fibrinolytic drug is used for treatment atorvastatin, it is important to routinely
of acute myocardial infarction, the monitor serum concentrations of
adverse drug effect that is most likely to (a) Blood urea nitrogen (BUN)
occur is (b) Alanine and aspartate aminotransferase
(a) Acute renal failure (c) Platelets
(b) Development of antiplatelet antibodies (d) Red blood cells
(c) Encephalitis secondary to liver dysfunction (e) Uric acid
(d) Hemorrhagic stroke
6. Six months after beginning atorvastatin,
(e) Neutropenia
the patient’s total and LDL cholesterol
3. Increased serum levels of which of the concentrations remained above normal
following may be associated with a and he continued to have anginal attacks
decreased risk of atherosclerosis? despite good adherence to his antianginal
medications. His physician decided for
(a) Very low-density lipoproteins (VLDL)
niacin. The major recognized mechanism
(b) Low-density lipoproteins (LDL) of action of niacin is
(c) Intermediate – density lipoproteins (IDL)
(a) Decreased lipid synthesis in adipose tissue
(d) High-density lipoproteins (HDL)
(b) Decreased oxidation of lipids in endothelial
(e) Cholesterol cells
4. If the patient has a history of gout, which (c) Decreased secretion of VLDL by the liver
of the following drugs is most likely to (d) Increased endocytosis of HDL by the liver
exacerbate this condition? (e) Increased lipid hydrolysis by lipoprotein
(a) Colestipol lipase
(b) Gemfibrozil
122 MCQs IN PHARMACOLOGY

7. Following drugs act on imidazoline (a) Alprostadil (b) Fluoxetine


receptor (c) Mifepristone (d) Sildenafil
(a) Moxonidine (b) Dexmedetomidine (e) Zafirlukast
(c) Tizanidine (d) All of the above
14. A treatment of angina that consistently
8. Which one of the following drugs increase decreases the heart rate and can prevent
digoxin plasma concentration by a vasospastic angina attacks is
pharmacokinetic mechanism? (a) Isosorbide dinitrate
(a) Captopril (b) Hydrochorothiazide (b) NIFedipine
(c) Lidocaine (d) Quinidine (c) Nitroglycerin
(e) Sulfasalazine (d) Propranolol
9. A 55-year-old patient currently receiving (e) Verapamil
other drugs for another condition is to
15. In a patient receiving digoxin for congestive
be started on diuretic therapy for mild
heart failure, condition that may facilitate
heart failure. Thiazides are known to
the appearance of toxicity include
reduce the excretion of
(a) Diazepam (b) Fluoxetine (a) Hyperkalemia (b) Hypernatremia
(c) Imipramine (d) Lithium (c) Hypocalcemia (d) Hypomagnesemia
(e) Potassium (e) All of the above

10. A hypertensive patient has been using 16. Activation of endothelin receptor ETA,
nifedipine for some time without unto- leads to
ward effects. If he experiences a rapidly (a) Vasoconstriction
developing enhancement of the antihyper- (b) Bronchoconstriction
tensive effect of the drug, it is probably
(c) Aldosterone release
due to
(d) All of the above
(a) Concomitant use of antacids
(b) Fods containing tyramine 17. Methylxanthine drugs such as aminophyl-
(c) Grapefruit juice line cause which one of the following?
(d) Induction of drug metabolism (a) Vasoconstriction in many vascular beds
(e) Over – the – counter decongestants (b) Decrease in the amount of cAMP in mast
cells
11. A drug lacking vasodilator properties that
(c) Bronchodilation
is useful in angina is
(d) Activation of the enzyme phosphodiesterase
(a) Isosorbide dinitrate
(e) Sedation
(b) Metoprolol
(c) NIfedipine 18. Drugs used in asthma that often cause
(d) Nitroglycerin tachycardia and tremor include
(e) Verapamil (a) Beclomethasone (b) Cromolyn sodium
(c) Ipratropium (d) Metaproterenol
12. Aldosterone release is stimulated by
(e) All of the above
(a) Angiotensin I (b) Angiotensin
(c) Angiotensin III (d) Both (b) and (c) 19. Following potassium sparing diuretic
inhibits action of aldosterone
13. Which one of the following drugs is used
in the treatment of male impotence and (a) Amiloride (b) Triamterene
activates prostaglandin E1 receptors? (c) Spironolactone (d) All of the above
CARDIO–VASCULAR DRUGS 123

20. In patients with chronic granulomatous (d) Slows the depolarization phase of the action
disease which of the following agents potential in AV nodal cells
increases the synthesis of tumor necrosis (e) Used in management of supraventricular
factor, leading to activation of phagocy- tachycardias
tosis?
26. What drug is used to prevent embolism in
(a) Aldesleukin (b) Cyclosporine
the lung and during myocardial infarction?
(c) Filgrastim (d) Infliximab
(a) Alteplase
(e) Interferon gamma
(b) Human growth hormone
21. The mechanism of action of cyclosporine (c) Granulocyte–macrophage colony – stimulating
involves factor (GM–CSF)
(a) Activation of calcineurin (d) EPOGEN (EPO)
(b) Binding to cyclophilin to cause inhibition of (e) None of the above
a cytoplasmic phosphatase
27. Which of the following cardiovascular
(c) Blockade of interleukin – 2- receptors
agents is classified chemically as a gly-
(d) Inhibition of phospholipase A2 coside?
(e) Suppression of bone marrow progenitors
(a) Nifedipine (b) Digoxin
22. Which one of the following drugs predictably (c) Flecainide (d) Cholestyramine
prolongs the PR interval and increases (e) Warfarin
cardiac contractility?
28. Inhibition of carbonic anhydrase results in
(a) Digoxin (b) Lidocaine
(a) Abolition of NaHCO 3 reabsorption in
(c) Propranolol (d) Quinidine
proximal tubule
(e) Verapamil
(b) Enhanced of NaHCO3 reabsorption in
23. Which of the following is the drug of proximal tubule
choice for management of cardiac (c) Enhanced NAHCO3 secretion in distal tubule
arrhythmias that occur in digitalis (d) None of the above
toxicity?
29. Which of the following cyclotron produced
(a) Amiodarone (b) Lidocaine
radiopharmaceuticals is used for assessing
(c) Propranolol (d) Sotalol
regional myocardial perfusion as part of
(e) Prazosin an exercise stress test?
24. A 54-year-old woman with severe (a) Thallous chloride 201TI USP
hypercholesterolemia is to be treated (b) Sodium iodide 123I
with a combination of niacin and (c) Gallium citrate 67Ga USP
atorvastatin. With this drug combination,
(d) Indium 111In pentetate
it is important that the patient be
monitored closely for signs of (e) Cobalt 57Co cyanocobalamin

(a) Agranulocytosis (b) Gallstones 30. Mary has a family history of heart disease
(c) Lactic acidosis (d) Myopathy and wonders if garlic would be beneficial
to her. Which of the following statements
(e) Thyrotoxicosis
is correct about garlic?
25. Regarding verapamil, which one of the (a) Enteric-coated tablets release their contents
following statements is false? in the stomach
(a) Angina pectoris is an important indication (b) Side effects include heartburn, flatulence, and
for the use of verapamil sweating
(b) Contraindicated in the asthmatic patient (c) The safety of garlic in pregnancy is unknown
(c) Relaxes vascular smooth muscle (d) Garlic does not interact with warfarin
124 MCQs IN PHARMACOLOGY

31. Exertion–induced angina, which is 37. Which of the following thrombolytic


relieved by rest, nitroglycerin, or both, is agents would be appropriate at this time?
referred to as (a) Anisoylated plasminogen streptokinase
(a) Prinzmetal’s angina activator complex (APSAC)
(b) Unstable angina (b) Streptokinase (SK)
(c) Classic angina (c) Recombinant tissue-type plasminogen
(d) Variant angina activator (t-PA)
(e) Preinfarction angina 38. Strong anticholinergic effects limit the
antiarrhythmic use of
32. Myocardial oxygen demand is increased
by all of the following factors except (a) Quinidine (b) Procainamide
(c) Tocainide (d) Flecainide
(a) Exercise
(e) Disopyramide
(b) Smoking
(c) Cold temperatures 39. Following loop diuretic is a phynoxy
(d) Isoproterenol acetic acid derivative
(e) Propranolol (a) Furosemide (b) Bumetanide
(c) Ethacrynic acid (d) All of the above
33. Which of the following agents used in
prinzmetal’s angina has spasmolytic 40. Following potassium sparing diuretic is
actions, which increase coronary blood a mineralocorticoid receptor antagonist
supply? (a) Amiloride (b) Triamterene
(a) Nitroglycerin (c) Spironolactone (d) All of the above
(b) Nifedipine 41. A patient receiving a class I antiarrhyth-
(c) Timolol mic agent on a chronic basis complains
(d) Isosorbide mononitrate of fatigue, low-grade fever, and joint
(e) Propranolol pain suggestive of systemic lupus erythe-
matosus (SLE). The patient is most likely
34. The oral absorption of following osmotic receiving
diuretic is negligible (a) Lidocaine (b) Procainamide
(a) Glycerin (b) Mannitol (c) Quinidine (d) Flecainide
(c) Isosorbide (d) All of the above (e) Propranolol
35. Maximal medical therapy for treating 42. Which of the following drugs is a class IV
angina pectoris is represented by which antiarrhythmic that is primarily indicated
of the following choices? for the treatment of supraventricular
(a) Diltiazem, verapamil, nitroglycerin tachyarrhythmias?
(b) Atenolol, isoproterenol, diltiazem (a) Lbutilide (b) Mexiletine
(c) Verapamil, nifedipine, propranolol (c) Diltiazem (d) Quinidine
(d) Isosorbide, atenolol, diltiazem (e) Propranolol
(e) Nitroglycerin, isosorbide, atenolol 43. Which of the following agents has a direct
effect on the AV mode, delaying calcium-
36. The term ischemic heart disease (IHD)
channel depolarization?
is used to designate all of the following
conditions except (a) Lidocaine (b) Diltiazem
(c) Bretylium (d) Quinidine
(a) Angina pectoris
(e) Lbutilide
(b) Sudden cardiac death
(c) Congestive heart failur (CHF) 44. Which of the following drugs is a class III
(d) Arrhythmias
CARDIO–VASCULAR DRUGS 125

antiarrhythmic agent that is effective in (c) Furosemide


the acute management of atrial fibrilla- (d) Carvedilol
tion or atrial flutter of recent onset ? (e) Bumetanide
(a) Bretylium (b) Lbutilide
49. For treating the patient with congestive
(c) Metoprolol (d) Disopyramide heart failure (CHF), which of the following
dosages of dopamine is selected for its
45. Which of the following groups of
positive inotropic effects?
symptoms is most often associated with
a patient who has right-sided heart (a) 2.0 mg/kg/min
failure? (b) 5–10 mg/kg/min
(a) Nocturia, rales, paroxysmal nocturnal dyspnea (c) 10–20 mg/kg/min
(b) Paroxysmal nocturnal dyspnea, pedal edema, (d) 40 mg/kg/min
jugular venous distention, hepatojugular reflux (e) 40 mg/kg/min
(c) Jugular venous distention, hepatojugular 50. Milrinone is an example of
reflux, pedal edema, shortness of breath
(a) Phosphodiesterase I inhibitor
(d) Hepatojugular reflux, jugular venous
(b) Phosphodiesterase II inhibitor
distension, pedal edema, abdominal
distention (c) Phosphodiesterase III inhibitor
(d) Phosphodiesterase IV inhibitor
(e) Paroxysmal nocturnal dyspnea, jugular
venous distention, abdominal distention, 51. Situations that predispose a digitalis-
shortness of breath treated patient to toxicity include
46. Which of the following combinations of (a) Hypercalcemia
drugs, when used together, reduce both (b) Hyperkalemia
preload and afterload? (c) Hypermagnesemia
(a) Nitroglycerin and isosorbide dinitrate 52. Unfractionated heparin binds to anti-
(b) Hydralazine and isosorbide dinitrate thrombin III and inactivates clotting
(c) Captopril and methyldopa factor(s)
(d) Prazosin and angiotension II (a) Xa (b) Ixa
(e) Hydralazine and methyldopa (c) Iia (d) All of the above
(e) None of the above
47. When digoxin is used in a patient with
congestive heart failure (CHF), it works 53. A patient to be commenced on oral anti-
by exerting a positive effect on coagulant therapy for DVT would be
treated with
(a) Stroke volume
(b) Total peripheral resistance (a) Oral anticoagulant therapy with warfarin for
a goal intenational normalized ration (INR)
(c) Heart rate
of 2–3
(d) Blood pressure
(b) Oral anticoagulant therapy with warfarin for
(e) Venous return a goal INR of 2.5–3.5
48. Because of proven beneficial effects on (c) Oral anticoagulant therapy with aspirin for
“cardiac remodeling”, these agents are a goal INR of 2–3
now indicated as first line therapy in CHF 54. A patient on oral anticoagulant therapy
patients. Which of the following is is commenced on sulfamethoxazole-
representative of this group of drugs? trimethoprim, double-strength twice daily.
(a) Hydrochlorothiazide One may expect to see the international
(b) Enalapril normalized ratio
126 MCQs IN PHARMACOLOGY

(a) Increase 61. In case of acute pain of angina pectoris


(b) Decrease the most effective treatment would be to
administer
(c) Remain unchanged
(a) Mannitol hexanitrate
55. When compared to unfractionated (b) Erythrityl tetranitrate
heparin, low molecular weight heparins
(c) Sodium nitrate
have
(d) Pentaerythritol tetranitrate
(a) Preferential binding affinity to factor Xa
(e) Nitroglycerin
relative to Iia (thrombin)
(b) Shorter half-lives 62. Which of the following is used to lower
(c) Dose – dependent renal clearance blood lipid levels?
(a) Trimethadione (b) Clofibrate
56. Acute renal failure (ARF) may be caused
(c) Flucytosine (d) Coumarin
by all of the following except
(e) Propranolol
(a) Acute tubular necrosis (ATN) due to drug
therapy (e.g., aminoglycosides, contrast 63. The chief use of levoarterenol is to treat
media) (a) Shock (b) Diabetes
(b) Severe hypotension or circulatolry collapse (c) Hypertension (d) Cardiac arrhythmias
(c) Decreased cardiac output, as from (e) Iron deficiencies
congestive heart failure
64. Tolerance to nitroglycerin may be
(d) Hemolysis, myoglobinuria overcome by
(e) Hyperkalemia
(a) Initially using the largest safe dose of the drug
57. During Phase 2 of action potential in (b) Using other nitrites
cardiac cell, depolarizing current through (c) Temporarily discontinuing the drug for one
calcium channels is balanced by or two weeks
(a) Delayed rectifier potassium current (d) Use of higher doses
(b) outward chloride channel (e) None of the above
(c) Both (a) and (b) 65. Quinidine can cause paradoxical tachy-
(d) None of the above cardia in a patient of
58. The action of quinidine differs from that (a) Stick sinus syndrome
of digitalis in (b) Auricular extrasystoles
(a) Decreasing irritability of cardiac muscle (c) Auricular fibrillation
(b) Preventing passage of impulses to the (d) Ventricular extrasystoles
ventricle 66. Quinidine is now used primarily for
(c) Increasing irritability of heart muscle (a) Conversion of auricular fibrillation to sinus
(d) Reducing conductivity rhythm
(e) None of the above (b) Control of ventricular rate in atrial flutter
(c) Termination of ventricular tachycardia
59. Overuse of digitalis may result in
(d) Prevention of recurrences of atrial and
(a) Habituation (b) Tolerance ventricular extrasystoles/tachycardias
(c) Addiction (d) Physical dependence
67. Procainamide differs from quinidine in the
(e) Cummulative poisoning
following respect
60. The action of digitalis is enhanced by (a) It does not cause paradoxical tachycardia
(a) Sodium (b) Calcium (b) It has no alfa adrenergic blocking activity
(c) Magnesium (d) Potassium (c) It has little antivagal action
(e) Chloride (d) Both (b) and (c)
CARDIO–VASCULAR DRUGS 127

68. In heart, potassium channels determine (c) They preferentially dilate autoregulatory
(a) Pacemaker function arterioles without affecting the larger arteries
(b) Resting potential (d) They increase subepicardial blood flow
without affecting subendocardial blood flow
(c) Action potential duration
(d) All of the above 75. Organic nitrates relax vascular smooth
muscle by
69. Lidocaine is the preferred antiarrhythmic
(a) Increasing intracellular cyclic AMP
for emergency control of cardiac arrhyth-
mias following acute myocardial infarction (b) Increasing intracellular cyclic GMP
because (c) Decreasing intracellular cyclic AMP
(a) It has a rapidly developing and titratable (d) Both (b) and (c)
antiarrhythmic action 76. Select the organic nitrate which undergoes
(b) It casues little myocardial depression and minimal first-pass metabolism in the liver
hypotension (a) Glyceryl trinitrate
(c) It has broad spectrum antiarrhythmic efficacy (b) Isosorbide dinitrate
in atrial as well as ventricular arrhythmias
(c) Isosorbide mononitrate
(d) Both (a) and (b)
(d) Erythrityl tetranitrate
70. Hypothyroidism is a possible consequence
77. The primary mechanism of beneficial
of prolonged therapy with
effect of glyceryl trinitrate in classical
(a) Procainamide (b) Mexiletine angina pectoris is
(c) Sotalol (d) Amiodarone (a) Increase in total coronary blood flow
71. Which of the following drugs is preferred (b) Redistribution of coronary blood flow
for termination of paroxysmal supraven- (c) Reduction of cardiac preload
tricular tachycardia (d) Reduction of cardiac after load
(a) Digoxin (b) Quinidine 78. Enhanced automaticity in cardiac cells
(c) Propranolol (d) Verapamil may occur because of
72. The following drug is used to reduce the (a) β-adrenergic stimulation
frequency of angina pectoris as well as (b) Hypokalemia
to terminate an acute attack (c) Mechanical stretch of cardiac muscles
(a) Digoxin (b) Furosemide (d) All of the above
(c) Enalapril (d) Amrinone
79. Glyceryl trinitrate is administratered by
73. Antianginal drugs afford the following all of the following routes except
benefit/benefits (a) Oral (b) Sublingual
(a) Terminate anginal attacks (c) Intramuscular (d) Intravenous
(b) Decrease the frequency of anginal attacks 80. A patient of acute myocardial infarction
(c) Retard the progression of coronary artery being treated in intensive care unit
disease developed left ventricular failure with raised
(d) Both (a) and (b) central venous pressure. It was decided to
use glyceryl trinitrate. Which route/method
74. Choose the correct statement about the of administration would be most suitable.
action of nitrates on coronary vessels
(a) Sublingual
(a) They mitigate angina pectoris by increasing
(b) Oral
total coronary flow
(c) Intravenous bolus injection
(b) They preferentially dilate conducting arteries
without affecting resistance arterioles (d) Slow intravenous infusion
128 MCQs IN PHARMACOLOGY

81. A patient suffers from spisodic pain 88. Which of the following drugs is a potassium
diffusely localized over the chest and upper channel opener ?
abdomen, which is relieved by sublingual (a) Pinacidil (b) Hydralazine
glyceryl trinitrate. He could be suffering
(c) Glibenclamide (d) Amiloride
from
(a) Angina pectoris 89. Though nitrates and calcium channel
blockers are both vasodilators, they are
(b) Biliary colic
used concurrently in angina pectoris
(c) Esophageal spasm
because
(d) All of the above
(a) They antagonize each other’s side effects
82. The dihydropyridines block the following (b) Nitrates primarily reduce preload while
type of calcium channels. calcium channel blockers primarily reduce
(a) L-type voltage sensitive channels after load
(b) T-type voltage sensitive channels (c) Nitrates increase coronary flow while
(c) N-type voltage sensitive channels calcium
(d) Receptor operated calcium channels (d) Both (b) and (c)

83. Which of the following drugs is most 90. ‘Coronary steal phenomenon’ has been
likely to accentuate varient (Prinzmetal) noted most frequently with
angina ? (a) Glyceryl trinitrate (b) Dipyridamole
(a) Digoxin (b) Furosemide (c) Propranolol (d) Diltiazem
(c) Enalapril (d) Amrinone 91. Which of the following drugs is believed
84. In cardiac cells, adenosine to improve microcirculation in peripheral
vascular diseases by promoting RBC
(a) Causes shortening of action potential duration flexibility ?
(b) Depolarization
(a) Cyclandelate (b) Theophyline
(c) Increase in normal automaticity
(c) Pentoxiphyline (d) Nicotinic acid
(d) All of the above
92. Higher incidence of myocardial infarc-
85. Which of the following antianginal drugs tion and increased mortality has been
is most likely to produce tachycardia as noted with the use of the following anti-
a side effect ? hypertensive drug
(a) Amlodipine (b) Nifedipine (a) Nifedipine (b) Verapamil
(c) Diltiazem (d) Verapamil (c) Diltiazem (d) Lisinopril
86. Which of the following is not an attribute 93. Cardiac glycosides are obtained from
of amlodipine ? following plant source.
(a) High and consistent oral bioavailability (a) Rauwolfia serpentina
(b) Large volume of distribution (b) Strophanthus gratus
(c) Generation of an active metabolite (c) Ricinus commounts.
(d) Long elimination half-life (d) Atropa belladonna
87. Propranolol should not be prescribed for 94. Therapeutic dose of digoxin in a normal
a patient of angina pectoris who is already individual has the following effects, except
receiving
(a) Increase in the speed of myocardial contractility
(a) Nifedipine
(b) No significant change in cardiac output
(b) Felodipine
(c) Relaxation of peripheral vascular bed
(c) Verapamil
(d) Increase in the force of myocardial
(d) Isosorbide mononitrate contractility
CARDIO–VASCULAR DRUGS 129

95. Digoxin given for cardiac failure is 102. Digitalis was discovered by
extremely valuable in patients of (a) William Withering
(a) Thyrotoxicosis (b) Beriberi (b) Ottolewi
(c) Cor pulmonale (d) Atrial fibrillation (c) Walksman
96. The agent given sublingually in an acute (d) Dale
attack of angina pectoris is 103. The cardiac slowing by digitalis in atrial
(a) Glyceryl trinitrate fibrillation and congestive failure
(b) Amyl nitrite (a) Is due partly to increased vagal activity
(c) Erythrital tetranitrate (b) Is due partly to decreased sympathetic
(d) Pentaerythritol tetranitrate activity
(c) Is due partly to depression of S.A. Node
97. Following statement is true about lidocaine
(d) May be partly related to a vagal mediated
(a) Reduce the slope of Phase 4
increse in atrial frequency
(b) Threshold excitability is not altered
(c) Action potential is not affected 104. After oral administration peak concentra-
tion of digoxin is reached in
(d) All of the above
(a) 1 to 2 mins (b) 30 to 60 mins
98. Aspirin in small doses (50 to 150 mg per
(c) 5 to 10 mins (d) 4 to 6 hours
day)
(a) Is of benefit in patients of unstable angina 105. The normal therapeutic plasma con-
centration of digitalis is
(b) Has thrombolytic action
(c) PREFerentially inhibits prostacyclin synthetase (a) 0.5 to 1.5 ng/ml and toxicity appears above
enzyme 3 mg/ml
(d) MAY Alleviate need for Verapamil in variant (b) 1 to 5 ng/µl and toxicity appears above 50
angina mg/ml
(c) 0.1 to 0.5 αg mg/ml and toxicity appears
99. Major beneficial effect of nitrates in above 5
classical angina is due to
(d) 0.5 to 1.5 αg mg/ml and toxicity appears
(a) Dilation of veins more than arteries above 5 αg mg/ml
(b) Increase in total coronary blood flow
106. Drug of choice for digitalis induced
(c) An increase in the end diastolic size of the arrhythmia is
heart
(a) Propranolol (b) Phenytoin
(d) An increase in the heart rate
(c) Xylocaine (d) Phenylephrine
100. Major lipid class present in chylomicrons
is 107. In which of the following conditions,
digitalis is most likely to be beneficial?
(a) Endogenous triglycerides
(a) Heart failure from valvular lesions
(b) Cholesterol esters
(b) Furosemide
(c) Dietary triglycerides
(c) Heart disease with anaemia
(d) All of the above
(d) Heart failure from thyrotoxicosis
101. Polymorphic ventricular tachycardia can
occur when terfenadine 108. Nicotinic acid

(a) Is coadministered with azithromycin (a) Decreases production of VLDL


(b) Is coadministered with fluconazole (b) Decreases LDL levels
(c) Is given in higher doses (c) Increases HDL levels
(d) Reduces QT interval (d) All of the above
130 MCQs IN PHARMACOLOGY

109. Which of the following would least likely (a) Probably be unrelated to drug administration
benefit a patient in acute pulmonary (b) Be the desired response of the drug
edema due to congestive heart failure?
(c) Probably be the result of drug-induced heart
(a) Intravenous morphine block
(b) Digoxin (d) Be an indication for increasing the drug dose.
(c) Oxygen
115. Digoxin differs from digitoxin in that
(d) Rotating tourniquets
digoxin
110. Monoamine oxidase inhibitors (a) Has a longer half life
(a) Do not cause postural hypotension (b) Is completely absorbed from the GI tract
(b) Are known to reduce blood pressure by (c) Is bound extensively to plasma proteins
depleting catecholamines
(d) Its half-life is more dependent on the
(c) Have a prompt hypotensive action adequacy of renal function
(d) Generally do not cause tachycardia
116. Diazoxide is administered by slow IV-
111. Digitalis is given before quinidine in the injection in emergency treatment of
treatment of atrial fibrillation, because hypertension because
digitalis increase A-V conduction and
counteracts (a) The drug is rapidly biotransformed by hepatic
microsomal enzymes
(a) The “Vagotonic” effect of quinidine and
(b) The drug is highly lipid-soluble and quickly
prevents “paradoxic” tachycardia
deposited in natural fat
(b) The “Vagolytic” effect of quinidine and
prevents “paradoxic” bradycardia (c) This measure is necessary to prevent severe
hyperglycemia
(c) The “Vagotonic” effect of quinidine and
prevents paradoxic bradycardia (d) The drug is quickly inactivated by serum
protein binding
(d) The “Vagolytic” effect of quinidine and
prevents paradoxic tachycardia 117. Glyceryl trinitrate is generally taken by
112. The earliest toxic symptom of chronic (a) Oral route (b) Intravenous route
digitalis administration is (c) Sublingual route (d) Subcutaneous route
(a) Altered color vision
118. Which of the following is longest acting
(b) Psychic symptoms nitrate?
(c) Anorexia, nausea, vomiting
(a) Glyceryl trinitrate
(d) Retrosternal pain
(b) Ethyt tetranitrate
113. In normal condition sequence of con- (c) Octyl nitrite
duction of cardiac action potential is
(d) Pentaerythritol tetranitrate
(a) SA node to AV node to bundle of His to atrium
119. Methaemoglobinaemia can occur with
(b) SA node to atrium to AV node to bundle of
high doses of
His
(c) SA node to atrium to bundle of His to AV (a) Digitalis (b) Nitrites
node (c) Propranolol (d) All of the above
(d) SA node to AV node to atrium to bundle of
120. Cardiac glycoside consists of
His
(a) Aminoacids and sugar
114. While treating a CHF patient with cardiac
(b) A steroid combined with sugar residue
glycoside, one notices that the patient’s
resting heart rate, which previously had (c) A polypeptide and sugar
been 86/min is now 40/min. This would (d) None of the above
CARDIO–VASCULAR DRUGS 131

121. Vascoconstriction is produced by following 127. Resperine is used in hypertension, only


agent in low doses because
(a) Sympathomimetic amines (a) It is a very strong antihypertensive
(b) Eicosanoids (b) Side effects become disproportionately
(c) Endothelin marked
(d) All of the above (c) High doses on chronic use cause addiction
(d) It has a very long duration of action
122. Perhexillene meleate is a
(a) Vasodilator drug 128. There is no rationale of giving digitoxin
parenterally because
(b) Cardiotonic drug
(c) Antihyperlipidaemic drug (a) It is very painful if given parenterally
(d) Urinary antiseptic (b) Its total initial oral and parenteral digitalization
dose is same
123. The dose of verapamil is (c) It is quickly metabolized if given I.V. or I.M.
(a) 4 to 6 gms per day (d) It is always preferred by I.V. route
(b) 40 to 80 gms thrice daily
129. For a longer duration of action nitroglycerine
(c) 1 to 10 mg thrice daily can be administered by which of the
(d) 250 mg once daily following routes
124. Chronic use of nitrites may lead to (a) Sublingual
(a) Allergic response (b) Inhalation
(b) Addietion (c) Intravenous
(c) Atherosclerotic changes (d) Cutaneous application
(d) Tolerance 130. Activation of prothrombin to thrombin is
carried out by
125. While treating auricular fibrillation, digi-
talization is necessary prior to quinidine (a) Factor VIII (b) Factor V
because (c) Factor IX (d) All of the above
(a) It prevents paradoxical increase in ventricular 131. Which of the following is not a calcium
rate channel blocker?
(b) Quinidine is only effective in the presence
(a) Verapamil (b) Diltiazam
of digitalis
(c) Prenylamine (d) Propranolol
(c) It reduces the dose and side effects of
quinidine 132. In the treatment of cardiac shock, the
(d) It antagonizes the extracardiac effects of drug of choice is
quinidine (a) Dopamine
126. Which of the following pharmacological (b) Propranolol
properties of quinidine is not useful (c) Phenoxybenzamine
clinically? (d) Metaraminol
(a) Shorterning of A-V nodal refractory period
133. Polymorphic ventricular tachycardia can
due to vagolytic action
be a complication of
(b) Depression of cardiac contractility
(a) Loratadine (b) Cetirizine
(c) Decrease in the automaticity of the normal
(c) Astemizole (d) C + D
pacemaker
(d) Reduction in the slope of slow diastolic 134. In hypertension which drug besides
depolarization lowering blood pressure arrests and even
reverses cardiac hypertrophy
132 MCQs IN PHARMACOLOGY

(a) ACE inhibitors 138. Which of the following is directly acting


(b) Betablockers vascular smooth muscle relaxant ?
(c) Calcium channel blockers (a) Verapamil (b) Minoxidil
(d) Diuretics (c) Clonidine (d) Diazoxide
135. The ACE inhibitor useful in hypertensive 139. The most potent stimulant for heart is
emergencies is
(a) Adrenaline (b) Noradrenaline
(a) Enalaprilat (b) Benzalaprilat
(c) Dopamine (d) Ephedrine
(c) Fosinoprilat (d) Quinoprilat
140. Fibrinolysis is mainly carried out by
136. Effect of decreased vitamin K on action
of warfarin is (a) Tissue – type plasminogen activator
(a) Enhanced anticoagulatory effect (b) Urokinase – type plasminogen activator
(b) Decreased anticoagulatory (c) Both (a) and (b)
(c) No effect (d) None of the above
(d) Unpredictable effect
141. Following is a phenothiazine analog with
137. Nitroglycerin ointment in clinical use is sodium channel blocking properties
(a) 1% (b) 2% (a) Moricizine (b) Mexiletine
(c) 5% (d) 10% (c) Tocainide (d) Propafenone

ANSWERS
1. a 2. d 3. d 4. d 5. d 6. c
7. a 8. d 9. d 10. c 11. b 12. d
13. a 14. e 15. d 16. d 17. c 18. d
19. c 20. e 21. b 22. a 23. b 24. d
25. b 26. a 27. b 28. a 29. a 30. b
31. c 32. e 33. b 34. b 35. d 36. c
37. c 38. e 39. c 40. c 41. b 42. c
43. b 44. b 45. d 46.b 47.a 48. b
49. b 50. c 51. a 52. d 53. a 54. a
55. a 56. e 57. a 58. a 59. e 60. b
61. e 62. b 63. a 64. c 65. c 66. d
67. d 68. d 69. d 70. d 71. d 72. a
73. d 74. b 75. b 76. c 77. c 78. d
79. c 80. d 81. d 82. a 83. a 84. a
85. b 86. c 87. c 88. a 89. b 90. b
91. c 92. a 93. b 94. c 95. d 96. a
97. a 98. a 99. a 100. c 101. c 102. a
103. a 104. d 105. a 106. b 107. a 108. b
109. d 110. d 111. d 112. c 113. b 114. c
115. d 116. d 117. c 118. d 119. b 120. b
121. d 122. a 123. b 124. d 125. a 126. a
127. b 128. b 129. d 130. b 131. d 132. a
133. d 134. a 135. a 136. b 137. b 138. a
139. a 140. c 141. a
CARDIO–VASCULAR DRUGS 133

EXPLANATIONS FOR THE ANSWERS

1. a Arterial blood pressure is directly proportional other phosphodiesterase isoforms are as follows:
to the blood flow, i.e., cardiac output and Phosphodiesterase I : Vinopocetine
peripheral vascular resistance. This relationship
Phosphodiesterase II : Not available
is correctly represented by equation in ‘A’
Phosphodiesterase IV : Rolipram
7. d Moxonidine acts mainly on imidazline I 1
57. a Phase 2 of action potential, also known as
receptors and produces antihypertensive effect.
‘plateau phase’ involves inward calcium current,
Dexmedetomidine is a α2-adrenergic receptor which has a slower inactivation. This inward
agonist. calcium current is balanced by outward delayed
Tizanidine is a spasmolytic agent. rectifier potassium current.
12. d Both angiotensin II and III promote aldosterone 68. d Potassium channels play an important role in
release. Aldosterone promotes the reabsorption various aspects of action potential in the heart
of sodium by distal renal tubules and this may viz. pacemaker function, resting potential and
increase the plasma volume and hypertension. action potential duration.
Angiotensin I is inactive. 78. d β-Adrenergic stimulation, hypokalemia and
16. d Activation of ETA receptors leads to vasocon- mechanical stretch produce increase in phase
striction, bronchoconstriction and stimulation of 4 slope, leading to acceleration of pacemaker
aldosterone release. rate, which results in enhanced automaticity.
Affinity of various endothelins for ETA receptor 84. a In cardiac cells, adenosine acts on its specific
is ET1 = ET2 > ET3. membrane receptors and activates acetylcholine-
19. c Spironolactone binds with cytoplasmic sensitive potassium current, specifically in atrium,
mineralocorticoid receptors. It also decreases sinus and AV node. This results in shortening of
the interacellular formation of active metabolites action potential duration. Activation of potassium
of aldosterone. current also results in hyperpolarization and
Triamterene and amiloride directly interfere with slowing of automaticity.
sodium entry. 97. a Lidocaine blocks both open and inactivated
28. a Carbonic anhydrase inhibitors inhibit both the cardiac sodium channels. It decreases
membrane and cytoplasmic forms of carbonic automaticity mainly by reducing the slope of
anhydrase and thus completely inhibit phase 4 and alters the threshold for cardiac
reabsorption of sodium bicarbonate in the excitability.
proximal tuble. Lidocaine may shorten the duration of action
34. b Both glycerin and isosorbide are orally active potential because of blockade of sodium
osmotic diuretics and hence show good oral channels, which get inactivated late during
absorption. action potential,
Mannitol shows negligible absorption. 100. c Chylomicrons contain dietary triglycerides as
39. c Ethacrynic acid is a phenoxy acetic acid the major lipid class.
derivative. Furosemide and bumetanide contain VLDL contains endogenous triglycerides and IDL,
sulfonamide moiety in their structure. LDL and HDL contain cholesterol esters as major
40. c Spironolactone is an aldosterone antagonist. lipid class.
Aldosterone is a mineralocorticoid and miner- 108. d Nicotinic acid produces decrease in LDL levels
alcorticoid receptors are present in epithelial because of decreased VLDL production and
cells of late distal tubule and collecting duct. enhanced clearance of LDL precursors in liver.
50. c Milrinone is an inhibitor of phosphodiesterase VLDL levels are decreased because of decreased
III and it produces positive inotropism via delivery of free fatty acids to liver, decrease in
inhibition of cGMP. Examples of inhibitors of triglyceride synthesis and decrease in VLDL-
134 MCQs IN PHARMACOLOGY

triglyceride transport. Mechnism of increased enzymatic reduction of vitamin K to its active


levels of HDL by nicotinamide is not known but hydroquinone form and the inhibition is
decreased clearance of apo A-I and decreased competitive. Thus, if there are decreased levels
synthesis of apoA-II may play important role. of vitamin K obviously there will be less
113. b The entire sequence of conduction of action competition for inhibition resulting in enhanced
potential under normal condition is as follows: anticoagulatory effect of warfarin.
SA node → Atria → AV node → Bundle of His, 140. c Fibrinolysis (thrombolysis) is carried out by
Purkinje fibers → Ventricles. various endogenous plasminogen activators e.g.
tissue-type plasminogen activator, urokinase-type
221. d Sympathomimetic amines, eicosanoids and
plasminogen activator and also kallikrein and
endothelins act on their respective receptors and
neutrophil elastase.
increase the calcium influx thereby production
contraction of smooth muscles and hence 141. a Moricizine is a phenothiazine analogue with
vasoconstriction. sodium channel blocking activity and is used in
the treatment of ventricular arrhythmias.
130. b Prothrombin, which is bound to phospholipid
surface of platelets, is activated by factor Xa in Mexiletine and tocainide are analogous of
the presence of actor V to thrombin. lidocaine.
136. a Warfarin acts as an anticoagulant by inhibiting Propafenone has some structural similarities to
propranolol.
DRUGS USED IN RESPIRATORY DISORDERS 135

CHAPTER 7
DRUGS USED IN RESPIRATORY
DISORDERS

1. Which of the following terms best (d) Decreased calcium influx into the mast cells
describes the antagonism of leukotriene’s (e) Decreased prostaglandin production
bronchoconstrictor effect (mediated at
leukotriene receptors) by terbutaline 4. Which of the following will result from
(acting α adrenoceptors) in a patient with blockade of H2 receptors?
asthma?
(a) Decreased camp in cardiac muscle
(a) Pharmacologic antagonist (b) Increased camp in cardiac muscle
(b) Partial agonist (c) Decreased IP3 in gastric mucosa
(c) Physiologic antagonist
(d) Increased IP3 in gastric mucosa
(d) Chemical antagonist
(e) Increased IP3 in smooth muscle
(e) Noncompetitive antagonist
5. Toxicities of H2 antihistamines include
2. If therapy with multiple drugs causes
which one of the following?
induction of drug metabolism in your
asthma patient, it will (a) Blurred vision
(a) Result in increased smooth endoplasmic (b) Diarrhea
reticulum (c) Orthostatic hypotension
(b) Result in increased rough endoplasmic (d) P450 inhibition
reticulum (e) Sleepiness
(c) Result in decreased enzymes in the soluble
cytoplasmic fraction 6. A patient undergoing cancer chemo-
(d) Require 3-4 months to reach completion therapy is vomiting frequently. A drug that
(e) Be irreversible might help in this situation is
(a) Bromocriptine (b) Cimetidine
3. The Symptoms of allergen-mediated
asthma result from which of the (c) Ketanserin (d) Loratadine
following? (e) Ondansetron
(a) Increased release of mediators from mast 7. Which of the following is most useful in
cells the treatment of hyperprolactinemia ?
(b) Increased adrenergic responsiveness of the
airways (a) Bromocriptine (b) Cimetidine
(c) Increased vascular permeability of bronchial (c) Ergotamine (d) Ketanserin
tissue (e) LSD
136 MCQs IN PHARMACOLOGY

8. Drugs that can dilate bronchi during an 15. Which one of the following drugs is most
acute asthmatic attack include all of the suitable for management of essential
following except tremor in a patient who has pulmonary
(a) Epinephrine (b) Terbutaline disease?
(c) Nedocromil (d) Theophyline (a) Diazepam (b) Levodopa
(e) Ipratropium (c) Metoprolol (d) Propranolol
(e) Terbutaline
9. Which of the following is a nonselective but
very potent and efficacious bonchodilator 16. A drug useful in the treatment of asthma
that is not active by the oral route? but lacking bronchodilator action, is
(a) Aminophyline (b) Cromolyn (a) Cromolyn (b) Ephedrine
(c) Epinephrine (d) Ipratropium (c) Isoproterenol (d) Metaproterenol
(e) Metaproterenol (e) Metoprolol
10. Which of the following is a prophylactic 17. Relative to fexofenadine, diphenhy-
agent that appears to stabilize mast cells? dramine is more likely to
(a) Aminophyline (b) Cromolyn (a) Be used for treatment of asthma
(c) Epinephrine (d) Ipratropium (b) Be used for treatment of gastroesophageal
(e) Metaproterenol reflux disease
(c) Cause cardiac arrhythmias in overdose
11. Which of the following is a direct bron-
(d) Have efficacy in the prevention of motion
chodilator that is most often used in
sickness
asthma by the oral route?
(e) Increase the serum concentration of warfarin
(a) Aminophyline (b) Cromolyn
(c) Epinephrine (d) Ipratropium 18. Bacteria that make either a fementative
(e) Metaproterenol or respiratory set of enzymes are known
as
12. Acute exacerbations of asthma can be
(a) Obligate anacerobes
triggered by all of the following except
(b) Obligate aerobes
(a) Bacterial of viral pneumonia
(c) Microacerophiles
(b) Hypertsensitivity reaction to penicillin
(d) Facultative organisms
(c) Discontinuation of asthma medication
(d) Hot, dry weather 19. The symptoms of allergen-mediated asthma
result from which of the following?
(e) Stressful emotional events
(a) Increased release of mediators from mast
13. Which of the following has overdose cells
toxicity that includes insomnia, arrhy-
(b) Increased adrenergic responsiveness of the
thmias, and convulsions?
airways
(a) Aminophyline (b) Cromolyn (c) Inceased vascular permeability of bronchial
(c) Epinephrine (d) Ipratropium tissue
(e) Metaproterenol (d) Decreased calcium influx into the mast cell
14. Which of the following is a very long- (e) Decreased prostaglandin production
acting β2 – selective agonist that is used 20. Acute exacerbations of asthma can be
for asthma prophylaxis? triggered by all of the following except
(a) Aminophyline (b) Cromolyn (a) Bacterial or viral pneumonia
(c) Epinephrine (d) Ipratropium (b) Hypersensitivity reaction to penicillin
(e) Salmeterol (c) Discontinuation of asthma medication
DRUGS USED IN RESPIRATORY DISORDERS 137

(d) Hot, dry weather 26. Terbutaline has a preference for stimulation
(e) Stressful emotional events of which of the following receptors?
(a) Alpha (b) Gamma
21. In the emergency department, the preferred
first-line therapy for asthma exacerbation (c) Beta 1 (d) Beta 2
is (e) Dopaminergic
(a) Theophyline (b) A β-agonist 27. The National Institutes of Health (NIH)
(c) A corticosteroid (d) Cromolyn sodium guidelines for the treatment of asthma
(e) An antihistamine recommended institution of routine
inhaled corticosteroids when patients are
22. Which of the following tests is used at home classified as having greater than or equal
to assess therapy and determine if a patient to which type of asthma?
with asthma should seek emergency care?
(a) Mild intermittent
(a) Forced expiratory volume in one second (b) Mild persistent
(FEV1)
(c) Moderate persistent
(b) Forced vital capacity (FVC)
(d) Severe persistent
(c) Total lung capacity (TLC)
(d) Peak expiratory flow rate (PEFR) 28. Isoxuprine is used to treat
(e) Residual volume (RV) (a) Asthma (b) Severe hypotension
(c) Nasal congestion (d) Premature labor
23. Which of the following agents and
dosage regimens is the best choice of (e) Hypertension
treatment for an asthma patient with 29. The most likely complication of prolonged
rheumatoid arthritis who is considered use of nasal decongestant drops is
sensitive to aspirin (experiences bron-
(a) Atrophic rhinitis
chospasm with use)?
(b) Hypertrophy of nasal mucosa
(a) Lbuprofen, 800 mg three times daily
(c) Naso-pharyngeal moniliasis
(b) Acetaminophen, 650 mg every 4 hours
(d) Blockage of Eustachian tubes
(c) Gold injections, 25 mg intramuscularly once
a week 30. Which of the following is a selective H1
(d) Azathioprine, 75 mg daily receptor agoinist ?
(e) Cyclophosphamide, 100 mg daily (a) 4-methyl histamine
(b) Impromidine
24. A drug administered by inhalation of
(c) 2-Thiazolyl ethylamine
powder as a prophylactic for asthma is
(d) Mepyramine
(a) Ephedrine (b) Disodium cromolyn
(c) Isoproterenol (d) Ocytriphylline 31. Fall in blood pressure caused by larger
(e) Epinephrine doses of histamine is blocked by
(a) H1 antihistaminics alone
25. Which of the following may precipitate
(b) H2 ANTAgonists alone
an asthma attack?
(c) Combination of H1 and H2 antagonists
(a) Respiratory acidosis
(d) None of the above
(b) Viran and bacterial infections
(c) Respiratory alkalosis 32. Histamine is involved as a mediator in
the following pathological conditon
(d) Cranberry jiuce
(e) Chocolate or Coca Cola (a) Cocaine (b) Dibucaine
(c) Lidocaine (d) Procaine
138 MCQs IN PHARMACOLOGY

33. In the emergency department, the 40. The action of 5-Hydroxy tryptamine
preferred first-line therapy for asthma mediated by the 5-HT3 receptor is
exacerbation is (a) Vasoconstriction (b) Bradycardia
(a) Theophylline (b) A β-agonist (c) EDRF release (d) Platelet aggregation
(c) A corticosteroid (d) Cromolyn sodium
41. Tachyphylaxis to many actions on
(e) An antihistamine
repeated injection is a feature of the
34. The capacity of an antihistaminic to produce following autocoid
sedation depends on the following except (a) Histamine (b) 5-Hydroxytryptamine
(a) Relative affinity for central versus peripheral (c) Bradykinin (d) Angiotensin
H1 receptors
42. The smooth muscle stimulating action of
(b) Ability to penetrate blood-brain barrier
5-HT is most marked in the
(c) Individual susceptibility
(a) Bronchi (b) Intestines
(d) Ratio of H1 : H2 blockade produced by the drug
(c) Ureter (d) Billary tract
35. While prescribing the conventional H1
43. The 5-HT antagonist that has anti-
antihistaminics the patient should be
hypertensive property is
advised to avoid
(a) Methysergide (b) Cyproheptadine
(a) Driving motor vehicles
(c) Ketanserin (d) Ondansetron
(b) Consuming processed cheese
(c) Strenuous physical exertion 44. The most important receptor involved in
(d) All of the above cytotoxic drug induced vomiting is
(a) Histamine H1 receptor
36. The antihistaminc which has calcium
channel blocking and labyrinthine (b) Serotonin 5-HT3 receptor
suppressant property is (c) Dopamine D2 receptor
(a) Cyproheptadine (b) Cinnarizine (d) Opioid µ receptor
(c) Clemastine (d) Cetirizine 45. Which of the following expectorants acts
both directly on the airway mucosa as
37. Erythromycin should not be given to patient
well as reflexly ?
being treated with terfenadine because
(a) Potassium iodide (b) Guaiphenesin
(a) Erythromycin induces the metabolism of
terfenadine (c) Terpin hydrate (d) Bromhexine
(b) Dangerous ventricular arrhythmias can occur 46. Bromhexine acts by
(c) Terfenadine inhibits metabolism of erythromycin (a) Inhibiting cough center
(d) Terfenadine antagonizes the antimicrobal (b) Irritating gastric mucosa and reflexly
action of erythromycin increasing bronchial secretion
38. Select the H1 antihistaminic which is used (c) Depolymerizing mucopolysaccharides present
topically in the nose for allergic rhinitis (d) Desensitizing stretch receptors in the lungs
(a) Deep intra-abdominal operation 47. The primary goals of asthma therapy
(b) Trachial intubation include all of the following except
(c) Tetanus (a) maintain normal activity levels
(d) Diagonosis of myasthenia gravis (b) maintain control of symptoms
39. H1 antihistaminics are beneficial in (c) avoid adverse effects of asthma medications
(a) All types of allergic disorders (d) prevent acute exacerbations and chronic
symptoms
(b) Certain type I allergic reactions only
(e) prevent destruction of lung tissue
(c) Anaphylactic shock(d) Bronchial asthma
DRUGS USED IN RESPIRATORY DISORDERS 139

48. Antitussives act by (b) It is a bronchodilator with anti-inflammatory


(a) Liquifying bronchial secretions property
(b) Raising the threshold of cough centre (c) It is a β blocker that can be safely given to
asthmatics
(c) Reducing cough inducing impulses from the
lungs (d) It is an antihistaminic with mast cells stabilizing
(d) Both (b) and (c) property

49. Which of the following antitussive is 55. Caffeine is more powerful than theophylline
present in opium but has no analgesic or in exerting the following action
addicting properties ? (a) Bronchodilatation
(a) Noscapine (b) Codeine (b) Cardiac stimulation
(c) Pholcodeine (d) Ethylmorphine (c) Diuresis
(d) Augmentation of skeletal muscle contractility
50. Bronchodilators are useful in cough
(a) Only when cough is non-productive 56. Methylxanthines exert the following
(b) Only when cough is associated with thick action (s) at cellular/molecular level
sticky secretions (a) Intracellular release of Ca2+
(c) Only when reflex bronchoconstriction is (b) Antagonism of adenosine
associated (c) Inhibition of phosphodiesterase
(d) Irrespective of nature of cough or associated (d) All of the above
features
57. Relatively higher dose of theophylline is
51 The common and dose related side effect required to attain therapeutics plasma
of salbutamol is concentration in
(a) Rise in blood pressure (a) Smokers
(b) Muscle tremor (b) Congestive heart failure patients
(c) Hyperglycaemia
(c) Those receiving erythromycin
(d) Central nervous system stimulation
(d) Those receiving cimetidine
52. Which of the following tests is used at home
58. The antiasthma drug which cannot be
to assess therapy and determine if a patient
administered by inhalation is
with asthma should seek emergency care?
(a) Theophylline (b) Ipratropium bromide
(a) Forced expiratory volume in one second (FEV1)
(c) Budesonide (d) Terbutaline
(b) Forced vital capacity (FVC)
(c) Total lung capacity (TLC) 59. A 10 year old child suffers from exercise
(d) Peak expiratory flow rate (PEFR) induced asthma: Which is the most suit-
(e) Residual volume (RV) able first line drug for regular prophy-
lactic therapy ?
53. In a patient of bronchial asthma, inhaled
(a) Oral salbutamol
salbutamol produces the following
effect(s). (b) Oral theophylline
(c) Inhaled sodium cromoglycate
(a) Inhibits antigen-antibody reaction in the lungs
(d) Inhaled beclomethasone dipropionate
(b) Causes bronchodilatation
(c) Reduces bronchial hyperreactivity 60. Which of the following drugs is neither
(d) Both (b) and (c) bronchodilator nor anti-inflammatory,
but has antihistaminic and mast cell
54. Select the correct statement about salmeterol. stabilizing activity ?
(a) It is a long acting selective β2 agonist bron- (a) Sodium cromoglycate
chodilator
(b) Ketotifen
140 MCQs IN PHARMACOLOGY

(c) Beclomethasone dipropionate 67. Histamine


(d) Mepyramine maleate (a) May be released from mast cells by a number
of therapeutic agents
61. The most consistent, pronounced and
sustained relief of symptoms in chronic (b) Causes sedation
bronchial asthma is afforded by (c) Decreases the force of contraction of
ventricular muscle
(a) β2 sympathomimetics
(d) Can cause strong contractions of the gravid
(b) Anticholinergics human uterus
(c) Sodium cromoglycate
68. Chronic Bronchitis is characterized by
(d) Corticosteroids
I. the distruction of central and peripheral
62. Intransasal spray of budesonide is portions of the acinus
indicated in
II. an increased number of mucous glands and
(a) Common cold goblet cells
(b) Acute vasomotor rhinitis III. edema and inflammation of the bronchioles
(c) Perennial vasomotor rhinitis (a) only I is correct
(d) Epistaxis (b) only III is correct
(c) I and II are correct
63. In patients of bronchial asthma inhaled
corticosteroids achieve the following (d) II and III are correct
except (e) I, II and III are correct
(a) Reduce the need for bronchodilator 69. Smooth muscle relaxation is due to
medication stimulation of which type of histamine
(b) Control an attack of refractory asthma receptors
(c) Reduce bronchial hyperreactivity (a) H1 (b) H2
(d) Reverse diminished responsiveness to (c) H3 (d) All of the above
sympathomimetic bronchodilators 70. All the following are employed in
64. Budesonide is a inhalation therapy of asthma except
(a) Nonsteroidal anti-inflammatory drug (a) Beclomethasone (b) Budesonide
(b) High ceiling diuretic (c) Dexamethasone (d) Triamcinolone
(c) Inhaled corticosteroid for asthma 71. Which xanthine derivative has no function
(d) Contraceptive in asthma ?

65. In an asthma patient treated with (a) Theophylline (b) Pentoxyphyllin


systemic corticosteroids, bronchodilator (c) Enprofylin (d) None of the above
drugs 72. What is symport ?
(a) Are not needed (a) Counter transport
(b) Are contraindicated (b) Contransport
(c) May be used on ‘as and when required’ basis (c) Carrier mediated diffusion
(d) Are ineffective (d) Solvent drug
66. To be a useful inhaled glucocorticoid the 73. Pyridostigmine differs from neostigmine
drug should have in that
(a) High oral bioavailability (a) More potent orally
(b) Low oral bioavailability (b) Longer acting
(c) Additional bronchodilator activity (c) Less muscarinic side effects
(d) Prodrug character (d) All of the above
DRUGS USED IN RESPIRATORY DISORDERS 141

ANSWERS
1. c 2. a 3. a 4. a 5. d 6. e
7. a 8. c 9. c 10. b 11. a 12. d
13. a 14. e 15. c 16. a 17. d 18. d
19. a 20. d 21. b 22. d 23. c 24. b
25. b 26. d 27. b 28. d 29. a 30. c
31. c 32. b 33. b 34. d 35. a 36. b
37. b 38. d 39. b 40. b 41. b 42. b
43. c 44. b 45. a 46. c 47. e 48. d
49. a 50. c 51. b 52. d 53. b 54. a
55. d 56. d 57. a 58. a 59. c 60. b
61. d 62. c 63. b 64. c 65. c 66. b
67. d 68. c 69. b 70. c 71. b 72. b
73. b

EXPLANATIONS FOR THE ANSWERS


3. a In asthma, airborne antigen binds to the mast onstriction. After an acute attack, in most cases
cell, activating the immunoglobulin E (IgE)- symptoms are minimal, and pathological chang-
mediated process. Mediators (e.g., histamine, es are not permanent. Unlike asthma, chronic
leukotrienes, prostaglandins) are then released, obstructive pulmonary disease does cause pro-
causing bornchoconstriction and tissue edema. gressive airway destruction, chronic bronchitis
12. d Exacerbations of asthma can be triggered by by excessive mucus production and other chang-
allergens, respiratory infections, occupational es, and emphysema by destruction of the aci-
stimuli (e. g., fumes from gasoline or paint), nus.
emotions, and environmental factors. Studies 52. d In monitoring of asthma therapy at home, peak
have shown that cold air can cause release of expiratory flow rate (PEFR) is the best test for
mast cells mediators by an undetermined assessment of therapy, trigger identification, and
mechanism. Hot, dry air does not cause this the need for referral to emergency care. It is
release. recommended for patients who have had severe
27. b The National Institutes of Health (NIH) exacerbations of asthma, who are poor
guidelines recommend routine use of inhaled perceivers of asthma symptoms, and those with
corticosteroids in patients who have mild moderate to severe disease.
persistent asthma. Short-acting β-agonist 68. d Chronic bronchitis is characterized by an
administered in a nebulizer or administered as increase in the number of mucous and goblet
a subcutaneous agent is the most appropriate cells due to bronchial irritation. This results
first line therapy. in increased mucus production. Other
33. b In an emergency situation, the most rapidly changes include edema and inflammation of
acting agent is used first. Selection of the route the bronchioles and changes in smooth
of administration depends on the severity of the muscle and cartilage. Emphysema is a
attack. An inhaled β-agonist administered in a permanent destruction of the central and
nebulizer or administered as a subcutaneous peripheral portions of the acinus distal to the
agent is the most appropriate first-line therapy. bronchioles. In this disease, adequate
47. e Asthma is characterized by reversible airway oxygen reaches the alveolar duct, due to
obstruction in response to specific stimuli. Mast increased rate of breathing, but perfusion is
cells release mediators, which trigger bronchoc- abnormal.
This page
intentionally left
blank
CHAPTER 8
ANTIBIOTICS

1. Isoniazid is a primary antitubercular (a) It is active against MRSA strains


agent that (b) It is the drug of choice in community-acquired
(a) Requires pyridoxine supplementation pneumonia
(b) May discolor the tears, saliva, urine or feces (c) It is a fourth-generation cephalosporin
orange-red (d) It decreases prothrombin time
(c) Causes ocular complications that are (e) Its antibacterial spectrum include Bacteroides
reversible if the drug is discontinued fragilis
(d) May be ototoxic and nephrotoxic 4. A patient needs antibiotic treatment for
(e) Should never be used due to hepatotoxic native valve, culture-positive infective
potential enterococcal endocarditis. His medical
history includes a severe anaphylactic
2. A 36-year-old woman recently treated for
reaction to penicillin G during the past year.
leukemia is admitted to hospital with
The best approach would be treatment with
malaise, chills and high fever. Gram stain
of blood reveals the presence of gram- (a) Amoxicillin/clavulanate
negative bacilli. The initial diagnosis is (b) Aztreonam
bacteremia and parenteral antibiotics are (c) Cefazolin plus genatamicin
indicated. The records of the patient (d) Meropenem
reveal that she had a severe uriticarial
(e) Vancomycin
risk, hypotension, and respirator y
difficulty following oral penicillin V about 5. This drug has activity against many strains
6 months ago. The most appropriate drug of Pseudomomas aeruginosa. However,
regimen for empiric treatment is when it is used alone, resistance has
emerged during the course of treatment.
(a) Ampicillin plus sulbactam
The drug should not be used in penicillin-
(b) Aztreonam allergic patients. Its activity against gram-
(c) Cefazolin negative rods is enhanced if it is given in
(d) Imipenem plus cilastatin combination with tazobactam
(e) Ticarcillin plus clavulanic acid (a) Amoxicillin (b) Aztreonam
3. Which one of the following statements (c) Imipenem (d) Piperacillin
about cefotetan is accurate? (e) Vancomycin
144 MCQs IN PHARMACOLOGY

6. Which of the following statements about (a) Methylation of binding sites on the 50S
vancomycin is accurate? ribosomal subunit
(a) It is bacteriostatic (b) Formation of esterases that hydrolyze the
lactone ring
(b) It binds to PBPs
(c) Increased activity of efflux mechanisms
(c) It is not susceptible to penicillinase
(d) Formation of drug-inactivating acetyltran-
(d) It has the advantage of oral bioavailability
ferases
(e) Staphylococcal enterocolitis occurs
(e) Decreased drug permeability of the
commonly with its use
cytoplasmic membrane
7. Which one of the following statements
11. The appearance of markedly vacuolated,
about ampicillin is false?
nucleated red cells in the marrow, anemia,
(a) Its activity is enhanced by sulbactam and reticulocytopenia are characteristic
(b) It causes maculopapular rashes dose-dependent side effects of
(c) It is the drug of choice for Listeria mono- (a) Azithromycin (b) Chloramphenicol
cytogenes infection (c) Clindamycin (d) Doxycycline
(d) It eradicates most strains of MRSA (e) Linezolid
(e) Pseudomembranous colitis may occur with
its use 12. In a patient with culture-positive entero-
coccal endocarditis who has failed to
8. The mechanism of antibacterial action of respond to vancomycin because of
tetracycline involves resistance, the treatment most likely to
(a) Binding to a component of the 50S ribosomal be effective is
subunit (a) Clarithromycin (b) Erythromycin
(b) Inhibition of translocase activity (c) Linezolid (d) Minocycline
(c) Blockade of binding of aminoacyl – tRNA to (e) Ticarcillin
bacterial ribosomes
13. All of the following factors may increase
(d) Selective inhibition of ribosomal peptidyl the risk of nephrotoxicity from gentamicin
transferases therapy except
(e) Inhibition of DNA–dependent RNA
(a) age over 70 years
polymerase
(b) prolonged courses of gentamicin therapy
9. Clarithromycin and erythromycin have very (c) concurrent amphotericin B therapy
similar spectrums of antimicrobial activity. (d) trough gentamicin levels below 2 mg/ml
The major advantage of clarithromytcin is
(e) concurrent cisplatin therapy
that it
14. Which one of the following antibiotics is
(a) Eradicates mycoplasmal infections in a single
likely to be most effective agent in the
dose
treatment of an infection due to enterococci
(b) Is active against strains of streptococci that if used in conjuction with penicillin G?
are resistant to erythromycin
(a) Amikacin (b) Gentamicin
(c) Is more active against Mycobacterium avium
(c) Netilmicin (d) Streptomycin
complex
(e) Tobramycin
(d) Does not inhibit live drug-metabolizing enzymes
(e) Acts on methicillin-resistant strains of 15. Regarding the antibacterial action of
staphylococci gentamicin, which one of the following
statements is most accurate?
10. The primary mechanism underlying the
(a) Efficacy is directly proportionate to the time
resistance of gram-positive organisms to that the plasma level of the drug is greater
macrolide antibiotics is than the minimal inhibitory concentration
ANTIBIOTICS 145

(b) The antibacterial action of gentamicin is not (d) It is often less toxic than conventional
concentration - dependent (multiple)dosing regimens
(c) Gentamicin continues to exert antibacterial (e) Under-dosing is less of a problem
effects even after plasma levels decrease
below detectable levels 21. Regarding the recently introduced lipid
formulation of amphotericin B, which one
(d) Antibacterial activity is often reduced by the
of the following statements is accurate?
presence of an inhibitor of cell wall synthesis
(e) The antibacterial action of gentamicin is time (a) Affinity of amphotericin B for these lipids is
– dependent greater than affinity for ergosterol
(b) They are less expensive to use than conventional
16. Which one of the following drugs is most amphotericin B
likely to be effective against multidrug –
(c) They are more effective in fungal infections
resistant strains of M tuberculosis,
because they increase tissue uptake of
including those resistant to streptomycin?
amphotericin B
(a) Amikacin (b) Clarithromycin
(d) They may decrease nephrotoxicity of
(c) Gentamicin (d) Meropenem amphotericin B
(e) Spectinomycin (e) They have wider spectrums of antifungal
17. Sterptomycin has no useful activity in the activity than conventional formulations of
treatment of amphotericin B
(a) Bubonic plague (b) Brucellosis 22. Which one of the following antimicrobial
(c) Lyme disease (d) Tuberculosis drugs does not require supplementation
(e) Tularemia of dosage following hemodialysis?
(a) Ampicillin (b) Cefazolin
18. Your 23-year-old female patient is
pregnant and has gonorrhea. The past (c) Ganciclovir (d) Tobramycin
medical history includes anaphylaxis (e) Vancomycin
following exposure to amoxicillin. 23. The persistent suppression of bacterial
Worried about compliance, you would like
growth that may occur following limited
to treat this patient with a singly dose, so
exposure to some antimicrobial drugs is
you chose
called
(a) Cefixime (b) Ceftriaxone
(a) Time-dependent killing
(c) Ciprofloxacin (d) Spectinomycin
(b) The post antibiotic effect
(e) Tetracycline
(c) Clinical synergy
19. In the empiric treatment of severe bacterial (d) Concentration – dependent killing
infections of unidentified entiology, this (e) Sequential blockade
drug, often used in combination with an
aminoglycoside, provides coverage against 24. In which of the following groups do all four
many staphylococci drugs warrant careful monitoring for drug-
related seizures in high-risk patients?
(a) Amoxicillin (b) Clavulanic acid
(c) Erythromycin (d) Nafcillin (a) Penicillin G, imipenem, amphotericin B,
(e) Tetracycline metronidazol
(b) Penicillin G, chlormphenicol, tetracycline,
20. Which one of the following statements vancomycin
about “one daily” dosing with aminogly-
(c) Imipenem, tetracycline, vancomycin,
cosides is false ?
sulfadiazine
(a) It is convenient for outpatient therapy (d) Cycloserine, metronidazole, vancomycin,
(b) Adjustment of dosage is less important in sulfadiazine
renal insufficiency (e) Metronidazole, imipenem, doxycycline,
(c) Less nursing time is required erythromycin
146 MCQs IN PHARMACOLOGY

25. Which one of the following antibiotics is a and a Gram stain reveals many white
potent inducer of hepatic drug-metabolizing cells but no organisms. Since this patient
enzymes? appears to have atypical pneumonia. You
(a) Ciprofloxacin (b) Cyclosporine should initiate treatment with
(c) Erythromycin (d) Rifampin (a) Cefazolin (b) Clindamycin
(e) Tetracycline (c) Erythromycin (d) Gentamicin
(e) Trovafloxacin
26. Beta – lactamase production by strains
of Haemophilus influenzae, Moraxella 30. Chloramphenicol is particularly effective
catarrhalis, and Neissera gonorrhoeae in the treatment of
confers resistance against penicillin G. (a) Diphtheria (b) Tuberculosis
which one of the following antibiotics is (c) Emphysema (d) Rickettsial diseases
most likely to be effective against all (e) Streptomyces pyogenes infections
strains of each of the above organisms?
31. An advantage of bleomycin over most
(a) Ampicillin (b) Ceftriaxone
antineoplastic drugs is its
(c) Clindamycin (d) Gentamicin
(a) Rapid onset of action
(e) Piperacillin
(b) Lack of toxicity to all normal cells
27. A 19-year-old woman with recurrent (c) Relative lack of bone marrow toxicity
sinusitis has been treated with different (d) Very broad spectrum of activity
antibiotics on several occasions. During
(e) Both (a) and (b)
the course of one such treatment she
developed a severe diarrhea and was 32. The drug of choice for scarlet fever is
hospitalized. Sigmoidoscopy revealed (a) Tetracycline (b) Sulfonamides
colitis, and pseudomembranes, were (c) Penicillin (d) Chloromycetin
confirmed histologically. Which of the
(e) Novobiocin
following drugs, administered orally, is
most likely to be effective in the treatment 33. An advantge of betamethasone acetate
of colitis due to C difficile? over betamethasone sodium phosphate
is
(a) Ampicillin (b) Cefazolin
(c) Clindamycin (d) Metonidazole (a) Fewer allergic reactions
(e) Tetracycline (b) Prompter action
(c) More sustained action
28. Bleomycin is used in most effective drug
(d) Greater solubility
combination regiment for the chemother-
(e) There is no advantage
apy of testicular carcinoma. Which one
of the following statements about the 34. Cyclosporine is used for
drug is accurate? (a) Allergies
(a) Acts mainly in the M phase of the cell cycle (b) Angina
(b) Derived from the bark of yew trees (c) Prevention of transplant rejection
(c) Myelosuppression is dose-limiting (d) Steroid deficiency
(d) Peripheral neuropathy occurs in more than (e) Treating led poisoning
50% of patients
35. The plasma half life of pencillin-G is
(e) Pulmonary infiltrates and fibrosis may occur longer in the new born because their
29. A high school student presents with head- (a) Plasma protein level is low
ache, fever, and cough of 2 day’s dura- (b) Drug metabolizing enzymes are immature
tion. Sputum is scant and nonpurulent
ANTIBIOTICS 147

(c) Glomerular filtration rate is low acquisition of multidrug resistance among


(d) Tubular transport mechanisms are not well bacteria ?
developed (a) Mutation (b) Conjugation
36. Choose the correct statement about the (c) Transduction (d) Transformation
role of opioid antimotility drugs in the 42. Methicillin resistant staphylococci do not
management of diarrhoeas
respond to β-lactam antibiotics because
(a) They are used to control diarrhoea irrespective
(a) They produce a β-lactamase which destroys
of its etiology
methicillin and related drugs
(b) They should be used only as a short-term
(b) They elaborate an amidase which destroys
measure after ensuring that enteroinvasive
methicillin and related drugs
organisms are not involved
(c) They have acquired a penicillin binding
(c) They are used as adjuvants to antimicrobial
protein which has low affinity for β-lactam
therapy of diarrhoea
antibiotics
(d) They are the drugs of choice in irritable bowel
(d) They are less permeable to β-lactam antibiotics
syndrome diarrhoea
43. A man has an Escherichia coli bacteremia
37. Spectinomycin is an aminoglycoside-like
with a low-grade fever (101.6°F).
antibiotic indicated for the treatment of
Appropriate management of his fever
(a) Gram-negative bacillary septicemia would be
(b) Tuberculosis
(a) Give acetaminophen 650 mg orally every 4
(c) Penicillin-resistant gonococcal infections hours
(d) Syphilis (b) Give aspirin 650 mg orally every 4 hours
(e) Gram-negative meningitis due to susceptible (c) Give alternating doses of aspirin and
organisms acetaminophen every 4 hours
38. Select the antibiotic with a high therapeutic (d) Withhold antipyretics, and use the fever curve
index to monitor his response to antibiotic therapy
(a) Streptomycin (b) Doxy (e) Use tepid water baths to reduce the fever
(c) Cephalexin (d) Polymyxin B 44. Superinfections are more common with
39. Which of the following organisms is (a) Use of narrow spectrum antibiotics
notorious for developing antimicrobial (b) Short courses of antibiotics
resistance rapidly ? (c) Use of antibiotics that are completely absorbed
(a) Streptococcus pyogenes from the small intestines
(b) Meningococcus (d) Use of antibiotic combinations covering both
(c) Treponema pallidum gram positive and gram negative bacteria
(d) Escherichia coli 45. Select the antibiotic whose dose must be
40. Widespread and prolonged used of an reduced in patients with renal insufficiency
antibiotic leads to emergence of drug (a) Ampicillin (b) Chloramphenicol
resistant strains because antibiotics (c) Tobramycin (d) Erythromycin
(a) Induce mutation in the bacteria
46. Which antimicrobial should be avoided
(b) Promote conjugation among bacteria
in patients of liver disease ?
(c) Allow resistant strains to propagate preferentially
(a) Chlortetracycline
(d) All of the above
(b) Cotrimoxazole
41. Of the following which is the most (c) Cephalexin
important mechanism of concurrent
(d) Ethambutol
148 MCQs IN PHARMACOLOGY

47. What is break point concentration of an 52. Adverse effects of ciprofloxacin are
antibiotic ? referable primarily to the following except
(a) Concentration at which the antibiotic lyses the (a) Gastrointestinal tract
bacteria (b) Kidney
(b) Concentration of the antibiotic which demarks (c) Skin
between sensitive and resistant bacteria (d) Nervous system
(c) Concentration of the antibiotic which 53. A single oral dose of the following drug
overcomes bacterial resistance can cure most cases of uncomplicated
(d) Concentration at which a bacteriostatic antibiotic gonorrhoea
becomes bactericidal (a) Ciprofloxacin (b) Cotrimoxazole
48. Which type of antimicrobial drug (c) Spectinomycin (d) Doxycycline
combination is most likely to exhibit 54. Which fluoroquinolone has markedly
antagonism ? enhanced activity against gram positive
(a) Bactericidal + Bactericidal bacteria and anaerobes ?
(b) Bactericidal + Bacteriostatic for a highly (a) Pefloxacin (b) Ciprofloxacin
sensitive organism (c) Sparfloxacin (d) Norfloxacin
(c) Bactericidal + Bacteriostatic for a marginally
55. A woman has an upper respiratory in-
sensitive organism fection. Six years ago, she experienced
(d) Bacteriostatic + Bacteriostatic an episode of bronchospasm follwing
penicillin V therapy. The cultures now
49. Indicate the sulfonamide whose sodium
reveal a strain of streptococcus pneumo-
salt yields a nearly neutral solution which
niae that is sensitive to all of the follow-
is suitable for topical use in the eye ing drugs. Which of these drugs would
(a) Sulfadiazine (b) Sulfacetamide be the best choice for this patient?
(c) Sulfamerazine (d) Sulfamethizole (a) Amoxicillin/clavulanate
(b) Erythromycin
50. Sulfamethoxypyridazine and other
related long acting sulfonamides have (c) Ampicillin
now gone into disuse because (d) cefaclor
(e) Cyclacillin
(a) They have produced serious cutaneous
reactions 56. Important microbiological features of
(b) They have produced high incidence of ciprofloxacin include the following except
crystalluria (a) Long postantibiotic effect
(c) They interact with many drugs (b) Marked suppression of intestinal anaerobes
(d) They do not penetrate blood-brain barrier (c) MBC values close to MIC values
(d) Slow development of resistance
51. Which of the following is not true of
sulfonamides ? 57. Currently the drug of choice for emperic
(a) They are primarily metabolized by acetylation treatment of typhoid fever is
(b) They are more likely to produce crystalluria (a) Chloramphenicol (b) Cotrimoxaxole
in alkaline urine in which they are less soluble (c) Ciprofloxacin (d) Ampicillin
(c) They may exert bactericidal action in the 58. In the treatment of typhoid fever, ciprof-
urinary tract loxacin has the following advantage(s)
(d) Used alone, they have become therapeutically (a) It is effective in nearly all cases
unreliable for serious infections (b) Early abetment of fever and other symptoms
ANTIBIOTICS 149

(c) Development of carrier state is less likely (d) All natural and semisynthetic penicillins are
(d) All of the above contraindicated for him

59. Penicillins interfere with bacterial cell wall 64. The most important reason for highly
synthesis by restricted use of penicillin G injections in
present day therapeutics is its
(a) Inhibiting synthesis of N-acetyl muramic acid
pentapeptide (a) Narrow spectrum of activity
(b) Inhibiting conjugation between N-acetyl (b) Potential to cause hypersensitivity reaction
muramic acid and N-acetyl glucosamine (c) Short duration of action
(c) Inhibiting transpeptidases and carboxy (d) Neurotoxicity
peptidases which cross link the peptidoglycan
65. Benzathine penicillin injected once every
residues
4 weeks for 5 years or more is the drug
(d) Counterfeiting for D-alanine in the bacterial of choice for
cell wall
(a) Agranulocytosis patients
60. All of the following drugs are suitable oral (b) Prophylaxis of bacterial endocarditis in
therapy for a lower urinary tract infection patients with valvular defects
due to Pseudomonas aeruginosa except (c) Prophylaxis of rheumatic fever
(a) Norfloxacin (d) Treatment of anthrax
(b) Trimethoprim-sulfamethxazole
66. Which of the following is not a semisyn-
(c) Ciprofloxacin
thetic penicillin ?
(d) Carbenicillin
(a) Procaine penicillin(b) Ampicillin
(e) Methenamine mandelate
(c) Cloxacillin (d) Carbenicillin
61. Gram negative organisms are largely
insensitive to benzyl penicillin because 67. Cloxacillin is indicated in infections
caused by the following organism(s)
(a) They produce large quantities of penicillinase
(a) Staphylococci (b) Streptococci
(b) They do not utilize D-alanine whose
(c) Gonococci (d) All of the above
incorporation in the cell wall is inhibited by
benzyl penicillin 68. The most frequent side effect of oral
(c) Benzyl penicillin is not able to penetrate ampicillin is
deeper into the lipoprotein-peptidoglycan (a) Nausea and vomiting
multiplayer cell wall of gram-negative
(b) Loose motions
bacteria
(c) Constipation
(d) Both (a) and (b)
(d) Urticaria
62. The penicillin G preparation with the
longest duration of action is 69. Amoxicillin is inferior to ampicillin for the
treatment of the following infection
(a) Benzathine penicillin
(a) Typhoid
(b) Sodium penicillin
(b) Shigella enteritis
(c) Potassium penicillin
(c) Subacute bacterial endocarditis
(d) Procaine penicillin
(d) Gonorrhoea
63. If a patient gives history of urticaria, itching
and swelling of lips following injection of 70. Piperacillin differs from carbenicillin in the
penicillin G, then following respect(s)

(a) He will develop similar reaction whenever (a) It is more active against pseudomonas
aeruginosa
penicillin is injected
(b) It is active against Klebsiella also
(b) He can be given ampicillin safely
(c) It is acid resistant
(c) He can be given oral phenoxymethyl
penicillin safely (d) Both (a) and (b)
150 MCQs IN PHARMACOLOGY

71. Clavulanic acid is combined with (c) They bind to 50S ribosomes and interfere
amoxicillin because with translocation of the growing peptide
chain in the bacteria
(a) It kills bacteria that are not killed by-moxicillin
(d) They interfere with DNA mediated RNA
(b) It reduces renal clearance of amoxicillin
synthesis in bacteria
(c) It counteracts the adverse effects of
amoxicillin 77. Select the most potent tetracycline
(d) It inhibits beta lactamases that destroy antibiotic
amoxicillin (a) Demeclocycline (b) Methacycline
72. Amoxicillin + Clavulanic acid is active (c) Minocycline (d) Doxycycline
against the following organism except 78. An 8-year-old child presented with
(a) Methicillin resistant Staph. aureus brownish discoloured and deformed
(b) Penicillinase producing Staph. aureus anterior teeth. History of having received
an antibiotic about 4 years earlier was
(c) Penicillinase producing n. gonorrhoeae
obtained. Which antibiotic could be
(d) β-lactamase producing E. coli responsible for the condition ?
73. A woman’s neglected hangnail has (a) Chloramphenicol (b) Tetracycline
developed into a mild staphylococcal (c) Erythromycin (d) Genetamicin
cellulites. Which of the following regimens
would be appropriate oral therapy? 79. The most suitable tetracycline for use in
a patient with impaired renal function is
(a) Dicloxacillin 125 mg q6h
(a) Chlortetracycline (b) Demeclocycline
(b) Vancomycin 250 mg q6h
(c) oxytetracycine (d) Doxycycline
(c) Methicillin 500 mg q6h
(d) Cefazolin 1 g q8h 80. Compared to older tetracyclines, doxycycline
produces a lower incidence of superinfection
(e) Penicillin V 500 mg q6h
diarrhoea because
74. Cefotaxime has the following properties (a) It is completely absorbed in the small
except intestines so that drug concentration in the
(a) It is highly active against aerobic gram colonic contents is low
negative bacteria (b) It is inactivated by the gut microflora
(b) It is the most active cephalosporin against (c) It is not active against the microbes of the
Pseudomonas aeruginosa normal gut flora
(c) It produces an active metabolite (d) It is a potent tetracycline and inhibits the
(d) It has achieved high cure rates in serious superinfection causing microbes as well
hospital acquired infections 81. Tetracyclines are still the first choice drugs
75. Select the fourth generation cephalospor- for the following disease.
in among the following (a) Granuloma linguinale
(a) Cefpirome (b) Ceftizoxime (b) Chancroid
(c) Ceftazidime (d) Cefuroxime (c) Syphilis
(d) Gonorrhoea in patients allergic to penicillin
76. The most important mechanism by which
tetracycline antibiotics exert antiimicrobial 82. Chloramphenicol is more active than
action is tetracyclines against
(a) They chelate Ca2+ ions and alter permeability (a) Bateroides fragilis
of bacterial cell membrane (b) Treponema pallidum
(b) They bind to 30S ribosomes and inhibit (c) Streptococci
bacterial protein synthesis (d) Staphylococci
ANTIBIOTICS 151

83. Which out of the following antibiotics 89. The most important mechanism of bacterial
penetrates blood-CSF barrier the best resistance to an aminoglycoside antibiotic
(a) Erythromycin (b) Gentamicin is
(c) Tetracycline (d) Chloramphenicol (a) Plasmid mediated acquisition of aminogly-
coside conjugating enzyme
84. The most important mechanism by which
gram negative bacilli acquire chloram- (b) Mutational acquisition of aminoglycoside
phenicol resistance is hydrolyzing enzyme
(a) Decreased permeability into the bacterial cell (c) Mutation reducing affinity of ribosomal
protein for the antibiotic
(b) Acquisition of a plasmid encoded for
chloramphenicol acetyl transferase (d) Mutational loss of porin channels
(c) Lowered affinity of the bacterial ribosome 90. Which toxic effect of aminoglycoside
for chloramphenicol antibiotics is most irreversible in nature ?
(d) Switching over from ribosomal to
(a) Vestibular damage
mitochondrial protein synthesis
(b) Hearing loss
85. What is the most important reason for
(c) Neuromuscular blockade
restricted clinical use of chloramphenicol ?
(d) Kidney damage
(a) Its narrow spectrum of activity
(b) Emergence of chloramphenicol resistance 91. Streptomycin sulfate is not absorbed
(c) Its potential to cause bone marrow depression orally because it is
(d) Its potential to cause superinfections (a) Degraded by gastrointestinal enzymes
(b) Destroyed by gastric acid
86. Aminoglycoside antibiotics have the
following common property (c) Highly ionized at a wide range of pH values
(a) They are primarily active against gram- (d) Insoluble in water
negative bacilli 92. The aminoglycoside antibiotic which is
(b) They are more active in acidic medium distinguished by its resistance to bacterial
(c) They readily enter cells and are distributed aminoglycoside inactivating enzymes is
in total body water
(a) Kanamycin (b) Sisomicin
(d) They are nearly completely metabolized in
(c) Amikacin (d) Tobramycin
liver
93. An aminoglycoside antibiotic should not
87. Which aminoglycoside antibiotic causes
be used concurrently with the following
more hearing loss than vestibular
disturabance as toxic effect ? drug

(a) Streptomycin (b) Gentamicin (a) Ampicillin (b) Vancomycin


(c) Kanamycin (d) Sisomicin (c) Ciprofloxacin (d) Rifampin

88. Which of the following drugs has 94. All of the following statements regarding
demonstrated in vitro activity against pentamidine isethionate are true except
Mycobacterium avium-intracellular (a) it is indicated for treatment or prophylaxis of
(MAI)? infection due to Pneumocystis carinii
(a) Vancomycin (b) it may be administered intramuscularly,
(b) Clarithromycin intravenously or by inhalation
(c) Erythromycin base (c) it has no clinically significant effect on serum
(d) Troleandomycin glucose
(e) Erythromycin estolate (d) it is effective in the treatment of leishmaniasis
152 MCQs IN PHARMACOLOGY

95. Prolonged oral therapy with the following (b) M.avium complex infection in AIDS patient
antibiotic can damage intestinal villi (c) M.tuberculosis infection in a patient who
resulting in statorrhoea and loose motions develops jaundice due to first line
(a) Ampicillin (b) Tetracycline antitubercular drugs
(c) Neomycin (d) Nystatin (d) Both (a) and (b)

96. Hepatitits with cholestatic jaundice occurs 103. A 23-year-old male with a history of
most frequently as an adverse reaction to influenza A infection. An outbreak of
the following preparation of erythromycin influenza A has just been reported in
his community and he is exhibiting
(a) Erythromycin base initial symptoms of influenza A. Which
(b) Erythromycin stearate agent would be the most useful to treat
(c) Erythromycin estolate him?
(d) Erythromycin ethylsuccinate (a) Cidofovir (b) Famciclovir
97. The following antibiotic is a first line drug (c) Rimantidine (d) Foscarnet
for treatment of Mycobacterium avium (e) Ribavirin
complex infection in AIDS patients 104. Addition of clavulanic acid to amoxicillin
(a) Clindamycin (b) Clarithromycin is to
(c) Roxithromycin (d) Erythromycin (a) Decrease the renal excretion of amoxicillin
98. Roxithromycin has the following (b) Enhance the anti-bacterial activity of
advantages over erythromycin except amoxicillin
(c) Decrease the biotransformation of amoxicillin
(a) It is more effective in whooping cough
(d) To increase oral absorption of amoxicillin
(b) It causes less gastric irritation
(c) It has longer plasma half-life 105. The aminoglycoside that can be used in
(d) It is unlikely to precipitate theophylline toxicity amoebiasis is
(a) Paromomycin (b) Framycetin
99. Highest incidence of antibiotic associated
(c) Amikacin (d) Netilmicin
pseudo membranous enterocolitis has
been noted with the use of 106. Tetracyclines are avoided in pregnancy
(a) Ampicillin (b) Chloramphenicol because they can
(c) Vancomycin (d) Clindamycin (a) Cause abortions
(b) Cause excessive postpartum hemorrhage
100. The drug of choice for treatment of
(c) Affect the bones and teeth of the fetus
methicillin resistant Staphylococcus
aureus infection is (d) Cause excessive vomiting in the mother

(a) Cloxacillin (b) Vancomycin 107. Ciprofloxacin inhibits the bacterial enzyme
(c) Erythromycin (d) Amikacin (a) Transpeptidase
101. ‘Red man syndrome’ has been associated (b) DNA gyrase
with rapid intravenous injection of the (c) DNA dependent RNA polymerase
following antibiotic (d) Dihydrofolate reductase
(a) Vancomycin (b) Clindamycin 108. Erythromycin is the drug of choice in
(c) Cefoperazone (d) Piperacillin (a) Pertussis
102. Clarithromycin is used for the following (b) Gonococcal urethritis
(a) Multidrug resistant M.tuberculosis infection (c) Prophylaxis of bacterial endocarditis
(d) Chlamydial infections
ANTIBIOTICS 153

109. Cyclosporine 117. Dr. Jones requests your help in prescrib-


(a) Is derived from a bacterium ing a protease inhibitor for his patient.
He has heard that not all agents are the
(b) Has a selective inhibitory effect on T-
lymphocytes same and asks for your recommendation
as to which agent would penetrate the
(c) Is not absorbed orally
blood-brain barrier. Which agent would
(d) Is excreted unchanged from the body you recomment?
110. Tetracycline is stored in the body in (a) Saquinavir (b) Ritonavir
(a) Protein bound form (c) Indinavir (d) Nelfinavir
(b) In hairs, nails and skin (e) Amprenavir
(c) In muscular tissues
118. Which of the following agents is not a
(d) In bones
broad spectrum antibiotic
111. Drug which interfere with the bacterial (a) Ampicillin
cell wall synthesis is
(b) Tetracycline
(a) Chloramphenicol (c) Chloremphenicol
(b) Tetracyclines (d) Gentamycin
(c) Colistin
(d) Penicillins and cephalosporins 119. Probenacid increases the plasma con-
centration of penicillin because
112. Antibiotic(s) which inhibit the protein
(a) It blocks the renal tubular excretion of
synthesis in cells is/are
penicillin
(a) Sulphonamides and PAS (b) It prevents the metabolism of penicillin
(b) Isonized and PAS (c) It displaces penicillin from protein binding
(c) Tetracyclines and chloramphenicol sites and thus increases free drug con-
(d) Penicillin and cephalosporins centration
113. Penicillins are not effective against (d) It acts by all above mechanisms
(a) Gram-positive cocci 120. Benzylpenicillin is not used orally because
(b) Gram-positive bacilli (a) It is destroyed by acid in stomach and
(c) Gram-negative bacilli absorption is also incomplete
(d) Gram-negative cocci (b) It is well absorbed from intestine but is
114. Pseudomonas aeruginosa organism is unpredictable
always sensitive to (c) It produces severe diarrhoea on oral
administration
(a) Streptomycin (b) Colistin
(d) All of the above
(c) Penicillin (d) Tetracyclines
115. Drug of choice for Brucella infection is 121. Penicillins should not be given by

(a) Tetracycline (a) Intramuscular route


(b) Gentamycin (b) Intravenous route
(c) Ampicillin (c) Intrathecal route
(d) Sulfonamides (d) Intra – articular route

116. Streptomycin is more active at 122. Anaphylactic shock to penicillin occurs


(a) pH 5.5 than pH 8.5 of urine (a) Almost at once after administration
(b) pH 8.5 than pH 5.5 of urine (b) Between 6 to 12 hrs of administration
(c) Equally active at all pH of urine (c) Between 4 to 6 hrs of administration
(d) All of the above (d) After 12 hrs of administration
154 MCQs IN PHARMACOLOGY

123. Drugs usually active against penicillinase 130. Penicillin was first used clinically for
producing Staphylococcus aureus include systemic infections in the year
which of the following?
(a) 1926 (b) 1935
I. Timentin (ticarcillin-clavulanate) (c) 1941 (d) 1957
II. Augmentin (amoxicillin-clavulanate)
III. Oxacillin 131. Antiviral agents that are active against
cytomegalovirus (CMV) include which of
(a) Only I is correct
the following?
(b) Only III is correct
I. Ganciclovir
(c) I and II are correct
(d) II and III are correct II. Foscarnet
(e) I, II and III are correct III. Acyclovir
(a) Only I is correct
124. Cephalosporins
(b) Only III is correct
(a) Are more vulnerable to beta–lactamase (c) I and II are correct
enzyme
(d) II and III are correct
(b) Are less vulnerable to beta–lactamase
enzyme (e) I, II and III are correct
(c) Do not possess beta lactam ring 132. Which of the following is a steroidal
(d) Have multiple beta lactam rings antibiotic
125. Which of the following antibiotics possess (a) Nalidixic acid (b) Fusidic acid
neuromuscular blocking action (c) Spectinomycin (d) Nitrofurantoin
(a) Ampicillin (b) Streptomycin 133. The antibiotic that enters brain freely is
(c) Chloramphenicol (d) Sulphonamides
(a) Tetracycline (b) Erythromycin
126. Oral neomycin and streptomycin are used (c) Chloramphenicol (d) Gentamicin
for sterlization of bowel and for dysen-
tery because 134. The tetracycline safe in renal failure is
(a) They are least absorbed from oral route (a) Digoxin (b) Furosemide
(b) The peak concentration in blood is achieved (c) Enalapril (d) Amrinone
quickly
135. The semisynthetic penicillin which is
(c) They are least toxic to G.I.T destroyed by acid is
(d) None of the above
(a) Phenoxymethyl penicillin
127. Chloramphenicol is the drug of choice in (b) Ampicillin
(a) Staphylococcal infection (c) Carbenicillin
(b) Salmonella infection (d) Coxacillin
(c) Viral infection
136. Why benzyl penicillin is ineffective in
(d) Amoebic dysentry gram negative infections ?
128. Drug of choice for ringworm infection is (a) Is destroyed by penicillinase of gram negative
(a) Griseofulvin (b) Amphotericin B organisms
(c) Nystatin (d) Neomycin (b) Gram negative organisms donot utilize D-
alanine whose incorporation is inhibited by
129. In renal failure safest tetracycline is benzyl penicillin
(a) Oxytetracycline (c) The lipoprotein – peptidoglycan cell wall of
(b) Chlortetracycline gram negative organisms prevents entry of
(c) Doxycycline penicillin
(d) Demethyl chlortetracycline (d) All of the above
ANTIBIOTICS 155

ANSWERS
1. a 2. b 3. e 4. e 5. d 6. c
7. d 8. c 9. c 10. a 11. b 12. c
13. d 14. d 15. c 16. a 17. c 18. d
19. d 20. b 21. d 22. e 23. b 24. a
25. d 26. b 27. d 28. e 29. c 30. d
31. c 32. c 33. c 34. c 35. d 36. b
37. c 38. c 39. d 40. c 41. b 42. c
43. d 44. d 45. c 46. a 47. b 48. b
49. b 50. a 51. b 52. b 53. a 54. c
55. b 56. b 57. c 58. d 59. c 60. b
61. c 62. a 63. d 64. b 65. c 66. a
67. a 68. b 69. b 70. d 71. d 72. a
73. a 74. b 75. a 76. b 77. c 78. b
79. d 80. a 81. a 82. a 83. d 84. b
85. c 86. a 87. c 88. b 89. a 90. b
91. c 92. c 93. b 94. c 95. c 96. c
97. b 98. a 99. d 100. b 101. a 102. b
103. c 104. b 105. a 106. c 107. b 108. a
109. b 110. d 111. d 112. c 113. c 114. c
115. d 116. b 117. d 118. d 119. a 120. a
121. b 122. a 123. e 124. b 125. b 126. a
127. b 128. a 129. c 130. c 131. c 132. b
133. c 134. b 135. c 136. c

EXPLANATIONS FOR THE ANSWERS


1. a Isoniazid increases the excretion or pyridoxine, seizures and in patients with impaired drug
which can lead to peripheral neuritis, elimination.
particularly in poorly nourished patients. 37. c Although active against various gram-negative
Pyridoxine (a form of vitamin B6) organisms, spectinomycin is approved only for
deficiency may cause convulsions as well as the treatment of gonorrhea and is particularly
the neuritis, involving synovial tenderness and recommended for treatment of uncomplicated
swelling. Treatment with the vitamin can reverse forms of the disease.
the neuritis and prevent or cure the seizures. 43. d The fever curve is very useful for monitoring a
13. d Trough serum levels below 2 mg/ml are patient’s response to antimicrobial therapy.
considered appropriate for gentamicin and are Antipyretics can be used to reduce high fever in
recommended to minimize the risk of toxicity from patients at risk for complications (e. g., seizures)
this aminoglycoside. Because aminoglycosides or in some cases to make the patient more
accumulate in the proximal tubule of the kidney, comfortable.
nephrotoxicity can occur. 55. b Amoxicillin, ampicillin and cyclacillin are all
24. a Seizures have been attributed to the use of penicillins and should be avoided in patients
penicillin G, imipenem, amphotericin B and with histories of hypersensitivity to other
metronidazole. Seizures are especially likely penicillin compounds. Although the risk of cross-
with high doses in patients with a history of reactivity with cephalosporins (e. g., cefaclor)
156 MCQs IN PHARMACOLOGY

is now considered very low, most clinicians 94. c Pentamidine isethionate is indicated for both
avoid the use of these agents in patients with treatment and prophylaxis of infection due to
histories of type I hypersensitivity reactions (e.g., Pneumocystis carinii. It can be administered
anaphylaxis, bronchospasm, giant hives) intramuscularly, intravenously or by inhalation.
60. b Norfloxacin, ciprofloxacin, carbenicillin and Inhalation may produce bronchospasm. Blood
methenamine mandelate achieve urine glucose should be carefully monitored because
concentrations high enough to treat urinary tract pentamidine may produce either hyperglycemia
infections due to Pseudomonas aeruginosa. or hypoglycemia.
Trimethoprim-sulfamethoxazole is not useful for 103. c Cidofovir, famciclovir and foscarnet have little
treating infections due to this organism, although or no in vivo activity agains influenza A.
the combination is useful for treating certain Ribavirin has some activity but is a second line
other urinary tract infections. agent for influenza A and is mainly indicated
73. a Although vancomycin, methicillin and cefazolin for treatment of RSV. Rimantidine is a derivative
have excellent activity against staphylococci, of amantidine with excellent activity against
they are not effective orally for systemic influenza A. it is indicated for the prophylaxis
infections. Vancomycin is prescribed orally for and treatment of influenza A viral infections.
infections limited to the gastrointestinal tract, but 117. d All are protease inhibitors, but only nelfinavir
because it is poorly absorbed orally, it is not has been shown to cross the blood-brain barrier,
effective for systemic infections. Most hospital a known reservoid for HIV viruses.
and community acquired staphylococci are 123. e Timentin and augmentin each include a β-
currently resistant to penicillinV. Thus, of the lactamase inhibitor, combined with ticarcillin
drugs listed in the question, the most appropriate and amoxicillin respectively. These combinations
drug for oral therapy of staphylococcai cellulites offer activity against Staphylococcus aureus
is dicloxacillin. similar to that of the penicillinase-resistant
88. b Clarithromycin, an alternative to erythromycin, penicillins, such as oxacillin.
has demonstrated in vitro activity against 131. c Only ganciclovir and foscarnet are active
Mycobacterium avium-intracellulare (MAI). against cytomegalovirus (CMV) infections. These
Clarithromycin is also used against Toxoplasma agents are virustatic and arrest DNA synthesis
gondii and Cryptosporidium species, and it is by inhibiting viral DNA polymerase. Although
more active than erythromycin against ganciclovir in the treatment of CMV infections
staphylococci and streptococci. Vancomycin is in the CNs has not been successful. Foscarnet
used to treat staphylococci and streptococci, but is a broad-spectrum antiviral agent and is used
has no demonstrated activity versus MAI. in patients with ganciclovir resistance. Acyclovir
Troleandomycin is similar to erythromycin but is not clinically useful for the treatment of CMV
is generally less active against these organisms. infections because CMV is relatively resistant to
acyclovir in vitro.
DRUGS USED IN GASTRO INTESTINAL TRACT DISORDERS 157

CHAPTER 9
DRUGS USED IN GASTROINTESTINAL
TRACT DISORDERS

1. Which of the following is most effective (e) Vasoactive intestinal peptide


in the treatment of peptic ulcer disease?
6. Which of the following is released in
(a) Bromocriptine (b) Cimetidine traumatized tissue, causes pain and
(c) Ergotamine (d) Ketanserin edema, and is inactivated by angiotensin
(e) LSD converting enzyme?

2. Which laxative should not be used to (a) Angiotensin I


treat acute constipation because of its (b) Angiotensin II
slow onset of action? (c) Atrial natriuretic peptide
(a) Glycerin (d) Bradykinin
(b) Bisacodyl suppository (e) Calcitonin gene-related peptide
(c) Psyllium 7. Which of the following is a decapeptide
(d) Milk of magnesia precursor of a vasoconstictor substance?
3. Which of the following–if given (a) Angiotensin I
intravenously–will cause increased (b) Angiotensin II
gastrointestinal motility and diarrhea? (c) Atrial natriuretic peptide
(a) Angiotensin II (b) Bethanechol (d) Bradykinin
(c) Bradykinin (d) Renin (e) Calcitonin gene-related peptide
(e) All of the above
8. Which is not a risk factor for hyperphos-
4. A peptide that causes increased capillary phatemia and death from sodium phos-
permeability and edema is phate enemas when used in children?
(a) Angiotensin II (b) Bradykinin (a) Renal insufficiency
(c) Captopril (d) Histamine (b) Hirschsprung’s disease
(e) Losartan (c) Anorectal malformations
(d) Children 6 to 12 years of age
5. A vasodilator that can be inactivated by
proteolytic enzymes is 9. Which of the following is the most potent
(a) Angiotensin I (b) Isoproterenol vasodilator discovered to date and is
found in high concentration in the
(c) Histamine (d) Neuropeptide Y
thyroid?
158 MCQs IN PHARMACOLOGY

(a) Angiotensin I mesis), and epigastric pain. Metabolic


(b) Angiotensin II acidosis and leukocytosis were also
present. This patient is most likely to have
(c) Atrial natriuretic peptide
ingested tablets containing
(d) Bradykinin
(a) Acetaminophen
(e) Calcitonin gene-related peptide
(b) Aspirin
10. Which of the following statements (c) Diphenhydramine
adequately describes bulk-forming
(d) Iron
laxaties?
(e) Vitamin C
(a) Can cause diarrhea if not taken with water
15. A patient is broght to the emergency room
(b) Are derived from polysaccharides and
suffering from nausea, vomiting, and
rersemble fiber (bran) in mechanism of
abdominal pain. He has muscle
action
weakness, which seems to be progressing
(c) Onset of action is in 4–8 hours downward from the head and neck. The
(d) Produce much more complete evacuation of patient has difficulty talking clearly and
constipation than stimulant products has ptosis and ophthalmoplegia. The most
likely cause of these symptoms is
11. Cardiac arrhythmias have occurred when
this drug was used by patients taking the (a) Accidental ingestion of paraquat
gastrointestinal promotility agent cisapride (b) An overdose of phenobarbital
(a) Amphotericin B (b) Clotrimazole (c) Excessive consumption of ethanol
(c) Griseofulvin (d) Ketoconazole (d) Food poisoning
(e) Voriconazole (e) Organophosphate poisoning

12. Metronidazole is least likely to be effective 16. A 55-year-old woman with insulin-
in the treatment of dependent diabetes of 40 years’ duration
complains of severe bloating and abdominal
(a) Amebiasis (b) Giardiasis distress, especially after meals. Evaluation
(c) Pneumocystosis (d) Psudomembranous is consistent with diabetic gastroparesis. The
colitis drug you would be most likely to
(e) Trichomoniasis recommend is

13. Which of the following statements about (a) Docusate (b) Dopamine
non-drug therapies for acute diarrhea is (c) Loperamide (d) Metoclopramide
not correct? (e) Sucralfate
(a) Breast feeding should be continued as normal 17. A patient who must take verapamil for
(b) Even if the patient is not vomiting, food should hypertension and angina has become
be withheld for 6–12 hours severely constipated. Which of the
(c) Fluids can be given to patients who following drugs would be most suitable
experience vomiting, but small amounts of as a cathartic?
fluid should be used. (a) Aluminum hydroxide
(d) Replacement fluids mainly consist of water, (b) Diphenoxylate
sugar, potassium, sodium and bicarbonates (c) Magnesium hydroxide
14. A 2-year-old child was brought to the (d) Metoclopramide
emergency room 1 hour after ingestion of (e) Mineral oil
tablets he had managed to obtain from a
18. Your cousin is planning a three-week trip
bottle on top of the refrigerator. His
overseas and asks your advice regarding
symptoms included marked gastro-
medications for traveler’s diarrhea. A drug
intestinal distress, vomiting (with hemate-
suitable for noninfectious diarrhea is
DRUGS USED IN GASTRO INTESTINAL TRACT DISORDERS 159

(a) Aluminum hydroxide (a) Useful for treatment of severe diarrhea


(b) Diphenoxylate (b) Very unsafe because not absorbed systemically
(c) Magnesium hydroxide (c) In general, small doses are needed to relieve
(d) Metoclopramide diarrhea
(e) Mineral oil (d) Kaolin is now generally recognized as a safe
and effective OTC antidiarrheal agent.
19. Which of the following products should
not be used to replenish lost fluids from 24. Which one of the following drugs has no
acute diarrhea? effect on prothrombin but increases the
(a) Pedialyte solution likelihood of bleeding in patients who are
also taking warfarin?
(b) Kool-Aid
(c) Gatorade (half-strength diluted with water) (a) Carbamazepine (b) Cholestyramine
(d) The world Health Organization (WHO) (c) Naproxen (d) Rifampin
solution (e) Vitamin K

20. A drug associated with the long QT 25. Which one of the following drugs has
syndrome and cardiac arrhythmias is resulted in severe hematotoxicity when
(a) Aluminum hydroxide administered to a patient being treated
with azathioprine?
(b) Cisapride
(c) Granisetron (a) Allopurinol (b) Cholestyramine
(d) Loperamide (c) Digoxin (d) Lithium
(e) Metromidazole (e) Theophyline

21. On your way to an examination you 26. Which one of the following antibodies has
experience that vulnerable feeling that the longest half-life?
an attack of diarrhea is imminent. If you (a) Black widow spider antivenin
stopped at a drugstore, you could buy (b) Botulinum antitoxin
this antidiarrheal drug without a
(c) Diphtheria antitoxin
prescription even though it is related
chemically to the strong opioid – analgesic (d) Hepatitits B immune globulin
meperidine (e) Snake bite antivenin
(a) Aluminum hydroxide 27. Hepatitis B vaccine is least likely to be
(b) Diphenoxylate recommended for prophylactic use in
(c) Loperamide (a) Dialysis patients
(d) Magnesium hydroxide (b) Intravenous drug abusers
(e) Metoclopramide (c) Newborns
22. This antibiotic is not appropriate for use (d) Raw oyster eaters
as an oral agent in the treatment of (e) Surgeons
recurrent peptic ulcer associated with
Helicobacter pylori 28. Which of the following statements
concerning traveler’s diarrhea (TD) is
(a) Amoxicillin (b) Clarithromycin
true?
(c) Metronidazole (d) Tetracycline
(a) TD can usually be avoided by not eating raw
(e) Vancomycin
vegetables, seafood or eggs when traveling
23. Which of the following statements about to third-world countries
adsorbant drugs used for diarrhea is (b) TD can be prevented by taking one dose of
true? antibiotic 1 day before a trip
160 MCQs IN PHARMACOLOGY

(c) A specific of Helicobacter pylori is the of bloating and sour bleaching after
primary pathogen responsible for TD meals. On several occasions, vomiting
(d) Phillip’s milk of magnesia is used to prevent/ has occurred after a meal. Evaluation
treat TD reveals delayed emptying of the stomach,
and you diagnose diabetic gastro paresis.
29. Dietary supplementation with DHEA is Which of the following drugs would be
best documented to have therapeutic most useful in this patient?
value in the treatment of
(a) Famotidine (b) Metoclopramide
(a) Acne (c) Misoprostol (d) Omeprazole
(b) Diabetes insipidus (e) Ondansetron
(c) Hirsutism in female patient
34. Which one of the following agents is least
(d) Postmenopausal osteoporosis
likely to protect the upper gastrointestinal
(e) Systemic lupus erythematosus tract from ulcer formation?
30. Which one of the following compounds has (a) Antacids (b) Celecoxib
been shown to have value in managing (c) Cimetidine (d) Misoprostol
symptoms of jet lag? (e) Sucralfate
(a) DHEA (b) Garlic
35. This agent is the drug of choice in severe
(c) Ginseng (d) Melatonin amebic disease and for hepatic abscess.
(e) Sassfras It is activated to toxic intermediates by
the pyruvate – ferredoxin oxidoreductase
31. A patient with Zollinger–Ellison syndrome
enzyme system present in the parasite.
has been receiving high doses of
cimetidine for 7 weeks. A frequent (a) Diloxanide furoate
adverse effect of cimetidine is (b) Emetine
(a) Agranulocytosis (c) Iodoquinol
(b) Systemic lupus erythematosus (d) Metronidazole
(c) Inhibition of hepatic metabolism of other (e) Paromomycin
drugs
36. All of the following agents are considered
(d) Antiestrogenic effects close to ideal laxatives except
(e) Hypertension (a) emollient laxatives
32. Most weak acid drugs as well as weak (b) bulk-forming laxatives
base drugs are absorbed primarily from (c) fiber
the small intestine after oral administration (d) stimulant laxatives
because
37. Which solution is used as an astringent?
(a) Both types are more ionized in the small
intestine (a) Strong iodine solution USP
(b) Bothe types are less ionized in the small (b) Aluminum acetate topical solution USP
intestine (c) Acetic acid NE
(c) The blood flow is greater in the small intestine (d) Aromatic ammonia spirit USP
than that of other parts of the gut (e) Benzalkonium chloride
(d) The surface area of the small intestine is
38. Mechanistically, which of the following
greater than other parts of the gut
drugs will decrease stomach acid secretion
(e) The small intestine has nonspecific carriers by blockade of H2 histaminic receptors?
for most drugs
(a) Pyrilamine (b) Hydroxyzine
33. A patient with a 30-year history of type (c) Cisapride (d) Omeprazole
1 diabetes comes to you with a complaint (e) Ranitidine
DRUGS USED IN GASTRO INTESTINAL TRACT DISORDERS 161

39. Lansoprazole would be effective in the 44. Which of the following statements
treatment of concerning traveler’s diarrhea (TD) is
(a) Gastroesophageal reflux disease true?
(b) Peptic ulcer disease (a) TD can usually be avoided by not eating raw
(c) Zollinger-Ellison syndrome vegetables, seafood, or eggs when traveling
to third-world countries.
(d) All of the above
(b) TD can be prevented by taking one dose of
(e) None of the above
antibiotic one day before a trip
40. All of the following statements about (c) A specie of Helicobacter pylori is the primary
stool softners are true except pathogen responsible for TD
(a) There is minimal systemic absorption (d) Phillip’s Milk of Magnesia is used to prevent
(b) the onset of action is usually 1–2 days / treat TD
(c) they are useful in patients with constipation 45. All of the following statements about
who have experienced an acute myocardial stool softeners are true except
infractioon
(a) There is minimal systemic absorption
(d) they can be taken with little or no water
(b) The onset of action is usually 1–2 days
41. All of the following statements adequately (c) They are useful in patients with constipation
describe bulk-forming laxatives except who have experienced an acute myocardial
(a) they produce a much more complete infarction
evacuation of constipation than stimulant (d) They can be taken with little or no water
products
(b) they can cause constipation if not taken with 46. Which of the following is an appropriate
water non-pharmacologic recommendation for
patients with gastroesophageal reflux
(c) they are derived from polysaccharides and
disease (GERD)?
resemble fiber (bran)in the mechanism of
action (a) Eat larger but fewer meals
(d) the onset of action is 24–72 hours (b) Avoid meals high in protein
(c) Eat evening meals at least 3 hours before
42. Which of the following statements
bed
adequately describes bulk-forming
laxatives? (d) Prop a patient’s head up with 2 pillows at
night
(a) Can cause diarrhea if not taken with water
(b) Are derived from polysaccharides and 47. Which of the following organisms has
resemble fiber (bran)in mechanism of action been implicated as a possible cause of
(c) Onset of action is in 4–8 hours chronic gastritis and peptic ulcer disease?
(d) Produce much more complete evacuation of (a) Campylobacter jejuni
constipation than stimulant products (b) Escherichia coli
43. Which of the following statements about (c) Helicobacter pylori
adsorbent drugs used for diarrhea is (d) Calymmatobacterium granulomatis
true? (e) Giardia lamblia
(a) Useful for treatment of severe diarrhea 48. A patient suffering from acute infectious
(b) Very safe because not absorbed systemically diarrhea caused by Shigella can be
(c) In general, small doses are needed to relieve managed in all of the following ways
diarrhea except
(d) Kaolin and pectin are considered to be very (a) no treatement because signs and symptoms
effective (category I)adsorbents usually resolve in 48 hours
162 MCQs IN PHARMACOLOGY

(b) use of glucose solutions (e.g., soda, apple (4) May cause dry mouth
juice)to settle the stomach and decrease the (5) Can be alternated with an antacid
number of stools mixture to control diarrhea
(c) avoiding food for at least 6 hours, then slowly
52. Aluminum hydroxide is used to treat
increasing fluid intake.
hyperphosphatemia associated with
(d) using antibiotics (e.g. Bactrim, doxycycline)
renal failure. Chronic use of aluminum
for 7 days
hydroxide may cause all of the following
49. As part of a comprehensive management conditions except
strategy to treat peptic ulcer disease, (a) Phosphate depletion
patients should be encouraged to do all (b) Calcium resorption and bone demineralization
of the following except
(c) Anorexia and constipation
(a) Decrease caffeine ingestion (d) Fluid retention
(b) Eat only bland foods
53. Which local anesthetic should be used to
(c) Stop smoking
treat symptoms of pain, itching, burning
(d) Avoid alcohol
and discomfort in patients with an
(e) Avoid the use of milk as a treatment modality established lidocaine allergy?
50. A gastric ulcer patient requires close (a) Tetracaine (b) Dibucaine
follow-up to document complete ulcer (c) Pramoxine (d) Benzocaine
healing because
54. Which of the following agents in most
(a) Perforation into the intestine is common
useful for carcinoma of the liver?
(b) Spontaneous healing of the ulcer may occur
(a) Vinblastine (b) Floxuridine (FUDR)
in 30% – 50% of cases
(c) Vincristine (d) Cytarabine (Cytosar)
(c) There is the risk of the ulcer being cancerous
(e) Mercaptopurine
(d) Symptoms tend to be chronic and recur
(e) Weight loss may be severe in gastric ulcer 55. All of the following vasoconstrictors are
patients deemed safe and effective for the temporary
relief of itching and swelling except
51. For each effect, select the agent that is
most likely associated with it (a) ephedrine 0.1% – 1.25%
(b) epinephrine 0.005% – 0.01%
(a) Sodium bicarbonate
(c) phenylpropanolamine 1% – 10%
(b) Aluminum hydroxide
(d) phenylephrine 0.25%
(c) Calcium carbonate
(d) Magnesium hydroxide 56. Bisacodyl frequently can cause
(e) Propantheline (a) Abdominal cramps
(1) May cause diarrhea (b) Constipation
(2) Cannot be used by patients with heart (c) Skin rashes
failure (d) Dizziness
(3) Use with milk and an alkaline substance (e) Nauseas
can cause milk-alkali syndrome
DRUGS USED IN GASTRO INTESTINAL TRACT DISORDERS 163

ANSWERS
1. c 2. c 3. b 4. b 5. e 6. d
7. a 8. d 9. e 10. b 11. d 12. c
13. b 14. d 15. d 16. d 17. c 18. b
19. b 20. b 21. c 22. e 23. d 24. c
25. a 26. d 27. d 28. a 29. e 30. d
31. c 32. d 33. b 34. b 35. d 36. d
37. b 38. e 39. d 40. d 41. a 42. b
43. b 44. a 45. d 46. c 47. c 48. b
49. b 50. c 51. 1.d, 2.a, 3.c, 4.e, 5.b 52. d 53. c
54. b 55. c 56. a

EXPLANATIONS FOR THE ANSWERS


1. c Glycerin and the bisacodyl suppository all Earlier Parents were told that children should
produce stools in one-half hour to a few hours, not receive food, milk-products or breast milk
whereas psyllium, a bulk-forming laxative, for 6–68 hours after the onset of diarrhea, but
produces stool in 24–72 hours in the same recent information shows that children should
manner as a normal bolus of food or fiber. remain on their normal dier or breast feeding
8. d The populr sodium phosphate enemas (e.g., during episodes of diarrhea because these do
fleet)are very effective but have resulted in not make the diarrhea worse and may actually
hyperphosphatemia, hypocalcemia (tetany), improve the diarrhea.
hypokaemia, metabolic acidosis and cardia 19. b Replacement fluids for diarrhea should contain
death usually due to conduction abnormalities the appropriate amount of electrolytes (K+, Na+,
in very small children. This has mainly occurred Cl -, citrate)and glucose per specified amount
in children younger than 2 years of age or of water as found in commercially available oral-
between 2 and 5 years of age with predisposing rehydration. The World Health Organization
factors. These factors include chronic renal (WHO)solution can provide necessar y
disease, anorectal disease, anorectal ingredients. In addition, one-half-strength
malformations, and/or Hirschsrprung’s disease, Gatorade will provide necessary electrolytes
which allow phosphate blood concentrations to and glucose. Kool-Aid does not contain
become abnormally potassium. Carbonated beverages are low in
potassium, and some are too high in glucose.
10. b Stimulant products result in a quicker, more
23. d Adsorbents are not effective for severe diarrhea
complete and often more violent evacuation of
because they simply cannot adsorb enough
the bowel than do the bulk-forming agents. Bulk
water and do not reverse the cause of the
forming agents are developed from complex
diarrhea. Large doses may decrease symptoms.
sugars, similar to fiber, that provide bulk to
Of all the adsorbents, kaolin is the most effective
increase gastrointestinal motility and water
and is now recognized by the FDA as safe and
absorption into the bowel. However, patients
effective. All adsorbents are safe because they
must drink plenty of water to facilitate the
are not adsorbed systemically.
absorption of water into the bowel, or they may
28. a Traveler’s diarrhea (TD)primarily is caused by
become more constipated.
bacteria. Prophylaxis and treatment regimens
13. b Treating acute diarrhea is the replacement of include oral antibiotics and bismuth subsalicylate.
lost fluids. If severe vomiting persists, then Helicobacter pylori is the organisms shown to
patients may need intra-venous rehydration. contribute to refractory pepetic ulcer disease.
164 MCQs IN PHARMACOLOGY

36. d The ideal laxative is natural and produces stool motility and increase water absorption into the
on a regular basis. The product produces stool bowel. However, patients must drink plenty of
quickly without adverse effects such as water to facilitate the absorption of water into
abdominal cramping or the formation of a hard the bowel, or they may become more constipated.
stool, which may be difficult to pass. Products 48. b Giving highly osmotic solutions of glucose can
such as fiber or bulk-forming agents produce a
result in more water absorbed into the intestinal
stool similar to a bolus of food, without adverse
tract and, thus further diarrhea. Many cases of
effects. Emollient laxatives produce soft stools
diarrhea resolve within 48 hours without treatment.
without difficult defecation. Stimulants produce
People with diarrhea can avoid food for atleast 6
a stool quickly, but patients often experience
hours, then increase their fluid intake slowly. Severe
severe abdominal cramping and hard stools.
cases of infectious diarrhea can be treated with
40. d Stool softeners are safe and do not produce
antibiotic or antiprotozoals, depending on the
any adverse systemic effects. Because stool
organism that caused the episode.
softners work as surfactants, they allow
absorption of water into the stool, which makes 53. c Due to its chemically distinct structure, promoxine
stool softer and easier to pass. These products exhibits less cross-sensitivity when compared to
are useful in patients who should avoid. the other anesthetics and should be used in
41. a Stimulant products result in a quicker, more patients with a lidocaine allergy.
complete and often more violent evacuation of 55. c Vasoconstrictors deemed safe and effective by
the bowel than do the bulk-forming agents are the FDA are ephedrine HCl 0.1% – 1.25%,
developed from complex sugars, similar to fiber, epinephrine HCl 0.005% – 0.01% and
that provide bulk to increase gastrointestinal phenylephrine HCl 0.25%.
OXYTOCICS & UTERINE MUSCLE RELAXANTS 165

CHAPTER 10
OXYTOCICS & UTERINE MUSCLE
RELAXANTS

1. A 29-year-old woman who was in her chemical analysis of the cancer reveals
41st week of gestation had been in labor the presence of estrogen and progester-
for 12 hours. Although her uterine one receptors. After this procedure, she
contractions had been strong and regular will probably receive
initially, they had diminished in force (a) Danazol (b) Flutamide
during the past hour. Which of the (c) Leuprolide (d) Mifepristone
following drugs would be administered (e) Tamoxifen
to facilitate this woman’s labor and
delivery? 5. A young woman complains of severe
abdominal pain at the time of menstruation.
(a) Dopamine (b) Leuprolide
Careful evaluation indicates the presence
(c) Oxytocin (d) Prolactin of significant endometrial deposits on the
(e) Vasopressin pelvic peritoneum. The most appropriate
therapy for this patient would be
2. Which one of the following agents is not
used in oral or implantable contraceptives? (a) Fultamide, orally
(b) Medroxyprogesterone acetate by intramuscular
(a) Clomiphene
injection
(b) Ethinyl estradiol (c) Norgestreal as an implant
(c) Mestranol (d) Oxandrolone by intramuscular injection
(d) Norethindrone (e) Raloxifene orally
(e) Norgestrel
6. Diethylstilbestrol should never be used in
3. All of the following are recognized effects pregnant women because it is associated
of combined oral contraceptives except with
(a) Breakthrough bleeding (a) Development of deep vein thrombosis in the
(b) Decreased risk of endometrial cancer pregnant woman
(c) Increased risk of ischemic stroke (b) Feminization of the external genitalia of male
offspring
(d) Increased risk of ovarian cancer
(c) Infertility and development of vaginal cancer
(e) Nausea in female offspring
4. A 50-year-old woman with a positive (d) Miscarriage
mammogram undergoes lumpectomy (e) Virilization of the external genitalia of female
and a small carcinoma is removed. Bio- offspring
166 MCQs IN PHARMACOLOGY

7. Finasteride has efficacy in the prevention (a) Desmopressin (b) Octreotide


of male-pattern baldness by virtue of its (c) Oxytocin (d) Prolactin
ability to (e) Triamcinolone
(a) Competitively antagonize androgen receptors
10. Raloxifene is a selective estrogen receptor
(b) Decrease the release of gonadotropins
modulator (SERM). Its characteristic
(c) Increase the serum concentration of SHBG properties make the drug most suitable
(d) Inhibit the synthesis of testosferone for treatment of a female patient who
(e) Reduce the production of dihydrotestosterone (a) Decides to start using an oral contraceptive
8. A 52-year-old postmenopausal patient (b) Has postmenopausal osteoporosis and is at
has evidence of low bone mineral risk for breast cancer
denisity. She and her physician are con- (c) Needs postcoital contraception
sidering therapy with raloxifene or a (d) Suffers from hirsutism
combination of conjugated estrogens
(e) Wants a therapeutic abortion
and medroxyprogesterone acetate.
Which of the following patient charac- 11. Select the drug which can improve
teristics is most likely to lead them to urinary flow rate in benign prostatic
select raloxifene? hypertrophy without affecting prostate
(a) Previous hysterectomy size
(b) Recurrent vaginitis (a) Amphetamine (b) Prazosin
(c) Rheumatoid arthritis (c) Finasteride (d) Goserelin
(d) Strong family history of breast cancer
12. The drug used for cervical priming to
(e) Troublesome hot flushes facilitate labour is
9. Following delivery of a healthy baby, a (a) Oxytocin
young woman begins to bleed extensively (b) Stilboestrol
because her uterus has failed to contract.
(c) Progesterone
Which one of the following drugs should
(d) Prostaglanding E2
be administered to this woman?

ANSWERS
1. c 2. a 3. d 4. e 5. b 6. c
7. e 8. d 9. c 10. b 11. b 12. d
OXYTOCICS & UTERINE MUSCLE RELAXANTS 167

CHAPTER 11
CHEMOTHERAPY

1. Twenty months after finishing her chemo- 4. A 31-year-old man has gonorrhea. He has
therapy, the woman had a relapse of breast no drug allergies, but he recalls that a few
cancer. The cancer was now unresponsive years ago while in Africa he had acute
to standard doses of chemotherapy. The hemolysis following use of an antimalarial
decision was made to treat the patient with drug. The physician is concerned that the
high-dose chemotherapy followed by au- patient has an accompanying urethritis
tologous stem cell transplantation. Which due to C trachomatis, though no cultures
or enzyme tests have been performed.
of the following drugs is most likely to be
Which of the following drugs is most likely
used to mobilize the peripheral blood stem
to be effective against gonococci and to
cells for the patient’s autologous stem cell
eradicate C trachomatis in this patient?
transplantation?
(a) Cefixime (b) Ciprofloxacin
(a) Erythropoietin (b) G – CSF
(c) Ofloxacin (d) Spectinomycin
(c) Interleukin - 11 (d) Intrinsic factor
(e) Sulfamethoxazole
(e) Thrombopoietin
5. The mechanism by which sulfasalazine
2. The combination of trimethoprim and exerts its primary action in ulcerative
sulfamethoxazole is effective against colitis is inhibition of
which one of the following opportunistic
(a) Folic acid synthesis
infections in the AIDS patient?
(b) The formation of leukotrienes and prostaglandins
(a) Disseminated herpes simplex (c) Phospholipase C
(b) Cryptococcal meningitis (d) Proton pump activity
(c) Toxoplasmosis (e) The formation of interleukind
(d) Oral cnadidiasis
6. Which one of the following adverse effects
(e) Tuberculosis
is most likely to occur with sulfonamides?
3. The top four most commonly diagnosed (a) Neurologic effects, including headache,
cancers include all of the following except dizziness, and lethargy
(a) Lung (b) Prostate (b) Hematuria
(c) Colon and rectum (d) Thyroid (c) Fanconi’s aminoaciduria syndrome
(e) Breast (d) Kernicterus in the newborn
(e) Skin reaction
168 MCQs IN PHARMACOLOGY

7. This drug is the preferred agent for teat- 12. A patient with AIDS and a CD4 cell count
ment of nocardiosis and, in combination of 100/µµL has persistent fever and weight
with pyrimethamine, is prophylactic loss associated with invasive pulmonary
against preumocystis carinii infections in disease that is due to M avium complex.
AIDS patiens. Optimal management of this patient is
(a) Ampicillin (b) Clindamycin to
(c) Norfloxacin (d) Sulfadiazine (a) Treat with rifabutin, since it prevents the
(e) Trimethoprim development of MAC bacteremia
(b) Select an antibiotic regimen based on drug
8. Supplementary folinic acid may prevent susceptibility of the cultured organism
anemia in folate-deficient persons who
(c) Start treatment with INH and pyrazinamide
use this drug; it is a weak base, and
achieves tissue levels similar to those in (d) Treat the patient with clarithromycin,
plasma ethambutol, and rifabutin
(a) Ciprofloxacin (b) Norfloxacin (e) Treat with trimethoprim – sulfamethoxazole
(c) Sulfacetamide (d) Trimethoprim 13. This drug has been used prophylactically
(e) Trovafloxacin in contacts of children with infection due to
Haemophilus influenzae type B. It is also
9. It is now recommended that trovafloxacin prophylactic in meningococcal and
be reserved for treatment of life-threatening staphylococcal carrier states. While the
infections because drug eliminates a majority of meningococci
(a) Bacterial resistance to the drug is very common from carriers, highly resistant strains may
(b) Complete liver failure has occured be selected out during treatment
(c) It is very expensive (a) Ciprofloxacin (b) Clofazimine
(d) Its use is associated with torsade de pointes (c) Dapsone (d) Rifampin
(e) Nephrotoxicity is dose-limiting (e) Streptomycin
10. The primary reason for the use of drug 14. Once–weekly administration of this
combinations in the treatment of tuberculosis antibiotic has prophylactic activity against
is to bacteremia due to M avium complex in
(a) Ensure patient compliance with the drug AIDS patients.
regimen (a) Azithromycin (b) Clarithromycin
(b) Reduce the incidence of adverse effects (c) Isoniazid (d) Kanamycin
(c) Enhance activity against metabolically (e) Rifabutin
inactive mycobacteria
(d) Delay or prevent the emergence of resistance 15. Which one of the following drugs is most
likely to cause loss of equilibrium and
(e) Provide prophylaxis against other bacterial
auditory damage?
infections
(a) Amikacin
11. The mechanism of high – level INH
(b) Ethambutol
resistance of M tuberculosis is
(c) Isonazid
(a) Formation of drug-inactivating N-acetyltrans-
(d) Para-aminosalicylic acid
ferase
(e) Rifabutin
(b) Reduced expression of the katG gene
(c) Decreased intracellular accumulation of INH 16. Chemical interactions between this drug
(d) Mutation in the inhA gene and cell membrane components can
result in the formation of pores lined by
(e) Change in the pathway of mucolic acid
hydrophilic groups present in the drug
synthesis
molecule.
CHEMOTHERAPY 169

(a) Dactinomycin (b) Griseofluvin (d) Increased activity of host cell phosphodi-
(c) Fluconazole (d) Nystatin esterases that degrade tRNA is one of the
(e) Terbinafine antiviral actions of interferons
(e) Foscarnet has no requirement for activation
17. Which one of the following statements by phosphorylation
about fluconazole is most accurate?
22. Which of the following drugs is most like-
(a) It is highly effective in treatment of aspergillosis ly to cause additive anemia and neutro-
(b) It does not penetrate the blood – brain barrier penia if administered to an AIDS patient
(c) Its oral bioavailability is less than that of taking zidovudine?
ketoconazole (a) Acyclovir (b) Amantadine
(d) It inhibits demethylation of lanosterol (c) Ganciclovir (d) Pentamidine
(e) It is potent inhibitor of hepatic drug– (e) Stavudine
metabolizing enzymes
23. The antiviral actions of this drug include
18. Which one of the following drugs is least inhibition of both RNA and DNA synthe-
likely to be effective in the treatment of sis. The drug is used for the treatment of
esophageal candidiasis, it is used by the severe respiratory syncytial virus infec-
oral route? tions in neonates.
(a) Amphotericin B (b) Clotrimazole (a) Amantadine (b) Amprenavir
(c) Fluconazole (d) Griseofulvin (c) Foscarnet (d) Ribavirin
(e) Ketoconazole (e) Ritonavir
19. Which one of the following statements 24. Regarding interferon alpha, which one
about flucytosine is accurate? of the following statements is least
(a) It is bioactivated by fungal cytosine deaminase accurate?
(b) It does not cross the blood–brain barrier (a) At the start of treatment, most patients
(c) It inhibits cytochrome P450 experience flu-like symptoms
(d) It is useful in esophageal candidiasis (b) Indications include treatment of genital warts
(e) It has a wide spectrum of antifungal activity (c) It is used in the management of hepatitis C
(d) Lamivudine interferes with its activity against
20. Which one of the following drugs is most hepatitis B
appropriate for oral use in vaginal
(e) Toxicity includes bone marrow suppression
candidiasis?
(a) Clotrimazole (b) Griseofluvin 25. Over 90% of this drug is excreted in the
urine in intact form. Because its urinary
(c) Fluconazole (d) Flucytosine
solubility is low, patients should be well
(e) Nystatin hydrated to prevent nephrotoxicity
21. Which one of the following statements (a) Acyclovir (b) Amantadine
about the mechanisms of action of (c) Indinavir (d) Zanamivir
antiviral drugs is least accurate? (e) Zidovudine
(a) The initial step in activation of famciclovir in
26. Used in the prophylaxis and treatment of
HSV-infected cells is its phosphorylation by
infection due to influenza viruses, this drug
viral thymidine kinase
facilitates clumping of mature virions and
(b) The reverse transcriptase of HIV is 30–50 their adhesion to infected cells.
times more sensitive to inhibition by indinavir
than host cell DNA polymerases (a) Amantadine (b) Efavirenz
(c) Gancicolvir inhibits viral DNA polymerase (c) Oseltamivir (d) Rimantadine
but does not cause chain termination (e) Saquinavir
170 MCQs IN PHARMACOLOGY

27. Which statement regarding phase-specific 32. A patient with AIDS has an extremely
chemotherapic agents is correct ? They high viral RNA titer. While blood is being
(a) Are most effective in one phase of the cell drawn from this patient, the syringe is
cycle accidentally dropped, contaminating the
floor, which is made of porous material.
(b) Effective in all phases of the cell cycle
The best way to deal with this is to
(c) Are only effective in G0 phase
(a) Completely replace the contaminated part
(d) Include the alkylating agents
of the floor
(e) Include the antituber antibiotics
(b) Clean the floor with soap and water
28. Infections due to gram–negative bacilli (c) Seal the room and decontaminate with
have occurred when this agent has been ethylene oxide
used as a skin antiseptic (d) Clean the floor with a 10% solution of
(a) Acetic acid household bleach
(b) Benzalkonium chloride (e) Neutralize the spill with a solution of
(c) Hexachlorophene potassium permanganate
(d) Merbromin 33. Certain anaerobic protozoan parasites
(e) Thimerosal lack mitochondria and generate energy–
rich compounds, such as acetyl–CoA, by
29. Which one of the following compounds means of enzymes present in organelles
is used topically to treat scabies and called hydrogenosomes. An important
pediculosis? enzyme involved in this process is
(a) Lindane (a) Cytochrome P450
(b) Mupirocin (b) Glycerol – 3 – phosphate oxidase
(c) Nitrofurazone (c) Hypoxanthine – guanine phosphoribosyltran-
(d) Polymyxin B ferase
(e) Silver sulfadiazine (d) Pyruvate – ferredoxin oxidoreductase
30. Methenamine salts are used as urinary (e) Thymidylate synthase
antiseptics. The reason why they lack
34. Which of the following compounds is a
systemic antibacterial action is that they
good substrate for hypoxanthine–
are
guanine phosphoribosyltransferase in
(a) Not absorbed into the systemic circulation trypanosomes (but not mammals) and is
following oral ingestion eventually converted into metabolites that
(b) Rapidly metabolized by liver drug– metabolizing are incorporated into RNA?
enzymes (a) Allopurinol
(c) Converted to formaldehyde only at low urinary (b) Alpha - difluoromethylornithine
pH (c) Glycerol
(d) Substrates for active tubular secretion (d) Mebendazole
(e) Over 90% bound to plasma proteins (e) Salicylhydroxamic acid
31. Which one of the following antiseptics 35. One chemotherapeutic strategy used to
promotes wound healing? eradicate the blood-stream form of
(a) Cetylpyridium African trypanosomes is based on the
(b) Chlorhexidine absolute dependence of the organism on
(c) Hexachlorophene (a) Cytochrome – dependent electron transfer
(d) Iodine (b) Dihydropteroate synthesis
(e) None of the above (c) Glycolysis
CHEMOTHERAPY 171

(d) Lactate dehydrogenase glucose – 6 – phosphate dehydrogenase


(e) Mitochondrial respiration deficiency.
(a) Chloroquine (b) Clindamycin
36. Which of the following drugs enhances
GABA actions on the neuromuscular (c) Mefloquine (d) Primaquine
junctions of nematodes and arthropods? (e) Quining
(a) Glutamic acid (b) Ivermectin 42. This drug can clear trypanosomes from
(c) Picrotoxin (d) Pyrantel pamoate the blood and lymph nodes and is active
(e) Pyrimethamine in the late CNS stages of African sleeping
sickness.
37. Which of the following drugs is an anti-
(a) Emetine (b) Melarsoprol
metabolite that inhibits a trypanosomal
enzyme involved in putrescine synthesis? (c) NIfurtimox (d) Pentamidine
(a) Alpha - difluoromethylornithine 43. Which one of the following drugs is
(b) Alpha - fluorodeoxyuridine recommended as a single agent for oral
(c) Metronidazole treatment of uncomplicated malaria due
to chloroquine–resistant P falciparum
(d) Polymyxin
strains?
(e) Thiopurinol riboside
(a) Doxycycline (b) Iodoquinol
38. Which one of the following enzymes is (c) Primaquine (d) Proguanil
not unique to parasites?
(e) Quinine
(a) Dihydropteridine pyrophosphokinase
44. A missionary from Chicago is sent to work
(b) Hypoxanthine–guanine phosphoribosyltrans-
in a geographic region of a Cental Ameri-
ferase
can country wher onchocerca volvulus is
(c) Lanosterol demethylase endemic. Infections due to this tissue
(d) Purine nucleoside phosphotransferase nematode (onchocerciasis) are a major
(e) Trypanothione reductase cause of “river blindness”, since microfi-
lariae migrate through subcutaneous tis-
39. Which of the following antimalarial drugs sues and concentrate in the eyes. Which
causes a dose–dependent toxic state that
one of the following durgs can be used
includes flushed and sweaty skin,
prophylactically to prevent onchocercia-
dizziness, nausea, diarrhea, tinnitus,
sis?
blurred vision, and impaired hearing?
(a) Bithionol (b) Ivermectin
(a) Amodiaquine (b) Primaquine
(c) Niclosamide (d) Oxamniquine
(c) Pyrimethamine (d) Quinine
(e) Suramin
(e) Sulfadoxine
45. In a patient with diffuse lymphoma, the
40. Plasmodial resistance to chloroquine is oncologist suggests a treatment strategy
due to that involves the initial administration of
(a) Change in receptor structure doxorubicin to obtain a significant log–
(b) Decreased carrier–mediated drug transport kill, followed by the cell cycle–specific
(c) Increase in the activity of DNA repair drugs cytarabine and vincristine. This
mechanisms therapeutic strategy is called
(d) Induction of inactivating enzymes (a) Pulse therapy
(e) Inhibition of dihydrofolate reductase (b) Recruitment
(c) Rescue therapy
41. This drug is the antimalarial agent most
commonly associated with causing an (d) Sequential blockade
acute hemolytic reaction in patients with (e) Synchrony
172 MCQs IN PHARMACOLOGY

46. Which one of the following statements (a) B lymphocyte


about the mechanisms of action of drugs (b) Cytotoxic T lymphocyte
used in cancer chemotherapy is least (c) Dendritic cell
accurate?
(d) Macrophage
(a) Alkylatine agents commonly attack the
(e) TH lymphocyte
nucleophilic N-7 position in guanine
(b) Anthracyclines intercalate with base pairs to 51. Cyclosporine is effective in organ trans-
block nucleic acid synthesis plantation. The immunosuppressant
(c) In steady doses, leuprolide inhibits the release action of the drug appears to be due to
of pituitary gonadotropins (a) Activation of natural killer (NK) cells
(d) Mercaptopurine is an irreversible inhibitor (b) Blockade of tissue responses to inflammatory
of HGPRT ase mediators
(e) Paclitaxel acts mainly in the M phase of the (c) Increased catabolism of IgG antibodies
cell cycle (d) Inhibition of the gene transcription of
47. Which of the following agents used in interleukins
drug combination regimens to treat (e) Interference with antigen recognition
testicular carcinoma is mot likely to cause
52. Azathiprone
nephrotoxicity?
(a) Binds avidly to a cytoplasmic immunophillin
(a) Bleomycin (b) Cisplatin
(b) Blocks formation of tetrahydrofolic acid
(c) Etoposide (d) Leuprolide
(c) Is a precursor of cytarabine
(e) Vinblastine
(d) Is markedly hematotoxic and has caused
48. Which one of the following is least likely neoplasms
to be a mechanism of cancer cell resistance
(e) Is a metabolite of mercaptopurine
to antineoplastic drugs?
(a) Change in properties of a target enzyme 53. Body Surface Area (BSA) is used in
calculating chemotherapy doses because
(b) Decreased activity of activating enzymes
(c) Increase in drug–metabolizing cytochrome (a) BSA is an indicator of tumor cell mass
P450 (b) BSA correlates with cardiac output
(d) Increase in DNA repair (c) BSA correlates with gastrointestinal transit
(e) Increase in production of drug–trapping time
molecules (d) the National Cancer Institute requires that
BSA be used
49. All of the following agents have been
(e) the Food and Drug Administration requires
used in drug regimens for the treatment
that BSA be used
of breast carcinoma. Which one has
specific activity in a subset of female 54. Which one of the following agents acts
breast cancers? at the step of antigen recognition?
(a) Anastrozole (b) Doxorubicin (a) Cyclosporine
(c) Fluoxymesterone (d) Methotrexate (b) Cyclophosphamide
(e) Trastuzumab (c) Methotrexate
50. Which cell involved in immune function (d) Rho(D)immune globulin
recognizes foreign peptides bound of (e) Tacrolimus
MHC class II molecules on the surface of
APC cells, secretes interleukin-2, and 55. Tumor necrosis factor - α - appears to play
initiates the cell-mediated immunity an important role in autoimmunity and
reaction responsible for host-versus-graft inflammatory diseases. Which of the
reactions? following is a humanized monoclonal
CHEMOTHERAPY 173

antibody that binds to TNF - a and inhibits 60. Which of the following drugs or drug
its action? groups is not useful in the prevention of
nausea and vomiting included by cancer
(a) Etanercept
chemotherapy ?
(b) Infliximab
(a) Dexamethasone (b) Dronabinol
(c) Muromonab – CD3
(c) Ketaserin (d) Ondansetron
(d) Sirolimus
(e) Phenothiazines
(e) Thalidomide
61. Passive immunization involves
56. Which one of the following agents is able
(a) Live immunogens
to suppress both B and T lymphocytes via
its inhibition of de novo synthesis of (b) Polysaccharide vaccines
purines? (c) Stimulation of antibody formation
(a) Cyclophosphamide (d) Use of antigens
(e) Use of preformed antibodies
(b) Methotrexate
(c) Mycophenolate mofetil 62. A businessman intends to travel abroad
(d) Prednisone in a geographical region where several
diseases are endemic. He would not be
(e) Tacrolimus
able to be vaccinated against
57. Which one of the following agents increases (a) Cholera
phagocytosis by macrophages in patients (b) Malaria
with chronic granulomatous disease?
(c) Meningococcal infection
(a) Aldesleukin (d) Typhoid fever
(b) Interferon - γ (e) Yellow fever
(c) Lymphocyte immune globulin
63. Which of the following is used in active
(d) Prednisone immunization of children and combines
(e) Trastuzumab bacterial toxoids with a bacterial
antigen?
58. A young woman employed as a dental
laboratory technician complains of (a) BCG (b) BSA
conjunctivitis, skin irritation, and hair loss. (c) DTP (d) ISG
On examination, she has perforation of (e) Rho(D)
the nasal septum and a “milk and roses”
64. Which of the following is a polysaccharide
complexion. These signs and symptoms
used for active immunization in patients
are most likely to be due to
with chronic cardiorespiratory ailments?
(a) Acute mercury poisoning (a) Antilymphocyte immune serum
(b) Chronic inorganic arsenic poisoning (b) BCG vaccine
(c) Chronic mercury poisoning (c) Mumps virus vaccine
(d) Excessive use of supplementary iron tablets (d) Pertussis immune globulin
(e) Lead poisoning (e) Pneumococcal vaccine
59. In the treatment of acute inorganic ar- 65. A needlestick injury is sustained by a
senic poisoning, the most likely drug to health care worker, and the blood is
be used is known to contain HBV surface antigens.
(a) Deferoxamine (b) Dimercaprol The health care worker should be given
(c) EDTA (d) Penicillamine (a) Nothing
(e) Succimer (b) Immune globulin
(c) Hepatitis B immune globulin
174 MCQs IN PHARMACOLOGY

(d) Hepatitis B vaccine 71. This agent, which is used in the chemo-
(e) Hepatitis B vaccine and hepatitis B immune therapy of Hodgkin’s lymphoma, is po-
globulin tentially leukemogenic
66. Which one of the following compounds (a) Dacarbazine (b) Doxorubicin
enhances immune function in vitro and (c) Prednisone (d) Procarbazine
in clinical trials decreases the symptoms (e) Vinblastine
of the common cold?
(a) Echinacea 72. A 54-year-old farmer has a 5-year
history of frequent, recurrent, and very
(b) Feverfew
painful kidney stones. Appropriate
(c) Garlic
chronic therapy for this man is
(d) Melatonin
(a) Furosemide
(e) Milk Thistle
(b) Hydrochlorothiazide
67. An important therapeutic or toxic effect
(c) Morphine
of loop diuretics is
(d) Spironolactone
(a) Decreased blood volume
(e) Triamterene
(b) Decreased heart rate
(c) Increased serum sodium 73. A 55-year-old executive has cardiomy-
(d) Increased total body potassium opathy and congestive heart failure. He
(e) Metabolic acidosis is being treated with diuretics. The mech-
anism of action of furosemide is best de-
68. The most appropriate drug for reversing scribed as
myasthenic crisis in a patient who is
experiencing diplopia, dysarthria, and (a) Interference with H+/HCO3 exchange
difficulty swallowing is (b) Blockade of a Na+/K+/2Cl- transporter
(a) Neostigmine (b) Pilocarpine (c) Blockade of a Na+/Cl- cotransporter
(c) Pralidoxime (d) Succinylcholine (d) Blockade of carbonic anhydrase
(e) Tubocurarine (e) Inhibition of genetic expression of DNA in
the kidney
69. In the management of patients with AIDS,
the sulfonamides are often used in 74. Which one of the following peptides is
combination with inhibitors of folate not a vasodilator?
reductase. However, such combinations
(a) Atrial natriuretic factor (ANF)
have minimal activity against
(b) Calcitonin gene-related peptide
(a) Escherichia coli
(c) Endothelin
(b) Nocardia species
(d) Substance P
(c) Pneumocystis carinii
(e) Vasoactive intestinal peptide
(d) Toxoplasma gondi
(e) Treponema pallidum 75. A young female patient using an oral
contraceptive is to be treated for pulmo-
70. Chemoprophylaxis for travelers to
geographic regions where chloroquine– nary tuberculosis. She is advised to use
resistant P falciparum is endemic is best an additional method of contraception
provided by since the efficacy of the oral agents is
commonly decreased if her drug regimen
(a) Atovaquone
includes
(b) Mefloquine
(a) Amikacin (b) Ethambutol
(c) Primaquine
(d) Pyrimethamine plus sulfadoxine (c) Isoniazid (d) Pyrazinamide
(e) Quinine (e) Rifampin
CHEMOTHERAPY 175

76. Which one of the following drugs is most (c) Increased likelihood of causing exophthalmo
likely to be effective in the treatment of during the first week of treatment
gonorrhea in this patient and safe to use? (d) Increased risk of fetal toxicity
(a) Amoxicillin - clavulanate (e) More sustained antithyroid activity when used
(b) Ceftriaxone continuously for several months
(c) Clarithromycin 82. In antiviral immunity, what directly
(d) Ofloxacin recognized and kills viral-infected cells?
(e) Tetracycline (a) Cytotoxic T cells (CTLs)
77. The rationale for combination chemother- (b) Antiviral antibodies
apy includes all of the following except (c) Interferons
(a) Biochemical enhancement of effect 83. An organ donor who is human leukocyte
(b) rescue of normal cells antigen (HLA)- matched with the recipient
(c) overcoming or preventing resistance of a graft is sought. Which individual is
(d) biochemical nullification of effect at least somewhat likely to provide a total
HLA match?
(e) cytotoxic to both resting and dividing cells
(a) A sibling of the graft recipient
78. This compound reduces the need for
(b) A parent of the graft recipient
platelet transfusions in patients undergoing
cancer chemotherapy (c) A cadaver

(a) Cyanocobalamin 84. Graft–versus–host (GVH) disease is


(b) Erythropoietin associated primarily with which type of
transplantation?
(c) Interleukin - II
(d) Iron dextran (a) Kidney
(e) Tranexamic acid (b) Heart
(c) Bone marrow
79. Which one of the following anticancer
drugs acts in the M-phase of the cell cycle 85. Which is a valid comparison of live, at-
to prevent disassembly of the mitotic tenuated and killed, inactivated active
spindle? vaccines?
(a) Dactinomycin (b) Etoposide (a) Replication of the organisms in a live,
(c) Paclitaxel (d) Procarbazine attenuated vaccine increases the stimulation
of the immune system, and a lower dose is
(e) Vinblastine
often required
80. While colchicines has been used in acute (b) Attenuated vaccines often require multiple
gout, the drug often causes severe doses
gastrointestinal distress. Consequently, (c) A killed, inactivated vaccine probably
many authorities now consider that the produces life-long immunity in one or two
drug of choice for acute gout is doses
(a) Acetaminophen (b) Aspirin
86. Which type of cell does not contain double
(c) Indomethacin (d) methotrexate standard deoxyribonucleic acid (DNA)?
(e) Sulfinpyrazone
(a) Human cells
81. Relative to Lugol’s solution propylthou- (b) Bacteria cells
racil has (c) Human immunodeficiency virus (HIV)
(a) A faster onset of antithyroid action cells
(b) A greater inhibitory effect on the proteolytic 87. Which enzyme is used by the Human
release of hormones from the thyroid gland Immunodeficiency Virus (HIV) to form
176 MCQs IN PHARMACOLOGY

deoxyribonucleic acid (DNA) in the host (b) Adenine (c) Guanine


cell? (d) Thymine (e) All of the above
(a) Restrictive endonuclease
94. Which of the following statements
(b) DNA-directed polymerase regarding signal transduction is incorrect?
(c) Reverse transcriptase
(a) Thyroxine-bound receptors act on DNA and
(d) Both (a) and (b) regulate specific transcription of genes.
(e) None of the above (b) Cyclic adenosine monophosphate can act
88. Gamma immunoglobulin is considered as second messenger.
(c) The level of drug receptors at the cell surface
(a) Deoxyribonucleic acid (DNA)
increases with chronic stimulation by receptor
(b) Ribonucleic acid (RNA) agonists
(c) Protein (d) Binding of ligand to cell-surface receptors
(d) None of the above can lead to synthesis of proteins
89. Glycoprotein is considered a protein linked (e) Antacids act by interacting with small ions
to normally found in the gastrointestinal tract.
(a) A carbohydrate 95. All of the following chemotherapy agents
(b) A hormone can be administered intrathecally except
(c) A lipid (a) Methotrexate (b) Cytrabine
(d) A deoxyribonucleic acid (DNA) (c) Hydrocortisone (d) Thiotepa
(e) None of the above (e) Vincristine

90. An enzyme that cleaves deoxyribonucleic 96. The efficacy rates for nonprescription
acid (DNA) at a specific site is called antifungal agents for vaginal yeast
(a) Restrictive endonuclease infections is
(b) Restrictive ribonuclease (a) 50% (b) 60%
(c) Trypsin (c) 70% (d) 80%
(d) None of the above 97. The best product to treat vulvar pruritus
91. An example of a cytokine is in a woman with a vaginal yeast infection
is
(a) Interleukin
(a) External miconazole (Monistat)
(b) Insulin
(b) External miconazole and intravaginal
(c) Gonadotropin
miconazole (Monistat-7 combination pack)
(d) Thyroxine
(c) Intravaginal tioconazole (Vagistat 1)
(e) None of the above
(d) Intravaginal butoconazole (Femstat 3)
92. A common storage condition for most
98. Which of the following patients complain-
biotechnology products after reconstitution
ing of vaginal yeast infection symptoms
is
should be referred to a physician?
(a) Room temperature
(a) If there is a history of recurrent vaginal yeast
(b) Cool place infection
(c) Warm place (b) If she is pregnant
(d) No excessive heat (c) If she is less than age 12
(e) Freezer (d) All of the above
93. What base is found in deoxyribonucleic 99. Which of the following herbs is known
acid (DNA)? to cause cancer?
(a) Cytosine
CHEMOTHERAPY 177

(a) Chaparral (b) Comfrey 104. Cisapride should not be used in combination
(c) Ma huang (d) Licorice with either fluconazole or indinavir because
(e) St. John’s wort of increased potential for

100. Which of the following herbs should be (a) Atrial fibrillation


used with caution while driving or (b) Atrial flutter
performing other tasks that require (c) Ventricular fibrillation
alertness and coordination? (d) Torsades de pointes
(a) Valerian (b) Echinacea (e) Angina pectoris
(c) Dong quai (d) Feverfew
105. Which statement reageding phase-
(e) Saw palmetto
specific chemotherapeutic agents is
101. Tom would like to try Echinacea to correct? They
prevent colds and flus during the winter
(a) Are most effective in one phase of the cell
months. Which of the following
cycle
statements is true about Echinacea?
(b) Are effective in all phases of the cell cycle
(a) It is contraindicated in patients allergic to
(c) Are only effective in G0 phase
parsley
(b) It should only be taken continuously for three (d) Include the alkylating agents
months (e) Include the antitumor antibiotics
(c) It is contraindicated in patients with lupus 106. Body surface area (BSA) is used in calcu-
and leucosis
lating chemotherapy doses because
(d) Prolonged use of Echinacea will upregulate
the immune system (a) BSA is an indicator of tumor cell mass
(e) Side effects include headache, rash, and (b) BSA correlates with cardiac output
dizziness (c) BSA correlates with gastrointestinal transit
time
102. Hematological testing of a patient with
acquired immune deficiency syndrome (d) The National Cancer Institute requires that
(AIDS) is most likely to show which of the BSA be used
following abnormalities? (e) The Food and Drug Administration (FDA)
(a) Basophilia (b) Eosinophilia requires that BSA be used
(c) Lymphopenia (d) Reticulocytosis 107. The rationale for combination chemother-
(e) Agranulocytosis apy includes all of the following except
103. Hematological studies are most likely to (a) Biochemical enhancement of effect
show a low reticulocyte count in a patient (b) Rescue of normal cells
who has which one of the following (c) Overcoming or preventing resistance
abnormalities ?
(d) Biochemical nullification of effect
(a) Aplastic anemia secondary to cance (e) Cytotoxic to both resting and dividing cells
chemotherapy
(b) Acute hemolytic anemia secondary to 108. Which of the following chemotherapeutic
quinidine treatment agents is classified as an alkylating agent?
(c) Severe bleeding secondary to an automobile (a) Cyclophosphamide
accident (b) Etoposide
(d) Iron deficiency anemia 1 week after
(c) Mechlorethamine
treatment with ferrous sulfate
(d) Paclitaxel
(e) Megaloblastic anemia due to folate
deficiency 1 week after treatment with folic (e) Cyclophosphamide and mechlorethamine
acid
178 MCQs IN PHARMACOLOGY

109. Which of the following chemotherapy (a) Ovarian carcinoma


agents acts by intercalation? (b) Psoriasis
(a) Vincristine (c) Breast carcinoma
(b) Paclitaxel (d) Melanoma
(c) Doxorubicin (e) Hodgkin’s disease
(d) Vincristine and paclitaxel 115. The principal active alkaloid of ipecac is
(e) Topotecan
(a) Yohimbine (b) Caffeine
110. How do antimetabolites exert their (c) Apomorphine (d) Lobeline
cytotoxic effect? (e) Emetine
(a) Inhibiting DNA synthesis by sliding between 116. Emetine is used in the treatment of
DNA base pairs
(a) Malalria (b) Schistosomiasis
(b) Inhibiting RNA synthesis by sliding between
RNA base pairs (c) Tuberculosis (d) Amebiasis
(c) Acting as false metabolites in the (e) None of the above
microtubules 117. The therapeutic use of neostigmine
(d) Acting as false substitutions in the production bromide is as a
of nucleic acids (a) Miotic
(e) Promoting microtubule assembly and (b) CNS depressant
stabilization
(c) Treatment of curare poisoning
111. Which of the following chemotherapy (d) Neuromuscular blocker
agents are correctly paired according to (e) Muscle relaxant
their mechanism of action?
118. Ethylenediamine-tetracetic acid is an
(a) Vincristine and paclitaxel antidote for which of the following?
(b) Etoposide and paclitaxel
(a) Sodium secobarbital
(c) Docetaxel and paclitaxel
(b) Aspirin
(d) Docetaxel and etoposide
(c) Paris green
(e) Vincristine and etoposide
(d) Phosphorus
112. When does not neutrophil nadir associated (e) Lead
with chemotherapy agents generally
119. Which of the following chemotherapeutic
occur?
agents is classified as an alkylating
(a) During administration of the chemotherapy agent?
(b) 1–2 days after therapy (a) Cyclophosphamide
(c) 10–14 days after therapy (b) Etoposide
(d) 1 month after therapy (c) Mechlorethamine
(e) When the platelet count begins to rise (d) Paclitaxel
113. N-acetylcysteine is classified as a (an) (e) Cyclophosphamide and mechlorethamine
(a) Analgesic 120. Which of the following is a gametocidal
(b) Antitussive agent in malaria?
(c) Mucolytic agent (a) Atabrine (b) Quinine
(d) Antitubercular agent (c) Paludrine (d) Primaquine
(e) Protein hydrolysate (e) Aralen

114. Procarbazine (Matulane) is used primarily 121. Which of the following is contraindicated
to treat in the presence of active tuberculosis?
CHEMOTHERAPY 179

(a) Hydrocortisone (b) Streptomycin (a) Irritable bowel syndrome


(c) INH (d) PAS (b) Cholera
(e) PABA (c) Salmonella diarrhoeas
122. A drug that is very effective in the (d) Traveller’s diarrhoea
treatment of tinea versicolor is 129. The therapeutic effect of sulfasalazine in
(a) Undecylenic acid ulcerative colitis is exerted by
(b) Clotrimazole (Lotrimin) (a) Inhibitory action of the unabsorbed drug on
(c) Acrisorcin (Akrinol) the abnormal colonic flora
(d) Penicillin G (b) Breakdown of the drug in colon to release
(e) Prednisolone 5-aminosalicylic acid which suppresses
inflammation locally
123. Which of the following is a first-line drug
to treat tuberculosis? (c) Release of sulfapyridine having antibacterial
property
(a) PAS (b) Rifampin
(d) Systemic immunomodulatory action of the
(c) Ethionamide (d) Cycloserine drug
(e) Ethambutol
130. The preferred drug for controlling an
124. Yellow pigmentation of the skin is most acute exacerbation of ulcerative colitis is
common with
(a) Prednisolone (b) Sulfasalazine
(a) Chloraquine (b) Pamaquin
(c) Mesalazine (d) Vancomycin
(c) Quinacrine (d) Quinine
(e) Atabrine 131. To be effective in ulcerative colitis, 5-
aminosalicylic acid has to be given as
125. The most effective single chemotherapeutic
(a) Acrylic polymer coated tablet which releases
agent of those indicated, in the treatement
of tuberculosis is the drug only in the lower bowel
(b) A complex of two molecules joined together
(a) Neomycin (b) Sulfones
by azo bond
(c) Streptomycin (d) Penicillin
(c) A retention enema
(e) Terramycin
(d) All of the above
126. Which is an antidote for malathion
poisoning? 132. A small amount of atropine is added to
the diphenoxylate tablet/syrup to
(a) Vitamin K
(a) Suppress associated vomiting of gastroenteritis
(b) Protamine sulfate
(b) Augment the antimotility action of diphen-
(c) Nalorphine (Nalline)
oxylate
(d) Pralidoxine (Protopam)
(c) Block side effects of diphenoxylate
(e) Edrophonium (Tensilon)
(d) Discourage overdose and abuse of
127. Antimicrobial treatment does not alter the diphenoxylate
course of the following diarrhoeas except
133. Select the sulfonamide drug which is
(a) Mild enterotoxigenic E.coli diarrhoea active against pseudomonas and is used
(b) Campylobacter diarrhoea by topical application for prophylaxis of
(c) Coeliac disease diarrhoea infection in burn cases
(d) Food poisoning diarrhoea (a) Sulfadiazine
128. Which of the following diarrhoeas is (b) Silver sulfadiazine
consistently benefited by antimicrobial (c) Sulfadoxine
therapy (d) Sulfamethoxazole
180 MCQs IN PHARMACOLOGY

134. The drug of choice for prophylaxis of 140. Indicate the drug which attain therapeutic
meningococcal meningitis during an antibacterial concentration in the urinary
epidemic is tract but not in other tissues
(a) Phenoxymethyl penicillin (a) Sulfasomidine (b) Piperacillin
(b) Tetracycline (c) Nitrofurantoin (d) Both (b) and (c)
(c) Rifampin
141. Acidic urine augments the antibacterial
(d) Ciprofloxacin action of the following drug
135. Trimethoprim inhibits bacteria without (a) Sulfadiazine (b) Cotrimoxazole
affecting mammalian cells because (c) Gentamicin (d) Nitrofurantoin
(a) It does not penetrate mammalian cells
142. Which of the following is not a first line
(b) It has high affinity for bacterial but low affinity
antitubercular drug ?
for mammalian dihydrofolate reductase
enzyme (a) Ciprofloxacin (b) Streptomycin
(c) It inhibits bacterial folate synthetase as well (c) Pyrazinamide (d) Ethambutol
as dihydrofolate reductase enzymes
143. The intermittently multiplying (spurter)
(d) All of the above tubercle bacilli present within caseous
136. Trimethoprim is combined with sulf- material having low oxygen tension are
amethoxazole in a ration of 1:5 to yield most susceptible to
a steady state plasma concentration ratio (a) Ethambutol (b) Rifampin
of (c) Streptomycin (d) Pyrazinamide
(a) Trimethoprin 1:Sulfamethoxazole 5
144. Occurrence of the following adverse
(b) Trimethoprin 1:Sulfamethoxazole 10 reaction absolutely contraindicates
(c) Trimethoprin 1:Sulfamethoxazole 20 further use of rifampin in the treatment
(d) Trimethoprin 1:Sulfamethoxazole 1 of tuberculosis
137. Indicate the condition in which neither (a) Respiratory syndrome
trimethoprim nor sulfamethoxazole alone (b) Cutaneous syndrome
are effective, but their combination (c) Flu syndrome
cotrimoxazole is (d) Abdominal syndrome
(a) Prostatitis
145. Which of the following chemotherapy
(b) Lymphogranuloma venereum
agents acts by intercalation?
(c) Pneumocystis carinii pneumonia
(a) Vincristine
(d) Bacillary dysentery
(b) Paclitaxel
138. Select the antimicrobial drug which is (c) Doxorubicin
used orally only for urinary tract infection
(d) Vincristine and paclitaxel
for bacterial diarrhoeas
(e) Topotecan
(a) Pefloxacin
(b) Azithromycin 146. Ethambutol is not used in children below
6 years of age because
(c) Bacampicillin
(d) Nalidixic acid (a) Young children are intolerant to ethambutol
(b) Ethambutol causes growth retardation in
139. Nalidixic acid is primarily active against
young children
(a) Cocci (c) It is difficult to detect ethambutol induced
(b) Bacilli visual impairment in young children
(c) Gram-positive bacteria (d) In young children visual toxicity of ethambutol
(d) Gram-negative bacteria is irreversible
CHEMOTHERAPY 181

147. In a patient of pulmonary tuberculosis, (d) Isoniazid + Rifampin for 6 months with
pyrazinamide is most active on the additional Pyrazinamide during the initial 2
following subpopulation of tubercle bacilli months
(a) Rapidly multiplying bacilli located on cavity
152. Corticosteroids are absolutely contrain-
walls
dicated in which of the following types
(b) Slow growing bacilli within macrophages
of tuberculosis
and at sites showing inflammatory response
(c) Intermittently multiplying bacilli within (a) Miliary (b) Meningeal
caseous material (c) Intestinal (d) Renal
(d) Dormant bacilli
153. Multi Drug Resistant (MDR) tuberculosis
148. The most important reason for using a is defined as resistance to
combination of chemotherapeutic agents
(a) Any two or more antitubercular drugs
in the treatment of tuberculosis is
(b) Isoniazid + any other antitubercular drug
(a) To obtain bactericidl effect
(c) Isoniazid + Rifampin ± any one or more
(b) To prevent developmet of resistance to the
antitubercular drugs
drugs
(c) To broaden the spectrum of activity (d) All five first line antitubercular drugs
(d) To reduce adverse effects of the drugs 154. Mycobat tuberculosis infection in a HI V
infected patient is treated with
149. Addition of pyrazinamide and ethambutol
for the first two months to the isoniazid + (a) The same antitubercular regimen as HIV
rifampin therapy of tuberculosis serves the negative patient
following purpose(s) (b) Four first line antitubercular drugs for 2
(a) Reduces the total duration of therapy to 6 months followed by a longer continuation
months phase of 7 months with rifampin + isoniazid
(b) Produces more rapid sputum conversion (c) All five first line antitubercular drugs for nine
(c) Permits reduction of rifampin dose months
(d) Both (a) and (b) (d) Clarithromycin + Ciprofloxacin + Rifabutin
150. What is true of short course DOTS(WHO) for 12 months
stategy for treatment of tuberculosis ? 155. The most important dose dependent
(a) It consists of an initial intensive phase and a toxicity of dapsone is
later continuation phase
(a) Methemoglobinemia
(b) The dose of antitubercular drugs is reduced
(b) Haemolysis
after clinical response occurs
(c) The patient himself is made responsible for (c) Hepatitis
administering antitubercular drugs (d) Dermatitis
(d) All of the above 156. The tetracycline with highest antileprotic
151. According to the currect WHO guidelines activity is
new (untreated) sputum smear positive (a) Minocylcine (b) Docycycline
cases of pulmonary tuberculosis are to
be treated with the following regimen (c) Methacycline (d) Oxytetracycline

(a) Isoniazid + Rifampin + Pyrazinamide for 6 157. Multidrug therapy with dapsone,
months rifampin and clofazimine is the treatment
(b) Isoniazid + Thiacetazone + Rifampin for 2 of choice of
months followed by isoniazid + thiacetazone (a) Multibacillary leprosy
for 6 months
(b) Paucibacillary leprosy
(c) Isoniazid + Rifampin for 6 months with
additional Pyrazinamide + Ethambutol/ (c) Dapsone resistant leprosy
Streptomycin during the initial 2 months (d) All forms of leprosy
182 MCQs IN PHARMACOLOGY

158. The newer lipid formulations of amphot- (c) Varicella-zoster virus


ericin B differ from the conventional for- (d) Epstein–Barr virus
mulation in the following respects except
166. Which of the following is true of acyclovir
(a) They are more efficacious
treatment of genital herpes simplex ?
(b) They produce milder acute reaction
(a) Topical treatment affords symptomatic relief
(c) They are less nephrotoxic
in primary as well as recurrent disease
(d) They produce milder anaemia
(b) Oral therapy for 10 days affords symptomatic
159. The drug of choice for monilial diarrhoea relief as well as prevents recurrences
is (c) Oral therapy for 10 days affords symptomatic
(a) Flucytosine (b) Nystatin relief but does not prevent recurrences
(c) Natamycin (d) Ketoconazole (d) Continuous long-term topical therapy is
recommended to prevent recurrences
160. Griseofulvin is indicated in.
(a) All types of tinea infection 167. The virus directed reverse transcriptase
enzyme is inhibited by
(b) Onychomycosis
(c) Pityriasis versicolor (a) Amantadine (b) Zidovudine
(d) Both (b) and (c) (c) Vidarabine (d) Acyclovir

161. Choose the azole antifungal drug which 168. How do antimetabolites exert their cytotoxic
is used only topically effect?

(a) Ketoconazole (b) Fluconazole (a) Inhibiting DNA synthesis by sliding between
(c) Itraconazole (d) Econazole DNA base pairs
(b) Inhibiting RNA synthesis by sliding between
162. Fluconazole differs from ketoconazole in RNA base pairs
that
(c) Acting as false metabolites in the microtubules
(a) It is not active by the oral route (d) Acting as false substitutions in the production
(b) It is a more potent inhibitor of drug metabolizm of nucleic acids
(c) It is not effective in cryptococcal meningitis (e) Promoting microtubule assembly and
(d) It is unlikely to produce anti-androgenic side stabilization
effects
169. Antiretroviral therapy is not recommend-
163. The only antifungal drug which has some ed in asymptomatic HIV infected subjects
activity against moulds like Mucor and because of the following reason(s).
Aspergillus is
(a) All antiretroviral drugs lose efficacy after
(a) Itraconazole (b) Fluconazole some time
(c) Miconazole (d) Ketoconazole (b) Adverse effects of antiretroviral drugs
164. Iodoxuridine is indicated in compromise the quality of life of
asymptomatic subjects
(a) Herpes simplex keratitis
(c) The treated subjects may produce and
(b) Herpes zoster transmit drug resistant virus
(c) Chicken pox (d) All of the above
(d) All of the above
170. Choose the correct statement about aman-
165. Which of the following viruses is most tadine
susceptible to acyclovir ?
(a) It is an antimetabolite used for viral infections
(a) Herpes simplex type I virus
(b) It prevents penetration of the virus into the
(b) Herpes simplex type II virus host cell
CHEMOTHERAPY 183

(c) Concurrent administration of amantadine (b) Residents of endemic areas


prevents antibody response to influenzavaccine (c) Travellers from nonendemic to endemic areas
(d) It is used to protect high risk subjects during (d) Travellers from endemic to nonendemic areas
an influenza A2 epidemic
177. Radical cure of vivax malaria should be
171. The antiviral action of amantadine is attempted in
exerted through
(a) Areas where only sporadic cases occur
(a) Interaction with a virus directed thymidine (b) Endemic areas with effective vector control
kinase measures
(b) Interaction with a viral M2 protein (c) Endemic areas not covered by vector control
(c) Inhibition of a viral protease enzyme (d) Both (a) and (b)
(d) Inhibition of viral RNA mediated DNA
synthesis 178. Chloroquine acts as
(a) Preerythrocytic schizontocide for both P.
172. Erythrocytic schizontocide antimalarial
falciparum and P. vivax
drugs are used as
` (b) Erythrocytic schizontocide for both P.
(a) Suppressive prophylactic falciparum and P. vivax
(b) Clinical curative (c) Exoerythrocytic schizontocide for P. vivax
(c) Radical curative for P. Vivax (d) Gametocidal for P. falciparum
(d) Both (a) and (b)
179. Which of the following drugs is suitable
173. Which of the following drugs is a causal for treatment of malaria during pregnancy
prophylactic for falciparum malaria and
(a) Quinine
suppressive prophylactic for vivax malaria
(b) Chloroquine
(a) Chloroquine (b) Mepacrine
(c) Pyrimethamine
(c) Quinine (d) Chloroguanide
(d) Primaquine
174. Recrudescence of malaria refers to
180. Amodiaquine differs from chloroquine in
recurrence of malarial fever due to
the following respect(s)
(a) Reinfection of the patient by mosquito bite
(a) It is currently not recommended for treatment
(b) Reinfection of blood by exoerythrocytic of clinical attacks of malaria
hyponozoites
(b) Its used as a suppressive prophylactic is
(c) Incomplete clearance of schizonts from blood prohibited
(d) Reinfection of blood by sporozoites (c) It is less bitter and causes less itching
175. If a drug is active against the preeryth- (d) Both (b) and (c)
rocytic stage of the malarial parasite it
181. Choose the correct statement about me-
will be useful as a
floquine
(a) Suppressive prophylactic
(a) In a single dose it affords clinical cure in all
(b) Causal prophylactic types of malaria
(c) Clinical curative (b) It is selectively active against chloroquine
(d) Radical curative resistant P. falciparum but not the chloroquine
sensitive Strains
176. Chemoprophylaxis of malaria is recom-
mended for the following category of (c) It is recommended only in areas where
subjects chloroquine resistant P.falciparum is prevalent
(d) Both (a) and (c)
(a) Residents of nonedemic areas
184 MCQs IN PHARMACOLOGY

182. The drug of choice for cerebral malaria (a) Pyrimethamine (b) Artemisinin
due to P. falciparum is (c) Primaquine (d) Mefloquine
(a) Quinine
189. All of the following chemotherapy agents
(b) Mefloquine work through affecting microtubule
(c) Chloroguanide function except
(d) Pyrimethamine + Sulfadoxine (a) Docotaxel (b) Vinblastine
183. Intravenous injection of quinine produces (c) Mitoxantrone (d) Vincristine
(a) Rise in blood pressure (e) Vinorelbine
(b) Neuromuscular block 190. Choose the correct statement(s) about
(c) Hyperglycaemia metronidazole
(d) Hypoglycaemia (a) It is a drug of choice for amoebic dysentery
184. The following is true of quinine as well as amoebic liver abscess
(b) It affords the most rapid symptom relief in
(a) It has a longer elimination half-life than
amoebi dysentery
chloroquine
(c) It is the most effective drug in eradicating
(b) It is not to be used for prophylaxis of malaria
amoebic cysts from the colon
(c) It is not active against P. vivax
(d) All of the above
(d) It should not be used along with sulfapy-
rimethamine 191. Metronidazole is used for
185. The fastest acting schizontocidal drug (a) Round worm infestation
among the following is (b) Hook worm infestation
(a) Artemether (c) Kala-azar
(b) Melfoquine (d) Giardiasis
(c) Chloroquine 192. In addition to having antiamoebic activity,
(d) Pyrimethamine tinidazole inhibits
186. Pyrimethamine + sulfadoxine should be (a) Anaerobic bacillin
used as a (b) Aerobic bacilli
(a) Clinical curative in areas with chloroquine (c) Gram-positive cocci
resistance (d) Gram-negative cocci
(b) Clinical curative in areas without chloroquine
193. Tinidazole differs from metronidazole in
resistance
that
(c) Prophylactic in areas with or without
chloroquine resistance (a) It is not active against anaerobic bacteria
(d) All of the above (b) It has a broader spectrum of activity
(c) It has a longer elimination half life
187. Which of the following antimalarial drugs
(d) It has better oral absorption
is more active against pre-and exoeryth-
rocytic stages of the malarial parasite 194. Emetine is now used only as a reserve
than against the erythrocytic stage ? drug for amoebiasis because
(a) Chloroguanide (b) Primaquine (a) It is less effective than metronidazole
(c) Pyrimethamine (d) Quinine (b) It produces a slower response than
metronidazole
188. Use of the following antimalarial drug
carrys high risk of adverse effect in (c) It has cardiotoxic potential
subjects with G-6-PD deficiency. (d) It does not clear stools of amoebic cysts
CHEMOTHERAPY 185

195. Choose the most effective drug for mild 202. Albendazole is less effective than
intestinal amoebiasis and asymptomatic mebendazole in the following helminthic
cyst passers infestation
(a) Metronidazole (b) Emetine (a) Hydatid disease (b) Trichuriasis
(c) Quiniodochlor (d) Diloxanide furoate (c) Strongyloidosis (d) Ascariasis
196. The following antiamoebic drug should 203. The following helminthic disease can be
not be used in children because of risk of treated by albendazole but not by
causing blindness mebendazole
(a) Quinidochlor (b) Diloxanide furoate (a) Hook worm infestation
(c) Tinidazole (d) Secnidazole (b) Thread worm infestation
(c) Trichuriasis
197. After treating intestinal amoebiasis with
metronidazole, a course of diloxanide (d) Neurocysticercosis
furoate is often advised to 204. Piperazine antagonizes the anthelmintic
(a) Cure any subclinical hepatic involvement action of the following drug
(b) Suppress the symbiotic intestinal flora (a) Pyrantel pamoate (b) Mebendazole
(c) Eradicate luminal cyst forming trophozoites (c) Albendazole (d) Niclosamide
(d) Both (b) and (c)
205. The following anthelmintic has been
198. The following drug is used for oral treatment found to be safe during pregnancy
of trichomonas vaginitis (a) Thialbendazole (b) Piperazine
(a) Diiodohydroxyquin (c) Albendazole (d) Pyrantel pamoate
(b) Tinidazole
206. A child has been brought with intestinal
(c) Clotrimazole
obstruction due to clumping of round
(d) Natamycin worms. Which of the following anthelm-
199. The drug of choice for Kala Azar is intics administered by intragastic tube can
relax the ascarids and relieve the obstruc-
(a) Pentamidine tion
(b) Amphotericin B
(a) Levamisole (b) Mebendazole
(c) Sodium stibogluconate
(c) Pyrantel pamoate (d) Piperazine
(d) Ketoconazole
207. Thialbendazole is rarely used now be-
200. Pentamidine + contrimoxazole is the cause
treatment of choice for the following
disease (a) It is not active against round worm and hook
worm
(a) Toxoplasmosis
(b) It produces lower cure rates in intestinal
(b) Pneumocystis carnii pneumonia helminthiasis than mebendazole or
(c) Actinomycosis albendazole
(d) Schistosomiasis (c) It needs pretreatment fasting and post
treatment purgative
201. Leishmania donovani is susceptible to
certain antifungal drugs because both (d) It frequently produces incapacitating side
fungi and Leishmania effects

(a) Utilize purine salvage pathway 208. Select the drug used in the treatment of
(b) Utilize similar glycolytic mechanisms filariasis
(c) Have similar topoisomerase II enzyme (a) Diethyl carbamazine citrate
(d) Have ergosterol in their cell membranes (b) Thialbendazole
186 MCQs IN PHARMACOLOGY

(c) Levamisole (a) Vincristine


(d) Piperazine citrate (b) Chlorambucil
(c) 6-Mercaptopurine
209. Diethyl carbamazine citrate has the
following action in filariasis (d) Cisplatin
(a) Rapidly kills adult filarial worms and stops 215. The most important target of action of
production of microfilariae chlorambucil is
(b) Kills circulating microfilariae (a) Myeloid tissue (b) Lymphoid tissue
(c) Kills microfilariae present in nodules and (c) Neural tissue (d) Skin
serous fluids
216. Which of the following antineoplastic
(d) Promotes phagocytosis of circulating
drugs is a mitotic inhibitor and causes
microfilariae
metaphase arrest ?
210. Hormonal agents that are useful in the (a) Busulfan (b) Vincristine
treatment of cancer include
(c) Cytarabine (d) Procarbazine
(a) Tamoxifen
217. Vinca alkaloids exert antitumor activity
(b) Prednisone
by
(c) Flutamide
(a) Activating topoisomerase II to cause breaks
(d) Tamoxifen and flutamide
in DNA strands
(e) Tamoxifen, prednisone and flutamide
(b) Crosslinking DNA strands
211. Praziquantel is preferred over niclosamide (c) Inhibiting DNA mediated RNA synthesis
for Taenia solium infestation because (d) Inhibiting polymerization of tubulin to form
(a) It achieves higher cure rates intracellular microtubules
(b) It produces fewer side effects
218. Vincristine differs from vinblastine in the
(c) It does not lead to digestion of worm and following respect(s)
kills encysted larvae, so that chances of
(a) Its prominent adverse effect is neuropathy
cysticercosis are minimized
(b) It frequently produces alopecia
(d) Both (a) and (b)
(c) It does not significantly depress bone marrow
212. For the treatment of Hymenolepis nana (d) All of the above
infestation, praziquantel has the following
advantage(s) over niclosamide 219. Patients treated with the following
anticancer drug are likely to develop a
(a) It is better tolerated
disulfiram like reaction on taking alcohol.
(b) It requires single dose treatment against 5
days treatment with niclosamide (a) Dacarbazine (b) Procarbazine
(c) A purgative is required after niclosamide but (c) Melphalan (d) Hydroxyurea
not after praziquantel 220. Select the cell cycle nonspecific antineo-
(d) All of the above plastic drug.
213. Praziquantel is effective against the (a) Vincristine (b) Bleomycin
following helminth(s) (c) Methotrexate (d) 5-Fluorouracil
(a) Taenia saginata 221. Biological response modifiers like GM-CSF
(b) Diphyllobothrium latum are used in conjunction with anticancer
(c) Schistosomes drugs for the following purpose(s).
(d) All of the above (a) To enhance antitumour activity of the drug
214. The following anticancer drug has high (b) To prevent hypersensitivity reactions to the
emetogenic potential drug
CHEMOTHERAPY 187

(c) To hasten recovery from druginduced 229. Isoniazid is ineffective in


myelosuppression (a) Treatment of M. avium complex
(d) Both (a) and (c) (b) Chemoprophylaxis of tuberculosis in HIV-
infected persons
222. Select the drug which is used exclusively
(c) Chemoprophylaxis of tuberculosis in children
in organ transplantation and autoim-
mune diseases, but not in cancers (d) Multi-drug resistant tuberculosis

(a) Cyclophosphamide 230. The agent useful in prevention of MAC


infection in HIV infected individuals is
(b) Cyclosporine
(c) Methotrexate (a) Isoniazid (b) Pyrazinamide
(c) Rifabutin (d) Streptomycin
(d) 6-Mercaptopurine
231. Dose of rifampicin recommended in
223. The following drug(s) is/are effective in
multibacillary leprosy is 600 mg
reducing the occurrence of embolic stroke
in auricular fibrillation patients (a) Once daily
(b) Once in a week
(a) Aspirin (b) Warfarin
(c) Twice weekly
(c) Digoxin (d) Both (a) and (c)
(d) Once in a month
224. Sulfonamide used in treatment of
232. The antimalarial agent that can be used
ulcerative colitis is during pregnancy is
(a) Sulfasalazine (a) Halofantrine (b) Chloroguanide
(b) Sulfacetamide (c) Mefloquine (d) Primaquine
(c) Silver sulfadiazine
233. The anti-fungal agent amphotericin B is
(d) Mafenide also active aganist
225. The cephalosporin that is active against (a) Anaerobic bacteria
pseudomonas aeruginosa is (b) Giardia lambila
(a) Cephalexin (b) Cephalothin (c) Leishmania
(c) Cefuroxime (d) Ceftazidime (d) Rickettsiae
234. The anti amoebic agent implicated in
226. To increase its blood concentration,
causing subacute myelo-optic neuropathy
imipenem is combined with
(SMON) is
(a) Clavulanic acid (b) Sulbactam (a) Diloxanide furoate
(c) Probenecid (d) Cilastatin (b) Iodochlorohydroxyquin
227. Characteristic toxicity of ethambutol is (c) Emetine
(d) Metronidazole
(a) Hepatitis
(b) Renal damage 235. The drug of choice for tropical eosinophilia
(c) Vestibular damage is
(d) Visual defects (a) Niridazole
(b) Niclosamide
228. Pyridoxine given to a patient of tubercu- (c) Diethyl carbamazine citrate
losis prevents
(d) Tetramisole
(a) INH induced peripheral neuritis
236. The agent used to treat tapeworm infes-
(b) Rifampicin induced hepatotoxicity tations is
(c) Ethambutol induced visual defects
(a) Praziquantel (b) Diloxanide furoate
(d) Streptomycin induced nephrotoxicity
(c) Pentamidine (d) Thialbendazole
188 MCQs IN PHARMACOLOGY

237. Methotrexate (c) To broaden the spectrum of antimicrobial


activity
(a) Is useful in choriocarcinoma
(d) All of the above
(b) Is safe in patients with renal dysfunction
(c) Induced neurotoxicity is reversed by leucovorin 244. The bactericidal drugs act most effectively
(d) Is not used in children on
(a) The toxin liberated by organisms
238. Prolactin
(b) Preventing liberation of toxins from organisms
(a) Has somatotropic activity
(c) Slowly dividing organisms
(b) If low can cause infertility in women
(d) Rapidly dividing organisms
(c) Can suppress menstrual cycle in lacting
women 245. Most of the laboratory sensitivity tests for
antimicrobials are conducted at pH
(d) Levels are increase by dopamine
(a) 4.6–4.8 (b) 8.4–8.6
239. Thioamide drugs
(c) 3.2–3.6 (d) 7.2–7.4
(a) Aggravate endocrine exophthalmos
246. Sulphonamides prevent the synthesis of
(b) Are safe in children
folic acid in bacterial cells because
(c) Produce irreversible hypothyroidism
(a) They compete with PABA which is a precursor
(d) Do not produce relapse of folic acid
240. Liothyronine is the drug of choice in (b) They potentiate action of PABA which is
treatment of natural antagonist of folic acid
(a) Myxoedema coma (c) They bind to PABA and form a complex
(b) Cretinism (d) None of the above
(c) Iodine deficiency goiter 247. Sulphonamide-trimethoprim combination
(d) Non-resectable cases of papillary thyroid becomes bactericidal because
carcinoma (a) Sulphonamide inhibits conversion of folic
acid to folinic acid, and trimethoprim from
241. When does the neutrophil nadir
PABA to folic acid
associated with chemotherapy agents
generally occur? (b) Sulphonamide inhibits conversion of PABA
to folic acid and trimethoprim from folic acid
(a) During administration of the chemotherapy to folinic acid
(b) 1–2 days after therapy (c) Trimethoprim increases the free drug
(c) 10–14 days after therapy concentration of sulphonamide
(d) 1 month after therapy (d) Sulphonamide increases the free drug
(e) When the platelet count begins to rise concentration of trimethoprim

242. Which of the following are the principal 248. Sulphonamide given in late pregnancy or
organisms in superinfection? to new born can result in kernicterous
because
(a) Salmonella, shigella
(a) It stimulates synthesis of bilirubin
(b) Candida, albicans, proteus and staphylococci
(b) It displaces bilirubin from plasma protein and
(c) E. Coli, Mycobacterium tuberculosis
and results in high free bilirubin concentration
(d) None of the above
(c) It prevents the metabolism of bilirubin
243. In tuberculosis combination of antimicro- (d) It is metabolized in bilirubin
bials is used
249. Concentration of sulphonamides in C.S.F.
(a) To delay the development of drug resistance is about 40 to 80% of that in blood
(b) To reduce severity of adverse reactions because
CHEMOTHERAPY 189

(a) In C.S.F. there is no protein to bind the drug (c) Interfering with plasmodium mitochondria
(b) C.S.F. has higher protein content than the (d) By all of the above mechanisms
blood
257. Drug resistance develops
(c) C.S.F has low protein content than the blood
(a) More readily to pyrimethamine than chloroquin
(d) In C.S.F sulphonamides are quickely
destroyed (b) More readily to chloroquin than pyrimethamine
(c) In both cases resistance does not develop
250. Sulphonamides are primarily metabolized
(d) In both cases resistance develops readily
by
(a) Acetylation 258. For suppressive prophylaxis of malaria
the drug used is
(b) Glucuronidation
(c) Etheral sulphate formation (a) Pyrimethamine 25 mg. once a week
(d) None of the above (b) Pyrimethamine 250 mg. once a week
(c) Primaquine 15 mg daily for 2 days
251. In an alkaline urine sulphonamides are
(d) All of the above drug can be used
(a) Less soluble
259. For radical cure of malaria the drug of
(b) More soluble
choice is
(c) Not at all soluble
(a) Primaquine 25 mg bd. × 7 days
(d) No effect of pH on stability of sulphonamides
(b) Primaquine 15 mg once daily × 14 days
252. The risk of crystallurea by suylphonamides (c) Chloroquine 400 mg. once is a month
can be reduced by
(d) None of the above
(a) Giving more soluble sulphonamides
260. Which of the following statements
(b) Making urine alkaline
describe hemorrhagic cystitis? It
(c) Ensuring urinary volume at least 2 liters a
day (a) Is caused by excretion of tumor cell
(d) All of the above breakdown products
(b) Is associated with iforfamide or cyclophos-
253. Sulphamethizole is principally used for phamide administration
(a) Topical application (c) Is caused by the administration of mesna
(b) Systemic infection (d) Can be prevented or treated with acrolein
(c) Occular infection (e) Can be treated with G-CSF
(d) Urinary infection
261. The less common causative organism in
254. Stomatitis is characterized by all of the urinary tract infection is
following signs and symptoms except (a) E. Coli (b) Pseudomonas
(a) headache (b) erythema (c) Tubercular bacilli (d) Proteus
(c) bleeding (d) ulcerations
262. In an acute U.T.I. with alkaline urine and
(e) dryness of mouth unidentified organism, the treatment
255 In co-trimoxazole, sulphamethoxazole should begin with
and trimethoprim are in ratio of (a) Co-trimoxazole (b) Sulphonamide
(a) 2 : 1 (b) 1 : 5 (c) Nitrofurantoin (d) Mendallic acid
(c) 1 : 1 (d) 5 : 1 263. Nitrofurantion is useful only in urinary
256. Pyrimethamine and progunil act by tract infection because

(a) Interfering with parasite’s ability to digest Hb (a) It is rapidly excreted and concentrated in
urine
(b) Inhibiting dihydrofolate reductase which
converts folic acid to folinic acid (b) It is rapidly metabolized in liver which
prevents effective plasma concentration
190 MCQs IN PHARMACOLOGY

(c) It’s concentration in renal tissue is high (c) It potentiates the antitubercular effect of other
because of renal tubular reabsorption drugs
(d) It is effective against common organisms of (d) It has reduced the duration of antitubercular
U.T.I e.g. proteins, and E. Coli therapy
(e) All above reasons
270. In cell targeted suicide mode of gene
264. Hexamine mandelate is only effective in therapy for cancer the prodrug used is
U.T.I. and not in systemic infection (a) Gancyclovir (b) Acyclovir
because (c) Ara c/m (d) All of the above
(a) It is only concentrated in urine
271. Drug of choice for guinea worm infestation
(b) In urine at pH less than 5.5, it liberates active
is
antibacterials
(a) Digoxin (b) Furosemide
(c) For systemic infection dose required is too
high which is toxic (c) Enalapril (d) Amrinone
(d) Organisms in systemic infection are not 272. Which of the following can not be treated
sensitive to it with danzol ?
265. All of the following chemotherapy agents (a) Endometriosis
are vesicants except (b) Menorrhagia
(a) Doxorubicin (b) Mechlorethamine (c) Fibrocystic disease
(c) Vincristine (d) Methotrexate (d) Hirsutism
(e) Idarubicin 273. Melphalan is the drug of choice in
266. Which of the following is most effective (a) Leukemia
antitubercular drug? (b) Multiple myeloma
(a) INH (b) Thioactazone (c) Hodgkin’s disease
(c) PAS (d) Pyrazinamide (d) Osteosarcoma
267. Which of the following drugs is effective 274. Prolonged chlorpromazine therapy
against intracellular organism of tuber- results in
culosis?
(a) Osteomalacta
(a) INH (b) Streptomycin (b) Photosensitivity
(c) PAS (d) Ethambutol (c) Yellowish discolouration
268. Isoniazid may cause peripheral (d) Altered renal function
neuropathy and anaemia as its side
275. Drug of choice for actionomycosis is
effects because
(a) Penicillin (b) Tetracycline
(a) It is toxic to nerves and to R.B.C
(c) Sulfonamide (d) Dapsone
(b) It interferes with pyridoxin metabolism and
induces its defeciency 276. Match each of the following toxicities to
(c) It prevents absorption of iron and ascorbic acid the agent most likely to cause the toxicity
(d) It is a type of hypersensitivity reaction (a) Cardiotixicity (1) Vincristine
269. Rifampicin has gained significance in (b) Hypersensitivity (2) Irinotecan
antitubercular therapy because (c) Diarrhea (3) Doxorubicin
(a) It is the cheapest and effective drug (d) Pulmonary toxicity (4) Paclitaxel
(b) It is least toxic, so suitable for chronic use (e) Constipation (5) Bleomycin
CHEMOTHERAPY 191

ANSWERS
1. d 2. c 3. b 4. c 5. b 6. e
7. d 8. d 9. b 10. d 11. b 12. d
13. d 14. a 15. a 16. d 17. d 18. d
19. a 20. c 21. b 22. c 23. d 24. d
25. a 26. c 27. a 28. b 29. a 30. c
31. e 32. d 33. d 34. a 35. c 36. b
37. a 38. b 39. d 40. b 41. d 42. b
43. e 44. b 45. b 46. d 47. b 48. c
49. e 50. e 51. d 52. d 53. b 54. d
55. b 56. c 57. b 58. b 59. b 60. c
61. e 62. b 63. c 64. e 65. e 66. a
67. a 68. a 69. e 70. b 71. d 72. b
73. b 74. c 75. e 76. d 77. d 78. c
79. c 80. c 81. e 82. a 83. a 84. c
85. a 86. c 87. c 88. c 89. a 90. a
91. a 92. b 93. e 94. c 95. e 96. d
97. b 98. d 99. b 100. a 101. b 102. c
103. a 104. d 105. a 106. b 107. d 108. e
109. c 110. d 111. c 112. c 113. c 114. e
115. e 116. d 117. c 118. e 119. e 120. d
121. a 122. c 123. b 124. e 125. c 126. d
127. b 128. c 129. b 130. a 131. d 132. d
133. b 134. c 135. b 136. c 137. c 138. d
139. d 140. c 141. d 142. a 143. b 144. a
145. c 146. c 147. b 148. b 149. d 150. a
151. c 152. c 153. c 154. b 155. b 156. a
157. d 158. a 159. b 160. b 161. d 162. d
163. a 164. a 165. a 166. c 167. b 168. d
169. d 170. d 171. b 172. d 173. d 174. c
175. b 176. c 177. d 178. b 179. b 180. d
181. d 182. a 183. d 184. b 185. a 186. a
187. b 188. c 189. c 190. a 191. d 192. a
193. c 194. c 195. b 196. a 197. c 198. b
199. c 200. b 201. d 202. b 203. d 204. a
205. b 206. d 207. d 208. a 209. d 210. e
211. c 212. b 213. d 214. d 215. b 216. b
217. d 218. d 219. b 220. d 221. c 222. b
223. d 224. a 225. d 226. d 227. d 228. a
229. a 230. c 231. d 232. b 233. c 234. b
235. c 236. a 237. a 238. c 239. b 240. a
241. c 242. b 243. a 244. d 245. d 246. a
192 MCQs IN PHARMACOLOGY

247. b 248. b 249. c 250. a 251. b 252. a


253. d 254. b 255. d 256. b 257. a 258. a
259. b 260. b 261. c 262. a 263. e 264. b
265. d 266. a 267. a 268. b 269. d 270. d
271. a 272. d 273. b 274. a 275. a
276. a.3, b.4, c.2, d.5, e.1

EXPLANATIONS FOR THE ANSWERS


1. d Prostate cancer is the most common cancer in in inhibition of cell division. Vincristine, vinblastine
men, breast cancer is the most common cancer and vinorelbine are vinca alkaloids, which work
in women, followed by lung, then colon and by preventing microtubule formation.
rectum for both men and women. Mitoxantrone is an antitumor antibiotic that works
27. a Phase-specific agents are most active in one by DNA intercalation.
specific phase of the cell cycle. These agents 210. e Tamoxifen is an antiestrogen used in the
have no activity against cell in G0, the resting treatment of breast cancer. Prednisone is used
phase. Examples of phase-specific agents for its antilymphocytic. Properties in the treatment
include the mitotic inhibitors, asparaginase, the of non-Hodgkin’s lymphoma. Flutamide is an
antimetabolites and etoposides. antiandrogen used in the treatment of prostate
53. b BSA correlates with cardiac output, which cancer.
determines renal and hepatic blood flow and 241. c Bone marrow suppression, particularly of the
thus affects drug elimination. neutrophils, ususlly is the most profound 10–
77. d Combination chemotherapy has been 14 days after chemotherapy.
developed to have maximum cytotoxicity to 252. a Stomatitis, or mucositis, is an inflammation of
tumor cells and minimal toxicity to normal cells. the mucous membranes, particularly the oral
The drugs are dosed and scheduled such that mucosa. Although the symptoms generally are
maximal cell kill and occurs, while sparing limited to the mouth and throat, stomatitis may
normal cells as much as possible. Combination effect any part of the gastrointestinal tract,
regimens often contain agents with different potentially causing diarrhea and anal fissures.
spectrums of toxicity.
260. b Hemorrhagic cystitis results from irritation of the
95 e Intrathecally administered vincristine is fatal. All lining of the bladder by acrolein, a metabolite
syringes of vincristine must be labeled “Fatal if of ifosfamide and cyclophosphamide. Mesna
given intrathecally. For intravenous use only.” may be used to inactivate the acrolein, thus
119. e Cyclophosphamide and mechlorethamine are preventing hemorrhagic cystitis.
nitrogen mustards, a subgroup of the alkylating 265. d Vesicant chemotherapy agents may cause local
agents. Etoposide is a topoisomerase II inhibitor necrosis if extravasated outside the vein.
and paclitaxel is a mitotic inhibitor. Doxorubicin, idarubicin, mechlorethamine and
145. c Doxorubicin is an antitumor antibiotic that vincristine are all classified as vesicants.
inhibits DNA synthesis by intercalation. 276.a. 3, b. 4, c. 2, d. 5, e.1: Cardiotoxicity is associated
Vincristine and paclitaxel are mitotic inhibitors with cumulative doses of doxorubicin and other
that act on microtubule assembly. Topotecan antitumor antibiotics. Hypersensitivity from
inhibits topoisomerase I. paclitaxel may be due to its cremophor diluent.
168. d Antimetabolites are structural analogues of Severe diarrhea, requiring treatment with atropine,
naturally occurring substrates for biochemical is associated with irrinotecan. Pulmonary toxicity
reactions. They inhibit DNA synthesis by acting is associated with cumulative doses of bleomycin.
as false substitutions in the production of DNA. Severe constipation and paralytic ileus is
189. c Docetaxel is a taxane, which works by promoting associated with the use of vincristine.
microtubule assembly and stabilization, resulting
DRUGS USED IN ENDOCRINE DISORDERS (HORMONES) 193

Chapter 12
DRUGS USED IN ENDOCRINE
DISORDERS (HORMONES)

1. Which one of the following compounds (a) Bromocriptine


is not a hormone? (b) Desmopressin
(a) Bromocriptine (b) Somatomedin (c) Human gonadotropin hormone
(c) Somatotropin (d) Thyroxine (d) Leuprolide
(e) Vasopressin (e) Octreotide
2. Which one of the following hormones not 5. Who is least likely to be treated with
synthesized in the hypothalamus? somatropin?
(a) Corticotropin–releasing hormone (a) A 3-year-old cow on a diary farm
(b) Luteinizing hormone (b) A 4-year-old girl with an XO genetic
(c) Oxytocin genotype
(d) Thyrotropin–releasing hormone (c) A 4-year-old boy with chronic renal failure
(e) Vasopressin and growth deficiency
3. An important difference between leuprolide (d) A 10-year-old boy with polydipsia and
and the new drug ganirelix is that ganirelix polyuria
(a) Can be administered as an oral formulation (e) A 37-year-old AIDS patient who is 180 cm
tall and weights 52 Kg
(b) Can be used alone to restore fertility to
hypogonadal men and women 6. Hormones that are useful in the diagnosis
(c) Immediately reduces gonadotropin secretion of endocrine insufficiency include
(d) Initially stimulates pituitary production of LH (a) Corticotropin-releasing hormone
and FSH
(b) Cosyntropin
(e) Must be administered in a pulsatile fashion
(c) Gonadotropin–releasing hormone
4. A 27-year-old woman with amenorrhea, (d) Thyrotropin–releasing hormone
infertility, and galactorrhea was treated (e) All of the above
with a drug that successful restored
ovulation and menstruation. Before being 7. All of the following substances are
given the drug, the woman was carefully endogenous topic hormones secreted by
questioned about previous mental health the pituitary gland except
problems, which she did not have. She was
(a) Somatotropin
advised to take the drug orally. The drug
used to treat this patient was probably. (b) Human chorionic gonadotropin (HCG)
194 MCQs IN PHARMACOLOGY

(c) Follicle-stimulating hormone (FSH) an adrenal tumor. Which of the following


(d) Thyroid-stimulating hormone (TSH) drugs would be expected to reduce the
(e) Adrenocorticotropic hormone (ACTH) signs and symptoms of this man’s
disease?
8. Which of the following drugs is least likely
(a) Betamethasone (b) Cortisol
to be used as part of a controlled ovarian
(c) Fludrocortisone (d) Ketoconazole
hyperstimulation protocol?
(e) Triamcinolone
(a) Human chorionic gonadotropin
(b) Leuprolide 14. In the treatment of congenital adrenal
(c) Menotropins hyperplasis in which there is excess
production of cortisol precursors due to
(d) Pergolide
a lack of 21ββ-hydroxylase activity, the
(e) Urofollitropin purpose of administration of a synthetic
9. Actions of thyroxine do not include glucocorticoid is
(a) Acceleration of cardiac rate (a) Inhibition of aldosterone synthesis
(b) Decreased glomerular filtration rate (b) Normalization of renal function
(c) Fine tremor of skeletal muscles (c) Prevention of hypoglycemia
(d) Increased appetite (d) Recovery of normal immune function
(e) Stimulation of oxygen consumption (e) Suppression of ACTH secretion

10. Effects of iodide salts given in large doses 15. Glucocorticoids have not been proved to
do not include be effective in the treatment of

(a) Decreased size of the thyroid gland (a) Acute lymphocytic leukemia
(b) Decreased vascularity of the thyroid gland (b) Addison’s disease
(c) Decreased hormone release (c) Asthma
(d) Decreased iodination of tyrosine (d) Chemotherapy-inhduced vomiting
(e) Increased 131I uptake (e) Osteoporosis

11. Synptoms of hypothyroidism (myxedema) 16. For patients who have been on long-term
do not include therapy with a glucocorticoid and who
now wish to discontinue the drug,
(a) Dry, puffy skin gradual tapering of the glucocorticoid is
(b) Increased appetite needed to allow recovery of
(c) Large tongue and drooping of the eyelids
(a) Depressed release of insulin from pancreatic
(d) Lethargy, sleepiness B cells
(e) Slow heart rate (b) Hematopoiesis in the bone marrow
12. When initiating thyroxine therapy for an (c) Normal osteoblast function
elderly patient with long-standing (d) The control by vasopressin of water excretion
hypothyroidism, it is important to begin (e) The hypothalamic-pituitary-adrenal system
with small doses to avoid
17. A 24-year-old woman with type 1 diabetes
(a) A flare of exophthalmos wishes to try tight control of her diabetes to
(b) Acute renal failure improve her long-term prognosis. Which of
(c) Hemolysis the following regimens is most appropriate?
(d) Overstimulation of the heart (a) Morning injections of mixed lente and
(e) Seizures ultralente insulins
13. A 46-year-old male patient has Cushing’s (b) Evening injections of mixed regular and lente
syndrome that is due to the presence of insulin
DRUGS USED IN ENDOCRINE DISORDERS (HORMONES) 195

(c) Morning and evening injections of regular (b) Has a longer half-life
insulin, supplemented by small amounts of (c) Has higher affinity for thyroid hormone
insulin at mealtimes receptors
(d) Morning injections of ultralente insulin, (d) Is faster acting
supplemented by small amounts of insulin (e) Is more likely to improve a patient’s mood
lispro at mealtimes
(e) Morning injection of semilente insulin and 23. A young woman seeks advice because she
evening injection of lente insulin had unprotected sexual intercourse 12
hours earlier. Based on her menstrual cycle,
18. Which of the following substances when she believes that conception is possible.
present in urine is the most likely positive Which of the following drugs should she
sign of pregnancy? use as a postcoital contraceptive?
(a) Thyroid-stimulating hormone (TSH) (a) Clompihene
(b) Corticotropin (b) Diethylstilbestrol plus raloxifene
(c) Human chronic gonadotropin (HCG) (c) Ethinyl estradiol combined with norethindrone
(d) Interstitial cell-stimulating hormone (ICSH) (d) Flutamide
(e) Protamine zinc insulin (PZI) (e) Letrozole plus finasteride
19. A 54-year-old obese patient with type 2 24. All of the following hormonal drugs
diabetes and a history of alcoholism possess a steroidal nucleus except
probably should not receive metformin
because it can increase his risk of (a) Ethinyl estradiol
(b) Nonethindrone
(a) A disulfiram-like reaction
(c) Liothyronine
(b) Excessive weight gain
(d) Prednisolone
(c) Hypoglycemia
(e) Fluoxymesterone
(d) Lactic acidosis
(e) Serious hepatotoxicity 25. Which one of the following statements
about hormone replacement therapy (HRT)
20. Which of the following drugs is taken regimens in menopause is accurate?
during the first part of a meal for the
purpose of delaying the absorption of (a) It commonly includes a progestin to reduce
dietary carbohydrates? the risk of endometrial cancer
(b) It has been shown in clinical trials to reduce
(a) Acarbose (b) Colestipol
migraine attacks
(c) Glipizide (d) Pioglitazone
(c) It includes steroids that induce cytochrome
(e) Repaglinide
P450
21. Which of the following drugs is most (d) It should be avoided in women with a history
likely to cause hypoglycemia when used of diabetes
as monotherapy in the treatment of type (e) It uses the same effective doses of steroids
2 diabetes? as those combined oral contraceptives
(a) Acarbose (b) Glyburide
26. Hypercoagulability and dermal vascular
(c) Metformin (d) Miglitol necrosis due to protein C deficiency is known
(e) Rosiglitazone to be an early–appearing adverse effect of
treatment with
22. In the treatment of hypothyroidism,
thyroxine is preferred over liothyronine (a) Aspirin (b) Clopidogrel
because thyroxine (c) Heparin (d) Streptokinase
(a) Can be made more easily by recombinant (e) Warfarin
DNA technology
196 MCQs IN PHARMACOLOGY

27. A 42-year-old woman requires treatment (c) Protamine zinc insulin (PZi)
for diabetes insipidus following surgical (d) Semilente insulin
removal of part of her pituitary gland. (e) Ultralente insulin
The advantage of treating this patient
with desmopressin instead of vasopressin 33. Which of the following classes of
is that desmopressin compounds stimulates the release of
(a) Causes less formation of factor VIII insulin from pancreatic β -cells?
(b) Causes less hypernatremia (a) Progestins
(c) Causes less hyperprolactinemia (b) Biguanides
(d) Is more selective for the V2 receptor subtype (c) α-Glucosidase inhibitors
(e) Provides greater relief of the excessive thirst (d) Thiourylenes
the patient experiencing (e) Sulfonylureas
28. All of the following substances are 34. Insulin preparations that contain a
endogenous tropic hormones secreted by modifying protein include
the pituitary gland except
(a) Lente insulin
(a) Somatotropin (b) Regular insulin
(b) Human Chorionic Gonadotropin (HCG) (c) Isophane insulin (NPH)
(c) Follicle-timulating hormone (FSH)
(d) Thyroid-stimulating hormone (TSH) 35. Metabolic reactions likely to be affected
by a protein–deficient diet include
(e) Corticotropin (ACTH)
(a) Glycine conjugation
29. Which of the following substances when
(b) Hydrolysis
present in urine is the most likely positive
sign of pregnancy? (c) Glucuronidation

(a) Thyroid–stimulating hormone (TSH) 36. What is the correct formula to use for
(b) Corticotropin (ACTH) calculating the free thyroxine index (FTI)?
(c) Human chorionic gonadotropin (HCG) (a) T4 × RT3 U/mean serum RT3 U
(d) Interstital cell-stimulating hormone (ICSH) (b) T3 × T3/mean serum RT3U
(e) Protamine zinc insulin (PZl) (c) T3 × RT3U/mean serum RT3U
(d) T4 × RT3U × mean serum RT3U
30. Which of the following glucocorticoids
produces the least sodium retention? (e) T3 × RT3U × mean serum RT3U
(a) Corticosone 37. Which of the following insulins can be
(b) Hydrocorticosone administered intravenously?
(c) Prednisolone (a) Regular insulin
(d) Dextramethasone (b) Isophane insulin (NPH)
(e) Fludrocortisone (c) Protamine zinc insulin (PZI)
31. Which of the following glucocorticoids (d) Semilente insulin
produces the least sodium retention? (e) Ultralente insulin
(a) Cortisone (b) Hydrocortisone 38. All of the following conditions are causes
(c) Prednisolone (d) Dexamethasone of hyperthyroidism except
(e) Fludrocortisone (a) Graves’ disease
32. Which of the following insulins can be (b) Hashimoto’s thyroiditis
administered intravenously? (c) Toxic multinodular goiter
(a) Regular insulin (d) Triiodothyronine tocicosis
(b) Isophane insulin (NPH) (e) Plummer’s disease
DRUGS USED IN ENDOCRINE DISORDERS (HORMONES) 197

39. Which of the following preparations is (d) Reverse triiodothyronine (rT3)


used to attain remission of thyrotoxicosis? (e) Total thyroxine
(a) Propranolol (b) Liotrix 45. Which of the following agents has been
(c) Levothyroxine (d) Propylthiouracil shown to interact with oral thyroxine (T4)
(e) Desiccated thyroid replacement therapy?
(a) Propylthiouracil (b) Cholestyramine
40. The thyroid gland normally secretes
which of the following substances into the (c) Thyrotropin (d) Levothyroxine
serum? (e) Lovastatin

(a) Thyrotropin-releasing hormone (TRH) 46. What laboratory tests are currently
(b) Thyrotropin (thyroid-stimulating hormone) recommended by the Thyroid association
to diagnose thyroid disease?
(c) Diiodothyronine (DIT)
(a) Resin triiodothyronine uptake (RT3U) and total
(d) Thyroglobulin
thyroxine (TT4)
(e) Thyroxine (T4)
(b) Thyrotropin (TSH)and free thyroxine index
41. All of the following conditions are causes (FTI)
of hypothyroidism except (c) Total thyroxine (TT4)and sensitive TSG assay
(a) Endemic goiter (d) Free T4 and sensitive TSH assay
(b) Surgical excision (e) Free T4 and RT3U
(c) Hashimoto’s thyroiditis 47. What patient population should be
(d) Goitrin-induced iodine deficiency screened for thyroid disease?
(e) Graves’s disease (a) Hospitalized patients
(b) Elderly patients with chronic disease
42. Common tests to monitor patients receiving
(c) Elderly hospitalized patients
replacement therapy for hypothyroidism
include all of the following except (d) College students
(e) Women over 20 years old
(a) Thyrotropin (TSH)stimulation test
(b) Sensitive TSH assay 48. What is the average replacement dose
of levothyroxine for an otherwise healthy
(c) Free thyroxine index (FTI)
adult?
(d) Resin triiodothyronine uptake (RT3U)
(a) 25–50 µg/day
(e) Total thyroxine (TT4)
(b) 50–100 µg/day
43. Which of the following pairs of prepara- (c) 75–150 µg/day
tions has been studies for bioequiva- (d) 100–200 µg/day
lence? (e) 200–400 µg/day
(a) Levoxyl–Thyrolar
49. In comparing levothyroxine to liothyro-
(b) Thyroglobulin - Proloid nine, which of the following statements
(c) Levothroid - Synthroid is not correct?
(d) Cytomel - Synthroid (a) Both levothyroxine and liothyronine are
(e) Desiccated thyroid–Armour thyroid naturally occurring thyroid hormones
(b) Liothyronine can be converted in the
44. The inhibition of pituitary thyrotropin
peripheral circulation to levothyroxine.
secretion iscontrolled by which of the
(c) Liothyronine is more potent than levothyroxine
following?
(d) The plasma concentration of liothyronine is
(a) Free thyroxine (T4) less than that of levothyroxine.
(b) Thyroid–releasing hormone (TRH) (e) Liothyronine has a shorter duration of action
(c) Free thyroxine index (FTI) than levothyroxine.
198 MCQs IN PHARMACOLOGY

50. Which of the values represents the lower (a) Muscle dystonia caused by triflupromazine
level of detection for the fourth generation (b) Insomnia after taking pentobarbitone
sensitive TSH assay as established by the (c) Precipitation of asthma by morphine
Thyroid Association?
(d) Gum hyperplasia caused by phenytoin
(a) 0.5–5 mlU/L
56. An immunologically mediated reaction to
(b) 1–2 mlU/L
a drug producing stereotyped symptoms
(c) 0.01–0.02 mlU/L unrelated to its pharmacodynamic actions
(d) 0.0001–0.002 mlU/L is
(e) 0.0001–0.0002 mlU/L (a) Hypersensitivity (b) Supersensitivity
51. In which of the following clinical (c) Intolerance (d) Idiosyncrasy
presentations should the sensitive TSH 57. Drugs producing allergic reactions
assay be used? generally act as
(a) Population screening for thyroid disease (a) Complete antigens
(b) Screening hospitalized patients (b) Haptenes
(c) Patients receiving thyroid replacement after (c) Antibodies
6 to 8 weeks of therapy (d) Mediators
(d) Patients who are human immunodeficiency
virus (HIV) positive 58. Which of the following allergic drug reaction
is caused by circulating antibodies ?
(e) Screening patients with psychiatric illness
(a) Serum sickness
52. Which of the following classes of com-
(b) Anaphylactic shock
pounds stimulates the release of insulin
(c) Systemic lupus erythematosus
from pancreatic β-cells?
(d) Angioedema
(a) Progestins
(b) Biguanides 59. The essential feature in drug addiction is
(c) α-Glucosides inhibitors (a) Physical dependence
(d) Thiourylenes (b) Psychological dependence
(e) Sulfonylureas (c) Both (a) and (b)
(d) Psychiatric abnormality
53. Which of the following is not an estrogenic
substance? 60. Adaptive neurophysiological changes
(a) Estradiol produced by repeated administration of
a drug, which result in the appearance
(b) Premarin
of characteristic withdrawal syndrome on
(c) Theelin discontinuation of the drug is called
(d) Follicle-stimulating hormone (a) Drug addiction
(e) Stilbesterol (b) Drug abuse
54. Propylthiouracil is useful in the treatment (c) Psychological dependence
of (d) Physical dependence
(a) Derangement toxicosis 61. What constitutes ‘drug abuse’?
(b) Hypothyroidism
(a) Physician prescribed use of penicillin G for
(c) Hypoparathyroidism the cure of viral fever
(d) Hyperthyroidism (b) Self administration of aspirin to relieve headache
(e) Thyroidits (c) Repeated self administration of morphine to
derive euphoria
55. Which of the following is an idiosyncratic
adverse drug reaction ? (d) All of the above
DRUGS USED IN ENDOCRINE DISORDERS (HORMONES) 199

62. Under physiological conditions the rate (a) Flutamide: Competitively blocks the binding
limiting enzyme in the generation of of androgens to their receptor
angiotensin II is (b) Finasteride: Inhibits 5α-reductase
(a) Renin (c) Miglitol: Inhibits α-glucosidase
(b) Angiotensin converting enzyme (d) Pioglitazone: Competitively blocks the
(c) Aminopeptidase binding of estrogens to their receptor
(d) Angiotensinase (e) Anastrazole: Inhibits aromatase
63. Angiotensin II causes rise in blood pressure 70. The most prominent action of bromocriptine
by is
(a) Direct vasoconstriction (a) Dopamine D2 agonism
(b) Releasing adrenaline from adrenal medulla (b) Dopamine D2 antagonism
(c) Increasing central sympathetic tone (c) Dopamine D1 antagonism
(d) All of the above (d) α adrenergic antagonism
64. Which of the following is a pressor peptide 71. Gynaecomastia can be treated with
that can be generated both in circulation
(a) Chlorpromazine
as well as locally in certain tissues ?
(b) Cimetidine
(a) Bradykinin (b) Angiotensin
(c) Bromocriptine
(c) Kallidin (d) Plasmin
(d) Metoclopramide
65. Angiotensin II palys a key role in the
72. Menotropins is a preparation of
following risk factor for ischaemic heart
disease (a) FSH + LH obtained from urine of menstruating
women
(a) Hypercholesterolemia
(b) LH obtained from urine of pregnant women
(b) Ventricular hypertrophy
(c) FSH + LH obtained from urine of menopausal
(c) Carbohydrate intolerance
women
(d) Cardiac arrhythmia
(d) LH obtained from serum of pregnant mare
66. Several actions of growth hormone are
exerted through the elaboration of 73. Serum TSH levels are high in most cases
of
(a) Cyclic AMP
(a) Myxoedema
(b) Cyclic GMP
(b) Grave’s disease
(c) Somatostatin
(c) Carcinoma thyroid
(d) Insulin like growth factor – 1
(d) Toxic nodular goiter
67. Somatostatin inhibits the release of
74. Adrenocorticotropic hormone is primarily
(a) Growth hormone (b) Insulin used for
(c) Thyrotropin (d) All of the above
(a) Treatment of Addison’s disease
68. For therapeutic use, growth hormone is (b) Treatment of congenital adrenal hyperplasia
obtained from (c) Treatment of autoimmune disease
(a) Recombinant DNA technique (d) Diagnosis of pituitary-adrenal axis disorders
(b) Human cadaver pituitaries
75. Which of the following compounds is
(c) Porcine pituitaries
incorrectly matched with one of its
(d) Chemical synthesis therapeutic uses?
69. Which of the following compounds is (a) Raloxifene: Advanced breast cancer
incorrectly matched with its mechanism (b) Metformin: Non-insulin dependent diabetes
of action? mellitus (NIDDM)
200 MCQs IN PHARMACOLOGY

(c) Finasteride : Benign prostatic hyperplasia I – resume carbimazole after 1 week for 2 –
(d) Propylthiouracil: Hyperthyroidism 3 months
(e) Tamoxifen : Estrogen-dependent breast (d) Propranolol + Lugol’s iodine for 2 weeks –
cancer
131
I – continue Lugol’s iodine for 2 – 3 months

76. Metabolic rate of the following organ is 82. The thyroid inhibitor which produces the
not significantly affected by thyroxine fastest response is

(a) Brain (b) Heart (a) Radioactive iodine


(c) Liver (d) Skeletal muscle (b) Lugol’s iodine
(c) Propylthiouracil
77. Actions of thyroxine include the following
(d) Lithium carbonate
except
(a) Rise in blood sugar level 83. In the treatment of hyperthyroidism,
carbimazole has the following advantage
(b) Reduction in plasma cholesterol level
over radioactive iodine.
(c) Fall in plasma free fatty acid level
(a) cost of treatment is lower
(d) Induction of negative nitrogen balance
(b) It is preferable in uncooperative patient
78. Triiodothyronine is preferred over (c) It is better tolerated by the patients
thyroxine in the treatment of (d) Hypothyrodism when in luced is reversible
(a) Endemic goiter
84. Glucose entry into the cells of the following
(b) Cretinism organ/ tissue is highly dependent on the
(c) Papillary carcinoma of thyroid presence of insulin.
(d) Myxoedema coma (a) Brain (b) Liver
79. Carbimazole acts by inhibiting (c) Adipose tissue (d) Kidney tubules
(a) Iodide trapping 85. Prolonged testosterone therapy can cause
(b) Oxidation of iodide (a) Hypertrophy of seminiferous tubules of testes
(c) Proteolysis of thyroglobulin (b) Hypertrophy of interstitial cells of testes
(d) Synthesis of thyroglobulin protein (c) Atrophy of interstitial cells of testes
80. Antithyroid drugs exert the following (d) Both (a) and (b)
action. 86. Testosterone therapy started in a boy of
(a) Block the action of thyroxine on peripheral 8 years and continued till puberty is likely
tissues to
(b) Block the action of thyroxine on pituitary (a) Increase adult stature
(c) Block the action of TSH on thyroid (b) Reduce adult stature
(d) Inhibit thyroxine synthesis (c) Have no effect on adult stature
(d) Cause hypertrophy of penis
81. A 60-year-old male presents with severe
hyperthyroidism and multinodular goiter. 87. Which of the following is true of anabolic
It was decided to treat him with 131I. The steroids ?
most appropriate course of treatment (a) They are testosterone congeners having
would be anabolic but no androgenic activity
(a) Immediate 131 I dosing with no other drug (b) They are androgens with relatively selective
before or after anabolic activity
(b) Propranolol for 1 week followed by 131 I (c) They are suitable for long-term therapy in
(c) Propranolol + carbimazole till severe children
thyrotoxicosis controlled – 1 week gap – 131 (d) Both (b) and (c)
DRUGS USED IN ENDOCRINE DISORDERS (HORMONES) 201

88. Which of the following drugs has potent (c) Osteoporosis


antiandrogenic and weak progestational (d) Atrophic vaginitis
activity ?
94. Addition of a progestin for 10-12 days
(a) Digoxin (b) Furosemide each month to estrogen replacement
(c) Enalapril (d) Amrinone therapy in menopausal women is
recommended because the progestin
89. Which of the following is a non-steroidal
antiandrogen that has been found to be (a) Blocks the increased risk of myocardial
palliative in advanced carcinoma prostate ? infarction due to estrogen
(b) Blocks the increased risk of endometrial
(a) Cyproterone acetate
carcinoma due to estrogen
(b) Danazol
(c) Reverses vulval atrophy occurring in
(c) Finasteride postmenopausal women
(d) Flutamide (d) Enhances the metabolic benefits of estrogen
treatment
90. Hormones that form lipophilic esters
without prior structural modifications 95. The preferred estrogen for hormone re-
include placement therapy in menopausal wom-
I. Hydrocortisone en is
II. Testosterone (a) Ethinylestradiol
III. Progesterone (b) Estradiol benzoate
(a) I only is correct (c) Diethylstilbestrol
(b) III only is correct (d) Conjugated estrogens
(c) I and II are correct 96. Select the compound which used for
(d) II and III are correct hormone replacement therapy in
postmenopausal women serves the
(e) I, II and III are correct
purpose of both estrogen and progestin
91. Finasteride acts by with weak androgenic activity.
(a) Blocking testosterone receptors in the prostate (a) Digoxin (b) Furosemide
gland (c) Enalapril (d) Amrinone
(b) Reducing testosterone secretion from testes
97. Estrogen replacement therapy fro post-
(c) Reducing LH secretion from pituitary menopausal women is contraindicated in
(d) Reducing circulating as well as prostatic subjects with
dihydrotestosterone concentration (a) Leg vein thrombosis
92. Transdermal estradiol differs from oral (b) Undiagnosed vaginal bleeding
estrogen therapy in that it (c) Migraine
(a) FSH + LH obtained from urine of menstruating (d) All of the above
women 98. Clomiphene citrate is indicated for the
(b) LH obtained from urine of pregnant women following condition/conditions
(c) FSH + LH obtained from urine of menopausal (a) Female infertility due to anovular cycles
women (b) Male infertility due to oligozoospermia
(d) LH obtained from serum of pregnant mare (c) Endometriosis
93. In which of the following conditions (d) Both (a) and (b)
estrogen is not the primary drug but is 99. Which of the following is an orally active
added to progestin as adjuvant ? ovulation inducing agent ?
(a) Dysfunctional uterine bleeding (a) Menotropin (b) Mifepristone
(b) Menopausal syndrome (c) Danazol (d) Clomiphene citrate
202 MCQs IN PHARMACOLOGY

100. Insulin preparations that contain a mod- (c) Danazol


ifying protein include (d) Medroxyprogesterone acetate
I. Lente insulin
107. Which hormone is secreted by pancreatic
II. Regular insulin β cells to facilitate glucose and amino acid
III. Isophane insulin (NPH) transport for normal cellular metabolic
(a) I only is correct processes ?
(b) III only is correct (a) Testosterone (b) Insulin
(c) I and II are correct (c) Corticotropin (d) Estradiol
(d) II and III are correct (e) Vasopressin
(e) I, II and III are correct
108. The purpose/purposes served by the
101. Progesterone administration progestin component of the combined
(a) Suppresses onset of menstruation estrogen + progestin contraceptive pill is/
(b) Induces watery cervical secretion are
(c) Sensitizes the uterus to oxygocin (a) Suppression of ovulation
(d) Cornifies vaginal epithelium (b) Prompt bleeding at the end of the course
102. Select the indication for which a progestin (c) Blockade of increased risk of endometrial
is used alone without combining with an carcinoma
estrogen. (d) All of the above
(a) Threatened abortion 109. In which of the following forms of oral
(b) Dysfunctional uterine bleeding contraception, pills are taken continuously
(c) Hormone replacement therapy without interruption ?
(d) Premenstrual tension (a) Combined pill (b) Sequential pill
103. Mifepristone possesses the following (c) Minipill (d) Phased pill
activities 110. A progestin and an estrogen are combined
(a) Potent antiprogestin + weak androgenic in oral contraceptive pill because
(b) Potent antiprogestin + weak antiglucocorticoid (a) The estrogen blocks the side effects of the
(c) Potent antiestrogenic + weak antiprogestin progestin
(d) Potent antiestrogenic + weak glucocorticoid (b) The progestin blocks the side effects of the
estrogen
104. Which of the following drugs is an anti-
progestin (c) Both synergise to suppress ovulation
(d) Both synergise to produce hostile cervical
(a) Gemeprost (b) Megestrol
mucus
(c) Mifepristone (d) Tamoxifen
111. The most common and important under-
105. The most important indication of mifepris-
sirable effect of injectable contraceptive
tone is
depot medroxyprogesterone acetate is
(a) Endometriosis
(a) Nausea and vomiting
(b) Cushings syndrome
(b) Disruption of cyclic menstraual bleeding
(c) First term abortion
(c) Venous thrombosis
(d) Second term abortion
(d) Hypertension
106. Which of the following can act as a single
dose postcoital contraceptive ? 112. The primary mechanism of action of the
combined estrogen-progestin oral contra-
(a) Clomiphene citrate
ceptive pill is
(b) Mifepristone
DRUGS USED IN ENDOCRINE DISORDERS (HORMONES) 203

(a) Production of cervical mucus hostile to sperm (a) Cyproterone acetate


penetration (b) Goserelin
(b) Suppression of FSH and LH release (c) Centchroman
(c) Making endometrium unsuitable for (d) Gossypol
implantation
119. Which of the following tissues is most sen-
(d) Enhancing uterine contractions to dislodge
sitive to oxytocin.
the fertilized ovum
(a) Myometrium
113. Which of the following is advised when (b) Myoepithelium of mammary alvioli
a woman on combined oral contraceptive
(c) Vascular smooth muscle
pill misses a dose
(d) Renal collecting ducts
(a) Continue with the course without regard to
the missed dose 120. Which class of drug closely relates to :
(b) Take 2 pills the next day and continue with “Peptic ulceration and gastrointestinal
the course hemorrhage; hyperglycemia, hyperten-
sion, and edema; “buffalo hump” and
(c) Take 2 pills every day for the remaining part “moon face”; psychological disturbanc-
of the course es; and increased susceptibility to infec-
(d) Discontinue the course and use alternative tion ?
method of contraception
(a) Antithyroid agents
114. Which side effect effect of the oral (b) Sulfonylurea oral hypoglyccemics
contraceptive subsides after 3-4 cycles of (c) Adrenocorticosteroids
continued use (d) Progestins
(a) Glucose intolerance (e) Androgens
(b) Rise in blood pressure
121. Oxytocin is preferred over ergometrine
(c) Headache for augmenting labour because
(d) Fluid retention
(a) It has brief and titratable action
115. Concurrent use of the following drug is (b) It is less likely to cause foetal anoxia
likely to cause failure of oral contraception (c) It is less likely to impede foetal descent
(a) Isoniazid (b) Rifampicin (d) All of the above
(c) Cimetidine (d) Propranolol
122. The drug of choice for controlling post-
116. On stoppage of the combined estrogen- partum haemorrhage is
progestin contraceptive pill, fertility (a) Oxytocin
returns after (b) Methylergometrine
(a) 1–2 months (b) 4–6 months (c) Dihydroergotamine
(c) 6–12 months (d) Uncertain period (d) Prostaglandin E2
117. Which hormone promotes the resorption 123. Ergometrine stops post partum haemor-
of water at the renal distal convoluted rhage by
tubule (a) Causing vasoconstriction of uterine arteries
(a) Testosterone (b) Insulin (b) Increasing tone of uterine muscle
(c) Corticotropin (d) Estradiol (c) Promoting coagulation
(e) Vasopressin (d) Inducing platelet aggregation
118. Which of the following has been found 124. Bone resorption is accelerated by
to act as a male contraceptive without (a) Estrogens (b) Parathormone
affecting libido or potency
(c) Bisphosphonates (d) Calcitonin
204 MCQs IN PHARMACOLOGY

125. The primary action of parathormone is (b) Breast cells


(a) To increase intestinal calcium absorption (c) Placenta
(b) To increase calcium reabsorption in kidney (d) Gonadotropes
tubules
133. Progesterone
(c) To promote calcium deposition in extraosseus
(a) Increases muscular contractility of fallopian
tissues
tubes
(d) To increase resorption of calcium from bone
(b) Decreases the frequency of LH pulses
126. The drug of choice for hypoparathyroid- (c) Increases the myometrial contractions
ism is (d) Increases the thickness of the endometrium
(a) Parathormone (b) Calcium lactate
134. Feminization is not a side effect of
(c) Vitamin D (d) Pamidronate
(a) Mesterolone
127. The most suitable Vitamin D preparation (b) Testosterone cypionate
for vitamin D dependent rickets is
(c) Testosterone
(a) Calciferol (b) Cholecalciferol (d) Testosterone propionate
(c) Calcifediol (d) Calcitriol
135. Anabolic steroids are contraindicated in
128. The vitamin that is regarded to be a hor-
(a) Prostatic carcinoma
mone is
(b) Carcinoma of the breast in females
(a) Vitamin D (b) Vitamin C
(c) Promoting growth in hypogonadal children
(c) Vitamin B12 (d) Vitamin A
(d) Refractory anaemias associated with
129. Which of the following drugs can cause hypoplastic bone marrow
rickets in children by interfering with
136. Potassium iodide
Vitamin D action ?
(a) Increases bronchial secretions
(a) Tetracycline (b) Phenylbutazone
(b) On prolonged use may result in hyper-
(c) Phenytoin (d) Ciprofloxacin
thyroidism
130. Estrogens (c) Is useful in inflammation of the bronchi
(a) Block bone resorption (d) Is contraindicated in thyroid storm
(b) Maintain negative calcium balance 137. Which of the following is physiological
(c) Decrease HDL levels role of vasopressin ?
(d) Increase bile acid secretion (a) Smooth muscle vasoconstriction
131. Which class of drug closely relates to: (b) Neurotransmission/neuromodulation
“Agranulocytosis and other blood (c) Increased factor VIII concentration
dyscrasias, cholestatic jaundice, nausea (d) All of the above
and vomiting, hypoglycemia and photo-
sensitivity” ? 138. Angiotensin increases peripheral resistance
by
(a) Antithyroid agents
(b) Sulfonylurea oral hypoglyccemics (a) Direct vasoconstriction
(c) Adrenocorticosteroids (b) Catecholamine release from adrenal medulla
(d) Progestins (c) Noradrenaline release from sympathetic
nerve terminals
(e) Androgens
(d) All of the above
132. The principle source of circulating
estrogen in menstructing women is 139. The drug of choice for hypoparathyroid-
ism is
(a) Granulosa cells
DRUGS USED IN ENDOCRINE DISORDERS (HORMONES) 205

(a) Parathormone (b) Pamidronate (a) Antithyroid agents


(c) Calcium (d) Vitamin D (b) Sulfonylurea oral hypoglyccemics
(c) Adrenocorticosteroids
140. Which class of drug closely relates to:
“Hepatotoxicity and jaundice, urinary (d) Progestins
retention and azoospermia, prostatic (e) Androgens
hypertrophy and priapism and paradox-
ical gynecomastia.”

ANSWERS
1. a 2. b 3. c 4. a 5. d 6. e
7. b 8. d 9. b 10. e 11. b 12. d
13. d 14. e 15. e 16. e 17. d 18. c
19. d 20. a 21. b 22. b 23. c 24. c
25. a 26. e 27. d 28. b 29. c 30. d
31. d 32. a 33. e 34. c 35. a 36. a
37. a 38. b 39. d 40. e 41. e 42. a
43. c 44. a 45. b 46. d 47. e 48. c
49. b 50. d 51. c 52. e 53. d 54. d
55. b 56. a 57. b 58. a 59. b 60. d
61. c 62. a 63. d 64. b 65. b 66. d
67. d 68. a 69. d 70. a 71. c 72. c
73. a 74. d 75. a 76. a 77. c 78. d
79. b 80. d 81. c 82. b 83. d 84. c
85. c 86. b 87. b 88. c 89.d 90. c
91. d 92. a 93. a 94. b 95. d 96. a
97. d 98. d 99. d 100. b 101. a 102. a
103. b 104. c 105. c 106. b 107. b 108. d
109. c 110. c 111. b 112. b 113. b 114. c
115. b 116. a 117. a 118. d 119. b 120. c
121. d 122. b 123. b 124. b 125. d 126. c
127. d 128. a 129. c 130. a 131. b 132. a
133. b 134. a 135. a 136. a 137. d 138. d
139. d 140. e

EXPLANATIONS FOR THE ANSWERS


7. b Human chorinic gonadotropin (hCG) is 18. c Human chorinic gonadotropin (hCG) is a
produced by placental tissue and serves to proteinaceous tropic hormone that is secreted
stimulate the secretion of progesterone during by chorinic (e. g., placental) tissue. Thus, hCG
pregnancy. Growth hormone (somatotropin), is present in the urine only after conception
follicle-stimulating hormone (FSH), thyroid- has occurred.
stimulating hormone (TSH) and corticotropin 24. c Liothyronine is a thyroid hormone. Thyroid
(ACTH) are all secreted by the anteriour pituitary hormones consist of iodinated aromatic amino
gland. acids and are not steroidal in nature. Ethinyl
206 MCQs IN PHARMACOLOGY

estradiol is a steroidal estrogen, norethindrone 75. a Raloxifene is a selective estrogen receptor


is a steroidal 19-norprogestin, prednisolone is modulator (SERM) and is not used to treat breast
an adrenocorticosteroid and fluoxymesterone cancer. Instead, because of its ability to reduce
is a steroidal adrogen. bone resorption and decrease bone turnover, it
30. d Glucocorticoids have varying degrees of is used for the prevention of osteoporosis. All of
mineralocorticoid activity. This mineralocorticoid the other four compounds are correctly matched
activity, which can result in sodium and fluid to their therapeutic use. Finasteride is also used
retention, can be blocked by the introduction of a to treat androgenic alopecia.
methyl or hydroxyl group in position 16 of the 90. c Hydrocortisone has a 21-hydroxyl group, and
steroidal nucleus. Dexamethasone has a 16 β-methyl testosterone has a 17-hydroxyl group; therefore,
substituent. both of these agents can form esters (e.g.,
37. a Most insulin preparation are suspensions; thus, hydrocor ticosone acetate, testosterone
they contain particulate matter. Only clear propionate). Progesterone does not have any
solutions may be administered intravenously. alcohol groups in its molecule; therefore, it
Regular insulin, which consists of water-soluble cannot directly form any esters.
crystalline zinc insulin is therefore suitable for 100. b Regular insulin, which is a rapid-acting insulin
intravenous administration. Insulin preparations preparation, contains only zinc insulin crystals.
are normally injected subcutaneously. All lente insulins are free of modifying proteins,
49. b The thyroid gland produces both levothyroxine (T4) which contributes to their hypoallergenic
and liothyronine (T3). The natural ratio of these properties. Isophane insulin is NPH insulin,
compounds is 4 to 1 in favor of levothyroxine; which contains protamina, a strongly basic
therefore, liothyronine is normally present at a lower protein. The protamine reduces the water
concentration thatn levothyroxine. Liothyronine is solubility of zinc insulin and lengthens its
more potent than levothyroxine, but has a shorter duration of action. Isophane insulin is classified
duration of action. Peripheral conversion involves as an intermediate-acting insulin preparation,
deiodination, thus levothyroxine is converted to having a duration of action of about 24 hours.
liothyronine. The reverse process is not possible. Common explanation for 107. b, 117. a:
52. e Of the five classes of compounds listed, only Insulin is required for the proper utilization of
biguanides, α-glucosidase inhibitors, and glucose and the transport of glucose and amino
sulfonylureas are used in the treatment of non-insulin- acids across cell membranes. Testosterone,
dependent mellitus (NIDDM). These classes provide which is produced principally from the Leydig
their beneficial effects through different mechanism cells of the testes, is responsible for male sexual
of action. Biguanides enhance the peripheral use characteristics. Vasopressin is secreted from the
of insulin, suppress gluconeogenesis, and are often posterior pituitary and is sometimes referred to
referred to as antiheperglycemic agents. α- as an antidiuratic hormone.
Glucosidase inhibitors decrease the absorption of Common explanation for 120. c, 131. b and 118. e:
glucose . sulfonylureas and the structurally unrelated Exogenously administered adrenocorticosteroids
compounds, repaglinide and nateglinide, stimulate are effective anti-inflammatory agents but give rise
the secretion of insulin from pancreatic β-cells. to a wide range of metabolic and
69. d Pioglitazone does not interact with estrogen immunosuppressive effects that result in severe
receptors. It is an oral hypoglycemic agent that adverse effects. Oral antidiabetic agents of the
produces its effects by binding to nuclear sulfonylurea type can cause blood dyscrasias,
peroxisome proliferator-activated receptors (PPARs) impaired liver function, and photosensitivity.
involved in transcription of insulin–responsive Exogenously administered adrogens suppress
genes and in regulation of adipocyte differentiation sperm formation and cause paradoxical
and lipid metabolism. The other four agents are gynecomastia. Most significant is the hepatotoxicity
correctly matched to their mechanisms. produced by alkyl-substituted adrogen compounds.
ANTIDIABETICS 207

CHAPTER 13
ANTIDIABETICS

1. Current criteria used in the diagnosis of patients neglect in rotating the insulin
diabetes mellitus (DM) include all of the injection site. This is known as
following symptoms except
(a) Lipoatrophy
(a) Fasting hyperglycemia
(b) Hypertrophic degenerative adiposity
(b) Polyuria
(c) Lipohypertrophy
(c) Polydipsia
(d) Atrophic skin lesion
(d) Tinnitus
(e) Weight loss (e) Dermatitis

2. The most useful glucose test used in moni- 5. Sulfonylureas are a primary mode of
toring diabetes mellitus (DM) therapy is therapy in the treatment of
(a) Urine monitoring (a) Insulin–dependent (type 1) diabetes mellitus
(b) Blood monitoring (DDM) patients
(c) Renal function monitoring (b) Diabetic patients experiencing severe hepatic
(d) Cardiovascular monitoring or renal dysfunction
(e) Vascular monitoring (c) Diabetic pregnant women
(d) Patient with diabetic ketoacidosis
3. Which of the following statements
concerning insulin replacement therapy (e) Non-insulin-dependent (type 2) DM patients
is most accurate?
6. Patients taking chlorpropamide should
(a) Most commercial insulin products vary little avoid products containing
with respect to time, course, and duration of
hypoglycemic activity (a) Acetaminophen (b) Ethanol
(b) Regular insulins cannot be mixed with NPH (c) Vitamin A (d) Penicillins
(isophane insulin suspension) (e) Milk products
(c) Regular insulin cannot be given intravenously
7. The standard recommended dose of gly-
(d) Counting or regulating carbohydrate con- buride is
sumption is a necessity for all diabetic pa-
tients (a) 0.5–2 mg/day
(e) Insulin therapy does not have to be monitored (b) 1.25–20mg/day
closely (c) 50–100 mg/day
4. A mass of adipose tissue that develops (d) 200 mg/day
at the injection site is usually due to the (e) 200–1000 mg/day
208 MCQs IN PHARMACOLOGY

8. Which of the following may increase the 16. The insulin preparation of choice in
insulin need of diabetics? diabetic ketoacidosis is
(a) Isoniazid (a) Regular insulin
(b) Penicillin (b) Lente insulin
(c) Glyceryl guaiacolate (c) Isophane insulin
(d) Aspirin (d) Monocomponent insulin
(e) Prednisone
17. Insulin resistance can be overcome by the
9. Excessive use of tolbutamide will lead to use of
(a) Diarrhea (a) Corticosteroids
(b) Prolonged hypoglycemia (b) Tolbutamide
(c) Tolerance to alcohol (c) Protamine
(d) Acidosis (d) Monocomponent insulin preparations
(e) Glycosuria 18. Human insulins are obtained by the
10. The insulin receptor is a following sources/methods except
(a) Ion channel regulating receptor (a) Cadaver pancreas
(b) Tyrosine protein kinase receptor (b) Proinsulin recombinant bacterial
(c) G-protein coupled receptor (c) Precursor yeast recombinant
(d) None of these (d) Enzyme modification of prok insulin

11. The duration of action of insulin–zinc 19. Compared to pork/beef insulins, the
suspension (lente insulin) is human insulins
(a) 2–4 hours (b) 8–10 hours (a) Are more potent
(c) 20–24 hours (d) 30–36 hours (b) Have a faster kinetics of absorption and
elimation
12. The most common adverse reaction to (c) Have longer biological action half-life
insulin is (d) Penetrated blood-brain barrier more
(a) Hypoglycaemia (b) Lipodystrophy efficiently
(c) Urticaria (d) Angioedema 20. The second generation sulfonylurea
13. Which of the following is a neuroglucopenic hypoglycaemics differ from the first
symptom of hypoglycaemia ? generation one in that they
(a) Sweating (b) Palpitation (a) Are more potent
(c) Tremor (d) Abnormal behavior (b) Are longer acting
(c) Do not lower blood sugar in nondiabetics
14. There is no alternative to insulin therapy subject
for
(d) Are less prone to cause hypoglycaemic
(a) All insulin dependent diabetes mellitus reaction
(IDDM) patients
(b) All noninsulin dependent diabetes mellitus 21. Metformin is preferred over phenformin
(NIDDM) patients because
(c) NIDDM patients not controlled by a (a) It is more potent
sulfonylurea drug (b) It is less liable to cause lactic acidosis
(d) NIDDM patients not controlled by a (c) It does not interfere with vitamin B 12
biguanide drug absorption
(d) It is not contraindicated in patients with
15. In a patient of diabetes mellitus maintained
kidney disease
on insulin therapy, administration of the
following drug can vitiate glycaemia 22. Sulfonylureas do not lower blood sugar
control. level in
(a) Prednisolone (b) Prazosin (a) Nondiabetics
(c) Paracetamol (d) Phenytoin (b) Noninsulin dependent diabetics
ANTIDIABETICS 209

(c) Insulin dependent diabetics (d) It raises circulating insulin levels


(d) None of these
29. Guargum limits post-prandial glycaemia
23. Sulfonylurea hypoglycaemics act by by
(a) Reducing intestinal absorption of glucose (a) Inhibiting intestinal brush border α-
(b) Increasing insulin secretion from pancreas glucosidases
(c) Reversing down-regulation of insulin (b) Slowing carbohydrate absorption from
receptors intestine
(d) Both (b) and (c) (c) Releasing incretins from the intestine
(d) Promoting uptake of glucose into skeletal
24. The hypoglycaemic action of sulfonylureas
is likely to be attenuated by the concurrent muscles
use of 30. Select the drug which tends to reverse
(a) Hydrochlorothiazid insulin resistance by increasing cellular
(b) Propranolol glucose transporters.
(c) Chloramphenicol (a) Glibenclamide (b) Troglitazone
(d) Aspirin (c) Acarbose (d) Prednisolone
25. Sulfonylureas are more commonly used 31. Glucagon release from pancreas is
than biguanides as oral hypoglycaemics stimulated by
because
(a) High blood glucose level
(a) Biguanides are less efficacious
(b) Insulin
(b) Sulfonylureas lower blood sugar in both
(c) Somatostatin
IDDM and NIDDM patients
(d) Adrenaline
(c) Sulfonylureas also aid weight reduction in
obese diabetics 32. Aldosterone enhances Na+ reabsorption
(d) Biguanides are prone to precipitate in renal tubules by
ketoacidosis
(a) Stimulating carbonic anhydrase
26. The present status of oral hypoglycaemics (b) Inhibitng Na+ K+ ATP ase
in diabetes mellitus is (c) Inducing the synthesis of Na+ K+ ATP ase
(a) They are the first choice drugs in all cases (d) Promoting K+ secretion
(b) They should be prescribed only if the patient
refuses insulin injections 33. Nephrogenic diabetes insipidus is seen
(c) They are used only in type I diabetes mellitus with
(d) They are used first in most uncomplicated (a) Demeclocycline (b) Doxycycline
mild to moderate type II diabetics (c) Minocycline (d) Oxytetracycline
27. Which of the following features disfavors 34. The antidiabetic agent most likely to
use of oral hypoglycaemics in diabetes cause lactic acidosis is
mellitus ?
(a) Chlorpropamide (b) Phenformin
(a) Age at onset of disease over 40 years (c) Glipizide (d) Metformin
(b) Insulin requirement more than 40 U/day
(c) Fasting blood sugar level between 100–200 35. The treatment of gestational diabetes
mg/dl would comprise of
(d) Associated obesity (a) Glibenclamide (b) Chlorpropamide
28. Which of the following is ture of acarbose ? (c) Glipizide (d) Insulin
(a) It reduces absorption of glucose from 36. Diabetic ketoacidosis is best managed by
intestines (a) Crystalline insulin given intravenously
(b) It produces hypoglycaemia in normal as well (b) Human insulin given intramusculary
as diabetic subjects
(c) Lente insulin given subcutaneously
(c) It limits postprandial hyperglycaemia in
diabetis (d) Isophane insulin given intradermally
210 MCQs IN PHARMACOLOGY

37. The sulfonylurea with a relatively longer 43. A diabetic on oral hypoglycaemic drug
duration of action is chlorpropamide, suffered from enteric
fever and was prescribed chloramphenicol.
(a) Chlorpropamide (b) Tolbutamide
He developed severe hypoglycaemia. This
(c) Glibenclamide (d) Glipizide is because
38. Insulin (a) Chloramphenicol itself has mild hypoglycaemic
effect
(a) Release is enhance by somatostatin
(b) Chloramphenicol increases the absorption
(b) Has an identical chemical structure in all the
of chlorpropamide
species
(c) Chloramphenicol causes release of insulin
(c) Release from the pancreas occurs only in the
postprandial state (d) Chloramphenicol inhibits the metabolism of
chlorpropamide
(d) Promotes synthesis of triglycerides
44. For increasing the excretion of weakly
39. Metformin acidic drugs, urine should be made
(a) Does not cause hypoglycemia even in large (a) Alkaline
doses (b) At neutral pH
(b) Should not be combined with glipizide (c) Acidic
(c) Is contraindicated in obese NIDDM patients (d) pH does not effect the urinary excretion of
(d) Causes release of insulin from the pancreas acidic drugs
40. The agent with neglible mineralocorticold 45. Longest acting insulin is
effect is (a) Insulin zinc suspension
(a) Prednisone (b) Betamethasone (b) Isophane insulin
(c) Fludrocortisone (d) Cortisol (c) Globin zinc insulin
41. Preferred route of insulin is (d) protamine zinc insulin

(a) Oral (b) Subcutaneous 46. Diuretic effective in diabetes insipidus is


(c) Sublingual (d) Enteric coated tabs (a) Thiazides
(b) Loop diuretic
42. Incorporation of vasoconstrictor substance
in a solution of a drug to be injected (c) Mercurial diuretic
subcutaneously retards absorption. This (d) Carbonic anhydrase inhibitor
principal is utilized in combination of 47. The insulin receptor is
(a) Epinephrine with local anaesthetics (a) Tyrosine protein kinase receptor
(b) Epinephrine with I.V. glucose (b) G protein coupled receptor
(c) With vaccines (c) Ion channel regulating receptor
(d) With insulin (d) None of these

ANSWERS
1. d 2. b 3. d 4. c 5. e 6. b
7. b 8. e 9. b 10. b 11. c 12. a
13. d 14. a 15. a 16. a 17. d 18. a
19. b 20. a 21. b 22. c 23. d 24. a
25. a 26. d 27. b 28. c 29. b 30. b
31. d 32. c 33. a 34. b 35. d 36. a
37. a 38. d 39. a 40. b 41. b 42. a
43. d 44. a 45. d 46. a 47. a
ANTICOAGULANTS 211

CHAPTER 14
ANTICOAGULANTS

1. Which preparation of Vitamin K should (c) They are metabolized slowly and have longer
not be injected in the new born ? duration of action
(a) Phytonadione (d) All of the above
(b) Menadione 6. Initiation of unfractionated heparin
(c) Menadione sod.diphosphate therapy for the patient in question 1
(d) Both (b) and (c) would best be achieved with
2. Unfractionated heparin binds to anti- (a) 5000-unit loading dose followed by 1000
thrombin III and inactivates clotting units/hr
factor(s). (b) 5000-unit loading dose followed by 1800
(a) Xa (b) Ixa units/hr
(c) Iia (d) All of the above (c) 8000-unit loading dose followed by 1800
(e) None of these units/hr
(d) 8000-unit loading dose followed by 1000
3. Low concentrations of heparin selectively units/hr
interfere with the following coagulation
patheay(s) 7. Which of the following can be used to
(a) Intrinsic pathway antagonize the action of heparin in case
of overdose ?
(b) Extrinsic pathway
(c) Common pathway (a) Heparin sulfate (b) Dextran sulfate
(d) Both (a) and (c) (c) Protamine sulfate (d) Ancrod

4. Low doses of heparin prolong 8. Blood level of which clotting factor


declines most rapidly after the initation
(a) Bleeding time
of warfarin therapy
(b) Activated partial thromboplastin time
(a) Factor VII (b) Factor IX
(c) Prothrombin time
(c) Factor X (d) Prothrombin
(d) Both (b) and (c)
9. The following drug reduces the effect of
5. Low molecular weight heparins differ
from conventional heparin in that oral anticoagulants

(a) They selectively inhibit factor Xa (a) Broad spectrum antibiotic


(b) They do not significantly prolong clotting time (b) Cimetidine
212 MCQs IN PHARMACOLOGY

(c) Aspirin (a) If I only is correct


(d) Oral contraceptive (b) If III only is correct
10. The most definite beneficial results are (c) If I and II are correct
obtained in the use of anticoagulants for (d) If II and III are correct
the following purpose (e) If I, II, and III are correct
(a) Prevention of recurrences of myocardial 16. Thrombolytic therapy instituted within 3-
infarction 6 hours of onset of acute myocardial
(b) Prevention of venous thrombosis and infarction affords the following benefit(s)
pulmonary embolism
(a) Reduces mortality
(c) Cerebrovascular accident
(b) Reduces area of myocardial necrosis
(d) Retinal artery thrombosis
(c) Preserves ventricular function
11. Anticoagulants are indicated in (d) All of these
(a) Immobilized elederly patients 17. The preferred route of administration of
(b) Buerger’s disease streptokinase in acute myocardial infarction
(c) Stroke due to cerebral thrombosis is
(d) All of these (a) Intravenous (b) Subcutaneous
12. Which of the following tests are used to (c) Intracoronary (d) Intracardiac
monitor antithrombotic therapy?
18. A patient on oral anticoagulant therapy
I. International normalized ratio (INR) is commenced on sulfamethoxazole-
II. Activated partial thromboplastin time (APTT) trimethoprim, double-strength twice
III. Heparin assay daily. One may expect to see the INR
(a) If I only is correct I. Increase
(b) If III only is correct II. Decrease
(c) If I and II are correct III. Remain unchanged
(d) If II and III are correct (a) If I only is correct
(e) If I, II, and III are correct (b) If III only is correct
13. Which fibrioolytic drug(s) is/are antigenic (c) If I and II are correct
(d) If II and III are correct
(a) Streptokinase (b) Urokinase
(e) If I, II, and III are correct
(c) Alteplase (d) Both (a) and (b)
19. Aspirin prolongs bleeding time by inhibiting
14. The most important complication of
the synthesis of
streptokinase therapy is
(a) Clotting factors in liver
(a) Hypotension (b) Bleeding
(b) Prostacyclin in vascular endothelium
(c) Fever (d) Anaphylaxis
(c) Cyclic AMP in platelets
15. A patient to be commenced on oral
(d) Thromboxane A2 in platelets
anticoagulant therapy for DVT would be
treated with: 20. Route of heparin administration is
I. Oral anticoagulant therapy with warfarin for (a) Oral (b) Subcutaneous
a goal international normalized ratio (INR) (c) Intramuscular (d) Sublingual
of 2–3
II. Oral anticoagulant therapy with warfarin for 21. The anticoagulant effects of heparin can
a goal INR of 2.5–3.5 be promptly arrested by administration of
III. Oral anticoagulant therapy with aspirin for (a) Epsilon amino caproic acid
a goal INR of 2–3 (b) Protamine sulfate
ANTICOAGULANTS 213

(c) Adrenaline I. Hold the drug therapy


(d) Vitamin K II. Administer vitamin K
III. Administer fresh frozen plasma
22. Severe cases of bleeding due to
fibrinolytic agents are treated with (a) If I only is correct
(b) If III only is correct
(a) Aspirin
(c) If I and II are correct
(b) Heparin
(d) If II and III are correct
(c) EACA (Epsilon Amino Caproic Acid)
(e) If I, II, and III are correct
(d) Vitamin K
23. If a patient has an INR greater than 20
and active bleeding that is clinically
significant (i.e., hematuria), the pharmacist
should

ANSWERS
1. d 2. b 3. a 4. b 5. d 6. a
7. c 8. a 9. d 10. b 11. a 12. c
13. a 14. b 15. b 16. d 17. a 18. a
19. d 20. b 21. b 22. c 23. b

EXPLANATIONS FOR THE ANSWERS


2. d Unifractionated heparin acts heparin acts as can be evaluated by heparin assay. Because of
an anticoagulant by catalyzing the the reliability of dose responsiveness seen with
inactivation of thrombin (factor Iia), activated LMWH therapy, the need to perform heparin
factor X (factor Xa), and activated factor IX assays is controversial.
(factor Ixa)by antithrombin III. 15. a Oral anticoagulant therapy is monitored by
6. c Varying nomograms for dosing continuous- measuring the prothrombin time (PT). The PT is
infusion unfractionated heparin exist in the responsive to depression of three of the four
medical and pharmaceutical literature. The vitamin K-dependent procoagulant clotting
loading dose is typically 70–100 units/kg. factors, these respective clotting factors take
In this case the loading dose is 80 units/kg. approximately 96 hours to be depleted, at which
Maintenance doses of 15–25 units/kg/hr are time the PT should be sufficient to arrive at an
typically used. In this case , the maintenance international normalized ratio (INR) of 2.0–3.0
dose is 18 units/kg/hr. for patients with deep venous thrombosis. Patients
12. d Unfractionated heparin may be appropriately with mechanical prosthetic heart valves have INRs
monitored by either the activated partial targeted in the 2.5–3.5 range. Aspirin therapy
thromboplastin time (aPTT) or heparin assay. is not monitored by INR determinations.
Because different laboratories use APTT 18. a Oral anticoagulant therapy with warfarin may be
reagents with varying sensitivities, the APTT complicated by a myriad of drug-drug interactions
range and its corresponding ratio must be owing to the highly protein-bound state of warfarin.
correlated to a heparin level of 0.2–0.4 units/ Such drug interactions may potentiate the
ml; or 0.3–0.7 units/ml by plasma-amidolytic prothrombin time/international normalized ration
assay. The safety and efficacy of low- (PT/INR), inhibit the anticoagulant effect of warfarin,
molecular-weight heparin (LMWH) or have no effect on the actions of warfarin.
cannot be reliably evaluated by APTT Sulfamethoxazole-trimethoprim and other antibiotics
determinations. LMWH safety and efficacy have the potential to augment the anticoagulant
214 MCQs IN PHARMACOLOGY

effect of warfarin by eliminating bacterial flora and surgery or invasive procedures will always
thereby, producing vitamin – K deficiency. hasten the urgency of warfarin reversal. In the
23. e Pharmacists may be called on to offer advice setting of active bleeding, its clinical significance
regarding reversal of warfarin therapy or may must be demonstrated by consultation with the
be empowered using Pharmacy and patient’s physician. If the international
Therapeutics Committee or Medical Board normalized ration is >20 and the patient has
approved protocols to reverse warfarin’s effect. active bleeding that is clinically significant, the
In all instances, the pharmacist must critically pharmacist must hold drug therapy, consider the
and clinically evaluate the situations and most appropriate dose and route of vitamin-K
communicate with the physician regarding delivery and administer fresh frozen plasma to
management issue. A need for immediate replete the vitamin-Kdependent clotting factors.
ANTIHYPERLIPEDEMIC AGENTS 215

Chapter 15
ANTIHYPERLIPEDEMIC AGENTS

1. Inhibition of thromboxane synthesis by (a) Clofibrate (b) Cholestyramine


aspirin in platelets lasts for 5–7 days (c) Lovastatin (d) Bezafibrate
because
5. Select the most appropriate hypolipidemic
(a) Aspirin persists in the body for 5–7 days drug for a patient with raised LDL-
(b) Aspirin induced depletion of arachidonic cholesterol level but normal triglyceride
acid lasts 5–7 days level
(c) Regeneration of aspirin inhibited cyclo- (a) A HMG – CoA reductase inhibitor
oxygenase takes 5–7 days (b) A fibric acid derivative
(d) Platelets cannot generate fresh thromboxane (c) Probucol
synthetase and their turnover time is 5-7 days (d) Nicotinic acid
2. Choose the drug which has a direct effect on 6. A patient with coronary artery disease
platelet membrane to inhibit aggregation, has raised serum triglyceride (600 mg/
release reaction and to improve platelet dl) but normal total cholesterol level (150
survival in extra-corporeal circulation mg/dl). Which hypolipidemic drug should
be prescribed
(a) Dipyridamole (b) Ticlopidine
(c) Aspirin (d) Sulfinpyrazone (a) Probulol (b) Gemfibrozil
(c) Cholestyramine (d) Lovastatin
3. Combined therapy with dipyridamole
and warfarin is recommended in subjects 7. Choose the correct statement about
with the following lovastatin

(a) Risk factors for coronary artery disease (a) It markedly lowers plasma triglyceride with
little effect on cholesterol level
(b) prosthetic heart valves
(b) It is used as an adjuvant to gemfibrozil for
(c) Chronic arteriovenous shunts for repeated type III hyperlipoproteinemia
haemodialysis
(c) It is not effective in diabetes associated
(d) Both (b) and (c) hypercholesterolemia
4. Select the hypocholesterolemic drug which (d) It is a competitive inhibitor of the rate limiting
interferes with intestinal absorption of bile step in cholesterol synthesis
salts and cholesterol, and secondarily 8. Select the drug which reduces cholesterol
increases cholesterol turnover in the liver synthesis in liver, increases expression of
216 MCQs IN PHARMACOLOGY

LDL receptors on hepatocytes and has 14. Hydroxyethyl starch is a


been found to reduce mortality due to (a) Plasma expander
coronary artery disease
(b) Haemostatic
(a) Simvastatin (b) Nicotinic acid (c) Heparin substitute
(c) Probucol (d) Colestipol (d) Bile acid sequestrant
9. What is true of nicotinic acid as well as 15. Cholestyramine resin
nicotinamide ?
(a) Significantly lowers plasma triglyceride levels
(a) Both possess vitamin B3 activity (b) Can enhance the oral absorption of digoxin
(b) Both cause cutaneous vasodilatation (c) May enhance the absorption of fat-soluble
(c) Both lower plasma triglyceride and VLDL vitamins
levels (d) Is the drug of choice in type II hyperlipopro-
(d) Both cause hyperglycaemia after prolonged teinemia
medication
16. Clofibrate
10. Which hypolipidemic drug has been used (a) May reduce the action of warfarin
to control and prevent pancreatitis in
(b) Can decrease gall stone formation
familial hypertriglyceridemia ?
(c) Can elevate plasma HDL cholesterol levels\
(a) Lovastatin (b) Clofibrate (d) Increases platelet aggregation
(c) Cholestyramine (d) Nicotinic acid
17. Probucol
11. The rare but characteristic adverse effect
(a) Can lower HDL cholesterol levels
of HMGCoA reductase inhibitors is
(b) Reduces clearance of plasma LDL by the liver
(a) Onycolysis (c) Can produce constipation
(b) Myopathy (d) Is benefician in patients with recent myocardial
(c) Alopecia infarction
(d) Oculomucocutaneous syndrome
18. Nicotinic acid
12. In a 50-year-old male without any other (a) Reduces production of VLDL
coronary artery disease risk factor, (b) Should not be used in combination with
hypocholesterolemic drugs are recom- cholestyramine resin
mended only when the serum LDL
(c) Can lower HDL levels
cholesterol level is higher than
(d) Can increase serum triglyceride levels
(a) 130 mg/dl (b) 160 mg/dl
(c) 190 mg/dl (d) 240 mg/dl 19. Which of the following statements is true
regarding cholestyramine?
13. High molecular weight, pharmacody- (a) It inhibits free fatty acid release from adipose
namically inert, nonantigenic substances tissue
which form colloidal solution are used as
(b) It releases lipoprotein lipase
(a) Osmotic purgatives (c) It blocks the final step in the synthesis of cholestrol
(b) Osmotic diuretics in the body
(c) Plasma expanders (d) When used in large doses, it decreases serum
(d) All of the above cholesterol

ANSWERS
1. d 2. b 3. d 4. b 5. a 6. b
7. d 8. a 9. a 10. c 11. b 12. c
13. c 14. a 15. d 16. c 17. a 18. a
19. d
ANTACIDS 217

Chapter 16
ANTACIDS

1. For healing duodenal ulcer the usual 5. Antacid combinations of magnesium and
duration of H2 blocker therapy is aluminium salts are superior to single
(a) 4 weeks component preparations because
(b) 6 weeks (a) They have rapid as well as sustained acid
(c) 8 weeks neutralizing action
(d) 12 weeks (b) They are less likely to affect gastric emptying
(c) They are less likely to alter bowel movement
2. What is true of acid control therapy with (d) All of the above
H2 blockers ?
(a) It generally heals duodenal ulcers faster than 6. In peptic ulcer, antacids are now primarily
gastric ulcers used for
(b) It checks bleeding in case of bleeding peptic (a) Prompt pain relief
ulcer (b) Ulcer healing
(c) It prevents gastroesophageal reflux (c) Preventing ulcer relapse
(d) Both (a) and (b) (d) Control of bleeding from the ulcer
3. The ‘acid neutralizing capacity’ of an 7. The following is a noncompetitive anta-
antacid is governed by gonist at the gastric jparietal cell H2
(a) The equivalent weight of the antacid receptor.
(b) The pH of 1N solution of the antacid (a) Cimetidine (b) Loxatidine
(c) The rate at which the antacid reacts with HCl (c) Roxatidine (d) Rantidine
(d) Both (a) and (c) 8. Gynaecomastia can occur as a side effect
4. An antacid with the following property of
would be desirable in the treatment of (a) Bromocriptine (b) Levodopa
peptic ulcer. (c) Famotidine (d) Cimetidine
(a) Which raises gastric pH to 4.0
9. Which histamine H2 blocker has maximum
(b) Which raises gastric pH to 7.0 inhibitory effect on microsomal cytochrome
(c) Which increases gastric motility and hastens P-450 enzyme ?
gastic emptying
(a) Cimetidine (b) Ranitidine
(d) Both (b) and (c)
(c) Roxatidine (d) Famotidine
218 MCQs IN PHARMACOLOGY

10. Choose the correct statement about H2 (a) Carbenoxolone sodium


receptor blockers. (b) Sucralfate
(a) They are the most efficacious drugs in (c) Famotidine
inhibiting gastric acid secretion (d) Lansoprazole
(b) They cause fastest healing of duodenal ulcers
17. Omeprazole exerts practically no other
(c) They are the most commonly used drugs for
action except inhibition of gastric acid
inhibiting gastric acid secretion
secretion because
(d) They afford most prompt relief of ulcer pain
(a) In transforms into the active cationic forms
11. Ranitidine is mostly given by intravenous only in the acidic pH of the gastric juice
infusion for the following indication. (b) Its active forms have selective affinity for the
(a) Rapid relief of gastric ulcer pain H+K+ ATP ase located in the apical canaliculi
(b) Prophylaxis of gastric erosion in patients with of gastric parietal cells
extensive burns (c) Its cationic forms are unable to diffuse out
(c) Zollinger Ellison syndrome from the gastric parietal cell canaliculi
(d) Reflux oesophagitis (d) All of the above

12. Ranitidine differs from cimetidine in the 18. The most efficacious drug for inhibiting
following manner. round the clock gastric acid output is

(a) It is less potent (a) Omeprazole (b) Cimetidine


(b) It is shorter acting (c) Pirenzepine (d) Misoprostol
(c) It does not have antiandrogenic action 19. The primar y mechanism by which
(d) It produces more CNS side effects prostaglandins promote ulcer healing is

13. Eradication of H pylori along with gastric (a) Inhibition of gastric secretion
antisecretory drugs affords the following (b) Augmentation of bicarbonate buffered mucus
benefit(s). layer covering gastroduodenal mucosa
(a) Faster relief of ulcer pain (c) Increased bicarbonate secretion in gastric
juice
(b) Faster ulcer healing
(d) Increased turnover of gastric mucosal cell
(c) Reduced chance of ulcer relapse
(d) Both (b) and (c) 20. Which of the following statements is true
about misoprostol ?
14. The drugs employed for anti H pylori
therapy include the following except (a) It relieves peptic ulcer pain, but does not
promote ulcer healing
(a) Ciprofloxacin (b) Clarithromycin
(b) It heals nonsteroidal anti-inflammatory drug
(c) Tinidazole (d) Amoxicillin induced gastric ulcer not responding to H2
15. The following class of gastric antisecretory blockers
drug have primary effect on juice volume, (c) It produces fewer side effects than H2 blockers
with less marked effects on acid and (d) It is the most effective drug for preventing
pepsin content, and also reduce gastric ulcer relapse
motility
21. Sucralfate promotes healing of duodenal
(a) Histamine H2 blockers ulcer by
(b) Anticholinergics
(a) Enhancing gastric mucus and bicarbonate
(c) Proton pump inhibitors secretion
(d) Prostaglandins (b) Coating the ulcer and preventing the action
16. The following drug is an inhibitor of of acid-pepsin on ulcer base
gastric mucosal proton pump.
ANTACIDS 219

(c) Promoting regeneration of mucosa (c) Senna (d) Magnesium sulfate


(d) Both (a) and (b)
29. Saline osmotic purgatives are used for
22. Antacids administered concurrently (a) Reatment of constipation
reduce efficacy of the following antipeptic (b) Prevention of constipation in patients of piles
ulcer drug.
(c) Avoidance of straining at stools in patients
(a) Cimetidine (b) Omeprazole of hernia
(c) Sucralfate (d) Pirenzepine (d) Tapeworm infestation: Following niclosamide
administration
23. The most important drawback of sucralfate
in the treatment of duodenal ulcer is 30. The most suitable laxative for a patient
(a) Low ulcer healing efficacy of irritable bowel syndrome with spastic
(b) Poor relief of ulcer pain constipation is
(c) Highincidence of side effects (a) Dietary fibre (b) Liquid paraffin
(d) Need for taking a big tablet four times a day (c) Bisacodyl (d) Senna

24. The preferred regimen for preventing 31. The success of oral rehydration therapy
duodenal ulcer relapse is of diarrhoea depends upon the following
process in the intestinal mucosa.
(a) Maintenance antacid regimen
(b) Maintenance H2 blocker regimen (a) Sodium pump mediated Na+ absorption
(c) On demand intermittent H2 blocker regimen (b) Glucose coupled Na+ absorption
(d) Mintenance sucralfate regimen (c) Bicarbonate coupled Na+ absorption
(d) Passive Na+ diffusion secondary to nutrient
25. Used as a laxative, liquid paraffin has absorption
the following drawbacks except
32. Saline laxatives containing magnesium
(a) It interferes with absorption of fat soluble
vitamins (a) Reduce the secretion of cholecystokinin
(b) It is unpleasant to swallow (b) Are more effective when administered on an
(c) It causes gripping empty stomach
(d) It may produce foreignbody granulomas (c) Are commonly used in the treatment in
functional constipation
26. Which of the following purgatives (d) Are safe in patients with impaired renal function
undergoes enterohepatic circulation to
produce prolonged action ? 33. Antimotility drugs are contraindicated in
(a) Docusates (b) Phenolphthalein (a) Mild traveler’s diarrhoea
(c) Castor oil (d) Lactulose (b) Acute infective diarrhoeas
(c) Ileostomy patients
27. The following laxative lowers blood
(d) Patients after anal surgery
ammonia level in hepatic encephalopathy.
(a) Bisacodyl (b) Liquid paraffin 34. Octreotide
(c) Lactulose (d) Magnesium sulfate (a) Has a short half-life of 2 minutes
(b) Is given orally three times in a day
28. Select the purgative that should not be
taken at bed time. (c) Is useful in carcinoid syndrome
(d) Increases intestinal smooth muscle
(a) Ispaghula (b) Bisacodyl
contractility
220 MCQs IN PHARMACOLOGY

35. Therapy of choice in Zollinger Ellison 38. Omeprazole is most useful in


syndrome is
(a) Gastric ulcer
(a) Omeprazole (b) Duodenal ulcer
(b) Ranitidine (c) Reflux esophagitis
(c) Sucralfate (d) Gastritis
(d) Carbenoxolone sodium
39. Antacids should not be prescribed concur-
36. Misoprostol rently with
(a) Is helpful in preventing ulcers induced by (a) Ranitidine (b) Pirenzipine
NSAIDs (c) Sucralfate (d) Omeprazole
(b) Can cause constipation
40. The antiulcer drug ver y helpful in
(c) Does not inhibit acid secretion
preventing NSAID induced gastric ulcer
(d) Can delay labor in a pregnant woman is
37. Ranitidine is a new histamine receptor (a) Roxatidine (b) Furosemide
blocker. It is (c) Enalapril (d) Amrinone
(a) Less potent, non competitive but selective
41. In peptic ulcer, antacids do have a
antagonist of histamine at gastric site than
primary role in
cimetidine
(b) More potent, non selective and most toxic (a) Prompt relief of pain
histamine antagonist of histamine at gastric (b) Ulcer healing
site (c) Control of bleeding
(c) More potent, competitive and selective (d) Prevention of ulcer relapse
antagonist of histamine at gastric site
(d) None of the above

ANSWERS
1. c 2. a 3. d 4. a 5. d 6. a
7. b 8. d 9. a 10. c 11. b 12. c
13. d 14. a 15. b 16. d 17. d 18. a
19. b 20. b 21. b 22. c 23. d 24. b
25. c 26. b 27. c 28. d 29. d 30. a
31. b 32. b 33. b 34. c 35. a 36. a
37. c 38. b 39. c 40. c 41. a
ANTIEMETICS 221

Chapter 17
ANTIEMETICS

1. The most dependable emetic used to (a) Metoclopramide


expel ingested poisons is (b) Cisapride
(a) Intramuscular emetine (c) Domperidone
(b) Oral syrup ipecacuanha (d) All of these
(c) Intramuscular apomorphine
6. A patient returning from dinner party
(d) Oral bromocriptine meets with road accident and has to be
2. The most effective antimotion sickness urgently operated upon under general
drug suitable for short brisk journies is anaesthesia. Which drug can be injected
intramuscularly to hasten his gastric
(a) Promethazine theoclate
emptying
(b) Cinnarizine
(a) Methypolysiloxane
(c) Prochlorperazine
(b) Promethazine
(d) Hyoscine
(c) Metoclopramide
3. In case of hill journey, antimotion sickness (d) Apomorphine
drugs are best administered at
7. Which antiemetic selectively blocks
(a) Twelve hours before commencing journey
levodopa induced vomiting without
(b) One hour before commencing journey
blocking its antiparkinsonian action ?
(c) Immediately after commencing journey
(a) Metoclopramide (b) Cisapride
(d) At the first feeling of motion sickness
(c) Domperidone (d) Ondansetron
4. Metoclopramide blocks apomorphine
induced vomiting, produces muscle 8. The fastest symptomatic relief as well as
dystonias and increases prolactin release highest healing rates in reflux esophagitis
indicates that it has have been obtained with

(a) Anticholinergic action (a) Cisapride (b) Ranitidine


(b) Antihistaminic action (c) Omeprazole (d) Sodium alginate
(c) Anti 5-HT3 action 9. Cisapride enhances gastrointestinal
(d) Antidopaminergic action motility by
5. Which prokinetic drug(s) produce(s) (a) Activating serotonin 5-HT4 receptor
extrapyramidal side effects ? (b) Activating muscarinic M3 receptor
222 MCQs IN PHARMACOLOGY

(c) Blocking serotonin 5-HT3 receptor (d) Three times the molar concentration of Na+
(d) Blocking dopamine D2 receptor
16. The concentration of sodium ions in WHO
10. The most effective antiemetic for oral rehydration solution is
controlling cisplatin induced vomiting is (a) 40 m moles/L (b) 60 m moles/L
(a) Prochlorperazine (b) Ondansetron (c) 90 m moles/L (d) 110 m moles/L
(c) Metoclopramide (d) Promethazine
17. The electrolyte composition of WHO oral
11. Ondansetron is a rehydration solution is based upon that of
(a) Second generation antihistaminic (a) Enterotoxigenic E. coli diarrhea stools
(b) Drug for peptic ulcer (b) Cholera stools in adults
(c) New antiarrhythmic (c) Cholera stools in children
(d) Antiemetic for cancer chemotherapy (d) Rotavirus diarrhea stools
12. Ondansetron blocks emetogenic impulses 18. A case of acute diarrhea presents with
at the following site(s). abdominal pain, fever, mucus and blood
(a) Vagal afferents in intestines in stools and is suspected to be suffering
(b) Nucleus tractus solitarius from Shigella enteritis. What antimicrobial
(c) Chemoreceptor trigger zone treatment would be most appropriate.
(d) All of the above (a) No antimicrobial treatment
(b) Metronidazole
13. Cancer chemotherapy induced vomiting
that is not controlled by metoclopramide (c) Norfloxacin
alone can be suppressed by combining it (d) Chloramphenicol
with
19. Ordansetron
(a) Amphetamine (b) Dexamethasone
(a) Can cause extrapyramidal side effects
(c) Hyoscine (d) Cyclizine
(b) Is not effective in control of acute emesis
14. Irrespective of the type, all laxatives exert (c) Can prevent emesis due to radiation
the following action. (d) Is not absorbed orally
(a) Increase the content of solids in the faeces
20. The prokinetic effects of metoclopramide
(b) Increase the water content of faeces
can be abolished by
(c) Reduce absorption of nutrients
(a) Vagotomy (b) Atropine
(d) Increase intestinal motility
(c) Bethanechol (d) Acetylcholine
15. For optimum rehydration, the molar
concentration of glucose in ORS should be 21. The most potent drug for prevention of
motion sickness is
(a) Equal to the molar concentration of Na+
(b) Somewhat lower than molar concentration (a) Dimenhydrinate (b) Tripelenamine
of Na+ (c) Scopolamine (d) Hydroxygene
(c) Somewhat higher than the molar
concentration of Na+

ANSWERS
1. c 2. d 3. b 4. d 5. a 6. c
7. c 8. c 9. a 10. b 11. d 12. d
13. b 14. b 15. c 16. c 17. c 18. c
19. c 20. b 21. c
MATCH THE FOLLOWING 223

CHAPTER 18
MATCH THE FOLLOWING

1. Match the correct mechanism of action for (4) Sheep plasma (d) Human antihaemophyllic
the diuretic agents mentioned below: fraction

(1) Acetazolamide (a) Increases serum K+ level 4. The drug A to E are used as diuretics.
(2) Chlorthiazide (b) Competitively antagonizes Match them to their classes.
aldodsterone
(1) Osmotic diuretic (a) Spiranolactone
(3) Spiranolactone (c) Inhibits active Na+ secretion,
(2) Loop diuretic (b) Isosorbide
decreasing K+ excretion in the
distal nephron (3) Potassium sparing (c) Merasalyl theophylline
diuretic (d) Furosemide
(4) Triamterene (d) Inhibits carbonic anhydrase
(4) Organomercurial (e) Probenecid
(e) Inhibits electrolyte reabsorp-
diuretic
tion in the distal portion of the
ascending limb of the loop of 5. Pharmacologicla activity of certain well
Henle known plant drugs are listed A to E.
2. Listed are some of the commonly used Match them.
drugs. Their pharmacological actions are (1) Papaverin (a) Weak analeptic
listed in A to E, match them. (2) Camphor (b) Vasodilator
(1) Aspirin (a) Rises body temperature (3) Veratrum alkaloids (c) Antineoplastic
(2) Acetaminophen (b) Non–analgesic anti-inflam- (4) Vincristine (d) Central vasoconstrictor
matory (e) Anxiolytic
(3) Phenylbutazone (c) Non–anti-inflammator y an-
6. Listed are drugs 1 to 4. Their appropriate
algesic
antihypertensive mechanisms are given
(4) Probenacid (d) Increases of depth of res-
piration
in A to F match them.
(e) Increases fluid retention (1) Pindalol (a) Vadoilator
(2) Minoxidil (b) Angiotiensin coverting en-
3. The following are the test animals or zyme inhibitor
substances used for the biological assay (3) Captopril (c) Diuretic
of preparations listed in A to E. Match (4) Amiloride (d) Beta–blocker
them correctly.
(e) Centrally acting alpha
(1) Mice (a) Vasopressin adrenoceptor agonist
(2) Albino rats (b) Diphtheria antitoxin f) Potassium induction
(3) Guinea pigs (c) Insulin
224 MCQs IN PHARMACOLOGY

7. Symptoms for the following diseases are (4) Antitussive (d) Diphenoxylate
indicated from A to E . Match them. (e) Codeine
(1) Cushing’s syndrome (a) Hyperthyroidism 13. Match the following antiarrhythmic drugs
(2) Addison’s disease (b) Inflammatory bowel to the respective categories A to E:
(3) Grave’s disease (c) Decreased production of cortisol
(1) Acebutalol (a) Calcium channel blocker
(4) Crohn’s disease (d) Decreased production of cortisol
(2) Verapamil (b) Potassium channel blocker
(e) Increased production of cortisol
(3) Kromakalem (c) Sodium channel blocker
8. The undersirable effects of the antibiotics (4) Phenytoin (d) Beta adrenoceptor blocker
are listed in A to E. Match them. (e) Chloride channel blocker
(1) Tetracycline (a) Gray–baby syndrome 14. Match the following categories of
(2) Streptomycin (b) Discolouration of teeth antibiotics with their respective examples
(3) Chloramphenicol (c) Jaundice A to E:
(4) Rifampicin (d) Obesity (1) Narrow spectrum (a) Neomycin
9. Match the following terms with their (2) Broad - spectrum (b) Cephalsporins
respective definitions A to E: (3) Acting on cell (c) Streptomycin
membrane
(1) Achyliagestrica (a) Decrease in alkali contents
(4) Inhibiting protein (d) Cycloserine
(2) Acidosis (b) Absence of hydrochloric acid
synthesis
(3) Actinomycosis (c) A deficient disorder of adrenal
contex with anaemia, weakness, 15. Match the following penicillins with their
dyspepsia, hypotension respective categories A to E:
(4) Addision’s disease (d) A fungal disease caused by (1) Penicillin G (a) Broad spectrum
Actinomyces
(2) Penicillin V (b) Penicillinase resistant
(e) A state of psychic and physical
(3) Methicillin (c) Natural
drug dependence
(4) Ampicillin (d) Semi–synthetic
10. Match the following categories of (e) Anti–pseudomonas
Adrenocorticosteroids with their
respective examples A to E: 16. Match the following categories of
anticancer hormones with their respective
(1) Glucocorticoids (a) Aldosterone, Fludrocortisone
examples A to E:
(2) Mineral corticoids (b) Ketokonazole, Metyapone
(1) Androgens (a) Methoxyestradiol
(3) Receptor (c) Cholesterol, β-Sitosterol
(2) Gonadotropin- (b) Prednisolone
(4) Synthetic Inhibitors (d) Cortisol, Triamcinolone
releasing
(e) Mifepristone, Spironolactone
(3) Estrogens (c) Testosterone
11. Match the following anaesthetics with (4) Adrenocortrocoids (d) Goserelin
their respective nature A to E: (e) Diethylstilbestrol
(1) Coacine (a) Other anaesthetic
17. Match the following antidepressant drugs
(2) Lignocaine (b) Synthetic non-nitrogenous with their respective categories A to E:
compound
(1) Amitriptyline (a) Non-specific
(3) Propanediol (c) Inhaled anaesthetic
(2) Nizoxetin (b) Tricyclic first generation
(4) Chlorpromazine (d) Synthetic nitrogenous com-
pound (3) Caffeine (c) Tricyclic second generation
(e) Natural alkaloid (4) Isoniazid (d) Psychomotor stimulant
(e) Heterocyclic
12. Match the following antiemetic drugs with
their respective categories A to E: 18. Match the following antidiabetic drugs
with their respective categories A to E
(1) Weak agonist (a) Pentazocine
(2) Strong agonist (b) Propoxyphene (1) Phenformin (a) Long acting
(3) Antidiarrhea (c) Heroin (2) Tolbutamide (b) Rapid acting
MATCH THE FOLLOWING 225

(3) NPH Insulin (c) Intermediate acting 24. Match the following antiasthamtic drugs
(4) Protamine zinc (d) Beguanide suspension with their respective categories A to E:
insulin
(1) Ephedrine (a) Purine base bronchodilator
(e) Sulphonylurea
(2) Theophyline (b) Symathomimetic bronchodi-
19. Match the following categories of lator
antifungal drugs with their respective (3) Ipratropium bromide (c) Polypetide
examples A to E: (4) Disodium (d) Anticholinergics bronchodilator
(1) For superficial (a) Amphotericin B cromoglycate
infection (e) Anti - inflammatory
(2) Topical (b) Nystatin
(3) For systemic (c) Undercylenic acid 25. Match the following terms with their
mycoses respective meaning A to E:
(4) For vaginal (d) Grisefulvin (1) Bradicardia (a) Irritation of the mucous
candidiasis meanings membrane
(e) Nystatin
(2) Bronchitis (b) Feeble state produced by a
20. Match the following antihyperlipidemic serious disease
drugs with their respective categories A (3) Chachexia (c) Inflammation of mucous
to E: membrane
(1) Cholesteryl amine (a) Fibric acid derivative (4) Catarrh (d) Glowness of the heart beat
(2) Clofibrate (b) Bile acid sesquestrant (e) Narrowing of the bronchi
(3) Lovastation (c) VLDL secreting inhibitor
26. Match the following drugs with therir
(4) Niacin (d) Antioxidant
respective mechanisms A to E:
(e) HMGCO – A reductase
(1) Methotrexate (a) Nucleic acid derivative
21. Match the following categories of binding to viral enzymes
antimalarial drugs with their respective (2) PAS (b) Inhibitor of dihydrofolate
examples A to E: reductase
(1) Acridine dyes (a) Biguanide (3) Acyclovir (c) Competitive inhibition of PABA
(2) Cinchona alkaloid (b) Primaquine uptake
(3) Aminoquinoline (c) Quinacrine (4) Rifumpicin (d) Reversible inhibition of protein
(4) Diaminopyrimidine (d) Quinine synthesis
(e) Mepacrine (e) Inhibition of DNA dependent
RNA polymerase
22. Match the following antiseptic drugs with
their respective categories A to E: 27. Match the following cholinoceptor acting
(1) Halazone (a) Dye drugs their categories A to E:
(2) Thimerosol (b) Anionic surfactant (1) Pilocarpine (a) Nicotinic blockers
(3) Delqualinium (c) Halogen containing (2) Physostigmine (b) Cholinesterase regenerator
(4) Proflavin (d) Heavy metals
(3) Pirenzepine (c) Direct acting agonist
(e) Cationic surfactant
(4) Pralidoxine (d) Indirect acting carbamate
23. Match the following terms with their (e) Muscarinic blocker
respective meaning A to E:
28. Match the following diseases with their
(1) Aphrodisiac (a) Pain in joint
respective diagnostic tests A to E:
(2) Arrhythmia (b) Loss of power of governing
(3) Arthralgia (c) Absence of spermatozoa in the (1) Leprosy (a) Shick test
sperm (2) Scarlet Fever (b) Lepromin Test
(4) Ataxia (d) Exciting sexual desire (3) Syphilis (c) Dick test
(e) Variation from the normal (4) Typhoid (d) VDRL and Widal test
regular rhythm of the heart (e) Widal test
beat
226 MCQs IN PHARMACOLOGY

29. Match the following drugs with their (1) Astemizole (a) 500 mg
respective mechanisms of action from (2) Captopril (b) 200 mg
A to E: (3) Ascorbic acid (c) 2g
(1) Local anaesthetics (a) Blocks calcium channels (4) Cephalothin (d) 10 mg
(2) Minoxidil (b) Prevents synthesis of prothrombin (e) 100 mg
(3) Proserpine (c) Blocks neuronal sodium 35. Match the following psychoactive drugs
channels with their respective categories A to E:
(4) Warfarin (d) Vasodilation by blocking
(1) Haloperidol (a) Leukotriene
potassium channels
(2) Meprobamate (b) Antipsychotic
(e) Depletes the catecholamine
store in neurons (3) Imipramine (c) Anti - anxiety
(4) LSD (d) Anti - depressant
30. Match the following drugs of abuse with (e) Psychotogenic
their respective categories A to E:
36. Match the following sympathomimetic
(1) Scopolamine (a) Inhalant
drugs with their respective categories A
(2) Nitrous oxide (b) Marijuana
to E:
(3) Hashish (c) Hallucinogen
(1) Amphetamine (a) Dopamine
(4) Cocaine (d) Sedative hyponotic
(2) Phenylephrine (b) Indirect releaser
(e) Stimulant
(3) Dobutamine (c) Alpha–1 selective
31. Match the following enzymes inhibited by (4) Albuterol (d) Beta–1 selective
the corresponding agents A to E: (e) Beta–2 selective
(1) Acetylcholine (a) Acetazolamide
37. Given below are the ailments and the
esterase
drugs used. Match them correctly.
(2) Carbonic anhydrase (b) Methotrexate
(1) Parkinson’s disease (a) Probenecid
(3) Dihydrofoleate (c) Trimethoprim
reducatase in human (2) Glaucoma (b) Ampicillin
(4) Dihydrofoleate (d) Physostigmine (3) Gout (c) Nitroglycerin
reducatase in microbes (4) Angina (d) Pilocarpine
(e) Primethamine (e) Levo dopa

32. Match the following neurotransmitters with 38. Given below are the drugs and their
their respective localizations A to E: antagonists. Match them correctly.
(1) GABA (a) Neuromuscular junction (1) 5–HT (a) Bemegride
(2) L - Glutamate (b) Hypothalamus (2) Codeine (b) Atropine
(3) Neuropeptide (c) Substantia nigra and striatum (3) Phenobarbitone (c) Cyproheptadine
(4) Dopamine (d) Cerebral cortex (4) Muscarine (d) Naloxone
(e) Cerebral nerve (e) Pyridoxine

33. Match the following drugs used in 39. Choose the most appropriate from the
Parkinsonism with their respective group below to match drugs:
categories A to E: (1) Cocaine (a) Central stimulant
(1) Levodopa (a) Dopa decarboxylase inhibitor (2) Codeine (b) Acetylcholine estrase inhibitor
(2) Carbidopa (b) MAU inhibitor (3) Physostigmine (c) Cardiotonic
(3) Biperiden (c) Used in Wilson’s disease (4) Atropine (d) Relief of mild pain
(4) Penicillamine (d) Dopamine prodrug (e) Mydriatic
(e) Used in Tremor 40. Given below are some of the drugs and
their mode of action in A to E. Match them
34. Match the following drugs with their
correctly.
respective usual doses A to E:
MATCH THE FOLLOWING 227

(1) Hydralazine (a) Vasodilator by direct action (2) Streptomycin (b) An antibiotic cotaining nitro
(2) Phenothiazine (b) Inhibits the vasconstrictor and group which binds to 50 S
pressor effects of 5-HT ribosomal subunit
(3) Methysergide (c) Antagonist to HT receptor of (3) Chloramphenicol (c) A dimeric indole alkaloid which
Histamine binds avidily to tubulin, a class
(4) Tolazamide (d) Stimulate the islet tissue to of protein that forms the miotic
secrete insulin spindle
(e) Inhibiting the enzyme carbonic (4) Rifampicin (d) A quinoline alkalid which inhibits
anhydrase the growth of Plasmodium vivax
41. Givern below are the hypotensive agents. (e) A naphthalene antibiotic which
Match their modes of action: inhibits cell wall synthesis

(1) Minoxidil (a) Alpha adereno receptor 46. For the following drugs, specific mechanism
antagonist of action is given in A to D. Match them:
(2) Parazosin (b) Beta adereno receptor
antagonist (1) Spironolactone (a) Non-competitively inhibit the
enzyme carbonic anhydrase
(3) Alpha – methyl dopa (c) From alpha methyl norepi-
nepherine (2) Acetazolamide (b) Inhibit the cotransport of Na+
(4) Clonidine (d) Direct action on blood vessel and Cl- in loop of Henle
(e) Decreases sympathetic activity (c) Competitive inhibitor of
through brain aldosterone at the receptors in
the distal tubule
42. Given below are the drugs and their (d) Direct inhibition of Na+ and Cl-
enzymes inhibited by them. Match them reabsorption
correctly.
(1) Physostigmine (a) COMT 47. Match the following terms with their
(2) Imipramine (b) Acetaldehyde dehydrogenase respective definitions A to E:
(3) Pyrogallol (c) Carbonic anhydrase (1) Drug allergy (a) Excessive pharmacological
(4) Disulfiram (d) Cholinesterase action of the drug due to
(e) MAO overdosage or prolonged use
(2) Toxic effect (b) Characteristic toxic effects of
43. Choose the most appropriate drug for the a drug in an individual at
following: therapeutic doses
(1) Potassium sparing (a) Spironolactone (3) Idiosyncrasy (c) An immunologically mediated
diuretic reaction producing sterotype
(2) Loop diuretic (b) Mannitol symptoms
(3) Osmotic diuretic (c) Furosemide (4) Intolerance (d) Abnormal reactivity to a
(4) Carbonic anhydrase (d) Acetazolamide inhibitor chemical
(e) Aldosterone (e) Alteration of mood and feelings
44. Match the following regions in GIT with 48. Match the following terms with their
the pH levels indicated from A to E: respective meanings A to E:
(1) Mouth (a) = 5.0–6.0 (1) Carcinogenicity (a) Drugs causing genetic defects
(2) Stomach (b) = 6.8–7.5
(2) Mutagenicity (b) Drugs causing foetal absor-
(3) Deodenum (c) = 6.8–7.0 malities
(4) Large intestine (d) = 3.0–5.0 (3) Teratogenicity (c) Drugs capable of altering
(e) = 1.5–3.0 mood and feelings
45. For the drugs listed 1 to 4, mechanism of (4) Drug dependence (d) Drugs causing functional
action is indicated from A to E. Match them: disturbances which persist,
even after the offending drug
(1) Vincristine (a) Macrolide antibiotic which
inhibits DNA dependent RNA (e) Drug causing cancer
polymerase
228 MCQs IN PHARMACOLOGY

49. Match the following teratogenic drugs and (1) Aminoglycosides (a) Polymyxin–B Colistin, Bacitracin
their respective produced abnormalies (2) Macrolide (b) Streptomycin, Gentamycin,
A to D: antibiotics Neomycin
(1) Aspirin (a) Nose, eye and hand defects, (3) Polypeptide (c) Erythromycin, Oleandomycin,
growth retardation antibiotics Roxi-thromycin
(2) Antithyroid drugs (b) Hypoplastic phalanges, cleft (4) Nitrofuran (d) Vancomycin, Lincomycin,
lip, micro-cephaly derivatives Viomycin
(3) Phenytoin (c) Premature closure of ductus (e) Nitrofurantoin, Furazolidone
arteriosus
55. Match the following categories of anti-
(4) Warfarin (d) Foetal goiter and hypothyroidism
microbial agents with their respective
50. Match the following drugs and their examples A to E:
respective resulted diseases A to E: (1) Antibacterial (a) Chloroquine, Metronidazole,
(1) Isoniazid (a) Peptic ulcers Diloxanide
(2) Phenothiazines (b) Hepatitis (2) Antifungal (b) Sodium fusidate, Thiacetazone
(3) Salicylates (c) Parkinsonism (3) Antiviral (c) (Acyclovir), Amantadine,
Idoxuridine, Zidovudin
(4) Tetracyclines (d) Phocomelia, multiple defects
(4) Antiprotozoal (d) Amphotericin B, Griseofulvin,
(e) Discoloured and deformed teeth, Ketoconazole
retarded bone growth
(e) Aminoglycosides, Erythromy-
51. Match the following oral hypoglycaemic cin, Penicillins
drugs with their respective features A to 56. Match the following categories of
E laxatives/purgati es with their respective
(1) Tolbutamide (a) Potent examples A to E:
(2) Penicillin V (b) Penicillinase resistant (1) Bulk forming (a) Docusates
(3) Methicillin (c) Natural (2) Stimulant (b) Magnesium and sodium salts
(4) Ampicillin (d) Semi–synthetic (3) Osmotic (c) Bisacodyl, Senna, Castor Oil
(e) Anti–pseudomonas (4) Stool sofener (d) Dietar y fibre, Psyllium
Methylcellulose
52. Match the following sulfonamides with (e) Liquid paraffin
their duration of actions from A to E
(1) Sulphadiazine (a) Long acting
57. Match the following categories of antiulcer
drugs with their respective examples A to E:
(2) Sulfadimethoxine (b) Toxic
(3) Sulfaphenazole (c) Short acting (1) Ulcer heating drugs (a) Sodium bicarbonate, sodium
citrate
(4) Succinyl (d) Intermediate acting
(2) Ulcer protective (b) Carbenoxolone sodium,
sulfathiazole
Deglycyrrhi-zinized liquorice
(e) Poorly absorbed
(3) H2 antihistamines (c) Sucralfate, colloidal bismuth
53. Match the following modes of action with subctirate
their respective antimicrobial drugs A to E (4) Anticholinergics (d) Atropine Pirenzepine Trimi-
pramine
(1) Interfere with (a) Rifampicin, Norfloxacin,
(e) Cimetidine, Ranitiding,
intermediary meta- Metronidazole Roxatidine
bolism
(2) Interfere with DNA (b) Penicillins, Cephalosporins, 58. Match the following fat-soluble vitamins
synthesis Vancomycin Bactracin with their respective chemical constituents
(3) Intefere with DNA (c) Idoxuridine, Acylovir, Zidovudin A to E:
(4) Inhibit cell wall (d) Sulfonamides, PAS, Ethambutol (1) Vitamin A (a) Pyridoxine
synthesis (2) Vitamin D (b) Methyl Phytyl naphthaquinone
(e) Polymyxins, Colistins, Bacitracin (3) Vitamin E (c) Alpha tocopherol
(4) Vitamin K (d) Calciferol
54. Match the following categories with their
(e) Retinol
respective antimicrobial drugs A to E:
MATCH THE FOLLOWING 229

ANSWERS
1. 1-d, 2-e, 3-b, 4-c 2. 1-d, 2-c, 3-b, 4-e 3. 1-c, 2-d, 3-a, 4-c
4. 1-b, 2-d, 3-a, 4-c 5. 1- b, 2-a, 3-b, 4-c 6. 1-d, 2-a, 3-b, 4c
7. 1- e, 2-c, 3-a, 4-b 8. 1-b, 2-e, 3-a, 4-c 9. 1-b, 2-a, 3-d, 4-c
10. 1-d, 2-a, 3-e, 4-b 11. 1-c, 2-e, 3-d, 4-b 12. 1-b, 2-c, 3-d, 4-e
13. 1-d, 2-c, 3-e, 4-a 14. 1-d, 2-a, 3-b, 4-c 15. 1-c, 2-d, 3-b, 4-a
16. 1-c, 2-d, 3-e, 4-b 17. 1-b, 2-c, 3-d, 4-a 18. 1-d, 2-e, 3-c, 4-a
19. 1-d, 2-e, 3-a, 4-b 20. 1-b, 2-a, 3-e, 4-c 21. 1-e, 2-d, 3-b, 4-c
22. 1-c, 2-d, 3-e, 4-a 23. 1-d, 2-e, 3-a, 4-b 24. 1-b, 2-a, 3-d, 4-e
25. 1-d, 2-c, 3- b, 4- a 26. 1-b, 2-c, 3-a, 4-e 27. 1-c, 2-d, 3-e, 4-b
28. 1-b, 2-c, 3-d, 4-e 29. 1-c, 2-d, 3-e, 4-b 30. 1-c, 2-a, 3-b, 4-e
31. 1-d, 2-a, 3-b, 4-c 32. 1-e, 2-d, 3-b, 4-c 33. 1-d, 2-a, 3-b, 4-c
34. 1-d, 2-e, 3-a, 4-c 35. 1-b, 2-c, 3-d, 4-e 36. 1-b, 2-c, 3-d, 4-e
37. 1-e, 2-d, 3-a, 4-c 38. 1-c, 2-d, 3-a, 4-b 39. 1-a, 2-d, 3-b, 4-e
40. 1-a, 2-a, 3-a, 4-a 41. 1-b, 2-a, 3-a, 4-e 42. 1-e, 2-d, 3-b, 4-c
43. 1-a, 2-c, 3-b, 4-d 44. 1-b, 2-e, 3-d, 4-c 45. 1-c, 2-d, 3-b, 4-a
46. 1-X, 2-X, 3-X, 4-X 47. 1-c, 2-a, 3-d, 4-b 48. 1-e, 2-a, 3-b, 4-c
49. 1-c, 2-a, 3-b, 4-a 50. 1-b, 2-c, 3-a, 4-e 51. 1-e, 2-c, 3-b, 4-a
52. 1-c, 2-a, 3-d, 4-e 53. 1-d, 2-c, 3-a, 4-b 54. 1-b, 2-c, 3-a, 4-e
55. 1-e, 2-d, 3-c, 4-a 56. 1-d, 2-a, 3-b, 4-a 57. 1-b, 2-c, 3-e, 4-d
58. 1-e, 2-d, 3-c, 4-b

Вам также может понравиться